You are on page 1of 2309

‫ﺑﻨﻚ أﺳﺌﻠﺔ اﻟﺪﻛﺘﻮر أﻣﺠﺪ ﻋﻔﻴﻒ‬

‫‪Internal Medicine -‬‬

‫ﻋﺪد اﻷﺳﺌﻠﺔ ‪1081‬‬

‫ﺗﺎرﻳﺦ ﺗﺤﺪﻳﺚ اﻷﺳﺌﻠﺔ ‪08-04-2023‬‬

‫ﻫﺬا اﻟﺘﺠﻤﻴﻊ ﻳﺤﺘﻮي ﻋﲆ ﺟﻤﻴﻊ اﻷﺳﺌﻠﺔ اﳌﻮﺟﻮدة ﰲ ﺗﻄﺒﻴﻖ د‪ .‬أﻣﺠﺪ ﻋﻔﻴﻒ‬

‫‪Page - 1‬‬
‫ﻣﻮاﺿﻴﻊ اﻷﺳﺌﻠﺔ وأﻋﺪادﻫﺎ‬

1) Cardiology => 155 Questions


2) Endocrinology => 162 Questions
3) Gastroenterology => 106 Questions
4) Hematology => 155 Questions
5) Hepatology => 66 Questions
6) Infectious diseases => 44 Questions
7) Nephrology => 96 Questions
8) Neurology => 82 Questions
9) Pulmonology => 144 Questions
10) Rheumatology => 71 Questions

Page - 2
Cardiology

Page - 3
Cardiology

‫ﻣﻮاﺿﻴﻊ اﻷﺳﺌﻠﺔ وأﻋﺪادﻫﺎ‬

1) Acute and decompensated heart failure (DHF) => 5 Questions


2) Acute coronary syndrome (ACS) => 11 Questions
3) Anaphylaxis => 9 Questions
4) Brain Natriuretic Peptide (BNP) => 3 Questions
5) Cardiac Transplant => 2 Questions
6) Cardiac tumors => 1 Questions
7) Cardiomyopathies => 11 Questions
8) Cardiopulmonary resuscitation (CPR) => 4 Questions
9) Chest Pain => 3 Questions
10) Cocaine-induced Chest Pain => 3 Questions
11) Complications of MI => 2 Questions
12) Congestive heart failure (CHF) => 19 Questions
13) Digoxin and Digoxin toxicity => 2 Questions
14) Dysrhythmias => 40 Questions
15) Infective Endocarditis (IE) => 7 Questions
16) Nitroglycerin => 2 Questions
17) Normal heart conduction and basic ECG => 5 Questions
18) Pericardial eusion and Tamponade => 3 Questions
19) Pericarditis => 4 Questions
20) Prinzmetal Angina => 1 Questions
21) Stable angina => 6 Questions
22) Syncope => 3 Questions
23) Valvular diseases and heart murmurs => 9 Questions

Page - 4
Internal Medicine - Cardiology

Question 1/155

Question #1

A 55-year-old male patient presents with acute typical chest pain and is diagnosed with STEMI.
Which of the following mostly indicates a diagnosis of cardiogenic shock in this patient?

a. Acute refractory pulmonary edema


b. High CVP and hypotension
c. Low CVP and hypotension
d. Sinus bradycardia and hypotension
e. Ventricular arrhythmia and hypotension

‫اﻹﺟﺎﺑﺔ ﻋﲆ اﻟﺼﻔﺤﺔ اﻟﺘﺎﻟﻴﺔ‬

Page - 5
Internal Medicine - Cardiology - Acute and decompensated heart failure (DHF)

Question 1/155

Question #1

A 55-year-old male patient presents with acute typical chest pain and is diagnosed with STEMI.
Which of the following mostly indicates a diagnosis of cardiogenic shock in this patient?

a. Acute refractory pulmonary edema


b. High CVP and hypotension √
c. Low CVP and hypotension
d. Sinus bradycardia and hypotension
e. Ventricular arrhythmia and hypotension

Description

Cardiogenic shock is sustained hypotension with tissue hypoperfusion despite adequate le
ventricular filling pressure.

e main signs of cardiogenic shock are hypotension, oliguria, cold extremities, altered mental
status, hypotension, rapid and weak thready pulse, and jugular venous distension.

Page - 6
Internal Medicine - Cardiology

Question 2/155

Question #2

A 65-year-old patient with a history of CHF presents with acute shortness of breath, orthopnea, and
diaphoresis. His blood pressure is 210/120, pulse 120 bpm, and ECG and cardiac enzymes are
unremarkable. What is the most appropriate initial treatment?

a. Furosemide
b. Metoprolol
c. Enalapril
d. Morphine
e. Aldactone

‫اﻹﺟﺎﺑﺔ ﻋﲆ اﻟﺼﻔﺤﺔ اﻟﺘﺎﻟﻴﺔ‬

Page - 7
Internal Medicine - Cardiology - Acute and decompensated heart failure (DHF)

Question 2/155

Question #2

A 65-year-old patient with a history of CHF presents with acute shortness of breath, orthopnea, and
diaphoresis. His blood pressure is 210/120, pulse 120 bpm, and ECG and cardiac enzymes are
unremarkable. What is the most appropriate initial treatment?

a. Furosemide √
b. Metoprolol
c. Enalapril
d. Morphine
e. Aldactone

Description

Elevated blood pressure here is the main trigger of pulmonary edema. is may occur due to non-
compliance with medications, a high salt diet, cardiac insult such as acute myocardial infarction, etc.

Intravenous furosemide is the first-line medication to help the kidneys remove excess fluid from the
body.

e treatment of pulmonary edema is achieved by reducing the preload and the aerload of the
heart:

Diuretics (intravenous furosemide)


Morphine (anxiolytic and reduced catecholamine production leading to reduced vascular
resistance)
Nitroprusside (reduces the preload and aerload)
Noninvasive ventilation may have a rule
Dobutamine and intra-aortic balloon pump can be used in the presence of concomitant
hypotension

Page - 8
Internal Medicine - Cardiology

Question 3/155

Question #3

A 66-year-old male patient has shortness of breath, diaphoresis, and tachypnea. He is a known case
of CHF and is compliant with his medications. However, physical examination reveals lower limb
edema, increased JVP, and lung crepitations; his heart rate is 115/ minute, his respiratory rate is
32/minute, and his blood pressure is 170/100 mmHg. What is the most appropriate medication to
prescribe at this time?

a. Inderal 40mg PO
b. Digoxin 500 mic intravenously
c. Intravenous mannitol
d. Intravenous morphine
e. Intravenous normal saline

‫اﻹﺟﺎﺑﺔ ﻋﲆ اﻟﺼﻔﺤﺔ اﻟﺘﺎﻟﻴﺔ‬

Page - 9
Internal Medicine - Cardiology - Acute and decompensated heart failure (DHF)

Question 3/155

Question #3

A 66-year-old male patient has shortness of breath, diaphoresis, and tachypnea. He is a known case
of CHF and is compliant with his medications. However, physical examination reveals lower limb
edema, increased JVP, and lung crepitations; his heart rate is 115/ minute, his respiratory rate is
32/minute, and his blood pressure is 170/100 mmHg. What is the most appropriate medication to
prescribe at this time?

a. Inderal 40mg PO
b. Digoxin 500 mic intravenously
c. Intravenous mannitol
d. Intravenous morphine √
e. Intravenous normal saline

Description

is case is typical for decompensated heart failure and acute pulmonary edema.

e treatment of pulmonary edema is achieved by reducing the preload and the aerload of the
heart:

Diuretics (intravenous furosemide)


Morphine (anxiolytic and reduce catecholamine production, leading to reduced vascular
resistance)
Nitroprusside (reduces the preload and aerload)
Noninvasive ventilation may have a rule
Dobutamine and intra-aortic balloon pump can be used in the presence of concomitant
hypotension

Beta-blockers are not used in acute pulmonary edema because they can limit the heart rate. is will
lead to more reduction in cardiac output and worsen the condition.

Page - 10
Internal Medicine - Cardiology

Question 4/155

Question #4

A 72-year-old male patient with a history of COPD, CHF, and HTN presents with dyspnea. On
examination, reduced breath sounds and scattered wheezes are noted, but there is no Jugular
venous distention. e heart sound is distant, and lower limb edema is noted. his BNP level is
850pg/mL. Which of the following is the best initial management of this patient?

a. Intravenous heparin
b. Oral high-dose aspirin
c. Start dobutamine infusion
d. Short-acting beta-agonists
e. Furosemide intravenously

‫اﻹﺟﺎﺑﺔ ﻋﲆ اﻟﺼﻔﺤﺔ اﻟﺘﺎﻟﻴﺔ‬

Page - 11
Internal Medicine - Cardiology - Acute and decompensated heart failure (DHF)

Question 4/155

Question #4

A 72-year-old male patient with a history of COPD, CHF, and HTN presents with dyspnea. On
examination, reduced breath sounds and scattered wheezes are noted, but there is no Jugular
venous distention. e heart sound is distant, and lower limb edema is noted. his BNP level is
850pg/mL. Which of the following is the best initial management of this patient?

a. Intravenous heparin
b. Oral high-dose aspirin
c. Start dobutamine infusion
d. Short-acting beta-agonists
e. Furosemide intravenously √

Description

Given the high level of BNP, this patient is suering from acute heart failure exacerbation. e best
initial treatment in the treatment of decompensated heart failure and pulmonary edema is
furosemide

BNP (brain natriuretic peptide) is an enzyme secreted by heart muscles due to stretching or
infarction of these muscles. It acts as a diuretic to decrease blood pressure and decrease the load on
the heart. Helpful to investigate if you are not sure if shortness of breath is due to a cardiac or
respiratory cause

If the BNP level is high (> 500 pg/ml) à cardiac cause


If BNP is not elevated à non-cardiac cause

Causes of high BNP:

Heart failure
Acute MI
Mitral valve rupture
Constrictive pericarditis
Large pulmonary embolus

Page - 12
Internal Medicine - Cardiology

Question 5/155

Question #5

A non-compliant with his medications 60-year-old male known for CHF presents with increasing
shortness of breath, cyanosis, distress, and sweating. e patient is agitated, and his vital signs are
as follows: blood pressure is 230/130, pulse is 130 bpm (regular), and temperature is 37.8 °C. His chest
examination reveals diuse crepitations and wheezes bilaterally. All the following are appropriate
management except:

a. Brain natriuretic peptide will dierentiate the cardiac from the respiratory source of the shortness
of breath
b. Morphine is important to be given to this patient
c. Furosemide is an important symptomatic reliever for his condition
d. Beta-blockers are indicated for treating this patient at this time
e. Place the patient in a semi-setting position

‫اﻹﺟﺎﺑﺔ ﻋﲆ اﻟﺼﻔﺤﺔ اﻟﺘﺎﻟﻴﺔ‬

Page - 13
Internal Medicine - Cardiology - Acute and decompensated heart failure (DHF)

Question 5/155

Question #5

A non-compliant with his medications 60-year-old male known for CHF presents with increasing
shortness of breath, cyanosis, distress, and sweating. e patient is agitated, and his vital signs are
as follows: blood pressure is 230/130, pulse is 130 bpm (regular), and temperature is 37.8 °C. His chest
examination reveals diuse crepitations and wheezes bilaterally. All the following are appropriate
management except:

a. Brain natriuretic peptide will dierentiate the cardiac from the respiratory source of the shortness
of breath
b. Morphine is important to be given to this patient
c. Furosemide is an important symptomatic reliever for his condition
d. Beta-blockers are indicated for treating this patient at this time √
e. Place the patient in a semi-setting position

Description

Because they can suppress the compensatory tachycardia, beta-blockers are not allowed to be given
in a patient with acute decompensated heart failure

Beta-blockers help reduce the mortality rate in congestive heart failure, but it is only used when the
patient is out of decompensation

Page - 14
Internal Medicine - Cardiology

Question 6/155

Question #6

A 76-year-old male patient presents with chest pain, and his ECG with back leads is shown below.
What is the most likely diagnosis?

a. Low Lateral MI
b. Inferior MI
c. Posterior MI
d. High lateral MI
e. Normal ECG

‫اﻹﺟﺎﺑﺔ ﻋﲆ اﻟﺼﻔﺤﺔ اﻟﺘﺎﻟﻴﺔ‬

Page - 15
Internal Medicine - Cardiology - Acute coronary syndrome (ACS)

Question 6/155

Question #6

A 76-year-old male patient presents with chest pain, and his ECG with back leads is shown below.
What is the most likely diagnosis?

a. Low Lateral MI
b. Inferior MI
c. Posterior MI √
d. High lateral MI
e. Normal ECG

Description

is ECG shows ST depression and prominent R wave on the anterior leads and ST elevation on V7 –
V9 (back leads).

is ECG indicates posterior MI.

Page - 16
Internal Medicine - Cardiology

Question 7/155

Question #7

A 66-yaar-old male patient presents with STEMI on leads II, III, aVF, and the right V4. Which of the
following arteries is most likely occluded in this patient?

a. Le circumflex artery


b. Right coronary artery
c. Le anterior descending artery
d. Post descending artery
e. Le the main artery

‫اﻹﺟﺎﺑﺔ ﻋﲆ اﻟﺼﻔﺤﺔ اﻟﺘﺎﻟﻴﺔ‬

Page - 17
Internal Medicine - Cardiology - Acute coronary syndrome (ACS)

Question 7/155

Question #7

A 66-yaar-old male patient presents with STEMI on leads II, III, aVF, and the right V4. Which of the
following arteries is most likely occluded in this patient?

a. Le circumflex artery


b. Right coronary artery √
c. Le anterior descending artery
d. Post descending artery
e. Le the main artery

Description

Anatomy of the coronary artery:

e le and right coronary arteries arise from the aortic root.
e le coronary artery branches:
Le circumflex artery (provide blood supply to the le lateral wall of the heart
Anterior descending (LAD): the main blood supply to the anterior and septal walls
e right coronary artery branches:
It supplies the right side of the heart
It supplies the inferior wall of the heart.

e following figures demonstrate the anatomy of the coronary arteries and the correlation
between myocardial infarction and ECG changes in dierent leads.

Page - 18
Page - 19
Internal Medicine - Cardiology

Question 8/155

Question #8

A 45-year-old male presents with typical chest pain and ST elevation on inferior leads. Which of the
following is shown to reduce the mortality rate in this condition?

a. Nitrates
b. Digoxin
c. iazide diuretics
d. Calcium channel blockers
e. Beta-blockers

‫اﻹﺟﺎﺑﺔ ﻋﲆ اﻟﺼﻔﺤﺔ اﻟﺘﺎﻟﻴﺔ‬

Page - 20
Internal Medicine - Cardiology - Acute coronary syndrome (ACS)

Question 8/155

Question #8

A 45-year-old male presents with typical chest pain and ST elevation on inferior leads. Which of the
following is shown to reduce the mortality rate in this condition?

a. Nitrates
b. Digoxin
c. iazide diuretics
d. Calcium channel blockers
e. Beta-blockers √

Description

Beta-blockers and ACE inhibitors have been found to decrease the mortality rate in patients with
myocardial infarction

Nitrates, digoxin, thiazide diuretics, and calcium channel blockers have not been found to reduce
the mortality rate aer myocardial infarction

Page - 21
Internal Medicine - Cardiology

Question 9/155

Question #9

A patient is diagnosed with Anterior STEMI. His Blood pressure is 130/80. His heart rate is 69 bpm,
O2 saturation is 96%. Which of the following is the initial treatment for his condition?

a. Aspirin, Clopidogrel, Heparin, metoprolol, Oxygen, Nitrates


b. Aspirin, Clopidogrel, Heparin, metoprolol, Nitrates
c. Aspirin, Clopidogrel, metoprolol, Oxygen
d. Aspirin, metoprolol, Oxygen, Nitrates
e. Aspirin, Clopidogrel, Heparin, Oxygen, Nitrates

‫اﻹﺟﺎﺑﺔ ﻋﲆ اﻟﺼﻔﺤﺔ اﻟﺘﺎﻟﻴﺔ‬

Page - 22
Internal Medicine - Cardiology - Acute coronary syndrome (ACS)

Question 9/155

Question #9

A patient is diagnosed with Anterior STEMI. His Blood pressure is 130/80. His heart rate is 69 bpm,
O2 saturation is 96%. Which of the following is the initial treatment for his condition?

a. Aspirin, Clopidogrel, Heparin, metoprolol, Oxygen, Nitrates


b. Aspirin, Clopidogrel, Heparin, metoprolol, Nitrates √
c. Aspirin, Clopidogrel, metoprolol, Oxygen
d. Aspirin, metoprolol, Oxygen, Nitrates
e. Aspirin, Clopidogrel, Heparin, Oxygen, Nitrates

Description

Oxygen is not indicated in patients with ACS whose O2 saturation is more than 94%.

Dual antiplatelet agents are indicated in all cases of ACS.

Unless contraindicated, beta-blockers are indicated to reduce the mortality rate in patients with
ACS.

Nitrates will be indicated to reduce the infarction size but never given in a patient with inferior wall
Myocardial infarction

Page - 23
Internal Medicine - Cardiology

Question 10/155

Question #10

A 44-year-old male presents to the ER with typical chest pain and ST elevation on leads V4 – V6.
Which of the following is the most important step in managing this patient?

a. Administration of heparin
b. Administration of nitroglycerine
c. Cardiac enzymes
d. Cardiac monitoring
e. Oxygen administration

‫اﻹﺟﺎﺑﺔ ﻋﲆ اﻟﺼﻔﺤﺔ اﻟﺘﺎﻟﻴﺔ‬

Page - 24
Internal Medicine - Cardiology - Acute coronary syndrome (ACS)

Question 10/155

Question #10

A 44-year-old male presents to the ER with typical chest pain and ST elevation on leads V4 – V6.
Which of the following is the most important step in managing this patient?

a. Administration of heparin
b. Administration of nitroglycerine
c. Cardiac enzymes
d. Cardiac monitoring √
e. Oxygen administration

Description

Monitoring cardiac electrical activity is the most important step in the management because the
patient is at risk of sudden death due to arrhythmia.

If ventricular arrhythmia develops, early DC cardioversion is indicated to control the patient’s


condition

Page - 25
Internal Medicine - Cardiology

Question 11/155

Question #11

A patient is just diagnosed with acute coronary syndrome, which of the following would be an
indication for thrombolytic therapy?

a. A 42-year-old male with new ST-segment depression


b. A 45-year-old male with a new le bundle branch block
c. A 49-year-old with new ST elevation and brain hemorrhage
d. A 50-year-old with a new-onset first-degree AV block
e. A 39-year-old with new-onset ectopic beats

‫اﻹﺟﺎﺑﺔ ﻋﲆ اﻟﺼﻔﺤﺔ اﻟﺘﺎﻟﻴﺔ‬

Page - 26
Internal Medicine - Cardiology - Acute coronary syndrome (ACS)

Question 11/155

Question #11

A patient is just diagnosed with acute coronary syndrome, which of the following would be an
indication for thrombolytic therapy?

a. A 42-year-old male with new ST-segment depression


b. A 45-year-old male with a new le bundle branch block √
c. A 49-year-old with new ST elevation and brain hemorrhage
d. A 50-year-old with a new-onset first-degree AV block
e. A 39-year-old with new-onset ectopic beats

Description

rombolytic therapy is contraindicated in patients with a brain hemorrhage

New-onset Le bundle branch block is considered ST-elevation equivalent, and thrombolytic
treatment could be given unless contraindicated

Page - 27
Internal Medicine - Cardiology

Question 12/155

Question #12

A 48-year-old female patient with a known panic disorder presents with le-side chest tightness
radiating to the le shoulder for the last 3 hours. In addition, she complains of shortness of breath.
Which of the following is the best next step in managing this patient?

a. Reassurance and explanation that this is a panic attack


b. Admission for further evaluation
c. Obtain CBC, chemistry profile, LFT, and ECG
d. Obtain ECG and cardiac enzymes
e. Refer for cardiac catheterization immediately

‫اﻹﺟﺎﺑﺔ ﻋﲆ اﻟﺼﻔﺤﺔ اﻟﺘﺎﻟﻴﺔ‬

Page - 28
Internal Medicine - Cardiology - Acute coronary syndrome (ACS)

Question 12/155

Question #12

A 48-year-old female patient with a known panic disorder presents with le-side chest tightness
radiating to the le shoulder for the last 3 hours. In addition, she complains of shortness of breath.
Which of the following is the best next step in managing this patient?

a. Reassurance and explanation that this is a panic attack


b. Admission for further evaluation
c. Obtain CBC, chemistry profile, LFT, and ECG
d. Obtain ECG and cardiac enzymes √
e. Refer for cardiac catheterization immediately

Description

ECG and cardiac enzymes are the first steps to managing any patient with chest pain.

is patient is suering from typical chest pain. Acute coronary syndrome can’t be excluded, and
ECG should be done immediately

Page - 29
Internal Medicine - Cardiology

Question 13/155

Question #13

A 65-year-old male patient is a known case of DM, HTN, and old MI 2 years ago. He had exertional
angina since the myocardial infarction happened. His medications include propranolol and aspirin.
Today he presents to you with chest pain at rest for 30 hours. ECG reveals an old myocardial
infarction. What is the most appropriate treatment for this patient?

a. Admission and adjustment of the treatment


b. Admission and CABG
c. Add nitrate and discharge home
d. Exercise stress test
e. Reduce the dose of propranolol

‫اﻹﺟﺎﺑﺔ ﻋﲆ اﻟﺼﻔﺤﺔ اﻟﺘﺎﻟﻴﺔ‬

Page - 30
Internal Medicine - Cardiology - Acute coronary syndrome (ACS)

Question 13/155

Question #13

A 65-year-old male patient is a known case of DM, HTN, and old MI 2 years ago. He had exertional
angina since the myocardial infarction happened. His medications include propranolol and aspirin.
Today he presents to you with chest pain at rest for 30 hours. ECG reveals an old myocardial
infarction. What is the most appropriate treatment for this patient?

a. Admission and adjustment of the treatment √


b. Admission and CABG
c. Add nitrate and discharge home
d. Exercise stress test
e. Reduce the dose of propranolol

Description

is patient has a typical presentation of unstable angina with a possible progression to myocardial
infarction. erefore, he needs to be admitted, and his treatment should be adjusted.

CABG is not indicated at this point.

Indications for CABG:

ree vessels disease


Le main artery disease
Proximal LAD occlusion with EF% less than 50%
Chronic disabling angina

Page - 31
Internal Medicine - Cardiology

Question 14/155

Question #14

A 65-year-old male patient has had chest pain for the last 30 minutes, and his ECG is shown in the
picture below. What is the most likely occluded artery?

a. Right coronary artery


b. Le Anterior Descending artery
c. Le main coronary artery
d. Le circumflex coronary artery
e. Posterior descending artery

‫اﻹﺟﺎﺑﺔ ﻋﲆ اﻟﺼﻔﺤﺔ اﻟﺘﺎﻟﻴﺔ‬

Page - 32
Internal Medicine - Cardiology - Acute coronary syndrome (ACS)

Question 14/155

Question #14

A 65-year-old male patient has had chest pain for the last 30 minutes, and his ECG is shown in the
picture below. What is the most likely occluded artery?

a. Right coronary artery


b. Le Anterior Descending artery √
c. Le main coronary artery
d. Le circumflex coronary artery
e. Posterior descending artery

Description

e ECG shows ST elevation on V2 – V4

is indicates anterior wall myocardial infarction

e most common artery aected in this case is LAD

e locations of infarctions and their relation to ECG are shown in the following table:

Page - 33
Page - 34
Internal Medicine - Cardiology

Question 15/155

Question #15

A 65-year-old male patient has had chest pain for the last 30 minutes, and his ECG is shown in the
picture below. What is the most likely occluded artery?

a. Right coronary artery


b. Le Anterior Descending artery
c. Le main coronary artery
d. Le circumflex coronary artery
e. Posterior descending artery

‫اﻹﺟﺎﺑﺔ ﻋﲆ اﻟﺼﻔﺤﺔ اﻟﺘﺎﻟﻴﺔ‬

Page - 35
Internal Medicine - Cardiology - Acute coronary syndrome (ACS)

Question 15/155

Question #15

A 65-year-old male patient has had chest pain for the last 30 minutes, and his ECG is shown in the
picture below. What is the most likely occluded artery?

a. Right coronary artery √


b. Le Anterior Descending artery
c. Le main coronary artery
d. Le circumflex coronary artery
e. Posterior descending artery

Description

e ECG shows ST elevation on II, III, and aVF

is indicates Inferior wall myocardial infarction

e most common artery aected in this case is RCA

e locations of infarctions and their relation to ECG are shown in the following table:

Page - 36
Page - 37
Internal Medicine - Cardiology

Question 16/155

Question #16

A 48-year-old male patient has sudden shortness of breath and diaphoresis. On auscultation,
bibasilar lung crepitations are heard. ECG is shown below. What is the most likely diagnosis?

a. Acute pericarditis
b. Myocardial infarction
c. Pulmonary embolism
d. Pneumonia
e. Spontaneous pneumothorax

‫اﻹﺟﺎﺑﺔ ﻋﲆ اﻟﺼﻔﺤﺔ اﻟﺘﺎﻟﻴﺔ‬

Page - 38
Internal Medicine - Cardiology - Acute coronary syndrome (ACS)

Question 16/155

Question #16

A 48-year-old male patient has sudden shortness of breath and diaphoresis. On auscultation,
bibasilar lung crepitations are heard. ECG is shown below. What is the most likely diagnosis?

a. Acute pericarditis
b. Myocardial infarction √
c. Pulmonary embolism
d. Pneumonia
e. Spontaneous pneumothorax

Description

is ECG shows elevated ST-segment on leads II, III, and aVF. It also has reciprocal changes on leads I,
aVL, and V6

ese changes, along with the history, are typical for acute myocardial infarction.

Page - 39
Internal Medicine - Cardiology

Question 17/155

Question #17

A 22-year-old male patient developed facial swelling, stridor, and hypotension immediately aer
eating shrimp at a restaurant. Which one of the following treatments should be initiated
immediately?

a. Hydrocortisone
b. Antihistamine
c. Epinephrine
d. Aminophylline
e. Dexamethasone

‫اﻹﺟﺎﺑﺔ ﻋﲆ اﻟﺼﻔﺤﺔ اﻟﺘﺎﻟﻴﺔ‬

Page - 40
Internal Medicine - Cardiology - Anaphylaxis

Question 17/155

Question #17

A 22-year-old male patient developed facial swelling, stridor, and hypotension immediately aer
eating shrimp at a restaurant. Which one of the following treatments should be initiated
immediately?

a. Hydrocortisone
b. Antihistamine
c. Epinephrine √
d. Aminophylline
e. Dexamethasone

Description

is is a case of anaphylactic shock.

e treatment of choice for anaphylactic shock is adrenaline intramuscularly.

Hydrocortisone, dexamethasone, and antihistamines are used to treat anaphylactic shock but have
slower and lower eects.

Subcutaneous administration of drugs in hypotensive patients is inappropriate because of the


reduced skin perfusion that slows systemic absorption.

Anaphylaxis:

It is an IgE-mediated severe, life-threatening, generalized, or systemic hypersensitivity


reaction; the patient must be already sensitized to the antigen
Food (e.g., Nuts) - the most common cause in children
e rash is present in all types of anaphylaxis, while other symptoms like hypotension,
tachycardia, and respiratory symptoms are variable.
e first-line treatment is adrenalin 1/1,000 (best to give IM)

Page - 41
Internal Medicine - Cardiology

Question 18/155

Question #18

A 15-year-old boy develops anaphylactic shock due to a food allergy. You would administrate which
of the following intramuscular drugs?

a. Adrenalin 1/10
b. Adrenalin 1/100
c. Adrenalin 1/1000
d. Adrenalin 1/10000
e. Adrenalin 1/100000

‫اﻹﺟﺎﺑﺔ ﻋﲆ اﻟﺼﻔﺤﺔ اﻟﺘﺎﻟﻴﺔ‬

Page - 42
Internal Medicine - Cardiology - Anaphylaxis

Question 18/155

Question #18

A 15-year-old boy develops anaphylactic shock due to a food allergy. You would administrate which
of the following intramuscular drugs?

a. Adrenalin 1/10
b. Adrenalin 1/100
c. Adrenalin 1/1000 √
d. Adrenalin 1/10000
e. Adrenalin 1/100000

Description

Adrenalin 1/1000 is the most appropriate intramuscular dose for anaphylactic shock.

Treatment of anaphylaxis includes the following:

First-line treatment is adrenalin 1/1,000 → best to give IM


Hydrocortisone: (its action needs up to 4 hours)
Chlorphenamine (Allerfrin)
Nebulized Beta-2 agonist
Supportive measures (O2, IV Fluids)
Emergent airway protection if needed (intubation, cricothyroidotomy)

Page - 43
Internal Medicine - Cardiology

Question 19/155

Question #19

A 15-year-old male presented to you aer a bee sting while playing in the garden. e patient is tired,
his skin is flushing, his blood pressure is 80/70, and his pulse rate is 110 bpm. What is the most
appropriate initial medication to administer to this patient?

a. Hydrocortisone
b. Inhaled SABA
c. Subcutaneous adrenaline
d. Intramuscular adrenaline
e. IV normal saline

‫اﻹﺟﺎﺑﺔ ﻋﲆ اﻟﺼﻔﺤﺔ اﻟﺘﺎﻟﻴﺔ‬

Page - 44
Internal Medicine - Cardiology - Anaphylaxis

Question 19/155

Question #19

A 15-year-old male presented to you aer a bee sting while playing in the garden. e patient is tired,
his skin is flushing, his blood pressure is 80/70, and his pulse rate is 110 bpm. What is the most
appropriate initial medication to administer to this patient?

a. Hydrocortisone
b. Inhaled SABA
c. Subcutaneous adrenaline
d. Intramuscular adrenaline √
e. IV normal saline

Description

is is a case of anaphylactic shock.

e treatment of choice for anaphylactic shock is adrenaline intramuscularly.

Hydrocortisone, inhaled SABA, and normal saline are also used to treat anaphylactic shock, but they
have slower and lower eects.

Subcutaneous administration of drugs in hypotensive patients is inappropriate because of the


reduced skin perfusion that slows systemic absorption.

Anaphylaxis:

It is an IgE-mediated severe, life-threatening, generalized, or systemic hypersensitivity


reaction; the patient must be already sensitized to the antigen
Food (e.g., Nuts) - the most common cause in children
e rash is present in all types of anaphylaxis, while other symptoms like hypotension,
tachycardia, and respiratory symptoms are variable.
e first-line treatment is adrenalin 1/1,000 (best to give IM)

Page - 45
Internal Medicine - Cardiology

Question 20/155

Question #20

e most appropriate route of administration for adrenalin in patients with anaphylaxis is:

a. Intravenous
b. Subcutaneous
c. Oral
d. Intramuscular
e. Inhalational

‫اﻹﺟﺎﺑﺔ ﻋﲆ اﻟﺼﻔﺤﺔ اﻟﺘﺎﻟﻴﺔ‬

Page - 46
Internal Medicine - Cardiology - Anaphylaxis

Question 20/155

Question #20

e most appropriate route of administration for adrenalin in patients with anaphylaxis is:

a. Intravenous
b. Subcutaneous
c. Oral
d. Intramuscular √
e. Inhalational

Description

e treatment of choice for anaphylactic shock is adrenaline intramuscularly.

Subcutaneous administration of drugs in hypotensive patients is inappropriate because of the


reduced skin perfusion that slows systemic absorption.

Anaphylaxis:

It is an IgE-mediated severe, life-threatening, generalized, or systemic hypersensitivity


reaction; the patient must be already sensitized to the antigen
Food (e.g., Nuts) - the most common cause in children
e rash is present in all types of anaphylaxis, while other symptoms like hypotension,
tachycardia, and respiratory symptoms are variable.
e first-line treatment is adrenalin 1/1,000 (best to give IM)

Page - 47
Internal Medicine - Cardiology

Question 21/155

Question #21

A 17-year-old male patient is going to a camp next week. He has a history of anaphylactic shock
secondary to a bee sting 1 year ago. Which of the following would you recommend preventing
severe adverse reactions if exposed to a sting during camping?

a. He should never go to the camp


b. He should take antihistamine if stung
c. Self-administrated adrenalin should be with the patient
d. He should be indoors all the time
e. He should wear a beekeeping suit when he is outdoor

‫اﻹﺟﺎﺑﺔ ﻋﲆ اﻟﺼﻔﺤﺔ اﻟﺘﺎﻟﻴﺔ‬

Page - 48
Internal Medicine - Cardiology - Anaphylaxis

Question 21/155

Question #21

A 17-year-old male patient is going to a camp next week. He has a history of anaphylactic shock
secondary to a bee sting 1 year ago. Which of the following would you recommend preventing
severe adverse reactions if exposed to a sting during camping?

a. He should never go to the camp


b. He should take antihistamine if stung
c. Self-administrated adrenalin should be with the patient √
d. He should be indoors all the time
e. He should wear a beekeeping suit when he is outdoor

Description

Adrenalin is the treatment of choice for anaphylactic shock.

Adrenalin is the primary treatment and should be provided immediately

self-injectable epinephrine should be with patients with anaphylaxis all the time to avoid severe
anaphylactic reactions.

Self-injectable epinephrine can be delivered IM or SC immediately aer exposure to an allergen that


is usually causing anaphylaxis.

Anaphylaxis:

It is an IgE-mediated severe, life-threatening, generalized, or systemic hypersensitivity


reaction; the patient must be already sensitized to the antigen
Food (e.g., Nuts) - the most common cause in children
e rash is present in all types of anaphylaxis, while other symptoms like hypotension,
tachycardia, and respiratory symptoms are variable.
e first-line treatment is adrenalin 1/1,000 (best to give IM)

Page - 49
Internal Medicine - Cardiology

Question 22/155

Question #22

A 66-year-old has urine retention for 15 hours, and you ordered Foley catheter insertion. ree
minutes later, the patient started to have a wheezy chest, hypotension, and skin rash. What is the
most likely cause for this clinical scenario?

a. Pulmonary embolism
b. Acute myocardial infarction
c. Urinary bladder rupture
d. Latex allergy
e. Septic shock

‫اﻹﺟﺎﺑﺔ ﻋﲆ اﻟﺼﻔﺤﺔ اﻟﺘﺎﻟﻴﺔ‬

Page - 50
Internal Medicine - Cardiology - Anaphylaxis

Question 22/155

Question #22

A 66-year-old has urine retention for 15 hours, and you ordered Foley catheter insertion. ree
minutes later, the patient started to have a wheezy chest, hypotension, and skin rash. What is the
most likely cause for this clinical scenario?

a. Pulmonary embolism
b. Acute myocardial infarction
c. Urinary bladder rupture
d. Latex allergy √
e. Septic shock

Description

e timing here is the most important; this patient has symptoms of anaphylaxis immediately aer
the insertion of the Foley catheter.

e most likely diagnosis here is anaphylaxis secondary to latex exposure.

Anaphylaxis:

It is an IgE-mediated severe, life-threatening, generalized, or systemic hypersensitivity


reaction; the patient must be already sensitized to the antigen
Food (e.g., Nuts) - the most common cause in children
e rash is present in all types of anaphylaxis, while other symptoms like hypotension,
tachycardia, and respiratory symptoms are variable.
e first-line treatment is adrenalin 1/1,000 (best to give IM)

Page - 51
Internal Medicine - Cardiology

Question 23/155

Question #23

An 18-year-old female was bitten by a scorpion 30 minutes ago. Her vital signs at the hospital are
blood pressure of 70/30 mmHg, her pulse is 145 bpm, her temperature is 37.3 °C, and her respiratory
rate is 33 per minute and labored. Which of the following would not be an option in her
management?

a. Adrenalin
b. Atropine
c. Salbutamol
d. Diphenhydramine
e. Methylprednisolone

‫اﻹﺟﺎﺑﺔ ﻋﲆ اﻟﺼﻔﺤﺔ اﻟﺘﺎﻟﻴﺔ‬

Page - 52
Internal Medicine - Cardiology - Anaphylaxis

Question 23/155

Question #23

An 18-year-old female was bitten by a scorpion 30 minutes ago. Her vital signs at the hospital are
blood pressure of 70/30 mmHg, her pulse is 145 bpm, her temperature is 37.3 °C, and her respiratory
rate is 33 per minute and labored. Which of the following would not be an option in her
management?

a. Adrenalin
b. Atropine √
c. Salbutamol
d. Diphenhydramine
e. Methylprednisolone

Description

Atropine here has no rule in the treatment and may exacerbate tachycardia in this patient.

Treatment of anaphylaxis includes the following:

First-line treatment is adrenalin 1/1,000 → best to give IM


Hydrocortisone: (its action needs up to 4 hours)
Chlorphenamine (Allerfrin)
Nebulized Beta-2 agonist
Supportive measures (O2, IV Fluids)
Emergent airway protection if needed (intubation, cricothyroidotomy)

Note that methylprednisolone is a steroid that may be used in this case

Note that diphenhydramine is an antihistamine that may be used in this case too

Page - 53
Internal Medicine - Cardiology

Question 24/155

Question #24

Shortly aer a bee sting, a 22-year-old male started to have a rash, shortness of breath, and
hypotension. is reaction is mediated by which of the following antibodies?

a. IgA antibodies
b. IgM antibodies
c. IgG antibodies
d. IgD antibodies
e. IgE antibodies

‫اﻹﺟﺎﺑﺔ ﻋﲆ اﻟﺼﻔﺤﺔ اﻟﺘﺎﻟﻴﺔ‬

Page - 54
Internal Medicine - Cardiology - Anaphylaxis

Question 24/155

Question #24

Shortly aer a bee sting, a 22-year-old male started to have a rash, shortness of breath, and
hypotension. is reaction is mediated by which of the following antibodies?

a. IgA antibodies
b. IgM antibodies
c. IgG antibodies
d. IgD antibodies
e. IgE antibodies √

Description

Anaphylaxis:

It is an IgE-mediated severe, life-threatening, generalized, or systemic hypersensitivity


reaction; the patient must be already sensitized to the antigen
It is a type 1 hypersensitivity reaction
Food (e.g., Nuts) - the most common cause in children
e rash is present in all types of anaphylaxis, while other symptoms like hypotension,
tachycardia, and respiratory symptoms are variable.
e first-line treatment is adrenalin 1/1,000 (best to give IM)

Pathogenesis:

Exposure to an antigen → immune system activation → IgE binds to mast cells → histamine,
prostaglandins, and leukotrienes release → vasodilatation → hypotension and shock.

Page - 55
Internal Medicine - Cardiology

Question 25/155

Question #25

A 66-year-old male patient was admitted as a case of warfarin overdose and was started on
intravenous fresh frozen plasma. Unfortunately, 30 minutes later, he began to have continuous
flushing, wheezing, pruritus, and urticaria. His blood pressure was 80/44mmHg, and his pulse was
130 bpm. Adrenalin was administrated, and the FFP transfusion was discontinued. Which of the
following is the most likely explanation for this condition?

a. Adrenal crisis
b. e patient has IgA deficiency
c. Gram-negative sepsis
d. Gram-positive sepsis
e. Internal bleeding due to warfarin overdose

‫اﻹﺟﺎﺑﺔ ﻋﲆ اﻟﺼﻔﺤﺔ اﻟﺘﺎﻟﻴﺔ‬

Page - 56
Internal Medicine - Cardiology - Anaphylaxis

Question 25/155

Question #25

A 66-year-old male patient was admitted as a case of warfarin overdose and was started on
intravenous fresh frozen plasma. Unfortunately, 30 minutes later, he began to have continuous
flushing, wheezing, pruritus, and urticaria. His blood pressure was 80/44mmHg, and his pulse was
130 bpm. Adrenalin was administrated, and the FFP transfusion was discontinued. Which of the
following is the most likely explanation for this condition?

a. Adrenal crisis
b. e patient has IgA deficiency √
c. Gram-negative sepsis
d. Gram-positive sepsis
e. Internal bleeding due to warfarin overdose

Description

In patients with IgA deficiency, anti-IgA autoantibodies of the IgG and IgE are oen present and are
the most likely cause of the anaphylactic reaction

In FFP transfusion, you have to use IgA-deficient plasma for the IgA-deficient recipients

Anaphylaxis:

It is an IgE-mediated severe, life-threatening, generalized, or systemic hypersensitivity


reaction; the patient must be already sensitized to the antigen
Food (e.g., Nuts) - the most common cause in children
e rash is present in all types of anaphylaxis, while other symptoms like hypotension,
tachycardia, and respiratory symptoms are variable.
e first-line treatment is adrenalin 1/1,000 (best to give IM)

Page - 57
Internal Medicine - Cardiology

Question 26/155

Question #26

A 72-year-old male patient with a history of COPD, CHF, and HTN presents with dyspnea. On
examination, reduced breath sounds and scattered wheezes are noted, but there is no Jugular
venous distention. e heart sound is distant, and lower limb edema is noted. What is the next step
to dierentiate the cause of dyspnea in this patient?

a. Order a chest radiograph


b. Order brain natriuretic peptide level
c. Order Liver function test
d. Order Pulmonary function test
e. Order urea and creatinine levels

‫اﻹﺟﺎﺑﺔ ﻋﲆ اﻟﺼﻔﺤﺔ اﻟﺘﺎﻟﻴﺔ‬

Page - 58
Internal Medicine - Cardiology - Brain Natriuretic Peptide (BNP)

Question 26/155

Question #26

A 72-year-old male patient with a history of COPD, CHF, and HTN presents with dyspnea. On
examination, reduced breath sounds and scattered wheezes are noted, but there is no Jugular
venous distention. e heart sound is distant, and lower limb edema is noted. What is the next step
to dierentiate the cause of dyspnea in this patient?

a. Order a chest radiograph


b. Order brain natriuretic peptide level √
c. Order Liver function test
d. Order Pulmonary function test
e. Order urea and creatinine levels

Description

BNP (brain natriuretic peptide) is an enzyme secreted by heart muscles due to stretching or
infarction of these muscles. It acts as a diuretic to decrease blood pressure and decrease the load on
the heart; useful to investigate if you are not sure if shortness of breath is due to a cardiac or
respiratory cause

If the BNP level is high (> 500 pg/ml) à cardiac cause


If BNP is not high à non-cardiac cause

Causes of high BNP:

Heart failure
Acute MI
Mitral valve rupture
Constrictive pericarditis
Large pulmonary embolus

Page - 59
Internal Medicine - Cardiology

Question 27/155

Question #27

A patient with shortness of breath was found to have a high brain natriuretic peptide level. Which of
the following is the least likely cause of his shortness of breath?

a. Heart failure
b. Acute myocardial infarction
c. Massive pulmonary embolism
d. Chronic obstructive lung disease
e. Constrictive pericarditis

‫اﻹﺟﺎﺑﺔ ﻋﲆ اﻟﺼﻔﺤﺔ اﻟﺘﺎﻟﻴﺔ‬

Page - 60
Internal Medicine - Cardiology - Brain Natriuretic Peptide (BNP)

Question 27/155

Question #27

A patient with shortness of breath was found to have a high brain natriuretic peptide level. Which of
the following is the least likely cause of his shortness of breath?

a. Heart failure
b. Acute myocardial infarction
c. Massive pulmonary embolism
d. Chronic obstructive lung disease √
e. Constrictive pericarditis

Description

Page - 61
Internal Medicine - Cardiology

Question 28/155

Question #28

A 62-year-old male patient with a history of COPD, CAD, and CKD presents with worsening shortness
of breath. His BNP level is 1200 pg/mL. Which of the following would be eective in the treatment of
this patient?

a. Inhaled SAMA
b. Hemodialysis
c. Intravenous mannitol
d. Loop diuretic
e. Nifedipine

‫اﻹﺟﺎﺑﺔ ﻋﲆ اﻟﺼﻔﺤﺔ اﻟﺘﺎﻟﻴﺔ‬

Page - 62
Internal Medicine - Cardiology - Brain Natriuretic Peptide (BNP)

Question 28/155

Question #28

A 62-year-old male patient with a history of COPD, CAD, and CKD presents with worsening shortness
of breath. His BNP level is 1200 pg/mL. Which of the following would be eective in the treatment of
this patient?

a. Inhaled SAMA
b. Hemodialysis
c. Intravenous mannitol
d. Loop diuretic √
e. Nifedipine

Description

A high BNP level is indicative of the presence of stretch on the cardiac muscles, mostly due to heart
failure

BNP (brain natriuretic peptide) is an enzyme secreted by heart muscles due to stretching or
infarction of these muscles. It acts as a diuretic to decrease blood pressure and decrease the load on
the heart; useful to investigate if you are not sure if shortness of breath is due to a cardiac or
respiratory cause

If the BNP level is high (> 500 pg/ml) à cardiac cause


If BNP is not high à non-cardiac cause

Causes of high BNP:

Heart failure
Acute MI
Mitral valve rupture
Constrictive pericarditis
Large pulmonary embolus

Page - 63
Internal Medicine - Cardiology

Question 29/155

Question #29

A 65-year-old male patient with advanced heart failure had cardiac transplant surgery 10 years ago.
Which is the most common cause of late death following heart transplant surgery?

a. Coronary artery disease in the gra


b. Gra rejection
c. Infection
d. Malignancy
e. Side eects of the treatments

‫اﻹﺟﺎﺑﺔ ﻋﲆ اﻟﺼﻔﺤﺔ اﻟﺘﺎﻟﻴﺔ‬

Page - 64
Internal Medicine - Cardiology - Cardiac Transplant

Question 29/155

Question #29

A 65-year-old male patient with advanced heart failure had cardiac transplant surgery 10 years ago.
Which is the most common cause of late death following heart transplant surgery?

a. Coronary artery disease in the gra √


b. Gra rejection
c. Infection
d. Malignancy
e. Side eects of the treatments

Description

Coronary artery disease in the gra (allogra coronary artery disease) is the most common cause of
late death aer cardiac transplant surgery

Infection is the most common cause of death within the first year aer cardiac transplant (25% of
cases), with the greatest risk in the first few months aer the surgery

Infection à 25% of cases

Rejection à 5% of cases

Malignancy à 15% of cases

Page - 65
Internal Medicine - Cardiology

Question 30/155

Question #30

A 65-year-old male patient with advanced heart failure had cardiac transplant surgery 9 months ago
and died today. Which is the most common cause of morbidity and mortality within the first year
following heart transplant surgery?

a. Coronary artery disease in the gra


b. Gra rejection
c. Infection
d. Malignancy
e. Side eects of the treatments

‫اﻹﺟﺎﺑﺔ ﻋﲆ اﻟﺼﻔﺤﺔ اﻟﺘﺎﻟﻴﺔ‬

Page - 66
Internal Medicine - Cardiology - Cardiac Transplant

Question 30/155

Question #30

A 65-year-old male patient with advanced heart failure had cardiac transplant surgery 9 months ago
and died today. Which is the most common cause of morbidity and mortality within the first year
following heart transplant surgery?

a. Coronary artery disease in the gra


b. Gra rejection
c. Infection √
d. Malignancy
e. Side eects of the treatments

Description

Infection is the most common cause of death within the first year aer cardiac transplant (25% of
cases), with the greatest risk in the first few months aer the surgery

Coronary artery disease in the gra (allogra coronary artery disease) is the most common cause of
late death aer cardiac transplant surgery

Infection à 25% of cases

Rejection à 5% of cases

Malignancy à 15% of cases

Page - 67
Internal Medicine - Cardiology

Question 31/155

Question #31

A 49-year-old patient was incidentally discovered to have atrial myxoma by an echocardiogram. e


following findings may be associated with this condition except:

a. Clubbing
b. Fever
c. Mid-diastolic murmur
d. Embolization and CVA
e. Premature ventricular tachycardia

‫اﻹﺟﺎﺑﺔ ﻋﲆ اﻟﺼﻔﺤﺔ اﻟﺘﺎﻟﻴﺔ‬

Page - 68
Internal Medicine - Cardiology - Cardiac tumors

Question 31/155

Question #31

A 49-year-old patient was incidentally discovered to have atrial myxoma by an echocardiogram. e


following findings may be associated with this condition except:

a. Clubbing
b. Fever
c. Mid-diastolic murmur
d. Embolization and CVA
e. Premature ventricular tachycardia √

Description

Atrial fibrillation (Not PVC) is associated with atrial myxoma.

It is a heart tumor that most commonly occurs in the le atrium and may be associated with
clubbing, Embolization, atrial fibrillation, fever, and mid-diastolic murmur. It is diagnosed by
echocardiogram

Page - 69
Internal Medicine - Cardiology

Question 32/155

Question #32

A 17-year-old male patient suddenly died while he was playing football at school. His father died at
the age of 40 years suddenly. What is the most common cause of death in this case?

a. Dilated cardiomyopathy
b. Hypertrophic obstructive cardiomyopathy
c. Myocardial infarction
d. Myocarditis
e. Arrhythmogenic right ventricular cardiomyopathy

‫اﻹﺟﺎﺑﺔ ﻋﲆ اﻟﺼﻔﺤﺔ اﻟﺘﺎﻟﻴﺔ‬

Page - 70
Internal Medicine - Cardiology - Cardiomyopathies

Question 32/155

Question #32

A 17-year-old male patient suddenly died while he was playing football at school. His father died at
the age of 40 years suddenly. What is the most common cause of death in this case?

a. Dilated cardiomyopathy
b. Hypertrophic obstructive cardiomyopathy √
c. Myocardial infarction
d. Myocarditis
e. Arrhythmogenic right ventricular cardiomyopathy

Description

Sudden death in a young person with a family history of sudden death will raise suspicion of HOCM
as the most common cause. Note that the second most common cause, in this case, is ARVC
(arrhythmogenic right ventricular cardiomyopathy)

HOCM is the most common cause of sudden death in athletes

Page - 71
Internal Medicine - Cardiology

Question 33/155

Question #33

A 17-year-old male patient complains of dizziness and recurrent syncope while playing basketball in
school. e school doctor refers the patient to you for evaluation because he suspects a cardiac
cause. However, the patient’s past medical records are unremarkable, but his father suddenly died
at the age of 39. e following findings in the echocardiogram are typical for the most likely
diagnosis except:

a. Mitral valve leaflet hypertrophy


b. A systolic anterior motion of the anterior mitral valve leaflet
c. Asymmetrical septal hypertrophy
d. A degree of diastolic heart failure
e. Mitral stenosis

‫اﻹﺟﺎﺑﺔ ﻋﲆ اﻟﺼﻔﺤﺔ اﻟﺘﺎﻟﻴﺔ‬

Page - 72
Internal Medicine - Cardiology - Cardiomyopathies

Question 33/155

Question #33

A 17-year-old male patient complains of dizziness and recurrent syncope while playing basketball in
school. e school doctor refers the patient to you for evaluation because he suspects a cardiac
cause. However, the patient’s past medical records are unremarkable, but his father suddenly died
at the age of 39. e following findings in the echocardiogram are typical for the most likely
diagnosis except:

a. Mitral valve leaflet hypertrophy


b. A systolic anterior motion of the anterior mitral valve leaflet
c. Asymmetrical septal hypertrophy
d. A degree of diastolic heart failure
e. Mitral stenosis √

Description

e patient most likely suers from Hypertrophic Obstructive Cardiomyopathy (HOCM).

e family history of sudden paternal death and the exertional dizziness and syncope strongly
suggest the condition.

Mitral regurgitation, not stenosis, is the likely finding in this scenario.

e patient will have le ventricular outflow obstruction due to septal or mitral leaflet hypertrophy.

Any degree of heart failure (usually diastolic) may present.

e echocardiogram findings in HOCM are (MR SAM ASH):

Mitral Regurgitation (MR)


Systolic anterior motion (SAM) of the anterior mitral valve
Asymmetric septal hypertrophy (ASH) > 1.5 times the thickness of the posterior wall.

Page - 73
Internal Medicine - Cardiology

Question 34/155

Question #34

A patient is diagnosed with Dilated cardiomyopathy (DCM), the following are more likely to cause
this condition except:

a. Alcoholism
b. Coxsackie B virus
c. Hemochromatosis
d. Doxorubicin
e. Wilson’s disease

‫اﻹﺟﺎﺑﺔ ﻋﲆ اﻟﺼﻔﺤﺔ اﻟﺘﺎﻟﻴﺔ‬

Page - 74
Internal Medicine - Cardiology - Cardiomyopathies

Question 34/155

Question #34

A patient is diagnosed with Dilated cardiomyopathy (DCM), the following are more likely to cause
this condition except:

a. Alcoholism
b. Coxsackie B virus
c. Hemochromatosis
d. Doxorubicin
e. Wilson’s disease √

Description

Wilson’s disease is a possible cause of DCM, but it is extremely rare. It is likely to occur in
hemochromatosis and the other mentioned choices.

Page - 75
Internal Medicine - Cardiology

Question 35/155

Question #35

A 35-year-old male patient with a family history of sudden cardiac death presents with recurrent
syncope and palpitation for the past 6 months. His ECG reveals T wave inversion in V1 – V3 and a
small positive deflection at the end of the QRS complex. What is the most likely diagnosis?

a. Hypertrophic obstructive cardiomyopathy


b. Arrhythmogenic right ventricular dysplasia
c. Long QT syndrome
d. Ischemic heart disease
e. Mitral valve prolapse

‫اﻹﺟﺎﺑﺔ ﻋﲆ اﻟﺼﻔﺤﺔ اﻟﺘﺎﻟﻴﺔ‬

Page - 76
Internal Medicine - Cardiology - Cardiomyopathies

Question 35/155

Question #35

A 35-year-old male patient with a family history of sudden cardiac death presents with recurrent
syncope and palpitation for the past 6 months. His ECG reveals T wave inversion in V1 – V3 and a
small positive deflection at the end of the QRS complex. What is the most likely diagnosis?

a. Hypertrophic obstructive cardiomyopathy


b. Arrhythmogenic right ventricular dysplasia √
c. Long QT syndrome
d. Ischemic heart disease
e. Mitral valve prolapse

Description

History of sudden death, recurrent palpitations, and syncope, along with the inverted T wave in
leads V1 – V3 and epsilon wave, suggest ARVC as the most likely diagnosis.

Arrhythmogenic right ventricular cardiomyopathy (ARVC) is an inherited cardiovascular disease that


may result in arrhythmia and sudden death

It is the second most common cause of sudden death in athletes

Page - 77
Internal Medicine - Cardiology

Question 36/155

Question #36

A patient is suspected of having hypertrophic obstructive cardiomyopathy. Which of the following is


the definitive diagnostic test for this condition?

a. Electrocardiogram
b. Family history of sudden death and history of syncope
c. Chest radiograph
d. Electrophysiological study
e. Echocardiogram

‫اﻹﺟﺎﺑﺔ ﻋﲆ اﻟﺼﻔﺤﺔ اﻟﺘﺎﻟﻴﺔ‬

Page - 78
Internal Medicine - Cardiology - Cardiomyopathies

Question 36/155

Question #36

A patient is suspected of having hypertrophic obstructive cardiomyopathy. Which of the following is


the definitive diagnostic test for this condition?

a. Electrocardiogram
b. Family history of sudden death and history of syncope
c. Chest radiograph
d. Electrophysiological study
e. Echocardiogram √

Description

Family history of sudden death and personal history of exertional syncope Strongly suggest HOCM,
but they are not providing definitive evidence of the disease

e definitive diagnosis is made by a two-dimensional echocardiogram

Page - 79
Internal Medicine - Cardiology

Question 37/155

Question #37

A 17-year-old male patient presents with recurrent syncopal attacks. e last one occurred while he
was running for a bus. His father died suddenly at the age of 38-year-old. What is the most likely
cause?

a. Hysterical conversion disorder


b. Vasovagal attack
c. Epilepsy
d. Cardiogenic syncope
e. Long QT syndrome

‫اﻹﺟﺎﺑﺔ ﻋﲆ اﻟﺼﻔﺤﺔ اﻟﺘﺎﻟﻴﺔ‬

Page - 80
Internal Medicine - Cardiology - Cardiomyopathies

Question 37/155

Question #37

A 17-year-old male patient presents with recurrent syncopal attacks. e last one occurred while he
was running for a bus. His father died suddenly at the age of 38-year-old. What is the most likely
cause?

a. Hysterical conversion disorder


b. Vasovagal attack
c. Epilepsy
d. Cardiogenic syncope √
e. Long QT syndrome

Description

In case of sudden death or unusual collapse in a young patient, you should think about HOCM as the
most common cause, especially if there is a family history of the sudden death of a first-degree
relative

Page - 81
Internal Medicine - Cardiology

Question 38/155

Question #38

A 22-year-old male patient with a history of syncope and palpitation was diagnosed with HOCM. His
24-hour Holter monitoring reveals non-sustained ventricular tachycardia. What is the most
appropriate for his management?

a. Start amiodarone
b. Place an ICD
c. Start Sotalol
d. Ablation of the accessory bundle
e. Observation

‫اﻹﺟﺎﺑﺔ ﻋﲆ اﻟﺼﻔﺤﺔ اﻟﺘﺎﻟﻴﺔ‬

Page - 82
Internal Medicine - Cardiology - Cardiomyopathies

Question 38/155

Question #38

A 22-year-old male patient with a history of syncope and palpitation was diagnosed with HOCM. His
24-hour Holter monitoring reveals non-sustained ventricular tachycardia. What is the most
appropriate for his management?

a. Start amiodarone
b. Place an ICD √
c. Start Sotalol
d. Ablation of the accessory bundle
e. Observation

Description

e most eective means of preventing sudden death in high-risk patients with hypertrophic
cardiomyopathy is an implantable cardioverter-defibrillator

Page - 83
Internal Medicine - Cardiology

Question 39/155

Question #39

A 17-year-old male patient presents with recurrent syncopal attacks; the last occurred while running
for a bus. His father died suddenly at the age of 38-year-old. His ECG shows le ventricular
hypertrophy and widespread T-wave inversion. What is the most appropriate investigation to be
done next in managing this patient?

a. Coronary angiography
b. Transthoracic echocardiogram
c. Transesophageal echocardiogram
d. ECG exercise stress test
e. 24-hour Holter monitor

‫اﻹﺟﺎﺑﺔ ﻋﲆ اﻟﺼﻔﺤﺔ اﻟﺘﺎﻟﻴﺔ‬

Page - 84
Internal Medicine - Cardiology - Cardiomyopathies

Question 39/155

Question #39

A 17-year-old male patient presents with recurrent syncopal attacks; the last occurred while running
for a bus. His father died suddenly at the age of 38-year-old. His ECG shows le ventricular
hypertrophy and widespread T-wave inversion. What is the most appropriate investigation to be
done next in managing this patient?

a. Coronary angiography
b. Transthoracic echocardiogram √
c. Transesophageal echocardiogram
d. ECG exercise stress test
e. 24-hour Holter monitor

Description

A transthoracic echocardiogram will be the next step in the management of HOCM. Echocardiogram
in HOCM will show the following features:

Mitral regurgitation
Systolic anterior motion of the anterior mitral valve leaflet
Asymmetrical hypertrophy

Page - 85
Internal Medicine - Cardiology

Question 40/155

Question #40

A 17-year-old male patient was diagnosed with asymptomatic hypertrophic cardiomyopathy. Which
of the following is the most eective in preventing sudden death in his condition?

a. Insertable cardiac monitor


b. Verapamil
c. Metoprolol
d. Implantable cardioverter defibrillator
e. Amiodaron

‫اﻹﺟﺎﺑﺔ ﻋﲆ اﻟﺼﻔﺤﺔ اﻟﺘﺎﻟﻴﺔ‬

Page - 86
Internal Medicine - Cardiology - Cardiomyopathies

Question 40/155

Question #40

A 17-year-old male patient was diagnosed with asymptomatic hypertrophic cardiomyopathy. Which
of the following is the most eective in preventing sudden death in his condition?

a. Insertable cardiac monitor


b. Verapamil
c. Metoprolol
d. Implantable cardioverter defibrillator √
e. Amiodaron

Description

ICD is the most eective means of preventing sudden death in high-risk patients with HOCM

Medications (verapamil, beta-blockers, and diltiazem) are used for symptom management but do
not decrease the risk of sudden death.

Amiodarone may decrease the risk of sudden death because of its eects on reducing dysrhythmias,
but it is less than ICD

Note that an Insertable cardiac monitor is a diagnostic tool implanted under the skin and records
cardiac events for up to 30 years. It is used to diagnose arrhythmias but isn’t used to treat acute
arrhythmia and to prevent sudden death

Page - 87
Internal Medicine - Cardiology

Question 41/155

Question #41

A 17-year-old male patient was diagnosed with asymptomatic hypertrophic cardiomyopathy.


Unfortunately, he died at 28 while running for a bus. His mother came to consult you about this
condition. What is the most appropriate advice for the patient’s mother?

a. His brother can participate in a non-contact sport


b. Cardiac catheterization should be performed immediately for his brother
c. e hypertrophy, in this case, regresses with age
d. His brother should undergo echocardiography
e. ere is no familial tendency in this condition

‫اﻹﺟﺎﺑﺔ ﻋﲆ اﻟﺼﻔﺤﺔ اﻟﺘﺎﻟﻴﺔ‬

Page - 88
Internal Medicine - Cardiology - Cardiomyopathies

Question 41/155

Question #41

A 17-year-old male patient was diagnosed with asymptomatic hypertrophic cardiomyopathy.


Unfortunately, he died at 28 while running for a bus. His mother came to consult you about this
condition. What is the most appropriate advice for the patient’s mother?

a. His brother can participate in a non-contact sport


b. Cardiac catheterization should be performed immediately for his brother
c. e hypertrophy, in this case, regresses with age
d. His brother should undergo echocardiography √
e. ere is no familial tendency in this condition

Description

is patient died due to HOCM. HOCM is an autosomal dominant condition that may lead to sudden
death, so siblings should be monitored for the disease by doing a two-dimensional echocardiogram.

Patient with suspected HOMC should avoid strenuous sports, even those considered non-contact, to
prevent sudden death until he is properly evaluated.

Cardiac catheterization should be performed in patients with HCM who have angina, syncope,
resuscitated sudden death, or a worrisome stress test.

Page - 89
Internal Medicine - Cardiology

Question 42/155

Question #42

A 66-year-old male has exertional dyspnea, lower limb edema, and orthopnea. His echocardiogram
demonstrates an ejection fraction of 23% and a thin-walled enlarged le ventricle. What is the most
likely diagnosis?

a. Arrhythmogenic right ventricular dysplasia


b. Dilated cardiomyopathy
c. Hypertrophic cardiomyopathy
d. Primary pulmonary HTN
e. Restrictive cardiomyopathy

‫اﻹﺟﺎﺑﺔ ﻋﲆ اﻟﺼﻔﺤﺔ اﻟﺘﺎﻟﻴﺔ‬

Page - 90
Internal Medicine - Cardiology - Cardiomyopathies

Question 42/155

Question #42

A 66-year-old male has exertional dyspnea, lower limb edema, and orthopnea. His echocardiogram
demonstrates an ejection fraction of 23% and a thin-walled enlarged le ventricle. What is the most
likely diagnosis?

a. Arrhythmogenic right ventricular dysplasia


b. Dilated cardiomyopathy √
c. Hypertrophic cardiomyopathy
d. Primary pulmonary HTN
e. Restrictive cardiomyopathy

Description

ese are typical echocardiogram findings of dilated cardiomyopathy (DCM).

e presence of an enlarged LV chamber and thin wall are suggestive of DCM

Hypertrophic cardiomyopathy will demonstrate hypertrophied wall of the LV

Restrictive cardiomyopathy will show reduced ventricular volume, normal le ventricular wall
thickness, normal systolic function, and impaired ventricular filling.

Page - 91
Internal Medicine - Cardiology

Question 43/155

Question #43

A 21-year-old male patient was brought to the emergency department aer he collapsed in the
street. He is not responsive, has a dilated right pupil, and is not breathing. What is the most
appropriate step to take?

a. Brain CT scan without contrast


b. Brain MRI
c. Immediate intravenous fluid administration
d. Immediate intubation
e. Intravenous mannitol

‫اﻹﺟﺎﺑﺔ ﻋﲆ اﻟﺼﻔﺤﺔ اﻟﺘﺎﻟﻴﺔ‬

Page - 92
Internal Medicine - Cardiology - Cardiopulmonary resuscitation (CPR)

Question 43/155

Question #43

A 21-year-old male patient was brought to the emergency department aer he collapsed in the
street. He is not responsive, has a dilated right pupil, and is not breathing. What is the most
appropriate step to take?

a. Brain CT scan without contrast


b. Brain MRI
c. Immediate intravenous fluid administration
d. Immediate intubation √
e. Intravenous mannitol

Description

If any patient is unresponsive and not breathing, you should start the BLS protocol of CAB
(compression, airway, and breathing).

Note that the order of CPR steps rearranged to become CAB instead of ABC

Intubation here is the best choice to answer

Page - 93
Internal Medicine - Cardiology

Question 44/155

Question #44

While performing CPR on an adult by two rescuers, the ratio of compression to respiration should
be:

a. 3:1
b. 10:3
c. 15:1
d. 30:1
e. 30:2

‫اﻹﺟﺎﺑﺔ ﻋﲆ اﻟﺼﻔﺤﺔ اﻟﺘﺎﻟﻴﺔ‬

Page - 94
Internal Medicine - Cardiology - Cardiopulmonary resuscitation (CPR)

Question 44/155

Question #44

While performing CPR on an adult by two rescuers, the ratio of compression to respiration should
be:

a. 3:1
b. 10:3
c. 15:1
d. 30:1
e. 30:2 √

Description

It is reasonable for rescuers trained in CPR using chest compressions and ventilation (rescue breaths)
to provide a compression-to-ventilation ratio of 30:2 for adults in cardiac arrest

Aer placement of an advanced airway, it is reasonable for the provider to deliver 1 breath every 6
seconds (10 breaths per min) while continuous chest compressions are being performed

You should minimize interruptions in chest compressions when placing or ventilating with an
advanced airway. In addition, excessive ventilation should be avoided.

Page - 95
Internal Medicine - Cardiology

Question 45/155

Question #45

You are a part of a 3-persons team providing cardiopulmonary resuscitation for a 16-year-old guy,
and no advanced airway; according to the AHA recommendations, the compression-to-ventilation
ratio should be:

a. 30:2
b. 15:2
c. 15:1
d. 30:1
e. 30:15

‫اﻹﺟﺎﺑﺔ ﻋﲆ اﻟﺼﻔﺤﺔ اﻟﺘﺎﻟﻴﺔ‬

Page - 96
Internal Medicine - Cardiology - Cardiopulmonary resuscitation (CPR)

Question 45/155

Question #45

You are a part of a 3-persons team providing cardiopulmonary resuscitation for a 16-year-old guy,
and no advanced airway; according to the AHA recommendations, the compression-to-ventilation
ratio should be:

a. 30:2 √
b. 15:2
c. 15:1
d. 30:1
e. 30:15

Description

According to the recent AHA recommendations for CPR in adults, if there is no advanced airway, the
compression-to-ventilation ratio should be 30:2 regardless of the number of rescuers.

Note that the scenario will be dierent in children < 12 years old.

Compression-to-ventilation ratio:

For adults:
No advanced airway: the ratio should be 30:2 regardless of the number of rescuers.
If an advanced airway is present: continuous compression with one ventilation every 5 –
6 seconds
For pediatrics (>12 years old):
No advanced airway one rescuer, the ratio should be 30:2
No advanced airway and ≥ 2 rescuers: the ratio should be 15:2
If an advanced airway is present, continuous compression with one ventilation every 2 –
3 seconds
Excessive ventilation should be avoided

Page - 97
Internal Medicine - Cardiology

Question 46/155

Question #46

A 5-year-old male patient is found to have pulseless electrical activity (PEA). You started CPR
immediately. Which of the following is not a cause of PEA?

a. Cardiac tamponade
b. Tension pneumothorax
c. Hyperglycemia
d. Hypothermia
e. Hyperkalemia

‫اﻹﺟﺎﺑﺔ ﻋﲆ اﻟﺼﻔﺤﺔ اﻟﺘﺎﻟﻴﺔ‬

Page - 98
Internal Medicine - Cardiology - Cardiopulmonary resuscitation (CPR)

Question 46/155

Question #46

A 5-year-old male patient is found to have pulseless electrical activity (PEA). You started CPR
immediately. Which of the following is not a cause of PEA?

a. Cardiac tamponade
b. Tension pneumothorax
c. Hyperglycemia √
d. Hypothermia
e. Hyperkalemia

Description

Hypoglycemia, not hyperglycemia, is a cause of PEA

the possible causes of PEA are shown in the table below:

Page - 99
Internal Medicine - Cardiology

Question 47/155

Question #47

A 35-year-old female presents to your department with severe retrosternal chest pain; the pain is
aggravated by lying down and bending forward and relieved by aluminum-containing antacids,
with no dysphagia or shortness of breath. What is the most likely diagnosis?

a. Unstable angina
b. Reflux disease
c. Achalasia
d. Costochondritis
e. Pericarditis

‫اﻹﺟﺎﺑﺔ ﻋﲆ اﻟﺼﻔﺤﺔ اﻟﺘﺎﻟﻴﺔ‬

Page - 100
Internal Medicine - Cardiology - Chest Pain

Question 47/155

Question #47

A 35-year-old female presents to your department with severe retrosternal chest pain; the pain is
aggravated by lying down and bending forward and relieved by aluminum-containing antacids,
with no dysphagia or shortness of breath. What is the most likely diagnosis?

a. Unstable angina
b. Reflux disease √
c. Achalasia
d. Costochondritis
e. Pericarditis

Description

It is unlikely to see IHD in young females. e chest pain is atypical because it changes with body
position and is relieved by antacid

e most likely diagnosis here is GERD

Features of typical chest pain:

Central, retrosternal
Heavy, dull, squeezing in character, or pressure-like.
Radiated to neck, jaw, epigastrium, shoulders, or back
Increased by exercise and relieved by rest or nitrates

Page - 101
Internal Medicine - Cardiology

Question 48/155

Question #48

e following mechanisms can aggravate the ischemic chest pain by increasing myocardial oxygen
demand except:

a. Increased myocardial contractility


b. Aortic valve stenosis
c. Increased the heart rate
d. Ventricular hypertrophy
e. Decreased systemic vascular resistance

‫اﻹﺟﺎﺑﺔ ﻋﲆ اﻟﺼﻔﺤﺔ اﻟﺘﺎﻟﻴﺔ‬

Page - 102
Internal Medicine - Cardiology - Chest Pain

Question 48/155

Question #48

e following mechanisms can aggravate the ischemic chest pain by increasing myocardial oxygen
demand except:

a. Increased myocardial contractility


b. Aortic valve stenosis
c. Increased the heart rate
d. Ventricular hypertrophy
e. Decreased systemic vascular resistance √

Description

Ischemic pain of the heart develops when the oxygen demand by the myocardium exceeds the
supply to it by coronary arteries either due to

coronary artery disease or blockage due to atherosclerosis or clot formation.

Increased “not decreased” systemic vascular resistance is responsible for more workload on the
heart to pump blood; thus, more oxygen is required for the process

Page - 103
Internal Medicine - Cardiology

Question 49/155

Question #49

A non-smoker 32-year-old female presents to you with sharp, stabbing chest pain at the le sternal
border. e pain is non-radiating and aggravated by coughing and palpation. ere is no history or
family history of DM, HTN, or IHD. Her physical examination, ECG, and chest x-ray are otherwise
normal. What is the next step in the evaluation of this patient?

a. Admission for serial ECG and cardiac enzymes


b. Exercise stress test
c. No additional testing
d. Refer to cardiac catheterization
e. Start aspirin and clopidogrel

‫اﻹﺟﺎﺑﺔ ﻋﲆ اﻟﺼﻔﺤﺔ اﻟﺘﺎﻟﻴﺔ‬

Page - 104
Internal Medicine - Cardiology - Chest Pain

Question 49/155

Question #49

A non-smoker 32-year-old female presents to you with sharp, stabbing chest pain at the le sternal
border. e pain is non-radiating and aggravated by coughing and palpation. ere is no history or
family history of DM, HTN, or IHD. Her physical examination, ECG, and chest x-ray are otherwise
normal. What is the next step in the evaluation of this patient?

a. Admission for serial ECG and cardiac enzymes


b. Exercise stress test
c. No additional testing √
d. Refer to cardiac catheterization
e. Start aspirin and clopidogrel

Description

is patient has atypical chest pain (non-cardiac). It is most likely due to a musculoskeletal problem.
No risk factors for IHD and the probability of coronary diseases is very low.

Features of typical chest pain:

Central, retrosternal
Heavy, dull, squeezing in character, or pressure-like.
Radiated to neck, jaw, epigastrium, shoulders, or back
Increased by exercise and relieved by rest or nitrates

Page - 105
Internal Medicine - Cardiology

Question 50/155

Question #50

A 26-year-old male patient had severe chest pain, diaphoresis, and HTN for 2 hours while at a party.
His friends told you that the patient was hyper during the party. His blood pressure is 165/115 mmHg,
his pulse is 115 bpm, and his pupils are dilated bilaterally. His ECG shows diuse ST elevation. Which
of the following is contraindicated in the treatment of this patient?

a. Verapamil
b. Propranolol
c. Enalapril
d. Valsartan
e. Hydrochlorothiazide

‫اﻹﺟﺎﺑﺔ ﻋﲆ اﻟﺼﻔﺤﺔ اﻟﺘﺎﻟﻴﺔ‬

Page - 106
Internal Medicine - Cardiology - Cocaine-induced Chest Pain

Question 50/155

Question #50

A 26-year-old male patient had severe chest pain, diaphoresis, and HTN for 2 hours while at a party.
His friends told you that the patient was hyper during the party. His blood pressure is 165/115 mmHg,
his pulse is 115 bpm, and his pupils are dilated bilaterally. His ECG shows diuse ST elevation. Which
of the following is contraindicated in the treatment of this patient?

a. Verapamil
b. Propranolol √
c. Enalapril
d. Valsartan
e. Hydrochlorothiazide

Description

is patient is most likely to have a cocaine overdose. e clues here are “the party, the chest pain
with ST-elevation, and feeling hyper during the party.”

Beta-blockers are contraindicated in the case of a cocaine overdose.

Cocaine inhibits the reuptake of norepinephrine, and there is a strong alpha-1-receptor activity
causing systemic and coronary vasoconstriction.

If we inhibit the vasodilatory action using propranolol, there is unopposed alpha-1 activity leading to
coronary vasoconstriction and worsening myocardial ischemia.

Page - 107
Internal Medicine - Cardiology

Question 51/155

Question #51

A heavy smoker 32-year-old male patient presents to the ER with 45 minutes history of pressure-like
chest pain that radiates to the le shoulder. e pain is associated with dyspnea, anxiety,
palpitations, and sweating. He admits that he used intranasal cocaine 2 hours ago. His blood
pressure is 185/114, pulse 110, and ECG shows sinus tachycardia with early repolarization. You
administered aspirin, nitrates, and oxygen. Which of the following is indicated at this point?

a. Captopril
b. Lorazepam
c. Metoprolol
d. Nifedipine
e. Tissue plasminogen activator

‫اﻹﺟﺎﺑﺔ ﻋﲆ اﻟﺼﻔﺤﺔ اﻟﺘﺎﻟﻴﺔ‬

Page - 108
Internal Medicine - Cardiology - Cocaine-induced Chest Pain

Question 51/155

Question #51

A heavy smoker 32-year-old male patient presents to the ER with 45 minutes history of pressure-like
chest pain that radiates to the le shoulder. e pain is associated with dyspnea, anxiety,
palpitations, and sweating. He admits that he used intranasal cocaine 2 hours ago. His blood
pressure is 185/114, pulse 110, and ECG shows sinus tachycardia with early repolarization. You
administered aspirin, nitrates, and oxygen. Which of the following is indicated at this point?

a. Captopril
b. Lorazepam √
c. Metoprolol
d. Nifedipine
e. Tissue plasminogen activator

Description

In cocaine-induced chest pain, Hypertension, tachycardia, and chest pain will oen respond to
intravenous benzodiazepines.

e use of beta-blockers is contraindicated in this case as they aggravate the vasospasms

Calcium channel blockers can be used in this situation if the patient is not responding to
benzodiazepines and nitrates

Page - 109
Internal Medicine - Cardiology

Question 52/155

Question #52

A 30-year-old male is brought to the ER with a crushing sensation in his chest, and his nasal mucous
membranes are found to be dry due to suspected cocaine use. e following choices about cocaine-
induced chest pain are false except:

a. e patient should be treated with beta-blockers


b. e main mechanism is a dopamine-dependent state resulting in vasospasm
c. ere are predicted ECG abnormalities
d. Transient ECG changes will exclude the diagnoses of cocaine toxicity
e. A high dose of cocaine is required to cause this condition

‫اﻹﺟﺎﺑﺔ ﻋﲆ اﻟﺼﻔﺤﺔ اﻟﺘﺎﻟﻴﺔ‬

Page - 110
Internal Medicine - Cardiology - Cocaine-induced Chest Pain

Question 52/155

Question #52

A 30-year-old male is brought to the ER with a crushing sensation in his chest, and his nasal mucous
membranes are found to be dry due to suspected cocaine use. e following choices about cocaine-
induced chest pain are false except:

a. e patient should be treated with beta-blockers


b. e main mechanism is a dopamine-dependent state resulting in vasospasm √
c. ere are predicted ECG abnormalities
d. Transient ECG changes will exclude the diagnoses of cocaine toxicity
e. A high dose of cocaine is required to cause this condition

Description

e main mechanism of cocaine-induced chest pain is believed to be due to a dopamine-dependent


state resulting in coronary vasospasm.

Beta-blockers are contraindicated in case of cocaine-induced chest pain because this will aggravate
the vasospasm

ECG changes are variable and may be persistent ST elevation or maybe transient changes

Page - 111
Internal Medicine - Cardiology

Question 53/155

Question #53

A patient presents to you with persistent ST elevation aer sustaining a myocardial infarction 2
years ago. Which of the following is the most likely diagnosis?

a. Pericarditis
b. LV aneurysm
c. Re-infarction
d. Papillary muscle rupture
e. LV free wall rupture

‫اﻹﺟﺎﺑﺔ ﻋﲆ اﻟﺼﻔﺤﺔ اﻟﺘﺎﻟﻴﺔ‬

Page - 112
Internal Medicine - Cardiology - Complications of MI

Question 53/155

Question #53

A patient presents to you with persistent ST elevation aer sustaining a myocardial infarction 2
years ago. Which of the following is the most likely diagnosis?

a. Pericarditis
b. LV aneurysm √
c. Re-infarction
d. Papillary muscle rupture
e. LV free wall rupture

Description

Complications of MI are shown in the following table

Page - 113
Internal Medicine - Cardiology

Question 54/155

Question #54

12-hours aer myocardial infarction, a 45-year-old man develops shortness of breath, mued heart
sound, raised JVP, and hypotension. His ECG is shown below. Which of the following is the most
likely diagnosis?

a. Re-infarction
b. Papillary muscle rupture
c. LV free wall rupture
d. LV aneurysm
e. Dressler’s syndrome

‫اﻹﺟﺎﺑﺔ ﻋﲆ اﻟﺼﻔﺤﺔ اﻟﺘﺎﻟﻴﺔ‬

Page - 114
Internal Medicine - Cardiology - Complications of MI

Question 54/155

Question #54

12-hours aer myocardial infarction, a 45-year-old man develops shortness of breath, mued heart
sound, raised JVP, and hypotension. His ECG is shown below. Which of the following is the most
likely diagnosis?

a. Re-infarction
b. Papillary muscle rupture
c. LV free wall rupture √
d. LV aneurysm
e. Dressler’s syndrome

Description

Complications of MI are shown in the following table

Page - 115
Page - 116
Internal Medicine - Cardiology

Question 55/155

Question #55

A 63-year-old male patient with a history of systolic heart failure presents for routine evaluation. He
is on Aldactone and carvedilol. However, his Enalapril was stopped 3 weeks ago because he was
diagnosed with chronic kidney disease. Which medications are allowed and have mortality and
symptomatic benefits for this patient?

a. Digoxin
b. Nitrate
c. Hydralazine/Isosorbide
d. Losartan
e. Amlodipine

‫اﻹﺟﺎﺑﺔ ﻋﲆ اﻟﺼﻔﺤﺔ اﻟﺘﺎﻟﻴﺔ‬

Page - 117
Internal Medicine - Cardiology - Congestive heart failure (CHF)

Question 55/155

Question #55

A 63-year-old male patient with a history of systolic heart failure presents for routine evaluation. He
is on Aldactone and carvedilol. However, his Enalapril was stopped 3 weeks ago because he was
diagnosed with chronic kidney disease. Which medications are allowed and have mortality and
symptomatic benefits for this patient?

a. Digoxin
b. Nitrate
c. Hydralazine/Isosorbide √
d. Losartan
e. Amlodipine

Description

ACE inhibitors and beta-blockers are the usual treatment for heart failure. Patients who are
intolerant to ACE inhibitors due to hyperkalemia should take a combination of hydralazine and oral
nitrates

Hydralazine and oral nitrates combination has both mortality and symptomatic benefits

Digoxin and nitrates have only symptomatic benefits in heart failure.

ARBs have the same risk of Hyperkalemia as ACE inhibitors in this patient and should be avoided.

Amlodipine will increase mortality and lower limb edema in patients with systolic heart failure.

Page - 118
Internal Medicine - Cardiology

Question 56/155

Question #56

e ejection fraction that matches the diagnosis of diastolic heart failure should be greater than:

a. 25%
b. 35%
c. 45%
d. 55%
e. 75%

‫اﻹﺟﺎﺑﺔ ﻋﲆ اﻟﺼﻔﺤﺔ اﻟﺘﺎﻟﻴﺔ‬

Page - 119
Internal Medicine - Cardiology - Congestive heart failure (CHF)

Question 56/155

Question #56

e ejection fraction that matches the diagnosis of diastolic heart failure should be greater than:

a. 25%
b. 35%
c. 45%
d. 55% √
e. 75%

Description

Diastolic heart failure is also known as heart failure with preserved ejection fraction. e ejection
fraction, in this case, will be normal (>55%)

Page - 120
Internal Medicine - Cardiology

Question 57/155

Question #57

A 66-year-old male patient has a history of HTN, CKD, and heart failure (last Ejection Fraction is
34%). He presents with shortness of breath and fatigue on exertion. His medications include
furosemide and metoprolol. He used to be on Enalapril but was stopped because of hyperkalemia.
Which drugs will have the most mortality and symptomatic benefit in this patient?

a. Digoxin
b. Hydralazine/Isosorbide
c. Nifedipine
d. Valsartan
e. Verapamil

‫اﻹﺟﺎﺑﺔ ﻋﲆ اﻟﺼﻔﺤﺔ اﻟﺘﺎﻟﻴﺔ‬

Page - 121
Internal Medicine - Cardiology - Congestive heart failure (CHF)

Question 57/155

Question #57

A 66-year-old male patient has a history of HTN, CKD, and heart failure (last Ejection Fraction is
34%). He presents with shortness of breath and fatigue on exertion. His medications include
furosemide and metoprolol. He used to be on Enalapril but was stopped because of hyperkalemia.
Which drugs will have the most mortality and symptomatic benefit in this patient?

a. Digoxin
b. Hydralazine/Isosorbide √
c. Nifedipine
d. Valsartan
e. Verapamil

Description

ACE inhibitors and beta-blockers are the usual treatment for heart failure. Patients who are
intolerant to ACE inhibitors due to hyperkalemia should take a combination of hydralazine and oral
nitrates

Hydralazine and oral nitrates combination has both mortality and symptomatic benefits

Page - 122
Internal Medicine - Cardiology

Question 58/155

Question #58

A 50-year-old male patient is on captopril for his heart failure. In addition, he has dyspnea on
significant exertion. Which of the following agents should be added to improve the survival of this
patient?

a. Amiodarone
b. Beta-blockers
c. Digitalis
d. Verapamil
e. Warfarin

‫اﻹﺟﺎﺑﺔ ﻋﲆ اﻟﺼﻔﺤﺔ اﻟﺘﺎﻟﻴﺔ‬

Page - 123
Internal Medicine - Cardiology - Congestive heart failure (CHF)

Question 58/155

Question #58

A 50-year-old male patient is on captopril for his heart failure. In addition, he has dyspnea on
significant exertion. Which of the following agents should be added to improve the survival of this
patient?

a. Amiodarone
b. Beta-blockers √
c. Digitalis
d. Verapamil
e. Warfarin

Description

Beta-blockers are recommended to reduce the mortality rate in patients with heart failure. But beta-
blockers should be avoided in case of acute decompensated heart failure because they will suppress
the compensatory tachycardia and worsen the condition of the patient

Page - 124
Internal Medicine - Cardiology

Question 59/155

Question #59

A 68-year-old male patient presents with palpitations and exertional dyspnea. His blood pressure is
146/89. His pulse is 110 and regular. His echocardiogram shows a high le ventricular end-diastolic
volume and high ejection fraction. What investigation would be expected to be most beneficial in
the diagnosis?

a. Cardiac enzymes
b. Kidney function test
c. yroid function test
d. Liver function test
e. Pulmonary function test

‫اﻹﺟﺎﺑﺔ ﻋﲆ اﻟﺼﻔﺤﺔ اﻟﺘﺎﻟﻴﺔ‬

Page - 125
Internal Medicine - Cardiology - Congestive heart failure (CHF)

Question 59/155

Question #59

A 68-year-old male patient presents with palpitations and exertional dyspnea. His blood pressure is
146/89. His pulse is 110 and regular. His echocardiogram shows a high le ventricular end-diastolic
volume and high ejection fraction. What investigation would be expected to be most beneficial in
the diagnosis?

a. Cardiac enzymes
b. Kidney function test
c. yroid function test √
d. Liver function test
e. Pulmonary function test

Description

e high LVEDV and high ejection fraction suggests the diagnosis of increased cardiac output heart
failure

TSH is the investigation expected to have the greatest diagnostic yield. High cardiac output heart
failure is most likely due to thyrotoxicosis in this patient

Page - 126
Internal Medicine - Cardiology

Question 60/155

Question #60

Which of the following agents is indicated in a patient with congestive heart failure associated with
reduced ejection fraction?

a. Valsartan
b. Hydralazine
c. Warfarin
d. Digoxin
e. Amiodarone

‫اﻹﺟﺎﺑﺔ ﻋﲆ اﻟﺼﻔﺤﺔ اﻟﺘﺎﻟﻴﺔ‬

Page - 127
Internal Medicine - Cardiology - Congestive heart failure (CHF)

Question 60/155

Question #60

Which of the following agents is indicated in a patient with congestive heart failure associated with
reduced ejection fraction?

a. Valsartan √
b. Hydralazine
c. Warfarin
d. Digoxin
e. Amiodarone

Description

All patients with systolic CHF should receive treatment of ACE inhibitory or ARBs as they can
significantly reduce the mortality rate in such patients

Page - 128
Internal Medicine - Cardiology

Question 61/155

Question #61

A 66-year-old male with a history of HTN and IHD presents with exertional dyspnea and orthopnea.
On examination, he is found to have pedal edema and bibasilar rales. His chest x-ray demonstrates
cardiomegaly and lung congestion. His echocardiogram shows cardiomegaly and an ejection
fraction of 40%. Which of the following are indicated to reduce the mortality rate in this patient?

a. Digoxin
b. Frusemide
c. Amlodipine
d. Aldactone
e. iazide diuretics

‫اﻹﺟﺎﺑﺔ ﻋﲆ اﻟﺼﻔﺤﺔ اﻟﺘﺎﻟﻴﺔ‬

Page - 129
Internal Medicine - Cardiology - Congestive heart failure (CHF)

Question 61/155

Question #61

A 66-year-old male with a history of HTN and IHD presents with exertional dyspnea and orthopnea.
On examination, he is found to have pedal edema and bibasilar rales. His chest x-ray demonstrates
cardiomegaly and lung congestion. His echocardiogram shows cardiomegaly and an ejection
fraction of 40%. Which of the following are indicated to reduce the mortality rate in this patient?

a. Digoxin
b. Frusemide
c. Amlodipine
d. Aldactone √
e. iazide diuretics

Description

HF treatment with mortality benefits are: ACEI/ARB, Beta-blockers, Spironolactone, Eplerenone


(hydralazine + Nitrates), ICD

Digoxin and Loop Diuretics have no mortality benefits (only symptomatic)

Page - 130
Internal Medicine - Cardiology

Question 62/155

Question #62

A 62-year-old male patient with a known case of CHF presents for routine follow-up. He says he has
no problems in ordinary physical activities and can perform everyday activities without limitations.
According to the NYHA classification of heart failure, which class most accurately describes the
patient’s condition?

a. Class I
b. Class II
c. Class III
d. Class IV
e. Class V

‫اﻹﺟﺎﺑﺔ ﻋﲆ اﻟﺼﻔﺤﺔ اﻟﺘﺎﻟﻴﺔ‬

Page - 131
Internal Medicine - Cardiology - Congestive heart failure (CHF)

Question 62/155

Question #62

A 62-year-old male patient with a known case of CHF presents for routine follow-up. He says he has
no problems in ordinary physical activities and can perform everyday activities without limitations.
According to the NYHA classification of heart failure, which class most accurately describes the
patient’s condition?

a. Class I √
b. Class II
c. Class III
d. Class IV
e. Class V

Description

NYHA classification of HF:

Class I: no limitations on ordinary physical activities


Class II: slight limitation of ordinary physical activities
Class III: marked limitation of physical activity
Class IV: symptoms of CHF present at rest

ere is no class V in the NYHA classification of heart failure.

Page - 132
Internal Medicine - Cardiology

Question 63/155

Question #63

e following treatments of systolic le ventricular heart failure can lower the mortality rate except:

a. Aldactone
b. Digoxin
c. ACE inhibitors / ARBs
d. Beta-blockers
e. Sacubitril/Valsartan (entresto)

‫اﻹﺟﺎﺑﺔ ﻋﲆ اﻟﺼﻔﺤﺔ اﻟﺘﺎﻟﻴﺔ‬

Page - 133
Internal Medicine - Cardiology - Congestive heart failure (CHF)

Question 63/155

Question #63

e following treatments of systolic le ventricular heart failure can lower the mortality rate except:

a. Aldactone
b. Digoxin √
c. ACE inhibitors / ARBs
d. Beta-blockers
e. Sacubitril/Valsartan (entresto)

Description

HF treatment with mortality benefits are: ACEI/ARB, Beta-blockers, Spironolactone,


Eplerenone (hydralazine + Nitrates), ICD, Sacubitril/Valsartan
Digoxin and Loop Diuretics have no mortality benefits (only symptomatic)
CCB may increase the mortality rate and worsen edema in systolic heart failure but reduces
the mortality in isolated diastolic heart failure
Digoxin and spironolactone are not beneficial in diastolic heart failure

Page - 134
Internal Medicine - Cardiology

Question 64/155

Question #64

A 60-year-old male patient with chronic heart failure presents with shortness of breath, easy
fatigability, and along with pulmonary and peripheral edema. What is the pathophysiology of this
disease?

a. Concentric cardiac hypertrophy


b. High cardiac output
c. High coronary perfusion
d. Increased myocardial contractility
e. Physiologic hypertrophy or athlete’s heart

‫اﻹﺟﺎﺑﺔ ﻋﲆ اﻟﺼﻔﺤﺔ اﻟﺘﺎﻟﻴﺔ‬

Page - 135
Internal Medicine - Cardiology - Congestive heart failure (CHF)

Question 64/155

Question #64

A 60-year-old male patient with chronic heart failure presents with shortness of breath, easy
fatigability, and along with pulmonary and peripheral edema. What is the pathophysiology of this
disease?

a. Concentric cardiac hypertrophy √


b. High cardiac output
c. High coronary perfusion
d. Increased myocardial contractility
e. Physiologic hypertrophy or athlete’s heart

Description

Increasing the workload on the heart or injury on the heart muscles will lead to a compensatory
increase in the cell size and decreased contractility à the increase in the mass of the heart muscle
will lead to a reduction in the chamber size

Physiologic hypertrophy or athlete’s heart is a normal response of healthy exercise or pregnancy


(requiring more blood pumping) à enhance the pumping ability of the heart

Key point: cardiac hypertrophy related to stress will reduce the chamber size and the ability of the
heart to pump blood, while physiologic hypertrophy will lead to better heart function

Page - 136
Internal Medicine - Cardiology

Question 65/155

Question #65

A 42-year-old male patient presents to you for a pre-life insurance examination. You performed a 2-
dimensional Echocardiogram. Which of the following readings are considered normal ejection
fraction for this patient?

a. 35%
b. 45%
c. 65%
d. 85%
e. 90%

‫اﻹﺟﺎﺑﺔ ﻋﲆ اﻟﺼﻔﺤﺔ اﻟﺘﺎﻟﻴﺔ‬

Page - 137
Internal Medicine - Cardiology - Congestive heart failure (CHF)

Question 65/155

Question #65

A 42-year-old male patient presents to you for a pre-life insurance examination. You performed a 2-
dimensional Echocardiogram. Which of the following readings are considered normal ejection
fraction for this patient?

a. 35%
b. 45%
c. 65% √
d. 85%
e. 90%

Description

e severity of HF according to EF%:

Grade I (EF > 60%) (Normal)


Grade II (EF = 40-59%)
Grade III (EF = 21-39%)
Grade IV (EF ≤ 20%)

Page - 138
Internal Medicine - Cardiology

Question 66/155

Question #66

An asymptomatic 55-year-old male presents for a routine examination. His blood pressure is 155/98,
ECG shows sinus rhythm and le ventricular hypertrophy. An echocardiogram confirms the findings.
Which of the following is the best management of this patient at this time?

a. Reassurance
b. Start nifedipine
c. Diclofenac sodium as needed
d. Valsartan
e. Hydrochlorothiazide

‫اﻹﺟﺎﺑﺔ ﻋﲆ اﻟﺼﻔﺤﺔ اﻟﺘﺎﻟﻴﺔ‬

Page - 139
Internal Medicine - Cardiology - Congestive heart failure (CHF)

Question 66/155

Question #66

An asymptomatic 55-year-old male presents for a routine examination. His blood pressure is 155/98,
ECG shows sinus rhythm and le ventricular hypertrophy. An echocardiogram confirms the findings.
Which of the following is the best management of this patient at this time?

a. Reassurance
b. Start nifedipine
c. Diclofenac sodium as needed
d. Valsartan √
e. Hydrochlorothiazide

Description

ACE inhibitors and ARBs will result in regression of ventricular hypertrophy.

Note that ACE inhibitors, ARBs, beta-blockers, and thiazide diuretics are the most helpful drugs in
this setting

NSAIDs should be avoided as they will cause sodium and water retention leading to worsening of
HTN and heart failure

Page - 140
Internal Medicine - Cardiology

Question 67/155

Question #67

An intern doctor in the hospital wants to discuss the treatment options for heart failure. You told
him that the following conditions are contraindications for beta-blocker use in heart failure except:

a. e presence of heart block


b. Concomitant use of verapamil
c. Presence of bradycardia
d. Hemodynamic instability
e. NYHA class II heart failure

‫اﻹﺟﺎﺑﺔ ﻋﲆ اﻟﺼﻔﺤﺔ اﻟﺘﺎﻟﻴﺔ‬

Page - 141
Internal Medicine - Cardiology - Congestive heart failure (CHF)

Question 67/155

Question #67

An intern doctor in the hospital wants to discuss the treatment options for heart failure. You told
him that the following conditions are contraindications for beta-blocker use in heart failure except:

a. e presence of heart block


b. Concomitant use of verapamil
c. Presence of bradycardia
d. Hemodynamic instability
e. NYHA class II heart failure √

Description

Contraindications to beta-blocker use include:

Hemodynamic instability
Heart block
Bradycardia
Acute decompensated heart failure
Severe asthma (but can be used in Asthma and COPD patients if they are monitored closely for
possible exacerbations)

Page - 142
Internal Medicine - Cardiology

Question 68/155

Question #68

A patient presents to you complaining of a gradually increasing Shortness of breath, orthopnea,


recurrent syncope, and easy fatigability. What does this patient have?

a. Right-side heart failure


b. Le-side heart failure
c. Biventricular heart failure
d. Acute myocardial infarction
e. Chronic obstructive pulmonary disease

‫اﻹﺟﺎﺑﺔ ﻋﲆ اﻟﺼﻔﺤﺔ اﻟﺘﺎﻟﻴﺔ‬

Page - 143
Internal Medicine - Cardiology - Congestive heart failure (CHF)

Question 68/155

Question #68

A patient presents to you complaining of a gradually increasing Shortness of breath, orthopnea,


recurrent syncope, and easy fatigability. What does this patient have?

a. Right-side heart failure


b. Le-side heart failure √
c. Biventricular heart failure
d. Acute myocardial infarction
e. Chronic obstructive pulmonary disease

Description

is patient has signs and symptoms of le-sided heart failure

Symptoms of HF are as the following:

Symptoms of low cardiac output – le side (hypotension, syncope, cold extremities)
Symptoms of backward congestion, as the following:
Le-side heart failure: dyspnea, orthopnea, PND, pulmonary edema, pleural eusion,
cardiomegaly, le side S3/S4 Gallop rhythm.
Right-side heart failure: peripheral and lower limb edema, liver congestion,
hepatojugular reflux, ascites, high JVP, right side S3/S4 gallop rhythm

Page - 144
Internal Medicine - Cardiology

Question 69/155

Question #69

A 70-year-old male patient with an echocardiogram documented systolic dysfunction presents for
evaluation. His lab investigations are normal, including kidney and liver function tests. Which of the
following is the best initial medication for this patient?

a. Digoxin
b. Furosemide
c. Isosorbide dinitrates
d. Spironolactone
e. Valsartan

‫اﻹﺟﺎﺑﺔ ﻋﲆ اﻟﺼﻔﺤﺔ اﻟﺘﺎﻟﻴﺔ‬

Page - 145
Internal Medicine - Cardiology - Congestive heart failure (CHF)

Question 69/155

Question #69

A 70-year-old male patient with an echocardiogram documented systolic dysfunction presents for
evaluation. His lab investigations are normal, including kidney and liver function tests. Which of the
following is the best initial medication for this patient?

a. Digoxin
b. Furosemide
c. Isosorbide dinitrates
d. Spironolactone
e. Valsartan √

Description

ACE inhibitors or ARBs are indicated for all patients with systolic heart failure, regardless of the
severity. ese drugs are shown to reduce both mortality and morbidity in both symptomatic and
asymptomatic patients

Page - 146
Internal Medicine - Cardiology

Question 70/155

Question #70

Which of the following heart failure treatments will reduce both preload and aerload?

a. Dobutamine
b. Enalapril
c. Furosemide
d. Hydralazine
e. Isosorbide dinitrates

‫اﻹﺟﺎﺑﺔ ﻋﲆ اﻟﺼﻔﺤﺔ اﻟﺘﺎﻟﻴﺔ‬

Page - 147
Internal Medicine - Cardiology - Congestive heart failure (CHF)

Question 70/155

Question #70

Which of the following heart failure treatments will reduce both preload and aerload?

a. Dobutamine
b. Enalapril √
c. Furosemide
d. Hydralazine
e. Isosorbide dinitrates

Description

(ACE inhibitors, ARB’s, Prazosin, Nitroprusside) ese drugs improve cardiac output by reducing total
peripheral resistance and ventricular filling pressure by facilitating venous pooling.

Page - 148
Internal Medicine - Cardiology

Question 71/155

Question #71

A 55-year-old male patient with a history of HTN presents with gradually increasing shortness of breath and
easy fatigability. On examination, he has bilateral basilar crepitations and lower limb edema. However, his
vital signs are within normal limits. Which is the most preferred tool for evaluating this patient’s condition?

a. ECG
b. Chest x-ray
c. 2-dimensional Echocardiogram
d. Electrophysiological study
e. Cardiac MRI

‫اﻹﺟﺎﺑﺔ ﻋﲆ اﻟﺼﻔﺤﺔ اﻟﺘﺎﻟﻴﺔ‬

Page - 149
Internal Medicine - Cardiology - Congestive heart failure (CHF)

Question 71/155

Question #71

A 55-year-old male patient with a history of HTN presents with gradually increasing shortness of breath and
easy fatigability. On examination, he has bilateral basilar crepitations and lower limb edema. However, his
vital signs are within normal limits. Which is the most preferred tool for evaluating this patient’s condition?

a. ECG
b. Chest x-ray
c. 2-dimensional Echocardiogram √
d. Electrophysiological study
e. Cardiac MRI

Description

e most preferred diagnostic tool for evaluating heart failure is a 2-dimensional echocardiogram

is test can (not exclusively) demonstrate the following details:

e cause of heart failure:


Akinesia (infarction), hypokinesia (ischemia)
Structural heart disease (ASD, VSD, valvular stenosis, valvular insuciency)
e presence of heart failure:
e ejection fraction of the heart
e systolic and diastolic dysfunction
e presence of complications of heart failure:
Ventricular hypertrophy
Pericardial eusion

Page - 150
Internal Medicine - Cardiology

Question 72/155

Question #72

A patient with class III CHF, whose ejection fraction is 39%, asks you about using high-dose aspirin to
relieve aches and pains. What is the most appropriate advice you provide?

a. e patient should avoid NSAIDS, but high-dose aspirin is appropriate to use in heart failure
b. NSAIDs, including aspirin, are good choices as they enhance the diuretic eect
c. High dose Aspirin is preferred over other NSAIDs for this patient
d. NSAIDs, including high-dose aspirin, should be avoided in CHF patients because they can cause
fluid retention
e. Low-dose aspirin is indicated to relieve the pain and aches in a patient with CHF

‫اﻹﺟﺎﺑﺔ ﻋﲆ اﻟﺼﻔﺤﺔ اﻟﺘﺎﻟﻴﺔ‬

Page - 151
Internal Medicine - Cardiology - Congestive heart failure (CHF)

Question 72/155

Question #72

A patient with class III CHF, whose ejection fraction is 39%, asks you about using high-dose aspirin to
relieve aches and pains. What is the most appropriate advice you provide?

a. e patient should avoid NSAIDS, but high-dose aspirin is appropriate to use in heart failure
b. NSAIDs, including aspirin, are good choices as they enhance the diuretic eect
c. High dose Aspirin is preferred over other NSAIDs for this patient
d. NSAIDs, including high-dose aspirin, should be avoided in CHF patients because they can cause
fluid retention √
e. Low-dose aspirin is indicated to relieve the pain and aches in a patient with CHF

Description

NSAIDs should be avoided if possible in case of CHF because they can aggravate sodium and water
retention and will increase systemic vascular resistance, leading to increased workload on the heart
and thus increasing the mortality and symptoms in patients with congestive heart failure

Page - 152
Internal Medicine - Cardiology

Question 73/155

Question #73

A 59-year-old male patient with T2DM, HTN, and dyslipidemia presents with exertional dyspnea,
orthopnea, and pedal edema. His medications are atorvastatin, metformin, Aldactone, captopril,
and bisoprolol. His ejection fraction is 25%, his blood pressure is now 159/88, and his pulse is 72 bpm.
Which of the following agents are not important in the mortality reduction in this patient’s
condition?

a. Captopril
b. Bisoprolol
c. Aldactone
d. Implantable cardiac defibrillator
e. Digoxin

‫اﻹﺟﺎﺑﺔ ﻋﲆ اﻟﺼﻔﺤﺔ اﻟﺘﺎﻟﻴﺔ‬

Page - 153
Internal Medicine - Cardiology - Congestive heart failure (CHF)

Question 73/155

Question #73

A 59-year-old male patient with T2DM, HTN, and dyslipidemia presents with exertional dyspnea,
orthopnea, and pedal edema. His medications are atorvastatin, metformin, Aldactone, captopril,
and bisoprolol. His ejection fraction is 25%, his blood pressure is now 159/88, and his pulse is 72 bpm.
Which of the following agents are not important in the mortality reduction in this patient’s
condition?

a. Captopril
b. Bisoprolol
c. Aldactone
d. Implantable cardiac defibrillator
e. Digoxin √

Description

Digoxin can improve the symptoms and reduce the recurrence of hospitalization in patients with
heart failure, but it never prolongs survival or improve the mortality rate.

HF treatment with mortality benefits are: ACEI/ARB, Beta-blockers, Spironolactone,


Eplerenone (hydralazine + Nitrates), ICD, Sacubitril/Valsartan
Digoxin and Loop Diuretics have no mortality benefits (only symptomatic)
CCB may increase the mortality rate and worsen edema in systolic heart failure but reduces
the mortality in isolated diastolic heart failure
Digoxin and spironolactone are not beneficial in diastolic heart failure

Page - 154
Internal Medicine - Cardiology

Question 74/155

Question #74

A 56-year-old male patient is on lisinopril, digoxin, Aldactone, and beta-blockers to treat heart
failure. Recently he was symptomatic, and furosemide was added. Today he presents with nausea,
vomiting, and palpitations. What is the most logical diagnosis that will jump into your mind?

a. Myocardial infarction
b. Digoxin toxicity
c. Hyperkalemia induced arrhythmia
d. Hyperthyroidism
e. Atrial fibrillation

‫اﻹﺟﺎﺑﺔ ﻋﲆ اﻟﺼﻔﺤﺔ اﻟﺘﺎﻟﻴﺔ‬

Page - 155
Internal Medicine - Cardiology - Digoxin and Digoxin toxicity

Question 74/155

Question #74

A 56-year-old male patient is on lisinopril, digoxin, Aldactone, and beta-blockers to treat heart
failure. Recently he was symptomatic, and furosemide was added. Today he presents with nausea,
vomiting, and palpitations. What is the most logical diagnosis that will jump into your mind?

a. Myocardial infarction
b. Digoxin toxicity √
c. Hyperkalemia induced arrhythmia
d. Hyperthyroidism
e. Atrial fibrillation

Description

Digoxin toxicity is the most likely diagnosis here. It is most likely induced by hypokalemia due to
adding furosemide to the patient’s drugs

Page - 156
Internal Medicine - Cardiology

Question 75/155

Question #75

A patient with heart failure and chronic kidney diseases diagnosed to have digoxin toxicity, all the
following can manifest in this disease except:

a. Orthopnea
b. Xanthopsia
c. Severe bradycardia
d. rombocytopenia
e. Nausea and vomiting

‫اﻹﺟﺎﺑﺔ ﻋﲆ اﻟﺼﻔﺤﺔ اﻟﺘﺎﻟﻴﺔ‬

Page - 157
Internal Medicine - Cardiology - Digoxin and Digoxin toxicity

Question 75/155

Question #75

A patient with heart failure and chronic kidney diseases diagnosed to have digoxin toxicity, all the
following can manifest in this disease except:

a. Orthopnea √
b. Xanthopsia
c. Severe bradycardia
d. rombocytopenia
e. Nausea and vomiting

Description

Digoxin is used to treat orthopnea, dyspnea, wheezing, and ankle edema related to heart failure.

Digoxin has a narrow therapeutic range and can cause toxicity leading to the following features:

Anorexia
Anxiety
Atrioventricular block
Confusion
Delirium and hallucinations
Diarrhea, nausea, and vomiting
Dizziness and headache
Rash
Severe bradycardia
Tachycardia
rombocytopenia
Ventricular arrhythmias
Visual disturbance (Xanthopsia)

Page - 158
Internal Medicine - Cardiology

Question 76/155

Question #76

A 22-year-old male patient complains of palpitations that occur every 10 – 14 days. e palpitations
occur for about 20 seconds and resolve spontaneously. Which of the following is the most
appropriate cardiac study to order at this time?

a. Echocardiogram
b. Electrocardiogram
c. Electrophysiological study
d. Implantable loop recorder
e. Insertable Cardiac Monitor

‫اﻹﺟﺎﺑﺔ ﻋﲆ اﻟﺼﻔﺤﺔ اﻟﺘﺎﻟﻴﺔ‬

Page - 159
Internal Medicine - Cardiology - Dysrhythmias

Question 76/155

Question #76

A 22-year-old male patient complains of palpitations that occur every 10 – 14 days. e palpitations
occur for about 20 seconds and resolve spontaneously. Which of the following is the most
appropriate cardiac study to order at this time?

a. Echocardiogram
b. Electrocardiogram √
c. Electrophysiological study
d. Implantable loop recorder
e. Insertable Cardiac Monitor

Description

ECG is the first step in this scenario. You have to assess the rhythm and screen for chamber
enlargement or previous myocardial infarctions

Common types of cardiac monitoring:

Holter monitor: portable external monitor that is used to record cardiac activity for 24 – 48
hours
Event recorder: only record the cardiac activity when the patient activates it. It is worn on the
body for up to 30 days
Mobile cardiac telemetry: it automatically detects and records the heart rhythm for up to 30
days. It transmits the recordings to a mobile device to be reviewed by the physician
Insertable cardiac monitor: it is inserted under the skin and records cardiac events up to 30
years

Page - 160
Internal Medicine - Cardiology

Question 77/155

Question #77

A 42-year-old female patient presents with palpitations and dizziness for 2 hours duration. Her
blood pressure is 116/66, her pulse is 162 bpm, and her ECG is below. What is the treatment of choice
for her condition?

a. Digoxin PO
b. Digoxin IV
c. Synchronized Cardioversion
d. Adenosine
e. Verapamil

‫اﻹﺟﺎﺑﺔ ﻋﲆ اﻟﺼﻔﺤﺔ اﻟﺘﺎﻟﻴﺔ‬

Page - 161
Internal Medicine - Cardiology - Dysrhythmias

Question 77/155

Question #77

A 42-year-old female patient presents with palpitations and dizziness for 2 hours duration. Her
blood pressure is 116/66, her pulse is 162 bpm, and her ECG is below. What is the treatment of choice
for her condition?

a. Digoxin PO
b. Digoxin IV
c. Synchronized Cardioversion
d. Adenosine √
e. Verapamil

Description

Adenosine is the treatment of choice for stable patients with SVT.

It is contraindicated in asthma patient

It has a very short half-life

If the patient is not responsive to adenosine so verapamil can be used

If the patient is unstable, synchronized cardioversion is indicated

Digoxin is used to treat atrial fibrillation and heart failure

Supraventricular Tachycardia (SVT)

SVT: regular, no P wave, narrow QRS, and heart rate > 150 bpm
Causes: (Hypokalemia, Hyperthyroidism, Excessive alcohol)

Page - 162
Treatment of SVT:

For unstable patients: synchronized cardioversion.


For stable patients:
Step 1: Vagal maneuvers: (carotid massage, ocular massage, Ice on the forehead,
Valsalva maneuver)
Step 2: Adenosine (6, 12, then 12 mg IV push)
Step 3: I.V verapamil, Diltiazem, or metoprolol.
Step 4: synchronized cardioversion (if unresponsive)
Definitive treatment: catheter ablation.

Page - 163
Internal Medicine - Cardiology

Question 78/155

Question #78

A 78-year-old female with atrial fibrillation presents for evaluation. Her heart rate is 130 bpm with
irregular irregularity on ECG. What would you do in the management of this patient?

a. Rate control and warfarin


b. Rate control and no anticoagulation
c. Rhythm control and warfarin
d. Rhythm control and anticoagulation
e. Synchronized cardioversion and LMWH

‫اﻹﺟﺎﺑﺔ ﻋﲆ اﻟﺼﻔﺤﺔ اﻟﺘﺎﻟﻴﺔ‬

Page - 164
Internal Medicine - Cardiology - Dysrhythmias

Question 78/155

Question #78

A 78-year-old female with atrial fibrillation presents for evaluation. Her heart rate is 130 bpm with
irregular irregularity on ECG. What would you do in the management of this patient?

a. Rate control and warfarin √


b. Rate control and no anticoagulation
c. Rhythm control and warfarin
d. Rhythm control and anticoagulation
e. Synchronized cardioversion and LMWH

Description

In old patients with atrial fibrillation, you do not have to control the rhythm

Rhythm control will carry no more benefit than rate control, so you pay attention to the patient’s
symptoms.

According to the CHA2DS2VAS score, this patient has 2 points for age and 1 point for her gender, so
she needs to be put on anticoagulation to reduce the risk of stroke.

e following diagram shows the approach to the treatment of atrial fibrillation

Page - 165
Page - 166
Internal Medicine - Cardiology

Question 79/155

Question #79

A 60-year-old male presents with a pounding sensation in his chest. His ECG is shown in the
following figure. What does this patient have?

a. Atrial fibrillation
b. Atrial flutter
c. Ventricular fibrillation
d. Normal sinus rhythm
e. Ventricular tachycardia

‫اﻹﺟﺎﺑﺔ ﻋﲆ اﻟﺼﻔﺤﺔ اﻟﺘﺎﻟﻴﺔ‬

Page - 167
Internal Medicine - Cardiology - Dysrhythmias

Question 79/155

Question #79

A 60-year-old male presents with a pounding sensation in his chest. His ECG is shown in the
following figure. What does this patient have?

a. Atrial fibrillation √
b. Atrial flutter
c. Ventricular fibrillation
d. Normal sinus rhythm
e. Ventricular tachycardia

Description

e presence of irregular irregularity on ECG along with absent P wave is the main feature of atrial
fibrillation on ECG.

Page - 168
Internal Medicine - Cardiology

Question 80/155

Question #80

A 49-year-old male patient has developed dizziness and palpitations for the past 6 hours, and his
ECG is shown below. What is the most likely diagnosis?

a. Le bundle branch block


b. Right bundle branch block
c. Ventricular tachycardia
d. Extrasystoles
e. Normal sinus rhythm

‫اﻹﺟﺎﺑﺔ ﻋﲆ اﻟﺼﻔﺤﺔ اﻟﺘﺎﻟﻴﺔ‬

Page - 169
Internal Medicine - Cardiology - Dysrhythmias

Question 80/155

Question #80

A 49-year-old male patient has developed dizziness and palpitations for the past 6 hours, and his
ECG is shown below. What is the most likely diagnosis?

a. Le bundle branch block


b. Right bundle branch block
c. Ventricular tachycardia
d. Extrasystoles √
e. Normal sinus rhythm

Description

Extrasystole, premature ventricular contractions, also called ectopic beats. ey arise from a
ventricular focus. ey have bizarre shape-wide QRS. If they present alternatively every other beat,
they called bigeminy.

Page - 170
Internal Medicine - Cardiology

Question 81/155

Question #81

A 43-year-old female came with recurrent palpitations and chest discomfort. However, her ECG is
shown below. Her medications include Enalapril and spironolactone for hypertension and NYHA
class 1 heart failure. What is the anticoagulation of choice for her condition?

a. No anticoagulation needed
b. Start Warfarin therapy with an INR target of 2 – 3
c. Start Apixaban or dabigatran
d. Use only aspirin for this patient
e. e data in the question are not enough to decide

‫اﻹﺟﺎﺑﺔ ﻋﲆ اﻟﺼﻔﺤﺔ اﻟﺘﺎﻟﻴﺔ‬

Page - 171
Internal Medicine - Cardiology - Dysrhythmias

Question 81/155

Question #81

A 43-year-old female came with recurrent palpitations and chest discomfort. However, her ECG is
shown below. Her medications include Enalapril and spironolactone for hypertension and NYHA
class 1 heart failure. What is the anticoagulation of choice for her condition?

a. No anticoagulation needed
b. Start Warfarin therapy with an INR target of 2 – 3
c. Start Apixaban or dabigatran √
d. Use only aspirin for this patient
e. e data in the question are not enough to decide

Description

is patient has a CHA2DS2-VASc score of 3 (for her gender, history of HTN, and heart failure).

e first-line anticoagulation in patients with atrial fibrillation and high CHA2DS2-VASc score are
DOACs.

For many years, warfarin was the first line, but now DOACs are recommended, and warfarin has
become the second line for those who do not tolerate DOACs.

Aspirin is not recommended for reducing stroke risk in patients with AF.

e following table demonstrates the CHA2DS2-VASc score and its calculations.

Page - 172
Page - 173
Internal Medicine - Cardiology

Question 82/155

Question #82

An adult presents to you for follow-up. He was diagnosed with arrhythmia 3 years ago but is not
compliant with follow-up, and no medications have been taken for 3 years. His ECG is positive for
Delta wave and short PR interval. Which of the following is contraindicated in his condition?

a. Procainamide
b. Amiodarone
c. Digoxin
d. Nifedipine
e. Cardioversion

‫اﻹﺟﺎﺑﺔ ﻋﲆ اﻟﺼﻔﺤﺔ اﻟﺘﺎﻟﻴﺔ‬

Page - 174
Internal Medicine - Cardiology - Dysrhythmias

Question 82/155

Question #82

An adult presents to you for follow-up. He was diagnosed with arrhythmia 3 years ago but is not
compliant with follow-up, and no medications have been taken for 3 years. His ECG is positive for
Delta wave and short PR interval. Which of the following is contraindicated in his condition?

a. Procainamide
b. Amiodarone
c. Digoxin √
d. Nifedipine
e. Cardioversion

Description

Medications that may shorten the refractory period and enhance conduction over the bypass tract
(the bundle of Kent) are contraindicated to be given in patients with WPW Syndrome. is is because
they may cause even faster tachyarrhythmias or deterioration into ventricular fibrillation.

Page - 175
Internal Medicine - Cardiology

Question 83/155

Question #83

A 32-year-old male patient with diarrhea developed hypokalemia. He has been on an antihistamine
for recurrent itching and erythromycin for the last 4 days. He develops a brief episode of syncope
requiring cardiopulmonary resuscitation. Which of the following is the most likely responsible for
his condition?

a. Sinus bradycardia
b. Atrial fibrillation
c. Supraventricular tachycardia
d. Torsade de points
e. Asystole

‫اﻹﺟﺎﺑﺔ ﻋﲆ اﻟﺼﻔﺤﺔ اﻟﺘﺎﻟﻴﺔ‬

Page - 176
Internal Medicine - Cardiology - Dysrhythmias

Question 83/155

Question #83

A 32-year-old male patient with diarrhea developed hypokalemia. He has been on an antihistamine
for recurrent itching and erythromycin for the last 4 days. He develops a brief episode of syncope
requiring cardiopulmonary resuscitation. Which of the following is the most likely responsible for
his condition?

a. Sinus bradycardia
b. Atrial fibrillation
c. Supraventricular tachycardia
d. Torsade de points √
e. Asystole

Description

e presence of hypokalemia, antihistamine use, and erythromycin will raise the suspicion of long
QT syndrome and Torsade de Points.

Torsade de points (TdP):

It is also known as (twisting points)


It is associated with Long QT syndrome
May result in sudden death

Treatment of TdP:

For unstable patients: cardioversion


For stable patients: IV magnesium sulfate
Always correct the underlying cause.

Page - 177
Internal Medicine - Cardiology

Question 84/155

Question #84

A 68-year-old male patient presents because of palpitation; he is found to have atrial fibrillation on
ECG with a heart rate of 130 bpm, his vital signs are stable, and his echocardiogram reveals mitral
valve regurgitation and dilated le atrium. What is the best treatment option for this patient’s
condition?

a. Start a geriatric dose of digoxin


b. Start Atenolol
c. Start nifedipine
d. Start Aspirin therapy
e. Amiodarone

‫اﻹﺟﺎﺑﺔ ﻋﲆ اﻟﺼﻔﺤﺔ اﻟﺘﺎﻟﻴﺔ‬

Page - 178
Internal Medicine - Cardiology - Dysrhythmias

Question 84/155

Question #84

A 68-year-old male patient presents because of palpitation; he is found to have atrial fibrillation on
ECG with a heart rate of 130 bpm, his vital signs are stable, and his echocardiogram reveals mitral
valve regurgitation and dilated le atrium. What is the best treatment option for this patient’s
condition?

a. Start a geriatric dose of digoxin


b. Start Atenolol √
c. Start nifedipine
d. Start Aspirin therapy
e. Amiodarone

Description

is is a stable, elderly patient who needs rate control for his atrial fibrillation.

Digoxin will be the best choice if the patient has heart failure

Nifedipine will cause more tachycardia and should be avoided at this stage

Page - 179
Page - 180
Internal Medicine - Cardiology

Question 85/155

Question #85

A 55-year-old male patient presents to the ER with shortness of breath and palpitation. His ECG
shows irregular irregularity; the patient’s blood pressure is 85/50, and the pulse is 133 and irregular.
What would you do next in the management of this patient?

a. Amiodarone IV infusion
b. Start CPR
c. Asynchronized cardioversion
d. Synchronized cardioversion
e. Digoxin infusion

‫اﻹﺟﺎﺑﺔ ﻋﲆ اﻟﺼﻔﺤﺔ اﻟﺘﺎﻟﻴﺔ‬

Page - 181
Internal Medicine - Cardiology - Dysrhythmias

Question 85/155

Question #85

A 55-year-old male patient presents to the ER with shortness of breath and palpitation. His ECG
shows irregular irregularity; the patient’s blood pressure is 85/50, and the pulse is 133 and irregular.
What would you do next in the management of this patient?

a. Amiodarone IV infusion
b. Start CPR
c. Asynchronized cardioversion
d. Synchronized cardioversion √
e. Digoxin infusion

Description

Synchronized cardioversion is the treatment of choice for any patient with atrial fibrillation and
hemodynamic instability.

Page - 182
Internal Medicine - Cardiology

Question 86/155

Question #86

A 19-year-old male patient has palpitations and lightheadedness, which are related to exertion. In
addition, he has a family history of an autopsy-negative sudden death of his brother while playing
football, and his mother has recurrent syncope. Which of the following is the best initial test in
management?

a. Resting ECG
b. Stress ECG
c. Echocardiogram
d. Electrophysiological study
e. no further testing

‫اﻹﺟﺎﺑﺔ ﻋﲆ اﻟﺼﻔﺤﺔ اﻟﺘﺎﻟﻴﺔ‬

Page - 183
Internal Medicine - Cardiology - Dysrhythmias

Question 86/155

Question #86

A 19-year-old male patient has palpitations and lightheadedness, which are related to exertion. In
addition, he has a family history of an autopsy-negative sudden death of his brother while playing
football, and his mother has recurrent syncope. Which of the following is the best initial test in
management?

a. Resting ECG √
b. Stress ECG
c. Echocardiogram
d. Electrophysiological study
e. no further testing

Description

Family history of sudden death and recurrent syncope is highly suggestive of inherited long QT
syndrome.

HOCM will be positive in the autopsy of his brother. So, long QT syndrome is more likely.

Resting ECG showing the corrected QT of > 440 ms is the best to diagnose long QT syndrome

Long QT syndrome is a risk for sudden death in young patients, and the management includes beta-
blockers, ICD, and avoiding competitive sports

Page - 184
Internal Medicine - Cardiology

Question 87/155

Question #87

A 65-year-old male patient was evaluated at the emergency room for dizziness, and his ECG is
shown below. What is the most likely diagnosis?

a. Heart block 1st-degree


b. Heart block 2nd-degree type 1
c. Heart block 2nd-degree type 2
d. Complete heart block
e. Atrial fibrillation

‫اﻹﺟﺎﺑﺔ ﻋﲆ اﻟﺼﻔﺤﺔ اﻟﺘﺎﻟﻴﺔ‬

Page - 185
Internal Medicine - Cardiology - Dysrhythmias

Question 87/155

Question #87

A 65-year-old male patient was evaluated at the emergency room for dizziness, and his ECG is
shown below. What is the most likely diagnosis?

a. Heart block 1st-degree


b. Heart block 2nd-degree type 1 √
c. Heart block 2nd-degree type 2
d. Complete heart block
e. Atrial fibrillation

Description

is ECG shows regular irregularity and a dropped beat. Progressive prolongation of PR followed by a
dropped beat is typical for first-degree AV block Mobitz type 1.

Heart block:

Also known as AV block (atrioventricular block)


Decreased or absent conduction through AV node
It could be first, second, or third-degree
Maybe due to RCA-mediated ischemia
First-degree AV block: long PR interval > 200ms, asymptomatic, need no treatment
Page - 186
Second-degree AV block (Mobitz type I): progressive prolongation of the PR interval, followed
by failure of conduction of P wave, no need for a pacemaker.
Second degree AV block (Mobitz type II): prolongation of the PR interval followed by failure of
conduction of P wave, need Pacemaker
ird-degree AV block (complete AV block): P and QRS waves are not related to each other;
regular P – P intervals and R – R intervals usually come with bradycardia and need Pacemaker

Page - 187
Internal Medicine - Cardiology

Question 88/155

Question #88

A 42-year-old female patient presents with palpitations and dizziness for 2 hours duration. At ER, her
blood pressure is 74/66, her pulse is 162 bpm, and her ECG is shown below. What is the next step in
the management of this patient?

a. Digoxin PO
b. Digoxin IV
c. Synchronized Cardioversion
d. Adenosine
e. Verapamil

‫اﻹﺟﺎﺑﺔ ﻋﲆ اﻟﺼﻔﺤﺔ اﻟﺘﺎﻟﻴﺔ‬

Page - 188
Internal Medicine - Cardiology - Dysrhythmias

Question 88/155

Question #88

A 42-year-old female patient presents with palpitations and dizziness for 2 hours duration. At ER, her
blood pressure is 74/66, her pulse is 162 bpm, and her ECG is shown below. What is the next step in
the management of this patient?

a. Digoxin PO
b. Digoxin IV
c. Synchronized Cardioversion √
d. Adenosine
e. Verapamil

Description

Any unstable patient with SVT should be immediately treated with synchronized cardioversion. e
patient’s blood pressure indicates instability here.

Vagal maneuvers, adenosine, and verapamil are used in stable patients.

Supraventricular Tachycardia (SVT)

SVT: regular, no P wave, narrow QRS, and heart rate > 150 bpm
Causes: (Hypokalemia, Hyperthyroidism, Excessive alcohol)

Treatment of SVT:

For unstable patients: synchronized cardioversion.


For stable patients:
Step 1: Vagal maneuvers: (carotid massage, ocular massage, Ice on the forehead,
Valsalva maneuver)
Page - 189
Step 2: Adenosine (6, 12, then 12 mg IV push)
Step 3: I.V verapamil, Diltiazem, or metoprolol.
Step 4: synchronized cardioversion (if unresponsive)
Definitive treatment: catheter ablation.

Page - 190
Internal Medicine - Cardiology

Question 89/155

Question #89

Regarding the risk of stroke in patients with atrial fibrillation. In comparison with patients with
permanent AF, patients with paroxysmal AF have a risk of stroke that is:

a. 25% less
b. 25% more
c. 50% less
d. 50% more
e. Similar risk of stroke for both groups

‫اﻹﺟﺎﺑﺔ ﻋﲆ اﻟﺼﻔﺤﺔ اﻟﺘﺎﻟﻴﺔ‬

Page - 191
Internal Medicine - Cardiology - Dysrhythmias

Question 89/155

Question #89

Regarding the risk of stroke in patients with atrial fibrillation. In comparison with patients with
permanent AF, patients with paroxysmal AF have a risk of stroke that is:

a. 25% less
b. 25% more
c. 50% less
d. 50% more
e. Similar risk of stroke for both groups √

Description

Regardless of the type or frequency of atrial fibrillation paroxysms, the risk of stroke is the same. e
risk of stroke is determined by applying the CHA2DS2VAS score

Page - 192
Internal Medicine - Cardiology

Question 90/155

Question #90

A patient presents to the ER with chest pain. While you take a history, he suddenly becomes
unresponsive. His ECG is shown below. What is the treatment of choice for this condition?

a. Amiodaron infusion
b. DC cardioversion
c. IV dextrose
d. IV naloxone
e. IV verapamil

‫اﻹﺟﺎﺑﺔ ﻋﲆ اﻟﺼﻔﺤﺔ اﻟﺘﺎﻟﻴﺔ‬

Page - 193
Internal Medicine - Cardiology - Dysrhythmias

Question 90/155

Question #90

A patient presents to the ER with chest pain. While you take a history, he suddenly becomes
unresponsive. His ECG is shown below. What is the treatment of choice for this condition?

a. Amiodaron infusion
b. DC cardioversion √
c. IV dextrose
d. IV naloxone
e. IV verapamil

Description

is patient has ventricular tachycardia, and he is unstable. is condition should be immediately
treated with DC cardioversion.

Ventricular tachycardia (VT):

VT: Regular, wide QRS (> 120 millisecond), no P wave, no T wave


Rate > 100 (usually 140-200)
Associated with (AV dissociation + capture beats)
It can be monomorphic or polymorphic

Treatment of VT:

For unstable patients: DC cardioversion


For stable patients: amiodarone, Lidocaine, procainamide

Page - 194
Internal Medicine - Cardiology

Question 91/155

Question #91

A 76-year-old male with a case of HTN and DM presents with atrial fibrillation. His blood pressure is
130.80, his pulse is 67 bpm, and the patient is conscious, oriented, and alert with no chest pain or
shortness of breath. What is the target level of INR for his condition?

a. 1.5 – 2
b. 2 – 3
c. 2.5 – 3.5
d. 3 – 4
e. No need for anticoagulation

‫اﻹﺟﺎﺑﺔ ﻋﲆ اﻟﺼﻔﺤﺔ اﻟﺘﺎﻟﻴﺔ‬

Page - 195
Internal Medicine - Cardiology - Dysrhythmias

Question 91/155

Question #91

A 76-year-old male with a case of HTN and DM presents with atrial fibrillation. His blood pressure is
130.80, his pulse is 67 bpm, and the patient is conscious, oriented, and alert with no chest pain or
shortness of breath. What is the target level of INR for his condition?

a. 1.5 – 2
b. 2 – 3 √
c. 2.5 – 3.5
d. 3 – 4
e. No need for anticoagulation

Description

According to the CHA2DS2VAS score, this patient has a score of 4 and needs anticoagulation

e target INR level for patients with atrial fibrillation is 2 – 3

Studies demonstrate that low-intensity warfarin (target 2 – 3) is equivalent to higher-intensity


anticoagulation (INR > 3) and is associated with a lower risk of major bleeding events

Page - 196
Internal Medicine - Cardiology

Question 92/155

Question #92

A patient is diagnosed with atrial fibrillation. Which is better for controlling heart rate at rest and
exercise?

a. Digoxin
b. Beta-blockers
c. Calcium channel blockers
d. Amiodaron
e. Flecainide

‫اﻹﺟﺎﺑﺔ ﻋﲆ اﻟﺼﻔﺤﺔ اﻟﺘﺎﻟﻴﺔ‬

Page - 197
Internal Medicine - Cardiology - Dysrhythmias

Question 92/155

Question #92

A patient is diagnosed with atrial fibrillation. Which is better for controlling heart rate at rest and
exercise?

a. Digoxin
b. Beta-blockers √
c. Calcium channel blockers
d. Amiodaron
e. Flecainide

Description

Beta-blockers are the best to control heart rate in both rest and exercise

Calcium channel blockers are used for the same purpose as well, but they are less eective than
beta-blockers

Digoxin primarily controls heart rate at rest only

Amiodaron and flecainide are used for rhythm control, not rate control

Page - 198
Internal Medicine - Cardiology

Question 93/155

Question #93

An asthmatic 42-year-old female patient presents with palpitations and dizziness for 2 hours; her
blood pressure is 116/66, her pulse is 162 bpm, and her ECG is shown below. What is the most likely
diagnosis?

a. Atrial fibrillation
b. Atrial flutter
c. Supraventricular tachycardia
d. Multifocal atrial tachycardia
e. Sinus tachycardia

‫اﻹﺟﺎﺑﺔ ﻋﲆ اﻟﺼﻔﺤﺔ اﻟﺘﺎﻟﻴﺔ‬

Page - 199
Internal Medicine - Cardiology - Dysrhythmias

Question 93/155

Question #93

An asthmatic 42-year-old female patient presents with palpitations and dizziness for 2 hours; her
blood pressure is 116/66, her pulse is 162 bpm, and her ECG is shown below. What is the most likely
diagnosis?

a. Atrial fibrillation
b. Atrial flutter
c. Supraventricular tachycardia √
d. Multifocal atrial tachycardia
e. Sinus tachycardia

Description

SVT (supraventricular tachycardia) on ECG demonstrated as an absent P wave, narrow QRS complex,
high heart rate, and regular rhythm.

Supraventricular Tachycardia (SVT)

SVT: regular, no P wave, narrow QRS, and heart rate > 150 bpm
Causes: (Hypokalemia, Hyperthyroidism, Excessive alcohol)

Treatment of SVT:

For unstable patients: synchronized cardioversion.


For stable patients:
Step 1: Vagal maneuvers: (carotid massage, ocular massage, Ice on the forehead,
Valsalva maneuver)
Step 2: Adenosine (6, 12, then 12 mg IV push)
Step 3: I.V verapamil, Diltiazem, or metoprolol.
Page - 200
Step 4: synchronized cardioversion (if unresponsive)
Definitive treatment: catheter ablation.

Page - 201
Internal Medicine - Cardiology

Question 94/155

Question #94

A 34-year-old male patient presents with intermittent palpitations. His ECG is shown below. What is
the most likely diagnosis?

a. Normal ECG
b. Le ventricular hypertrophy
c. WPW syndrome
d. LBBB
e. RBBB

‫اﻹﺟﺎﺑﺔ ﻋﲆ اﻟﺼﻔﺤﺔ اﻟﺘﺎﻟﻴﺔ‬

Page - 202
Internal Medicine - Cardiology - Dysrhythmias

Question 94/155

Question #94

A 34-year-old male patient presents with intermittent palpitations. His ECG is shown below. What is
the most likely diagnosis?

a. Normal ECG
b. Le ventricular hypertrophy
c. WPW syndrome √
d. LBBB
e. RBBB

Description

is ECG shows Delta wave and short PR interval, which is typical for WPW syndrome

It is a congenital defect characterized by the presence of an accessory bundle between the


atrium and ventricle,
is bundle is called the “bundle of Kent.”
Type A (Le Side) or type B (Right Side)

Short PR interval and early ventricular depolarization (Delta wave)

Page - 203
Internal Medicine - Cardiology

Question 95/155

Question #95

A 42-year-old female patient presents with palpitations and dizziness for 2 hours duration. Her
blood pressure is 116/66, her pulse is 162 bpm, and her ECG is below. e carotid massage was tried,
and adenosine 6, 12, then 12 mg VI push was given, but no improvement. What is the next step in the
treatment of this patient?

a. Repeat 12 mg adenosine IV push


b. Give 24 mg adenosine IV push
c. Start IV verapamil
d. Synchronized cardioversion
e. Amiodarone infusion

‫اﻹﺟﺎﺑﺔ ﻋﲆ اﻟﺼﻔﺤﺔ اﻟﺘﺎﻟﻴﺔ‬

Page - 204
Internal Medicine - Cardiology - Dysrhythmias

Question 95/155

Question #95

A 42-year-old female patient presents with palpitations and dizziness for 2 hours duration. Her
blood pressure is 116/66, her pulse is 162 bpm, and her ECG is below. e carotid massage was tried,
and adenosine 6, 12, then 12 mg VI push was given, but no improvement. What is the next step in the
treatment of this patient?

a. Repeat 12 mg adenosine IV push


b. Give 24 mg adenosine IV push
c. Start IV verapamil √
d. Synchronized cardioversion
e. Amiodarone infusion

Description

is patient has SVT but no improvement on vagal maneuvers and adenosine. erefore, verapamil
is the next step in the management of this patient.

e recommended dose of adenosine is 6mg, 12mg, then 12mg IV push

Only 3 doses are used. If the patient improves at any time, further doses should not be needed.

Supraventricular Tachycardia (SVT)

SVT: regular, no P wave, narrow QRS, and heart rate > 150 bpm
Causes: (Hypokalemia, Hyperthyroidism, Excessive alcohol)

Treatment of SVT:

For unstable patients: synchronized cardioversion.


For stable patients:
Page - 205
Step 1: Vagal maneuvers: (carotid massage, ocular massage, Ice on the forehead,
Valsalva maneuver)
Step 2: Adenosine (6, 12, then 12 mg IV push)
Step 3: I.V verapamil, Diltiazem, or metoprolol.
Step 4: synchronized cardioversion (if unresponsive)
Definitive treatment: catheter ablation.

Page - 206
Internal Medicine - Cardiology

Question 96/155

Question #96

A 33-year-old male patient has had chest discomfort and palpitation for 30 minutes; his ECG is
shown below. What is the most likely diagnosis?

a. Atrial fibrillation
b. Ventricular tachycardia
c. Sinus tachycardia
d. Atrial flutter
e. Sick sinus syndrome

‫اﻹﺟﺎﺑﺔ ﻋﲆ اﻟﺼﻔﺤﺔ اﻟﺘﺎﻟﻴﺔ‬

Page - 207
Internal Medicine - Cardiology - Dysrhythmias

Question 96/155

Question #96

A 33-year-old male patient has had chest discomfort and palpitation for 30 minutes; his ECG is
shown below. What is the most likely diagnosis?

a. Atrial fibrillation √
b. Ventricular tachycardia
c. Sinus tachycardia
d. Atrial flutter
e. Sick sinus syndrome

Description

is ECG shows irregular irregularity in the rhythm. ere is no P wave.

Fibrillatory waves (f waves) are present, and the QRS complexes are narrow.

is represents a case of atrial fibrillation.

Page - 208
Internal Medicine - Cardiology

Question 97/155

Question #97

A patient was admitted for major surgery. Which of the following is considered a risk factor for
postoperative ventricular arrhythmia?

a. Age of more than 60 years


b. History of hypothyroidism
c. History of SVT
d. Presence of congestive heart failure
e. Presence of PACs

‫اﻹﺟﺎﺑﺔ ﻋﲆ اﻟﺼﻔﺤﺔ اﻟﺘﺎﻟﻴﺔ‬

Page - 209
Internal Medicine - Cardiology - Dysrhythmias

Question 97/155

Question #97

A patient was admitted for major surgery. Which of the following is considered a risk factor for
postoperative ventricular arrhythmia?

a. Age of more than 60 years


b. History of hypothyroidism
c. History of SVT
d. Presence of congestive heart failure √
e. Presence of PACs

Description

Intraoperative and postoperative ventricular arrhythmias are expected in patients with the
following risk factors:

Preoperative ventricular ectopic beats


History of CHF
History of smoking

Page - 210
Internal Medicine - Cardiology

Question 98/155

Question #98

An asthmatic 42-year-old female patient presents with palpitations and dizziness for 2 hours; her
blood pressure is 116/66, her pulse is 162 bpm, and her ECG is shown below. What is the treatment of
choice for her condition?

a. Digoxin PO
b. Digoxin IV
c. Synchronized Cardioversion
d. Adenosine
e. Verapamil

‫اﻹﺟﺎﺑﺔ ﻋﲆ اﻟﺼﻔﺤﺔ اﻟﺘﺎﻟﻴﺔ‬

Page - 211
Internal Medicine - Cardiology - Dysrhythmias

Question 98/155

Question #98

An asthmatic 42-year-old female patient presents with palpitations and dizziness for 2 hours; her
blood pressure is 116/66, her pulse is 162 bpm, and her ECG is shown below. What is the treatment of
choice for her condition?

a. Digoxin PO
b. Digoxin IV
c. Synchronized Cardioversion
d. Adenosine
e. Verapamil √

Description

is patient has SVT; the first treatment of SVT in stable patients is adenosine, but this patient is
asthmatic, and adenosine is contraindicated with asthma.

Verapamil is the treatment of choice for stable patients with SVT and asthma.

Note that digoxin is used to treat AF and heart failure

Synchronized cardioversion is indicated if the patient is unstable.

Supraventricular Tachycardia (SVT)

SVT: regular, no P wave, narrow QRS, and heart rate > 150 bpm
Causes: (Hypokalemia, Hyperthyroidism, Excessive alcohol)

Treatment of SVT:

For unstable patients: synchronized cardioversion.


Page - 212
For stable patients:
Step 1: Vagal maneuvers: (carotid massage, ocular massage, Ice on the forehead,
Valsalva maneuver)
Step 2: Adenosine (6, 12, then 12 mg IV push)
Step 3: I.V verapamil, Diltiazem, or metoprolol.
Step 4: synchronized cardioversion (if unresponsive)
Definitive treatment: catheter ablation.

Page - 213
Internal Medicine - Cardiology

Question 99/155

Question #99

A 32-year-old female patient has had palpitations for 2 hours, and her ECG is shown below. What is
the most likely diagnosis?

a. Atrial fibrillation with rapid ventricular response


b. Supraventricular tachycardia
c. Sinus tachycardia
d. Ventricular tachycardia
e. Normal ECG

‫اﻹﺟﺎﺑﺔ ﻋﲆ اﻟﺼﻔﺤﺔ اﻟﺘﺎﻟﻴﺔ‬

Page - 214
Internal Medicine - Cardiology - Dysrhythmias

Question 99/155

Question #99

A 32-year-old female patient has had palpitations for 2 hours, and her ECG is shown below. What is
the most likely diagnosis?

a. Atrial fibrillation with rapid ventricular response


b. Supraventricular tachycardia √
c. Sinus tachycardia
d. Ventricular tachycardia
e. Normal ECG

Description

Regular, no P wave, narrow QRS, and heart rate > 150 bpm. ese features on ECG indicate SVT.

Supraventricular Tachycardia (SVT)

SVT: regular, no P wave, narrow QRS, and heart rate > 150 bpm
Causes: (Hypokalemia, Hyperthyroidism, Excessive alcohol)

Treatment of SVT:

For unstable patients: synchronized cardioversion.


For stable patients:
Page - 215
Step 1: Vagal maneuvers: (carotid massage, ocular massage, Ice on the forehead,
Valsalva maneuver)
Step 2: Adenosine (6, 12, then 12 mg IV push)
Step 3: I.V verapamil, Diltiazem, or metoprolol.
Step 4: synchronized cardioversion (if unresponsive)
Definitive treatment: catheter ablation.

Page - 216
Internal Medicine - Cardiology

Question 100/155

Question #100

Intravenous magnesium sulfate is used in treating which of the following arrhythmias?

a. Second-degree AV block
b. 3rd degree AV block
c. Junctional tachycardia
d. Torsade de points
e. Ventricular fibrillation

‫اﻹﺟﺎﺑﺔ ﻋﲆ اﻟﺼﻔﺤﺔ اﻟﺘﺎﻟﻴﺔ‬

Page - 217
Internal Medicine - Cardiology - Dysrhythmias

Question 100/155

Question #100

Intravenous magnesium sulfate is used in treating which of the following arrhythmias?

a. Second-degree AV block
b. 3rd degree AV block
c. Junctional tachycardia
d. Torsade de points √
e. Ventricular fibrillation

Description

Torsade de points or twisting points usually result from long QT syndrome and is treated with
intravenous magnesium sulfate

Torsade de points (TdP):

It is also known as (twisting points)


It is associated with Long QT syndrome
May result in sudden death

Treatment of TdP:

For unstable patients: cardioversion


For stable patients: IV magnesium sulfate
Always correct the underlying cause.

Page - 218
Internal Medicine - Cardiology

Question 101/155

Question #101

e following ECG was recorded on a 45-year-old male patient. He has no symptoms at all. What is
the most likely diagnosis?

a. Atrial fibrillation
b. Normal sinus rhythm
c. Heart block 1st degree
d. Complete Heart block
e. Junctional rhythm

‫اﻹﺟﺎﺑﺔ ﻋﲆ اﻟﺼﻔﺤﺔ اﻟﺘﺎﻟﻴﺔ‬

Page - 219
Internal Medicine - Cardiology - Dysrhythmias

Question 101/155

Question #101

e following ECG was recorded on a 45-year-old male patient. He has no symptoms at all. What is
the most likely diagnosis?

a. Atrial fibrillation
b. Normal sinus rhythm
c. Heart block 1st degree √
d. Complete Heart block
e. Junctional rhythm

Description

is ECG shows regular beats at a rate of 60 per minute

e axis deviation is normal, and the QRS complexes are narrow

e most important here is to note that the PR interval is more than 200 ms (1 large square). is is
typical for 1st-degree AV block.

Heart block:

Also known as AV block (atrioventricular block)


Decreased or absent conduction through AV node
It could be first, second, or third-degree
Maybe due to RCA-mediated ischemia
First-degree AV block: long PR interval > 200ms, asymptomatic, need no treatment
Second-degree AV block (Mobitz type I): progressive prolongation of the PR interval, followed
by failure of conduction of P wave, no need for a pacemaker.
Second degree AV block (Mobitz type II): prolongation of the PR interval followed by failurePage
of - 220
conduction of P wave, need Pacemaker
ird-degree AV block (complete AV block): P and QRS waves are not related to each other;
regular P – P intervals and R – R intervals usually come with bradycardia and need Pacemaker

Page - 221
Internal Medicine - Cardiology

Question 102/155

Question #102

e following figure is an ECG of a 32-year-old male with hypokalemia. What is the most likely
diagnosis?

a. Ventricular fibrillation
b. Atrial fibrillation
c. Extrasystoles
d. Torsade de points
e. Monomorphic ventricular tachycardia

‫اﻹﺟﺎﺑﺔ ﻋﲆ اﻟﺼﻔﺤﺔ اﻟﺘﺎﻟﻴﺔ‬

Page - 222
Internal Medicine - Cardiology - Dysrhythmias

Question 102/155

Question #102

e following figure is an ECG of a 32-year-old male with hypokalemia. What is the most likely
diagnosis?

a. Ventricular fibrillation
b. Atrial fibrillation
c. Extrasystoles
d. Torsade de points √
e. Monomorphic ventricular tachycardia

Description

Long QT (mostly here due to hypokalemia) and twisted QRS complexes are typical of torsade de
points. is patient should receive magnesium sulfate intravenously, and the hypokalemia should
be corrected.

Torsade de points (TdP):

It is also known as (twisting points)


Page - 223
It is associated with Long QT syndrome
May result in sudden death

Treatment of TdP:

For unstable patients: cardioversion


For stable patients: IV magnesium sulfate
Always correct the underlying cause.

Page - 224
Internal Medicine - Cardiology

Question 103/155

Question #103

A 17-year-old male patient presents with recurrent palpitation. He is found to have a long QT on ECG.
All the following can be a cause except:

a. Hyperkalemia
b. Hypomagnesemia
c. Hypocalcemia
d. Romano ward syndrome
e. Erythromycin use

‫اﻹﺟﺎﺑﺔ ﻋﲆ اﻟﺼﻔﺤﺔ اﻟﺘﺎﻟﻴﺔ‬

Page - 225
Internal Medicine - Cardiology - Dysrhythmias

Question 103/155

Question #103

A 17-year-old male patient presents with recurrent palpitation. He is found to have a long QT on ECG.
All the following can be a cause except:

a. Hyperkalemia √
b. Hypomagnesemia
c. Hypocalcemia
d. Romano ward syndrome
e. Erythromycin use

Description

e following table shows the possible causes of torsade de points and long QT syndrome.

Torsade de points (TdP):

It is also known as (twisting points)


It is associated with Long QT syndrome
May result in sudden death

Treatment of TdP:

For unstable patients: cardioversion


For stable patients: IV magnesium sulfate
Always correct the underlying cause.

Page - 226
Internal Medicine - Cardiology

Question 104/155

Question #104

A 33-year-old male patient has had chest discomfort and palpitation for 30 minutes; his ECG is
shown below. What is the most likely diagnosis?

a. Atrial fibrillation
b. Ventricular tachycardia
c. Sinus tachycardia
d. Atrial flutter
e. Sick sinus syndrome

‫اﻹﺟﺎﺑﺔ ﻋﲆ اﻟﺼﻔﺤﺔ اﻟﺘﺎﻟﻴﺔ‬

Page - 227
Internal Medicine - Cardiology - Dysrhythmias

Question 104/155

Question #104

A 33-year-old male patient has had chest discomfort and palpitation for 30 minutes; his ECG is
shown below. What is the most likely diagnosis?

a. Atrial fibrillation
b. Ventricular tachycardia
c. Sinus tachycardia
d. Atrial flutter √
e. Sick sinus syndrome

Description

is ECG shows a saw-tooth appearance of the P wave; this represents a case of atrial flutter

Atrial flutter:

Large re-entry circuit in Right Atrium


Atrial rate: 300 BPM
Ventricular rate: 300 bpm (1:1), 150 bpm (2:1), or 100 bpm (3:1)
Rhythm: regular or maybe irregular
P waves: saw-tooth appearance, best seen in leads II, III or AVF
QRS: normal (less than 120 ms)

Page - 228
Internal Medicine - Cardiology

Question 105/155

Question #105

e definitive treatment of WPW syndrome is ablation in which of the following sites?

a. e pulmonary vein bifurcation


b. e right ventricle
c. e AV node
d. e bundle of HIS
e. e bundle of Kent

‫اﻹﺟﺎﺑﺔ ﻋﲆ اﻟﺼﻔﺤﺔ اﻟﺘﺎﻟﻴﺔ‬

Page - 229
Internal Medicine - Cardiology - Dysrhythmias

Question 105/155

Question #105

e definitive treatment of WPW syndrome is ablation in which of the following sites?

a. e pulmonary vein bifurcation


b. e right ventricle
c. e AV node
d. e bundle of HIS
e. e bundle of Kent √

Description

e bundle of Kent is an accessory bundle bypassing the AV node in case of WPW syndrome

Page - 230
Internal Medicine - Cardiology

Question 106/155

Question #106

A 42-year-old male patient was diagnosed with SVT, and now the heart rate is controlled on
verapamil. He presents to you for follow-up. What is the definitive treatment of this patient?

a. Adenosine
b. Verapamil
c. Cardioversion
d. Cardiac ablation
e. Amiodarone

‫اﻹﺟﺎﺑﺔ ﻋﲆ اﻟﺼﻔﺤﺔ اﻟﺘﺎﻟﻴﺔ‬

Page - 231
Internal Medicine - Cardiology - Dysrhythmias

Question 106/155

Question #106

A 42-year-old male patient was diagnosed with SVT, and now the heart rate is controlled on
verapamil. He presents to you for follow-up. What is the definitive treatment of this patient?

a. Adenosine
b. Verapamil
c. Cardioversion
d. Cardiac ablation √
e. Amiodarone

Description

Cardiac ablation is considered the definitive treatment of SVT.

An electrophysiological study of the heart is done, and ablation of the abnormal focus is done by a
catheter aer it is determined.

Supraventricular Tachycardia (SVT)

SVT: regular, no P wave, narrow QRS, and heart rate > 150 bpm
Causes: (Hypokalemia, Hyperthyroidism, Excessive alcohol)

Treatment of SVT:

For unstable patients: synchronized cardioversion.


For stable patients:
Step 1: Vagal maneuvers: (carotid massage, ocular massage, Ice on the forehead,
Valsalva maneuver)
Step 2: Adenosine (6, 12, then 12 mg IV push)
Step 3: I.V verapamil, Diltiazem, or metoprolol.
Step 4: synchronized cardioversion (if unresponsive)
Definitive treatment: catheter ablation.

Page - 232
Internal Medicine - Cardiology

Question 107/155

Question #107

Which of the following options carries the most significant risk of stroke in a patient with atrial
fibrillation?

a. Age of 74-year-old
b. History of stroke
c. Female sex
d. Heart failure
e. Diabetes mellitus

‫اﻹﺟﺎﺑﺔ ﻋﲆ اﻟﺼﻔﺤﺔ اﻟﺘﺎﻟﻴﺔ‬

Page - 233
Internal Medicine - Cardiology - Dysrhythmias

Question 107/155

Question #107

Which of the following options carries the most significant risk of stroke in a patient with atrial
fibrillation?

a. Age of 74-year-old
b. History of stroke √
c. Female sex
d. Heart failure
e. Diabetes mellitus

Description

According to the CHA2DS2VAS score system, the age of more than 75 and the presence of Stroke or
TIA has the highest risk (score 2 points).

e rest of mentioned choices has a score of 1 point on the CHA2DS2VAS score, so we consider them
of lower risk for stroke

Page - 234
Internal Medicine - Cardiology

Question 108/155

Question #108

A 32-year-old male patient was diagnosed with WPW syndrome. e episodic symptomatic
supraventricular tachycardia or atrial fibrillation benefits most from which of the following?

a. Beta-blockers
b. Digoxin
c. Amiodarone
d. Flecainide
e. Radiofrequency catheter ablation of bypass tracts

‫اﻹﺟﺎﺑﺔ ﻋﲆ اﻟﺼﻔﺤﺔ اﻟﺘﺎﻟﻴﺔ‬

Page - 235
Internal Medicine - Cardiology - Dysrhythmias

Question 108/155

Question #108

A 32-year-old male patient was diagnosed with WPW syndrome. e episodic symptomatic
supraventricular tachycardia or atrial fibrillation benefits most from which of the following?

a. Beta-blockers
b. Digoxin
c. Amiodarone
d. Flecainide
e. Radiofrequency catheter ablation of bypass tracts √

Description

Radiofrequency catheter ablation of the Kent bundle is safer and more cost eective than surgery,
with a similar success rate.

Beta-blockers and digoxin are contraindicated in WPW syndrome as they can shorten the refractory
period of the accessory pathway and increase the ventricular rate.

Page - 236
Internal Medicine - Cardiology

Question 109/155

Question #109

An asthmatic 23-year-old female patient presents to you with palpitations, and her ECG
demonstrates regular narrow complex tachycardia with an absent P wave. Her blood pressure is
130/80, her pulse is 180 bpm, and she is conscious. However, she returns to her sinus rhythm aer
carotid massage and vagal maneuvers. What is the definitive treatment for her condition?

a. Adenosine 6, 12, 12 mg intravenously


b. Verapamil administration
c. Catheter ablation
d. Digoxin administration
e. Levothyroxine therapy

‫اﻹﺟﺎﺑﺔ ﻋﲆ اﻟﺼﻔﺤﺔ اﻟﺘﺎﻟﻴﺔ‬

Page - 237
Internal Medicine - Cardiology - Dysrhythmias

Question 109/155

Question #109

An asthmatic 23-year-old female patient presents to you with palpitations, and her ECG
demonstrates regular narrow complex tachycardia with an absent P wave. Her blood pressure is
130/80, her pulse is 180 bpm, and she is conscious. However, she returns to her sinus rhythm aer
carotid massage and vagal maneuvers. What is the definitive treatment for her condition?

a. Adenosine 6, 12, 12 mg intravenously


b. Verapamil administration
c. Catheter ablation √
d. Digoxin administration
e. Levothyroxine therapy

Description

is is a case of supraventricular tachycardia, which was improved aer noninvasive treatment.

Note that the question is about the definitive treatment of SVT, so that the answer will be catheter
ablation.

Adenosine is contraindicated in asthmatic patients, and it is not a definitive treatment but rather a
symptomatic reliever of the SVT attack

Verapamil is useful for her condition, but again it is not a definitive treatment but can control
symptoms and reduce the recurrence of the attacks.

Page - 238
Internal Medicine - Cardiology

Question 110/155

Question #110

A 69-year-old patient is found to have premature ventricular contractions on routine examination.


What is the treatment for this condition according to the recent guidelines?

a. Amiodarone oral tablet


b. Flecainide oral tablet
c. Verapamil oral tablet
d. Metoprolol oral tablet
e. Reassurance and no drug therapy

‫اﻹﺟﺎﺑﺔ ﻋﲆ اﻟﺼﻔﺤﺔ اﻟﺘﺎﻟﻴﺔ‬

Page - 239
Internal Medicine - Cardiology - Dysrhythmias

Question 110/155

Question #110

A 69-year-old patient is found to have premature ventricular contractions on routine examination.


What is the treatment for this condition according to the recent guidelines?

a. Amiodarone oral tablet


b. Flecainide oral tablet
c. Verapamil oral tablet
d. Metoprolol oral tablet
e. Reassurance and no drug therapy √

Description

e use of beta-blockers in patients with PVCs is reserved for symptomatic cases only;
asymptomatic patients with PVCs are not indicated to receive any drug therapy according to the
recent guidelines.

Page - 240
Internal Medicine - Cardiology

Question 111/155

Question #111

A 32-year-old patient presented with palpitation and was diagnosed with SVT, which is now
controlled by adenosine. e ECG is demonstrated below. Which of the following is the most likely
diagnosis?

a. Ventricular fibrillation
b. Premature ventricular contractions
c. Heart block
d. Sinus bradycardia
e. WPW syndrome

‫اﻹﺟﺎﺑﺔ ﻋﲆ اﻟﺼﻔﺤﺔ اﻟﺘﺎﻟﻴﺔ‬

Page - 241
Internal Medicine - Cardiology - Dysrhythmias

Question 111/155

Question #111

A 32-year-old patient presented with palpitation and was diagnosed with SVT, which is now
controlled by adenosine. e ECG is demonstrated below. Which of the following is the most likely
diagnosis?

a. Ventricular fibrillation
b. Premature ventricular contractions
c. Heart block
d. Sinus bradycardia
e. WPW syndrome √

Description

e presence of delta wave and short PR interval indicate WPW syndrome

is patient will primarily benefit from the ablation of the Kent bundle

Page - 242
Internal Medicine - Cardiology

Question 112/155

Question #112

What does this ECG show?

a. Torsade de points
b. Ventricular tachycardia
c. Ventricular fibrillation
d. Atrial fibrillation
e. Atrial flutter

‫اﻹﺟﺎﺑﺔ ﻋﲆ اﻟﺼﻔﺤﺔ اﻟﺘﺎﻟﻴﺔ‬

Page - 243
Internal Medicine - Cardiology - Dysrhythmias

Question 112/155

Question #112

What does this ECG show?

a. Torsade de points
b. Ventricular tachycardia √
c. Ventricular fibrillation
d. Atrial fibrillation
e. Atrial flutter

Description

Ventricular Tachycardia (VT): Regular, wide QRS (> 120 millisecond), no P wave, no T wave, and the
rate is > 100 (usually 140 – 200 bpm).

Ventricular tachycardia (VT):

VT: Regular, wide QRS (> 120 millisecond), no P wave, no T wave


Rate > 100 (usually 140-200)
Associated with (AV dissociation + capture beats)
It can be monomorphic or polymorphic
Page - 244
Treatment of VT:

For unstable patients: DC cardioversion


For stable patients: amiodarone, Lidocaine, procainamide

Page - 245
Internal Medicine - Cardiology

Question 113/155

Question #113

A 65-year-old male patient was evaluated at the emergency room for dizziness, and his ECG is
shown below. What is the most likely diagnosis?

a. Heart block 1st-degree


b. Heart block 2nd-degree type 1
c. Heart block 2nd-degree type 2
d. Complete heart block
e. Normal sinus rhythm

‫اﻹﺟﺎﺑﺔ ﻋﲆ اﻟﺼﻔﺤﺔ اﻟﺘﺎﻟﻴﺔ‬

Page - 246
Internal Medicine - Cardiology - Dysrhythmias

Question 113/155

Question #113

A 65-year-old male patient was evaluated at the emergency room for dizziness, and his ECG is
shown below. What is the most likely diagnosis?

a. Heart block 1st-degree


b. Heart block 2nd-degree type 1
c. Heart block 2nd-degree type 2
d. Complete heart block √
e. Normal sinus rhythm

Description

Bradycardia, AV dissociation, and wide QRS complex are present in this ECG. Note that AV
dissociation is typical for a complete AV block.

Heart block:

Also known as AV block (atrioventricular block)


Decreased or absent conduction through AV node
It could be first, second, or third-degree
Maybe due to RCA-mediated ischemia
First-degree AV block: long PR interval > 200ms, asymptomatic, need no treatment
Second-degree AV block (Mobitz type I): progressive prolongation of the PR interval, followed
Page - 247
by failure of conduction of P wave, no need for a pacemaker.
Second degree AV block (Mobitz type II): prolongation of the PR interval followed by failure of
conduction of P wave, need Pacemaker
ird-degree AV block (complete AV block): P and QRS waves are not related to each other;
regular P – P intervals and R – R intervals usually come with bradycardia and need Pacemaker

Page - 248
Internal Medicine - Cardiology

Question 114/155

Question #114

A 42-year-old female patient presents with palpitations and dizziness for 2 hours duration. Her
blood pressure is 116/66, her pulse is 162 bpm, and her ECG is below. What is the first-line treatment
for her condition?

a. Vagal maneuvers
b. Adenosine
c. Verapamil
d. Synchronized cardioversion
e. Amiodarone

‫اﻹﺟﺎﺑﺔ ﻋﲆ اﻟﺼﻔﺤﺔ اﻟﺘﺎﻟﻴﺔ‬

Page - 249
Internal Medicine - Cardiology - Dysrhythmias

Question 114/155

Question #114

A 42-year-old female patient presents with palpitations and dizziness for 2 hours duration. Her
blood pressure is 116/66, her pulse is 162 bpm, and her ECG is below. What is the first-line treatment
for her condition?

a. Vagal maneuvers √
b. Adenosine
c. Verapamil
d. Synchronized cardioversion
e. Amiodarone

Description

In stable patients with SVT, vagal maneuvers are the first-line treatment.

If not improved, start adenosine unless contraindicated.

Use verapamil in stable patients who don’t improve on adenosine or when adenosine is
contraindicated.

Supraventricular Tachycardia (SVT)

SVT: regular, no P wave, narrow QRS, and heart rate > 150 bpm
Causes: (Hypokalemia, Hyperthyroidism, Excessive alcohol)

Treatment of SVT:

For unstable patients: synchronized cardioversion.


For stable patients:
Step 1: Vagal maneuvers: (carotid massage, ocular massage, Ice on the forehead,
Page - 250
Valsalva maneuver)
Step 2: Adenosine (6, 12, then 12 mg IV push)
Step 3: I.V verapamil, Diltiazem, or metoprolol.
Step 4: synchronized cardioversion (if unresponsive)
Definitive treatment: catheter ablation.

Page - 251
Internal Medicine - Cardiology

Question 115/155

Question #115

A patient presents with palpitation, and his ECG is shown below. Which of the following is the
treatment of choice for his condition?

a. Amiodarone infusion
b. Intravenous Magnesium sulfate
c. Intravenous calcium gluconate
d. Synchronized cardioversion
e. Intravenous Adenosine

‫اﻹﺟﺎﺑﺔ ﻋﲆ اﻟﺼﻔﺤﺔ اﻟﺘﺎﻟﻴﺔ‬

Page - 252
Internal Medicine - Cardiology - Dysrhythmias

Question 115/155

Question #115

A patient presents with palpitation, and his ECG is shown below. Which of the following is the
treatment of choice for his condition?

a. Amiodarone infusion
b. Intravenous Magnesium sulfate √
c. Intravenous calcium gluconate
d. Synchronized cardioversion
e. Intravenous Adenosine

Description

is ECG is showing Torsade de points. e best treatment for this condition is magnesium sulfate
intravenously.

Torsade de points (TdP):

It is also known as (twisting points)


It is associated with Long QT syndrome
May result in sudden death

Treatment of TdP:

For unstable patients: cardioversion


For stable patients: IV magnesium sulfate
Always correct the underlying cause.

Page - 253
Internal Medicine - Cardiology

Question 116/155

Question #116

A 59-year-old male patient with fever and weight loss is diagnosed with infective endocarditis. His
blood culture shows streptococcus bovis twice. He should be evaluated for which of the following
conditions?

a. Colorectal cancer
b. Lung cancer
c. Prostate cancer
d. B-cell lymphoma
e. T-cell lymphoma

‫اﻹﺟﺎﺑﺔ ﻋﲆ اﻟﺼﻔﺤﺔ اﻟﺘﺎﻟﻴﺔ‬

Page - 254
Internal Medicine - Cardiology - Infective Endocarditis (IE)

Question 116/155

Question #116

A 59-year-old male patient with fever and weight loss is diagnosed with infective endocarditis. His
blood culture shows streptococcus bovis twice. He should be evaluated for which of the following
conditions?

a. Colorectal cancer √
b. Lung cancer
c. Prostate cancer
d. B-cell lymphoma
e. T-cell lymphoma

Description

Streptococcus bovis bacteremia or endocarditis is associated with colorectal cancer and upper
gastrointestinal malignancies as well.

In this patient, endoscopy is indicated to rule out colorectal cancer

Page - 255
Internal Medicine - Cardiology

Question 117/155

Question #117

A 66-year-old male patient has a fever and new-onset murmur and is diagnosed with infective
endocarditis. His blood culture is positive for Streptococcus bovis. What is the most appropriate
screening test to do at this point?

a. Chest x-ray
b. Colonoscopy
c. Repeat the blood culture
d. Perform prostate-specific antigen (PSA)
e. Bone marrow aspiration

‫اﻹﺟﺎﺑﺔ ﻋﲆ اﻟﺼﻔﺤﺔ اﻟﺘﺎﻟﻴﺔ‬

Page - 256
Internal Medicine - Cardiology - Infective Endocarditis (IE)

Question 117/155

Question #117

A 66-year-old male patient has a fever and new-onset murmur and is diagnosed with infective
endocarditis. His blood culture is positive for Streptococcus bovis. What is the most appropriate
screening test to do at this point?

a. Chest x-ray
b. Colonoscopy √
c. Repeat the blood culture
d. Perform prostate-specific antigen (PSA)
e. Bone marrow aspiration

Description

Streptococcus bovis bacteremia or endocarditis is associated with colorectal cancer and upper
gastrointestinal malignancies as well.

In this patient, endoscopy is indicated to rule out colorectal cancer

Page - 257
Internal Medicine - Cardiology

Question 118/155

Question #118

According to culture results, A 32-year-old male patient is treated with amoxicillin and low-dose
gentamycin for infective endocarditis. He is still febrile and has shortness of breath. His ECG shows a
Prolonged PR interval. What is the most appropriate at this time?

a. Continue treatment with antibiotics


b. Repeat blood culture and switch to dierent antibiotics
c. Start diuretics and beta-blockers
d. Refer to cardiac surgery
e. Add rifampicin to his antibiotic regimen

‫اﻹﺟﺎﺑﺔ ﻋﲆ اﻟﺼﻔﺤﺔ اﻟﺘﺎﻟﻴﺔ‬

Page - 258
Internal Medicine - Cardiology - Infective Endocarditis (IE)

Question 118/155

Question #118

According to culture results, A 32-year-old male patient is treated with amoxicillin and low-dose
gentamycin for infective endocarditis. He is still febrile and has shortness of breath. His ECG shows a
Prolonged PR interval. What is the most appropriate at this time?

a. Continue treatment with antibiotics


b. Repeat blood culture and switch to dierent antibiotics
c. Start diuretics and beta-blockers
d. Refer to cardiac surgery √
e. Add rifampicin to his antibiotic regimen

Description

is patient has infective endocarditis, and the ECG findings suggest an aortic root abscess. is
patient is a candidate for surgery and should have Valve debridement, valve repair, or replacement

Rifampicin is beneficial in infective endocarditis in a prosthetic heart valve.

Beta-blockers will aggravate the prolongation of PR on ECG and may result in a higher degree of AV
block

Indications for surgery in IE:

Severe valvular incompetence


An aortic abscess (oen indicated by a lengthening PR interval)
Infections resistant to antibiotics/fungal infections
Cardiac failure refractory to standard medical treatment

Recurrent emboli aer antibiotic therapy

Page - 259
Internal Medicine - Cardiology

Question 119/155

Question #119

A 65-year-old male patient presents with fever, chills, and weight loss for a 1-month duration. His
temperature is 39°C, his pulse is 105 bpm, and his chest examination reveals bibasilar crepitations
and pan-systolic murmur. His hand examination reveals a splinter hemorrhage. His WBC is
13,000/mm3, and his urinalysis is normal. What is the most likely diagnosis?

a. Infective endocarditis
b. Urinary tract infection
c. Bacterial pneumonia
d. Lymphoma
e. Diverticulitis

‫اﻹﺟﺎﺑﺔ ﻋﲆ اﻟﺼﻔﺤﺔ اﻟﺘﺎﻟﻴﺔ‬

Page - 260
Internal Medicine - Cardiology - Infective Endocarditis (IE)

Question 119/155

Question #119

A 65-year-old male patient presents with fever, chills, and weight loss for a 1-month duration. His
temperature is 39°C, his pulse is 105 bpm, and his chest examination reveals bibasilar crepitations
and pan-systolic murmur. His hand examination reveals a splinter hemorrhage. His WBC is
13,000/mm3, and his urinalysis is normal. What is the most likely diagnosis?

a. Infective endocarditis √
b. Urinary tract infection
c. Bacterial pneumonia
d. Lymphoma
e. Diverticulitis

Description

Page - 261
Internal Medicine - Cardiology

Question 120/155

Question #120

A 32-year-old male patient known as an IV drug user presents with fever, chills, and heart murmur.
What is the most appropriate test to do for this patient?

a. ECG
b. Blood culture
c. Urine culture
d. Chest x-ray
e. CBC

‫اﻹﺟﺎﺑﺔ ﻋﲆ اﻟﺼﻔﺤﺔ اﻟﺘﺎﻟﻴﺔ‬

Page - 262
Internal Medicine - Cardiology - Infective Endocarditis (IE)

Question 120/155

Question #120

A 32-year-old male patient known as an IV drug user presents with fever, chills, and heart murmur.
What is the most appropriate test to do for this patient?

a. ECG
b. Blood culture √
c. Urine culture
d. Chest x-ray
e. CBC

Description

is patient is suspected of having infective endocarditis. Blood culture and echocardiogram are the
most appropriate for diagnosis at this time

Page - 263
Internal Medicine - Cardiology

Question 121/155

Question #121

A 39-year-old intravenous drug user has a fever and a new-onset murmur. Which of the following
statements is true?

a. e prevalence of endocarditis in this patient is equal to the average population


b. e most common site is the mitral valve
c. Streptococcus pneumonia is the most common cause
d. It is typically found in the tricuspid valve
e. e treatment of choice is NSAIDs and bed rest

‫اﻹﺟﺎﺑﺔ ﻋﲆ اﻟﺼﻔﺤﺔ اﻟﺘﺎﻟﻴﺔ‬

Page - 264
Internal Medicine - Cardiology - Infective Endocarditis (IE)

Question 121/155

Question #121

A 39-year-old intravenous drug user has a fever and a new-onset murmur. Which of the following
statements is true?

a. e prevalence of endocarditis in this patient is equal to the average population


b. e most common site is the mitral valve
c. Streptococcus pneumonia is the most common cause
d. It is typically found in the tricuspid valve √
e. e treatment of choice is NSAIDs and bed rest

Description

Infective endocarditis (IE) is more prevalent in intravenous drug users, the most common cause of IE
in IV drug users is staphylococcus aureus, and it most commonly aects the tricuspid valve

e most common valve aected is the mitral valve, but the tricuspid is more likely to be involved in
IV drug users.

e treatment of choice for IE is intravenous antibiotics, not mere NSAIDs and bed rest

Page - 265
Internal Medicine - Cardiology

Question 122/155

Question #122

A 23-year-old female is an intravenous drug user who presents with fever, systolic murmur, and
general weakness. She is suspected of having infective endocarditis. What is the most common
valve to be aected in this patient?

a. Aortic valve
b. Mitral valve
c. Tricuspid valve
d. Pulmonary valve
e. ey are equally aected

‫اﻹﺟﺎﺑﺔ ﻋﲆ اﻟﺼﻔﺤﺔ اﻟﺘﺎﻟﻴﺔ‬

Page - 266
Internal Medicine - Cardiology - Infective Endocarditis (IE)

Question 122/155

Question #122

A 23-year-old female is an intravenous drug user who presents with fever, systolic murmur, and
general weakness. She is suspected of having infective endocarditis. What is the most common
valve to be aected in this patient?

a. Aortic valve
b. Mitral valve
c. Tricuspid valve √
d. Pulmonary valve
e. ey are equally aected

Description

Infective endocarditis in IV drug users most commonly involves the tricuspid valve

In cases of infective endocarditis in non-IV drug users patients, the Mitral valve is the most
commonly aected.

Page - 267
Internal Medicine - Cardiology

Question 123/155

Question #123

A patient is taking Isosorbide mononitrate for his stable angina. All the following about this drug are
true except:

a. It has excellent bioavailability when administrated orally


b. e elimination half-life is 4 – 5 hours
c. Mononitrate has a longer half-life than Isosorbide dinitrate
d. It is better to be taken sublingually.
e. 2% of isosorbide mononitrate is excreted unchanged in the urine

‫اﻹﺟﺎﺑﺔ ﻋﲆ اﻟﺼﻔﺤﺔ اﻟﺘﺎﻟﻴﺔ‬

Page - 268
Internal Medicine - Cardiology - Nitroglycerin

Question 123/155

Question #123

A patient is taking Isosorbide mononitrate for his stable angina. All the following about this drug are
true except:

a. It has excellent bioavailability when administrated orally


b. e elimination half-life is 4 – 5 hours
c. Mononitrate has a longer half-life than Isosorbide dinitrate
d. It is better to be taken sublingually. √
e. 2% of isosorbide mononitrate is excreted unchanged in the urine

Description

Isosorbide mononitrate does not undergo significant first-pass hepatic metabolism, so it has
excellent bioavailability when given orally.

Sublingual preparations are not available for mononitrates (not needed)

Mononitrates have a longer half-life and slower elimination than dinitrates. It is formulated as a
plain tablet and as a sustained-release preparation.

Sublingual isosorbide dinitrate produces maximal plasma concentrations of the drug by 6 minutes,
and the fall in concentration is rapid (half-life of approximately 45 minutes).

Glyceryl nitrate is an alternative term for nitroglycerine. In humans, peak concentrations of


nitroglycerin are found in plasma within 4 minutes of sublingual administration

Page - 269
Internal Medicine - Cardiology

Question 124/155

Question #124

e following are known mechanisms in which nitrates relieve the myocardial ischemic pain except:

a. Vasodilatation of the coronary artery


b. Reduce the systemic vascular resistance
c. Reduce the ventricular volume
d. Increase the oxygen demand
e. Reduce the preload and aerload

‫اﻹﺟﺎﺑﺔ ﻋﲆ اﻟﺼﻔﺤﺔ اﻟﺘﺎﻟﻴﺔ‬

Page - 270
Internal Medicine - Cardiology - Nitroglycerin

Question 124/155

Question #124

e following are known mechanisms in which nitrates relieve the myocardial ischemic pain except:

a. Vasodilatation of the coronary artery


b. Reduce the systemic vascular resistance
c. Reduce the ventricular volume
d. Increase the oxygen demand √
e. Reduce the preload and aerload

Description

Nitrates dilate the veins, arteries, and arterioles (including the coronary artery). is will lead to a
reduction in systemic vascular resistance and a reduction of the cardiac workload, and thus a
reduction of the oxygen demands.

Nitrates reduce the preload and aerload.

It is indicated in the case of acute MI (to reduce the infarction size), heart failure and acute
pulmonary edema, and recurrent chronic anginal pain.

Page - 271
Internal Medicine - Cardiology

Question 125/155

Question #125

A 54-year-old female has had chest pain for the last 2 hours, and she was diagnosed with pulmonary
embolism. Her ECG is shown below. All the following features of PE are seen in her ECG except:

a. Right bundle branch block


b. Extreme axis deviation
c. S1Q3T3
d. Inverted T wave in V1 – V4
e. Sinus tachycardia

‫اﻹﺟﺎﺑﺔ ﻋﲆ اﻟﺼﻔﺤﺔ اﻟﺘﺎﻟﻴﺔ‬

Page - 272
Internal Medicine - Cardiology - Normal heart conduction and basic ECG

Question 125/155

Question #125

A 54-year-old female has had chest pain for the last 2 hours, and she was diagnosed with pulmonary
embolism. Her ECG is shown below. All the following features of PE are seen in her ECG except:

a. Right bundle branch block


b. Extreme axis deviation
c. S1Q3T3
d. Inverted T wave in V1 – V4
e. Sinus tachycardia √

Description

Sinus tachycardia is the most common ECG finding in pulmonary embolism, but here it is not
present in the ECG of this patient.

e other mentioned choices are present in pulmonary embolism and are clearly shown in the ECG
above.

Page - 273
Internal Medicine - Cardiology

Question 126/155

Question #126

Which of the following is considered the normal pacemaker of the heart?

a. Atrioventricular node
b. Sinoatrial node
c. e interstitial cell of Cajal
d. Kent bundle
e. HIS bundle

‫اﻹﺟﺎﺑﺔ ﻋﲆ اﻟﺼﻔﺤﺔ اﻟﺘﺎﻟﻴﺔ‬

Page - 274
Internal Medicine - Cardiology - Normal heart conduction and basic ECG

Question 126/155

Question #126

Which of the following is considered the normal pacemaker of the heart?

a. Atrioventricular node
b. Sinoatrial node √
c. e interstitial cell of Cajal
d. Kent bundle
e. HIS bundle

Description

e normal pacemaker of the heart is the Sinoatrial (SA) node.

Typically, it generates impulses at a rate of 60 – 100 per minute

e normal conduction system of the heart is as the following:

SA node generates impulse → spread through atria → delayed at the AV node → spread down through
the HIS bundle → spread to Right and Le Bundle branches → Purkinje fibers

Note that the interstitial cell of Cajal is the normal pacemaker of the GI system, not the heart

Page - 275
Internal Medicine - Cardiology

Question 127/155

Question #127

About ECG interpretation, which of the following statements is true?

a. e P wave represents atrial depolarization


b. e QRS complex represents ventricular depolarization
c. e T wave represents ventricular repolarization
d. e heart rate “if regular” is calculated by dividing 300 by the RR interval
e. All of the above

‫اﻹﺟﺎﺑﺔ ﻋﲆ اﻟﺼﻔﺤﺔ اﻟﺘﺎﻟﻴﺔ‬

Page - 276
Internal Medicine - Cardiology - Normal heart conduction and basic ECG

Question 127/155

Question #127

About ECG interpretation, which of the following statements is true?

a. e P wave represents atrial depolarization


b. e QRS complex represents ventricular depolarization
c. e T wave represents ventricular repolarization
d. e heart rate “if regular” is calculated by dividing 300 by the RR interval
e. All of the above √

Description

Normally on ECG:

P wave: represents atrial depolarization (contraction)


QRS waves: represents ventricular depolarization (contraction)
T wave: represents ventricular repolarization (relaxation)

Rate assessment:

Normal heart rate is 60 – 100 bpm


A rate of more than 100 bpm is called tachycardia
A rate less than 60 bpm is called bradycardia
Calculate the rate by dividing 300/the number of large squares between each QRS complex

Page - 277
Internal Medicine - Cardiology

Question 128/155

Question #128

A 66-year-old male patient has had COPD for a long time, and his ECG is shown below. What is the
name of this condition?

a. Electrical alternans
b. P mitrale
c. P Pulmonale
d. Sinus bradycardia
e. Normal ECG morphology

‫اﻹﺟﺎﺑﺔ ﻋﲆ اﻟﺼﻔﺤﺔ اﻟﺘﺎﻟﻴﺔ‬

Page - 278
Internal Medicine - Cardiology - Normal heart conduction and basic ECG

Question 128/155

Question #128

A 66-year-old male patient has had COPD for a long time, and his ECG is shown below. What is the
name of this condition?

a. Electrical alternans
b. P mitrale
c. P Pulmonale √
d. Sinus bradycardia
e. Normal ECG morphology

Description

e presence of high P wave amplitude here indicates right atrial hypertrophy. is condition is
called P Pulmonale

e high amplitude P wave and COPD indicate elevated pulmonary pressure, leading to right-side
heart failure and right-side hypertrophy of the heart.

Page - 279
Internal Medicine - Cardiology

Question 129/155

Question #129

e following figure shows an ECG of a 62-year-old male patient. What is the most likely diagnosis?

a. Diuse ST elevation (pericarditis)


b. Normal sinus rhythm
c. Right bundle branch block
d. Le bundle branch block
e. Atrioventricular block

‫اﻹﺟﺎﺑﺔ ﻋﲆ اﻟﺼﻔﺤﺔ اﻟﺘﺎﻟﻴﺔ‬

Page - 280
Internal Medicine - Cardiology - Normal heart conduction and basic ECG

Question 129/155

Question #129

e following figure shows an ECG of a 62-year-old male patient. What is the most likely diagnosis?

a. Diuse ST elevation (pericarditis)


b. Normal sinus rhythm
c. Right bundle branch block
d. Le bundle branch block √
e. Atrioventricular block

Description

is ECG is irregularly irregular.

e heart rate is approximately 62 bpm

No P waves

f waves are present

QRS complexes are wide with a morphology of bundle branch block

Positive R in V6 indicates LBBB

Page - 281
is ECG shows atrial fibrillation and LBBB

Page - 282
Internal Medicine - Cardiology

Question 130/155

Question #130

A 67-year-old male patient with a history of lymphoma is diagnosed with cardiac tamponade. Which
of the following findings is inconsistent with this condition?

a. Hypotension
b. High JVP
c. Pulsus paradoxus
d. Quiet heart sound
e. Pericardial friction rub

‫اﻹﺟﺎﺑﺔ ﻋﲆ اﻟﺼﻔﺤﺔ اﻟﺘﺎﻟﻴﺔ‬

Page - 283
Internal Medicine - Cardiology - Pericardial eusion and Tamponade

Question 130/155

Question #130

A 67-year-old male patient with a history of lymphoma is diagnosed with cardiac tamponade. Which
of the following findings is inconsistent with this condition?

a. Hypotension
b. High JVP
c. Pulsus paradoxus
d. Quiet heart sound
e. Pericardial friction rub √

Description

Pericardial friction rub presents in the case of pericarditis, not cardiac tamponade.

e friction rub results from the friction between the parietal and visceral layers of the pericardium,
which are not in contact with each other in the case of tamponade

Cardiac tamponade:

Cardiac tamponade is a rapid or massive accumulation of pericardial eusion or blood


collection in the pericardial space
It causes decreased venous return, filling, and stroke volume.

It presents with Beck’s tirade (hypotension, increased JVP, and mued heart sounds)
Other symptoms include dyspnea, pulsus paradoxus, shock, and peripheral edema
Pulsus paradoxus (inspiratory fall in systolic BP > 10 mmHg during quiet breathing)

Electrical alternans (high and low voltage of QRSs on ECG alternatively)

e best treatment of cardiac tamponade is urgent pericardiocentesis (it is diagnostic and


therapeutic)

Page - 284
Internal Medicine - Cardiology

Question 131/155

Question #131

Which of the following is considered a diagnostic and therapeutic procedure for pericardial
eusion?

a. Transthoracic echocardiogram
b. Transesophageal echocardiogram
c. Pericardiocentesis
d. oracentesis
e. Beck’s triad

‫اﻹﺟﺎﺑﺔ ﻋﲆ اﻟﺼﻔﺤﺔ اﻟﺘﺎﻟﻴﺔ‬

Page - 285
Internal Medicine - Cardiology - Pericardial eusion and Tamponade

Question 131/155

Question #131

Which of the following is considered a diagnostic and therapeutic procedure for pericardial
eusion?

a. Transthoracic echocardiogram
b. Transesophageal echocardiogram
c. Pericardiocentesis √
d. oracentesis
e. Beck’s triad

Description

Pericardial eusion is defined as a fluid collection within the pericardium

Pericardiocentesis is the most appropriate diagnostic and therapeutic procedure for pericardial
eusion (it determines exudates vs. transudate)

It can be transudate due to (CHF, hypothyroidism, or hypoalbuminemia) or exudate due to (the


same causes of pericarditis).

Symptoms depend on the amount of fluid and may be asymptomatic, dyspnea, cough, signs of
pericarditis, elevated JVP and Ewart’s sign, or mued heart sounds

Echocardiograms are diagnostic only. While becks triad is diagnostic for cardiac tamponade.

oracentesis is used for pleural eusion, not pericardial one.

Page - 286
Internal Medicine - Cardiology

Question 132/155

Question #132

A patient was exposed to chest trauma during a road trac accident; his ECG is shown below. What
does this patient most likely have?

a. Myocardial infarction
b. Atrial fibrillation
c. Normal ECG
d. Cardiac tamponade
e. Cardiac muscle contusion

‫اﻹﺟﺎﺑﺔ ﻋﲆ اﻟﺼﻔﺤﺔ اﻟﺘﺎﻟﻴﺔ‬

Page - 287
Internal Medicine - Cardiology - Pericardial eusion and Tamponade

Question 132/155

Question #132

A patient was exposed to chest trauma during a road trac accident; his ECG is shown below. What
does this patient most likely have?

a. Myocardial infarction
b. Atrial fibrillation
c. Normal ECG
d. Cardiac tamponade √
e. Cardiac muscle contusion

Description

is ECG shows dierent QRS complexes amplitude alternatively.

e condition here is called electrical alternans, which is characteristic in the case of cardiac
tamponade

Cardiac tamponade:

Cardiac tamponade is a rapid or massive accumulation of pericardial eusion or blood


collection in the pericardial space
It causes decreased venous return, filling, and stroke volume.
Page - 288
It presents with Beck’s tirade (hypotension, increased JVP, and mued heart sounds)
Other symptoms include dyspnea, pulsus paradoxus, shock, and peripheral edema
Pulsus paradoxus (inspiratory fall in systolic BP > 10 mmHg during quiet breathing)

Electrical alternans (high and low voltage of QRSs on ECG alternatively)

e best treatment of cardiac tamponade is urgent pericardiocentesis (it is diagnostic and


therapeutic)

Page - 289
Internal Medicine - Cardiology

Question 133/155

Question #133

A 22-year-old male patient has a mild fever and chest pain relieved by leaning forward. His ECG is
shown below. What is the most likely diagnosis?

a. Acute myocardial infarction


b. Pericarditis
c. Myocarditis
d. Pneumonia
e. Muscular spasms

‫اﻹﺟﺎﺑﺔ ﻋﲆ اﻟﺼﻔﺤﺔ اﻟﺘﺎﻟﻴﺔ‬

Page - 290
Internal Medicine - Cardiology - Pericarditis

Question 133/155

Question #133

A 22-year-old male patient has a mild fever and chest pain relieved by leaning forward. His ECG is
shown below. What is the most likely diagnosis?

a. Acute myocardial infarction


b. Pericarditis √
c. Myocarditis
d. Pneumonia
e. Muscular spasms

Description

Diuse saddle shape ST-elevation on ECG and the presence of positional chest pain are suggestive of
pericarditis.

Pericarditis:

Pericarditis is an inflammation of the pericardium


Most commonly – idiopathic, but the most common cause is coxsackie B virus infection
A friction rub is the earliest sign of pericarditis
Atypical chest pain (positional; increased by lying back and decreased by bending forward)
e most specific ECG change is depressed PR interval (elevated on AVR)
Page - 291
Other ECG changes: ST elevation in all leads (diuse ST elevation) (Depression on AVR)

Page - 292
Internal Medicine - Cardiology

Question 134/155

Question #134

A 30-year-old male patient has positional chest pain and is diagnosed with acute pericarditis. Which
of the following drugs is most commonly used to treat this condition?

a. Paracetamol
b. Prednisolone
c. Heparin
d. Ibuprofen
e. Cefuroxime

‫اﻹﺟﺎﺑﺔ ﻋﲆ اﻟﺼﻔﺤﺔ اﻟﺘﺎﻟﻴﺔ‬

Page - 293
Internal Medicine - Cardiology - Pericarditis

Question 134/155

Question #134

A 30-year-old male patient has positional chest pain and is diagnosed with acute pericarditis. Which
of the following drugs is most commonly used to treat this condition?

a. Paracetamol
b. Prednisolone
c. Heparin
d. Ibuprofen √
e. Cefuroxime

Description

Pericarditis:

Pericarditis is an inflammation of the pericardium


Most commonly – idiopathic, but the most common cause is coxsackie B virus infection
A friction rub is the earliest sign of pericarditis
Atypical chest pain (positional; increased by lying back and decreased by bending forward)
e most specific ECG change is depressed PR interval (elevated on AVR)
Other ECG changes: ST elevation in all leads (diuse ST elevation) (Depression on AVR)

Treatment of acute pericarditis includes:

Bed-rest, NSAIDs
Colchicine decreased recurrence
Steroids for severe or recurrent cases
Treatment of the underlying disease

NSAIDs are the first-line treatment, e.g., ibuprofen for 3 weeks.

Other NSAIDs can be used, but steroids are better avoided because they can cause the steroid-
dependent syndrome.

A combination of ibuprofen and colchicine may be ideal

Anticoagulants are not used in pericarditis; they increase the risk of cardiac tamponade
Page - 294
Page - 295
Internal Medicine - Cardiology

Question 135/155

Question #135

A 40-year-old male patient presents with acute pleuritic chest pain relieved by leaning forward, and
a physical examination reveals a pericardial friction rub. Pericarditis is suspected. Which of the
following ECG findings is the most specific for pericarditis?

a. Widespread ST elevation
b. PR depression
c. T inversion
d. Pathological Q wave
e. Short PR interval

‫اﻹﺟﺎﺑﺔ ﻋﲆ اﻟﺼﻔﺤﺔ اﻟﺘﺎﻟﻴﺔ‬

Page - 296
Internal Medicine - Cardiology - Pericarditis

Question 135/155

Question #135

A 40-year-old male patient presents with acute pleuritic chest pain relieved by leaning forward, and
a physical examination reveals a pericardial friction rub. Pericarditis is suspected. Which of the
following ECG findings is the most specific for pericarditis?

a. Widespread ST elevation
b. PR depression √
c. T inversion
d. Pathological Q wave
e. Short PR interval

Description

e most specific ECG change is depressed PR interval (elevated on AVR).

Pericarditis:

Pericarditis is an inflammation of the pericardium


Most commonly – idiopathic, but the most common cause is coxsackie B virus infection
A friction rub is the earliest sign of pericarditis
Atypical chest pain (positional; increased by lying back and decreased by bending forward)
e most specific ECG change is depressed PR interval (elevated on AVR)
Other ECG changes: ST elevation in all leads (diuse ST elevation) (Depression on AVR)

Page - 297
Internal Medicine - Cardiology

Question 136/155

Question #136

A 40-year-old male patient presents with acute pleuritic chest pain relieved by leaning forward, and
a physical examination reveals a pericardial friction rub. What is the most appropriate
management?

a. Antibiotics
b. Antivirals
c. Anti-inflammatory agents
d. Bronchodilators
e. Beta-blockers

‫اﻹﺟﺎﺑﺔ ﻋﲆ اﻟﺼﻔﺤﺔ اﻟﺘﺎﻟﻴﺔ‬

Page - 298
Internal Medicine - Cardiology - Pericarditis

Question 136/155

Question #136

A 40-year-old male patient presents with acute pleuritic chest pain relieved by leaning forward, and
a physical examination reveals a pericardial friction rub. What is the most appropriate
management?

a. Antibiotics
b. Antivirals
c. Anti-inflammatory agents √
d. Bronchodilators
e. Beta-blockers

Description

is is a typical condition of pericarditis; its treatment depends on the cause. In this case, viral
infection is the most likely. e treatment contains analgesics, anti-inflammatory agents (NSAIDs),
and sometimes surgery.

Pericarditis:

Pericarditis is an inflammation of the pericardium


Most commonly – idiopathic, but the most common cause is coxsackie B virus infection
A friction rub is the earliest sign of pericarditis
Atypical chest pain (positional; increased by lying back and decreased by bending forward)
e most specific ECG change is depressed PR interval (elevated on AVR)
Other ECG changes: ST elevation in all leads (diuse ST elevation) (Depression on AVR)

Page - 299
Internal Medicine - Cardiology

Question 137/155

Question #137

A previously healthy 23-year-old presented to the ER with typical chest pain at rest and ST elevation
on ECG and was admitted to the CCU for treatment. e second ECG 15 minutes later shows normal
sinus rhythm with no ST/T changes, and his cardiac enzymes are flat. Which of the following is the
most likely diagnosis?

a. Stable angina
b. Unstable angina
c. Prinzmetal angina
d. Non-ST elevation MI
e. ST-elevation MI

‫اﻹﺟﺎﺑﺔ ﻋﲆ اﻟﺼﻔﺤﺔ اﻟﺘﺎﻟﻴﺔ‬

Page - 300
Internal Medicine - Cardiology - Prinzmetal Angina

Question 137/155

Question #137

A previously healthy 23-year-old presented to the ER with typical chest pain at rest and ST elevation
on ECG and was admitted to the CCU for treatment. e second ECG 15 minutes later shows normal
sinus rhythm with no ST/T changes, and his cardiac enzymes are flat. Which of the following is the
most likely diagnosis?

a. Stable angina
b. Unstable angina
c. Prinzmetal angina √
d. Non-ST elevation MI
e. ST-elevation MI

Description

A young, previously healthy patient with transient chest pain and ST elevation is suggestive of
Prinzmetal angina (coronary vasospasm)

Stable angina is unlikely in a young patient with chest pain not related to exertion

Unstable angina usually presents with non-specific ST/T changes, and chest pain at rest in elderly
patients or patients with a risk factor for IHD

Unlike this case, STEMI and NSTEMI will always manifest with high cardiac enzymes

Page - 301
Internal Medicine - Cardiology

Question 138/155

Question #138

All the following are primary prevention of coronary heart disease, except:

a. Regular exercise
b. Blood pressure control
c. Blood sugar control
d. Not to smoke
e. ACE inhibitors use

‫اﻹﺟﺎﺑﺔ ﻋﲆ اﻟﺼﻔﺤﺔ اﻟﺘﺎﻟﻴﺔ‬

Page - 302
Internal Medicine - Cardiology - Stable angina

Question 138/155

Question #138

All the following are primary prevention of coronary heart disease, except:

a. Regular exercise
b. Blood pressure control
c. Blood sugar control
d. Not to smoke
e. ACE inhibitors use √

Description

Primary prevention means preventing the disease from developing

Secondary prevention means preventing the disease from getting worse aer it already occurs

Tertiary prevention means improving the quality of life and reducing the symptoms of the disease
aer it appears.

Regular exercise, stopping smoking, and controlling DM and HTN will prevent IHD and are
considered primary prevention measures.

e use of ACE inhibitors is considered a secondary prevention measure (not primary)

Page - 303
Internal Medicine - Cardiology

Question 139/155

Question #139

An obese 63-year-old male with a history of HTN and DM presents with a squeezing, pressure-like
sensation in his chest radiating to the jaw, shoulders, and neck. e pain sustains for several
minutes and then subsides. e patient has normal blood pressure, heart rate, and ECG. What is the
most likely diagnosis?

a. Myocardial infarction
b. Angina pectoris
c. Transient ischemic attack
d. GERD
e. Bradyarrhythmia

‫اﻹﺟﺎﺑﺔ ﻋﲆ اﻟﺼﻔﺤﺔ اﻟﺘﺎﻟﻴﺔ‬

Page - 304
Internal Medicine - Cardiology - Stable angina

Question 139/155

Question #139

An obese 63-year-old male with a history of HTN and DM presents with a squeezing, pressure-like
sensation in his chest radiating to the jaw, shoulders, and neck. e pain sustains for several
minutes and then subsides. e patient has normal blood pressure, heart rate, and ECG. What is the
most likely diagnosis?

a. Myocardial infarction
b. Angina pectoris √
c. Transient ischemic attack
d. GERD
e. Bradyarrhythmia

Description

is patient has typical chest pain and is most likely suering from stable angina.

Features of typical chest pain:

Central, retrosternal
Heavy, dull, squeezing in character, or pressure-like.
Radiated to neck, jaw, epigastrium, shoulders, or back
Increased by exercise and relieved by rest or nitrates

Page - 305
Internal Medicine - Cardiology

Question 140/155

Question #140

A 52-year-old male patient presents with episodic, recurrent chest pain related to exercise. e pain
radiates to both shoulders and back. It is usually subsided within 5 minutes of rest. His lab
investigations, including lipid profile, liver function test, kidney function test, ECG, and chest x-ray,
are normal. What is the next step in the management of this patient?

a. Cardiac enzyme measurement


b. ECG stress test
c. Echocardiogram
d. Coronary angiography
e. Chest CT scan

‫اﻹﺟﺎﺑﺔ ﻋﲆ اﻟﺼﻔﺤﺔ اﻟﺘﺎﻟﻴﺔ‬

Page - 306
Internal Medicine - Cardiology - Stable angina

Question 140/155

Question #140

A 52-year-old male patient presents with episodic, recurrent chest pain related to exercise. e pain
radiates to both shoulders and back. It is usually subsided within 5 minutes of rest. His lab
investigations, including lipid profile, liver function test, kidney function test, ECG, and chest x-ray,
are normal. What is the next step in the management of this patient?

a. Cardiac enzyme measurement


b. ECG stress test √
c. Echocardiogram
d. Coronary angiography
e. Chest CT scan

Description

e patient is complaining of typical chest pain and is suspected of having chronic stable angina.
erefore, the best next step in his condition is to do an exercise ECG stress test.

Coronary angiography is accurate, but it is not the next step. Echocardiogram will be needed in
conditions when a structural heart disease or cardiac dysfunction is suspected.

Page - 307
Internal Medicine - Cardiology

Question 141/155

Question #141

In which of the following patients is coronary angiography indicated?

a. A 55-year-old female with sharp chest pain and late systolic murmur with mid-systolic click
b. A 66-year-old male with pleuritic chest pain at rest
c. A 75-year-old female with HTN, dyslipidemia, asymptomatic PVCs, and Reflux disease pain
d. A 42-year-old male with intermittent angina pain and a negative exercise stress test
e. An asymptomatic 39-year-old male with a family history of heart disease

‫اﻹﺟﺎﺑﺔ ﻋﲆ اﻟﺼﻔﺤﺔ اﻟﺘﺎﻟﻴﺔ‬

Page - 308
Internal Medicine - Cardiology - Stable angina

Question 141/155

Question #141

In which of the following patients is coronary angiography indicated?

a. A 55-year-old female with sharp chest pain and late systolic murmur with mid-systolic click
b. A 66-year-old male with pleuritic chest pain at rest
c. A 75-year-old female with HTN, dyslipidemia, asymptomatic PVCs, and Reflux disease pain
d. A 42-year-old male with intermittent angina pain and a negative exercise stress test √
e. An asymptomatic 39-year-old male with a family history of heart disease

Description

Coronary angiography is the most accurate test in the case of coronary artery disease, but it is not
indicated to be done routinely.

e typical clinical presentation, ECG findings, Cardiac enzyme levels, and the ECG stress test should
be used appropriately in ischemic heart diseases to reach the diagnosis.

If the patient has recurrent anginal pain and no alternative diagnosis is suspected, coronary
angiography can be used to establish the diagnosis.

Page - 309
Internal Medicine - Cardiology

Question 142/155

Question #142

A 66-year-old male patient complains of angina chest pain unrelated to exertion. It occurs
intermittently and usually while he is lying on the couch or bed. Which type of angina does the
patient experience?

a. Stable angina
b. Unstable angina
c. Angina decubitus
d. Prinzmetal angina
e. Angina equivalent

‫اﻹﺟﺎﺑﺔ ﻋﲆ اﻟﺼﻔﺤﺔ اﻟﺘﺎﻟﻴﺔ‬

Page - 310
Internal Medicine - Cardiology - Stable angina

Question 142/155

Question #142

A 66-year-old male patient complains of angina chest pain unrelated to exertion. It occurs
intermittently and usually while he is lying on the couch or bed. Which type of angina does the
patient experience?

a. Stable angina
b. Unstable angina
c. Angina decubitus √
d. Prinzmetal angina
e. Angina equivalent

Description

Angina decubitus is angina on changing position. Angina decubitus occurs because gravity
redistributes fluids in the body. is redistribution makes the heart work harder

Page - 311
Internal Medicine - Cardiology

Question 143/155

Question #143

A 55-year-old male patient has recurrent, central, crushing chest pain that manifests aer significant
exertion and is relieved by rest. In addition, he mentioned that his pain radiates to the jaw and both
shoulders. Physical examination is unremarkable except for obvious central obesity and acanthosis
nigricans. However, his ECG and cardiac enzymes are unremarkable. What is the first-line treatment
for his condition?

a. Beta-blockers
b. Calcium channel blockers
c. Sublingual nitrates on demand
d. Salbutamol inhaler on demand
e. Refer for Percutaneous coronary intervention (PCI)

‫اﻹﺟﺎﺑﺔ ﻋﲆ اﻟﺼﻔﺤﺔ اﻟﺘﺎﻟﻴﺔ‬

Page - 312
Internal Medicine - Cardiology - Stable angina

Question 143/155

Question #143

A 55-year-old male patient has recurrent, central, crushing chest pain that manifests aer significant
exertion and is relieved by rest. In addition, he mentioned that his pain radiates to the jaw and both
shoulders. Physical examination is unremarkable except for obvious central obesity and acanthosis
nigricans. However, his ECG and cardiac enzymes are unremarkable. What is the first-line treatment
for his condition?

a. Beta-blockers √
b. Calcium channel blockers
c. Sublingual nitrates on demand
d. Salbutamol inhaler on demand
e. Refer for Percutaneous coronary intervention (PCI)

Description

is patient suers from stable angina, and he has strong indications of insulin resistance.

e first-line treatment of stable angina is beta-blockers.

Calcium channel blockers are the second-line treatment in this case

Nitrates alone will not reduce the mortality but only symptoms.

In the stable angina, PCI is found not to reduce the mortality rate more than the drug therapy alone,
so it is not indicated unless the patient has a poor quality of life and worsening symptoms despite
medical treatment.

Salbutamol inhaler is a short-acting beta agonist used to treat asthma; our patient is not asthmatic
and is aware that SABA will increase the heart rate, the oxygen demands and thus worsening his
symptoms.

Page - 313
Internal Medicine - Cardiology

Question 144/155

Question #144

A 77-year-old male patient has passed out 5 times in the last 6 months. All 5 attacks occurred while
the patient was walking upstairs. According to his history alone, which of the following is the most
likely cause?

a. Vasovagal syncope
b. Transient ischemic attacks (TIA)
c. Hypotension
d. Aortic stenosis
e. Hypoglycemia

‫اﻹﺟﺎﺑﺔ ﻋﲆ اﻟﺼﻔﺤﺔ اﻟﺘﺎﻟﻴﺔ‬

Page - 314
Internal Medicine - Cardiology - Syncope

Question 144/155

Question #144

A 77-year-old male patient has passed out 5 times in the last 6 months. All 5 attacks occurred while
the patient was walking upstairs. According to his history alone, which of the following is the most
likely cause?

a. Vasovagal syncope
b. Transient ischemic attacks (TIA)
c. Hypotension
d. Aortic stenosis √
e. Hypoglycemia

Description

Patients who present with syncope with exercise should be evaluated for organic structural heart
diseases.

About 50% of patients with severe aortic stenosis have reported syncope on exertion.

Vasovagal attacks are associated with unpleasant stimuli which are not present here.

TIA usually presents with a neurological deficit and is not related to the

Exertion.

ere is no evidence of hypotension or hypoglycemia in this patient.

Page - 315
Internal Medicine - Cardiology

Question 145/155

Question #145

Which of the following is the most common cause of syncope?

a. Cardiac dysrhythmia
b. Medications eects
c. Orthostatic hypotension
d. Psychiatric disorders
e. Vasovagal syncope

‫اﻹﺟﺎﺑﺔ ﻋﲆ اﻟﺼﻔﺤﺔ اﻟﺘﺎﻟﻴﺔ‬

Page - 316
Internal Medicine - Cardiology - Syncope

Question 145/155

Question #145

Which of the following is the most common cause of syncope?

a. Cardiac dysrhythmia
b. Medications eects
c. Orthostatic hypotension
d. Psychiatric disorders
e. Vasovagal syncope √

Description

Neurocardiogenic is also known as vasovagal syncope.

It is the largest group of disorders causing syncope.

ese disorders result from reflex-mediated changes in vascular tone or heart rate.

Page - 317
Internal Medicine - Cardiology

Question 146/155

Question #146

e tilt table test is used to assess which of the following conditions?

a. Vasovagal syncope
b. Coronary artery disease
c. Transient ischemic attack
d. Elevated jugular venous pressure
e. Complete AV block

‫اﻹﺟﺎﺑﺔ ﻋﲆ اﻟﺼﻔﺤﺔ اﻟﺘﺎﻟﻴﺔ‬

Page - 318
Internal Medicine - Cardiology - Syncope

Question 146/155

Question #146

e tilt table test is used to assess which of the following conditions?

a. Vasovagal syncope √
b. Coronary artery disease
c. Transient ischemic attack
d. Elevated jugular venous pressure
e. Complete AV block

Description

e tilt table test is used to record blood pressure, heart rhythm, and heart rate on a beat-by-beat
basis as the table is tilted to dierent angles; the table always stays head-up.

e table will be tilted to 30 degrees for 2 to 3 minutes, 45 degrees for 2 to 3 minutes, and 70 degrees
for up to 45 minutes.

is test is used to evaluate patients with syncope.

Page - 319
Internal Medicine - Cardiology

Question 147/155

Question #147

A 17-year-old male presents due to recurrent loss of consciousness while playing sports. On
examination, a systolic ejection murmur is present at the right second intercostal space and
radiated to the neck. What is the most common cause of this patient’s condition?

a. Mitral valve prolapse


b. Calcific aortic valve stenosis
c. Tricuspid valve stenosis
d. Bicuspid aortic valve
e. Aortic regurgitation

‫اﻹﺟﺎﺑﺔ ﻋﲆ اﻟﺼﻔﺤﺔ اﻟﺘﺎﻟﻴﺔ‬

Page - 320
Internal Medicine - Cardiology - Valvular diseases and heart murmurs

Question 147/155

Question #147

A 17-year-old male presents due to recurrent loss of consciousness while playing sports. On
examination, a systolic ejection murmur is present at the right second intercostal space and
radiated to the neck. What is the most common cause of this patient’s condition?

a. Mitral valve prolapse


b. Calcific aortic valve stenosis
c. Tricuspid valve stenosis
d. Bicuspid aortic valve √
e. Aortic regurgitation

Description

Exercise-induced syncope is usually due to fixed cardiac output, which is not rising during exercise
and high demand. is presentation is most likely to be due to aortic stenosis

e presence of a systolic ejection murmur supports this diagnosis as well.

Calcific aortic stenosis is more common in old age, but the bicuspid aortic valve is more common in
young age patients

Page - 321
Internal Medicine - Cardiology

Question 148/155

Question #148

A 69-yar-old male patient presents with lightheadedness, typical chest pain, and systolic ejection
murmur at the right second intercostal space radiating to the neck. Echocardiogram shows le
ventricular hypertrophy. What is the most likely diagnosis?

a. Mitral stenosis
b. Aortic stenosis
c. Aortic insuciency
d. Mitral regurgitation
e. Infective endocarditis

‫اﻹﺟﺎﺑﺔ ﻋﲆ اﻟﺼﻔﺤﺔ اﻟﺘﺎﻟﻴﺔ‬

Page - 322
Internal Medicine - Cardiology - Valvular diseases and heart murmurs

Question 148/155

Question #148

A 69-yar-old male patient presents with lightheadedness, typical chest pain, and systolic ejection
murmur at the right second intercostal space radiating to the neck. Echocardiogram shows le
ventricular hypertrophy. What is the most likely diagnosis?

a. Mitral stenosis
b. Aortic stenosis √
c. Aortic insuciency
d. Mitral regurgitation
e. Infective endocarditis

Description

Angina pectoris, syncope, and le ventricular hypertrophy and failure are the classic presentation of
aortic stenosis.

Systolic ejection murmur is one more piece of evidence of the presence of aortic stenosis

Page - 323
Internal Medicine - Cardiology

Question 149/155

Question #149

A 71-year-old male patient with a known case of HTN presents with exertional shortness of breath
for 3 months. On examination, a decrescendo diastolic murmur is heard at the le 3rd intercostal
space. Of the following, what is the most likely associated with this patient’s condition?

a. Narrow pulse pressure


b. Bisferiens pulse
c. Pulsus alternans
d. Wide pulse pressure
e. Pulsus paradoxus

‫اﻹﺟﺎﺑﺔ ﻋﲆ اﻟﺼﻔﺤﺔ اﻟﺘﺎﻟﻴﺔ‬

Page - 324
Internal Medicine - Cardiology - Valvular diseases and heart murmurs

Question 149/155

Question #149

A 71-year-old male patient with a known case of HTN presents with exertional shortness of breath
for 3 months. On examination, a decrescendo diastolic murmur is heard at the le 3rd intercostal
space. Of the following, what is the most likely associated with this patient’s condition?

a. Narrow pulse pressure


b. Bisferiens pulse
c. Pulsus alternans
d. Wide pulse pressure √
e. Pulsus paradoxus

Description

e diastolic murmur best heard when the patient is leaning forward at Erb’s point, located at the
3rd intercostal space on the le, is most likely to be aortic regurgitation.

Wide pulse pressure is a sign that is usually seen in aortic regurgitation. It may be seen in
hyperthyroidism as well

Page - 325
Internal Medicine - Cardiology

Question 150/155

Question #150

A 29-year-old male patient complained of recurrent intermittent short-lasting palpitations. His


physical examination reveals normal S1 and S2, displaced apex beat 1 cm laterally, and systolic
ejection murmur at the le lower sternal border with no radiation. e murmur is aggravated by the
Valsalva maneuver and reduced by hand grip. What is the most likely diagnosis?

a. Aortic stenosis
b. Hypertrophic obstructive cardiomyopathy (HOCM)
c. Ventricular septal defect
d. Pulmonary HTN
e. Aortic regurgitation

‫اﻹﺟﺎﺑﺔ ﻋﲆ اﻟﺼﻔﺤﺔ اﻟﺘﺎﻟﻴﺔ‬

Page - 326
Internal Medicine - Cardiology - Valvular diseases and heart murmurs

Question 150/155

Question #150

A 29-year-old male patient complained of recurrent intermittent short-lasting palpitations. His


physical examination reveals normal S1 and S2, displaced apex beat 1 cm laterally, and systolic
ejection murmur at the le lower sternal border with no radiation. e murmur is aggravated by the
Valsalva maneuver and reduced by hand grip. What is the most likely diagnosis?

a. Aortic stenosis
b. Hypertrophic obstructive cardiomyopathy (HOCM) √
c. Ventricular septal defect
d. Pulmonary HTN
e. Aortic regurgitation

Description

Of the choices mentioned above, a systolic ejection murmur is present in aortic stenosis and HOCM.
e age of the patient, along with the eect of the Valsalva maneuvre and hand grip on the murmur,
is suggestive of HOCM

Ventricular septal defect presents with a pan-systolic murmur, while Aortic regurgitation presents
with a decrescendo diastolic murmur (early diastolic murmur)

Page - 327
Internal Medicine - Cardiology

Question 151/155

Question #151

Based on the echocardiogram, an elderly patient is diagnosed with aortic regurgitation. Which of
the following murmurs are expected to be present in this patient?

a. Systolic ejection murmur radiated to the neck


b. Early diastolic decrescendo murmur at the le sternal border
c. Pan-systolic murmur at the 5th intercostal space radiated to the axilla
d. Continuous machine-like murmur at le infraclavicular area
e. Late systolic murmur with mid-systolic click

‫اﻹﺟﺎﺑﺔ ﻋﲆ اﻟﺼﻔﺤﺔ اﻟﺘﺎﻟﻴﺔ‬

Page - 328
Internal Medicine - Cardiology - Valvular diseases and heart murmurs

Question 151/155

Question #151

Based on the echocardiogram, an elderly patient is diagnosed with aortic regurgitation. Which of
the following murmurs are expected to be present in this patient?

a. Systolic ejection murmur radiated to the neck


b. Early diastolic decrescendo murmur at the le sternal border √
c. Pan-systolic murmur at the 5th intercostal space radiated to the axilla
d. Continuous machine-like murmur at le infraclavicular area
e. Late systolic murmur with mid-systolic click

Description

Early diastolic decrescendo murmur at the le sternal border is the characteristic murmur in aortic
regurgitations

Page - 329
Internal Medicine - Cardiology

Question 152/155

Question #152

A patient was found to have a systolic ejection murmur at the le 2nd intercostal space. e second
heart sound is split widely during expiration as it is during inspiration. e rest of the physical
examination is normal. What is the most likely diagnosis?

a. Patent ductus arteriosus


b. Aortic stenosis
c. Aortic regurgitation
d. Atrial septal defect
e. Hypertrophic Obstructive Cardiomyopathy (HOCM)

‫اﻹﺟﺎﺑﺔ ﻋﲆ اﻟﺼﻔﺤﺔ اﻟﺘﺎﻟﻴﺔ‬

Page - 330
Internal Medicine - Cardiology - Valvular diseases and heart murmurs

Question 152/155

Question #152

A patient was found to have a systolic ejection murmur at the le 2nd intercostal space. e second
heart sound is split widely during expiration as it is during inspiration. e rest of the physical
examination is normal. What is the most likely diagnosis?

a. Patent ductus arteriosus


b. Aortic stenosis
c. Aortic regurgitation
d. Atrial septal defect √
e. Hypertrophic Obstructive Cardiomyopathy (HOCM)

Description

Systolic ejection murmur and fixed splitting are features of Atrial septal defect.

Fixed splitting: the splitting in S2 is equal in both expiration and inspiration

Note that splitting during inspiration is normal

Page - 331
Internal Medicine - Cardiology

Question 153/155

Question #153

A 69-year-old male patient with recurrent exertional syncope and chest pain has systolic thrill and
systolic ejection murmur radiated to both carotids. e patient is afebrile, and his vital signs and lab
investigations are normal. What is the next step in the management of this patient?

a. Cardiac enzymes
b. Chest x-ray
c. ECG stress tests
d. Echocardiogram
e. Refer to cardiac catheterization

‫اﻹﺟﺎﺑﺔ ﻋﲆ اﻟﺼﻔﺤﺔ اﻟﺘﺎﻟﻴﺔ‬

Page - 332
Internal Medicine - Cardiology - Valvular diseases and heart murmurs

Question 153/155

Question #153

A 69-year-old male patient with recurrent exertional syncope and chest pain has systolic thrill and
systolic ejection murmur radiated to both carotids. e patient is afebrile, and his vital signs and lab
investigations are normal. What is the next step in the management of this patient?

a. Cardiac enzymes
b. Chest x-ray
c. ECG stress tests
d. Echocardiogram √
e. Refer to cardiac catheterization

Description

is patient is suspected of having aortic stenosis. e presence of exertional syncope, angina, and
systolic ejection murmur that is radiated to the neck is a classic presentation of aortic stenosis.

Echocardiogram is the best next step in the management

Page - 333
Internal Medicine - Cardiology

Question 154/155

Question #154

A 71-year-old male patient with a known case of HTN presents with exertional shortness of breath
for 3 months. On examination, a decrescendo diastolic murmur is heard at the le 3rd intercostal
space. What is the most likely diagnosis?

a. Aortic stenosis
b. Mitral regurgitation
c. Tricuspid regurgitation
d. Mitral prolapse
e. Aortic regurgitation

‫اﻹﺟﺎﺑﺔ ﻋﲆ اﻟﺼﻔﺤﺔ اﻟﺘﺎﻟﻴﺔ‬

Page - 334
Internal Medicine - Cardiology - Valvular diseases and heart murmurs

Question 154/155

Question #154

A 71-year-old male patient with a known case of HTN presents with exertional shortness of breath
for 3 months. On examination, a decrescendo diastolic murmur is heard at the le 3rd intercostal
space. What is the most likely diagnosis?

a. Aortic stenosis
b. Mitral regurgitation
c. Tricuspid regurgitation
d. Mitral prolapse
e. Aortic regurgitation √

Description

e diastolic murmur best heard when the patient is leaning forward at Erb’s point, located at the
3rd intercostal space on the le, is most likely to be aortic regurgitation

Page - 335
Internal Medicine - Cardiology

Question 155/155

Question #155

A 73-year-old female presents for evaluation because she has had syncope 3 times in the last 2
months. All are related to exertion. What is the most likely cause of her syncope?

a. Vasovagal attack
b. Transient ischemic attack
c. Aortic stenosis
d. Non-sustained arrhythmia
e. Atrial myxoma

‫اﻹﺟﺎﺑﺔ ﻋﲆ اﻟﺼﻔﺤﺔ اﻟﺘﺎﻟﻴﺔ‬

Page - 336
Internal Medicine - Cardiology - Valvular diseases and heart murmurs

Question 155/155

Question #155

A 73-year-old female presents for evaluation because she has had syncope 3 times in the last 2
months. All are related to exertion. What is the most likely cause of her syncope?

a. Vasovagal attack
b. Transient ischemic attack
c. Aortic stenosis √
d. Non-sustained arrhythmia
e. Atrial myxoma

Description

Exercise-induced syncope is usually due to fixed cardiac output, which is not rising during exercise
and high demand. is presentation is most likely to be due to aortic stenosis

Page - 337
Endocrinology

Page - 338
Endocrinology

‫ﻣﻮاﺿﻴﻊ اﻷﺳﺌﻠﺔ وأﻋﺪادﻫﺎ‬

1) Acromegaly => 3 Questions


2) Adrenal insuciency and crisis => 9 Questions
3) Counterregulatory hormones => 1 Questions
4) Cushing’s syndrome => 6 Questions
5) Diabetes Mellitus and its complications => 39 Questions
6) Disorders of ADH => 13 Questions
7) Dyslipidemia => 3 Questions
8) Familial hypocalciuric hypercalcemia (FHH) => 1 Questions
9) Hypercalcemia => 16 Questions
10) Hyperprolactinemia => 5 Questions
11) Hyperthyroidism => 16 Questions
12) Hypocalcemia => 2 Questions
13) Hypothyroidism => 14 Questions
14) Incidentaloma => 1 Questions
15) Multiple Endocrine Neoplasias (MEN) => 2 Questions
16) Non-thyroidal illness => 2 Questions
17) Parathyroid gland => 1 Questions
18) Pheochromocytoma => 6 Questions
19) Pituitary Adenoma => 3 Questions
20) Primary hyperaldosteronism => 6 Questions
21) Subacute yroiditis => 4 Questions
22) yroid storm => 1 Questions
23) Vitamin D deficiency => 1 Questions
24) Hypoglycemia => 7 Questions

Page - 339
Internal Medicine - Endocrinology

Question 1/162

Question #1

A 49-year-old male patient complains of general weakness, bad body smells, and snoring at night. In
addition, he noted that his shoe size had become larger than previously. On examination, the
patient has a deep voice, large protruded mandible, maxilla, and orbital ridges. In addition,
acanthosis nigricans is noted on his neck and axilla, overweight. What is the most likely diagnosis?

a. Pituitary adenoma
b. Hypothyroidism
c. Adrenal gland adenoma
d. Exogenous steroid use
e. Type 2 diabetes mellitus

‫اﻹﺟﺎﺑﺔ ﻋﲆ اﻟﺼﻔﺤﺔ اﻟﺘﺎﻟﻴﺔ‬

Page - 340
Internal Medicine - Endocrinology - Acromegaly

Question 1/162

Question #1

A 49-year-old male patient complains of general weakness, bad body smells, and snoring at night. In
addition, he noted that his shoe size had become larger than previously. On examination, the
patient has a deep voice, large protruded mandible, maxilla, and orbital ridges. In addition,
acanthosis nigricans is noted on his neck and axilla, overweight. What is the most likely diagnosis?

a. Pituitary adenoma √
b. Hypothyroidism
c. Adrenal gland adenoma
d. Exogenous steroid use
e. Type 2 diabetes mellitus

Description

ese are typical features of acromegaly.

Acromegaly is a Disease of adult life, characterized by the growth of bulk but not in the height of
bone due to over-secretion of GH aer puberty.

Pituitary adenoma is the most common cause - usually macroadenoma.

Acromegaly is associated with increased body dimensions, compression eects of the adenoma, and
metabolic eects of the growth hormone (including glucose intolerance, weight gain, DM, and
HTN).

According to the case, treatment is achieved by Dopamine agonists, surgery, or radiotherapy.

Page - 341
Internal Medicine - Endocrinology

Question 2/162

Question #2

Which is the most common cause of death in a patient with acromegaly?

a. Renal failure
b. Cardiovascular complications
c. Colorectal cancer
d. Diabetes mellitus
e. Osteoarthritis

‫اﻹﺟﺎﺑﺔ ﻋﲆ اﻟﺼﻔﺤﺔ اﻟﺘﺎﻟﻴﺔ‬

Page - 342
Internal Medicine - Endocrinology - Acromegaly

Question 2/162

Question #2

Which is the most common cause of death in a patient with acromegaly?

a. Renal failure
b. Cardiovascular complications √
c. Colorectal cancer
d. Diabetes mellitus
e. Osteoarthritis

Description

Acromegaly can cause several complications, including arthritis, amenorrhea in women,


cardiomegaly, hypertension, carpal tunnel syndrome, colonic polyps,

kidney failure, and diabetes. However, cardiovascular diseases are the most common cause of death
in patients with acromegaly.

Acromegaly is a Disease of adult life, characterized by the growth of bulk but not in the height of
bone due to over-secretion of GH aer puberty.

Pituitary adenoma is the most common cause - usually macroadenoma.

Acromegaly is associated with increased body dimensions, compression eects of the adenoma, and
metabolic eects of the growth hormone (including glucose intolerance, weight gain, DM, and
HTN).

According to the case, treatment is achieved by Dopamine agonists, surgery, or radiotherapy.

Page - 343
Internal Medicine - Endocrinology

Question 3/162

Question #3

A 40-year-old male patient presents with hand swelling, enlargement of his nose, and deepening of
his voice. Physical examination demonstrates above-average body dimensions and bitemporal
hemianopia. What is the most likely confirmatory test for his condition?

a. Spot serum growth hormone level


b. Serum prolactin level
c. Oral glucose tolerance with serial growth hormone measurements
d. Insulin-like growth factor level
e. Pituitary MRI

‫اﻹﺟﺎﺑﺔ ﻋﲆ اﻟﺼﻔﺤﺔ اﻟﺘﺎﻟﻴﺔ‬

Page - 344
Internal Medicine - Endocrinology - Acromegaly

Question 3/162

Question #3

A 40-year-old male patient presents with hand swelling, enlargement of his nose, and deepening of
his voice. Physical examination demonstrates above-average body dimensions and bitemporal
hemianopia. What is the most likely confirmatory test for his condition?

a. Spot serum growth hormone level


b. Serum prolactin level
c. Oral glucose tolerance with serial growth hormone measurements √
d. Insulin-like growth factor level
e. Pituitary MRI

Description

is is a case of pituitary adenoma resulting in acromegaly.

e presence of increased body dimensions and bitemporal hemianopia strongly suggests the
condition.

If elevated, the insulin-like growth factor level is the most appropriate to screen for this condition.

75g OGTT with GH measurements with serial GH measurements is most appropriate for confirming
the diagnosis.

Pituitary MRI will show the macroadenoma, but it is not the most accurate test but is used to
localize the pathology only.

Growth hormone secretion is pulsatile, so spot level is not indicated to diagnose any condition but
rather a suppression test.

Page - 345
Internal Medicine - Endocrinology

Question 4/162

Question #4

A 30-year-old male patient complains of abdominal pain, vomiting, and dizziness for 2 days. Her past
medical history is significant for rheumatoid arthritis, for which she has received steroids for the last
4 months. What is the most likely explanation for her symptoms?

a. Steroids side eects


b. Cerebrovascular accident
c. Cushing’s syndrome
d. Adrenal insuciency
e. Hyperaldosteronism

‫اﻹﺟﺎﺑﺔ ﻋﲆ اﻟﺼﻔﺤﺔ اﻟﺘﺎﻟﻴﺔ‬

Page - 346
Internal Medicine - Endocrinology - Adrenal insuciency and crisis

Question 4/162

Question #4

A 30-year-old male patient complains of abdominal pain, vomiting, and dizziness for 2 days. Her past
medical history is significant for rheumatoid arthritis, for which she has received steroids for the last
4 months. What is the most likely explanation for her symptoms?

a. Steroids side eects


b. Cerebrovascular accident
c. Cushing’s syndrome
d. Adrenal insuciency √
e. Hyperaldosteronism

Description

is clinical scenario suggests steroid cessation and adrenal insuciency.

Patients using steroids for an extended period will develop adrenal atrophy and under-secretion of
endogenous steroids. If steroids are stopped suddenly, there will be cortisol deficiency manifesting
as hypotension, hypoglycemia, hyponatremia, hyperkalemia, and a non-anion gap metabolic
acidosis.

Page - 347
Internal Medicine - Endocrinology

Question 5/162

Question #5

A 35-year-old male patient presented to the emergency department aer being found unconscious
in the street. e initial workup shows severe dehydration, hypotension, a temperature of 40 °C,
serum sodium of 122 mEq/L, serum potassium of 5.9 mEq/L, serum glucose of 45 mg/dL, and normal
ECG. What is the most appropriate initial treatment for his condition?

a. Dextrose 10% water


b. 3% hypertonic saline
c. Intravenous calcium gluconate
d. Hydrocortisone 100 mg IV
e. Intravenous antibiotics

‫اﻹﺟﺎﺑﺔ ﻋﲆ اﻟﺼﻔﺤﺔ اﻟﺘﺎﻟﻴﺔ‬

Page - 348
Internal Medicine - Endocrinology - Adrenal insuciency and crisis

Question 5/162

Question #5

A 35-year-old male patient presented to the emergency department aer being found unconscious
in the street. e initial workup shows severe dehydration, hypotension, a temperature of 40 °C,
serum sodium of 122 mEq/L, serum potassium of 5.9 mEq/L, serum glucose of 45 mg/dL, and normal
ECG. What is the most appropriate initial treatment for his condition?

a. Dextrose 10% water


b. 3% hypertonic saline
c. Intravenous calcium gluconate
d. Hydrocortisone 100 mg IV √
e. Intravenous antibiotics

Description

is patient is suering from an adrenal crisis.

Hydrocortisone is the first-line treatment.

Calcium gluconate is the first line in hyperkalemia, but it is indicated if the serum potassium is more
than 6.5 mEq/L or if there are ECG changes.

Adrenal crisis is a medical emergency caused by sudden and marked insuciency of adrenocortical
hormones

e precipitating factors are stress (trauma, infection, bleeding, surgery, hypotension) or sudden
chronic high-dose steroid therapy withdrawal.

Clinical features:

Dehydration, headache, confusion, coma


Hypotension (shock)
Hypoglycemia
Nausea, vomiting, diarrhea, abdominal pain
Hyponatremia, hyperkalemia
Page - 349
High urea

Treatment:

Hydrocortisone injection (first step)


IV fluids
Treatment of hypoglycemia
Antibiotics
Shock management

Page - 350
Internal Medicine - Endocrinology

Question 6/162

Question #6

A 60-year-old male patient complains of abdominal pain and dizziness. Your evaluation
demonstrates orthostatic hypotension, hyperkalemia, and hyponatremia. Which is the best initial
test to rule out adrenal insuciency?

a. Morning serum cortisol


b. Cosyntropin stimulation test
c. Adrenal MRI
d. Growth hormone suppression test
e. Glucose tolerance test

‫اﻹﺟﺎﺑﺔ ﻋﲆ اﻟﺼﻔﺤﺔ اﻟﺘﺎﻟﻴﺔ‬

Page - 351
Internal Medicine - Endocrinology - Adrenal insuciency and crisis

Question 6/162

Question #6

A 60-year-old male patient complains of abdominal pain and dizziness. Your evaluation
demonstrates orthostatic hypotension, hyperkalemia, and hyponatremia. Which is the best initial
test to rule out adrenal insuciency?

a. Morning serum cortisol √


b. Cosyntropin stimulation test
c. Adrenal MRI
d. Growth hormone suppression test
e. Glucose tolerance test

Description

A single morning cortisone level of more than 13 g/dL is a reliable and sensitive finding to rule out
adrenal insuciency.

A Cosyntropin stimulation test is used to confirm the diagnosis if the serum morning cortisol level is
reduced.

Page - 352
Internal Medicine - Endocrinology

Question 7/162

Question #7

A 40-year-old male patient presents to the ER with nausea, vomiting, poor appetite, fatigue, and
dizziness. Physical examination demonstrates blood pressure of 139/82 mmHg supine and 105/63
mmHg standing. His serum electrolytes show serum sodium of 122 mEq/L, potassium of 5.9 mEq/L,
urea of 8 mmol/L, and creatinine of 90 mmol/L. What is the most common pathology of his
condition?

a. Infection
b. Infarction
c. Autoimmune
d. Protein infiltration
e. Metastasis

‫اﻹﺟﺎﺑﺔ ﻋﲆ اﻟﺼﻔﺤﺔ اﻟﺘﺎﻟﻴﺔ‬

Page - 353
Internal Medicine - Endocrinology - Adrenal insuciency and crisis

Question 7/162

Question #7

A 40-year-old male patient presents to the ER with nausea, vomiting, poor appetite, fatigue, and
dizziness. Physical examination demonstrates blood pressure of 139/82 mmHg supine and 105/63
mmHg standing. His serum electrolytes show serum sodium of 122 mEq/L, potassium of 5.9 mEq/L,
urea of 8 mmol/L, and creatinine of 90 mmol/L. What is the most common pathology of his
condition?

a. Infection
b. Infarction
c. Autoimmune √
d. Protein infiltration
e. Metastasis

Description

Hypoadrenalism, Addison’s disease, or chronic primary adrenal insuciency, is characterized by


reduced cortisol, resulting in specific metabolic features.

In 80% of cases, it is caused by an autoimmune process leading to adrenal gland destruction and
malfunction.

Clinical features include the following:

Weakness, weight loss, fatigue, vomiting, fever, constipation, abdominal pain, impotence,
syncope, postural hypotension, loss of axillary hair
Hyperpigmentation of skin (due to increased MSH), Not present in secondary adrenal
insuciency
Hypotension
Vitiligo (autoimmune process) is associated with this condition; autoimmune conditions are
associated with each other.

e main metabolic problems are:

Hyperuricemia, hyperkalemia, hypercalcemia


Hyponatremia, hypoglycemia

Page - 354
Investigations:

CT abdomen to assess the adrenal glands


Serum cortisol is the best initial test (sensitive test)
Cosyntropin stimulation test (Cosyntropin is a synthetic analog of ACTH) → (most specific test)
Anti-adrenal antibodies in 50% of cases
Low serum aldosterone

Treatment: steroid hormone replacement

Page - 355
Internal Medicine - Endocrinology

Question 8/162

Question #8

A 33-year-old male patient was admitted yesterday to the surgical department for observation aer
sustaining a road trac accident. Today the patient develops blood pressure of 90/40 mmHg and a
heart rate of 140 bpm, but no source of bleeding is detected. His lab investigations demonstrate
hemoglobin of 15.0 g/dL (within baseline), serum sodium of 121 mEq/L, and serum potassium of 7
mEq/L. What is the most appropriate treatment at this time?

a. Intravenous ringer lactate


b. Start blood transfusion
c. Order abdominal CT scan
d. Start intravenous beta-blockers
e. Give hydrocortisone 100 mg intravenously

‫اﻹﺟﺎﺑﺔ ﻋﲆ اﻟﺼﻔﺤﺔ اﻟﺘﺎﻟﻴﺔ‬

Page - 356
Internal Medicine - Endocrinology - Adrenal insuciency and crisis

Question 8/162

Question #8

A 33-year-old male patient was admitted yesterday to the surgical department for observation aer
sustaining a road trac accident. Today the patient develops blood pressure of 90/40 mmHg and a
heart rate of 140 bpm, but no source of bleeding is detected. His lab investigations demonstrate
hemoglobin of 15.0 g/dL (within baseline), serum sodium of 121 mEq/L, and serum potassium of 7
mEq/L. What is the most appropriate treatment at this time?

a. Intravenous ringer lactate


b. Start blood transfusion
c. Order abdominal CT scan
d. Start intravenous beta-blockers
e. Give hydrocortisone 100 mg intravenously √

Description

is patient is suering from an adrenal crisis. erefore, hydrocortisone is the first-line treatment.

Adrenal crisis is a medical emergency caused by sudden and marked insuciency of adrenocortical
hormones

e precipitating factors are stress (trauma, infection, bleeding, surgery, hypotension) or sudden
chronic high-dose steroid therapy withdrawal.

Clinical features:

Dehydration, headache, confusion, coma


Hypotension (shock)
Hypoglycemia
Nausea, vomiting, diarrhea, abdominal pain
Hyponatremia, hyperkalemia
High urea

Treatment:

Hydrocortisone injection (first step)


IV fluids
Page - 357
Treatment of hypoglycemia
Antibiotics
Shock management

Page - 358
Internal Medicine - Endocrinology

Question 9/162

Question #9

A 55-year-old male patient complains of dizziness. Aer an appropriate workup, you diagnose him
with adrenal insuciency. Which of the following are the most likely expected abnormalities in this
patient?

a. Hyperkalemia, hypernatremia
b. Hypokalemia, hypernatremia
c. Hypokalemia, hyponatremia
d. Normal potassium, hyponatremia
e. Hyperkalemia, hyponatremia

‫اﻹﺟﺎﺑﺔ ﻋﲆ اﻟﺼﻔﺤﺔ اﻟﺘﺎﻟﻴﺔ‬

Page - 359
Internal Medicine - Endocrinology - Adrenal insuciency and crisis

Question 9/162

Question #9

A 55-year-old male patient complains of dizziness. Aer an appropriate workup, you diagnose him
with adrenal insuciency. Which of the following are the most likely expected abnormalities in this
patient?

a. Hyperkalemia, hypernatremia
b. Hypokalemia, hypernatremia
c. Hypokalemia, hyponatremia
d. Normal potassium, hyponatremia
e. Hyperkalemia, hyponatremia √

Description

Adrenal insuciency is defined as a reduced cortisol serum level. It may be caused by a primary
pathology in the adrenal gland, sudden steroid withdrawal, or reduced ACTH secretion (secondary
adrenal insuciency).

Adrenal insuciency presents with hyperkalemia, hyponatremia, hypoglycemia, hypotension, and


non-anion gap metabolic acidosis

Page - 360
Internal Medicine - Endocrinology

Question 10/162

Question #10

A 40-year-old female patient presents with abdominal pain and diarrhea. Her physical examination
shows a blood pressure of 80/49 mmHg, pulse rate of 80 bpm, and temperature of 36.8 °C. in
addition, you noted hyperpigmented skin at the palmar creases and the buccal mucosa. What is the
most likely diagnosis?

a. Addison’s disease
b. Cushing’s syndrome
c. Pheochromocytoma
d. Hypothyroidism
e. Hypoparathyroidism

‫اﻹﺟﺎﺑﺔ ﻋﲆ اﻟﺼﻔﺤﺔ اﻟﺘﺎﻟﻴﺔ‬

Page - 361
Internal Medicine - Endocrinology - Adrenal insuciency and crisis

Question 10/162

Question #10

A 40-year-old female patient presents with abdominal pain and diarrhea. Her physical examination
shows a blood pressure of 80/49 mmHg, pulse rate of 80 bpm, and temperature of 36.8 °C. in
addition, you noted hyperpigmented skin at the palmar creases and the buccal mucosa. What is the
most likely diagnosis?

a. Addison’s disease √
b. Cushing’s syndrome
c. Pheochromocytoma
d. Hypothyroidism
e. Hypoparathyroidism

Description

Hypoadrenalism, Addison’s disease, or chronic primary adrenal insuciency, is characterized by


reduced cortisol, resulting in specific metabolic features.

In 80% of cases, it is caused by an autoimmune process leading to adrenal gland destruction and
malfunction.

Clinical features include the following:

Weakness, weight loss, fatigue, vomiting, fever, constipation, abdominal pain, impotence,
syncope, postural hypotension, loss of axillary hair
Hyperpigmentation of skin (due to increased MSH), Not present in secondary adrenal
insuciency
Hypotension
Vitiligo (autoimmune process) is associated with this condition; autoimmune conditions are
associated with each other.

e main metabolic problems are:

Hyperuricemia, hyperkalemia, hypercalcemia


Hyponatremia, hypoglycemia

Investigations:
Page - 362
CT abdomen to assess the adrenal glands
Serum cortisol is the best initial test (sensitive test)
Cosyntropin stimulation test (Cosyntropin is a synthetic analog of ACTH) → (most specific test)
Anti-adrenal antibodies in 50% of cases
Low serum aldosterone

Treatment: steroid hormone replacement

Page - 363
Internal Medicine - Endocrinology

Question 11/162

Question #11

A 60-year-old male patient is a known chronic steroid user due to COPD asthma overlap syndrome.
He presents to you with a 3 days history of dizziness. His physical examination and lab
investigations show hypotension, hyperkalemia, and hyponatremia. What is the most likely
diagnosis?

a. Steroid side eect


b. Cushing’s syndrome
c. Hyperaldosteronism
d. Adrenal insuciency
e. Hypoxic brain injury

‫اﻹﺟﺎﺑﺔ ﻋﲆ اﻟﺼﻔﺤﺔ اﻟﺘﺎﻟﻴﺔ‬

Page - 364
Internal Medicine - Endocrinology - Adrenal insuciency and crisis

Question 11/162

Question #11

A 60-year-old male patient is a known chronic steroid user due to COPD asthma overlap syndrome.
He presents to you with a 3 days history of dizziness. His physical examination and lab
investigations show hypotension, hyperkalemia, and hyponatremia. What is the most likely
diagnosis?

a. Steroid side eect


b. Cushing’s syndrome
c. Hyperaldosteronism
d. Adrenal insuciency √
e. Hypoxic brain injury

Description

is clinical scenario suggests steroid cessation and adrenal insuciency.

Patients using steroids for an extended period will develop adrenal atrophy and under-secretion of
endogenous steroids. If steroids are stopped suddenly, there will be cortisol deficiency manifesting
as hypotension, hypoglycemia, hyponatremia, hyperkalemia, and a non-anion gap metabolic
acidosis.

Page - 365
Internal Medicine - Endocrinology

Question 12/162

Question #12

A 36-year-old female patient presents to the emergency department with severe distress. However,
his history is only significant for chronic fatigue for 6 months. Physical exam demonstrates
orthostatic hypotension and hyperpigmentation of the knuckles, palmar creases, and abdominal
scar. Laboratory investigations show hyponatremia and hyperkalemia. What is the most appropriate
test to confirm the diagnosis?

a. Serum cortisol level


b. Cosyntropin Stimulation test
c. 17-hydroxyprogesterone
d. Metanephrine level in 24-hour urine collection.
e. Low-dose dexamethasone suppression test

‫اﻹﺟﺎﺑﺔ ﻋﲆ اﻟﺼﻔﺤﺔ اﻟﺘﺎﻟﻴﺔ‬

Page - 366
Internal Medicine - Endocrinology - Adrenal insuciency and crisis

Question 12/162

Question #12

A 36-year-old female patient presents to the emergency department with severe distress. However,
his history is only significant for chronic fatigue for 6 months. Physical exam demonstrates
orthostatic hypotension and hyperpigmentation of the knuckles, palmar creases, and abdominal
scar. Laboratory investigations show hyponatremia and hyperkalemia. What is the most appropriate
test to confirm the diagnosis?

a. Serum cortisol level


b. Cosyntropin Stimulation test √
c. 17-hydroxyprogesterone
d. Metanephrine level in 24-hour urine collection.
e. Low-dose dexamethasone suppression test

Description

Hypoadrenalism, Addison’s disease, or chronic primary adrenal insuciency, is characterized by


reduced cortisol, resulting in specific metabolic features.

In 80% of cases, it is caused by an autoimmune process leading to adrenal gland destruction and
malfunction.

Clinical features include the following:

Weakness, weight loss, fatigue, vomiting, fever, constipation, abdominal pain, impotence,
syncope, postural hypotension, loss of axillary hair
Hyperpigmentation of skin (due to increased MSH), Not present in secondary adrenal
insuciency
Hypotension
Vitiligo (autoimmune process) is associated with this condition; autoimmune conditions are
associated with each other.

e main metabolic problems are:

Hyperuricemia, hyperkalemia, hypercalcemia


Hyponatremia, hypoglycemia

Page - 367
Investigations:

CT abdomen to assess the adrenal glands


Serum cortisol is the best initial test (sensitive test)
Cosyntropin stimulation test (Cosyntropin is a synthetic analog of ACTH) → (most specific test)
Anti-adrenal antibodies in 50% of cases
Low serum aldosterone

Treatment: steroid hormone replacement

Low dose dexamethasone suppression test is used in Cushing’s syndrome workup

17-hydroxyprogesterone is used in adrenal hyperplasia workup

Metanephrine level is an indicator of pheochromocytoma

Page - 368
Internal Medicine - Endocrinology

Question 13/162

Question #13

e following are considered stress hormones that participate in glucose Counterregulation except:

a. Growth hormone (GH)


b. Glucagon
c. Cortisol
d. Epinephrine and norepinephrine
e. yroid-stimulating hormone (TSH)

‫اﻹﺟﺎﺑﺔ ﻋﲆ اﻟﺼﻔﺤﺔ اﻟﺘﺎﻟﻴﺔ‬

Page - 369
Internal Medicine - Endocrinology - Counterregulatory hormones

Question 13/162

Question #13

e following are considered stress hormones that participate in glucose Counterregulation except:

a. Growth hormone (GH)


b. Glucagon
c. Cortisol
d. Epinephrine and norepinephrine
e. yroid-stimulating hormone (TSH) √

Description

Description:

TSH is not a stress hormone and does not aect blood sugar.

e action of insulin is counter-regulated by

Glucagon
Epinephrine (adrenaline)
Norepinephrine (noradrenaline)
Cortisol
Growth hormone

ese counterregulatory hormones raise the level of glucose in the blood by promoting
glycogenolysis, gluconeogenesis, ketogenesis, and other catabolic processes.

Page - 370
Internal Medicine - Endocrinology

Question 14/162

Question #14

A 50-year-old female is diagnosed with HTN, hirsutism, and acne. She is referred to you by her family
physician to evaluate her laboratory findings. Her lab investigations demonstrate elevated serum
cortisol, normal ACTH, and aldosterone. What is the most likely diagnosis?

a. Cushing’s syndrome
b. Bartter’s syndrome
c. Conn’s syndrome
d. Liddle’s syndrome
e. Addison’s disease

‫اﻹﺟﺎﺑﺔ ﻋﲆ اﻟﺼﻔﺤﺔ اﻟﺘﺎﻟﻴﺔ‬

Page - 371
Internal Medicine - Endocrinology - Cushing’s syndrome

Question 14/162

Question #14

A 50-year-old female is diagnosed with HTN, hirsutism, and acne. She is referred to you by her family
physician to evaluate her laboratory findings. Her lab investigations demonstrate elevated serum
cortisol, normal ACTH, and aldosterone. What is the most likely diagnosis?

a. Cushing’s syndrome √
b. Bartter’s syndrome
c. Conn’s syndrome
d. Liddle’s syndrome
e. Addison’s disease

Description

Cushing’s syndrome results from increased secretion of cortisone by adrenal glands

Clinical features of Cushing’s syndrome:

Hypertension, hyperglycemia, hypokalemia


Fat redistribution: Moon face, weight gain / central obesity, bualo hump, thin extremities
Skin: inning of skin, abdominal striae, decreased wound healing, easy bruising, hirsutism,
acne, skin pigmentation
Muscle weakness (proximal muscle)
Sexual disturbances, edema, amenorrhea
Psychiatric symptoms (e.g., depression)

Page - 372
Internal Medicine - Endocrinology

Question 15/162

Question #15

A 30-year-old male patient complains of hirsutism, truncal obesity, easy bruising, and
hyperglycemia. His blood pressure is found to be 151/89 mmHg. What is the most helpful test in
localizing the cause of Cushing’s syndrome?

a. Urinary cortisol level


b. Serum cortisol level
c. Low-dose dexamethasone suppression test
d. High-dose dexamethasone suppression test
e. Plasma ACTH level

‫اﻹﺟﺎﺑﺔ ﻋﲆ اﻟﺼﻔﺤﺔ اﻟﺘﺎﻟﻴﺔ‬

Page - 373
Internal Medicine - Endocrinology - Cushing’s syndrome

Question 15/162

Question #15

A 30-year-old male patient complains of hirsutism, truncal obesity, easy bruising, and
hyperglycemia. His blood pressure is found to be 151/89 mmHg. What is the most helpful test in
localizing the cause of Cushing’s syndrome?

a. Urinary cortisol level


b. Serum cortisol level
c. Low-dose dexamethasone suppression test
d. High-dose dexamethasone suppression test √
e. Plasma ACTH level

Description

A high-dose dexamethasone suppression test to dierentiate between a pituitary adenoma and an


ectopic source that is producing ACTH

Diagnosis of Cushing’s syndrome

Low dose (1 mg) dexamethasone suppression test


ACTH levels
High dose (8 mg) dexamethasone suppression test
CT or MRI to localize the pathology
Petrosal sinus sampling to confirm Cushing’s disease when microadenoma does not show in
imaging
Other labs: hyperglycemia, hyperlipidemia, hypokalemia, metabolic alkalosis

Page - 374
Page - 375
Internal Medicine - Endocrinology

Question 16/162

Question #16

A 49-year-old male patient complains of proximal muscle weakness and tiredness over the past 6
months. she was recently diagnosed with HTN and is suspected of having Cushing’s disease. Which
of the following is not found in Cushing’s disease?

a. Failure to suppress morning cortisol with low-dose dexamethasone


b. High serum ACTH level
c. Skin pigmentation
d. Bilateral adrenal hypertrophy
e. Failure to suppress ACTH by high dose dexamethasone

‫اﻹﺟﺎﺑﺔ ﻋﲆ اﻟﺼﻔﺤﺔ اﻟﺘﺎﻟﻴﺔ‬

Page - 376
Internal Medicine - Endocrinology - Cushing’s syndrome

Question 16/162

Question #16

A 49-year-old male patient complains of proximal muscle weakness and tiredness over the past 6
months. she was recently diagnosed with HTN and is suspected of having Cushing’s disease. Which
of the following is not found in Cushing’s disease?

a. Failure to suppress morning cortisol with low-dose dexamethasone


b. High serum ACTH level
c. Skin pigmentation
d. Bilateral adrenal hypertrophy
e. Failure to suppress ACTH by high dose dexamethasone √

Description

Cushing’s disease diers from Cushing’s syndrome

Cushing’s disease is a disease caused by ACTH-secreting pituitary adenoma

is will lead to a high cortisol level that fails to suppress with a low dose dexamethasone
suppression test and heavy adrenal workload leading to hypertrophy of both adrenal glands.

High-dose dexamethasone suppression test will fail to suppress ACTH in ectopic ACTH secretion.

e following diagram describes the approach to the diagnosis of Cushing’s syndrome:

Page - 377
Page - 378
Internal Medicine - Endocrinology

Question 17/162

Question #17

A 25-year-old female patient presents with secondary amenorrhea for 10 months, feeling weak and
lethargic. On examination, dark skin pigmentation is noted, and her BMI is 36 kg/m2. Lab
investigations demonstrate hyperglycemia and hypokalemia. What is the most likely diagnosis?

a. Hypothyroidism
b. Cushing’s syndrome
c. Hemochromatosis
d. Polycystic ovarian syndrome
e. Hyperaldosteronism

‫اﻹﺟﺎﺑﺔ ﻋﲆ اﻟﺼﻔﺤﺔ اﻟﺘﺎﻟﻴﺔ‬

Page - 379
Internal Medicine - Endocrinology - Cushing’s syndrome

Question 17/162

Question #17

A 25-year-old female patient presents with secondary amenorrhea for 10 months, feeling weak and
lethargic. On examination, dark skin pigmentation is noted, and her BMI is 36 kg/m2. Lab
investigations demonstrate hyperglycemia and hypokalemia. What is the most likely diagnosis?

a. Hypothyroidism
b. Cushing’s syndrome √
c. Hemochromatosis
d. Polycystic ovarian syndrome
e. Hyperaldosteronism

Description

e clinical scenario fits with the diagnosis of Cushing’s syndrome.

Hypercortisolism (Cushing’s syndrome) results from increased secretion of cortisone by adrenal


glands

Cushing’s disease is a term used for pituitary overproduction of the ACTH hormone

Clinical features of Cushing’s syndrome:

Hypertension, hyperglycemia, hypokalemia


Fat redistribution: Moon face, weight gain / central obesity, bualo hump, thin extremities
Skin: inning of skin, abdominal striae, decreased wound healing, easy bruising, hirsutism,
acne, skin pigmentation
Muscle weakness (proximal muscle)
Sexual disturbances, edema, amenorrhea
Psychiatric symptoms (e.g., depression)

Page - 380
Internal Medicine - Endocrinology

Question 18/162

Question #18

A 33-year-old female is suspected of having Cushing’s syndrome. Which of the following is the least
likely to present in this patient?

a. Truncal obesity
b. Abdominal striation
c. Proximal muscle weakness
d. Hyperkalemia
e. Depression

‫اﻹﺟﺎﺑﺔ ﻋﲆ اﻟﺼﻔﺤﺔ اﻟﺘﺎﻟﻴﺔ‬

Page - 381
Internal Medicine - Endocrinology - Cushing’s syndrome

Question 18/162

Question #18

A 33-year-old female is suspected of having Cushing’s syndrome. Which of the following is the least
likely to present in this patient?

a. Truncal obesity
b. Abdominal striation
c. Proximal muscle weakness
d. Hyperkalemia √
e. Depression

Description

Description:

Cushing’s syndrome is associated with hypokalemia, not hyperkalemia

Cushing’s syndrome results from increased secretion of cortisone by adrenal glands

Clinical features of Cushing’s syndrome:

Hypertension, hyperglycemia, hypokalemia


Fat redistribution: Moon face, weight gain / central obesity, bualo hump, thin extremities
Skin: inning of skin, abdominal striae, decreased wound healing, easy bruising, hirsutism,
acne, skin pigmentation
Muscle weakness (proximal muscle)
Sexual disturbances, edema, amenorrhea
Psychiatric symptoms (e.g., depression)

Page - 382
Internal Medicine - Endocrinology

Question 19/162

Question #19

A patient is suspected of having Cushing’s syndrome. Which of the following is the test of choice to
confirm the diagnosis?

a. Abdominal CT scan
b. Adrenal MRI
c. Dexamethasone suppression test
d. Urinary cortisol level
e. Serum cortisol level

‫اﻹﺟﺎﺑﺔ ﻋﲆ اﻟﺼﻔﺤﺔ اﻟﺘﺎﻟﻴﺔ‬

Page - 383
Internal Medicine - Endocrinology - Cushing’s syndrome

Question 19/162

Question #19

A patient is suspected of having Cushing’s syndrome. Which of the following is the test of choice to
confirm the diagnosis?

a. Abdominal CT scan
b. Adrenal MRI
c. Dexamethasone suppression test √
d. Urinary cortisol level
e. Serum cortisol level

Description

e dexamethasone suppression test confirms the diagnosis by administering 1 mg of


dexamethasone around 11 pm and measuring plasma cortisol at 8 am the next morning.

Diagnosis of Cushing’s syndrome

Low dose (1 mg) dexamethasone suppression test


ACTH levels
High dose (8 mg) dexamethasone suppression test
CT or MRI to localize the pathology
Petrosal sinus sampling to confirm Cushing’s disease when microadenoma does not show in
imaging
Other labs: hyperglycemia, hyperlipidemia, hypokalemia, metabolic alkalosis

Page - 384
Page - 385
Internal Medicine - Endocrinology

Question 20/162

Question #20

A patient was admitted to the ICU with a case of sepsis, and he was critically ill. Which of the
following is the most appropriate blood glucose target to achieve?

a. 70 – 90 mg/dL
b. 90 – 126 mg/dL
c. 120 – 140 mg/dL
d. 140 – 180 mg/dL
e. 180 – 220 mg/dL

‫اﻹﺟﺎﺑﺔ ﻋﲆ اﻟﺼﻔﺤﺔ اﻟﺘﺎﻟﻴﺔ‬

Page - 386
Internal Medicine - Endocrinology - Diabetes Mellitus and its complications

Question 20/162

Question #20

A patient was admitted to the ICU with a case of sepsis, and he was critically ill. Which of the
following is the most appropriate blood glucose target to achieve?

a. 70 – 90 mg/dL
b. 90 – 126 mg/dL
c. 120 – 140 mg/dL
d. 140 – 180 mg/dL √
e. 180 – 220 mg/dL

Description

In critically ill patients, to avoid marked hyperglycemia and minimize the risk of hypoglycemia, it is
recommended to achieve the blood glucose target of 140 – 180 mg/dL

Page - 387
Internal Medicine - Endocrinology

Question 21/162

Question #21

A 50-year-old male patient was admitted to the hospital with upper GI bleeding. He was NPO for 12
hours aer admission, and then investigations demonstrated the following: upper endoscopy
showed gastritis, lab investigations showed a glucose level of 170 mg/dL, and hemoglobin level of 14
g/dL. en, 6 weeks later, he presents for follow-up and is found to have a fasting glucose level of 88
mg/dL and normal oral GTT. What is the most common cause of his hyperglycemia while admitted
to the hospital?

a. Type 1 DM
b. Type 2 Dm
c. Stress hyperglycemia
d. Impaired glucose tolerance
e. Impaired fasting glucose

‫اﻹﺟﺎﺑﺔ ﻋﲆ اﻟﺼﻔﺤﺔ اﻟﺘﺎﻟﻴﺔ‬

Page - 388
Internal Medicine - Endocrinology - Diabetes Mellitus and its complications

Question 21/162

Question #21

A 50-year-old male patient was admitted to the hospital with upper GI bleeding. He was NPO for 12
hours aer admission, and then investigations demonstrated the following: upper endoscopy
showed gastritis, lab investigations showed a glucose level of 170 mg/dL, and hemoglobin level of 14
g/dL. en, 6 weeks later, he presents for follow-up and is found to have a fasting glucose level of 88
mg/dL and normal oral GTT. What is the most common cause of his hyperglycemia while admitted
to the hospital?

a. Type 1 DM
b. Type 2 Dm
c. Stress hyperglycemia √
d. Impaired glucose tolerance
e. Impaired fasting glucose

Description

Remember that not any hyperglycemia is a DM.

Stress hyperglycemia is a transient elevation of blood sugar due to stress. is condition
spontaneously resolved within several hours.

e pathogenesis is related to glucose counterregulation and the elevation of the stress hormones
during stress or acute illnesses.

Patients with pneumonia, myocardial infarction, stroke, burns, or even postoperative state are at
risk of stress hyperglycemia.

Stress hyperglycemia needs no treatment; you should treat the main illness.

Page - 389
Internal Medicine - Endocrinology

Question 22/162

Question #22

A 50-year-old male patient complains of feeling tired and frequent urination for the past month. On
examination, a deep painless ulcer is noted on his right heel. In addition, the patient complains of
unexplained weight loss during the past month. What is the most appropriate initial investigation?

a. Lower limb arteriography


b. Lower limb venography
c. Fasting blood sugar
d. Biopsy from the base of the ulcer
e. Swap culture from the ulcer base

‫اﻹﺟﺎﺑﺔ ﻋﲆ اﻟﺼﻔﺤﺔ اﻟﺘﺎﻟﻴﺔ‬

Page - 390
Internal Medicine - Endocrinology - Diabetes Mellitus and its complications

Question 22/162

Question #22

A 50-year-old male patient complains of feeling tired and frequent urination for the past month. On
examination, a deep painless ulcer is noted on his right heel. In addition, the patient complains of
unexplained weight loss during the past month. What is the most appropriate initial investigation?

a. Lower limb arteriography


b. Lower limb venography
c. Fasting blood sugar √
d. Biopsy from the base of the ulcer
e. Swap culture from the ulcer base

Description

is clinical scenario strongly suggests type 2 diabetes.

Polyuria, chronic fatigue, and lower limb ulcer are characteristic

the best initial investigation will be a fasting blood glucose level which, if elevated on two separate
occasions, is diagnostic of DM

Page - 391
Internal Medicine - Endocrinology

Question 23/162

Question #23

A 55-year-old male patient presents to the ER aer sustaining burns on his right leg while taking a
bath. He stated that he could not tell whether the water was hot enough and had recently started to
suer from pins and needles in his hands. His past history is significant for type 2 DM for the past 10
years. His physical examination demonstrates decreased touch sensation bilaterally, the power is
intact, but the tendon reflexes are reduced bilaterally. What is the most likely cause of his
symptoms?

a. Ischemic stroke
b. Multiple sclerosis
c. Spinal cord compression
d. Guillain-Barre syndrome
e. Diabetic neuropathy

‫اﻹﺟﺎﺑﺔ ﻋﲆ اﻟﺼﻔﺤﺔ اﻟﺘﺎﻟﻴﺔ‬

Page - 392
Internal Medicine - Endocrinology - Diabetes Mellitus and its complications

Question 23/162

Question #23

A 55-year-old male patient presents to the ER aer sustaining burns on his right leg while taking a
bath. He stated that he could not tell whether the water was hot enough and had recently started to
suer from pins and needles in his hands. His past history is significant for type 2 DM for the past 10
years. His physical examination demonstrates decreased touch sensation bilaterally, the power is
intact, but the tendon reflexes are reduced bilaterally. What is the most likely cause of his
symptoms?

a. Ischemic stroke
b. Multiple sclerosis
c. Spinal cord compression
d. Guillain-Barre syndrome
e. Diabetic neuropathy √

Description

is patient has physical findings of lower motor neuron damage due to diabetes.

Ischemic CVA will demonstrate unilateral symptoms with hyperreflexia

Guillain-Barre Syndrome is characterized by ascending paralysis and paresthesia, which is not the
case here.

Multiple sclerosis is diagnosed in the presence of neurological deficits that are disseminated in time
and place. Furthermore, it is not common in old age.

Page - 393
Internal Medicine - Endocrinology

Question 24/162

Question #24

An 18-year-old lady complains of dizziness, deep breathing, and abdominal pain. Her medical history
is significant for type 1 DM, for which she receives insulin. What is the best next step in the
management of this patient?

a. Serum urea and creatinine


b. Blood culture and sensitivity
c. Abdominal ultrasound
d. HbA1c
e. Blood sugar and arterial blood gas

‫اﻹﺟﺎﺑﺔ ﻋﲆ اﻟﺼﻔﺤﺔ اﻟﺘﺎﻟﻴﺔ‬

Page - 394
Internal Medicine - Endocrinology - Diabetes Mellitus and its complications

Question 24/162

Question #24

An 18-year-old lady complains of dizziness, deep breathing, and abdominal pain. Her medical history
is significant for type 1 DM, for which she receives insulin. What is the best next step in the
management of this patient?

a. Serum urea and creatinine


b. Blood culture and sensitivity
c. Abdominal ultrasound
d. HbA1c
e. Blood sugar and arterial blood gas √

Description

Page - 395
Internal Medicine - Endocrinology

Question 25/162

Question #25

A 40-year-old male patient presented to your oce for a review of his lab investigations done
yesterday. His fasting glucose is 109 mg/dL and his 2h 75 g OGTT is 130 mg/dL. What is the most
likely diagnosis?

a. Impaired fasting glucose


b. Impaired glucose tolerance
c. Clinical Diabetes
d. Maturity onset diabetes of youth
e. Normal readings

‫اﻹﺟﺎﺑﺔ ﻋﲆ اﻟﺼﻔﺤﺔ اﻟﺘﺎﻟﻴﺔ‬

Page - 396
Internal Medicine - Endocrinology - Diabetes Mellitus and its complications

Question 25/162

Question #25

A 40-year-old male patient presented to your oce for a review of his lab investigations done
yesterday. His fasting glucose is 109 mg/dL and his 2h 75 g OGTT is 130 mg/dL. What is the most
likely diagnosis?

a. Impaired fasting glucose √


b. Impaired glucose tolerance
c. Clinical Diabetes
d. Maturity onset diabetes of youth
e. Normal readings

Description

- Impaired fasting glucose (IFG): FBS 90-125 mg/dl

- Impaired glucose tolerance (IGT): 2h 75 g OGTT140-199 mg/dl

Both impaired fasting glucose and impaired glucose tolerance are considered prediabetes. However,
they are not a disease but rather a risk category.

Clinical diabetes is diagnosed by the following criteria:

Diagnosis of DM: any one of the following is diagnostic:

Symptoms of DM + RBS ≥ 200 mg/dl or,


One of the following On at least two separate occasions:

a. FBS ≥ 126 mg/dl

b. 2h 75 g OGTT ≥ 200 mg/dl

c. RBS ≥ 200 mg/dl

d. HbA1c ≥ 6.5%

Page - 397
Internal Medicine - Endocrinology

Question 26/162

Question #26

A 32-year-old female patient is found to have a fasting blood glucose of 155 mg/dL during her
routine examination. e patient is asymptomatic, previously healthy, and takes no medications.
What is the most appropriate next step in the management of this patient?

a. Start insulin therapy


b. Start oral hypoglycemic agents
c. Only lifestyle modification
d. Repeat the fasting blood glucose
e. Test for diabetic complications

‫اﻹﺟﺎﺑﺔ ﻋﲆ اﻟﺼﻔﺤﺔ اﻟﺘﺎﻟﻴﺔ‬

Page - 398
Internal Medicine - Endocrinology - Diabetes Mellitus and its complications

Question 26/162

Question #26

A 32-year-old female patient is found to have a fasting blood glucose of 155 mg/dL during her
routine examination. e patient is asymptomatic, previously healthy, and takes no medications.
What is the most appropriate next step in the management of this patient?

a. Start insulin therapy


b. Start oral hypoglycemic agents
c. Only lifestyle modification
d. Repeat the fasting blood glucose √
e. Test for diabetic complications

Description

Diagnosis of DM: any one of the following is diagnostic:

Symptoms of DM + RBS ≥ 200 mg/dl or,

One of the following On at least two separate occasions:

FBS ≥ 126 mg/dl


2h 75 g OGTT ≥ 200 mg/dl
RBS ≥ 200 mg/dl
HbA1c ≥ 6.5%

Note that asymptomatic patients with one fasting blood glucose reading need another reading to
confirm DM.

Page - 399
Internal Medicine - Endocrinology

Question 27/162

Question #27

A diabetic 66-year-old male patient has pain and pins and needle sensation in his hands and feet.
You diagnose him with diabetic nephropathy. Besides proper diabetic control, which drugs can be
used to treat his symptoms?

a. Metformin
b. Metoclopramide
c. Pregabalin
d. Folic acid and B12
e. Statin

‫اﻹﺟﺎﺑﺔ ﻋﲆ اﻟﺼﻔﺤﺔ اﻟﺘﺎﻟﻴﺔ‬

Page - 400
Internal Medicine - Endocrinology - Diabetes Mellitus and its complications

Question 27/162

Question #27

A diabetic 66-year-old male patient has pain and pins and needle sensation in his hands and feet.
You diagnose him with diabetic nephropathy. Besides proper diabetic control, which drugs can be
used to treat his symptoms?

a. Metformin
b. Metoclopramide
c. Pregabalin √
d. Folic acid and B12
e. Statin

Description

Diabetic neuropathy is present in at least 50% of diabetes patients. While its primary symptoms are
unpleasant, secondary complications such as falls, foot ulcers, arrhythmias, and ileus are even more
serious.

e best option to reduce neuropathic pain in this situation is pregabalin.

Page - 401
Internal Medicine - Endocrinology

Question 28/162

Question #28

A 66-year-old male patient has a body mass index of 33 kg/m2 and has been diagnosed with type 2
DM. Unfortunately, diet and lifestyle modifications fail to achieve his proper A1C target. Considering
normal lab tests and physical examinations other than DM findings, what is the most appropriate
next step?

a. Start biguanides
b. Start insulin therapy
c. Start a sulfonylurea drug
d. Continue lifestyle modifications and exercise
e. Start regular acarbose

‫اﻹﺟﺎﺑﺔ ﻋﲆ اﻟﺼﻔﺤﺔ اﻟﺘﺎﻟﻴﺔ‬

Page - 402
Internal Medicine - Endocrinology - Diabetes Mellitus and its complications

Question 28/162

Question #28

A 66-year-old male patient has a body mass index of 33 kg/m2 and has been diagnosed with type 2
DM. Unfortunately, diet and lifestyle modifications fail to achieve his proper A1C target. Considering
normal lab tests and physical examinations other than DM findings, what is the most appropriate
next step?

a. Start biguanides √
b. Start insulin therapy
c. Start a sulfonylurea drug
d. Continue lifestyle modifications and exercise
e. Start regular acarbose

Description

Biguanides (Metformin) is the first line in the treatment of T2DM

It works by blocking gluconeogenesis and increasing insulin sensitivity but does not cause
hypoglycemia.

It is contraindicated if > 80 years old or GFR less than 30 ml/min (Risk of lactic acidosis)

It is considered the best for the treatment of DM in obese patients (Metformin help reduce weight)

e most common side eect is Gastrointestinal Upset (diarrhea, vomiting, nausea) and, rarely,
Lactic acidosis

Page - 403
Internal Medicine - Endocrinology

Question 29/162

Question #29

Which of the following insulins does the onset of action at 13 minutes and the peak at 1 – 3 hours?

a. Aspart
b. Premixed insulin
c. NPH
d. Detemir
e. Glargine

‫اﻹﺟﺎﺑﺔ ﻋﲆ اﻟﺼﻔﺤﺔ اﻟﺘﺎﻟﻴﺔ‬

Page - 404
Internal Medicine - Endocrinology - Diabetes Mellitus and its complications

Question 29/162

Question #29

Which of the following insulins does the onset of action at 13 minutes and the peak at 1 – 3 hours?

a. Aspart √
b. Premixed insulin
c. NPH
d. Detemir
e. Glargine

Description

Insulin Aspart starts its action 5 – 20 minutes aer administration and peaks at 0.5 to 3 hours.
Insulin can be rapid-acting, short-acting, intermediate-acting, or long-acting
Premixed insulin composed of rapid or short-acting combined with intermediated-acting
insulin

e type of insulin that can be given IV is called regular insulin

Page - 405
Internal Medicine - Endocrinology

Question 30/162

Question #30

A 29-year-old male patient is noted to have acanthosis nigricans and an HbA1c level of 8%. Which of
the following is the most likely type of DM he has?

a. Type 1 DM
b. Type 2 DM
c. Latent autoimmune DM of adults (LADA)
d. DM secondary to hemochromatosis
e. Maturity onset DM of youth (MODY)

‫اﻹﺟﺎﺑﺔ ﻋﲆ اﻟﺼﻔﺤﺔ اﻟﺘﺎﻟﻴﺔ‬

Page - 406
Internal Medicine - Endocrinology - Diabetes Mellitus and its complications

Question 30/162

Question #30

A 29-year-old male patient is noted to have acanthosis nigricans and an HbA1c level of 8%. Which of
the following is the most likely type of DM he has?

a. Type 1 DM
b. Type 2 DM √
c. Latent autoimmune DM of adults (LADA)
d. DM secondary to hemochromatosis
e. Maturity onset DM of youth (MODY)

Description

Acanthosis nigricans is a skin condition that results from hyperinsulinemia.

High insulin level stimulates epidermal keratocytes and dermal fibroblast proliferation leading to a
darkening of the skin in particular areas of the body.

Acanthosis nigricans is strong evidence of insulin resistance which points to the diagnosis of Type 2
DM.

Despite the young age of this patient, the presence of acanthosis nigricans is strongly suggestive of
Type 2 DM

Page - 407
Page - 408
Internal Medicine - Endocrinology

Question 31/162

Question #31

A 72-year-old male patient complains of morbid obesity and type 2 DM. His medications include
metformin and insulin, and his A1C level is 6.6%. e additional routine investigations demonstrate
persistent albuminuria and a high cholesterol level. Which of the following are the most appropriate
medications to be added to his regimen?

a. Atorvastatin and Valsartan


b. Atorvastatin and nitrates
c. Valsartan and niacin
d. B complex and folic acid
e. Sulfonylurea and enalapril

‫اﻹﺟﺎﺑﺔ ﻋﲆ اﻟﺼﻔﺤﺔ اﻟﺘﺎﻟﻴﺔ‬

Page - 409
Internal Medicine - Endocrinology - Diabetes Mellitus and its complications

Question 31/162

Question #31

A 72-year-old male patient complains of morbid obesity and type 2 DM. His medications include
metformin and insulin, and his A1C level is 6.6%. e additional routine investigations demonstrate
persistent albuminuria and a high cholesterol level. Which of the following are the most appropriate
medications to be added to his regimen?

a. Atorvastatin and Valsartan √


b. Atorvastatin and nitrates
c. Valsartan and niacin
d. B complex and folic acid
e. Sulfonylurea and enalapril

Description

e presence of persistent albuminuria suggests diabetic nephropathy.

Both diabetic nephropathy and high cholesterol level should be treated to reduce the risk of
complications in DM patients.

ACE inhibitors or ARBs are the best options to delay nephropathy as they reduce the pressure inside
the nephrons and delay the progression of the damage.

Statin is the first-line treatment of dyslipidemia in a patient with DM as it reduces the mortality and
the risk of complications in this patient.

Page - 410
Internal Medicine - Endocrinology

Question 32/162

Question #32

A 66-year-old male patient complains of being thirsty and urinating more than usual. His body mass
index is 35 kg/m2, and his urine dipstick is positive for glucose. What is the most appropriate next
step in the management of this patient?

a. Renal biopsy
b. KFT and electrolytes
c. Blood sugar measurement
d. Urinalysis for microscopy
e. Vasopressin serum level

‫اﻹﺟﺎﺑﺔ ﻋﲆ اﻟﺼﻔﺤﺔ اﻟﺘﺎﻟﻴﺔ‬

Page - 411
Internal Medicine - Endocrinology - Diabetes Mellitus and its complications

Question 32/162

Question #32

A 66-year-old male patient complains of being thirsty and urinating more than usual. His body mass
index is 35 kg/m2, and his urine dipstick is positive for glucose. What is the most appropriate next
step in the management of this patient?

a. Renal biopsy
b. KFT and electrolytes
c. Blood sugar measurement √
d. Urinalysis for microscopy
e. Vasopressin serum level

Description

is is a classic scenario of type 2 DM in which the symptoms of DM (polyuria and polydipsia) are
present, and the glucose is detected to be high in the urine.

Serum glucose measurement is the most appropriate at this point because it is more sensitive and
specific than urinary glucose level in diagnosing DM.

Page - 412
Internal Medicine - Endocrinology

Question 33/162

Question #33

A 23-year-old male patient with type 1 Dm presents with abdominal pain and rapid breathing.
Physical examination demonstrates deep, rapid breathing, dehydration, drowsiness, and fruity-
smelling breath. His plasma glucose is elevated, and the urine is positive for ketonuria. What is the
most appropriate initial treatment?

a. Normal saline followed by insulin infusion and KCL


b. Dextrose saline followed by an insulin
c. Normal saline and potassium infusion
d. Normal saline and subcutaneous insulin
e. Normal saline and sodium bicarbonate infusion

‫اﻹﺟﺎﺑﺔ ﻋﲆ اﻟﺼﻔﺤﺔ اﻟﺘﺎﻟﻴﺔ‬

Page - 413
Internal Medicine - Endocrinology - Diabetes Mellitus and its complications

Question 33/162

Question #33

A 23-year-old male patient with type 1 Dm presents with abdominal pain and rapid breathing.
Physical examination demonstrates deep, rapid breathing, dehydration, drowsiness, and fruity-
smelling breath. His plasma glucose is elevated, and the urine is positive for ketonuria. What is the
most appropriate initial treatment?

a. Normal saline followed by insulin infusion and KCL √


b. Dextrose saline followed by an insulin
c. Normal saline and potassium infusion
d. Normal saline and subcutaneous insulin
e. Normal saline and sodium bicarbonate infusion

Description

Description:

is patient suers from DKA.

e main treatments for DKA patients include normal saline, insulin infusion, and KCL.

Insulin and KCL should be initiated aer the first 1 liter of normal saline and until the serum
potassium results are known.

Page - 414
Internal Medicine - Endocrinology

Question 34/162

Question #34

A 22-year-old male patient was admitted to the hospital due to a road trac accident. However, his
investigations demonstrate glucosuria. Which of the following is the most appropriate investigation
for following up on his glucose level aer successfully treating his injury?

a. Random blood glucose


b. Fasting blood glucose
c. Repeat urinary dipstick for glucose
d. 24 hours urinary collection for glucose
e. Glycated hemoglobin (A1C)

‫اﻹﺟﺎﺑﺔ ﻋﲆ اﻟﺼﻔﺤﺔ اﻟﺘﺎﻟﻴﺔ‬

Page - 415
Internal Medicine - Endocrinology - Diabetes Mellitus and its complications

Question 34/162

Question #34

A 22-year-old male patient was admitted to the hospital due to a road trac accident. However, his
investigations demonstrate glucosuria. Which of the following is the most appropriate investigation
for following up on his glucose level aer successfully treating his injury?

a. Random blood glucose


b. Fasting blood glucose √
c. Repeat urinary dipstick for glucose
d. 24 hours urinary collection for glucose
e. Glycated hemoglobin (A1C)

Description

Urinary glucose level is not a reliable indicator for DM. However; it increases due to a decreased
urinary threshold (e.g., stress, pregnancy, and young people)

If an acutely ill or injured patient has glucosuria, you better perform fasting blood glucose
concentration to follow up and exclude DM.

Fasting blood sugar is more accurate than random blood sugar

HbA1C is a reliable screening and follow-up test used in the diagnosis, but it is more beneficial in
following up the treatment of known DM cases.

Page - 416
Internal Medicine - Endocrinology

Question 35/162

Question #35

An obese patient is diagnosed with type 2 DM. What is the first-line treatment if no
contraindications are present?

a. Pioglitazone
b. Sulfonylurea
c. Insulin
d. Metformin
e. Acarbose

‫اﻹﺟﺎﺑﺔ ﻋﲆ اﻟﺼﻔﺤﺔ اﻟﺘﺎﻟﻴﺔ‬

Page - 417
Internal Medicine - Endocrinology - Diabetes Mellitus and its complications

Question 35/162

Question #35

An obese patient is diagnosed with type 2 DM. What is the first-line treatment if no
contraindications are present?

a. Pioglitazone
b. Sulfonylurea
c. Insulin
d. Metformin √
e. Acarbose

Description

Biguanides (Metformin) is the first line in the treatment of T2DM

It works by blocking gluconeogenesis and increasing insulin sensitivity but does not cause
hypoglycemia.

It is contraindicated if > 80 years old or GFR less than 30 ml/min (Risk of lactic acidosis)

It is considered the best for the treatment of DM in obese patients (Metformin helps decrease
weight)

e most common side eect is Gastrointestinal Upset (diarrhea, vomiting, nausea) and, rarely,
Lactic acidosis

Page - 418
Internal Medicine - Endocrinology

Question 36/162

Question #36

Regarding diabetic ketoacidosis, which of the following statements is true?

c. e fluid of choice used in the treatment is Ringer lactate


d. e target treatment is to reduce the serum glucose to normal.
e. e lower the HCO3 reading, the more severe the disease
a. Insulin bolus of 0.1 unit per kg should be used
b. Ketogenesis results from suppressed lipolysis

‫اﻹﺟﺎﺑﺔ ﻋﲆ اﻟﺼﻔﺤﺔ اﻟﺘﺎﻟﻴﺔ‬

Page - 419
Internal Medicine - Endocrinology - Diabetes Mellitus and its complications

Question 36/162

Question #36

Regarding diabetic ketoacidosis, which of the following statements is true?

c. e fluid of choice used in the treatment is Ringer lactate


d. e target treatment is to reduce the serum glucose to normal.
e. e lower the HCO3 reading, the more severe the disease √
a. Insulin bolus of 0.1 unit per kg should be used
b. Ketogenesis results from suppressed lipolysis

Description

e lower HCO3 readings indicate more severe DKA (it correlates with the severity)
DKA usually occurs in T1DM, characterized by insulin deficiency with increasing
counterregulatory hormones (GH, Cortisone, Adrenalin, Glucagon)
Ketogenesis in this situation results from lipolysis which is activated by the low insulin levels.
Insulin boluses are not used in the treatment of DKA, but insulin infusion is the correct
treatment
e fluid of choice is normal saline; switch to dextrose containing fluid if the serum glucose is
less than 250 mg/dL
e target of the DKA treatment is to treat acidosis, HCO3 should be more than 15 and the
anion gap returns to normal. Aer that, the blood glucose should be controlled and then the
patient should be weaned from the DKA protocol

Page - 420
Internal Medicine - Endocrinology

Question 37/162

Question #37

Which of the following conditions is treated with metformin other than DM?

a. Diabetes insipidus
b. Hyperthyroidism
c. Hypothyroidism
d. Polycystic kidney disease
e. Polycystic ovarian syndrome

‫اﻹﺟﺎﺑﺔ ﻋﲆ اﻟﺼﻔﺤﺔ اﻟﺘﺎﻟﻴﺔ‬

Page - 421
Internal Medicine - Endocrinology - Diabetes Mellitus and its complications

Question 37/162

Question #37

Which of the following conditions is treated with metformin other than DM?

a. Diabetes insipidus
b. Hyperthyroidism
c. Hypothyroidism
d. Polycystic kidney disease
e. Polycystic ovarian syndrome √

Description

Besides diabetes, metformin is used to treat polycystic ovarian syndrome (PCOS)

Biguanides (Metformin) is the first line in the treatment of T2DM

It works by blocking gluconeogenesis and increasing insulin sensitivity but does not cause
hypoglycemia.

It is contraindicated if > 80 years old or GFR less than 30 ml/min (Risk of lactic acidosis)

It is considered the best for the treatment of DM in obese patients (Metformin helps decrease
weight)

e most common side eect is Gastrointestinal Upset (diarrhea, vomiting, nausea) and, rarely,
Lactic acidosis

Page - 422
Internal Medicine - Endocrinology

Question 38/162

Question #38

A 50-year-old male patient was recently diagnosed with type 2 DM. Which laboratory test should be
ordered before starting him on metformin?

a. Serum potassium and sodium


b. Serum creatinine
c. Complete blood count
d. Lipid profile
e. ECG stress test

‫اﻹﺟﺎﺑﺔ ﻋﲆ اﻟﺼﻔﺤﺔ اﻟﺘﺎﻟﻴﺔ‬

Page - 423
Internal Medicine - Endocrinology - Diabetes Mellitus and its complications

Question 38/162

Question #38

A 50-year-old male patient was recently diagnosed with type 2 DM. Which laboratory test should be
ordered before starting him on metformin?

a. Serum potassium and sodium


b. Serum creatinine √
c. Complete blood count
d. Lipid profile
e. ECG stress test

Description

Biguanides (Metformin) is the first line in the treatment of T2DM

It works by blocking gluconeogenesis and increasing insulin sensitivity but does not cause
hypoglycemia.

It is contraindicated if > 80 years old or GFR less than 30 ml/min (Risk of lactic acidosis)

It is considered the best for the treatment of DM in obese patients (Metformin helps decrease
weight)

e most common side eect is Gastrointestinal Upset (diarrhea, vomiting, nausea) and, rarely,
Lactic acidosis

Page - 424
Internal Medicine - Endocrinology

Question 39/162

Question #39

In patients with DKA, which of the following is true about the potassium levels in the body?

a. It remains unaected
b. It can be normal despite low total body potassium
c. It can be normal despite high total body potassium
d. It will naturally correct when insulin administrated
e. Hyperkalemia is always present

‫اﻹﺟﺎﺑﺔ ﻋﲆ اﻟﺼﻔﺤﺔ اﻟﺘﺎﻟﻴﺔ‬

Page - 425
Internal Medicine - Endocrinology - Diabetes Mellitus and its complications

Question 39/162

Question #39

In patients with DKA, which of the following is true about the potassium levels in the body?

a. It remains unaected
b. It can be normal despite low total body potassium √
c. It can be normal despite high total body potassium
d. It will naturally correct when insulin administrated
e. Hyperkalemia is always present

Description

Description:

In DKA patients, the total body potassium is deficient, but normal serum potassium is due to shiing
to extracellular fluids.

e insulin administration will shi the serum potassium intracellularly, causing hypokalemia, so
KCL should be provided even if the serum potassium is normal during the treatment of DKA.

Page - 426
Internal Medicine - Endocrinology

Question 40/162

Question #40

A 40-year-old male patient is found to have a fasting blood glucose level of 130 mg/dL during the
routine examination. However, on further evaluation, he has a second reading of 140 mg/dL aer 72
hours. What is the most appropriate at this time?

a. e patient has type 1 DM and should be treated with insulin


b. e patient has type 2 DM and should be started on lifestyle modification and exercise
c. A glucose tolerance test should be ordered at this time
d. is patient has prediabetes and should change his lifestyle
e. Fasting blood sugar should be repeated aer 1 month to ensure the diagnosis

‫اﻹﺟﺎﺑﺔ ﻋﲆ اﻟﺼﻔﺤﺔ اﻟﺘﺎﻟﻴﺔ‬

Page - 427
Internal Medicine - Endocrinology - Diabetes Mellitus and its complications

Question 40/162

Question #40

A 40-year-old male patient is found to have a fasting blood glucose level of 130 mg/dL during the
routine examination. However, on further evaluation, he has a second reading of 140 mg/dL aer 72
hours. What is the most appropriate at this time?

a. e patient has type 1 DM and should be treated with insulin


b. e patient has type 2 DM and should be started on lifestyle modification and exercise √
c. A glucose tolerance test should be ordered at this time
d. is patient has prediabetes and should change his lifestyle
e. Fasting blood sugar should be repeated aer 1 month to ensure the diagnosis

Description

Even in asymptomatic patients, two readings of fasting blood sugar of more than 126 are diagnostic
for DM.

Note that type 1 DM presents at a younger age and is not diagnosed during the routine examination
but presents with DKA.

Prediabetes is diagnosed when the fasting blood sugar is 90 – 125 mg/dL

A glucose tolerance test will not add value to the diagnosis at this stage; two elevated fasting
glucose readings are enough for the diagnosis.

Page - 428
Internal Medicine - Endocrinology

Question 41/162

Question #41

A 40-year-old male patient was diagnosed to have impaired glucose tolerance. Which of the
following lab test is most consistent with this diagnosis?

a. Fasting blood glucose of 105 mg/dL


b. Fasting blood glucose of 250 mg/dL
c. Random blood glucose of 130 mg/dL
d. 2h-75g-OGTT of 180 mg/dL
e. 2h-75g-OGTT of 280 mg/dL

‫اﻹﺟﺎﺑﺔ ﻋﲆ اﻟﺼﻔﺤﺔ اﻟﺘﺎﻟﻴﺔ‬

Page - 429
Internal Medicine - Endocrinology - Diabetes Mellitus and its complications

Question 41/162

Question #41

A 40-year-old male patient was diagnosed to have impaired glucose tolerance. Which of the
following lab test is most consistent with this diagnosis?

a. Fasting blood glucose of 105 mg/dL


b. Fasting blood glucose of 250 mg/dL
c. Random blood glucose of 130 mg/dL
d. 2h-75g-OGTT of 180 mg/dL √
e. 2h-75g-OGTT of 280 mg/dL

Description

Impaired glucose tolerance and impaired fasting glucose are not a disease but rather risk categories.

ey are called prediabetes (at risk of having DM in the future)

Fasting blood glucose of 105 mg/dL is called impaired fasting glucose

Fasting blood glucose of 250 mg/dL suggests DM

2h-75g-OGTT of 280 mg/dL suggests DM (not pre-DM)

Diagnosis of DM: any one of the following is diagnostic:

Symptoms of DM + RBS ≥ 200 mg/dl or,


One of the following On at least two separate occasions:
FBS ≥ 126 mg/dl
2h 75 g OGTT ≥ 200 mg/dl
RBS ≥ 200 mg/dl
HbA1c ≥ 6.5%

Diagnosis of Prediabetes:

Impaired fasting glucose (IFG): FBS 90-125 mg/dl


Impaired glucose tolerance (IGT): 2h 75 g OGTT 140-199 mg/dl
Page - 430
Page - 431
Internal Medicine - Endocrinology

Question 42/162

Question #42

One of the following lab tests is diagnostic for type 2 DM:

e. 2-hour postprandial 75g OGTT of 160 mg/dL


a. Symptoms of DM and random blood sugar of 180 mg/dl
b. Fasting plasma glucose of 126 mg/dL on two separate occasions
c. Random blood glucose of 200 mg/dL
d. HbA1C of 6% on 2 dierent occasions

‫اﻹﺟﺎﺑﺔ ﻋﲆ اﻟﺼﻔﺤﺔ اﻟﺘﺎﻟﻴﺔ‬

Page - 432
Internal Medicine - Endocrinology - Diabetes Mellitus and its complications

Question 42/162

Question #42

One of the following lab tests is diagnostic for type 2 DM:

e. 2-hour postprandial 75g OGTT of 160 mg/dL


a. Symptoms of DM and random blood sugar of 180 mg/dl
b. Fasting plasma glucose of 126 mg/dL on two separate occasions √
c. Random blood glucose of 200 mg/dL
d. HbA1C of 6% on 2 dierent occasions

Description

Diagnosis of DM: any one of the following is diagnostic:

Symptoms of DM + RBS ≥ 200 mg/dl or,

One of the following On at least two separate occasions:

FBS ≥ 126 mg/dl


2h 75 g OGTT ≥ 200 mg/dl
RBS ≥ 200 mg/dl
HbA1c ≥ 6.5%

e choices d and e are considered prediabetes, while one RBS reading of 200 without symptoms of
DM is not diagnostic.

Page - 433
Internal Medicine - Endocrinology

Question 43/162

Question #43

Glycated hemoglobin (HbA1C) is an appropriate screening test to assess the adequacy of the
treatment of DM patients. In which of the following patients does the HbA1C assay is inaccurate?

a. A 62-year-old male with Type 2 DM and dyslipidemia


b. A 52-year-old male with Type 2 DM and HTN
c. A 40-year-old with type 1 DM and Hypothyroidism
d. A 39-year-old male with Type 2 DM and morbid obesity
e. A 20-year-old male with Type 1 DM and sickle cell anemia

‫اﻹﺟﺎﺑﺔ ﻋﲆ اﻟﺼﻔﺤﺔ اﻟﺘﺎﻟﻴﺔ‬

Page - 434
Internal Medicine - Endocrinology - Diabetes Mellitus and its complications

Question 43/162

Question #43

Glycated hemoglobin (HbA1C) is an appropriate screening test to assess the adequacy of the
treatment of DM patients. In which of the following patients does the HbA1C assay is inaccurate?

a. A 62-year-old male with Type 2 DM and dyslipidemia


b. A 52-year-old male with Type 2 DM and HTN
c. A 40-year-old with type 1 DM and Hypothyroidism
d. A 39-year-old male with Type 2 DM and morbid obesity
e. A 20-year-old male with Type 1 DM and sickle cell anemia √

Description

e short RBC lifespan makes the HbA1C level show false low values.

e glycosylated hemoglobin assay is inaccurate by conditions aecting red blood cell survival, such
as sickle cell disease or the presence of hemoglobin C.

Page - 435
Internal Medicine - Endocrinology

Question 44/162

Question #44

In diabetic ketoacidosis, the formation of ketone bodies results from which of the following?

a. Reduced urinary ketone excretion


b. Destruction of the free fatty acids
c. Reduced cortisol level
d. High glucose level
e. High insulin level

‫اﻹﺟﺎﺑﺔ ﻋﲆ اﻟﺼﻔﺤﺔ اﻟﺘﺎﻟﻴﺔ‬

Page - 436
Internal Medicine - Endocrinology - Diabetes Mellitus and its complications

Question 44/162

Question #44

In diabetic ketoacidosis, the formation of ketone bodies results from which of the following?

a. Reduced urinary ketone excretion


b. Destruction of the free fatty acids √
c. Reduced cortisol level
d. High glucose level
e. High insulin level

Description

Description:

In the case of DKA, the lack of insulin is the main cause of lipolysis that leads to ketone bodies
formation and metabolic acidosis

DKA usually occurs in T1DM, characterized by insulin deficiency with increasing counterregulatory
hormones (GH, Cortisone, Adrenalin, Glucagon). It can present in T2DM, but that is very rare.

Page - 437
Internal Medicine - Endocrinology

Question 45/162

Question #45

A 49-year-old male patient complains of excessive thirst and urination. However, his vital signs are
normal, and his serum glucose level is 370 mg/dL. What is the most appropriate treatment?

a. Oral hypoglycemic agent


b. Long-acting insulin
c. Short-acting insulin
d. Intravenous regular insulin
e. Get an additional blood sugar reading

‫اﻹﺟﺎﺑﺔ ﻋﲆ اﻟﺼﻔﺤﺔ اﻟﺘﺎﻟﻴﺔ‬

Page - 438
Internal Medicine - Endocrinology - Diabetes Mellitus and its complications

Question 45/162

Question #45

A 49-year-old male patient complains of excessive thirst and urination. However, his vital signs are
normal, and his serum glucose level is 370 mg/dL. What is the most appropriate treatment?

a. Oral hypoglycemic agent √


b. Long-acting insulin
c. Short-acting insulin
d. Intravenous regular insulin
e. Get an additional blood sugar reading

Description

e presence of random blood glucose (RBS) of ≥ 200 mg/dL and the classic DM symptoms are
diagnostic of DM without the need for additional RBS reading.

Lifestyle modifications are the first-line treatment in this situation. However, it is not listed in the
choices, so starting an oral hypoglycemic agent is the first line in this scenario.

Page - 439
Internal Medicine - Endocrinology

Question 46/162

Question #46

A 70-year-old male patient was diagnosed recently with type 2 DM. His past history is significant for
CKD, and his baseline GFR is 25 ml/min. His body mass index is 37 kg/m2. What is the drug of choice
to control his DM?

a. Biguanides
b. Sulfonylurea
c. Pioglitazone
d. SGLT2 inhibitors
e. Insulin

‫اﻹﺟﺎﺑﺔ ﻋﲆ اﻟﺼﻔﺤﺔ اﻟﺘﺎﻟﻴﺔ‬

Page - 440
Internal Medicine - Endocrinology - Diabetes Mellitus and its complications

Question 46/162

Question #46

A 70-year-old male patient was diagnosed recently with type 2 DM. His past history is significant for
CKD, and his baseline GFR is 25 ml/min. His body mass index is 37 kg/m2. What is the drug of choice
to control his DM?

a. Biguanides
b. Sulfonylurea
c. Pioglitazone
d. SGLT2 inhibitors
e. Insulin √

Description

Biguanides, sulfonylurea, and sodium-glucose cotransporter 2 inhibitors are contraindicated


because of poor renal function.

Pioglitazone causes weight gain, so it is better to be avoided in this situation

Insulin is the most appropriate drug for this patient.

Page - 441
Internal Medicine - Endocrinology

Question 47/162

Question #47

A 49-year-old male patient complains of polyuria, polyphagia, and polydipsia for 4 months. His
physical examination demonstrates acanthosis nigricans and high BMI. His lab investigations show
a random blood sugar of 292 mg/dL. What is the best next step in the management of this patient?

a. Repeat random blood sugar


b. Perform fasting blood sugar
c. Urinary cortisol level
d. CBC, KFT, LFT, Electrolytes, A1C, and start DM treatment
e. Start insulin therapy

‫اﻹﺟﺎﺑﺔ ﻋﲆ اﻟﺼﻔﺤﺔ اﻟﺘﺎﻟﻴﺔ‬

Page - 442
Internal Medicine - Endocrinology - Diabetes Mellitus and its complications

Question 47/162

Question #47

A 49-year-old male patient complains of polyuria, polyphagia, and polydipsia for 4 months. His
physical examination demonstrates acanthosis nigricans and high BMI. His lab investigations show
a random blood sugar of 292 mg/dL. What is the best next step in the management of this patient?

a. Repeat random blood sugar


b. Perform fasting blood sugar
c. Urinary cortisol level
d. CBC, KFT, LFT, Electrolytes, A1C, and start DM treatment √
e. Start insulin therapy

Description

is patient has diabetes, and he needs to be treated.

e appropriate workup is to take baseline lab readings and start DM treatment.

Diagnosis of DM: any one of the following is diagnostic:

Symptoms of DM + RBS ≥ 200 mg/dl or,

One of the following On at least two separate occasions:

FBS ≥ 126 mg/dl


2h 75 g OGTT ≥ 200 mg/dl
RBS ≥ 200 mg/dl
HbA1c ≥ 6.5%

Symptomatic patients with elevated random blood sugar do not need additional readings to
confirm DM.

Insulin therapy is not indicated at this stage of type 2 DM; you better start on lifestyle modifications
and oral agents, and then insulin will be a choice.

Page - 443
Internal Medicine - Endocrinology

Question 48/162

Question #48

A diabetic 60-year-old male patient has complained of diarrhea for the past 4 months. His past
history is significant for type 2 DM treated with insulin alone. His lab tests show glycated
hemoglobin of 12%. What is the most likely cause of his diarrhea?

a. Gastroenteritis
b. Autonomic neuropathy
c. Inflammatory bowel disease
d. Irritable bowel syndrome
e. Giardiasis

‫اﻹﺟﺎﺑﺔ ﻋﲆ اﻟﺼﻔﺤﺔ اﻟﺘﺎﻟﻴﺔ‬

Page - 444
Internal Medicine - Endocrinology - Diabetes Mellitus and its complications

Question 48/162

Question #48

A diabetic 60-year-old male patient has complained of diarrhea for the past 4 months. His past
history is significant for type 2 DM treated with insulin alone. His lab tests show glycated
hemoglobin of 12%. What is the most likely cause of his diarrhea?

a. Gastroenteritis
b. Autonomic neuropathy √
c. Inflammatory bowel disease
d. Irritable bowel syndrome
e. Giardiasis

Description

Autonomic neuropathy is a well-known consequence of diabetes mellitus.

It aects both the sympathetic and parasympathetic branches of the autonomic nervous system.

Natural aging and alcoholism are other possible causes of neuropathy.

e damage to the sympathetic nervous system in the GI tract will lead to diarrhea, while the
damage to the parasympathetic typically results in constipation.

Page - 445
Internal Medicine - Endocrinology

Question 49/162

Question #49

e risk of diabetic complications is best reduced by which of the following measures?

a. Tight diabetic control


b. e usage of statin
c. e usage of ACE inhibitors
d. e usage of Aspirin
e. Lifestyle modification

‫اﻹﺟﺎﺑﺔ ﻋﲆ اﻟﺼﻔﺤﺔ اﻟﺘﺎﻟﻴﺔ‬

Page - 446
Internal Medicine - Endocrinology - Diabetes Mellitus and its complications

Question 49/162

Question #49

e risk of diabetic complications is best reduced by which of the following measures?

a. Tight diabetic control √


b. e usage of statin
c. e usage of ACE inhibitors
d. e usage of Aspirin
e. Lifestyle modification

Description

Tight glycemic control is the most important and eective method in reducing both microvascular
and macrovascular complications of diabetes mellitus.

ACE inhibitors can reduce the progression of diabetic nephropathy

Aspirin and statins can reduce the mortality rate and the cardiovascular events resulting from DM.

Page - 447
Internal Medicine - Endocrinology

Question 50/162

Question #50

Regarding diabetic ketoacidosis, which of the following statements is true?

a. Insulin bolus of 0.1 unit per kg should be used


b. Ketogenesis results from suppressed lipolysis
c. e fluid of choice used in the treatment is Ringer lactate
d. e target treatment is to reduce the serum glucose to normal.
e. e lower the HCO3 reading, the more severe the disease

‫اﻹﺟﺎﺑﺔ ﻋﲆ اﻟﺼﻔﺤﺔ اﻟﺘﺎﻟﻴﺔ‬

Page - 448
Internal Medicine - Endocrinology - Diabetes Mellitus and its complications

Question 50/162

Question #50

Regarding diabetic ketoacidosis, which of the following statements is true?

a. Insulin bolus of 0.1 unit per kg should be used


b. Ketogenesis results from suppressed lipolysis
c. e fluid of choice used in the treatment is Ringer lactate
d. e target treatment is to reduce the serum glucose to normal.
e. e lower the HCO3 reading, the more severe the disease √

Description

e lower HCO3 readings indicate more severe DKA (it correlates with the severity)
DKA usually occurs in T1DM, characterized by insulin deficiency with increasing
counterregulatory hormones (GH, Cortisone, Adrenalin, Glucagon)
Ketogenesis in this situation results from lipolysis which is activated by the low insulin levels.
Insulin boluses are not used in the treatment of DKA, but insulin infusion is the correct
treatment
e fluid of choice is normal saline; switch to dextrose containing fluid if the serum glucose is
less than 250 mg/dL
e target of the DKA treatment is to treat acidosis, HCO3 should be more than 15 and the
anion gap returns to normal. Aer that, the blood glucose should be controlled and then the
patient should be weaned from the DKA protocol

Page - 449
Internal Medicine - Endocrinology

Question 51/162

Question #51

A 22-yer-old female patient presents to the emergency department with abdominal pain. Her
physical examination demonstrates fruity-smelling breath and tachypnea. In addition, her lab
investigations show a blood sugar of 380 mg/dL, urinary ketones +++, and arterial blood PH of 7.3.
what is the most appropriate initial management?

a. Intravenous normal saline


b. Intravenous KCL infusion
c. Intravenous insulin infusion
d. Intravenous antibiotics
e. Intravenous bicarbonates

‫اﻹﺟﺎﺑﺔ ﻋﲆ اﻟﺼﻔﺤﺔ اﻟﺘﺎﻟﻴﺔ‬

Page - 450
Internal Medicine - Endocrinology - Diabetes Mellitus and its complications

Question 51/162

Question #51

A 22-yer-old female patient presents to the emergency department with abdominal pain. Her
physical examination demonstrates fruity-smelling breath and tachypnea. In addition, her lab
investigations show a blood sugar of 380 mg/dL, urinary ketones +++, and arterial blood PH of 7.3.
what is the most appropriate initial management?

a. Intravenous normal saline √


b. Intravenous KCL infusion
c. Intravenous insulin infusion
d. Intravenous antibiotics
e. Intravenous bicarbonates

Description

Normal saline is the first-line treatment for any patient suering from DKA.

Insulin infusion and KCL infusion should be started aer the initiation of normal saline or until the
serum potassium status is known.

All the mentioned choices are used in DKA treatment, but intravenous normal saline should be
started early before other treatments.

Page - 451
Internal Medicine - Endocrinology

Question 52/162

Question #52

A diabetic patient was prescribed metformin as a part of his management. All of the following are
true except:

a. It carries a risk of hypoglycemia


b. It is contraindicated if the GFR < 30 ml/min
c. Gastrointestinal ax are the most common side eects
d. It can help losing weight
e. It Works by blocking gluconeogenesis and increasing insulin sensitivity

‫اﻹﺟﺎﺑﺔ ﻋﲆ اﻟﺼﻔﺤﺔ اﻟﺘﺎﻟﻴﺔ‬

Page - 452
Internal Medicine - Endocrinology - Diabetes Mellitus and its complications

Question 52/162

Question #52

A diabetic patient was prescribed metformin as a part of his management. All of the following are
true except:

a. It carries a risk of hypoglycemia √


b. It is contraindicated if the GFR < 30 ml/min
c. Gastrointestinal ax are the most common side eects
d. It can help losing weight
e. It Works by blocking gluconeogenesis and increasing insulin sensitivity

Description

Metformin does not carry the risk of hypoglycemia.

Biguanides (Metformin) is the first line in the treatment of T2DM

It works by blocking gluconeogenesis and increasing insulin sensitivity but does not cause
hypoglycemia.

It is contraindicated if > 80 years old or GFR less than 30 ml/min (Risk of lactic acidosis)

It is considered the best for the treatment of DM in obese patients (Metformin helps decrease
weight)

e most common side eect is Gastrointestinal Upset (diarrhea, vomiting, nausea) and, rarely,
Lactic acidosis

Page - 453
Internal Medicine - Endocrinology

Question 53/162

Question #53

A 62-year-old male patient with a history of Type 2 DM requires urgent major abdominal surgery.
Which of the following drugs should be withheld until the normal KFT is documented 48 hours
postoperatively?

a. Acarbose
b. Sulfonylurea
c. Biguanides
d. Insulin
e. Pioglitazone

‫اﻹﺟﺎﺑﺔ ﻋﲆ اﻟﺼﻔﺤﺔ اﻟﺘﺎﻟﻴﺔ‬

Page - 454
Internal Medicine - Endocrinology - Diabetes Mellitus and its complications

Question 53/162

Question #53

A 62-year-old male patient with a history of Type 2 DM requires urgent major abdominal surgery.
Which of the following drugs should be withheld until the normal KFT is documented 48 hours
postoperatively?

a. Acarbose
b. Sulfonylurea
c. Biguanides √
d. Insulin
e. Pioglitazone

Description

e administration of general anesthesia may result in hypotension, which leads to kidney and
peripheral hypoperfusion, with subsequent lactate buildup. As a result, if radiocontrast material is
administered or urgent surgery is required, metformin should be delayed, and hydration should be
maintained until intact kidney function is confirmed at 24 and 48 hours following the intervention.

Page - 455
Internal Medicine - Endocrinology

Question 54/162

Question #54

A 43-year-old male patient is a known history of type 1 DM. He was presented by his wife to the
emergency department aer falling downstairs and becoming unconscious. What is the most
appropriate initial investigation?

a. Brain CT scan
b. Random blood sugar
c. Arterial blood gas
d. Brain MRI
e. ECG

‫اﻹﺟﺎﺑﺔ ﻋﲆ اﻟﺼﻔﺤﺔ اﻟﺘﺎﻟﻴﺔ‬

Page - 456
Internal Medicine - Endocrinology - Diabetes Mellitus and its complications

Question 54/162

Question #54

A 43-year-old male patient is a known history of type 1 DM. He was presented by his wife to the
emergency department aer falling downstairs and becoming unconscious. What is the most
appropriate initial investigation?

a. Brain CT scan
b. Random blood sugar √
c. Arterial blood gas
d. Brain MRI
e. ECG

Description

It is not uncommon to encounter hypoglycemia in diabetic patients because of high insulin doses or
taking insulin without adequate food intake.

Random blood sugar is the first rational investigation to be done in this situation to exclude
hypoglycemia.

Hypoglycemia is a priority and should be treated rapidly to avoid prolonged hypoglycemia and brain
damage.

Page - 457
Internal Medicine - Endocrinology

Question 55/162

Question #55

A 50-year-old female patient is a known case of type 2 DM. She presented for routine investigations
and was found to have a blood pressure of 142/91 mmHg and persistent albuminuria. What is the
most appropriate drug management?

a. ACE inhibitors
b. Calcium channel blockers
c. Beta-blockers
d. Statins
e. Aspirin

‫اﻹﺟﺎﺑﺔ ﻋﲆ اﻟﺼﻔﺤﺔ اﻟﺘﺎﻟﻴﺔ‬

Page - 458
Internal Medicine - Endocrinology - Diabetes Mellitus and its complications

Question 55/162

Question #55

A 50-year-old female patient is a known case of type 2 DM. She presented for routine investigations
and was found to have a blood pressure of 142/91 mmHg and persistent albuminuria. What is the
most appropriate drug management?

a. ACE inhibitors √
b. Calcium channel blockers
c. Beta-blockers
d. Statins
e. Aspirin

Description

Persistent albuminuria (previously called microalbuminuria) is an early sign of diabetic


nephropathy. ACE inhibitors and ARBs are the drugs of choice for this situation because they are
nephroprotective.

ACE inhibitors and ARBs delay the progression of diabetic nephropathy by reducing the pressure
inside the nephrons and reducing the damage.

Page - 459
Internal Medicine - Endocrinology

Question 56/162

Question #56

A 22-year-old male patient complains of central obesity and acanthosis nigricans. His family history
is significant for type 2 DM in his father and mother, and his lab tests show a fasting glucose level of
110 mg/dL. What is the most likely diagnosis?

a. Type 1 DM
b. Type 2 Dm
c. Prediabetes
d. Maturity onset diabetes of youth (MODY)
e. Latent autoimmune diabetes in adults (LADA)

‫اﻹﺟﺎﺑﺔ ﻋﲆ اﻟﺼﻔﺤﺔ اﻟﺘﺎﻟﻴﺔ‬

Page - 460
Internal Medicine - Endocrinology - Diabetes Mellitus and its complications

Question 56/162

Question #56

A 22-year-old male patient complains of central obesity and acanthosis nigricans. His family history
is significant for type 2 DM in his father and mother, and his lab tests show a fasting glucose level of
110 mg/dL. What is the most likely diagnosis?

a. Type 1 DM
b. Type 2 Dm
c. Prediabetes √
d. Maturity onset diabetes of youth (MODY)
e. Latent autoimmune diabetes in adults (LADA)

Description

is patient is at risk of having DM in the future.

His family history of DM, the presence of acanthosis nigricans, and the blood glucose reading of 90 –
125 mg/dL strongly suggest prediabetes

Diagnosis of Prediabetes:

Impaired fasting glucose (IFG): FBS 90-125 mg/dl


Impaired glucose tolerance (IGT): 2h 75 g OGTT140-199 mg/dl

Page - 461
Internal Medicine - Endocrinology

Question 57/162

Question #57

A 40-year-old male patient with DM comes for a follow-up. He recently has increased appetite and
weight loss. He takes insulin in divided doses but takes no other medications. His serum glucose
concentrations were tested at home for the last 3 weeks, ranging from 300 – 350 mg/dL. What is the
most appropriate next step in the management of his condition?

a. Add metformin
b. Change to another insulin type
c. Increase his insulin dose
d. Reduce his insulin dose
e. Provide his daily insulin as a single dose

‫اﻹﺟﺎﺑﺔ ﻋﲆ اﻟﺼﻔﺤﺔ اﻟﺘﺎﻟﻴﺔ‬

Page - 462
Internal Medicine - Endocrinology - Diabetes Mellitus and its complications

Question 57/162

Question #57

A 40-year-old male patient with DM comes for a follow-up. He recently has increased appetite and
weight loss. He takes insulin in divided doses but takes no other medications. His serum glucose
concentrations were tested at home for the last 3 weeks, ranging from 300 – 350 mg/dL. What is the
most appropriate next step in the management of his condition?

a. Add metformin
b. Change to another insulin type
c. Increase his insulin dose √
d. Reduce his insulin dose
e. Provide his daily insulin as a single dose

Description

is patient has inadequate blood glucose control and needs more insulin.

Reducing insulin dose will result in more elevated blood sugar

Adding metformin is not appropriate because it will be of a little benefit

Single-dose insulin carries a risk of hypoglycemia and will not help reduce blood sugar control in this
situation.

Page - 463
Internal Medicine - Endocrinology

Question 58/162

Question #58

In a patient with diabetes, which of the following is the most appropriate test for long-term follow-
up of his blood sugar control state?

a. Glycated hemoglobin
b. Fasting blood glucose
c. Random blood glucose
d. C-Peptide
e. GAD antibodies titer

‫اﻹﺟﺎﺑﺔ ﻋﲆ اﻟﺼﻔﺤﺔ اﻟﺘﺎﻟﻴﺔ‬

Page - 464
Internal Medicine - Endocrinology - Diabetes Mellitus and its complications

Question 58/162

Question #58

In a patient with diabetes, which of the following is the most appropriate test for long-term follow-
up of his blood sugar control state?

a. Glycated hemoglobin √
b. Fasting blood glucose
c. Random blood glucose
d. C-Peptide
e. GAD antibodies titer

Description

Glucose can bind irreversibly to hemoglobin via a process known as glycation. erefore, the
hemoglobin with glucose attached is known as glycated hemoglobin, oen abbreviated to HbA1C.

HbA1C gives an Idea about the last 2 – 3 months of glycemic control, and it is an appropriate test to
assess the adequacy of DM treatment.

Another method is the frequent measurements of blood glucose, but this is a short-term method
and results in multiple punctures and pain in the skin.

Page - 465
Internal Medicine - Endocrinology

Question 59/162

Question #59

A 40-year-old male patient complains of feeling thirsty despite drinking plenty of water. Which of
the following is the most likely laboratory finding associated with central diabetes insipidus?

a. Increased urine osmolality with the administration of vasopressin


b. Decreased urine osmolality with the administration of vasopressin
c. Increased urine osmolality with water restriction
d. Decreased urine osmolality with water restriction
e. Increased urine osmolality with intravenous fluid administration

‫اﻹﺟﺎﺑﺔ ﻋﲆ اﻟﺼﻔﺤﺔ اﻟﺘﺎﻟﻴﺔ‬

Page - 466
Internal Medicine - Endocrinology - Disorders of ADH

Question 59/162

Question #59

A 40-year-old male patient complains of feeling thirsty despite drinking plenty of water. Which of
the following is the most likely laboratory finding associated with central diabetes insipidus?

a. Increased urine osmolality with the administration of vasopressin √


b. Decreased urine osmolality with the administration of vasopressin
c. Increased urine osmolality with water restriction
d. Decreased urine osmolality with water restriction
e. Increased urine osmolality with intravenous fluid administration

Description

Positive water deprivation test is present in both types of DI, while vasopressin challenge test is
positive in central DI, not nephrogenic DI

Increased urine osmolality with water restriction is a normal finding that indicates no DI.

Diabetes insipidus manifests as low urinary sodium, urine osmolality, and specific gravity. In
addition, serum sodium and serum osmolality are elevated.

Diabetes insipidus (DI) results in insucient (central) or ineective (nephrogenic) ADH → high
volume water loss in urine

ere are two types of the disease:

Central DI: Any CNS disorder (stroke, tumor, trauma, hypoxia, infection, histiocytosis) that
impairs the production of ADH in the hypothalamus or storage in the posterior pituitary gland.
Nephrogenic DI: less common, characterized by loss of ADH eect on collecting ducts, caused
by (lithium, demeclocycline, CKD, Hypokalemia, hypercalcemia, pyelonephritis, renal
amyloidosis, Sjögren’s syndrome)

Page - 467
Internal Medicine - Endocrinology

Question 60/162

Question #60

A 33-year-old male patient develops polyuria aer his pituitary surgery. Physical examination
demonstrates dehydration despite receiving 5 liters of fluid in the past 24 hours. What is the most
appropriate management?

a. Provide vasopressin
b. Provide more intravenous fluid
c. Provide tap water orally in addition to the intravenous fluid
d. Provide thiazide diuretics
e. Start demeclocycline

‫اﻹﺟﺎﺑﺔ ﻋﲆ اﻟﺼﻔﺤﺔ اﻟﺘﺎﻟﻴﺔ‬

Page - 468
Internal Medicine - Endocrinology - Disorders of ADH

Question 60/162

Question #60

A 33-year-old male patient develops polyuria aer his pituitary surgery. Physical examination
demonstrates dehydration despite receiving 5 liters of fluid in the past 24 hours. What is the most
appropriate management?

a. Provide vasopressin √
b. Provide more intravenous fluid
c. Provide tap water orally in addition to the intravenous fluid
d. Provide thiazide diuretics
e. Start demeclocycline

Description

Persistent polyuria, despite appropriate fluid therapy in a patient with pituitary gland surgery,
suggests diabetes insipidus.

Diabetes insipidus (DI) results in insucient (central) or ineective (nephrogenic) ADH → high
volume water loss in urine

ere are two types of the disease:

Central DI: Any CNS disorder (stroke, tumor, trauma, hypoxia, infection, histiocytosis) that
impairs the production of ADH in the hypothalamus or storage in the posterior pituitary gland.
Nephrogenic DI: less common, characterized by loss of ADH eect on collecting ducts, caused
by (lithium, demeclocycline, CKD, Hypokalemia, hypercalcemia, pyelonephritis, renal
amyloidosis, Sjögren’s syndrome)

Central type DI is treated by vasopressin (DDAVP), while nephrogenic DI is treated by the correction
of the cause and stopping the oending agent

Page - 469
Internal Medicine - Endocrinology

Question 61/162

Question #61

A 65-year-old male patient complains of confusion. His lab investigations demonstrate serum
sodium of 116 mEq/L, serum osmolality of 248 mOsm/Kg H2O, and urine osmolality of 286
mOsm/Kg H2O. However, his liver, kidney, cardiac, and hepatic functions are normal. What is the
most appropriate treatment at this time?

a. Hypertonic 3% saline
b. Normal saline 0.9%
c. Water restriction only
d. Demeclocycline
e. Vasopressin

‫اﻹﺟﺎﺑﺔ ﻋﲆ اﻟﺼﻔﺤﺔ اﻟﺘﺎﻟﻴﺔ‬

Page - 470
Internal Medicine - Endocrinology - Disorders of ADH

Question 61/162

Question #61

A 65-year-old male patient complains of confusion. His lab investigations demonstrate serum
sodium of 116 mEq/L, serum osmolality of 248 mOsm/Kg H2O, and urine osmolality of 286
mOsm/Kg H2O. However, his liver, kidney, cardiac, and hepatic functions are normal. What is the
most appropriate treatment at this time?

a. Hypertonic 3% saline
b. Normal saline 0.9%
c. Water restriction only
d. Demeclocycline √
e. Vasopressin

Description

In the presence of hyponatremia, kidneys are expected to dilute urine maximally to less than 100
mOsm/kg water.

is patient has reduced serum sodium and serum osmolality and a urine osmolality of more than
100 mOsm/Kg water (not maximally diluted urine). is is typical for SIADH.

Syndrome of inappropriate anti-diuretic hormone (SIADH) is a syndrome in which there is over-


secretion of ADH leading to water retention, hyponatremia, hypoosmolality, high urine sodium, and
high urine osmolality.

Page - 471
Internal Medicine - Endocrinology

Question 62/162

Question #62

A patient complains of confusion secondary to hyponatremia. Which of the following fits with the
diagnosis of SIADH?

a. Elevated serum glucose


b. Elevated serum osmolality
c. Reduced urine osmolality
d. Elevated urine osmolality
e. Low plasma arginine vasopressin

‫اﻹﺟﺎﺑﺔ ﻋﲆ اﻟﺼﻔﺤﺔ اﻟﺘﺎﻟﻴﺔ‬

Page - 472
Internal Medicine - Endocrinology - Disorders of ADH

Question 62/162

Question #62

A patient complains of confusion secondary to hyponatremia. Which of the following fits with the
diagnosis of SIADH?

a. Elevated serum glucose


b. Elevated serum osmolality
c. Reduced urine osmolality
d. Elevated urine osmolality √
e. Low plasma arginine vasopressin

Description

In the presence of hyponatremia, kidneys are expected to dilute urine maximally to less than 100
mOsm/kg water.

Syndrome of inappropriate anti-diuretic hormone (SIADH) is a syndrome in which there is over-


secretion of ADH leading to water retention, hyponatremia, hypoosmolality, high urine sodium, and
high urine osmolality.

Page - 473
Internal Medicine - Endocrinology

Question 63/162

Question #63

A 66-year-old male patient complains of confusion for a one-day duration. His lab investigations
demonstrate serum sodium of 116 mEq/L, random blood sugar of 130 mg/dL, and serum osmolality
of 248 mOsm/kg H2O. Which of the following is the most likely diagnosis?

a. Diabetes insipidus
b. Inappropriate ADH secretion
c. Hyperosmolar hyperglycemic state
d. Pseudohyponatremia
e. Congestive heart failure

‫اﻹﺟﺎﺑﺔ ﻋﲆ اﻟﺼﻔﺤﺔ اﻟﺘﺎﻟﻴﺔ‬

Page - 474
Internal Medicine - Endocrinology - Disorders of ADH

Question 63/162

Question #63

A 66-year-old male patient complains of confusion for a one-day duration. His lab investigations
demonstrate serum sodium of 116 mEq/L, random blood sugar of 130 mg/dL, and serum osmolality
of 248 mOsm/kg H2O. Which of the following is the most likely diagnosis?

a. Diabetes insipidus
b. Inappropriate ADH secretion √
c. Hyperosmolar hyperglycemic state
d. Pseudohyponatremia
e. Congestive heart failure

Description

Hyponatremia and hypoosmolality suggest SIADH as the most likely diagnosis.

Syndrome of inappropriate anti-diuretic hormone (SIADH) is a syndrome in which there is over-


secretion of ADH leading to water retention, hyponatremia, hypoosmolality, high urine sodium, and
high urine osmolality.

Page - 475
Internal Medicine - Endocrinology

Question 64/162

Question #64

A 32-year-old female is known to have bipolar disorder. She receives her treatment regularly. She
presents with polyuria, hypernatremia, and features of dehydration. Her water deprivation test fails
to raise urine osmolality. What is the most likely diagnosis?

a. Central diabetes insipidus


b. Nephrogenic diabetes insipidus
c. Inappropriate ADH secretion
d. Renal impairment
e. Heart failure

‫اﻹﺟﺎﺑﺔ ﻋﲆ اﻟﺼﻔﺤﺔ اﻟﺘﺎﻟﻴﺔ‬

Page - 476
Internal Medicine - Endocrinology - Disorders of ADH

Question 64/162

Question #64

A 32-year-old female is known to have bipolar disorder. She receives her treatment regularly. She
presents with polyuria, hypernatremia, and features of dehydration. Her water deprivation test fails
to raise urine osmolality. What is the most likely diagnosis?

a. Central diabetes insipidus


b. Nephrogenic diabetes insipidus √
c. Inappropriate ADH secretion
d. Renal impairment
e. Heart failure

Description

e use of lithium (a mood stabilizer used in bipolar disorder) is associated with nephrogenic DI

Diabetes insipidus (DI) results in insucient (central) or ineective (nephrogenic) ADH → high
volume water loss in urine

ere are two types of the disease:

Central DI: Any CNS disorder (stroke, tumor, trauma, hypoxia, infection, histiocytosis) that
impairs the production of ADH in the hypothalamus or storage in the posterior pituitary gland.
Nephrogenic DI: less common, characterized by loss of ADH eect on collecting ducts, caused
by (lithium, demeclocycline, CKD, Hypokalemia, hypercalcemia, pyelonephritis, renal
amyloidosis, Sjögren’s syndrome)

Central type DI is treated by vasopressin (DDAVP), while nephrogenic DI is treated by the correction
of the cause and stopping the oending agent

Page - 477
Internal Medicine - Endocrinology

Question 65/162

Question #65

In a 22-year-old female, the diagnosis of diabetes insipidus is suspected. What is the most likely
laboratory pattern to manifest in this patient?

a. High plasma osmolality, low urine osmolality, high serum sodium


b. High plasma osmolality, low urine osmolality, low serum sodium
c. High plasma osmolality, high urine osmolality, high serum sodium
d. Low plasma osmolality, low urine osmolality, high serum sodium
e. Low plasma osmolality, high urine osmolality, high serum sodium

‫اﻹﺟﺎﺑﺔ ﻋﲆ اﻟﺼﻔﺤﺔ اﻟﺘﺎﻟﻴﺔ‬

Page - 478
Internal Medicine - Endocrinology - Disorders of ADH

Question 65/162

Question #65

In a 22-year-old female, the diagnosis of diabetes insipidus is suspected. What is the most likely
laboratory pattern to manifest in this patient?

a. High plasma osmolality, low urine osmolality, high serum sodium √


b. High plasma osmolality, low urine osmolality, low serum sodium
c. High plasma osmolality, high urine osmolality, high serum sodium
d. Low plasma osmolality, low urine osmolality, high serum sodium
e. Low plasma osmolality, high urine osmolality, high serum sodium

Description

Diabetes insipidus manifests as low urinary sodium, urine osmolality, and specific gravity. In
addition, serum sodium and serum osmolality are elevated.

Diabetes insipidus (DI) results in insucient (central) or ineective (nephrogenic) ADH → high
volume water loss in urine

ere are two types of the disease:

Central DI: Any CNS disorder (stroke, tumor, trauma, hypoxia, infection, histiocytosis) that
impairs the production of ADH in the hypothalamus or storage in the posterior pituitary gland.
Nephrogenic DI: less common, characterized by loss of ADH eect on collecting ducts, caused
by (lithium, demeclocycline, CKD, Hypokalemia, hypercalcemia, pyelonephritis, renal
amyloidosis, Sjögren’s syndrome)

Central type DI is treated by vasopressin (DDAVP), while nephrogenic DI is treated by the correction
of the cause and stopping the oending agent

Page - 479
Internal Medicine - Endocrinology

Question 66/162

Question #66

A 20-year-old male patient is treated for meningitis. On the third day aer admission, the patient
becomes confused and develops convulsions that fail to respond to anticonvulsant therapy. Which
is the most likely cause of refractory seizure in meningitis patients?

a. Brain tissue ischemia


b. Brain abscess
c. Elevated intracranial pressure
d. Hyponatremia from SIADH
e. Subdural empyema

‫اﻹﺟﺎﺑﺔ ﻋﲆ اﻟﺼﻔﺤﺔ اﻟﺘﺎﻟﻴﺔ‬

Page - 480
Internal Medicine - Endocrinology - Disorders of ADH

Question 66/162

Question #66

A 20-year-old male patient is treated for meningitis. On the third day aer admission, the patient
becomes confused and develops convulsions that fail to respond to anticonvulsant therapy. Which
is the most likely cause of refractory seizure in meningitis patients?

a. Brain tissue ischemia


b. Brain abscess
c. Elevated intracranial pressure
d. Hyponatremia from SIADH √
e. Subdural empyema

Description

Hyponatremia from the syndrome of inappropriate secretion of ADH (SIADH) is a common


complication of bacterial meningitis.

Hyponatremia may cause refractory seizures that should be treated with an appropriate hypertonic
saline solution.

Syndrome of inappropriate anti-diuretic hormone (SIADH) is a syndrome in which there is over-


secretion of ADH leading to water retention, hyponatremia, hypoosmolality, high urine sodium, and
high urine osmolality.

Page - 481
Internal Medicine - Endocrinology

Question 67/162

Question #67

A 50-year-old male patient with a history of lung cancer presents with confusion. His lab
investigations demonstrate serum sodium of 119 mEq/L and normal blood sugar. In addition, his
urine Osmolality is high. What is the most likely diagnosis?

a. Renal impairment
b. Hyperaldosteronism
c. Diabetes insipidus
d. Inappropriate ADH secretion
e. Pseudohyponatremia

‫اﻹﺟﺎﺑﺔ ﻋﲆ اﻟﺼﻔﺤﺔ اﻟﺘﺎﻟﻴﺔ‬

Page - 482
Internal Medicine - Endocrinology - Disorders of ADH

Question 67/162

Question #67

A 50-year-old male patient with a history of lung cancer presents with confusion. His lab
investigations demonstrate serum sodium of 119 mEq/L and normal blood sugar. In addition, his
urine Osmolality is high. What is the most likely diagnosis?

a. Renal impairment
b. Hyperaldosteronism
c. Diabetes insipidus
d. Inappropriate ADH secretion √
e. Pseudohyponatremia

Description

Syndrome of inappropriate anti-diuretic hormone (SIADH) is a syndrome in which there is over-


secretion of ADH leading to water retention, hyponatremia, hypo-osmolality, high urine sodium,
and high urine osmolality.

In this patient, the ADH source is mostly from lung cancer (as a paraneoplastic feature).

Page - 483
Internal Medicine - Endocrinology

Question 68/162

Question #68

A 33-year-old man with Sjögren’s syndrome came to your clinic with polyuria and polydipsia for the
past week. His lab investigations demonstrate a serum fasting glucose of 70 mg/dL, serum calcium
of 9.1 mg/dL, serum sodium of 152 mEq/L, serum creatinine of 0.8 mg/dL and serum urea of 15
mmol/L. what is the best initial test to do for this patient?

a. Desmopressin challenge test


b. Water deprivation test
c. Brain CT scan
d. Sweat chloride test
e. Schirmer’s test

‫اﻹﺟﺎﺑﺔ ﻋﲆ اﻟﺼﻔﺤﺔ اﻟﺘﺎﻟﻴﺔ‬

Page - 484
Internal Medicine - Endocrinology - Disorders of ADH

Question 68/162

Question #68

A 33-year-old man with Sjögren’s syndrome came to your clinic with polyuria and polydipsia for the
past week. His lab investigations demonstrate a serum fasting glucose of 70 mg/dL, serum calcium
of 9.1 mg/dL, serum sodium of 152 mEq/L, serum creatinine of 0.8 mg/dL and serum urea of 15
mmol/L. what is the best initial test to do for this patient?

a. Desmopressin challenge test


b. Water deprivation test √
c. Brain CT scan
d. Sweat chloride test
e. Schirmer’s test

Description

e history of Sjögren’s syndrome, polyurea, hypernatremia, hyperosmolality, and normal glucose


and calcium strongly suggest diabetes insipidus (DI).

e best initial test in this situation is water deprivation test.

e most accurate to dierentiate central vs. peripheral DI is desmopressin challenge test.

Note that sweat chloride test is used for cystic fibrosis while Schirmer’s test is used for the diagnosis
of Sjögren’s syndrome which is already diagnosed here.

Page - 485
Internal Medicine - Endocrinology

Question 69/162

Question #69

A 45-year-old male patient complains of polyuria of about 5 liters per day. His urinalysis shows a
urine specific gravity of 1.001, absent RBC, absent WBC, normal glucose, and absent albumin. His
serum sodium is 146 mEq/L, and his serum potassium is 4 mEq/L. e water deprivation test results
in a urine specific gravity of 1.001 and serum sodium of 150 mEq/L. What is the most likely diagnosis?

a. Primary polydipsia
b. Diabetes insipidus
c. Syndrome of inappropriate ADH secretion
d. Chronic renal failure
e. Diabetes mellitus

‫اﻹﺟﺎﺑﺔ ﻋﲆ اﻟﺼﻔﺤﺔ اﻟﺘﺎﻟﻴﺔ‬

Page - 486
Internal Medicine - Endocrinology - Disorders of ADH

Question 69/162

Question #69

A 45-year-old male patient complains of polyuria of about 5 liters per day. His urinalysis shows a
urine specific gravity of 1.001, absent RBC, absent WBC, normal glucose, and absent albumin. His
serum sodium is 146 mEq/L, and his serum potassium is 4 mEq/L. e water deprivation test results
in a urine specific gravity of 1.001 and serum sodium of 150 mEq/L. What is the most likely diagnosis?

a. Primary polydipsia
b. Diabetes insipidus √
c. Syndrome of inappropriate ADH secretion
d. Chronic renal failure
e. Diabetes mellitus

Description

Polyuria, hyponatremia, and diluted urine that fails to concentrate aer the water deprivation test
strongly suggest diabetes insipidus.

Diabetes insipidus (DI) results in insucient (central) or ineective (nephrogenic) ADH → high
volume water loss in urine

ere are two types of the disease:

Central DI: Any CNS disorder (stroke, tumor, trauma, hypoxia, infection, histiocytosis) that
impairs the production of ADH in the hypothalamus or storage in the posterior pituitary gland.
Nephrogenic DI: less common, characterized by loss of ADH eect on collecting ducts, caused
by (lithium, demeclocycline, CKD, Hypokalemia, hypercalcemia, pyelonephritis, renal
amyloidosis, Sjögren’s syndrome)

Central type DI is treated by vasopressin (DDAVP), while nephrogenic DI is treated by the correction
of the cause and stopping the oending agent

Page - 487
Internal Medicine - Endocrinology

Question 70/162

Question #70

A 45-year-old male patient complains of polyuria of about 5 liters per day. His urinalysis shows a
urine specific gravity of 1.001, absent RBC, absent WBC, normal glucose, and absent albumin. His
serum sodium is 140 mEq/L, and his serum potassium is 4 mEq/L. Water deprivation test results in a
urine specific gravity of 1.025 and serum sodium of 142 mEq/L. What is the most likely diagnosis?

a. Primary polydipsia
b. Diabetes insipidus
c. Syndrome of inappropriate ADH secretion
d. Chronic renal failure
e. Diabetes mellitus

‫اﻹﺟﺎﺑﺔ ﻋﲆ اﻟﺼﻔﺤﺔ اﻟﺘﺎﻟﻴﺔ‬

Page - 488
Internal Medicine - Endocrinology - Disorders of ADH

Question 70/162

Question #70

A 45-year-old male patient complains of polyuria of about 5 liters per day. His urinalysis shows a
urine specific gravity of 1.001, absent RBC, absent WBC, normal glucose, and absent albumin. His
serum sodium is 140 mEq/L, and his serum potassium is 4 mEq/L. Water deprivation test results in a
urine specific gravity of 1.025 and serum sodium of 142 mEq/L. What is the most likely diagnosis?

a. Primary polydipsia √
b. Diabetes insipidus
c. Syndrome of inappropriate ADH secretion
d. Chronic renal failure
e. Diabetes mellitus

Description

e elevation of specific gravity with water restriction indicates that there is no diabetes insipidus.

Primary polydipsia (psychogenic polydipsia) is the presence of high fluid intake leading to polyuria
and diluted urine that will concentrate with a water deprivation test.

Page - 489
Internal Medicine - Endocrinology

Question 71/162

Question #71

A previously healthy 33-year-old male patient complains of polyuria and polydipsia for several days.
On examination, the patient is euvolemic and has no significant findings. His lab investigations
show serum calcium, potassium, glucose, and kidney function of normal limits. However, urinalysis
demonstrates low specific gravity and low urine osmolality. Which of the following is the most likely
deficient hormone in this patient?

a. Renin
b. Cortisol
c. Insulin
d. Arginine vasopressin
e. Growth hormone

‫اﻹﺟﺎﺑﺔ ﻋﲆ اﻟﺼﻔﺤﺔ اﻟﺘﺎﻟﻴﺔ‬

Page - 490
Internal Medicine - Endocrinology - Disorders of ADH

Question 71/162

Question #71

A previously healthy 33-year-old male patient complains of polyuria and polydipsia for several days.
On examination, the patient is euvolemic and has no significant findings. His lab investigations
show serum calcium, potassium, glucose, and kidney function of normal limits. However, urinalysis
demonstrates low specific gravity and low urine osmolality. Which of the following is the most likely
deficient hormone in this patient?

a. Renin
b. Cortisol
c. Insulin
d. Arginine vasopressin √
e. Growth hormone

Description

Polyuria, low specific gravity, and low urine osmolality suggest diabetes insipidus

Diabetes insipidus (DI) results in insucient (central) or ineective (nephrogenic) ADH → high
volume water loss in urine

ere are two types of the disease:

Central DI: Any CNS disorder (stroke, tumor, trauma, hypoxia, infection, histiocytosis) that
impairs the production of ADH in the hypothalamus or storage in the posterior pituitary gland.
Nephrogenic DI: less common, characterized by loss of ADH eect on collecting ducts, caused
by (lithium, demeclocycline, CKD, Hypokalemia, hypercalcemia, pyelonephritis, renal
amyloidosis, Sjögren’s syndrome)

Central type DI is treated by vasopressin (DDAVP), while nephrogenic DI is treated by the correction
of the cause and stopping the oending agent

Page - 491
Internal Medicine - Endocrinology

Question 72/162

Question #72

e following lipid-lowering agents are matched with the correct statement except:

a. Statin → the best to treat high LDL


b. Niacin → the best to rise HDL
c. Fibrates → the best to reduce Triglycerides
d. Cholestyramine → bile acid sequestrant
e. Ezetimibe → high mortality benefit

‫اﻹﺟﺎﺑﺔ ﻋﲆ اﻟﺼﻔﺤﺔ اﻟﺘﺎﻟﻴﺔ‬

Page - 492
Internal Medicine - Endocrinology - Dyslipidemia

Question 72/162

Question #72

e following lipid-lowering agents are matched with the correct statement except:

a. Statin → the best to treat high LDL


b. Niacin → the best to rise HDL
c. Fibrates → the best to reduce Triglycerides
d. Cholestyramine → bile acid sequestrant
e. Ezetimibe → high mortality benefit √

Description

Ezetimibe (Cholesterol absorption inhibitor) Lowers LDL but with no mortality benefit

e drugs used in dyslipidemia are as the following:

Statins (HMG CO-A inhibitors)


e best treatment for a high LDL level
e lipid-lowering agent with the strongest mortality benefit
ey have an antioxidant eect on the endothelial lining
E.g., Atorvastatin, Simvastatin, Lovastatin, Rosuvastatin
Side eects: Myositis, hepatotoxicity, warfarin potentiation
Niacin:
Excellent drug to add the statin
Niacin is stronger than statin, exercise, and smoking cessation in raising HDL
Associated with Glucose intolerance, the elevation of uric acid, and itching
Fibrates (lipoprotein lipase stimulators)
E.g., Gemfibrozil, fenofibrate
Lower TG level more than statin
e combination with a statin increases the risk of Myositis
Cholestyramine:
Bile acid Sequestrant
Has significant interaction with other medications in the gut (decrease their absorption)
Side eects include constipation and Flatus.
Ezetimibe (Cholesterol absorption inhibitor)
Page - 493
Lowers LDL but with no benefit
Not better than placebo in the clinical endpoint
Protein Convertase Subtilisin Kexin type 9 (PCSK9) inhibitors
E.g., Evolocumab, Alirocumab
Injectable medication that can decrease LDL strongly
Used if the patient is not responsive or can’t tolerate statin

Page - 494
Internal Medicine - Endocrinology

Question 73/162

Question #73

Which of the following is the most atherogenic?

a. High-density lipoprotein (HDL)


b. Very Low-density lipoprotein (VLDL)
c. Chylomicrons
d. Low-density lipoprotein (LDL)
e. Triglycerides

‫اﻹﺟﺎﺑﺔ ﻋﲆ اﻟﺼﻔﺤﺔ اﻟﺘﺎﻟﻴﺔ‬

Page - 495
Internal Medicine - Endocrinology - Dyslipidemia

Question 73/162

Question #73

Which of the following is the most atherogenic?

a. High-density lipoprotein (HDL)


b. Very Low-density lipoprotein (VLDL)
c. Chylomicrons
d. Low-density lipoprotein (LDL) √
e. Triglycerides

Description

LDL is the leading cause of atherosclerosis


HDL is good lipoprotein and does not cause atherosclerosis

Page - 496
Internal Medicine - Endocrinology

Question 74/162

Question #74

Regarding dyslipidemia, all of the following are true except:

a. LDL is the most atherogenic


b. HMG CO-A inhibitors are the best for the treatment of High LDL levels and have a mortality
benefit in CAD-equivalent patients
c. Niacin is more vital than statins, exercise, and smoking cessation to raise HDL
d. Fibrates are used in combination with statin and are better for lowering Triglycerides level
e. Eruptive xanthoma is a specific skin lesion for familial hypercholesterolemia.

‫اﻹﺟﺎﺑﺔ ﻋﲆ اﻟﺼﻔﺤﺔ اﻟﺘﺎﻟﻴﺔ‬

Page - 497
Internal Medicine - Endocrinology - Dyslipidemia

Question 74/162

Question #74

Regarding dyslipidemia, all of the following are true except:

a. LDL is the most atherogenic


b. HMG CO-A inhibitors are the best for the treatment of High LDL levels and have a mortality
benefit in CAD-equivalent patients
c. Niacin is more vital than statins, exercise, and smoking cessation to raise HDL
d. Fibrates are used in combination with statin and are better for lowering Triglycerides level
e. Eruptive xanthoma is a specific skin lesion for familial hypercholesterolemia. √

Description

Eruptive xanthoma is a specific skin lesion for hypertriglyceridemia, not familial


hypercholesterolemia. In contrast, Tendinous xanthoma is typical for familial hypercholesterolemia.

Page - 498
Internal Medicine - Endocrinology

Question 75/162

Question #75

A patient was found to have the following laboratory tests:

Serum calcium: 11.8 mg/dL

Serum PTH: 97 ng/L

Urine Ca:creatinine rato is low

What is the most likely diagnosis?

a. Primary hyperparathyroidism
b. Secondary hyperparathyroidism
c. Familial hypocalciuric hypercalcemia (FHH)
d. Vitamin D excess
e. Pseudohypoparathyroidism

‫اﻹﺟﺎﺑﺔ ﻋﲆ اﻟﺼﻔﺤﺔ اﻟﺘﺎﻟﻴﺔ‬

Page - 499
Internal Medicine - Endocrinology - Familial hypocalciuric hypercalcemia (FHH)

Question 75/162

Question #75

A patient was found to have the following laboratory tests:

Serum calcium: 11.8 mg/dL

Serum PTH: 97 ng/L

Urine Ca:creatinine rato is low

What is the most likely diagnosis?

a. Primary hyperparathyroidism
b. Secondary hyperparathyroidism
c. Familial hypocalciuric hypercalcemia (FHH) √
d. Vitamin D excess
e. Pseudohypoparathyroidism

Description

High serum sodium, low urine sodium, and high parathyroid hormone are typical for two conditions:
FHH and primary hyperparathyroidism.

e main dierence between these two conditions is the urinary calcium:creatinine ratio which is
low in FHH and high in primary hyperparathyroidism.

e following table demonstrates the dierent lab investigations in the disorders of parathyroid
gland and calcium:

Page - 500
Page - 501
Internal Medicine - Endocrinology

Question 76/162

Question #76

A patient is found to have elevated both calcium and parathyroid levels. However, he was previously
healthy and had no significant medical history of note. Which of the following is an indication of
parathyroid surgery?

a. Patients age of 60-year-old


b. Presence of renal stones
c. Serum calcium of 10.8 mg/dL
d. Increased bone density
e. Visible adenoma on neck CT scan

‫اﻹﺟﺎﺑﺔ ﻋﲆ اﻟﺼﻔﺤﺔ اﻟﺘﺎﻟﻴﺔ‬

Page - 502
Internal Medicine - Endocrinology - Hypercalcemia

Question 76/162

Question #76

A patient is found to have elevated both calcium and parathyroid levels. However, he was previously
healthy and had no significant medical history of note. Which of the following is an indication of
parathyroid surgery?

a. Patients age of 60-year-old


b. Presence of renal stones √
c. Serum calcium of 10.8 mg/dL
d. Increased bone density
e. Visible adenoma on neck CT scan

Description

e indications for parathyroid surgery in primary hyperparathyroidism are:

e presence of kidney stones


Age less than 50-year-old
Serum calcium of 1 mg/dL (0.25 mmol/L) above the normal upper limit
Reduced bone density

Page - 503
Internal Medicine - Endocrinology

Question 77/162

Question #77

A 40-year-old female patient is treated for metastatic lung cancer with chemotherapy.
Unfortunately, she develops polyuria and confusion and is found to have hypercalcemia. What is the
first-line treatment in this clinical scenario?

a. Start bisphosphonates
b. Change the chemotherapy regimen
c. Normal saline and diuretics
d. Normal saline alone
e. Dextrose 5% in water solution

‫اﻹﺟﺎﺑﺔ ﻋﲆ اﻟﺼﻔﺤﺔ اﻟﺘﺎﻟﻴﺔ‬

Page - 504
Internal Medicine - Endocrinology - Hypercalcemia

Question 77/162

Question #77

A 40-year-old female patient is treated for metastatic lung cancer with chemotherapy.
Unfortunately, she develops polyuria and confusion and is found to have hypercalcemia. What is the
first-line treatment in this clinical scenario?

a. Start bisphosphonates
b. Change the chemotherapy regimen
c. Normal saline and diuretics
d. Normal saline alone √
e. Dextrose 5% in water solution

Description

Patients with hypercalcemia should initially receive normal saline intravenously, but you should be
aware not to exacerbate dehydration with diuretics.

Furosemide may be used in this case, but you should postpone its use until the patient is euvolemic
and out of dehydration

Treatment of hypercalcemia is as the following:

IV normal saline (first line)


Furosemide (Lasix) – use with caution because it may exacerbate dehydration
Calcitonin has a faster eect than Bisphosphonates
Bisphosphonates (inhibit bone resorption)
Surgical removal of the adenoma
Mithramycin (used to treatment of hypercalcemia in malignancy)

Steroids (treat hypercalcemia in sarcoidosis) 

Page - 505
Internal Medicine - Endocrinology

Question 78/162

Question #78

A 40-year-old female patient was referred to you because of hypercalcemia. Her medical history is
significant for chronic kidney disease. However, further investigations demonstrate a calcium level
of 2.8 mmol/L, PTH level of 79 ng/mL, Vitamin D of 70 nmol/L, and elevated urea and creatinine.
What is the most likely diagnosis?

a. Multiple myeloma
b. Familial hypocalciuric hypercalcemia
c. Secondary hyperparathyroidism
d. Tertiary hyperparathyroidism
e. Hypervitaminosis D

‫اﻹﺟﺎﺑﺔ ﻋﲆ اﻟﺼﻔﺤﺔ اﻟﺘﺎﻟﻴﺔ‬

Page - 506
Internal Medicine - Endocrinology - Hypercalcemia

Question 78/162

Question #78

A 40-year-old female patient was referred to you because of hypercalcemia. Her medical history is
significant for chronic kidney disease. However, further investigations demonstrate a calcium level
of 2.8 mmol/L, PTH level of 79 ng/mL, Vitamin D of 70 nmol/L, and elevated urea and creatinine.
What is the most likely diagnosis?

a. Multiple myeloma
b. Familial hypocalciuric hypercalcemia
c. Secondary hyperparathyroidism
d. Tertiary hyperparathyroidism √
e. Hypervitaminosis D

Description

Tertiary hyperparathyroidism develops in patients who have had secondary hyperparathyroidism


for a long time (typically due to CKD)

Page - 507
Internal Medicine - Endocrinology

Question 79/162

Question #79

e main mechanism of hypercalcemia development in sarcoidosis patients is:

a. Reduced renal calcium excretion


b. Parathormone-related peptide secretion (PTHrP)
c. Osteoclastic activation
d. Vitamin D activation
e. Parathyroid hormone release

‫اﻹﺟﺎﺑﺔ ﻋﲆ اﻟﺼﻔﺤﺔ اﻟﺘﺎﻟﻴﺔ‬

Page - 508
Internal Medicine - Endocrinology - Hypercalcemia

Question 79/162

Question #79

e main mechanism of hypercalcemia development in sarcoidosis patients is:

a. Reduced renal calcium excretion


b. Parathormone-related peptide secretion (PTHrP)
c. Osteoclastic activation
d. Vitamin D activation √
e. Parathyroid hormone release

Description

In sarcoidosis, the pathological granuloma contains macrophages that are responsible for
converting 25(OH)D3 to 1.25(OH)D3 (the active form), leading to hypercalcemia.

Page - 509
Internal Medicine - Endocrinology

Question 80/162

Question #80

A 40-year-old female patient was referred to you because she was incidentally found to have
hypercalcemia. Her further investigations demonstrate a calcium level of 2.8 mmol/L, PTH level of 79
ng/mL, Vitamin D of 70 nmol/L, and normal urea and creatinine. What is the most likely diagnosis?

a. Multiple myeloma
b. Familial hypocalciuric hypercalcemia
c. Secondary hyperparathyroidism
d. Tertiary hyperparathyroidism
e. Hypervitaminosis D

‫اﻹﺟﺎﺑﺔ ﻋﲆ اﻟﺼﻔﺤﺔ اﻟﺘﺎﻟﻴﺔ‬

Page - 510
Internal Medicine - Endocrinology - Hypercalcemia

Question 80/162

Question #80

A 40-year-old female patient was referred to you because she was incidentally found to have
hypercalcemia. Her further investigations demonstrate a calcium level of 2.8 mmol/L, PTH level of 79
ng/mL, Vitamin D of 70 nmol/L, and normal urea and creatinine. What is the most likely diagnosis?

a. Multiple myeloma
b. Familial hypocalciuric hypercalcemia √
c. Secondary hyperparathyroidism
d. Tertiary hyperparathyroidism
e. Hypervitaminosis D

Description

e correct answer is familial hypocalciuric hypercalcemia (FHH). e patient suers from


hypercalcemia and has a high PTH level. erefore, possible dierentials are primary
hyperparathyroidism, tertiary hyperparathyroidism, and familial hypocalciuric hypercalcemia.

Tertiary hyperparathyroidism develops in patients with secondary hyperparathyroidism for a long


time (typically due to CKD, which is absent here).

Primary hyperparathyroidism is not listed in the options above, so FHH is the most likely diagnosis.

Page - 511
Internal Medicine - Endocrinology

Question 81/162

Question #81

A 60-year-old female with a history of breast cancer and bone metastasis complains of excessive
thirst, polyuria, nausea, vomiting, and confusion. Which of the following is the most likely
explanation?

a. Brain metastasis
b. Hypercalcemia
c. Hypocalcemia
d. Hypernatremia
e. SIADH

‫اﻹﺟﺎﺑﺔ ﻋﲆ اﻟﺼﻔﺤﺔ اﻟﺘﺎﻟﻴﺔ‬

Page - 512
Internal Medicine - Endocrinology - Hypercalcemia

Question 81/162

Question #81

A 60-year-old female with a history of breast cancer and bone metastasis complains of excessive
thirst, polyuria, nausea, vomiting, and confusion. Which of the following is the most likely
explanation?

a. Brain metastasis
b. Hypercalcemia √
c. Hypocalcemia
d. Hypernatremia
e. SIADH

Description

Hypercalcemia is the most prevalent cancer-related paraneoplastic condition. Hypercalcemia


manifestations may develop when total blood calcium is more than 12 mg/dL (> 3 mmol/L).
Anorexia, GI regurgitation, vomiting, lethargy, convulsions, restlessness, hypertension, constipation,
dehydration, and feeding intolerance are known features of hypercalcemia

Page - 513
Internal Medicine - Endocrinology

Question 82/162

Question #82

You consulted regarding a 46-year-old male patient admitted to the ICU for confusion and lethargy.
His lab investigations demonstrate a serum creatinine of 1.1 mg/dL, serum protein of 7.3 g/dL, serum
calcium of 3.2 mmol/L, and serum albumin of 4.2 g/L. What is the most appropriate treatment at
this time?

a. Intravenous normal saline


b. Furosemide intravenously
c. Bisphosphonates
d. Mithramycin
e. Steroids

‫اﻹﺟﺎﺑﺔ ﻋﲆ اﻟﺼﻔﺤﺔ اﻟﺘﺎﻟﻴﺔ‬

Page - 514
Internal Medicine - Endocrinology - Hypercalcemia

Question 82/162

Question #82

You consulted regarding a 46-year-old male patient admitted to the ICU for confusion and lethargy.
His lab investigations demonstrate a serum creatinine of 1.1 mg/dL, serum protein of 7.3 g/dL, serum
calcium of 3.2 mmol/L, and serum albumin of 4.2 g/L. What is the most appropriate treatment at
this time?

a. Intravenous normal saline √


b. Furosemide intravenously
c. Bisphosphonates
d. Mithramycin
e. Steroids

Description

Patients with hypercalcemia should initially receive normal saline intravenously, but you should be
aware not to exacerbate dehydration with diuretics.

Furosemide may be used in this case, but you should postpone its use until the patient is euvolemic
and out of dehydration

Treatment of hypercalcemia is as the following:

IV normal saline (first line)


Furosemide (Lasix) – use with caution because it may exacerbate dehydration
Calcitonin has a faster eect than Bisphosphonates
Bisphosphonates (inhibit bone resorption)
Surgical removal of the adenoma
Mithramycin (used to treatment of hypercalcemia in malignancy)
Steroids (treat hypercalcemia in sarcoidosis)

Page - 515
Internal Medicine - Endocrinology

Question 83/162

Question #83

A 55-year-old male patient was found to have hypercalcemia on routine investigations. Additional
tests demonstrate elevated alkaline phosphatase and hypophosphatemia. Which of the following is
the most likely diagnosis?

a. Hypervitaminosis D
b. Parathyroid adenoma
c. Excessive calcium intake
d. Multiple myeloma
e. Sarcoidosis

‫اﻹﺟﺎﺑﺔ ﻋﲆ اﻟﺼﻔﺤﺔ اﻟﺘﺎﻟﻴﺔ‬

Page - 516
Internal Medicine - Endocrinology - Hypercalcemia

Question 83/162

Question #83

A 55-year-old male patient was found to have hypercalcemia on routine investigations. Additional
tests demonstrate elevated alkaline phosphatase and hypophosphatemia. Which of the following is
the most likely diagnosis?

a. Hypervitaminosis D
b. Parathyroid adenoma √
c. Excessive calcium intake
d. Multiple myeloma
e. Sarcoidosis

Description

is is most likely due to primary hyperparathyroidism.

e high PTH level will cause high bone turnover leading to hypercalcemia and elevated alkaline
phosphatase levels. However, hypophosphatemia may present.

Parathyroid hormone (PTH) causes phosphate loss through the kidneys.

Vitamin D promotes calcium and phosphorus absorption in the gastrointestinal tract; hence high
levels are linked to raised calcium and phosphorus levels

Sarcoidosis enhances calcium absorption by activating vitamin D, so the same situation as vitamin D
excess will result.

Multiple myeloma is associated with normal alkaline phosphatase levels.

Page - 517
Internal Medicine - Endocrinology

Question 84/162

Question #84

A 70-year-old male patient with lung cancer presents with excessive urination and lower back pain.
His ECG demonstrates a short QT interval. What is the best next step in management?

a. Spinal MRI
b. PET scan
c. DEXA scan
d. Alkaline phosphatase level
e. Serum calcium level

‫اﻹﺟﺎﺑﺔ ﻋﲆ اﻟﺼﻔﺤﺔ اﻟﺘﺎﻟﻴﺔ‬

Page - 518
Internal Medicine - Endocrinology - Hypercalcemia

Question 84/162

Question #84

A 70-year-old male patient with lung cancer presents with excessive urination and lower back pain.
His ECG demonstrates a short QT interval. What is the best next step in management?

a. Spinal MRI
b. PET scan
c. DEXA scan
d. Alkaline phosphatase level
e. Serum calcium level √

Description

e presence of polyuria and short QT interval in a cancer patient strongly suggests hypercalcemia

Serum calcium level is the best initial investigation.

Page - 519
Internal Medicine - Endocrinology

Question 85/162

Question #85

A patient complains of primary hyperparathyroidism. Which of the following is the most common
case?

a. Lithium therapy
b. Parathyroid hyperplasia
c. MEN type 1
d. Parathyroid adenoma
e. Pituitary adenoma

‫اﻹﺟﺎﺑﺔ ﻋﲆ اﻟﺼﻔﺤﺔ اﻟﺘﺎﻟﻴﺔ‬

Page - 520
Internal Medicine - Endocrinology - Hypercalcemia

Question 85/162

Question #85

A patient complains of primary hyperparathyroidism. Which of the following is the most common
case?

a. Lithium therapy
b. Parathyroid hyperplasia
c. MEN type 1
d. Parathyroid adenoma √
e. Pituitary adenoma

Description

In 85 % of instances, a solitary parathyroid gland adenoma is the cause of primary


hyperparathyroidism. Multiple adenomas and enlargement of all four glands are also common
causes. Multiple endocrine neoplasia and cancer are uncommon. Hyperparathyroidism can be
caused by lithium medication; however, this is not considered primary hyperparathyroidism.

Pituitary adenoma has no relation to hyperparathyroidism.

Page - 521
Internal Medicine - Endocrinology

Question 86/162

Question #86

A 55-year-old female presents with flank pain, and her ultrasound demonstrates a 4 mm ureteral
stone. In addition, she has a serum calcium level of 3.1 mmol/L. e patient is not taking any
medication and has a free medical history. What is the most appropriate investigation at this time?

a. Urinary calcium level


b. Repeat serum calcium in 4 weeks
c. Serum vitamin D level
d. Serum calcitonin level
e. Serum PTH level

‫اﻹﺟﺎﺑﺔ ﻋﲆ اﻟﺼﻔﺤﺔ اﻟﺘﺎﻟﻴﺔ‬

Page - 522
Internal Medicine - Endocrinology - Hypercalcemia

Question 86/162

Question #86

A 55-year-old female presents with flank pain, and her ultrasound demonstrates a 4 mm ureteral
stone. In addition, she has a serum calcium level of 3.1 mmol/L. e patient is not taking any
medication and has a free medical history. What is the most appropriate investigation at this time?

a. Urinary calcium level


b. Repeat serum calcium in 4 weeks
c. Serum vitamin D level
d. Serum calcitonin level
e. Serum PTH level √

Description

Primary hyperparathyroidism and malignancy account for 90% of the cases of hypercalcemia.

e best initial step here is to check for PTH level

Elevated PTH level indicates primary hyperparathyroidism.

Vitamin D is necessary if the serum PTH is low

Serum calcitonin level is not indicated in this scenario

Repeating serum calcium in 4 weeks without action is inappropriate because the patient has
symptomatic severe hypercalcemia.

Page - 523
Internal Medicine - Endocrinology

Question 87/162

Question #87

A patient develops hypercalcemia, and you suspect a medication side eect. Which of the following
is the most likely cause?

d. Amlodipine
e. Valsartan
a. Furosemide
b. Allopurinol
c. Hydrochlorothiazide

‫اﻹﺟﺎﺑﺔ ﻋﲆ اﻟﺼﻔﺤﺔ اﻟﺘﺎﻟﻴﺔ‬

Page - 524
Internal Medicine - Endocrinology - Hypercalcemia

Question 87/162

Question #87

A patient develops hypercalcemia, and you suspect a medication side eect. Which of the following
is the most likely cause?

d. Amlodipine
e. Valsartan
a. Furosemide
b. Allopurinol
c. Hydrochlorothiazide √

Description

iazides reduce calcium clearance in the kidneys by increasing calcium reabsorption in the distal
tubule.

Furosemide is a diuretic that lowers blood calcium levels and is used to treat hypercalcemia.

Page - 525
Internal Medicine - Endocrinology

Question 88/162

Question #88

A 50-year-old male patient complains of thirst and polyuria. His lab investigations demonstrate
hypercalcemia. However, his medical records are significant for recurrent calcium oxalate stones.
What is the most appropriate at this time?

a. Repeat serum calcium in 1 month


b. Skeletal survey
c. Brain CT scan
d. Parathyroid hormone level
e. yroid function test

‫اﻹﺟﺎﺑﺔ ﻋﲆ اﻟﺼﻔﺤﺔ اﻟﺘﺎﻟﻴﺔ‬

Page - 526
Internal Medicine - Endocrinology - Hypercalcemia

Question 88/162

Question #88

A 50-year-old male patient complains of thirst and polyuria. His lab investigations demonstrate
hypercalcemia. However, his medical records are significant for recurrent calcium oxalate stones.
What is the most appropriate at this time?

a. Repeat serum calcium in 1 month


b. Skeletal survey
c. Brain CT scan
d. Parathyroid hormone level √
e. yroid function test

Description

Primary hyperparathyroidism and malignancy account for 90% of the cases of hypercalcemia.

e best initial step here is to check for PTH level

Elevated PTH level indicates primary hyperparathyroidism.

Repeating serum calcium in 1 month without action is inappropriate because the patient has
symptomatic hypercalcemia.

Page - 527
Internal Medicine - Endocrinology

Question 89/162

Question #89

A 60-year-old male patient who takes multiple medications presents to your oce for a routine
check-up. His lab investigations were unremarkable except for serum calcium of 2.8 mmol/L and
PTH level of 82 pg/mL. What is the most likely cause of his condition?

a. Lithium use
b. Furosemide use
c. Calcium carbonate use
d. Vitamin D excess
e. Hydrochlorothiazide

‫اﻹﺟﺎﺑﺔ ﻋﲆ اﻟﺼﻔﺤﺔ اﻟﺘﺎﻟﻴﺔ‬

Page - 528
Internal Medicine - Endocrinology - Hypercalcemia

Question 89/162

Question #89

A 60-year-old male patient who takes multiple medications presents to your oce for a routine
check-up. His lab investigations were unremarkable except for serum calcium of 2.8 mmol/L and
PTH level of 82 pg/mL. What is the most likely cause of his condition?

a. Lithium use √
b. Furosemide use
c. Calcium carbonate use
d. Vitamin D excess
e. Hydrochlorothiazide

Description

is patient complains of high PTH and calcium levels which is most likely due to Lithium usage

Lithium increases serum calcium by enhancing PTH secretion.

Furosemide is known to cause hypocalcemia and is used to treat hypercalcemia

Calcium carbonate, vitamin D, and thiazides increase the serum calcium leading to secondary
hypoparathyroidism (high calcium has negative feedback and reduces the PTH)

Page - 529
Internal Medicine - Endocrinology

Question 90/162

Question #90

e initial treatment of hypercalcemia in malignancy should be which of the following?

a. Calcium gluconate
b. Magnesium sulfate
c. Mithramycin
d. Aggressive hydration
e. Furosemide

‫اﻹﺟﺎﺑﺔ ﻋﲆ اﻟﺼﻔﺤﺔ اﻟﺘﺎﻟﻴﺔ‬

Page - 530
Internal Medicine - Endocrinology - Hypercalcemia

Question 90/162

Question #90

e initial treatment of hypercalcemia in malignancy should be which of the following?

a. Calcium gluconate
b. Magnesium sulfate
c. Mithramycin
d. Aggressive hydration √
e. Furosemide

Description

Patients with hypercalcemia should initially receive normal saline intravenously, but you should be
aware not to exacerbate dehydration with diuretics.

Furosemide may be used in this case, but you should postpone its use until the patient is euvolemic
and out of dehydration

Treatment of hypercalcemia is as the following:

IV normal saline (first line)


Furosemide (Lasix) – use with caution because it may exacerbate dehydration
Calcitonin has a faster eect than Bisphosphonates
Bisphosphonates (inhibit bone resorption)
Surgical removal of the adenoma
Mithramycin (used to treatment of hypercalcemia in malignancy)
Steroids (treat hypercalcemia in sarcoidosis)

Page - 531
Internal Medicine - Endocrinology

Question 91/162

Question #91

A 55-year-old male patient is known to have multiple myeloma. He presents with confusion and
polyuria. What is the most likely electrolyte abnormality in his condition?

a. Hypocalcemia
b. Hypercalcemia
c. Hyperkalemia
d. Hypernatremia
e. Hyponatremia

‫اﻹﺟﺎﺑﺔ ﻋﲆ اﻟﺼﻔﺤﺔ اﻟﺘﺎﻟﻴﺔ‬

Page - 532
Internal Medicine - Endocrinology - Hypercalcemia

Question 91/162

Question #91

A 55-year-old male patient is known to have multiple myeloma. He presents with confusion and
polyuria. What is the most likely electrolyte abnormality in his condition?

a. Hypocalcemia
b. Hypercalcemia √
c. Hyperkalemia
d. Hypernatremia
e. Hyponatremia

Description

Osteoclastic activation is the main mechanism of hypercalcemia in multiple myeloma patients.

is patient complains of polyuria and confusion mostly secondary to hypercalcemia, and he should
receive normal saline initially

Page - 533
Internal Medicine - Endocrinology

Question 92/162

Question #92

A 40-year-old female complains of unexplained milk expression in her nipples. Her last menstrual
period was 9 months ago, and she complains of reduced sexual drive. What is the most likely
diagnosis?

a. Hyperaldosteronism
b. Pheochromocytoma
c. Hypercortisolism
d. Hyperprolactinemia
e. Hypoparathyroidism

‫اﻹﺟﺎﺑﺔ ﻋﲆ اﻟﺼﻔﺤﺔ اﻟﺘﺎﻟﻴﺔ‬

Page - 534
Internal Medicine - Endocrinology - Hyperprolactinemia

Question 92/162

Question #92

A 40-year-old female complains of unexplained milk expression in her nipples. Her last menstrual
period was 9 months ago, and she complains of reduced sexual drive. What is the most likely
diagnosis?

a. Hyperaldosteronism
b. Pheochromocytoma
c. Hypercortisolism
d. Hyperprolactinemia √
e. Hypoparathyroidism

Description

Hyperprolactinemia is an Increased level of PRL hormone.

Prolactin’s function is to stimulate the production of milk by breasts.

Prolactin is physiologically stimulated by stress, lactation, and nipple stimulation and suppressed by
dopamine and dopamine agonists.

Infertility, galactorrhea, amenorrhea, or sometimes heavy menstruation are typical features of this
condition.

Pregnancy should be ruled out if any female patient presents with amenorrhea, galactorrhea, or
both.

Plasma PRL level is the best initial test to perform if hyperprolactinemia is suspected

TFT, KFT, and LFT are used to estimate the cause, while pituitary imaging is only used to localize
pituitary adenoma if diagnosed.

Page - 535
e majority of patients are treated medically with dopamine agonists (cabergoline, bromocriptine)

Transsphenoidal surgical adenoma removal should be done for those who cannot tolerate or failed
medical treatment.

Page - 536
Internal Medicine - Endocrinology

Question 93/162

Question #93

A 20-year-old female develops secondary amenorrhea and milky nipple discharge. Her pregnancy
test is negative. What is the best initial investigation at this time?

a. Serum ADH level


b. Serum ACTH level
c. Growth hormone suppression test
d. Serum prolactin level
e. Pituitary MRI

‫اﻹﺟﺎﺑﺔ ﻋﲆ اﻟﺼﻔﺤﺔ اﻟﺘﺎﻟﻴﺔ‬

Page - 537
Internal Medicine - Endocrinology - Hyperprolactinemia

Question 93/162

Question #93

A 20-year-old female develops secondary amenorrhea and milky nipple discharge. Her pregnancy
test is negative. What is the best initial investigation at this time?

a. Serum ADH level


b. Serum ACTH level
c. Growth hormone suppression test
d. Serum prolactin level √
e. Pituitary MRI

Description

Hyperprolactinemia is an Increased level of PRL hormone.

Prolactin’s function is to stimulate the production of milk by breasts.

Prolactin is physiologically stimulated by stress, lactation, and nipple stimulation and suppressed by
dopamine and dopamine agonists.

Infertility, galactorrhea, amenorrhea, or sometimes heavy menstruation are typical features of this
condition.

Pregnancy should be ruled out if any female patient presents with amenorrhea, galactorrhea, or
both.

Plasma PRL level is the best initial test to perform if hyperprolactinemia is suspected

TFT, KFT, and LFT are used to estimate the cause, while pituitary imaging is only used to localize
pituitary adenoma if diagnosed.

e majority of patients are treated medically with dopamine agonists (cabergoline, bromocriptine)

Transsphenoidal surgical adenoma removal should be done for those who cannot tolerate or failed
medical treatment.

Page - 538
Internal Medicine - Endocrinology

Question 94/162

Question #94

A 32-year-old female patient complains of infertility, galactorrhea, and visual field defect. Her
medical history is significant for ESRD, hepatitis C, and hypothyroidism. However, her lab
investigations demonstrate high levels of prolactin hormone. Which of the following is the most
likely diagnosis?

a. Primary hypothyroidism
b. Pituitary adenoma
c. Renal impairment
d. Liver disease
e. Oral contraceptive pills use

‫اﻹﺟﺎﺑﺔ ﻋﲆ اﻟﺼﻔﺤﺔ اﻟﺘﺎﻟﻴﺔ‬

Page - 539
Internal Medicine - Endocrinology - Hyperprolactinemia

Question 94/162

Question #94

A 32-year-old female patient complains of infertility, galactorrhea, and visual field defect. Her
medical history is significant for ESRD, hepatitis C, and hypothyroidism. However, her lab
investigations demonstrate high levels of prolactin hormone. Which of the following is the most
likely diagnosis?

a. Primary hypothyroidism
b. Pituitary adenoma √
c. Renal impairment
d. Liver disease
e. Oral contraceptive pills use

Description

e most common cause is adenoma (called prolactinoma)

Other causes include:

Primary hypothyroidism (high TRH will stimulate PRL secretion)


Pregnancy (high level but opposed by estrogen)
Chronic renal failure, liver cirrhosis
PCOS
Drugs (OCP, estrogen, TCA, Haldol, metoclopramide, dopamine antagonists, verapamil)

While other choices are still possible, the presence of a visual field defect in this patient suggests
prolactinoma as the most likely diagnosis.

Hyperprolactinemia is an Increased level of PRL hormone.

Prolactin’s function is to stimulate the production of milk by breasts.

Page - 540
Prolactin is physiologically stimulated by stress, lactation, and nipple stimulation and suppressed by
dopamine and dopamine agonists.

Infertility, galactorrhea, amenorrhea, or sometimes heavy menstruation are typical features of this
condition.

Pregnancy should be ruled out if any female patient presents with amenorrhea, galactorrhea, or
both.

Plasma PRL level is the best initial test to perform if hyperprolactinemia is suspected

TFT, KFT, and LFT are used to estimate the cause, while pituitary imaging is only used to localize
pituitary adenoma if diagnosed.

e majority of patients are treated medically with dopamine agonists (cabergoline, bromocriptine)

Transsphenoidal surgical adenoma removal should be done for those who cannot tolerate or failed
medical treatment.

Page - 541
Internal Medicine - Endocrinology

Question 95/162

Question #95

A 40-year-old female was treated with cabergoline for prolactinoma for the past year, but she still
has no response. Recently she developed a visual field defect and complained of reduced libido.
What is the most appropriate treatment at this time?

a. Bromocriptine
b. Transsphenoidal surgical resection
c. Radiotherapy
d. Chemotherapy
e. Continue cabergoline

‫اﻹﺟﺎﺑﺔ ﻋﲆ اﻟﺼﻔﺤﺔ اﻟﺘﺎﻟﻴﺔ‬

Page - 542
Internal Medicine - Endocrinology - Hyperprolactinemia

Question 95/162

Question #95

A 40-year-old female was treated with cabergoline for prolactinoma for the past year, but she still
has no response. Recently she developed a visual field defect and complained of reduced libido.
What is the most appropriate treatment at this time?

a. Bromocriptine
b. Transsphenoidal surgical resection √
c. Radiotherapy
d. Chemotherapy
e. Continue cabergoline

Description

Transsphenoidal surgical adenoma removal should be done for those who cannot tolerate or failed
medical treatment.

Hyperprolactinemia is an Increased level of PRL hormone.

Prolactin’s function is to stimulate the production of milk by breasts.

Prolactin is physiologically stimulated by stress, lactation, and nipple stimulation and suppressed by
dopamine and dopamine agonists.

Infertility, galactorrhea, amenorrhea, or sometimes heavy menstruation are typical features of this
condition.

Pregnancy should be ruled out if any female patient presents with amenorrhea, galactorrhea, or
both.

Plasma PRL level is the best initial test to perform if hyperprolactinemia is suspected

Page - 543
TFT, KFT, and LFT are used to estimate the cause, while pituitary imaging is only used to localize
pituitary adenoma if diagnosed.

e majority of patients are treated medically with dopamine agonists (cabergoline, bromocriptine)

Page - 544
Internal Medicine - Endocrinology

Question 96/162

Question #96

A 30-year-old female presents with amenorrhea and galactorrhea. Her pituitary MRI shows a 1 cm
mass. What is the most appropriate management?

a. Cabergoline
b. Transsphenoidal surgical resection
c. Chemotherapy
d. Radiotherapy
e. Reassurance

‫اﻹﺟﺎﺑﺔ ﻋﲆ اﻟﺼﻔﺤﺔ اﻟﺘﺎﻟﻴﺔ‬

Page - 545
Internal Medicine - Endocrinology - Hyperprolactinemia

Question 96/162

Question #96

A 30-year-old female presents with amenorrhea and galactorrhea. Her pituitary MRI shows a 1 cm
mass. What is the most appropriate management?

a. Cabergoline √
b. Transsphenoidal surgical resection
c. Chemotherapy
d. Radiotherapy
e. Reassurance

Description

Hyperprolactinemia is an Increased level of PRL hormone.

Prolactin’s function is to stimulate the production of milk by breasts.

Prolactin is physiologically stimulated by stress, lactation, and nipple stimulation and suppressed by
dopamine and dopamine agonists.

Infertility, galactorrhea, amenorrhea, or sometimes heavy menstruation are typical features of this
condition.

Pregnancy should be ruled out if any female patient presents with amenorrhea, galactorrhea, or
both.

Plasma PRL level is the best initial test to perform if hyperprolactinemia is suspected

TFT, KFT, and LFT are used to estimate the cause, while pituitary imaging is only used to localize
pituitary adenoma if diagnosed.

e majority of patients are treated medically with dopamine agonists (cabergoline, bromocriptine)

Transsphenoidal surgical adenoma removal should be done for those who cannot tolerate or failed
medical treatment.

Page - 546
Internal Medicine - Endocrinology

Question 97/162

Question #97

A lady with Grave’s disease is seeking treatment for her condition. Which of the following has been
found to aggravate ophthalmopathy?

a. Propylthiouracil
b. Methimazole
c. Radioactive iodine ablation
d. yroidectomy
e. Propranolol

‫اﻹﺟﺎﺑﺔ ﻋﲆ اﻟﺼﻔﺤﺔ اﻟﺘﺎﻟﻴﺔ‬

Page - 547
Internal Medicine - Endocrinology - Hyperthyroidism

Question 97/162

Question #97

A lady with Grave’s disease is seeking treatment for her condition. Which of the following has been
found to aggravate ophthalmopathy?

a. Propylthiouracil
b. Methimazole
c. Radioactive iodine ablation √
d. yroidectomy
e. Propranolol

Description

When treated with radioactive iodine, Graves’ disease ophthalmopathy may flare up and worsen
initially.

Other mentioned treatments are not associated with this problem.

Page - 548
Internal Medicine - Endocrinology

Question 98/162

Question #98

A young lady has high T4, suppressed TSH, and positive thyroid-stimulating antibodies. What is the
most likely diagnosis?

a. Toxic multinodular goiter


b. Hashimoto’s thyroiditis
c. Grave’s disease
d. Subacute thyroiditis
e. yroid cancer

‫اﻹﺟﺎﺑﺔ ﻋﲆ اﻟﺼﻔﺤﺔ اﻟﺘﺎﻟﻴﺔ‬

Page - 549
Internal Medicine - Endocrinology - Hyperthyroidism

Question 98/162

Question #98

A young lady has high T4, suppressed TSH, and positive thyroid-stimulating antibodies. What is the
most likely diagnosis?

a. Toxic multinodular goiter


b. Hashimoto’s thyroiditis
c. Grave’s disease √
d. Subacute thyroiditis
e. yroid cancer

Description

Grave’s disease is an autoimmune condition resulting from thyroid-stimulating antibodies.

ese antibodies will stimulate the thyroid gland via the TSH receptors leading to increased
thyroxine production and thyrotoxicosis with eye signs and pretibial myxedema.

e thyroid gland may diusely be enlarged, and there is a diuse increased radioactive iodine
uptake.

Lab investigations will show suppressed TSH, high T3 and T4, and positive thyroid-stimulating
antibodies.

Page - 550
Internal Medicine - Endocrinology

Question 99/162

Question #99

A patient with thyrotoxicosis has her radioactive iodine uptake results, and it was reduced. Which of
the following is the most likely diagnosis?

a. Grave’s disease
b. Multinodular goiter
c. Solitary toxic thyroid nodule
d. Subacute thyroiditis
e. None of the above

‫اﻹﺟﺎﺑﺔ ﻋﲆ اﻟﺼﻔﺤﺔ اﻟﺘﺎﻟﻴﺔ‬

Page - 551
Internal Medicine - Endocrinology - Hyperthyroidism

Question 99/162

Question #99

A patient with thyrotoxicosis has her radioactive iodine uptake results, and it was reduced. Which of
the following is the most likely diagnosis?

a. Grave’s disease
b. Multinodular goiter
c. Solitary toxic thyroid nodule
d. Subacute thyroiditis √
e. None of the above

Description

Radioactive iodine uptake is elevated in Graves’ disease, multinodular goiter, and autonomously
functioning solitary thyroid nodule. In addition, it will be low in subacute thyroiditis, silent
thyroiditis, and exogenous thyroxine use.

e destructed cells in subacute thyroiditis will not be able to uptake iodine.

Subacute thyroiditis is mostly due to viral infection of the thyroid gland leading to cell destruction
and release of the preformed thyroxine and features of thyrotoxicosis initially, which will be later
transformed into hypothyroidism.

Page - 552
Internal Medicine - Endocrinology

Question 100/162

Question #100

A 33-year-old female patient presents with double vision when looking upward and painful eye
movement. On examination, the patient has lid lag, a noticeable welling of her eyelids, and
tachycardia. What are the most appropriate investigations to perform?

a. yroid function test


b. Liver function test
c. Brain MRI
d. Fundoscopic examination
e. Autoantibodies test

‫اﻹﺟﺎﺑﺔ ﻋﲆ اﻟﺼﻔﺤﺔ اﻟﺘﺎﻟﻴﺔ‬

Page - 553
Internal Medicine - Endocrinology - Hyperthyroidism

Question 100/162

Question #100

A 33-year-old female patient presents with double vision when looking upward and painful eye
movement. On examination, the patient has lid lag, a noticeable welling of her eyelids, and
tachycardia. What are the most appropriate investigations to perform?

a. yroid function test √


b. Liver function test
c. Brain MRI
d. Fundoscopic examination
e. Autoantibodies test

Description

is patient is most likely suering from Graves disease.

yroid function test showing suppressed TSH and high T3 and T4 is characteristic

e presence of thyroid-stimulating antibodies is specific for this condition.

Page - 554
Internal Medicine - Endocrinology

Question 101/162

Question #101

A 25-year-old female patient presents with chronic diarrhea and palpitation. Her lab investigations
demonstrate high T4 and very low TSH. What is the most eective treatment for relieving
symptoms?

a. Propranolol
b. Verapamil
c. Diltiazem
d. Terazosin
e. yroxin

‫اﻹﺟﺎﺑﺔ ﻋﲆ اﻟﺼﻔﺤﺔ اﻟﺘﺎﻟﻴﺔ‬

Page - 555
Internal Medicine - Endocrinology - Hyperthyroidism

Question 101/162

Question #101

A 25-year-old female patient presents with chronic diarrhea and palpitation. Her lab investigations
demonstrate high T4 and very low TSH. What is the most eective treatment for relieving
symptoms?

a. Propranolol √
b. Verapamil
c. Diltiazem
d. Terazosin
e. yroxin

Description

is patient suers from thyrotoxicosis (high T4 and low TSH)

Propranolol is important in preventing cardiovascular complications of hypothyroidism.

Propranolol will act as an antiarrhythmic agent to prevent the conversion of T4 to the active T3; it
will prevent tachycardia and has a role in treating thyroid storm.

Page - 556
Internal Medicine - Endocrinology

Question 102/162

Question #102

A 32-year-old female complains of palpitations, tachycardia, restlessness, tremor, and weight loss.
Her physical examination is remarkable for lid lag and pretibial myxedema. In addition, her
investigations demonstrate undetectable TSH and increased radioactive iodine uptake. What is the
most likely diagnosis?

a. Hashimoto’s thyroiditis
b. Grave’s disease
c. Multinodular goiter
d. Subacute thyroiditis
e. Pheochromocytoma

‫اﻹﺟﺎﺑﺔ ﻋﲆ اﻟﺼﻔﺤﺔ اﻟﺘﺎﻟﻴﺔ‬

Page - 557
Internal Medicine - Endocrinology - Hyperthyroidism

Question 102/162

Question #102

A 32-year-old female complains of palpitations, tachycardia, restlessness, tremor, and weight loss.
Her physical examination is remarkable for lid lag and pretibial myxedema. In addition, her
investigations demonstrate undetectable TSH and increased radioactive iodine uptake. What is the
most likely diagnosis?

a. Hashimoto’s thyroiditis
b. Grave’s disease √
c. Multinodular goiter
d. Subacute thyroiditis
e. Pheochromocytoma

Description

e typical clinical features of thyrotoxicosis and the presence of eye signs and pretibial myxedema
strongly suggest Grave’s disease

Undetectable TSH indicates negative feedback from the very high thyroxine level.

Diusely increased iodine uptake is typical for Grave’s disease as well.

Page - 558
Internal Medicine - Endocrinology

Question 103/162

Question #103

A 32-year-old female complains of palpitations, tachycardia, restlessness, tremor, and weight loss.
Her physical examination is remarkable for lid lag and pretibial myxedema. In addition, her
investigations demonstrate undetectable TSH and increased radioactive iodine uptake. e most
specific test to make the expected diagnosis is:

a. Free T3 level.
b. Free T4 level.
c. Serum TSH level.
d. Serum yroid Stimulating Immunoglobulin.
e. Serum Total T4 level.

‫اﻹﺟﺎﺑﺔ ﻋﲆ اﻟﺼﻔﺤﺔ اﻟﺘﺎﻟﻴﺔ‬

Page - 559
Internal Medicine - Endocrinology - Hyperthyroidism

Question 103/162

Question #103

A 32-year-old female complains of palpitations, tachycardia, restlessness, tremor, and weight loss.
Her physical examination is remarkable for lid lag and pretibial myxedema. In addition, her
investigations demonstrate undetectable TSH and increased radioactive iodine uptake. e most
specific test to make the expected diagnosis is:

a. Free T3 level.
b. Free T4 level.
c. Serum TSH level.
d. Serum yroid Stimulating Immunoglobulin. √
e. Serum Total T4 level.

Description

Grave’s disease is the most likely diagnosis because of the clinical features of thyrotoxicosis,
undetectable TSH, and eye signs.

yroid-stimulating antibodies are the most specific in this scenario.

Page - 560
Internal Medicine - Endocrinology

Question 104/162

Question #104

A 29-year-old female patient complains of heat intolerance, tachycardia, tremor, and nervousness.
Despite a good appetite, she loses weight. What are the most likely lab results to present in her
condition?

a. Reduced TSH and high T4


b. Reduced TSH and normal T4
c. High TSH and normal T4
d. High TSH and low T4
e. Normal TSH and normal T4

‫اﻹﺟﺎﺑﺔ ﻋﲆ اﻟﺼﻔﺤﺔ اﻟﺘﺎﻟﻴﺔ‬

Page - 561
Internal Medicine - Endocrinology - Hyperthyroidism

Question 104/162

Question #104

A 29-year-old female patient complains of heat intolerance, tachycardia, tremor, and nervousness.
Despite a good appetite, she loses weight. What are the most likely lab results to present in her
condition?

a. Reduced TSH and high T4 √


b. Reduced TSH and normal T4
c. High TSH and normal T4
d. High TSH and low T4
e. Normal TSH and normal T4

Description

Description:

is patient is most likely suering from thyrotoxicosis, mostly primary hyperthyroidism.

Primary hyperthyroidism manifested as high T4 and suppressed TSH.

e following table demonstrates the lab findings of dierent thyroid diseases:

Page - 562
Internal Medicine - Endocrinology

Question 105/162

Question #105

A 35-year-old female presents with a neck mass, weight loss, increased appetite, tachycardia, and
chronic diarrhea. Physical examination demonstrates exophthalmos, lid retraction, lid lag, and
pretibial myxedema. What is the most likely finding in the neck ultrasound of this patient?

a. yroid cyst
b. yroid mass
c. Diuse thyroid enlargement
d. Lymph node enlargement
e. Normal thyroid gland

‫اﻹﺟﺎﺑﺔ ﻋﲆ اﻟﺼﻔﺤﺔ اﻟﺘﺎﻟﻴﺔ‬

Page - 563
Internal Medicine - Endocrinology - Hyperthyroidism

Question 105/162

Question #105

A 35-year-old female presents with a neck mass, weight loss, increased appetite, tachycardia, and
chronic diarrhea. Physical examination demonstrates exophthalmos, lid retraction, lid lag, and
pretibial myxedema. What is the most likely finding in the neck ultrasound of this patient?

a. yroid cyst
b. yroid mass
c. Diuse thyroid enlargement √
d. Lymph node enlargement
e. Normal thyroid gland

Description

is is a case of Grave’s disease, and It is associated with diuse homogenous thyroid enlargement

Grave’s disease is an autoimmune condition resulting from thyroid-stimulating antibodies.

ese antibodies will stimulate the thyroid gland via the TSH receptors leading to increased
thyroxine production and thyrotoxicosis with eye signs and pretibial myxedema.

e thyroid gland may diusely be enlarged, and there is a diuse increased radioactive iodine
uptake.

Lab investigations will show suppressed TSH, high T3 and T4, and positive thyroid-stimulating
antibodies.

Page - 564
Internal Medicine - Endocrinology

Question 106/162

Question #106

A 30-year-old lady presents with palpitation and atrial fibrillation. Her medical history is
unremarkable, but she has complained of heat intolerance and weight loss for the past month. Her
lab investigations demonstrate high free T4, suppressed TSH, and low thyroglobulin levels. What is
the most likely diagnosis?

a. Primary hyperthyroidism
b. Hypothyroidism
c. Secondary hyperthyroidism
d. Factitious disorder
e. Subacute thyroiditis

‫اﻹﺟﺎﺑﺔ ﻋﲆ اﻟﺼﻔﺤﺔ اﻟﺘﺎﻟﻴﺔ‬

Page - 565
Internal Medicine - Endocrinology - Hyperthyroidism

Question 106/162

Question #106

A 30-year-old lady presents with palpitation and atrial fibrillation. Her medical history is
unremarkable, but she has complained of heat intolerance and weight loss for the past month. Her
lab investigations demonstrate high free T4, suppressed TSH, and low thyroglobulin levels. What is
the most likely diagnosis?

a. Primary hyperthyroidism
b. Hypothyroidism
c. Secondary hyperthyroidism
d. Factitious disorder √
e. Subacute thyroiditis

Description

e laboratory values in this patient show hyperthyroidism, but the presence of low thyroglobulin
strongly suggests that the patient is taking a high dose of exogenous thyroxine.

A low yroglobulin level is useful in patients with thyrotoxicosis caused by the surreptitious use of
thyroid hormone.

Page - 566
Internal Medicine - Endocrinology

Question 107/162

Question #107

A 30-year-old female complains of tremors, tachycardia, weight loss, increased appetite, and heat
intolerance. Which of the following is the main mechanism of these symptoms?

a. Reduced thyroid hormone level


b. Increased calcitonin level
c. Increased metabolic rate
d. Increased insulin resistance
e. Reduced caloric intake

‫اﻹﺟﺎﺑﺔ ﻋﲆ اﻟﺼﻔﺤﺔ اﻟﺘﺎﻟﻴﺔ‬

Page - 567
Internal Medicine - Endocrinology - Hyperthyroidism

Question 107/162

Question #107

A 30-year-old female complains of tremors, tachycardia, weight loss, increased appetite, and heat
intolerance. Which of the following is the main mechanism of these symptoms?

a. Reduced thyroid hormone level


b. Increased calcitonin level
c. Increased metabolic rate √
d. Increased insulin resistance
e. Reduced caloric intake

Description

is patient complains of features of thyrotoxicosis.

Hyperthyroidism is an increased thyroid gland activity, leading to excessive thyroid hormone,


resulting in a high metabolic rate.

A high metabolic rate is the main mechanism of action to cause these symptoms.

Page - 568
Internal Medicine - Endocrinology

Question 108/162

Question #108

A 33-year-old female patient is receiving methimazole for Graves disease treatment. She started on
this drug 1 month ago and presents now for a follow-up. Which of the following is the most
appropriate indicator of her response to treatment?

a. TSH level
b. Free T4 level
c. Free T3 level
d. Total T4 level
e. Total T3 level

‫اﻹﺟﺎﺑﺔ ﻋﲆ اﻟﺼﻔﺤﺔ اﻟﺘﺎﻟﻴﺔ‬

Page - 569
Internal Medicine - Endocrinology - Hyperthyroidism

Question 108/162

Question #108

A 33-year-old female patient is receiving methimazole for Graves disease treatment. She started on
this drug 1 month ago and presents now for a follow-up. Which of the following is the most
appropriate indicator of her response to treatment?

a. TSH level
b. Free T4 level √
c. Free T3 level
d. Total T4 level
e. Total T3 level

Description

Free T4 has the highest sensitivity to the activity of the thyroid gland.

It is the best indicator and the test of choice for titration doses of anti-thyroid medications.

TSH is an insensitive index of treatment response because it remains suppressed by thyroid


hormones for several months.

Note that TSH is the most sensitive in diagnosing hyperthyroidism but not the most sensitive in
following up the anti-thyroid doses.

Page - 570
Internal Medicine - Endocrinology

Question 109/162

Question #109

A 30-year-old female patient presents to the endocrine clinic with a history of thyrotoxicosis. Her
last menstrual period was 4 weeks ago, and her pregnancy test was discovered to be positive
yesterday. Which of the following is the most appropriate treatment option for her condition at this
time?

a. Carbimazole alone
b. Propylthiouracil alone
c. Radioactive iodine ablation
d. yroidectomy
e. Propranolol alone

‫اﻹﺟﺎﺑﺔ ﻋﲆ اﻟﺼﻔﺤﺔ اﻟﺘﺎﻟﻴﺔ‬

Page - 571
Internal Medicine - Endocrinology - Hyperthyroidism

Question 109/162

Question #109

A 30-year-old female patient presents to the endocrine clinic with a history of thyrotoxicosis. Her
last menstrual period was 4 weeks ago, and her pregnancy test was discovered to be positive
yesterday. Which of the following is the most appropriate treatment option for her condition at this
time?

a. Carbimazole alone
b. Propylthiouracil alone √
c. Radioactive iodine ablation
d. yroidectomy
e. Propranolol alone

Description

Because of its lower teratogenic profile, propylthiouracil (PTU) is the drug of choice in thyrotoxicosis
in women in their first trimester. In addition, because of its ecacy, it is the drug of choice in thyroid
storm.

Carbimazole is used in pregnancy aer the first trimester and in non-pregnant women because it is
eective and less hepatotoxic than PTU.

Propranolol alone will control symptoms, but it should not be used without anti-thyroid therapy.

yroidectomy is rarely performed in pregnancy, it is reserved for patients not responding to anti-
thyroid medication, but if necessary, it is preferable in the second trimester

Page - 572
Internal Medicine - Endocrinology

Question 110/162

Question #110

A patient complains of hyperthyroidism, and you suspect a drug-induced etiology. Which of the
following is the most likely cause?

a. Propranolol
b. Verapamil
c. Propylthiouracil
d. Amiodarone
e. Methimazole

‫اﻹﺟﺎﺑﺔ ﻋﲆ اﻟﺼﻔﺤﺔ اﻟﺘﺎﻟﻴﺔ‬

Page - 573
Internal Medicine - Endocrinology - Hyperthyroidism

Question 110/162

Question #110

A patient complains of hyperthyroidism, and you suspect a drug-induced etiology. Which of the
following is the most likely cause?

a. Propranolol
b. Verapamil
c. Propylthiouracil
d. Amiodarone √
e. Methimazole

Description

Amiodarone is a drug used in treating and preventing arrhythmias. Amiodarone is 37% iodine; it is
known to cause either hypothyroidism or hyperthyroidism

Around 1 in 6 patients taking amiodarone to develop thyroid dysfunction

Propranolol, PTU, and methimazole are used to treat hyperthyroidism

Page - 574
Internal Medicine - Endocrinology

Question 111/162

Question #111

A 33-year-old female patient was diagnosed with Grave’s disease and is at your oce today to
discuss radioactive iodine therapy. is therapy increases the risk of which of the following?

a. Hyperthyroidism
b. Hypothyroidism
c. yroid cancer
d. Inappropriate ADH secretion
e. Bleeding

‫اﻹﺟﺎﺑﺔ ﻋﲆ اﻟﺼﻔﺤﺔ اﻟﺘﺎﻟﻴﺔ‬

Page - 575
Internal Medicine - Endocrinology - Hyperthyroidism

Question 111/162

Question #111

A 33-year-old female patient was diagnosed with Grave’s disease and is at your oce today to
discuss radioactive iodine therapy. is therapy increases the risk of which of the following?

a. Hyperthyroidism
b. Hypothyroidism √
c. yroid cancer
d. Inappropriate ADH secretion
e. Bleeding

Description

Radioactive iodine ablation is a method to treat hyperthyroidism without exposing the patient to
surgical complications. However, hypothyroidism is the main complication of this therapy; it aects
up to 70% of patients in 10 years.

Patients may need thyroxine replacement aer radioactive iodine ablation.

Page - 576
Internal Medicine - Endocrinology

Question 112/162

Question #112

A 33-year-old female patient complains of hyperactivity, irritability, insomnia, heat intolerance,


sweating, and palpitations. In addition, she has a neck mass that is mobile with swallowing. Which
of the following is the most likely diagnosis?

a. Lymphoma
b. yroglossal cyst
c. yroid cyst
d. Toxic nodular goiter
e. yroid cancer

‫اﻹﺟﺎﺑﺔ ﻋﲆ اﻟﺼﻔﺤﺔ اﻟﺘﺎﻟﻴﺔ‬

Page - 577
Internal Medicine - Endocrinology - Hyperthyroidism

Question 112/162

Question #112

A 33-year-old female patient complains of hyperactivity, irritability, insomnia, heat intolerance,


sweating, and palpitations. In addition, she has a neck mass that is mobile with swallowing. Which
of the following is the most likely diagnosis?

a. Lymphoma
b. yroglossal cyst
c. yroid cyst
d. Toxic nodular goiter √
e. yroid cancer

Description

is patient complains of thyrotoxicosis symptoms, and she has a thyroid mass.

is mass is likely a goiter rather than a cancer or cyst because it is functioning and has no features
suggestive of malignancy.

Page - 578
Internal Medicine - Endocrinology

Question 113/162

Question #113

A 38-year-old female patient underwent thyroidectomy for thyroid cancer a week ago. She presents
with spasms, numbness, and paresthesia. What is the most likely diagnosis?

a. yroid storm
b. Hyperparathyroidism
c. Hypothyroidism
d. Hypocalcemia
e. Hypokalemia

‫اﻹﺟﺎﺑﺔ ﻋﲆ اﻟﺼﻔﺤﺔ اﻟﺘﺎﻟﻴﺔ‬

Page - 579
Internal Medicine - Endocrinology - Hypocalcemia

Question 113/162

Question #113

A 38-year-old female patient underwent thyroidectomy for thyroid cancer a week ago. She presents
with spasms, numbness, and paresthesia. What is the most likely diagnosis?

a. yroid storm
b. Hyperparathyroidism
c. Hypothyroidism
d. Hypocalcemia √
e. Hypokalemia

Description

Primary hypothyroidism and hypocalcemia are among the most common complications of thyroid
gland surgeries.

Disruption or removal of parathyroid glands will reduce PTH production and primary
hypoparathyroidism.

Page - 580
Internal Medicine - Endocrinology

Question 114/162

Question #114

A 55-year-old male patient with a long history of T2DM presents with a tingling sensation around his
mouth and a muscle twitching. His symptoms have been worsening recently. What is the most likely
biochemical finding in this patient?

a. Hyponatremia
b. Hypernatremia
c. Hypercalcemia
d. Hypocalcemia
e. Hypokalemia

‫اﻹﺟﺎﺑﺔ ﻋﲆ اﻟﺼﻔﺤﺔ اﻟﺘﺎﻟﻴﺔ‬

Page - 581
Internal Medicine - Endocrinology - Hypocalcemia

Question 114/162

Question #114

A 55-year-old male patient with a long history of T2DM presents with a tingling sensation around his
mouth and a muscle twitching. His symptoms have been worsening recently. What is the most likely
biochemical finding in this patient?

a. Hyponatremia
b. Hypernatremia
c. Hypercalcemia
d. Hypocalcemia √
e. Hypokalemia

Description

Tingling sensation around the mouth (perioral paresthesia) and muscle twitching are typical
straightforward indicators of hypocalcemia.

e history of DM is also related to hypocalcemia; it is mostly associated with a degree of renal


impairment that leads to hypocalcemia.

Page - 582
Internal Medicine - Endocrinology

Question 115/162

Question #115

A previously healthy 55-year-old female has her lab tests for a routine check-up. Her TSH level is 12
μU/mL, and her T4 was normal. What is the most likely diagnosis?

a. Normal readings
b. Primary Hypothyroidism
c. Primary Hyperthyroidism
d. Subclinical hypothyroidism
e. Central hyperthyroidism

‫اﻹﺟﺎﺑﺔ ﻋﲆ اﻟﺼﻔﺤﺔ اﻟﺘﺎﻟﻴﺔ‬

Page - 583
Internal Medicine - Endocrinology - Hypothyroidism

Question 115/162

Question #115

A previously healthy 55-year-old female has her lab tests for a routine check-up. Her TSH level is 12
μU/mL, and her T4 was normal. What is the most likely diagnosis?

a. Normal readings
b. Primary Hypothyroidism
c. Primary Hyperthyroidism
d. Subclinical hypothyroidism √
e. Central hyperthyroidism

Description

In a patient not known to have hypothyroidism and not treated with thyroxine, high TSH and
normal T4 indicate subclinical hypothyroidism. In this condition, there are no symptoms of
hypothyroidism.

e following table demonstrates the lab findings of dierent thyroid diseases:

Page - 584
Internal Medicine - Endocrinology

Question 116/162

Question #116

A 27-year-old female with bipolar disorder presents with cold intolerance, general weakness, and
decreased appetite. What is the most appropriate investigation at this time?

a. Complete blood count


b. C-Reactive protein
c. Free T3 level
d. Lithium level
e. Serum TSH level

‫اﻹﺟﺎﺑﺔ ﻋﲆ اﻟﺼﻔﺤﺔ اﻟﺘﺎﻟﻴﺔ‬

Page - 585
Internal Medicine - Endocrinology - Hypothyroidism

Question 116/162

Question #116

A 27-year-old female with bipolar disorder presents with cold intolerance, general weakness, and
decreased appetite. What is the most appropriate investigation at this time?

a. Complete blood count


b. C-Reactive protein
c. Free T3 level
d. Lithium level
e. Serum TSH level √

Description

Lithium is a mood stabilizer used to treat bipolar disorder and is known to cause hypothyroidism.

e patient complains of typical features of hypothyroidism, so you better start with serum TSH
level.

TSH level is the most appropriate, accurate, and cost-eective screening test for thyroid diseases.

Page - 586
Internal Medicine - Endocrinology

Question 117/162

Question #117

A 55-year-old female presents for routine investigations. Which of the following is the most
appropriate screening test for thyroid diseases?

a. Free T3
b. Free T4
c. Serum TSH level
d. Serum TRH level
e. yroglobulin level

‫اﻹﺟﺎﺑﺔ ﻋﲆ اﻟﺼﻔﺤﺔ اﻟﺘﺎﻟﻴﺔ‬

Page - 587
Internal Medicine - Endocrinology - Hypothyroidism

Question 117/162

Question #117

A 55-year-old female presents for routine investigations. Which of the following is the most
appropriate screening test for thyroid diseases?

a. Free T3
b. Free T4
c. Serum TSH level √
d. Serum TRH level
e. yroglobulin level

Description

Periodic screening for thyroid diseases by performing TSH level is indicated in all female patients
who are 50-year-old or older.

TSH level is the most appropriate, accurate, and cost-eective screening test for thyroid diseases.

Page - 588
Internal Medicine - Endocrinology

Question 118/162

Question #118

A 50-year-old female with a history of Sheehan’s syndrome is on levothyroxine replacement therapy


due to secondary hypothyroidism. She was at a GP clinic, and the doctor referred her to you because
he found her TSH level of 0.05 mIU/mL. What is the most appropriate action at this time?

a. Reduce the dose of thyroxine


b. Increase the dose of thyroxine
c. Repeat TSH aer 6 weeks
d. Order free T4 level
e. Order thyroglobulin level

‫اﻹﺟﺎﺑﺔ ﻋﲆ اﻟﺼﻔﺤﺔ اﻟﺘﺎﻟﻴﺔ‬

Page - 589
Internal Medicine - Endocrinology - Hypothyroidism

Question 118/162

Question #118

A 50-year-old female with a history of Sheehan’s syndrome is on levothyroxine replacement therapy


due to secondary hypothyroidism. She was at a GP clinic, and the doctor referred her to you because
he found her TSH level of 0.05 mIU/mL. What is the most appropriate action at this time?

a. Reduce the dose of thyroxine


b. Increase the dose of thyroxine
c. Repeat TSH aer 6 weeks
d. Order free T4 level √
e. Order thyroglobulin level

Description

In this scenario, the TSH level will not be a reliable indicator for hypothyroidism because of the
pituitary gland infarction; TSH will be low even if the thyroxine therapy is adequate, insucient, or
more than required.

e best indicator in this scenario is the free T4 level.

Page - 590
Internal Medicine - Endocrinology

Question 119/162

Question #119

A 52-year-old female patient complains of general weakness and low energy for the past 3 months.
On examination, delayed tendon reflexes, bradycardia, and cold, dry extremities are noted. Her labs
show high TSH and low free T4. What is the most likely agent to cause her symptoms?

a. Estrogen replacement therapy


b. Lithium
c. Valsartan
d. Metformin
e. Erythromycin

‫اﻹﺟﺎﺑﺔ ﻋﲆ اﻟﺼﻔﺤﺔ اﻟﺘﺎﻟﻴﺔ‬

Page - 591
Internal Medicine - Endocrinology - Hypothyroidism

Question 119/162

Question #119

A 52-year-old female patient complains of general weakness and low energy for the past 3 months.
On examination, delayed tendon reflexes, bradycardia, and cold, dry extremities are noted. Her labs
show high TSH and low free T4. What is the most likely agent to cause her symptoms?

a. Estrogen replacement therapy


b. Lithium √
c. Valsartan
d. Metformin
e. Erythromycin

Description

High TSH, low free T4, and the clinical scenario in this patient are secondary to hypothyroidism.
Lithium is a mode stabilizer used in Bipolar disorder; it is well known to cause hypothyroidism.
erefore, thyroid function tests should be followed closely with patients taking lithium.

Page - 592
Internal Medicine - Endocrinology

Question 120/162

Question #120

A 32-year-old female patient complains of chronic fatigue, cold sensitivity, anorexia, weight gain,
and reduced appetite. Her TSH is elevated, and her T4 is low. What is the most appropriate
treatment?

a. Propranolol
b. Radioactive iodine ablation
c. yroxine
d. Methimazole
e. Propylthiouracil

‫اﻹﺟﺎﺑﺔ ﻋﲆ اﻟﺼﻔﺤﺔ اﻟﺘﺎﻟﻴﺔ‬

Page - 593
Internal Medicine - Endocrinology - Hypothyroidism

Question 120/162

Question #120

A 32-year-old female patient complains of chronic fatigue, cold sensitivity, anorexia, weight gain,
and reduced appetite. Her TSH is elevated, and her T4 is low. What is the most appropriate
treatment?

a. Propranolol
b. Radioactive iodine ablation
c. yroxine √
d. Methimazole
e. Propylthiouracil

Description

High TSH, low T4, and the mentioned clinical manifestations are typical for hypothyroidism.

Hypothyroidism is an inability of the thyroid gland to maintain the body’s requirement of thyroxin
hormone.

yroxine replacement is the treatment of choice for this condition.

Page - 594
Internal Medicine - Endocrinology

Question 121/162

Question #121

A 30-year-old female complains of weight gain, low energy, and constipation. Which of the
following lab findings fit with the diagnosis of primary hypothyroidism?

a. High TSH and high T4


b. High TSH and low T4
c. High TSH and normal T4
d. Low TSH and high T4
e. Low TSH and low T4

‫اﻹﺟﺎﺑﺔ ﻋﲆ اﻟﺼﻔﺤﺔ اﻟﺘﺎﻟﻴﺔ‬

Page - 595
Internal Medicine - Endocrinology - Hypothyroidism

Question 121/162

Question #121

A 30-year-old female complains of weight gain, low energy, and constipation. Which of the
following lab findings fit with the diagnosis of primary hypothyroidism?

a. High TSH and high T4


b. High TSH and low T4 √
c. High TSH and normal T4
d. Low TSH and high T4
e. Low TSH and low T4

Description

High TSH, low T4, and the mentioned clinical manifestations are typical for hypothyroidism.

Hypothyroidism is an inability of the thyroid gland to maintain the body’s requirement of thyroxin
hormone.

e following table demonstrates the lab findings of dierent thyroid diseases:

Page - 596
Internal Medicine - Endocrinology

Question 122/162

Question #122

A 55-year-old female patient complains of constipation for 6 months. on further questioning, she
complains of fatigue, feels tired, has significant weight gain, and cannot tolerate cold weather.
Which of the following is relevant to this patient’s condition?

a. Elevated TRH
b. Elevated thyroglobulin level
c. Elevated TSH level
d. Elevated TSH receptor-stimulating antibodies
e. Hypercalcemia

‫اﻹﺟﺎﺑﺔ ﻋﲆ اﻟﺼﻔﺤﺔ اﻟﺘﺎﻟﻴﺔ‬

Page - 597
Internal Medicine - Endocrinology - Hypothyroidism

Question 122/162

Question #122

A 55-year-old female patient complains of constipation for 6 months. on further questioning, she
complains of fatigue, feels tired, has significant weight gain, and cannot tolerate cold weather.
Which of the following is relevant to this patient’s condition?

a. Elevated TRH
b. Elevated thyroglobulin level
c. Elevated TSH level √
d. Elevated TSH receptor-stimulating antibodies
e. Hypercalcemia

Description

Description:

is patient mostly suers from primary hypothyroidism.

TSH level is the most appropriate, accurate, and cost-eective in the screening for thyroid diseases;
in primary hypothyroidism, it should be elevated.

yroglobulin levels are reduced in hypothyroidism.

TSH receptor stimulating antibodies are associated with Grave’s disease (hyperthyroidism)

Hypercalcemia and bone resorption results from hyperthyroidism, not hypothyroidism

Page - 598
Internal Medicine - Endocrinology

Question 123/162

Question #123

What is the most common cause of hypothyroidism?

a. Endemic iodine deficiency


b. Graves disease
c. Hashimoto thyroiditis
d. yroid agenesis/dysgenesis
e. Drug induced

‫اﻹﺟﺎﺑﺔ ﻋﲆ اﻟﺼﻔﺤﺔ اﻟﺘﺎﻟﻴﺔ‬

Page - 599
Internal Medicine - Endocrinology - Hypothyroidism

Question 123/162

Question #123

What is the most common cause of hypothyroidism?

a. Endemic iodine deficiency


b. Graves disease
c. Hashimoto thyroiditis √
d. yroid agenesis/dysgenesis
e. Drug induced

Description

Hashimoto thyroiditis is the most common cause of hypothyroidism. It is an autoimmune disease in


which anti-TPO antibodies attacks the thyroid gland leading to subclinical hypothyroidism and then
clinical hypothyroidism.

Endemic iodine deficiency was the most common in the past but today the iodine was added to salt
and the condition became less common.

Graves disease is the most common cause of hyperthyroidism, not hypothyroidism.

yroid agenesis/dysgenesis are the most common cause of cretinism (congenital hypothyroidism).

Page - 600
Internal Medicine - Endocrinology

Question 124/162

Question #124

A 29-year-old female patient is treated with daily 100 mcg of levothyroxine for primary
hypothyroidism. She presented to you with amenorrhea for 2 days, and you ordered a pregnancy
test that returned positive. Her TSH level is 2.1 μU/mL. What would you do regarding her
levothyroxine dose?

a. Increase the dose to 150 mic/day


b. Reduce the dose to 50 mic/day
c. Keep the same dose, and no changes are indicated
d. Hold levothyroxine during the first trimester
e. Hold levothyroxine and resume aer delivery

‫اﻹﺟﺎﺑﺔ ﻋﲆ اﻟﺼﻔﺤﺔ اﻟﺘﺎﻟﻴﺔ‬

Page - 601
Internal Medicine - Endocrinology - Hypothyroidism

Question 124/162

Question #124

A 29-year-old female patient is treated with daily 100 mcg of levothyroxine for primary
hypothyroidism. She presented to you with amenorrhea for 2 days, and you ordered a pregnancy
test that returned positive. Her TSH level is 2.1 μU/mL. What would you do regarding her
levothyroxine dose?

a. Increase the dose to 150 mic/day √


b. Reduce the dose to 50 mic/day
c. Keep the same dose, and no changes are indicated
d. Hold levothyroxine during the first trimester
e. Hold levothyroxine and resume aer delivery

Description

yroxine requirements increase by 30 – 50% during pregnancy. So, you should increase the dose of
thyroxine in this patient.

Levothyroxine is not teratogenic and should never be stopped during pregnancy because this will
aect fetal physical and brain development.

Page - 602
Internal Medicine - Endocrinology

Question 125/162

Question #125

A 40-year-old female patient complains of amenorrhea and galactorrhea. On further questioning,


she told you that she has chronic constipation, felt fatigued, and has been gaining weight for the
past 6 months. In addition, physical examination demonstrates dry skin, bradycardia, and loss of the
outer one-third of her eyebrows. Lab tests show hyperprolactinemia and a negative pregnancy test.
Which of the following is the most likely cause of her hyperprolactinemia?

a. Hypothyroidism
b. Pregnancy
c. Oral contraceptive use
d. Stress
e. Prolactinoma

‫اﻹﺟﺎﺑﺔ ﻋﲆ اﻟﺼﻔﺤﺔ اﻟﺘﺎﻟﻴﺔ‬

Page - 603
Internal Medicine - Endocrinology - Hypothyroidism

Question 125/162

Question #125

A 40-year-old female patient complains of amenorrhea and galactorrhea. On further questioning,


she told you that she has chronic constipation, felt fatigued, and has been gaining weight for the
past 6 months. In addition, physical examination demonstrates dry skin, bradycardia, and loss of the
outer one-third of her eyebrows. Lab tests show hyperprolactinemia and a negative pregnancy test.
Which of the following is the most likely cause of her hyperprolactinemia?

a. Hypothyroidism √
b. Pregnancy
c. Oral contraceptive use
d. Stress
e. Prolactinoma

Description

Hypothyroidism is a well-known cause of hyperprolactinemia because of the high TRH level that
stimulates prolactin hormone secretion.

Dry skin, bradycardia, chronic constipation, fatigue, and weight gain are typical hypothyroidism
features that are the most likely in this scenario.

In the absence of hypothyroidism symptoms, choose pituitary adenoma (prolactinoma) as the most
likely cause of hyperprolactinemia.

Page - 604
Internal Medicine - Endocrinology

Question 126/162

Question #126

A postmenopausal woman has been treated for hypothyroidism for years but has not been
investigated for many years. Her lab investigations today demonstrate suppressed TSH. Which
complications may result from chronic excess thyroid hormone replacement in this patient?

a. Diuse toxic goiter


b. Osteoarthritis
c. Osteoporosis
d. Hyperprolactinemia
e. Hyperparathyroidism

‫اﻹﺟﺎﺑﺔ ﻋﲆ اﻟﺼﻔﺤﺔ اﻟﺘﺎﻟﻴﺔ‬

Page - 605
Internal Medicine - Endocrinology - Hypothyroidism

Question 126/162

Question #126

A postmenopausal woman has been treated for hypothyroidism for years but has not been
investigated for many years. Her lab investigations today demonstrate suppressed TSH. Which
complications may result from chronic excess thyroid hormone replacement in this patient?

a. Diuse toxic goiter


b. Osteoarthritis
c. Osteoporosis √
d. Hyperprolactinemia
e. Hyperparathyroidism

Description

Bone resorption can result from even mild chronic excessive thyroxine replacement in
postmenopausal women. erefore, elevated serum calcium is expected in this clinical scenario.

Page - 606
Internal Medicine - Endocrinology

Question 127/162

Question #127

A 40-year-old female patient complains of weight gain, constipation, cold intolerance, and
bradycardia of 50 beats per minute. What is the most likely underlying mechanism?

a. Infarction
b. Adenoma
c. Autoimmune process
d. Infection
e. Nutritional condition

‫اﻹﺟﺎﺑﺔ ﻋﲆ اﻟﺼﻔﺤﺔ اﻟﺘﺎﻟﻴﺔ‬

Page - 607
Internal Medicine - Endocrinology - Hypothyroidism

Question 127/162

Question #127

A 40-year-old female patient complains of weight gain, constipation, cold intolerance, and
bradycardia of 50 beats per minute. What is the most likely underlying mechanism?

a. Infarction
b. Adenoma
c. Autoimmune process √
d. Infection
e. Nutritional condition

Description

is is a typical clinical scenario of hypothyroidism.

e most common cause of hypothyroidism in Jordan is Hashimoto’s thyroiditis.

Note that iodine deficiency was the most common in the past, but now the salt fortified with iodine
makes it less prevalent, and Hashimoto’s has become the most common.

Hashimoto’s thyroiditis is an autoimmune process against the thyroid tissue. It results from Anti-
TPO antibodies.

Pituitary infection, infarction, or adenoma are other possible causes of central (secondary)
hypothyroidism.

yroid infection (e.g., subacute thyroiditis is another possible cause.

Nutritional iodine deficiency is a possible cause, but Hashimoto’s thyroiditis is still the most
common and most likely in this case.

Page - 608
Internal Medicine - Endocrinology

Question 128/162

Question #128

A 34-year-old female complains of chronic fatigue and weight gain. Her physical examination shows
delayed deep tendon reflexes, and her lab investigations show elevated thyroid-stimulating
hormone. What is the most likely diagnosis?

a. Primary hyperthyroidism
b. Secondary hyperthyroidism
c. Hypothyroidism
d. Subacute thyroiditis
e. Chronic fatigue syndrome

‫اﻹﺟﺎﺑﺔ ﻋﲆ اﻟﺼﻔﺤﺔ اﻟﺘﺎﻟﻴﺔ‬

Page - 609
Internal Medicine - Endocrinology - Hypothyroidism

Question 128/162

Question #128

A 34-year-old female complains of chronic fatigue and weight gain. Her physical examination shows
delayed deep tendon reflexes, and her lab investigations show elevated thyroid-stimulating
hormone. What is the most likely diagnosis?

a. Primary hyperthyroidism
b. Secondary hyperthyroidism
c. Hypothyroidism √
d. Subacute thyroiditis
e. Chronic fatigue syndrome

Description

Delayed deep tendon reflex is the most important bedside test in hypothyroidism.

e presence of fatigue, weight gain, and high TSH level strongly suggests the diagnosis of primary
hypothyroidism.

Page - 610
Internal Medicine - Endocrinology

Question 129/162

Question #129

In a 52-year-old female patient, A 3-cm adrenal gland mass was accidentally discovered in a CT scan
performed to evaluate suspected renal stones and hematuria. e patient has no medical history
except for her flank pain and hematuria for the past 2 weeks. Her physical examinations are
unremarkable, and her lab investigations show normal urea, electrolytes, and kidney function. What
is the most appropriate at this time?

a. 1 mg dexamethasone suppression test and 24 hours urinary metanephrine


b. Repeat the abdominal CT scan aer 6 – 12 months
c. Fine needle aspiration of the mass
d. Surgical removal
e. Reassurance and discharge home

‫اﻹﺟﺎﺑﺔ ﻋﲆ اﻟﺼﻔﺤﺔ اﻟﺘﺎﻟﻴﺔ‬

Page - 611
Internal Medicine - Endocrinology - Incidentaloma

Question 129/162

Question #129

In a 52-year-old female patient, A 3-cm adrenal gland mass was accidentally discovered in a CT scan
performed to evaluate suspected renal stones and hematuria. e patient has no medical history
except for her flank pain and hematuria for the past 2 weeks. Her physical examinations are
unremarkable, and her lab investigations show normal urea, electrolytes, and kidney function. What
is the most appropriate at this time?

a. 1 mg dexamethasone suppression test and 24 hours urinary metanephrine √


b. Repeat the abdominal CT scan aer 6 – 12 months
c. Fine needle aspiration of the mass
d. Surgical removal
e. Reassurance and discharge home

Description

Description:

In patients with adrenal incidentaloma, you should perform a 1 mg dexamethasone suppression test
and 24 hours urinary Metanephrine levels for all patients.

Surgical removal should be done if the gland is > 4 cm in diameter (because of the risk of
malignancy)

If the gland is less than 4 cm in diameter and non-functioning, you should repeat the abdominal CT
scan aer 6 – 12 months (only once).

For the pituitary incidentaloma, you should test for PRL level, IGF-1 level, and cortisol excess.

Page - 612
Internal Medicine - Endocrinology

Question 130/162

Question #130

A 36-year-old male patient complains of recurrent extensive peptic ulcer disease. His lab tests
demonstrate elevated gastrin and hypercalcemia. Which of the following is the most appropriate
test to establish the diagnosis of Multiple endocrine neoplasia?

a. Abdominal CT scan
b. Head MRI
c. Upper GI endoscopy
d. Urinary metanephrines
e. Serum thyroid function test

‫اﻹﺟﺎﺑﺔ ﻋﲆ اﻟﺼﻔﺤﺔ اﻟﺘﺎﻟﻴﺔ‬

Page - 613
Internal Medicine - Endocrinology - Multiple Endocrine Neoplasias (MEN)

Question 130/162

Question #130

A 36-year-old male patient complains of recurrent extensive peptic ulcer disease. His lab tests
demonstrate elevated gastrin and hypercalcemia. Which of the following is the most appropriate
test to establish the diagnosis of Multiple endocrine neoplasia?

a. Abdominal CT scan
b. Head MRI √
c. Upper GI endoscopy
d. Urinary metanephrines
e. Serum thyroid function test

Description

e presence of pancreatic tumor (Gastrinoma) and the parathyroid adenoma that leads to
hypercalcemia is characteristic of MEN type 1

e missing piece of the MEN type 1 is the pituitary gland tumor which is better investigated by
pituitary MRI. erefore, a head MRI is the most appropriate answer.

Page - 614
Page - 615
Internal Medicine - Endocrinology

Question 131/162

Question #131

e presence of primary hyperparathyroidism, pheochromocytoma, and medullary thyroid cancer is


characteristic of which of the following diagnoses?

a. Multiple endocrine neoplasia type I


b. Multiple endocrine neoplasia type IIa
c. Multiple endocrine neoplasia type IIb
d. Multiple myeloma
e. Sarcoidosis

‫اﻹﺟﺎﺑﺔ ﻋﲆ اﻟﺼﻔﺤﺔ اﻟﺘﺎﻟﻴﺔ‬

Page - 616
Internal Medicine - Endocrinology - Multiple Endocrine Neoplasias (MEN)

Question 131/162

Question #131

e presence of primary hyperparathyroidism, pheochromocytoma, and medullary thyroid cancer is


characteristic of which of the following diagnoses?

a. Multiple endocrine neoplasia type I


b. Multiple endocrine neoplasia type IIa √
c. Multiple endocrine neoplasia type IIb
d. Multiple myeloma
e. Sarcoidosis

Description

Multiple Endocrine Neoplasia (MEN) is an inherited (Autosomal dominant) rare syndrome in which
multiple endocrine glands get neoplasia.

e following are the types of MEN:

Page - 617
Internal Medicine - Endocrinology

Question 132/162

Question #132

A 30-year-old female sustained a motor vehicle accident and was admitted to the ICU. His lab
investigations are normal except for low serum T3. However, his TSH and T4 levels are normal. What
is the most appropriate management?

a. Start intravenous T3
b. Prescribe oral T4
c. Radioactive iodine uptake thyroid scan
d. yroglobulin level
e. Observation and monitoring

‫اﻹﺟﺎﺑﺔ ﻋﲆ اﻟﺼﻔﺤﺔ اﻟﺘﺎﻟﻴﺔ‬

Page - 618
Internal Medicine - Endocrinology - Non-thyroidal illness

Question 132/162

Question #132

A 30-year-old female sustained a motor vehicle accident and was admitted to the ICU. His lab
investigations are normal except for low serum T3. However, his TSH and T4 levels are normal. What
is the most appropriate management?

a. Start intravenous T3
b. Prescribe oral T4
c. Radioactive iodine uptake thyroid scan
d. yroglobulin level
e. Observation and monitoring √

Description

Sick euthyroid syndrome (also known as Non-thyroidal illness):

Any acute, severe illness can aect the circulating TSH and thyroid hormones in the absence of
any thyroid disease
Unless a thyroid disorder is strongly suspected, routine thyroid function testing should be
avoided in acutely ill patients.

Lab investigations:

Low T3 syndrome: Low free T3 with normal T4 and TSH (the most common pattern)
Low T4 syndrome: reduced T3 and T4 due to accelerated consumption.
TSH may be reduced, normal or increased

Treatment:

Changes are reversible upon recovery from the systemic illness; hence, no treatment is usually
needed.

Page - 619
Internal Medicine - Endocrinology

Question 133/162

Question #133

A 60-year-old male patient is admitted to the medical ICU for sepsis. His history is significant for
HTN and T2DM. his lab investigations demonstrate a TSH level of 0.19 microunits/mL, free T4 of 0.3
ng/dL. what is the most likely explanation?

a. Secondary hypothyroidism
b. Hyperthyroidism
c. Sick euthyroid syndrome
d. Subacute thyroiditis
e. Grave’s disease

‫اﻹﺟﺎﺑﺔ ﻋﲆ اﻟﺼﻔﺤﺔ اﻟﺘﺎﻟﻴﺔ‬

Page - 620
Internal Medicine - Endocrinology - Non-thyroidal illness

Question 133/162

Question #133

A 60-year-old male patient is admitted to the medical ICU for sepsis. His history is significant for
HTN and T2DM. his lab investigations demonstrate a TSH level of 0.19 microunits/mL, free T4 of 0.3
ng/dL. what is the most likely explanation?

a. Secondary hypothyroidism
b. Hyperthyroidism
c. Sick euthyroid syndrome √
d. Subacute thyroiditis
e. Grave’s disease

Description

Sick euthyroid syndrome (also known as Non-thyroidal illness):

Any acute, severe illness can aect the circulating TSH and thyroid hormones in the absence of
any thyroid disease
Unless a thyroid disorder is strongly suspected, routine thyroid function testing should be
avoided in acutely ill patients.

Lab investigations:

Low T3 syndrome: Low free T3 with normal T4 and TSH (the most common pattern)
Low T4 syndrome: reduced T3 and T4 due to accelerated consumption.
TSH may be reduced, normal or increased

Treatment:

Changes are reversible upon recovery from the systemic illness; hence, no treatment is usually
needed.

Page - 621
Internal Medicine - Endocrinology

Question 134/162

Question #134

A patient is suspected to have primary hyperparathyroidism, which of the following suggests the
disease?

a. High serum alkaline phosphatase


b. Serum calcium level above 11 mg/dL
c. Parathyroid hormone less than 10 pg/mL
d. High urinary calcium level
e. Low urine Ca:creatinine ratio

‫اﻹﺟﺎﺑﺔ ﻋﲆ اﻟﺼﻔﺤﺔ اﻟﺘﺎﻟﻴﺔ‬

Page - 622
Internal Medicine - Endocrinology - Parathyroid gland

Question 134/162

Question #134

A patient is suspected to have primary hyperparathyroidism, which of the following suggests the
disease?

a. High serum alkaline phosphatase


b. Serum calcium level above 11 mg/dL √
c. Parathyroid hormone less than 10 pg/mL
d. High urinary calcium level
e. Low urine Ca:creatinine ratio

Description

Hyperparathyroidism:

It can be primary, secondary


Primary: adenoma, hyperplasia, or carcinoma of the parathyroid gland
Secondary: high PTH in response to low serum calcium (CRF, malabsorption, rickets…)
Primary hyperparathyroidism:
Features of hypercalcemia
Bone pain and pathological fractures
Secondary hyperparathyroidism:
Features of hypocalcemia

e following table demonstrates the dierential diagnosis of parathyroid disorders

Page - 623
Page - 624
Internal Medicine - Endocrinology

Question 135/162

Question #135

A 60-year-old female presents with paroxysmal hypertension associated with headaches, sweating,
and tremors. His blood pressure is 160/97 mmHg, and his pulse rate is 103 bpm. He told you that he
has a strong family history of thyroid cancer. What is the most appropriate investigation?

a. 24 hours urinary collection for metanephrines


b. Adrenal gland CT scan
c. Brian CT scan
d. Pituitary CT scan
e. Serum vanillylmandelic acid

‫اﻹﺟﺎﺑﺔ ﻋﲆ اﻟﺼﻔﺤﺔ اﻟﺘﺎﻟﻴﺔ‬

Page - 625
Internal Medicine - Endocrinology - Pheochromocytoma

Question 135/162

Question #135

A 60-year-old female presents with paroxysmal hypertension associated with headaches, sweating,
and tremors. His blood pressure is 160/97 mmHg, and his pulse rate is 103 bpm. He told you that he
has a strong family history of thyroid cancer. What is the most appropriate investigation?

a. 24 hours urinary collection for metanephrines √


b. Adrenal gland CT scan
c. Brian CT scan
d. Pituitary CT scan
e. Serum vanillylmandelic acid

Description

is is a typical scenario of pheochromocytoma as a part of MEN type 2.

24-hour urine collection for Metanephrine is the most specific test for pheochromocytoma.

Vanillylmandelic acid is the least specific degradation product of epinephrine and norepinephrine.

Pheochromocytoma is a tumor of the medulla of the adrenal gland that secretes epinephrine and
norepinephrine,

It can be associated with other conditions like (MEN IIB, neurofibromatosis, and Von Hippel Lindau
syndrome)

Pheochromocytoma is 10% malignant, 10% bilateral, 10% familial, and 10% extra-adrenal tumor

Clinical features include HTN (paroxysmal or persistent), sweating, tachycardia, weight loss, tremor,
hyperglycemia, hypercalcemia, and erythrocytosis.

Free metanephrine level in plasma is the best initial test, while 24 hours urinary metanephrine is the
most accurate test.

Treatment of HTN in pheochromocytoma is achieved by alpha-blockers. However, beta-blockers and


calcium channel blockers may be used aerward.

Surgical removal of the adrenal adenoma is the most accurate test.


Page - 626
Internal Medicine - Endocrinology

Question 136/162

Question #136

A 39-year-old male patient complains of episodic headache, sweating, tremor, and palpitations. He
lost 7 kilograms in the past 2 months. His examination demonstrates HTN, tachycardia, and tremor.
In addition, his lab investigations show hyperglycemia and hypercalcemia. What is the most likely
diagnosis?

a. Pheochromocytoma
b. Hyperthyroidism
c. Essential tremor
d. Renal artery stenosis
e. Hyperaldosteronism

‫اﻹﺟﺎﺑﺔ ﻋﲆ اﻟﺼﻔﺤﺔ اﻟﺘﺎﻟﻴﺔ‬

Page - 627
Internal Medicine - Endocrinology - Pheochromocytoma

Question 136/162

Question #136

A 39-year-old male patient complains of episodic headache, sweating, tremor, and palpitations. He
lost 7 kilograms in the past 2 months. His examination demonstrates HTN, tachycardia, and tremor.
In addition, his lab investigations show hyperglycemia and hypercalcemia. What is the most likely
diagnosis?

a. Pheochromocytoma √
b. Hyperthyroidism
c. Essential tremor
d. Renal artery stenosis
e. Hyperaldosteronism

Description

is is a typical scenario of pheochromocytoma.

Pheochromocytoma is a tumor of the medulla of the adrenal gland that secretes epinephrine and
norepinephrine,

It can be associated with other conditions like (MEN IIB, neurofibromatosis, and Von Hippel Lindau
syndrome)

Pheochromocytoma is 10% malignant, 10% bilateral, 10% familial, and 10% extra-adrenal tumor

Clinical features include HTN (paroxysmal or persistent), sweating, tachycardia, weight loss, tremor,
hyperglycemia, hypercalcemia, and erythrocytosis.

Free metanephrine level in plasma is the best initial test, while 24 hours urinary metanephrine is the
most accurate test.

Treatment of HTN in pheochromocytoma is achieved by alpha-blockers. However, beta-blockers and


calcium channel blockers may be used aerward.

Page - 628
Surgical removal of the adrenal adenoma is the most accurate test.

Page - 629
Internal Medicine - Endocrinology

Question 137/162

Question #137

A 33-year-old male patient is suspected of having pheochromocytoma because of paroxysmal HTN,


headaches, and sweatings. Which of the following will most likely be present during the episode of
these symptoms?

a. Hypoglycemia
b. Hypercalcemia
c. Anemia
d. Reduced catecholamines in urine
e. Bradycardia

‫اﻹﺟﺎﺑﺔ ﻋﲆ اﻟﺼﻔﺤﺔ اﻟﺘﺎﻟﻴﺔ‬

Page - 630
Internal Medicine - Endocrinology - Pheochromocytoma

Question 137/162

Question #137

A 33-year-old male patient is suspected of having pheochromocytoma because of paroxysmal HTN,


headaches, and sweatings. Which of the following will most likely be present during the episode of
these symptoms?

a. Hypoglycemia
b. Hypercalcemia √
c. Anemia
d. Reduced catecholamines in urine
e. Bradycardia

Description

Laboratory tests in pheochromocytoma may demonstrate hyperglycemia, hypercalcemia, and


erythrocytosis.

Pheochromocytoma is a tumor of the medulla of the adrenal gland that secretes epinephrine and
norepinephrine,

It can be associated with other conditions like (MEN IIB, neurofibromatosis, and Von Hippel Lindau
syndrome)

Pheochromocytoma is 10% malignant, 10% bilateral, 10% familial, and 10% extra-adrenal tumor

Clinical features include HTN (paroxysmal or persistent), sweating, tachycardia, weight loss, tremor,
hyperglycemia, hypercalcemia, and erythrocytosis.

Free metanephrine level in plasma is the best initial test, while 24 hours urinary metanephrine is the
most accurate test.

Treatment of HTN in pheochromocytoma is achieved by alpha-blockers. However, beta-blockers and


calcium channel blockers may be used aerward.

Surgical removal of the adrenal adenoma is the most accurate test.

Page - 631
Internal Medicine - Endocrinology

Question 138/162

Question #138

Pheochromocytoma is known to be associated with all the following except:

a. Hypercalcemia
b. Multiple endocrine neoplasia type 2
c. Neurofibromatosis
d. Hypoglycemia
e. Tremor and tachycardia

‫اﻹﺟﺎﺑﺔ ﻋﲆ اﻟﺼﻔﺤﺔ اﻟﺘﺎﻟﻴﺔ‬

Page - 632
Internal Medicine - Endocrinology - Pheochromocytoma

Question 138/162

Question #138

Pheochromocytoma is known to be associated with all the following except:

a. Hypercalcemia
b. Multiple endocrine neoplasia type 2
c. Neurofibromatosis
d. Hypoglycemia √
e. Tremor and tachycardia

Description

Hyperglycemia, not hypoglycemia, is a manifestation of pheochromocytoma.

Pheochromocytoma is a tumor of the medulla of the adrenal gland that secretes epinephrine and
norepinephrine,

It can be associated with other conditions like (MEN IIB, neurofibromatosis, and Von Hippel Lindau
syndrome)

Pheochromocytoma is 10% malignant, 10% bilateral, 10% familial, and 10% extra-adrenal tumor

Clinical features include HTN (paroxysmal or persistent), sweating, tachycardia, weight loss, tremor,
hyperglycemia, hypercalcemia, and erythrocytosis.

Free metanephrine level in plasma is the best initial test, while 24 hours urinary metanephrine is the
most accurate test.

Treatment of HTN in pheochromocytoma is achieved by alpha-blockers. However, beta-blockers and


calcium channel blockers may be used aerward.

Surgical removal of the adrenal adenoma is the most accurate test.

Page - 633
Internal Medicine - Endocrinology

Question 139/162

Question #139

A 33-year-old male patient complains of tremors, palpitation, headache, and sweating. His physical
examination demonstrates elevated blood pressure, and you suspect pheochromocytoma. What is
the most appropriate agent to control blood pressure in this patient?

a. Alpha-blockers
b. Beta-blockers
c. Diuretics
d. Calcium channel blockers
e. Angiotensin receptor blockers

‫اﻹﺟﺎﺑﺔ ﻋﲆ اﻟﺼﻔﺤﺔ اﻟﺘﺎﻟﻴﺔ‬

Page - 634
Internal Medicine - Endocrinology - Pheochromocytoma

Question 139/162

Question #139

A 33-year-old male patient complains of tremors, palpitation, headache, and sweating. His physical
examination demonstrates elevated blood pressure, and you suspect pheochromocytoma. What is
the most appropriate agent to control blood pressure in this patient?

a. Alpha-blockers √
b. Beta-blockers
c. Diuretics
d. Calcium channel blockers
e. Angiotensin receptor blockers

Description

Alpha-blockers are the drugs of choice to control blood pressure in pheochromocytoma patients.

Beta-blockers and calcium channel blockers can be used later on aer the use of alpha-blockers

Diuretics should be avoided in pheochromocytoma patients because these patients are volume-
depleted, and diuretics may exacerbate this depletion.

Page - 635
Internal Medicine - Endocrinology

Question 140/162

Question #140

A 39-year-old male patient complains of episodic headache, sweating, tremor, and palpitations. He
lost 7 kilograms in the past 2 months. His examination demonstrates HTN, tachycardia, and tremor.
In addition, his lab investigations show hyperglycemia and hypercalcemia. Which of the following is
the most appropriate diagnostic test?

a. Brain CT scan
b. Adrenal gland CT scan
c. 24-hour Metanephrine level
d. yroid function test
e. Aldosteron to renin ratio

‫اﻹﺟﺎﺑﺔ ﻋﲆ اﻟﺼﻔﺤﺔ اﻟﺘﺎﻟﻴﺔ‬

Page - 636
Internal Medicine - Endocrinology - Pheochromocytoma

Question 140/162

Question #140

A 39-year-old male patient complains of episodic headache, sweating, tremor, and palpitations. He
lost 7 kilograms in the past 2 months. His examination demonstrates HTN, tachycardia, and tremor.
In addition, his lab investigations show hyperglycemia and hypercalcemia. Which of the following is
the most appropriate diagnostic test?

a. Brain CT scan
b. Adrenal gland CT scan
c. 24-hour Metanephrine level √
d. yroid function test
e. Aldosteron to renin ratio

Description

is is a typical scenario of pheochromocytoma.

Pheochromocytoma is a tumor of the medulla of the adrenal gland that secretes epinephrine and
norepinephrine,

It can be associated with other conditions like (MEN IIB, neurofibromatosis, and Von Hippel Lindau
syndrome)

Pheochromocytoma is 10% malignant, 10% bilateral, 10% familial, and 10% extra-adrenal tumor

Clinical features include HTN (paroxysmal or persistent), sweating, tachycardia, weight loss, tremor,
hyperglycemia, hypercalcemia, and erythrocytosis.

Free metanephrine level in plasma is the best initial test, while 24 hours urinary metanephrine is the
most accurate test.

Treatment of HTN in pheochromocytoma is achieved by alpha-blockers. However, beta-blockers and


calcium channel blockers may be used aerward.

Surgical removal of the adrenal adenoma is the most accurate test.

Page - 637
Internal Medicine - Endocrinology

Question 141/162

Question #141

A 40-year-old female patient complains of amenorrhea, infertility, and galactorrhea. On further


questioning, the patient complains of a recurrent, progressive headache. Which of the following
visual abnormalities is most likely present in this patient?

a. Homonymous hemianopia
b. Homonymous upper quadrantanopia
c. Homonymous lower quadrantanopia
d. Bitemporal hemianopia
e. Cortical blindness

‫اﻹﺟﺎﺑﺔ ﻋﲆ اﻟﺼﻔﺤﺔ اﻟﺘﺎﻟﻴﺔ‬

Page - 638
Internal Medicine - Endocrinology - Pituitary Adenoma

Question 141/162

Question #141

A 40-year-old female patient complains of amenorrhea, infertility, and galactorrhea. On further


questioning, the patient complains of a recurrent, progressive headache. Which of the following
visual abnormalities is most likely present in this patient?

a. Homonymous hemianopia
b. Homonymous upper quadrantanopia
c. Homonymous lower quadrantanopia
d. Bitemporal hemianopia √
e. Cortical blindness

Description

is patient most likely suers from hyperprolactinemia, most commonly caused by prolactinoma.

Prolactinoma is a pituitary gland adenoma that secrets prolactin and may cause compression.

Clinical features of pituitary adenoma are:

e compression eect of the tumor:


Visual field defect: (bitemporal hemianopia or upper temporal quadrantanopia): due to
compression on the optic chiasma
Lateral extension to cavernous sinus → Cranial nerves 3, 4 & 6 dysfunction
Compression of normal tissue of pituitary gland → Hyposecretion of the gland hormones
(Panhypopituitarism)
Hypersecretion conditions:
Gigantism or acromegaly (excessive GH secretion)
Cushing’s disease (with high ACTH)
Hyperprolactinemia
Some adenomas are called non-functioning adenomas (no excessive hormone secretion but
can cause compression eect)

Page - 639
Internal Medicine - Endocrinology

Question 142/162

Question #142

A 29-year-old female patient complains of recurrent headaches and bitemporal hemianopia. In


addition, she has had amenorrhea for the last 4 months. What is the most likely explanation?

a. Dilatation of the third ventricle


b. Berry aneurysm of the circle of Willis
c. Pituitary adenoma
d. Pregnancy
e. Polycystic ovarian syndrome

‫اﻹﺟﺎﺑﺔ ﻋﲆ اﻟﺼﻔﺤﺔ اﻟﺘﺎﻟﻴﺔ‬

Page - 640
Internal Medicine - Endocrinology - Pituitary Adenoma

Question 142/162

Question #142

A 29-year-old female patient complains of recurrent headaches and bitemporal hemianopia. In


addition, she has had amenorrhea for the last 4 months. What is the most likely explanation?

a. Dilatation of the third ventricle


b. Berry aneurysm of the circle of Willis
c. Pituitary adenoma √
d. Pregnancy
e. Polycystic ovarian syndrome

Description

e presence of headache, bitemporal hemianopia, and the failure of the HPO axis strongly suggest
compression due to pituitary adenoma.

Clinical features of pituitary tumors:

e compression eect of the tumor:


Visual field defect: (bitemporal hemianopia or upper temporal quadrantanopia): due to
compression on the optic chiasma
Lateral extension to cavernous sinus → Cranial nerves 3, 4 & 6 dysfunction
Compression of normal tissue of pituitary gland → Hyposecretion of the gland hormones
(Panhypopituitarism)
Hypersecretion conditions:
Gigantism or acromegaly (excessive GH secretion)
Cushing’s disease (with high ACTH)
Hyperprolactinemia
Some adenomas are called non-functioning adenomas (no excessive hormone secretion but
can cause compression eect)

Page - 641
Internal Medicine - Endocrinology

Question 143/162

Question #143

A 35-year-old female presents with 2 months history of nervousness, tachycardia, heat intolerance,
and tremor. Her lab investigations demonstrate a TSH level of 15.4 μU/mL, and her free T4 level is 25
ng/dL. What is the most likely diagnosis?

a. Grave’s disease
b. Subacute thyroiditis
c. TSH secreting pituitary adenoma
d. Factitious hyperthyroidism
e. Hashimoto’s thyroiditis

‫اﻹﺟﺎﺑﺔ ﻋﲆ اﻟﺼﻔﺤﺔ اﻟﺘﺎﻟﻴﺔ‬

Page - 642
Internal Medicine - Endocrinology - Pituitary Adenoma

Question 143/162

Question #143

A 35-year-old female presents with 2 months history of nervousness, tachycardia, heat intolerance,
and tremor. Her lab investigations demonstrate a TSH level of 15.4 μU/mL, and her free T4 level is 25
ng/dL. What is the most likely diagnosis?

a. Grave’s disease
b. Subacute thyroiditis
c. TSH secreting pituitary adenoma √
d. Factitious hyperthyroidism
e. Hashimoto’s thyroiditis

Description

e clinical features and the elevated T4 level suggest the diagnosis of hyperthyroidism.

TSH level will be suppressed in the case of primary hyperthyroidism but elevated in the case of
secondary hyperthyroidism.

is is secondary hyperthyroidism mostly due to pituitary over-secretion of TSH, leading to elevated
thyroxine levels.

Page - 643
Internal Medicine - Endocrinology

Question 144/162

Question #144

A 40-year-old male patient is suspected of having primary hyperaldosteronism. e following are


characteristics of this condition except:

a. Hypertension
b. Reduced urinary sodium
c. Normal to elevated urinary potassium
d. High aldosterone to renin ratio
e. All of the above

‫اﻹﺟﺎﺑﺔ ﻋﲆ اﻟﺼﻔﺤﺔ اﻟﺘﺎﻟﻴﺔ‬

Page - 644
Internal Medicine - Endocrinology - Primary hyperaldosteronism

Question 144/162

Question #144

A 40-year-old male patient is suspected of having primary hyperaldosteronism. e following are


characteristics of this condition except:

a. Hypertension
b. Reduced urinary sodium
c. Normal to elevated urinary potassium √
d. High aldosterone to renin ratio
e. All of the above

Description

Aldosterone:renin ratio of > 20 indicates primary hyperaldosteronism.

It is the best test to dierentiate primary from secondary hyperaldosteronism.

Aldosterone, a hormone produced and secreted by the adrenal glands, signals the kidneys to excrete
less sodium and more potassium.

Primary hyperaldosteronism is the hypersecretion of aldosterone despite low renin and


hypertension.

Adrenal adenoma (Conn’s syndrome), adrenal hyperplasia, or rarely adrenal malignancy are possible
causes.

You should suspect this condition if HTN, Hypokalemia, and metabolic alkalosis are present.

e best initial test is to measure the aldosterone/renin ratio (levels of > 20 are suggestive of the
disease)

24-hour urinary collection for aldosterone should be done next.

CT or MRI are the best to localize the pathology/

Treatment is either surgery for tumors or spironolactone for hyperplasia.

Page - 645
Internal Medicine - Endocrinology

Question 145/162

Question #145

A 35-year-old guy has felt weak, lethargic, and suering from headaches. He has a blood pressure
of 190/103 mmHg. He is not taking any medication. His blood test results indicate hypokalemia but
are otherwise normal. What is the most likely diagnosis?

a. Pheochromocytoma
b. Primary hyperaldosteronism
c. Hyperthyroidism
d. Chronic kidney disease
e. Renal artery stenosis

‫اﻹﺟﺎﺑﺔ ﻋﲆ اﻟﺼﻔﺤﺔ اﻟﺘﺎﻟﻴﺔ‬

Page - 646
Internal Medicine - Endocrinology - Primary hyperaldosteronism

Question 145/162

Question #145

A 35-year-old guy has felt weak, lethargic, and suering from headaches. He has a blood pressure
of 190/103 mmHg. He is not taking any medication. His blood test results indicate hypokalemia but
are otherwise normal. What is the most likely diagnosis?

a. Pheochromocytoma
b. Primary hyperaldosteronism √
c. Hyperthyroidism
d. Chronic kidney disease
e. Renal artery stenosis

Description

is is a typical scenario of primary hyperaldosteronism.

Aldosterone, a hormone produced and secreted by the adrenal glands, signals the kidneys to excrete
less sodium and more potassium.

Primary hyperaldosteronism is the hypersecretion of aldosterone despite low renin and


hypertension.

Adrenal adenoma (Conn’s syndrome), adrenal hyperplasia, or rarely adrenal malignancy are possible
causes.

You should suspect this condition if HTN, Hypokalemia, and metabolic alkalosis are present.

e best initial test is to measure the aldosterone/renin ratio (levels of > 20 are suggestive of the
disease)

24-hour urinary collection for aldosterone should be done next.

CT or MRI are the best to localize the pathology/

Treatment is either surgery for tumors or spironolactone for hyperplasia.

Page - 647
Internal Medicine - Endocrinology

Question 146/162

Question #146

A 39-year-old lady has been tired and sluggish. Despite taking enalapril, she has a blood pressure of
159/99 mmHg. Her blood test results are as follows: Hemoglobin 13.2 g/dL, Serum sodium 142
mEq/L, Serum potassium 3.2 mEq/L. What is the most likely diagnosis?

a. Cushing’s syndrome
b. Pheochromocytoma
c. Chronic kidney disease
d. Primary hyperaldosteronism
e. Hypoadrenalism

‫اﻹﺟﺎﺑﺔ ﻋﲆ اﻟﺼﻔﺤﺔ اﻟﺘﺎﻟﻴﺔ‬

Page - 648
Internal Medicine - Endocrinology - Primary hyperaldosteronism

Question 146/162

Question #146

A 39-year-old lady has been tired and sluggish. Despite taking enalapril, she has a blood pressure of
159/99 mmHg. Her blood test results are as follows: Hemoglobin 13.2 g/dL, Serum sodium 142
mEq/L, Serum potassium 3.2 mEq/L. What is the most likely diagnosis?

a. Cushing’s syndrome
b. Pheochromocytoma
c. Chronic kidney disease
d. Primary hyperaldosteronism √
e. Hypoadrenalism

Description

Primary hyperaldosteronism is the hypersecretion of aldosterone despite low renin and


hypertension.

Adrenal adenoma (Conn’s syndrome), adrenal hyperplasia, or rarely adrenal malignancy are possible
causes.

You should suspect this condition if HTN, Hypokalemia, and metabolic alkalosis are present.

e best initial test is to measure the aldosterone/renin ratio (levels of > 20 are suggestive of the
disease)

24-hour urinary collection for aldosterone should be done next.

CT or MRI are the best to localize the pathology/

Treatment is either surgery for tumors or spironolactone for hyperplasia.

Page - 649
Internal Medicine - Endocrinology

Question 147/162

Question #147

A 35-year-old male patient complains of HTN and headache. His recent blood tests show serum
potassium of 2.6 mEq/L. What is the most appropriate test at this time?

a. Cortisol level
b. yroxine level
c. Aldosterone level
d. Testosterone level
e. yrotropin level

‫اﻹﺟﺎﺑﺔ ﻋﲆ اﻟﺼﻔﺤﺔ اﻟﺘﺎﻟﻴﺔ‬

Page - 650
Internal Medicine - Endocrinology - Primary hyperaldosteronism

Question 147/162

Question #147

A 35-year-old male patient complains of HTN and headache. His recent blood tests show serum
potassium of 2.6 mEq/L. What is the most appropriate test at this time?

a. Cortisol level
b. yroxine level
c. Aldosterone level √
d. Testosterone level
e. yrotropin level

Description

Hypokalemia and HTN strongly suggest primary hyperaldosteronism as the most likely diagnosis.

24-hour urinary collection for aldosterone is the most appropriate initial hormone test.

Aldosterone:renin ratio of > 20 indicates primary hyperaldosteronism.

It is the best test to dierentiate primary from secondary hyperaldosteronism.

Aldosterone, a hormone produced and secreted by the adrenal glands, signals the kidneys to excrete
less sodium and more potassium.

Primary hyperaldosteronism is the hypersecretion of aldosterone despite low renin and


hypertension.

Adrenal adenoma (Conn’s syndrome), adrenal hyperplasia, or rarely adrenal malignancy are possible
causes.

You should suspect this condition if HTN, Hypokalemia, and metabolic alkalosis are present.

e best initial test is to measure the aldosterone/renin ratio (levels of > 20 are suggestive of the
disease)

24-hour urinary collection for aldosterone should be done next.

Page - 651
CT or MRI are the best to localize the pathology/

Treatment is either surgery for tumors or spironolactone for hyperplasia.

Page - 652
Internal Medicine - Endocrinology

Question 148/162

Question #148

A 39-year-old lady has been tired and sluggish. Despite taking enalapril, she has a blood pressure of
159/99 mmHg. Her blood test results are as follows: Hb 13.2 g/dL, Serum sodium 142 mEq/L, Serum
potassium 3.2 mEq/L. Which of the following is the most likely explanation for her hypokalemia?

a. Urinary potassium loss


b. Reduced potassium intake
c. Potassium loss in stool
d. Potassium shiing to the intracellular space
e. Metabolic alkalosis

‫اﻹﺟﺎﺑﺔ ﻋﲆ اﻟﺼﻔﺤﺔ اﻟﺘﺎﻟﻴﺔ‬

Page - 653
Internal Medicine - Endocrinology - Primary hyperaldosteronism

Question 148/162

Question #148

A 39-year-old lady has been tired and sluggish. Despite taking enalapril, she has a blood pressure of
159/99 mmHg. Her blood test results are as follows: Hb 13.2 g/dL, Serum sodium 142 mEq/L, Serum
potassium 3.2 mEq/L. Which of the following is the most likely explanation for her hypokalemia?

a. Urinary potassium loss √


b. Reduced potassium intake
c. Potassium loss in stool
d. Potassium shiing to the intracellular space
e. Metabolic alkalosis

Description

is is a typical scenario of primary hyperaldosteronism.

Aldosterone, a hormone produced and secreted by the adrenal glands, signals the kidneys to excrete
less sodium and more potassium.

Primary hyperaldosteronism is the hypersecretion of aldosterone despite low renin and


hypertension.

Adrenal adenoma (Conn’s syndrome), adrenal hyperplasia, or rarely adrenal malignancy are possible
causes.

You should suspect this condition if HTN, Hypokalemia, and metabolic alkalosis are present.

e best initial test is to measure the aldosterone/renin ratio (levels of > 20 are suggestive of the
disease)

24-hour urinary collection for aldosterone should be done next.

CT or MRI are the best to localize the pathology/

Treatment is either surgery for tumors or spironolactone for hyperplasia.

Page - 654
Internal Medicine - Endocrinology

Question 149/162

Question #149

A 40-year-old male patient complains of hypertension and hypokalemia. Which of the following is
the most sensitive means of dierentiating primary from secondary hyperaldosteronism?

a. Metanephrine urinary level


b. Cosyntropin stimulation test
c. Aldosterone:renin ratio
d. Aldosterone level
e. Renin level

‫اﻹﺟﺎﺑﺔ ﻋﲆ اﻟﺼﻔﺤﺔ اﻟﺘﺎﻟﻴﺔ‬

Page - 655
Internal Medicine - Endocrinology - Primary hyperaldosteronism

Question 149/162

Question #149

A 40-year-old male patient complains of hypertension and hypokalemia. Which of the following is
the most sensitive means of dierentiating primary from secondary hyperaldosteronism?

a. Metanephrine urinary level


b. Cosyntropin stimulation test
c. Aldosterone:renin ratio √
d. Aldosterone level
e. Renin level

Description

Aldosterone:renin ratio of > 20 indicates primary hyperaldosteronism.

It is the best test to dierentiate primary from secondary hyperaldosteronism.

Aldosterone, a hormone produced and secreted by the adrenal glands, signals the kidneys to excrete
less sodium and more potassium.

Primary hyperaldosteronism is the hypersecretion of aldosterone despite low renin and


hypertension.

Adrenal adenoma (Conn’s syndrome), adrenal hyperplasia, or rarely adrenal malignancy are possible
causes.

You should suspect this condition if HTN, Hypokalemia, and metabolic alkalosis are present.

e best initial test is to measure the aldosterone/renin ratio (levels of > 20 are suggestive of the
disease)

24-hour urinary collection for aldosterone should be done next.

CT or MRI are the best to localize the pathology/

Treatment is either surgery for tumors or spironolactone for hyperplasia.

Page - 656
Internal Medicine - Endocrinology

Question 150/162

Question #150

29-year-old female patient presents with neck pain, fever, and tachycardia, and you suspect
subacute thyroiditis. Which of the following is false about this condition?

a. It results from viral infection


b. ESR is elevated
c. e patient could need a thyroxine replacement
d. Features of hyperthyroidism are initial
e. Radioactive iodine uptake is elevated

‫اﻹﺟﺎﺑﺔ ﻋﲆ اﻟﺼﻔﺤﺔ اﻟﺘﺎﻟﻴﺔ‬

Page - 657
Internal Medicine - Endocrinology - Subacute yroiditis

Question 150/162

Question #150

29-year-old female patient presents with neck pain, fever, and tachycardia, and you suspect
subacute thyroiditis. Which of the following is false about this condition?

a. It results from viral infection


b. ESR is elevated
c. e patient could need a thyroxine replacement
d. Features of hyperthyroidism are initial
e. Radioactive iodine uptake is elevated √

Description

Radioactive iodine uptake is elevated in Graves’ disease, multinodular goiter, and autonomously
functioning solitary thyroid nodule. In addition, it will be low in subacute thyroiditis, silent
thyroiditis, and exogenous thyroxine use.

e destructed cells in subacute thyroiditis will not be able to uptake iodine.

Subacute thyroiditis is mostly due to viral infection of the thyroid gland leading to cell destruction
and release of the preformed thyroxine and features of thyrotoxicosis initially, which will be later
transformed into hypothyroidism.

Page - 658
Internal Medicine - Endocrinology

Question 151/162

Question #151

A 35-year-old lady complains of neck pain and tenderness associated with low-grade fever and
tachycardia. Her lab investigations show high WBC, high Free T4, high ESR, and normal TSH. What is
the most likely diagnosis?

a. Neck cellulitis
b. Single hot nodule
c. Subacute thyroiditis
d. Grave’s disease
e. Hashimoto’s thyroiditis

‫اﻹﺟﺎﺑﺔ ﻋﲆ اﻟﺼﻔﺤﺔ اﻟﺘﺎﻟﻴﺔ‬

Page - 659
Internal Medicine - Endocrinology - Subacute yroiditis

Question 151/162

Question #151

A 35-year-old lady complains of neck pain and tenderness associated with low-grade fever and
tachycardia. Her lab investigations show high WBC, high Free T4, high ESR, and normal TSH. What is
the most likely diagnosis?

a. Neck cellulitis
b. Single hot nodule
c. Subacute thyroiditis √
d. Grave’s disease
e. Hashimoto’s thyroiditis

Description

Subacute thyroiditis is mostly due to viral infection of the thyroid gland leading to cell destruction
and release of the preformed thyroxine and features of thyrotoxicosis initially, which will be later
transformed into hypothyroidism.

Subacute thyroiditis is characterized by elevated ESR, neck pain, and tenderness associated with
features of hyperthyroidism followed by hypothyroidism.

Page - 660
Internal Medicine - Endocrinology

Question 152/162

Question #152

A 40-year-old female has recovered from a one-week illness of upper respiratory infection. Today,
she presents to your oce with a racing heart, neck pain, and feeling hot. Physical examination
demonstrates tachycardia and a tender neck. Her lab investigations show high T4, T3, WBC, and ESR,
and negative thyroid antibodies. What is the most likely diagnosis?

a. Hashimoto’s thyroiditis
b. Grave’s disease
c. Subacute thyroiditis
d. Toxic nodular goiter
e. yroid fibrosis

‫اﻹﺟﺎﺑﺔ ﻋﲆ اﻟﺼﻔﺤﺔ اﻟﺘﺎﻟﻴﺔ‬

Page - 661
Internal Medicine - Endocrinology - Subacute yroiditis

Question 152/162

Question #152

A 40-year-old female has recovered from a one-week illness of upper respiratory infection. Today,
she presents to your oce with a racing heart, neck pain, and feeling hot. Physical examination
demonstrates tachycardia and a tender neck. Her lab investigations show high T4, T3, WBC, and ESR,
and negative thyroid antibodies. What is the most likely diagnosis?

a. Hashimoto’s thyroiditis
b. Grave’s disease
c. Subacute thyroiditis √
d. Toxic nodular goiter
e. yroid fibrosis

Description

Description:

Subacute thyroiditis is mostly due to viral infection of the thyroid gland leading to cell destruction
and release of the preformed thyroxine and features of thyrotoxicosis initially, which will be later
transformed into hypothyroidism.

Subacute thyroiditis is characterized by elevated ESR, neck pain, and tenderness associated with
features of hyperthyroidism followed by hypothyroidism.

Page - 662
Internal Medicine - Endocrinology

Question 153/162

Question #153

A 35-year-old female complains of neck pain, fever, and palpitations. Her physical examination
demonstrates neck tenderness and tachycardia. In addition, her lab investigations show
hyperthyroidism and reduced radioactive iodine uptake. What is the most helpful treatment at this
time?

a. Levothyroxine and NSAIDs


b. Prednisolone
c. Propylthiouracil
d. Carbimazole
e. yroidectomy

‫اﻹﺟﺎﺑﺔ ﻋﲆ اﻟﺼﻔﺤﺔ اﻟﺘﺎﻟﻴﺔ‬

Page - 663
Internal Medicine - Endocrinology - Subacute yroiditis

Question 153/162

Question #153

A 35-year-old female complains of neck pain, fever, and palpitations. Her physical examination
demonstrates neck tenderness and tachycardia. In addition, her lab investigations show
hyperthyroidism and reduced radioactive iodine uptake. What is the most helpful treatment at this
time?

a. Levothyroxine and NSAIDs


b. Prednisolone √
c. Propylthiouracil
d. Carbimazole
e. yroidectomy

Description

Subacute thyroiditis is mostly due to viral infection of the thyroid gland leading to cell destruction
and release of the preformed thyroxine and features of thyrotoxicosis initially, which will be later
transformed into hypothyroidism.

Subacute thyroiditis is characterized by elevated ESR, neck pain, and tenderness associated with
features of hyperthyroidism followed by hypothyroidism.

Treatment of subacute thyroiditis is achieved by:

Steroids, NSAIDs (aspirin)


Treatment of hyperthyroidism initially
Later, the patient will remain hypothyroidism and will need thyroxin

Page - 664
Internal Medicine - Endocrinology

Question 154/162

Question #154

A 30-year-old female presents to the hospital with palpitations and profuse perspiration for 3 days.
On examination, you found a temperature of 39 °C, and the heart rate is irregular, 155 beats per
minute. What is the most appropriate management?

a. DC cardioversion
b. Amiodarone
c. Flecainide
d. Propranolol
e. Verapamil

‫اﻹﺟﺎﺑﺔ ﻋﲆ اﻟﺼﻔﺤﺔ اﻟﺘﺎﻟﻴﺔ‬

Page - 665
Internal Medicine - Endocrinology - yroid storm

Question 154/162

Question #154

A 30-year-old female presents to the hospital with palpitations and profuse perspiration for 3 days.
On examination, you found a temperature of 39 °C, and the heart rate is irregular, 155 beats per
minute. What is the most appropriate management?

a. DC cardioversion
b. Amiodarone
c. Flecainide
d. Propranolol √
e. Verapamil

Description

is patient is most likely suering from a thyroid storm, so beta-blockers are the first line in this
scenario.

yroid storm is a medical emergency of hyperthyroidism, Precipitated by stress, infection, trauma

Symptoms include fever, nausea, vomiting, and diarrhea. In addition, the patient may suer from
arrhythmias, tachycardia, HTN, heart failure, and even coma

Propranolol is used in the treatment because it:

Blocks the target organ eect of thyroxine


Blocks the peripheral conversion of T4 to T3

Propylthiouracil (PTU) is the drug of choice for thyroid storm.

PTU is preferred over methimazole


It blocks the thyroid hormone’s production

Steroids and radioactive iodine ablation are used in the treatment.

Page - 666
Internal Medicine - Endocrinology

Question 155/162

Question #155

Which of the following is the most appropriate diagnostic test to evaluate the vitamin D level?

a. Serum total calcium level


b. Serum ionized calcium level
c. 1,25 (OH)D
d. 25 (OH)D
e. Urinary calcium level

‫اﻹﺟﺎﺑﺔ ﻋﲆ اﻟﺼﻔﺤﺔ اﻟﺘﺎﻟﻴﺔ‬

Page - 667
Internal Medicine - Endocrinology - Vitamin D deficiency

Question 155/162

Question #155

Which of the following is the most appropriate diagnostic test to evaluate the vitamin D level?

a. Serum total calcium level


b. Serum ionized calcium level
c. 1,25 (OH)D
d. 25 (OH)D √
e. Urinary calcium level

Description

e best test to evaluate vitamin D deficiency is 25 (OH)D3.

Note that 1,25 (OH)D is the active form of vitamin D synthesized by the kidneys and is aected by
kidney function.

Page - 668
Internal Medicine - Endocrinology

Question 156/162

Question #156

A 32-year-old nurse is brought to the emergency department with confusion and sweating.
However, you draw laboratory samples during the events, his serum glucose level is 25mg/dL, and
he improves aer the administration of intravenous 10% dextrose. His medical records show
recurrent episodes of the same condition for which he admitted to the hospital multiple times
without any final diagnosis. His other lab tests later show the following:

Serum insulin: high

Serum C-Peptide: high

Urinary toxicology screen for oral hypoglycemic agents: negative.

What is the most likely diagnosis?

a. Insulinoma
b. Surreptitious Insulin use
c. Surreptitious Oral hypoglycemic agents use
d. Anorexia nervosa
e. Post-prandial hypoglycemia

‫اﻹﺟﺎﺑﺔ ﻋﲆ اﻟﺼﻔﺤﺔ اﻟﺘﺎﻟﻴﺔ‬

Page - 669
Internal Medicine - Endocrinology - Hypoglycemia

Question 156/162

Question #156

A 32-year-old nurse is brought to the emergency department with confusion and sweating.
However, you draw laboratory samples during the events, his serum glucose level is 25mg/dL, and
he improves aer the administration of intravenous 10% dextrose. His medical records show
recurrent episodes of the same condition for which he admitted to the hospital multiple times
without any final diagnosis. His other lab tests later show the following:

Serum insulin: high

Serum C-Peptide: high

Urinary toxicology screen for oral hypoglycemic agents: negative.

What is the most likely diagnosis?

a. Insulinoma √
b. Surreptitious Insulin use
c. Surreptitious Oral hypoglycemic agents use
d. Anorexia nervosa
e. Post-prandial hypoglycemia

Description

is patient has recurrent episodes of the same condition that fulfills the Whipple’s triad
(Hypoglycemia).

e high C-Peptide and insulin levels indicate endogenous source of the insulin.

e negative urinary screen for OHA indicates that the patient’s condition is not surreptitious.

Always ensure that the insulin and C peptide samples are drawn during the hypoglycemia event;
they will give you a false negative results if they are taken while the blood sugar is normal.

Page - 670
Internal Medicine - Endocrinology

Question 157/162

Question #157

A 60-year-old male patient presents to the emergency department with decreased consciousness
and sweating. He is a known case of type 2 DM for which he receives metformin and sulfonylurea.
However, you found his blood sugar of 20 mg/dL. erefore, you provided him with an ampule of
D50%, and the patient recovered consciousness. What is the most appropriate management?

a. Provide another ampule of D50% and discharge


b. Provide 2 ampules of D50% and discharge
c. Admission for continuous dextrose infusion and glucose monitoring
d. Observe at the emergency department for 2 hours, then discharge home if no hypoglycemia
occurs
e. Observe at the emergency department for 6 hours, then discharged home if no hypoglycemia
occurs

‫اﻹﺟﺎﺑﺔ ﻋﲆ اﻟﺼﻔﺤﺔ اﻟﺘﺎﻟﻴﺔ‬

Page - 671
Internal Medicine - Endocrinology - Hypoglycemia

Question 157/162

Question #157

A 60-year-old male patient presents to the emergency department with decreased consciousness
and sweating. He is a known case of type 2 DM for which he receives metformin and sulfonylurea.
However, you found his blood sugar of 20 mg/dL. erefore, you provided him with an ampule of
D50%, and the patient recovered consciousness. What is the most appropriate management?

a. Provide another ampule of D50% and discharge


b. Provide 2 ampules of D50% and discharge
c. Admission for continuous dextrose infusion and glucose monitoring √
d. Observe at the emergency department for 2 hours, then discharge home if no hypoglycemia
occurs
e. Observe at the emergency department for 6 hours, then discharged home if no hypoglycemia
occurs

Description

Whipple’s triad here is obvious and diagnostic of hypoglycemia.

In managing hypoglycemia, you should address the cause and determine the risk of recurrence.
However, in this situation, the patient is on sulfonylurea, so he has a risk of recurrence of
hypoglycemia because of the drug’s prolonged half-life.

When the risk of recurrence is high, you should admit the patient and continue intravenous
dextrose and blood sugar monitoring.

If the patient is on insulin alone, there will be no need for admission as there is no risk of recurrence
because of the half-life of insulin.

Page - 672
Internal Medicine - Endocrinology

Question 158/162

Question #158

A 23-year-old male patient collapses while playing football. He is brought to the emergency
department and found to have normal blood pressure, pulse rate, and respiratory rate. What is the
most appropriate next step in management?

a. Brain CT scan
b. Brain MRI
c. Blood glucose measurement
d. Intravenous fluid administration
e. Perform ECG

‫اﻹﺟﺎﺑﺔ ﻋﲆ اﻟﺼﻔﺤﺔ اﻟﺘﺎﻟﻴﺔ‬

Page - 673
Internal Medicine - Endocrinology - Hypoglycemia

Question 158/162

Question #158

A 23-year-old male patient collapses while playing football. He is brought to the emergency
department and found to have normal blood pressure, pulse rate, and respiratory rate. What is the
most appropriate next step in management?

a. Brain CT scan
b. Brain MRI
c. Blood glucose measurement √
d. Intravenous fluid administration
e. Perform ECG

Description

Exercise can result in hypoglycemia, manifesting as a reduced level of consciousness. erefore,


before taking the patient for a CT scan or any other study, check his blood sugar to ensure adequate
serum glucose level.

e next step in any case scenario should be easy, accessible, informative, and make a dierence in
the management.

Page - 674
Internal Medicine - Endocrinology

Question 159/162

Question #159

A 40-year-old female patient complains of fainting and is found to have a blood sugar of 32mg/dL.
Which of the following criteria indicates hypoglycemia?

a. Cushing’s triad
b. Whipple triad
c. Virchow’s triad
d. Charcot triad
e. Beck’s triad

‫اﻹﺟﺎﺑﺔ ﻋﲆ اﻟﺼﻔﺤﺔ اﻟﺘﺎﻟﻴﺔ‬

Page - 675
Internal Medicine - Endocrinology - Hypoglycemia

Question 159/162

Question #159

A 40-year-old female patient complains of fainting and is found to have a blood sugar of 32mg/dL.
Which of the following criteria indicates hypoglycemia?

a. Cushing’s triad
b. Whipple triad √
c. Virchow’s triad
d. Charcot triad
e. Beck’s triad

Description

Whipple’s triad is used to diagnose hypoglycemia, and it includes:

Decreased level of consciousness


Serum glucose < 45 mg/dl
Improvement aer administration of glucose

Cushing triad presents in increased intracranial pressure

Virchow’s triad presents in patients with hypercoagulability

Charcot triad presents in patients with cholangitis

Beck’s triad presents in cardiac tamponade

Page - 676
Internal Medicine - Endocrinology

Question 160/162

Question #160

A 22-year-old male patient is brought to the emergency department with a decreased level of
consciousness. His friends told you he was at a party drinking alcohol and then found unconscious.
His blood pressure is 120/80, and his pulse is 94 bpm; he is afebrile and semi-conscious but has no
obvious neurological deficit. What is the most appropriate next step in the management of this
patient?

a. Brain CT scan
b. Brain MRI
c. Blood glucose measurement
d. Intravenous fluid administration
e. Perform ECG

‫اﻹﺟﺎﺑﺔ ﻋﲆ اﻟﺼﻔﺤﺔ اﻟﺘﺎﻟﻴﺔ‬

Page - 677
Internal Medicine - Endocrinology - Hypoglycemia

Question 160/162

Question #160

A 22-year-old male patient is brought to the emergency department with a decreased level of
consciousness. His friends told you he was at a party drinking alcohol and then found unconscious.
His blood pressure is 120/80, and his pulse is 94 bpm; he is afebrile and semi-conscious but has no
obvious neurological deficit. What is the most appropriate next step in the management of this
patient?

a. Brain CT scan
b. Brain MRI
c. Blood glucose measurement √
d. Intravenous fluid administration
e. Perform ECG

Description

Alcohol consumption can result in hypoglycemia, manifesting as a reduced level of consciousness.


erefore, before taking the patient for a CT scan or any other study, check his blood sugar to ensure
adequate serum glucose level.

e next step in any case scenario should be easy, accessible, informative, and make a dierence in
the management.

Page - 678
Internal Medicine - Endocrinology

Question 161/162

Question #161

A 40-year-old male patient has been diagnosed with type 1 DM for the past 23 years. His blood sugar
was well controlled on insulin, but he recently started having recurrent hypoglycemia episodes.
Which of the following is the most likely cause of his recurrent hypoglycemia?

a. Ineectiveness of his insulin


b. Increased insulin sensitivity
c. Renal impairment
d. Spontaneous beta cells improvement
e. Reduced physical activity

‫اﻹﺟﺎﺑﺔ ﻋﲆ اﻟﺼﻔﺤﺔ اﻟﺘﺎﻟﻴﺔ‬

Page - 679
Internal Medicine - Endocrinology - Hypoglycemia

Question 161/162

Question #161

A 40-year-old male patient has been diagnosed with type 1 DM for the past 23 years. His blood sugar
was well controlled on insulin, but he recently started having recurrent hypoglycemia episodes.
Which of the following is the most likely cause of his recurrent hypoglycemia?

a. Ineectiveness of his insulin


b. Increased insulin sensitivity
c. Renal impairment √
d. Spontaneous beta cells improvement
e. Reduced physical activity

Description

Diabetic renal disease is the most prevalent cause of hypoglycemia in a previously stable, well-
controlled diabetic patient who has not changed his or her diet or insulin dose. is is because the
kidney usually metabolizes exogenous insulin. However, when kidney function is impaired, the half-
life of insulin is prolonged due to reduced levels of its degradation. As a result, hypoglycemia
episodes may frequently occur in diabetic patients (treated with insulin) and moderate to severe
renal failure.

Page - 680
Internal Medicine - Endocrinology

Question 162/162

Question #162

A 32-year-old male patient complains of sweating and drowsiness for the past 30 minutes. However,
his blood glucose level was 25mg/dL, and he responded quickly to intravenous glucose
administration. His further evaluation demonstrates a reduced C-peptide level measured during the
attack. Shat is the most likely cause?

a. Insulinoma
b. Rebound hypoglycemia from eating
c. Sulfonylurea overdose
d. Surreptitious insulin administration
e. Glucagonoma

‫اﻹﺟﺎﺑﺔ ﻋﲆ اﻟﺼﻔﺤﺔ اﻟﺘﺎﻟﻴﺔ‬

Page - 681
Internal Medicine - Endocrinology - Hypoglycemia

Question 162/162

Question #162

A 32-year-old male patient complains of sweating and drowsiness for the past 30 minutes. However,
his blood glucose level was 25mg/dL, and he responded quickly to intravenous glucose
administration. His further evaluation demonstrates a reduced C-peptide level measured during the
attack. Shat is the most likely cause?

a. Insulinoma
b. Rebound hypoglycemia from eating
c. Sulfonylurea overdose
d. Surreptitious insulin administration √
e. Glucagonoma

Description

During the attack of hypoglycemia, if the C-peptide is high, this indicates that the insulin causing
this condition is endogenous (e.g., insulinoma, sulfonylurea overdose, or rebound hypoglycemia
from eating).

In this scenario, the low C-peptide level indicates exogenous insulin as the most likely diagnosis.

Glucagonoma is not a cause of hypoglycemia.

Page - 682
Gastroenterology

Page - 683
Gastroenterology

‫ﻣﻮاﺿﻴﻊ اﻷﺳﺌﻠﺔ وأﻋﺪادﻫﺎ‬

1) Achalasia => 11 Questions


2) Barrette's Esophagus (BE) => 3 Questions
3) Celiac Disease => 15 Questions
4) Diarrhea => 18 Questions
5) Epigastric pain and dyspepsia => 7 Questions
6) Esophageal spasm => 3 Questions
7) Esophagitis => 3 Questions
8) Gastroesophageal Reflux Disease (GERD) => 4 Questions
9) Gastroparesis => 2 Questions
10) H. pylori Infection => 6 Questions
11) Inflammatory Bowel Diseases (IBD) => 21 Questions
12) Irritable Bowel syndrome (IBS) => 4 Questions
13) Peptic ulcer disease (PUD) => 5 Questions
14) Upper Gastrointestinal bleeding (UGIB) => 3 Questions
15) Vitamin A Deficiency => 1 Questions

Page - 684
Internal Medicine - Gastroenterology

Question 1/106

Question #1

A patient was diagnosed recently with achalasia. Which of the following symptoms is not expected
to manifest in this patient?

a. Dysphagia to solid food


b. Dysphagia to liquid food
c. Regurgitation of food
d. Odynophagia
e. Night cough

‫اﻹﺟﺎﺑﺔ ﻋﲆ اﻟﺼﻔﺤﺔ اﻟﺘﺎﻟﻴﺔ‬

Page - 685
Internal Medicine - Gastroenterology - Achalasia

Question 1/106

Question #1

A patient was diagnosed recently with achalasia. Which of the following symptoms is not expected
to manifest in this patient?

a. Dysphagia to solid food


b. Dysphagia to liquid food
c. Regurgitation of food
d. Odynophagia √
e. Night cough

Description

Achalasia is the most common esophageal motility disorder

Caused by degeneration of myenteric plexus →Hypertonic LES that fails to relax during swallowing

Long-standing, painless, slowly progressive dysphagia (to liquid more than solid foods) with periods
of remission and relapse is the most common symptom

Some patients developed severe chest pain due to esophageal spasms, Non-acidic regurgitation of
undigested food, Pulmonary aspiration (night cough, aspiration pneumonia)

Page - 686
Internal Medicine - Gastroenterology

Question 2/106

Question #2

A 39-year-old woman has a productive cough and a moderate fever. She has found it challenging to
swallow both food and liquids for the last 4 months. She complains of central chest pain and
regurgitation of undigested food. A chest X-ray shows a megaesophagus. What is the most likely
diagnosis?

a. Achalasia
b. Esophageal carcinoma
c. Esophageal web
d. Barrett’s esophagus
e. Oropharyngeal dysphagia

‫اﻹﺟﺎﺑﺔ ﻋﲆ اﻟﺼﻔﺤﺔ اﻟﺘﺎﻟﻴﺔ‬

Page - 687
Internal Medicine - Gastroenterology - Achalasia

Question 2/106

Question #2

A 39-year-old woman has a productive cough and a moderate fever. She has found it challenging to
swallow both food and liquids for the last 4 months. She complains of central chest pain and
regurgitation of undigested food. A chest X-ray shows a megaesophagus. What is the most likely
diagnosis?

a. Achalasia √
b. Esophageal carcinoma
c. Esophageal web
d. Barrett’s esophagus
e. Oropharyngeal dysphagia

Description

Achalasia is a motility disorder of the esophagus caused by degeneration of the myenteric plexus
leading to a hypertonic LES that fails to relax during swallowing.

e condition should be suspected when a patient presents with the following presentation:

Dysphagia to solid and liquid food


History of the cause, e.g., Chagas disease, systemic sclerosis, etc.
Regurgitation of undigested food
Rat tail (bird’s peak) sign on barium swallow

Esophageal manometry is the most accurate test in the diagnosis as it shows a failure of the lower
esophageal sphincter (LES) to relax.

Upper endoscopy should be done to rule out pseudoachalasia.

Treatment is achieved by either endoscopic pneumatic dilatation, endoscopic Botulinum toxin


injection, or surgical myotomy of the lower esophageal sphincter

Page - 688
Internal Medicine - Gastroenterology

Question 3/106

Question #3

For the past few months, a 27-year-old female has suered from recurrent chest infection and
dysphagia to solid and liquid. Her medical history was significant only for Chagas disease 8 years
ago. So, what is the most likely diagnosis?

a. Achalasia cardia
b. Cystic fibrosis
c. Peptic stricture
d. Esophageal cancer
e. Hypogammaglobulinemia

‫اﻹﺟﺎﺑﺔ ﻋﲆ اﻟﺼﻔﺤﺔ اﻟﺘﺎﻟﻴﺔ‬

Page - 689
Internal Medicine - Gastroenterology - Achalasia

Question 3/106

Question #3

For the past few months, a 27-year-old female has suered from recurrent chest infection and
dysphagia to solid and liquid. Her medical history was significant only for Chagas disease 8 years
ago. So, what is the most likely diagnosis?

a. Achalasia cardia √
b. Cystic fibrosis
c. Peptic stricture
d. Esophageal cancer
e. Hypogammaglobulinemia

Description

Achalasia is a motility disorder of the esophagus caused by degeneration of the myenteric plexus
leading to a hypertonic LES that fails to relax during swallowing.

e condition should be suspected when a patient presents with the following presentation:

Dysphagia to solid and liquid food


History of the cause, e.g., Chagas disease, systemic sclerosis, etc.
Regurgitation of undigested food
Rat tail (bird’s peak) sign on barium swallow

Esophageal manometry is the most accurate test in the diagnosis as it shows a failure of the lower
esophageal sphincter (LES) to relax.

Upper endoscopy should be done to rule out pseudoachalasia.

Treatment is achieved by either endoscopic pneumatic dilatation, endoscopic Botulinum toxin


injection, or surgical myotomy of the lower esophageal sphincter

Page - 690
Internal Medicine - Gastroenterology

Question 4/106

Question #4

A 43-year-old woman complains of dysphagia to solids and liquids. Her chest x-ray shows wide
mediastinum, and the barium swallow shows dilated esophagus with a tapering distal end. What is
the next step in the management of this patient?

a. Reassurance
b. Upper GI endoscopy
c. Start nifedipine
d. Start Proton Pump inhibitors
e. Test for H pylori

‫اﻹﺟﺎﺑﺔ ﻋﲆ اﻟﺼﻔﺤﺔ اﻟﺘﺎﻟﻴﺔ‬

Page - 691
Internal Medicine - Gastroenterology - Achalasia

Question 4/106

Question #4

A 43-year-old woman complains of dysphagia to solids and liquids. Her chest x-ray shows wide
mediastinum, and the barium swallow shows dilated esophagus with a tapering distal end. What is
the next step in the management of this patient?

a. Reassurance
b. Upper GI endoscopy √
c. Start nifedipine
d. Start Proton Pump inhibitors
e. Test for H pylori

Description

is patient is suering from achalasia.

Endoscopy should be done for this patient to rule out pseudoachalasia and then treat with
pneumatic dilatation of the lower esophageal sphincter

Achalasia is a motility disorder of the esophagus caused by degeneration of the myenteric plexus
leading to a hypertonic LES that fails to relax during swallowing.

e condition should be suspected when a patient presents with the following presentation:

Dysphagia to solid and liquid food


History of the cause, e.g., Chagas disease, systemic sclerosis, etc.
Regurgitation of undigested food
Rat tail (bird’s peak) sign on barium swallow

Esophageal manometry is the most accurate test in the diagnosis as it shows a failure of the lower
esophageal sphincter (LES) to relax.

Upper endoscopy should be done to rule out pseudoachalasia.

Treatment is achieved by either endoscopic pneumatic dilatation, endoscopic Botulinum toxin


injection, or surgical myotomy of the lower esophageal sphincter

Page - 692
Internal Medicine - Gastroenterology

Question 5/106

Question #5

A 35-year-old patient complains of dysphagia to liquids more than solids. Barium swallow and
manometry are done and confirm the presence of achalasia. Which of the following is the best next
step in managing this patient?

a. Barium swallow
b. Bronchoscopy
c. Chest CT scan
d. Esophageal manometry
e. Esophagogastroduodenoscopy

‫اﻹﺟﺎﺑﺔ ﻋﲆ اﻟﺼﻔﺤﺔ اﻟﺘﺎﻟﻴﺔ‬

Page - 693
Internal Medicine - Gastroenterology - Achalasia

Question 5/106

Question #5

A 35-year-old patient complains of dysphagia to liquids more than solids. Barium swallow and
manometry are done and confirm the presence of achalasia. Which of the following is the best next
step in managing this patient?

a. Barium swallow
b. Bronchoscopy
c. Chest CT scan
d. Esophageal manometry
e. Esophagogastroduodenoscopy √

Description

A barium swallow is the best initial step to diagnose achalasia

Esophageal manometry is the most accurate test in the diagnosis

Upper endoscopy is used to rule out pseudo-achalasia before starting treatment

Achalasia:

Achalasia is the most common esophageal motility disorder


It is caused by degeneration of the myenteric plexus leading to hypertonic LES that fails to
relax during swallowing
It is a premalignant condition; it can result in squamous cell carcinoma of the esophagus.
Long-standing, painless, slowly progressive dysphagia (to liquid more than solid foods) with
periods of remission and relapse (the most common symptom)
Non-acidic regurgitation of undigested food (heartburn is usually absent)
Chest x-ray, barium swallow (the best initial), and esophageal manometry (the most accurate)
are used in the diagnosis, while upper endoscopy is used to exclude pseudoachalasia.

Page - 694
Internal Medicine - Gastroenterology

Question 6/106

Question #6

A 33-year-old male patient has had worsening dysphagia for solid and liquid food for 2 months, and
he lost 5 kg in the past 2 months. His history is only remarkable for systemic sclerosis. What is the
most accurate method to confirm the diagnosis?

a. Upper GI endoscopy
b. Chest x-ray
c. Barium swallow
d. Esophageal manometry
e. Positive response to PPI

‫اﻹﺟﺎﺑﺔ ﻋﲆ اﻟﺼﻔﺤﺔ اﻟﺘﺎﻟﻴﺔ‬

Page - 695
Internal Medicine - Gastroenterology - Achalasia

Question 6/106

Question #6

A 33-year-old male patient has had worsening dysphagia for solid and liquid food for 2 months, and
he lost 5 kg in the past 2 months. His history is only remarkable for systemic sclerosis. What is the
most accurate method to confirm the diagnosis?

a. Upper GI endoscopy
b. Chest x-ray
c. Barium swallow
d. Esophageal manometry √
e. Positive response to PPI

Description

is patient is most likely suering from achalasia. is is suggested by the history of systemic
sclerosis and the typical dysphagia for liquid and solid food.

Esophageal manometry is the most accurate test to diagnose achalasia.

Achalasia:

Achalasia is the most common esophageal motility disorder


It is caused by degeneration of the myenteric plexus leading to hypertonic LES that fails to
relax during swallowing
It is a premalignant condition; it can result in squamous cell carcinoma of the esophagus.
Long-standing, painless, slowly progressive dysphagia (to liquid more than solid foods) with
periods of remission and relapse (the most common symptom)
Non-acidic regurgitation of undigested food (heartburn is usually absent)
Chest x-ray, barium swallow (the best initial), and esophageal manometry (the most accurate)
are used in the diagnosis, while upper endoscopy is used to exclude pseudoachalasia.

Page - 696
Internal Medicine - Gastroenterology

Question 7/106

Question #7

A 30-year-old male patient was diagnosed with achalasia. e following symptoms favor the
diagnosis of achalasia, except:

a. Dysphagia to solid food


b. Dysphagia to liquids
c. Odynophagia
d. Regurgitation of undigested food
e. Cough and aspiration

‫اﻹﺟﺎﺑﺔ ﻋﲆ اﻟﺼﻔﺤﺔ اﻟﺘﺎﻟﻴﺔ‬

Page - 697
Internal Medicine - Gastroenterology - Achalasia

Question 7/106

Question #7

A 30-year-old male patient was diagnosed with achalasia. e following symptoms favor the
diagnosis of achalasia, except:

a. Dysphagia to solid food


b. Dysphagia to liquids
c. Odynophagia √
d. Regurgitation of undigested food
e. Cough and aspiration

Description

Achalasia usually does not present with odynophagia.

Achalasia:

Achalasia is the most common esophageal motility disorder


It is caused by degeneration of the myenteric plexus leading to hypertonic LES that fails to
relax during swallowing
It is a premalignant condition; it can result in squamous cell carcinoma of the esophagus.
Long-standing, painless, slowly progressive dysphagia (to liquid more than solid foods) with
periods of remission and relapse (the most common symptom)
Non-acidic regurgitation of undigested food (heartburn is usually absent)
Chest x-ray, barium swallow (the best initial), and esophageal manometry (the most accurate)
are used in the diagnosis, while upper endoscopy is used to exclude pseudoachalasia.

Page - 698
Page - 699
Internal Medicine - Gastroenterology

Question 8/106

Question #8

A 35-year-old patient with a history of dysphagia to liquids more than solids is suspected of having
achalasia. Which of the following is the most accurate test for this patient?

a. Barium swallow
b. Bronchoscopy
c. Chest CT scan
d. Esophageal manometry
e. Esophagogastroduodenoscopy

‫اﻹﺟﺎﺑﺔ ﻋﲆ اﻟﺼﻔﺤﺔ اﻟﺘﺎﻟﻴﺔ‬

Page - 700
Internal Medicine - Gastroenterology - Achalasia

Question 8/106

Question #8

A 35-year-old patient with a history of dysphagia to liquids more than solids is suspected of having
achalasia. Which of the following is the most accurate test for this patient?

a. Barium swallow
b. Bronchoscopy
c. Chest CT scan
d. Esophageal manometry √
e. Esophagogastroduodenoscopy

Description

A barium swallow is the best initial step to diagnose achalasia

Esophageal manometry is the most accurate test in the diagnosis

Upper endoscopy is used to rule out pseudo-achalasia before starting treatment

Achalasia:

Achalasia is the most common esophageal motility disorder


It is caused by degeneration of the myenteric plexus leading to hypertonic LES that fails to
relax during swallowing
It is a premalignant condition; it can result in squamous cell carcinoma of the esophagus.
Long-standing, painless, slowly progressive dysphagia (to liquid more than solid foods) with
periods of remission and relapse (the most common symptom)
Non-acidic regurgitation of undigested food (heartburn is usually absent)
Chest x-ray, barium swallow (the best initial), and esophageal manometry (the most accurate)
are used in the diagnosis, while upper endoscopy is used to exclude pseudoachalasia.

Page - 701
Internal Medicine - Gastroenterology

Question 9/106

Question #9

A 35-year-old patient with a history of dysphagia to liquids more than solids is suspected of having
achalasia. Which of the following is the best initial test for this patient?

a. Barium swallow
b. Bronchoscopy
c. Chest CT scan
d. Esophageal manometry
e. Esophagogastroduodenoscopy

‫اﻹﺟﺎﺑﺔ ﻋﲆ اﻟﺼﻔﺤﺔ اﻟﺘﺎﻟﻴﺔ‬

Page - 702
Internal Medicine - Gastroenterology - Achalasia

Question 9/106

Question #9

A 35-year-old patient with a history of dysphagia to liquids more than solids is suspected of having
achalasia. Which of the following is the best initial test for this patient?

a. Barium swallow √
b. Bronchoscopy
c. Chest CT scan
d. Esophageal manometry
e. Esophagogastroduodenoscopy

Description

A barium swallow is the best initial step to diagnose achalasia

Esophageal manometry is the most accurate test in the diagnosis

Upper endoscopy is used to rule out pseudo-achalasia before starting treatment

Achalasia:

Achalasia is the most common esophageal motility disorder


It is caused by degeneration of the myenteric plexus leading to hypertonic LES that fails to
relax during swallowing
It is a premalignant condition; it can result in squamous cell carcinoma of the esophagus.
Long-standing, painless, slowly progressive dysphagia (to liquid more than solid foods) with
periods of remission and relapse (the most common symptom)
Non-acidic regurgitation of undigested food (heartburn is usually absent)
Chest x-ray, barium swallow (the best initial), and esophageal manometry (the most accurate)
are used in the diagnosis, while upper endoscopy is used to exclude pseudoachalasia.

Page - 703
Internal Medicine - Gastroenterology

Question 10/106

Question #10

A 39-year-old male patient has had worsening diculty swallowing for the past 6 months. He has
not experienced weight loss and complains of regurgitation of undigested food. In addition, the
patient has had a history of recurrent respiratory infections over the past 3 years. What is the most
likely diagnosis?

a. Achalasia
b. Esophageal carcinoma
c. Esophageal web
d. Barrett’s esophagus
e. Oropharyngeal dysphagia

‫اﻹﺟﺎﺑﺔ ﻋﲆ اﻟﺼﻔﺤﺔ اﻟﺘﺎﻟﻴﺔ‬

Page - 704
Internal Medicine - Gastroenterology - Achalasia

Question 10/106

Question #10

A 39-year-old male patient has had worsening diculty swallowing for the past 6 months. He has
not experienced weight loss and complains of regurgitation of undigested food. In addition, the
patient has had a history of recurrent respiratory infections over the past 3 years. What is the most
likely diagnosis?

a. Achalasia √
b. Esophageal carcinoma
c. Esophageal web
d. Barrett’s esophagus
e. Oropharyngeal dysphagia

Description

Achalasia is a motility disorder of the esophagus caused by degeneration of the myenteric plexus
leading to a hypertonic LES that fails to relax during swallowing.

e condition should be suspected when a patient presents with the following presentation:

Dysphagia to solid and liquid food


History of the cause, e.g., Chagas disease, systemic sclerosis, etc.
Regurgitation of undigested food
Rat tail (bird’s peak) sign on barium swallow

Esophageal manometry is the most accurate test in the diagnosis as it shows a failure of the lower
esophageal sphincter (LES) to relax.

Upper endoscopy should be done to rule out pseudoachalasia.

Treatment is achieved by either endoscopic pneumatic dilatation, endoscopic Botulinum toxin


injection, or surgical myotomy of the lower esophageal sphincter

Page - 705
Internal Medicine - Gastroenterology

Question 11/106

Question #11

A previously healthy 58-year-old male comes to your oce to evaluate his dysphagia to liquid and
solid food. He has had this condition for 2 years. On barium swallow, a smooth narrowing on the
lower esophagus is noted. What is the most likely diagnosis?

a. Myasthenia gravis
b. Cerebrovascular accident
c. Esophageal web
d. Achalasia
e. Gastroesophageal reflux

‫اﻹﺟﺎﺑﺔ ﻋﲆ اﻟﺼﻔﺤﺔ اﻟﺘﺎﻟﻴﺔ‬

Page - 706
Internal Medicine - Gastroenterology - Achalasia

Question 11/106

Question #11

A previously healthy 58-year-old male comes to your oce to evaluate his dysphagia to liquid and
solid food. He has had this condition for 2 years. On barium swallow, a smooth narrowing on the
lower esophagus is noted. What is the most likely diagnosis?

a. Myasthenia gravis
b. Cerebrovascular accident
c. Esophageal web
d. Achalasia √
e. Gastroesophageal reflux

Description

Achalasia is a motility disorder of the esophagus caused by degeneration of the myenteric plexus
leading to a hypertonic LES that fails to relax during swallowing.

e condition should be suspected when a patient presents with the following presentation:

Dysphagia to solid and liquid food


History of the cause, e.g., Chagas disease, systemic sclerosis, etc.
Regurgitation of undigested food
Rat tail (bird’s peak) sign on barium swallow

Esophageal manometry is the most accurate test in the diagnosis as it shows a failure of the lower
esophageal sphincter (LES) to relax.

Upper endoscopy should be done to rule out pseudoachalasia.

Treatment is achieved by the endoscopic pneumatic dilatation, endoscopic Botulinum toxin


injection, or surgical myotomy of the lower esophageal sphincter.

Page - 707
Internal Medicine - Gastroenterology

Question 12/106

Question #12

A 54-year-old male patient has had severe GERD for 20 years. Upper endoscopy confirms the
presence of Barrett’s esophagus (BE). Which of the following statements is true regarding this
condition?

a. e severity of symptoms here is due to Barrett’s esophagus (BE)


b. BE is an inflammation of the lower esophagus due to GERD
c. e risk of esophageal adenocarcinoma is less than 1% per year
d. Clinically, the patient with BE presents with dysphagia
e. Anti-reflux surgery prevents BE from progressing to adenocarcinoma

‫اﻹﺟﺎﺑﺔ ﻋﲆ اﻟﺼﻔﺤﺔ اﻟﺘﺎﻟﻴﺔ‬

Page - 708
Internal Medicine - Gastroenterology - Barrette's Esophagus (BE)

Question 12/106

Question #12

A 54-year-old male patient has had severe GERD for 20 years. Upper endoscopy confirms the
presence of Barrett’s esophagus (BE). Which of the following statements is true regarding this
condition?

a. e severity of symptoms here is due to Barrett’s esophagus (BE)


b. BE is an inflammation of the lower esophagus due to GERD
c. e risk of esophageal adenocarcinoma is less than 1% per year √
d. Clinically, the patient with BE presents with dysphagia
e. Anti-reflux surgery prevents BE from progressing to adenocarcinoma

Description

In Barrett’s esophagus (BE), the Squamous lining of the lower esophagus is replaced by columnar
lining due to chronic exposure to acidity due to GERD (cellular metaplasia)

Barrett’s esophagus (BE) is a pre-malignant condition “0.5%/year risk of malignant transformation”


→ can cause adenocarcinoma of the esophagus

Anti-reflux surgery does not prevent the progression of BE to Adenocarcinoma.

BE is clinically asymptomatic, and it is only discovered by upper endoscopy.

Page - 709
Internal Medicine - Gastroenterology

Question 13/106

Question #13

A patient with a long-standing history of GERD was diagnosed by upper endoscopy and biopsy to
have Barrett’s esophagus (BE). Which of the following is false about this condition?

a. e risk of developing adenocarcinoma is <1%


b. e risk of having squamous cell carcinoma is 10%
c. Patient with Barrett’s esophagus has minimal symptoms
d. Anti-reflux therapy doesn’t prevent the progression of BE to a cancer
e. Endoscopic ablation is one of the treatment options

‫اﻹﺟﺎﺑﺔ ﻋﲆ اﻟﺼﻔﺤﺔ اﻟﺘﺎﻟﻴﺔ‬

Page - 710
Internal Medicine - Gastroenterology - Barrette's Esophagus (BE)

Question 13/106

Question #13

A patient with a long-standing history of GERD was diagnosed by upper endoscopy and biopsy to
have Barrett’s esophagus (BE). Which of the following is false about this condition?

a. e risk of developing adenocarcinoma is <1%


b. e risk of having squamous cell carcinoma is 10% √
c. Patient with Barrett’s esophagus has minimal symptoms
d. Anti-reflux therapy doesn’t prevent the progression of BE to a cancer
e. Endoscopic ablation is one of the treatment options

Description

e Squamous lining of the lower esophagus is replaced by columnar lining due to chronic exposure
to acidity due to GERD (cellular metaplasia)

Barrett’s esophagus (BE) has the potential to progress to an adenocarcinoma

0.5%/year risk of malignant transformation to adenocarcinoma, not squamous cell carcinoma

Page - 711
Internal Medicine - Gastroenterology

Question 14/106

Question #14

Which of the following is the most appropriate pathological description of Barrett’s Esophagus (BE)?

a. Squamous epithelium is replaced by columnar epithelium


b. e columnar epithelium is replaced by squamous epithelium
c. Hyperplasia of the lower esophageal squamous cells
d. Hyperplasia of the lower esophageal muscular layer
e. Ulcerations of the lower esophagus

‫اﻹﺟﺎﺑﺔ ﻋﲆ اﻟﺼﻔﺤﺔ اﻟﺘﺎﻟﻴﺔ‬

Page - 712
Internal Medicine - Gastroenterology - Barrette's Esophagus (BE)

Question 14/106

Question #14

Which of the following is the most appropriate pathological description of Barrett’s Esophagus (BE)?

a. Squamous epithelium is replaced by columnar epithelium √


b. e columnar epithelium is replaced by squamous epithelium
c. Hyperplasia of the lower esophageal squamous cells
d. Hyperplasia of the lower esophageal muscular layer
e. Ulcerations of the lower esophagus

Description

In Barrett’s esophagus (BE), the Squamous lining of the lower esophagus is replaced by columnar
lining due to chronic exposure to acidity due to GERD (cellular metaplasia)

Barrett’s esophagus (BE) is a pre-malignant condition “0.5%/year risk of malignant transformation”


→ can cause adenocarcinoma of the esophagus

Anti-reflux surgery does not prevent the progression of BE to Adenocarcinoma.

BE is clinically asymptomatic, and it is only discovered by upper endoscopy

Page - 713
Internal Medicine - Gastroenterology

Question 15/106

Question #15

A 22-year-old female patient has a 6 months history of diarrhea associated with weight loss and
easy bruising. Her examination is normal except for a chronic blistering skin consistent with
dermatitis herpetiformis. Which of the following abnormalities is expected to present in this
patient?

a. A low blood glucose level


b. Elevated total serum protein level
c. Low serum ferritin level
d. High Vitamin K level
e. Hypercalcemia

‫اﻹﺟﺎﺑﺔ ﻋﲆ اﻟﺼﻔﺤﺔ اﻟﺘﺎﻟﻴﺔ‬

Page - 714
Internal Medicine - Gastroenterology - Celiac Disease

Question 15/106

Question #15

A 22-year-old female patient has a 6 months history of diarrhea associated with weight loss and
easy bruising. Her examination is normal except for a chronic blistering skin consistent with
dermatitis herpetiformis. Which of the following abnormalities is expected to present in this
patient?

a. A low blood glucose level


b. Elevated total serum protein level
c. Low serum ferritin level √
d. High Vitamin K level
e. Hypercalcemia

Description

Iron deficiency anemia secondary to malabsorption is a known feature of celiac disease.

Malabsorption in celiac disease will result in low protein levels as well as a vitamin D deficiency
leading to low levels of calcium and elevated alkaline phosphatase

Celiac disease (gluten-sensitive enteropathy):

It is an autoimmune disease due to exposure to gliadin (gliadin is a product of gluten


breakdown) associated with HLA-DQ2 (chromosome 6) found in 80-90%, HLA-DQ8
It usually starts at age 6 months when adding food other than milk (It never manifests at
birth)
e clinical features include bloating, abdominal pain, weight loss, chronic diarrhea, and
symptoms of nutritional deficiencies.
e most accurate test is intestinal biopsy showing atrophic villi
e treatment is achieved by a strict gluten-free diet and the treatment of complications.
e most dangerous complications are small bowel lymphoma and adenocarcinoma

Page - 715
Internal Medicine - Gastroenterology

Question 16/106

Question #16

A 26-year-old female has had intermittent diarrhea and bloating for 6 months. Her stool is frothy
and dicult to flush down in the toilet. Further investigations show iron deficiency anemia and
positive endomysial antibodies. What is the most likely diagnosis?

a. Ulcerative colitis
b. Crohn’s disease
c. Celiac disease
d. Irritable bowel syndrome
e. Microscopic colitis

‫اﻹﺟﺎﺑﺔ ﻋﲆ اﻟﺼﻔﺤﺔ اﻟﺘﺎﻟﻴﺔ‬

Page - 716
Internal Medicine - Gastroenterology - Celiac Disease

Question 16/106

Question #16

A 26-year-old female has had intermittent diarrhea and bloating for 6 months. Her stool is frothy
and dicult to flush down in the toilet. Further investigations show iron deficiency anemia and
positive endomysial antibodies. What is the most likely diagnosis?

a. Ulcerative colitis
b. Crohn’s disease
c. Celiac disease √
d. Irritable bowel syndrome
e. Microscopic colitis

Description

Anti-gliadin, anti-endomysial, and anti-tissue transglutaminase antibodies are associated with


celiac disease

e presence of frothy stool is suggestive of steatorrhea secondary to malabsorption in celiac


disease

Iron deficiency anemia is a common complication of celiac disease.

Celiac disease (gluten-sensitive enteropathy):

It is an autoimmune disease due to exposure to gliadin (gliadin is a product of gluten


breakdown) associated with HLA-DQ2 (chromosome 6) found in 80-90%, HLA-DQ8
It usually starts at age 6 months when adding food other than milk (It never manifests at
birth)
e clinical features include bloating, abdominal pain, weight loss, chronic diarrhea, and
symptoms of nutritional deficiencies.
e most accurate test is intestinal biopsy showing atrophic villi
e treatment is achieved by a strict gluten-free diet and the treatment of complications.
e most dangerous complications are small bowel lymphoma and adenocarcinoma

Page - 717
Internal Medicine - Gastroenterology

Question 17/106

Question #17

A 33-year-old female patient has intermittent abdominal bloating and diarrhea that is more
prominent aer eating bread. She has no weight loss or anemia. What is the most likely diagnosis?

a. Ulcerative colitis
b. Crohn’s disease
c. Gastroenteritis
d. Pyelonephritis
e. Celiac disease

‫اﻹﺟﺎﺑﺔ ﻋﲆ اﻟﺼﻔﺤﺔ اﻟﺘﺎﻟﻴﺔ‬

Page - 718
Internal Medicine - Gastroenterology - Celiac Disease

Question 17/106

Question #17

A 33-year-old female patient has intermittent abdominal bloating and diarrhea that is more
prominent aer eating bread. She has no weight loss or anemia. What is the most likely diagnosis?

a. Ulcerative colitis
b. Crohn’s disease
c. Gastroenteritis
d. Pyelonephritis
e. Celiac disease √

Description

is is a classic scenario of celiac disease. It is more common in females, and it is more prominent
aer ingestion of gluten-containing food.

Celiac disease (gluten-sensitive enteropathy):

It is an autoimmune disease due to exposure to gliadin (gliadin is a product of gluten


breakdown) associated with HLA-DQ2 (chromosome 6) found in 80-90%, HLA-DQ8
It usually starts at age 6 months when adding food other than milk (It never manifests at
birth)
e clinical features include bloating, abdominal pain, weight loss, chronic diarrhea, and
symptoms of nutritional deficiencies.
e most accurate test is intestinal biopsy showing atrophic villi
e treatment is achieved by a strict gluten-free diet and the treatment of complications.
e most dangerous complications are small bowel lymphoma and adenocarcinoma

Page - 719
Internal Medicine - Gastroenterology

Question 18/106

Question #18

A 25-year-old female has had intermittent diarrhea and bloating for 6 months. she is a known case
of hypothyroidism and type-1-DM. Her investigations are positive for immunoglobulin A tissue
transglutaminase antibody. What is the most appropriate next step in the management of this
patient?

a. Duodenal biopsy
b. Colonoscopy
c. Sweat chloride test
d. Start gluten-free diet
e. Gastric emptying study

‫اﻹﺟﺎﺑﺔ ﻋﲆ اﻟﺼﻔﺤﺔ اﻟﺘﺎﻟﻴﺔ‬

Page - 720
Internal Medicine - Gastroenterology - Celiac Disease

Question 18/106

Question #18

A 25-year-old female has had intermittent diarrhea and bloating for 6 months. she is a known case
of hypothyroidism and type-1-DM. Her investigations are positive for immunoglobulin A tissue
transglutaminase antibody. What is the most appropriate next step in the management of this
patient?

a. Duodenal biopsy √
b. Colonoscopy
c. Sweat chloride test
d. Start gluten-free diet
e. Gastric emptying study

Description

ere are many points in the question that point to celiac disease as the most likely diagnosis:

- e patient is female (it is more common in females)

- e patient has associated autoimmune diseases (T1DM and Hypothyroidism)

- e typical symptoms of bloating and intermittent diarrhea

- e presence of positive Anti-tTG antibody

is patient should start on a gluten-free diet, but you should confirm the diagnosis by doing upper
endoscopy and duodenal biopsy.

e gastric emptying study is used in diabetic patients who are suspected of having gastroparesis,
but the patient here has positive Anti-tTG, which guides us to think about celiac disease

Celiac disease (gluten-sensitive enteropathy):

It is an autoimmune disease due to exposure to gliadin (gliadin is a product of gluten


breakdown) associated with HLA-DQ2 (chromosome 6) found in 80-90%, HLA-DQ8
It usually starts at age 6 months when adding food other than milk (It never manifests at
birth)
e clinical features include bloating, abdominal pain, weight loss, chronic diarrhea, and
Page - 721
symptoms of nutritional deficiencies.
e most accurate test is intestinal biopsy showing atrophic villi
e treatment is achieved by a strict gluten-free diet and the treatment of complications.
e most dangerous complications are small bowel lymphoma and adenocarcinoma

Page - 722
Internal Medicine - Gastroenterology

Question 19/106

Question #19

A 22-year-old female was diagnosed with celiac disease. Of the following, which foods are allowed
to eat by this patient?

a. Corn and rice


b. Oats
c. Wheat
d. Barley
e. Ray

‫اﻹﺟﺎﺑﺔ ﻋﲆ اﻟﺼﻔﺤﺔ اﻟﺘﺎﻟﻴﺔ‬

Page - 723
Internal Medicine - Gastroenterology - Celiac Disease

Question 19/106

Question #19

A 22-year-old female was diagnosed with celiac disease. Of the following, which foods are allowed
to eat by this patient?

a. Corn and rice √


b. Oats
c. Wheat
d. Barley
e. Ray

Description

Celiac disease is an autoimmune disease due to exposure to gliadin (gliadin is a product of gluten
breakdown)

Gluten is a protein present in Rye, Barley, Oats, Wheat

Rice and corn are gluten-free and are safe and allowed to be eaten by celiac disease patients

Celiac disease (gluten-sensitive enteropathy):

It is an autoimmune disease due to exposure to gliadin (gliadin is a product of gluten


breakdown) associated with HLA-DQ2 (chromosome 6) found in 80-90%, HLA-DQ8
It usually starts at age 6 months when adding food other than milk (It never manifests at
birth)
e clinical features include bloating, abdominal pain, weight loss, chronic diarrhea, and
symptoms of nutritional deficiencies.
e most accurate test is intestinal biopsy showing atrophic villi
e treatment is achieved by a strict gluten-free diet and the treatment of complications.
e most dangerous complications are small bowel lymphoma and adenocarcinoma

Page - 724
Internal Medicine - Gastroenterology

Question 20/106

Question #20

A 25-year-old female has had intermittent diarrhea and bloating for 6 months. she is a known case
of hypothyroidism and type-1-DM. Her investigations are positive for immunoglobulin A tissue
transglutaminase antibody. In addition, her duodenal biopsy confirms the suspected diagnosis.
What is the most appropriate next step in the management of this patient?

a. Repeat duodenal biopsy in 6 months


b. Colonoscopy
c. Sweat chloride test
d. Start gluten-free diet
e. Gastric emptying study

‫اﻹﺟﺎﺑﺔ ﻋﲆ اﻟﺼﻔﺤﺔ اﻟﺘﺎﻟﻴﺔ‬

Page - 725
Internal Medicine - Gastroenterology - Celiac Disease

Question 20/106

Question #20

A 25-year-old female has had intermittent diarrhea and bloating for 6 months. she is a known case
of hypothyroidism and type-1-DM. Her investigations are positive for immunoglobulin A tissue
transglutaminase antibody. In addition, her duodenal biopsy confirms the suspected diagnosis.
What is the most appropriate next step in the management of this patient?

a. Repeat duodenal biopsy in 6 months


b. Colonoscopy
c. Sweat chloride test
d. Start gluten-free diet √
e. Gastric emptying study

Description

ere are many points in the question that point to celiac disease as the most likely diagnosis:

- e patient is female (it is more common in females)

- e patient has associated autoimmune diseases (T1DM and Hypothyroidism)

- e typical symptoms of bloating and intermittent diarrhea

- e presence of positive Anti-tTG antibody

is patient should start on a gluten-free diet, but you should confirm the diagnosis by doing upper
endoscopy and duodenal biopsy.

e gastric emptying study is used in diabetic patients suspected of having gastroparesis, but the
patient here has positive Anti-tTG, which guides us to think about celiac disease.

Celiac disease (gluten-sensitive enteropathy):

It is an autoimmune disease due to exposure to gliadin (gliadin is a product of gluten


breakdown) associated with HLA-DQ2 (chromosome 6) found in 80-90%, HLA-DQ8
It usually starts at age 6 months when adding food other than milk (It never manifests at
birth)
e clinical features include bloating, abdominal pain, weight loss, chronic diarrhea, and
Page - 726
symptoms of nutritional deficiencies.
e most accurate test is intestinal biopsy showing atrophic villi
e treatment is achieved by a strict gluten-free diet and the treatment of complications.
e most dangerous complications are small bowel lymphoma and adenocarcinoma

Page - 727
Internal Medicine - Gastroenterology

Question 21/106

Question #21

A 22-year-old female patient has a 6 months history of diarrhea associated with weight loss and
easy bruising. Her examination is normal except for a chronic blistering skin consistent with
dermatitis herpetiformis. What is the most accurate diagnostic test to confirm the diagnosis?

a. Abdominal CT scan
b. Bowel biopsy
c. Colonoscopy
d. Gluten-free diet
e. Sweat chloride test

‫اﻹﺟﺎﺑﺔ ﻋﲆ اﻟﺼﻔﺤﺔ اﻟﺘﺎﻟﻴﺔ‬

Page - 728
Internal Medicine - Gastroenterology - Celiac Disease

Question 21/106

Question #21

A 22-year-old female patient has a 6 months history of diarrhea associated with weight loss and
easy bruising. Her examination is normal except for a chronic blistering skin consistent with
dermatitis herpetiformis. What is the most accurate diagnostic test to confirm the diagnosis?

a. Abdominal CT scan
b. Bowel biopsy √
c. Colonoscopy
d. Gluten-free diet
e. Sweat chloride test

Description

e presence of weight loss, fatty diarrhea, and features suggestive of vitamin K deficiency (easy
bruising) are all indicative of malabsorption

Dermatitis herpetiformis and features of malabsorption indicate Celiac disease as the most likely
diagnosis.

Endoscopy with biopsy is the most accurate test to do for celiac disease

e biopsy will show villous atrophy if the disease is present

Celiac disease (gluten-sensitive enteropathy):

It is an autoimmune disease due to exposure to gliadin (gliadin is a product of gluten


breakdown) associated with HLA-DQ2 (chromosome 6) found in 80-90%, HLA-DQ8
It usually starts at age 6 months when adding food other than milk (It never manifests at
birth)
e clinical features include bloating, abdominal pain, weight loss, chronic diarrhea, and
symptoms of nutritional deficiencies.
e most accurate test is intestinal biopsy showing atrophic villi
e treatment is achieved by a strict gluten-free diet and the treatment of complications.
e most dangerous complications are small bowel lymphoma and adenocarcinoma

Page - 729
Internal Medicine - Gastroenterology

Question 22/106

Question #22

A 25-year-old female has had intermittent diarrhea and bloating for 6 months. she is a known case
of hypothyroidism and type-1-DM. Her investigations are positive for immunoglobulin A tissue
transglutaminase antibody. What is the most likely finding in the duodenal biopsy?

a. Caseating granuloma
b. Non-caseating granuloma
c. Cystic formation
d. Villous atrophy
e. Eosinophilic infiltration

‫اﻹﺟﺎﺑﺔ ﻋﲆ اﻟﺼﻔﺤﺔ اﻟﺘﺎﻟﻴﺔ‬

Page - 730
Internal Medicine - Gastroenterology - Celiac Disease

Question 22/106

Question #22

A 25-year-old female has had intermittent diarrhea and bloating for 6 months. she is a known case
of hypothyroidism and type-1-DM. Her investigations are positive for immunoglobulin A tissue
transglutaminase antibody. What is the most likely finding in the duodenal biopsy?

a. Caseating granuloma
b. Non-caseating granuloma
c. Cystic formation
d. Villous atrophy √
e. Eosinophilic infiltration

Description

ere are many points in the question that point to celiac disease as the most likely diagnosis:

- e patient is female (it is more common in females)

- e patient has associated autoimmune diseases (T1DM and Hypothyroidism)

- e typical symptoms of bloating and intermittent diarrhea

- e presence of positive Anti-tTG antibody

In celiac disease, gluten exposure causes characteristic changes in the small intestine lining,
resulting in malabsorption. e proximal part of the small bowel is more aected than the distal
part, and the changes are reversible if a gluten-free diet is strictly adopted.

Celiac disease (gluten-sensitive enteropathy):

It is an autoimmune disease due to exposure to gliadin (gliadin is a product of gluten


breakdown) associated with HLA-DQ2 (chromosome 6) found in 80-90%, HLA-DQ8
It usually starts at age 6 months when adding food other than milk (It never manifests at
birth)
e clinical features include bloating, abdominal pain, weight loss, chronic diarrhea, and
symptoms of nutritional deficiencies.
e most accurate test is intestinal biopsy showing atrophic villi
Page - 731
e treatment is achieved by a strict gluten-free diet and the treatment of complications.
e most dangerous complications are small bowel lymphoma and adenocarcinoma

Page - 732
Internal Medicine - Gastroenterology

Question 23/106

Question #23

A 22-year-old female patient has a 6 months history of diarrhea associated with weight loss and
easy bruising. Her examination is normal except for a chronic blistering skin consistent with
dermatitis herpetiformis. Regarding the most likely diagnosis, all are false except:

a. Males are more aected than females


b. It may be associated with type 2 DM
c. Symptoms are limited to the GI tract
d. Bowel biopsy will be falsely negative if the patient is on a gluten-free diet
e. If the patient is asymptomatic, there is no need for a gluten-free diet

‫اﻹﺟﺎﺑﺔ ﻋﲆ اﻟﺼﻔﺤﺔ اﻟﺘﺎﻟﻴﺔ‬

Page - 733
Internal Medicine - Gastroenterology - Celiac Disease

Question 23/106

Question #23

A 22-year-old female patient has a 6 months history of diarrhea associated with weight loss and
easy bruising. Her examination is normal except for a chronic blistering skin consistent with
dermatitis herpetiformis. Regarding the most likely diagnosis, all are false except:

a. Males are more aected than females


b. It may be associated with type 2 DM
c. Symptoms are limited to the GI tract
d. Bowel biopsy will be falsely negative if the patient is on a gluten-free diet √
e. If the patient is asymptomatic, there is no need for a gluten-free diet

Description

If the patient is on a Gluten-free diet, the biopsy and his serology tests for celiac disease will be
negative (normal), and the symptoms will disappear

Like other autoimmune diseases, Females are more aected than males in celiac disease (1:13 more
common in women).

Celiac disease is associated with autoimmune diseases like Type 1 DM thyroid disease and
rheumatoid arthritis.

All patients should be treated with a gluten-free diet regardless of symptoms, including those with
isolated dermatitis herpetiformis

Page - 734
Internal Medicine - Gastroenterology

Question 24/106

Question #24

A 22-year-old female patient has had diarrhea, abdominal pain, and bloating for 4 months. she has a
blistering rash on her elbow. Her upper endoscopy and biopsy demonstrate shortening of the enteric
villi. What is the most likely diagnosis?

a. Crohn’s disease
b. Amebiasis
c. Giardiasis
d. Intestinal ischemia
e. Celiac disease

‫اﻹﺟﺎﺑﺔ ﻋﲆ اﻟﺼﻔﺤﺔ اﻟﺘﺎﻟﻴﺔ‬

Page - 735
Internal Medicine - Gastroenterology - Celiac Disease

Question 24/106

Question #24

A 22-year-old female patient has had diarrhea, abdominal pain, and bloating for 4 months. she has a
blistering rash on her elbow. Her upper endoscopy and biopsy demonstrate shortening of the enteric
villi. What is the most likely diagnosis?

a. Crohn’s disease
b. Amebiasis
c. Giardiasis
d. Intestinal ischemia
e. Celiac disease √

Description

is patient is a young female who has symptoms of celiac disease and the associated rash
(Dermatitis herpetiformis)

e biopsy, which is the most accurate, confirms the presence of atrophic villi

Celiac disease (gluten-sensitive enteropathy):

It is an autoimmune disease due to exposure to gliadin (gliadin is a product of gluten


breakdown) associated with HLA-DQ2 (chromosome 6) found in 80-90%, HLA-DQ8
It usually starts at age 6 months when adding food other than milk (It never manifests at
birth)
e clinical features include bloating, abdominal pain, weight loss, chronic diarrhea, and
symptoms of nutritional deficiencies.
e most accurate test is intestinal biopsy showing atrophic villi
e treatment is achieved by a strict gluten-free diet and the treatment of complications.
e most dangerous complications are small bowel lymphoma and adenocarcinoma

Page - 736
Internal Medicine - Gastroenterology

Question 25/106

Question #25

A 22-year-old female has been on a gluten-free diet for 6 months aer being suspected of having
celiac disease. She presents to confirm her diagnosis of the disease. What is the most appropriate
step in the management of this patient?

a. Upper endoscopy and biopsy


b. Stop the gluten-free diet
c. Sweat chloride test
d. Stool analysis
e. Anti-tTG antibody

‫اﻹﺟﺎﺑﺔ ﻋﲆ اﻟﺼﻔﺤﺔ اﻟﺘﺎﻟﻴﺔ‬

Page - 737
Internal Medicine - Gastroenterology - Celiac Disease

Question 25/106

Question #25

A 22-year-old female has been on a gluten-free diet for 6 months aer being suspected of having
celiac disease. She presents to confirm her diagnosis of the disease. What is the most appropriate
step in the management of this patient?

a. Upper endoscopy and biopsy


b. Stop the gluten-free diet √
c. Sweat chloride test
d. Stool analysis
e. Anti-tTG antibody

Description

e biopsy findings of celiac disease and even the serology test for antibodies will be falsely negative
if the patient is on a gluten-free diet. In addition, there will be a rapid response and reversibility if a
gluten-free diet is taken.

You should reintroduce the gluten for at least 6 weeks before testing for celiac disease.

Celiac disease (gluten-sensitive enteropathy):

It is an autoimmune disease due to exposure to gliadin (gliadin is a product of gluten


breakdown) associated with HLA-DQ2 (chromosome 6) found in 80-90%, HLA-DQ8
It usually starts at age 6 months when adding food other than milk (It never manifests at
birth)
e clinical features include bloating, abdominal pain, weight loss, chronic diarrhea, and
symptoms of nutritional deficiencies.
e most accurate test is intestinal biopsy showing atrophic villi
e treatment is achieved by a strict gluten-free diet and the treatment of complications.
e most dangerous complications are small bowel lymphoma and adenocarcinoma

Page - 738
Internal Medicine - Gastroenterology

Question 26/106

Question #26

A 42-year-old female has been on a gluten-free diet for 15 years aer being diagnosed with celiac
disease. She worsens the classic symptoms of celiac disease, such as abdominal pain, diarrhea, and
bloating. In addition, the patient has significant weight loss. e biopsy of her small intestine will
show which of the following?

a. Intestinal Tuberculosis
b. Non-caseating granuloma
c. Cystic formation
d. Lymphoma
e. Basophilic infiltration

‫اﻹﺟﺎﺑﺔ ﻋﲆ اﻟﺼﻔﺤﺔ اﻟﺘﺎﻟﻴﺔ‬

Page - 739
Internal Medicine - Gastroenterology - Celiac Disease

Question 26/106

Question #26

A 42-year-old female has been on a gluten-free diet for 15 years aer being diagnosed with celiac
disease. She worsens the classic symptoms of celiac disease, such as abdominal pain, diarrhea, and
bloating. In addition, the patient has significant weight loss. e biopsy of her small intestine will
show which of the following?

a. Intestinal Tuberculosis
b. Non-caseating granuloma
c. Cystic formation
d. Lymphoma √
e. Basophilic infiltration

Description

Celiac disease may be complicated by enteropathy-associated T-cell lymphoma.

e biopsy may show lymphomatous infiltrates, and mixed inflammatory infiltrate

Celiac disease (gluten-sensitive enteropathy):

It is an autoimmune disease due to exposure to gliadin (gliadin is a product of gluten


breakdown) associated with HLA-DQ2 (chromosome 6) found in 80-90%, HLA-DQ8
It usually starts at age 6 months when adding food other than milk (It never manifests at
birth)
e clinical features include bloating, abdominal pain, weight loss, chronic diarrhea, and
symptoms of nutritional deficiencies.
e most accurate test is intestinal biopsy showing atrophic villi
e treatment is achieved by a strict gluten-free diet and the treatment of complications.
e most dangerous complications are small bowel lymphoma and adenocarcinoma

Page - 740
Internal Medicine - Gastroenterology

Question 27/106

Question #27

A 19-year-old female with a known case of T1DM presents with weight loss, abdominal bloating, and
diarrhea for 6 months. She is also complaining of malodorous flatus and gurgling sounds in her
abdomen. Her lab investigations show microcytic hypochromic anemia but otherwise
unremarkable. If the patient has a similar family history, what is the most likely diagnosis?

a. Celiac disease
b. Food poisoning
c. Irritable bowel syndrome
d. Ulcerative colitis
e. Colorectal cancer

‫اﻹﺟﺎﺑﺔ ﻋﲆ اﻟﺼﻔﺤﺔ اﻟﺘﺎﻟﻴﺔ‬

Page - 741
Internal Medicine - Gastroenterology - Celiac Disease

Question 27/106

Question #27

A 19-year-old female with a known case of T1DM presents with weight loss, abdominal bloating, and
diarrhea for 6 months. She is also complaining of malodorous flatus and gurgling sounds in her
abdomen. Her lab investigations show microcytic hypochromic anemia but otherwise
unremarkable. If the patient has a similar family history, what is the most likely diagnosis?

a. Celiac disease √
b. Food poisoning
c. Irritable bowel syndrome
d. Ulcerative colitis
e. Colorectal cancer

Description

e presence of weight loss, bloating, diarrhea, Iron deficiency anemia, and the association with
T1DM and a positive family history point to a diagnosis of celiac disease.

Ulcerative colitis and IBS will never cause anemia or malabsorption

Colorectal cancer is unlikely in this age group

And food poisoning is acute and will not present for 6 months

Celiac disease (gluten-sensitive enteropathy):

It is an autoimmune disease due to exposure to gliadin (gliadin is a product of gluten


breakdown) associated with HLA-DQ2 (chromosome 6) found in 80-90%, HLA-DQ8
It usually starts at age 6 months when adding food other than milk (It never manifests at
birth)
e clinical features include bloating, abdominal pain, weight loss, chronic diarrhea, and
symptoms of nutritional deficiencies.
e most accurate test is intestinal biopsy showing atrophic villi
e treatment is achieved by a strict gluten-free diet and the treatment of complications.
e most dangerous complications are small bowel lymphoma and adenocarcinoma

Page - 742
Internal Medicine - Gastroenterology

Question 28/106

Question #28

A 32-year-old white female presents because of 6 years history of abdominal pain and smelly
diarrhea, not responding to the usual treatment. ere is no blood in the stool or weight loss. Her
physical examination, CBC, and electrolytes are normal. What is the most appropriate next step in
the management of this patient?

a. yroid function test


b. Kidney function test
c. Anti-tissue transglutaminase antibody
d. Stool analysis for ova or parasites
e. Measure bond density

‫اﻹﺟﺎﺑﺔ ﻋﲆ اﻟﺼﻔﺤﺔ اﻟﺘﺎﻟﻴﺔ‬

Page - 743
Internal Medicine - Gastroenterology - Celiac Disease

Question 28/106

Question #28

A 32-year-old white female presents because of 6 years history of abdominal pain and smelly
diarrhea, not responding to the usual treatment. ere is no blood in the stool or weight loss. Her
physical examination, CBC, and electrolytes are normal. What is the most appropriate next step in
the management of this patient?

a. yroid function test


b. Kidney function test
c. Anti-tissue transglutaminase antibody √
d. Stool analysis for ova or parasites
e. Measure bond density

Description

is is a suspected case of celiac sprue. e next step in the management is to have serology testing
for celiac disease antibodies

Note that all patients with celiac disease should be measured for bone density aer confirming the
diagnosis, but now the next step is to confirm the presence of celiac disease

Celiac disease (gluten-sensitive enteropathy):

It is an autoimmune disease due to exposure to gliadin (gliadin is a product of gluten


breakdown) associated with HLA-DQ2 (chromosome 6) found in 80-90%, HLA-DQ8
It usually starts at age 6 months when adding food other than milk (It never manifests at
birth)
e clinical features include bloating, abdominal pain, weight loss, chronic diarrhea, and
symptoms of nutritional deficiencies.
e most accurate test is intestinal biopsy showing atrophic villi
e treatment is achieved by a strict gluten-free diet and the treatment of complications.
e most dangerous complications are small bowel lymphoma and adenocarcinoma

Page - 744
Internal Medicine - Gastroenterology

Question 29/106

Question #29

A 22-year-old female patient has a 6 months history of diarrhea associated with weight loss and
easy bruising. Her examination is normal except for a chronic blistering skin consistent with
dermatitis herpetiformis. What is the most likely diagnosis?

a. Crohn’s disease
b. Ulcerative colitis
c. Celiac disease
d. H pylori infection
e. Von Willebrand disease

‫اﻹﺟﺎﺑﺔ ﻋﲆ اﻟﺼﻔﺤﺔ اﻟﺘﺎﻟﻴﺔ‬

Page - 745
Internal Medicine - Gastroenterology - Celiac Disease

Question 29/106

Question #29

A 22-year-old female patient has a 6 months history of diarrhea associated with weight loss and
easy bruising. Her examination is normal except for a chronic blistering skin consistent with
dermatitis herpetiformis. What is the most likely diagnosis?

a. Crohn’s disease
b. Ulcerative colitis
c. Celiac disease √
d. H pylori infection
e. Von Willebrand disease

Description

e presence of weight loss, fatty diarrhea, and features suggestive of vitamin K deficiency (easy
bruising) are all indicative of malabsorption

Dermatitis herpetiformis and features of malabsorption indicate Celiac disease as the most likely
diagnosis.

Celiac disease (gluten-sensitive enteropathy):

It is an autoimmune disease due to exposure to gliadin (gliadin is a product of gluten


breakdown) associated with HLA-DQ2 (chromosome 6) found in 80-90%, HLA-DQ8
It usually starts at age 6 months when adding food other than milk (It never manifests at
birth)
e clinical features include bloating, abdominal pain, weight loss, chronic diarrhea, and
symptoms of nutritional deficiencies.
e most accurate test is intestinal biopsy showing atrophic villi
e treatment is achieved by a strict gluten-free diet and the treatment of complications.
e most dangerous complications are small bowel lymphoma and adenocarcinoma

Page - 746
Internal Medicine - Gastroenterology

Question 30/106

Question #30

e most common organism associated with pseudomembranous colitis is:

a. Clostridium septicum
b. Clostridium perfringens
c. Clostridium dicile
d. Clostridium tetani
e. Clostridium sordellii

‫اﻹﺟﺎﺑﺔ ﻋﲆ اﻟﺼﻔﺤﺔ اﻟﺘﺎﻟﻴﺔ‬

Page - 747
Internal Medicine - Gastroenterology - Diarrhea

Question 30/106

Question #30

e most common organism associated with pseudomembranous colitis is:

a. Clostridium septicum
b. Clostridium perfringens
c. Clostridium dicile √
d. Clostridium tetani
e. Clostridium sordellii

Description

Pseudomembranous colitis is diarrhea that occurs as a result of antibiotic use due to the eradication
of the normal flora in the intestine leading to excessive growth of Clostridium dicile → release of a
toxin → bloody or watery diarrhea

Clostridia species and their diseases are as the following:

C. perfringens: food intoxication, diarrhea, gangrene, puerperal sepsis


C. dicile: pseudomembranous colitis
C. septicum: intestinal myonecrosis
C. botulinum: Botulism
C. sordellii: gangrene
C. tetani: Tetanus
C. novyi: gangrene

Page - 748
Internal Medicine - Gastroenterology

Question 31/106

Question #31

A 32-year-old male patient complains of loose, watery diarrhea and bloating since returning from his
trip to Yemen. Although he lost 3 kilograms since that time, his physical examination and lab
investigations are unremarkable. What is the most likely organism causing his symptoms?

a. Campylobacter jejuni
b. Giardia lamblia
c. Salmonella
d. Shigella
e. Staphylococcus aureus

‫اﻹﺟﺎﺑﺔ ﻋﲆ اﻟﺼﻔﺤﺔ اﻟﺘﺎﻟﻴﺔ‬

Page - 749
Internal Medicine - Gastroenterology - Diarrhea

Question 31/106

Question #31

A 32-year-old male patient complains of loose, watery diarrhea and bloating since returning from his
trip to Yemen. Although he lost 3 kilograms since that time, his physical examination and lab
investigations are unremarkable. What is the most likely organism causing his symptoms?

a. Campylobacter jejuni
b. Giardia lamblia √
c. Salmonella
d. Shigella
e. Staphylococcus aureus

Description

is case is typical for giardiasis

Giardiasis presents as traveler’s diarrhea with symptoms lasting more than 10 days
It can cause both acute or chronic diarrhea
is is chronic giardiasis (because it is associated with weight loss)
Campylobacter, Shigella, and Salmonella usually cause bloody diarrhea.

Giardiasis:

Giardiasis presents as traveler’s diarrhea with symptoms lasting more than 10 days
It can cause both acute or chronic diarrhea
Symptoms: bloating, flatulence, abdominal pain, loose stool, explosive watery diarrhea,
weight loss
Symptoms begin aer returning from a travel

Campylobacteriosis:

Campylobacter is the commonest bacterial cause of infectious intestinal disease


Clinical features:
Prodrome: headache, myalgia, and fevers (as high as 40°C)
Abdominal pain
Profuse diarrhea (oen bloody)

Page - 750
Internal Medicine - Gastroenterology

Question 32/106

Question #32

A 19-year-ola male patient presents with diarrhea. Which of the following is the least likely cause?

a. E. coli
b. Rotavirus
c. Norwalk virus
d. Giardiasis
e. Helicobacter pylori

‫اﻹﺟﺎﺑﺔ ﻋﲆ اﻟﺼﻔﺤﺔ اﻟﺘﺎﻟﻴﺔ‬

Page - 751
Internal Medicine - Gastroenterology - Diarrhea

Question 32/106

Question #32

A 19-year-ola male patient presents with diarrhea. Which of the following is the least likely cause?

a. E. coli
b. Rotavirus
c. Norwalk virus
d. Giardiasis
e. Helicobacter pylori √

Description

H pylori is a gastric pathogen that causes gastritis and peptic ulcer disease. It is unlikely to cause
diarrhea

Peptic ulcer disease (PUD):

It is an ulceration of the lower esophagus, stomach, or duodenum and can occur in the ileum
adjacent to Meckel’s diverticulum.
e most common site of gastric ulcer (GU) is at the lesser curvature (exactly at the Incisura
angularis)
e most common cause in developing countries is H. Pylori infection, while NSAIDs are
responsible for most cases in developed countries
All patients with peptic ulcer disease should be tested for H. pylori regardless of the use of
NSAIDs
Dyspepsia is the most common symptom, but epigastric pain, GI bleeding, and vomiting may
present.
Upper GI endoscopy is considered the most accurate test
e treatment is achieved by elimination of the cause and the use of PPIs

Page - 752
Internal Medicine - Gastroenterology

Question 33/106

Question #33

During a sigmoidoscopy, you found the morphology shown in the picture below. What is the most
likely diagnosis?

a. Melanosis coli
b. Colorectal cancer
c. Malignant lymphoma
d. Arteriovenous malformations
e. Colitis

‫اﻹﺟﺎﺑﺔ ﻋﲆ اﻟﺼﻔﺤﺔ اﻟﺘﺎﻟﻴﺔ‬

Page - 753
Internal Medicine - Gastroenterology - Diarrhea

Question 33/106

Question #33

During a sigmoidoscopy, you found the morphology shown in the picture below. What is the most
likely diagnosis?

a. Melanosis coli √
b. Colorectal cancer
c. Malignant lymphoma
d. Arteriovenous malformations
e. Colitis

Description

Black or brown discoloration of the mucosa of the colon is known as melanosis coli. It is caused by
laxative use.

It results from the presence of dark pigment in large mononuclear cells or macrophages in the
lamina propria of the mucosa

Page - 754
Page - 755
Internal Medicine - Gastroenterology

Question 34/106

Question #34

At a hotel, 5 guests developed nausea, vomiting, and watery diarrhea 2 hours aer eating, and the
symptoms resolved spontaneously within 48 hours. What is the most likely etiology of their
condition?

a. Rotavirus
b. Staph aureus toxin
c. C. botulinum
d. C. jejuni
e. C. Dicile

‫اﻹﺟﺎﺑﺔ ﻋﲆ اﻟﺼﻔﺤﺔ اﻟﺘﺎﻟﻴﺔ‬

Page - 756
Internal Medicine - Gastroenterology - Diarrhea

Question 34/106

Question #34

At a hotel, 5 guests developed nausea, vomiting, and watery diarrhea 2 hours aer eating, and the
symptoms resolved spontaneously within 48 hours. What is the most likely etiology of their
condition?

a. Rotavirus
b. Staph aureus toxin √
c. C. botulinum
d. C. jejuni
e. C. Dicile

Description

All of them have gastroenteritis secondary to food poisoning.

e dierential diagnosis includes (E. coli, Norwalk virus, Salmonella typhimurium, Salmonella
enteritidis, Campylobacter jejuni, and Yersinia enterocolitica)

Only Staph. aureus toxin would cause these symptoms in such a short time.

Staphylococcal toxins are fast-acting, sometimes causing illness in as little as 30 minutes aer
eating contaminated foods, but usually, symptoms present 1 to 6 hours aer the patient has
consumed the contaminated food

Page - 757
Internal Medicine - Gastroenterology

Question 35/106

Question #35

A 33-year-old female just returned from her trip to Europe. She develops symptoms of watery
diarrhea and cramping abdominal pain. What is the most likely organism causing her symptoms?

a. Campylobacteriosis
b. Escherichia coli
c. Giardiasis
d. Salmonella
e. Shigella

‫اﻹﺟﺎﺑﺔ ﻋﲆ اﻟﺼﻔﺤﺔ اﻟﺘﺎﻟﻴﺔ‬

Page - 758
Internal Medicine - Gastroenterology - Diarrhea

Question 35/106

Question #35

A 33-year-old female just returned from her trip to Europe. She develops symptoms of watery
diarrhea and cramping abdominal pain. What is the most likely organism causing her symptoms?

a. Campylobacteriosis
b. Escherichia coli √
c. Giardiasis
d. Salmonella
e. Shigella

Description

is is a case of traveler’s diarrhea.

E. coli is the most common cause


Other causes: Shigella, Salmonella, Campylobacter, Giardia lamblia

Campylobacteriosis presents with a prodrome followed by diarrhea

Giardiasis presents with watery diarrhea, flatulence, and weight loss

Salmonella and shigella present with bloody diarrhea.

Page - 759
Internal Medicine - Gastroenterology

Question 36/106

Question #36

A sigmoidoscopy in a 32-year-old female patient shows scattered areas of black discolorations. What
is the most likely description?

a. Endometriosis
b. Laxative use
c. Colitis
d. Malignant lymphoma
e. Arteriovenous malformations

‫اﻹﺟﺎﺑﺔ ﻋﲆ اﻟﺼﻔﺤﺔ اﻟﺘﺎﻟﻴﺔ‬

Page - 760
Internal Medicine - Gastroenterology - Diarrhea

Question 36/106

Question #36

A sigmoidoscopy in a 32-year-old female patient shows scattered areas of black discolorations. What
is the most likely description?

a. Endometriosis
b. Laxative use √
c. Colitis
d. Malignant lymphoma
e. Arteriovenous malformations

Description

Black or brown discoloration of the mucosa of the colon is known as melanosis coli. It is caused by
laxative use.

It results from the presence of dark pigment in large mononuclear cells or macrophages in the
lamina propria of the mucosa

Page - 761
Internal Medicine - Gastroenterology

Question 37/106

Question #37

A 34-year-old female complains of diarrhea for 2 days. She has been taking amoxicillin for 1 week for
pneumonia. On examination, she is well hydrated and has normal vital signs. However, her stool is
positive for occult blood, and her stool screening is positive for C. dicile toxin. What is the most
appropriate treatment at this time?

a. Oral vancomycin
b. Oral metronidazole
c. Oral TMP/SMX
d. Oral ciprofloxacin
e. Oral clindamycin

‫اﻹﺟﺎﺑﺔ ﻋﲆ اﻟﺼﻔﺤﺔ اﻟﺘﺎﻟﻴﺔ‬

Page - 762
Internal Medicine - Gastroenterology - Diarrhea

Question 37/106

Question #37

A 34-year-old female complains of diarrhea for 2 days. She has been taking amoxicillin for 1 week for
pneumonia. On examination, she is well hydrated and has normal vital signs. However, her stool is
positive for occult blood, and her stool screening is positive for C. dicile toxin. What is the most
appropriate treatment at this time?

a. Oral vancomycin √
b. Oral metronidazole
c. Oral TMP/SMX
d. Oral ciprofloxacin
e. Oral clindamycin

Description

Oral metronidazole used to be the first-line treatment for C. dicile antibiotic-associated colitis
(pseudomembranous colitis)

Recently, the guidelines have been changed, and the treatment of this condition is as the following:

Always stop the oending antibiotic


Oral vancomycin (the first-line treatment)
IV metronidazole and Vancomycin enema can be added if the severe disease is associated with
ileus.
If a fulminant or complicated disease is not responsive to the previous treatment, perform a
colectomy.
Fecal microbiota transplant is used for patients with multiple relapses.

Page - 763
Internal Medicine - Gastroenterology

Question 38/106

Question #38

A 23-year-old male patient presents with watery diarrhea, abdominal pain, and vomiting. A similar
history is found in his younger sister but with milder symptoms. Seven hours ago, at dinner, they ate
a cheeseburger. Which of the following is the most likely cause of his condition?

a. Rotavirus
b. Staph aureus toxin
c. C. botulinum
d. C. jejuni
e. C. Dicile

‫اﻹﺟﺎﺑﺔ ﻋﲆ اﻟﺼﻔﺤﺔ اﻟﺘﺎﻟﻴﺔ‬

Page - 764
Internal Medicine - Gastroenterology - Diarrhea

Question 38/106

Question #38

A 23-year-old male patient presents with watery diarrhea, abdominal pain, and vomiting. A similar
history is found in his younger sister but with milder symptoms. Seven hours ago, at dinner, they ate
a cheeseburger. Which of the following is the most likely cause of his condition?

a. Rotavirus
b. Staph aureus toxin √
c. C. botulinum
d. C. jejuni
e. C. Dicile

Description

Both of them have gastroenteritis secondary to food poisoning.

e dierential diagnosis includes (E. coli, Norwalk virus, Salmonella typhimurium, Salmonella
enteritidis, Campylobacter jejuni, and Yersinia enterocolitica)

Only Staph. aureus toxin would cause these symptoms in such a short time.

Staphylococcal toxins are fast-acting, sometimes causing illness in as little as 30 minutes aer
eating contaminated foods, but usually, symptoms present 1 to 6 hours aer the patient has
consumed the contaminated food

Page - 765
Internal Medicine - Gastroenterology

Question 39/106

Question #39

You suspect secretory diarrhea in a 22-year-old female patient. Which of the following is a feature of
this condition?

a. Small volume (<1L/day)


b. High stool osmotic gap
c. Blood or pus in stool analysis
d. It doesn’t stop with fasting
e. Foul-smelling stools

‫اﻹﺟﺎﺑﺔ ﻋﲆ اﻟﺼﻔﺤﺔ اﻟﺘﺎﻟﻴﺔ‬

Page - 766
Internal Medicine - Gastroenterology - Diarrhea

Question 39/106

Question #39

You suspect secretory diarrhea in a 22-year-old female patient. Which of the following is a feature of
this condition?

a. Small volume (<1L/day)


b. High stool osmotic gap
c. Blood or pus in stool analysis
d. It doesn’t stop with fasting √
e. Foul-smelling stools

Description

Osmotic diarrhea:

Stool osmotic gap > 100 mOsm


It stops on fasting and never awakes the patient from sleep
Usually not lead to dehydration

Secretory diarrhea:

Stool osmotic gap < 50 mOsm


Occurs at fasting or during sleep
Usually high volume > 1 L/day

Page - 767
Internal Medicine - Gastroenterology

Question 40/106

Question #40

A 62-year-old male patient has been on clindamycin for recent cellulitis for the last 3 days. He
presents with a 1-day history of watery diarrhea. He is afebrile and has no blood in the stool. What is
the best initial management for this patient?

a. Stop clindamycin
b. Start ciprofloxacin
c. Start vancomycin
d. Start metronidazole
e. Start loperamide

‫اﻹﺟﺎﺑﺔ ﻋﲆ اﻟﺼﻔﺤﺔ اﻟﺘﺎﻟﻴﺔ‬

Page - 768
Internal Medicine - Gastroenterology - Diarrhea

Question 40/106

Question #40

A 62-year-old male patient has been on clindamycin for recent cellulitis for the last 3 days. He
presents with a 1-day history of watery diarrhea. He is afebrile and has no blood in the stool. What is
the best initial management for this patient?

a. Stop clindamycin √
b. Start ciprofloxacin
c. Start vancomycin
d. Start metronidazole
e. Start loperamide

Description

Stopping the oending agent is the best initial step in this case, even before starting vancomycin or
metronidazole.

Oral metronidazole was the first-line treatment for C. dicile antibiotic-associated colitis
(pseudomembranous colitis).

Recently, the guidelines have been changed, and the treatment of this condition is as the following:

Always stop the oending antibiotic


Oral vancomycin (the first-line treatment)
IV metronidazole and Vancomycin enema can be added if the severe disease is associated with
ileus.
If a fulminant or complicated disease is not responsive to the previous treatment, perform a
colectomy.
Fecal microbiota transplant is used for patients with multiple relapses.

Page - 769
Internal Medicine - Gastroenterology

Question 41/106

Question #41

A 22-year-old male patient presents to your oce for evaluation regarding his 3 days of diarrhea.
You suspect a bacterial cause. Which of the following is the most common cause of acute bacterial
diarrhea?

a. E. coli O157:H7
b. Shigella dysentariae
c. Salmonella enterica
d. Campylobacter jejuni
e. C. dicile

‫اﻹﺟﺎﺑﺔ ﻋﲆ اﻟﺼﻔﺤﺔ اﻟﺘﺎﻟﻴﺔ‬

Page - 770
Internal Medicine - Gastroenterology - Diarrhea

Question 41/106

Question #41

A 22-year-old male patient presents to your oce for evaluation regarding his 3 days of diarrhea.
You suspect a bacterial cause. Which of the following is the most common cause of acute bacterial
diarrhea?

a. E. coli O157:H7
b. Shigella dysentariae
c. Salmonella enterica
d. Campylobacter jejuni √
e. C. dicile

Description

Campylobacter infection:

Campylobacter is the most common bacterial cause of infectious intestinal disease


Diagnosis is made by a positive stool culture but requires special media and handling

Clinical features:

Prodrome: headache, myalgia, and fevers (as high as 40°C)


Abdominal pain
Watery oensive diarrhea for 2 – 4 days, then become bloody

Treatment:

Usually, supportive treatment is adequate


Erythromycin or azithromycin can be used when there is a risk of complications

Page - 771
Internal Medicine - Gastroenterology

Question 42/106

Question #42

A 22-year-old male patient developed bloody diarrhea, abdominal cramping, and nausea 48 hours
aer returning from a camping trip. His stool study shows shigellosis. Which of the following is the
most important initial treatment for his condition?

a. Metronidazole
b. Fluid and electrolytes replacement
c. TMP/SMX
d. Ampicillin
e. Gastric lavage

‫اﻹﺟﺎﺑﺔ ﻋﲆ اﻟﺼﻔﺤﺔ اﻟﺘﺎﻟﻴﺔ‬

Page - 772
Internal Medicine - Gastroenterology - Diarrhea

Question 42/106

Question #42

A 22-year-old male patient developed bloody diarrhea, abdominal cramping, and nausea 48 hours
aer returning from a camping trip. His stool study shows shigellosis. Which of the following is the
most important initial treatment for his condition?

a. Metronidazole
b. Fluid and electrolytes replacement √
c. TMP/SMX
d. Ampicillin
e. Gastric lavage

Description

Shigellosis is an acute GI infection causing diarrhea. Symptoms include diarrhea, abdominal pain,
fever, nausea, and vomiting

In any kind of diarrhea, the priority is to maintain fluid status and electrolyte balance to avoid
dehydration and unwanted eects of fluid and electrolytes imbalance

Page - 773
Internal Medicine - Gastroenterology

Question 43/106

Question #43

A 62-year-old male patient has been on clindamycin for recent cellulitis for the last 3 days. He
presents with a 1-day history of watery diarrhea. He is afebrile and has no blood in the stool. What is
the best diagnostic test to be ordered for this patient?

a. Blood culture
b. C dicile cytotoxin assay in stool
c. Colonoscopy
d. Gram stain and culture of the stool
e. Clindamycin serum level

‫اﻹﺟﺎﺑﺔ ﻋﲆ اﻟﺼﻔﺤﺔ اﻟﺘﺎﻟﻴﺔ‬

Page - 774
Internal Medicine - Gastroenterology - Diarrhea

Question 43/106

Question #43

A 62-year-old male patient has been on clindamycin for recent cellulitis for the last 3 days. He
presents with a 1-day history of watery diarrhea. He is afebrile and has no blood in the stool. What is
the best diagnostic test to be ordered for this patient?

a. Blood culture
b. C dicile cytotoxin assay in stool √
c. Colonoscopy
d. Gram stain and culture of the stool
e. Clindamycin serum level

Description

C. dicile antibiotic-associated colitis (pseudomembranous colitis) should be suspected in any


patient who develops abdominal pain, diarrhea, and elevated white count aer the administration
of antibiotics.

Cytotoxin assay of the stool is the best test to order; it is sensitive and highly specific

A colonoscopy would show the classic Pseudomembranes, but it is invasive in this patient’s case.

Page - 775
Internal Medicine - Gastroenterology

Question 44/106

Question #44

A 29-year-old female with cellulitis started a course of clindamycin in the past 3 days. Unfortunately,
she came to you with bloody diarrhea. What is the first-line treatment for her condition?

a. Stop clindamycin and start oral metronidazole


b. Stop clindamycin and start oral vancomycin
c. Stop clindamycin and start intravenous vancomycin
d. Stop clindamycin and start intravenous metronidazole
e. Perform fecal microbiota transplant

‫اﻹﺟﺎﺑﺔ ﻋﲆ اﻟﺼﻔﺤﺔ اﻟﺘﺎﻟﻴﺔ‬

Page - 776
Internal Medicine - Gastroenterology - Diarrhea

Question 44/106

Question #44

A 29-year-old female with cellulitis started a course of clindamycin in the past 3 days. Unfortunately,
she came to you with bloody diarrhea. What is the first-line treatment for her condition?

a. Stop clindamycin and start oral metronidazole


b. Stop clindamycin and start oral vancomycin √
c. Stop clindamycin and start intravenous vancomycin
d. Stop clindamycin and start intravenous metronidazole
e. Perform fecal microbiota transplant

Description

Oral vancomycin has become the first-line treatment of pseudomembranous colitis.

In the past, oral metronidazole was the first line.

Treatment of pseudomembranous colitis:

If this is the first episode of Clostridium dicile infection


Oral vancomycin for 10 days (the first-line treatment)
Oral fidaxomicin (the second line)
Oral vancomycin and IV metronidazole (the third line)
If the patient has recurrent episodes:
Oral vancomycin and IV metronidazole
Consider surgery
Consider fecal microbiota transplant (if ≥ 2 episodes)

If fulminant or complicated disease is not responsive to the previous treatment, perform a


colectomy.

Page - 777
Internal Medicine - Gastroenterology

Question 45/106

Question #45

A 62-year-old male patient has been on clindamycin for recent cellulitis for the last 3 days. He
presents with a 1-day history of watery diarrhea. He is afebrile and has no blood in the stool. What is
the most likely causative organism of this condition?

a. Campylobacter jejuni
b. Giardia lamblia
c. Clostridium dicile
d. Salmonella
e. Rotavirus

‫اﻹﺟﺎﺑﺔ ﻋﲆ اﻟﺼﻔﺤﺔ اﻟﺘﺎﻟﻴﺔ‬

Page - 778
Internal Medicine - Gastroenterology - Diarrhea

Question 45/106

Question #45

A 62-year-old male patient has been on clindamycin for recent cellulitis for the last 3 days. He
presents with a 1-day history of watery diarrhea. He is afebrile and has no blood in the stool. What is
the most likely causative organism of this condition?

a. Campylobacter jejuni
b. Giardia lamblia
c. Clostridium dicile √
d. Salmonella
e. Rotavirus

Description

Pseudomembranous colitis is diarrhea that occurs as a result of antibiotic use due to the eradication
of the normal flora in the intestine leading to excessive growth of Clostridium dicile → release of a
toxin → bloody or watery diarrhea

Campylobacteriosis presents with a prodrome followed by diarrhea

Giardiasis presents with watery diarrhea, flatulence, and weight loss

Salmonella presents with bloody diarrhea.

Page - 779
Internal Medicine - Gastroenterology

Question 46/106

Question #46

A patient is diagnosed with traveler’s diarrhea, which of the following would be most eective in
treating this patient?

a. Amoxicillin
b. Ciprofloxacin
c. Erythromycin
d. Azithromycin
e. Ceriaxone

‫اﻹﺟﺎﺑﺔ ﻋﲆ اﻟﺼﻔﺤﺔ اﻟﺘﺎﻟﻴﺔ‬

Page - 780
Internal Medicine - Gastroenterology - Diarrhea

Question 46/106

Question #46

A patient is diagnosed with traveler’s diarrhea, which of the following would be most eective in
treating this patient?

a. Amoxicillin
b. Ciprofloxacin √
c. Erythromycin
d. Azithromycin
e. Ceriaxone

Description

Traveler’s diarrhea is most commonly but not exclusively caused by infection with E. coli in a traveler
person. Fluoroquinolones significantly reduce the duration and severity of traveler’s diarrhea when
given for 1 - 3 days.

Traveler’s diarrhea:

Traveler’s diarrhea occurs within 2 weeks of a visit to a tropical area


Usually, mild self-limiting diarrhea for less than 72 hours

Causes:

E. coli is the most common cause


Other causes: Shigella, Salmonella, Campylobacter, Giardia lamblia

Treatment:

Fluoroquinolones significantly reduce the duration and severity of traveler’s diarrhea when
given for 1 – 3 days.

Page - 781
Internal Medicine - Gastroenterology

Question 47/106

Question #47

A patient developed chronic watery diarrhea, weight loss, nausea, anorexia, and no fecal leukocytes.
Which of the following is the most likely diagnosis?

a. Amebiasis
b. Giardiasis
c. Salmonella
d. Shigellosis
e. Campylobacter jejuni

‫اﻹﺟﺎﺑﺔ ﻋﲆ اﻟﺼﻔﺤﺔ اﻟﺘﺎﻟﻴﺔ‬

Page - 782
Internal Medicine - Gastroenterology - Diarrhea

Question 47/106

Question #47

A patient developed chronic watery diarrhea, weight loss, nausea, anorexia, and no fecal leukocytes.
Which of the following is the most likely diagnosis?

a. Amebiasis
b. Giardiasis √
c. Salmonella
d. Shigellosis
e. Campylobacter jejuni

Description

Giardiasis is an infection with the protozoa Giardia lamblia. e infection could be asymptomatic
and may present with chronic malabsorption and watery diarrhea

e stool will show no inflammatory process, and the diagnosis is achieved by identifying the
flagellated organism in the fresh stool or duodenal contents.

Giardia antigen in stool is diagnostic as well

e treatment is with metronidazole, tinidazole or nitazoxanide

Page - 783
Internal Medicine - Gastroenterology

Question 48/106

Question #48

A 44-year-old male patient presents with epigastric pain aer eating his dinner. What is the best
next step to take for this patient?

a. Start PPI
b. Standing Chest x-ray
c. Abdominal Ultrasound
d. Electrocardiography
e. H pylori test

‫اﻹﺟﺎﺑﺔ ﻋﲆ اﻟﺼﻔﺤﺔ اﻟﺘﺎﻟﻴﺔ‬

Page - 784
Internal Medicine - Gastroenterology - Epigastric pain and dyspepsia

Question 48/106

Question #48

A 44-year-old male patient presents with epigastric pain aer eating his dinner. What is the best
next step to take for this patient?

a. Start PPI
b. Standing Chest x-ray
c. Abdominal Ultrasound
d. Electrocardiography √
e. H pylori test

Description

Myocardial infarction is the main concern in chest pain and epigastric pain. erefore, ECG should be
done on all patients with chest or epigastric pain to rule out myocardial infarction because it is the
most serious diagnosis.

Aer ruling out MI, you can check for GI causes of epigastric pain

Page - 785
Internal Medicine - Gastroenterology

Question 49/106

Question #49

A 33-year-old male patient has had epigastric pain and dyspepsia for 3 months. His H pylori test is
negative, and he doesn’t use NSAIDs. e patient was on esomeprazole 40mg daily for the last
month, but no improvement. Abdominal examination reveals mild epigastric tenderness. What is
the best next step in the management of this patient?

a. Repeat H pylori test again


b. Increase esomeprazole to twice daily
c. Change to another PPI
d. Perform an upper endoscopy
e. Start triple therapy

‫اﻹﺟﺎﺑﺔ ﻋﲆ اﻟﺼﻔﺤﺔ اﻟﺘﺎﻟﻴﺔ‬

Page - 786
Internal Medicine - Gastroenterology - Epigastric pain and dyspepsia

Question 49/106

Question #49

A 33-year-old male patient has had epigastric pain and dyspepsia for 3 months. His H pylori test is
negative, and he doesn’t use NSAIDs. e patient was on esomeprazole 40mg daily for the last
month, but no improvement. Abdominal examination reveals mild epigastric tenderness. What is
the best next step in the management of this patient?

a. Repeat H pylori test again


b. Increase esomeprazole to twice daily √
c. Change to another PPI
d. Perform an upper endoscopy
e. Start triple therapy

Description

is is a patient who is suering from dyspepsia. e patient is less than 55-year-old and has no red
flags

In a patient with dyspepsia, less than 55-year-old and no red flags:

Try lifestyle changes and antacids


If there is no improvement and a negative H pylori test, start PPI for 4 weeks
If there is no improvement and a positive H pylori test, start PPI and H pylori eradication
If there is no improvement aer 4 weeks of PPI, start twice-daily PPI for additional 4 weeks
If there is no improvement, do an upper endoscopy

Page - 787
Internal Medicine - Gastroenterology

Question 50/106

Question #50

A 30-year-old male patient presents with epigastric pain aggravated by food for the past 2 weeks. He
denies vomiting blood or blood in the stool. Considering the most likely diagnosis, what is the most
common upper endoscopy finding in this case?

a. Gastritis
b. Gastric Ulceration
c. Duodenal ulceration
d. Gastric cancer
e. Normal endoscopy

‫اﻹﺟﺎﺑﺔ ﻋﲆ اﻟﺼﻔﺤﺔ اﻟﺘﺎﻟﻴﺔ‬

Page - 788
Internal Medicine - Gastroenterology - Epigastric pain and dyspepsia

Question 50/106

Question #50

A 30-year-old male patient presents with epigastric pain aggravated by food for the past 2 weeks. He
denies vomiting blood or blood in the stool. Considering the most likely diagnosis, what is the most
common upper endoscopy finding in this case?

a. Gastritis
b. Gastric Ulceration
c. Duodenal ulceration
d. Gastric cancer
e. Normal endoscopy √

Description

Functional dyspepsia is the most likely diagnosis in this patient. It presents with epigastric pain and
dyspepsia along with normal mucosa on upper endoscopy

Page - 789
Internal Medicine - Gastroenterology

Question 51/106

Question #51

A 66-year-old male patient has had increasing dyspepsia and dysphagia for the past 3 months. ese
symptoms are associated with 10 kg weight loss. What is the most important investigation that
leads to the diagnosis?

a. Barium swallow
b. Chest x-ray
c. Upper endoscopy
d. Esophageal manometry
e. 24-hours PH monitoring

‫اﻹﺟﺎﺑﺔ ﻋﲆ اﻟﺼﻔﺤﺔ اﻟﺘﺎﻟﻴﺔ‬

Page - 790
Internal Medicine - Gastroenterology - Epigastric pain and dyspepsia

Question 51/106

Question #51

A 66-year-old male patient has had increasing dyspepsia and dysphagia for the past 3 months. ese
symptoms are associated with 10 kg weight loss. What is the most important investigation that
leads to the diagnosis?

a. Barium swallow
b. Chest x-ray
c. Upper endoscopy √
d. Esophageal manometry
e. 24-hours PH monitoring

Description

is patient is older than 55 years, has weight loss (a red flag), and has unexplained dyspepsia.

Any patient older than 55 years or those with alarming symptoms should have upper endoscopy
regardless of the results of H pylori testing.

Endoscopy here is indicated to rule out malignant conditions.

Page - 791
Internal Medicine - Gastroenterology

Question 52/106

Question #52

A 33-year-old male patient has had epigastric pain and dyspepsia for 3 months. His H pylori test is
negative, and he doesn’t use NSAIDs. e patient was on esomeprazole 40mg twice daily for the last
two months, but no improvement. Abdominal examination reveals mild epigastric tenderness.
What is the best next step in the management of this patient?

a. Repeat H pylori test again


b. Increase esomeprazole to twice daily
c. Change to another PPI
d. Perform an upper endoscopy
e. Start triple therapy

‫اﻹﺟﺎﺑﺔ ﻋﲆ اﻟﺼﻔﺤﺔ اﻟﺘﺎﻟﻴﺔ‬

Page - 792
Internal Medicine - Gastroenterology - Epigastric pain and dyspepsia

Question 52/106

Question #52

A 33-year-old male patient has had epigastric pain and dyspepsia for 3 months. His H pylori test is
negative, and he doesn’t use NSAIDs. e patient was on esomeprazole 40mg twice daily for the last
two months, but no improvement. Abdominal examination reveals mild epigastric tenderness.
What is the best next step in the management of this patient?

a. Repeat H pylori test again


b. Increase esomeprazole to twice daily
c. Change to another PPI
d. Perform an upper endoscopy √
e. Start triple therapy

Description

is is a patient who is suering from dyspepsia. e patient is less than 55-year-old and has no red
flags

In a patient with dyspepsia, less than 55-year-old and no red flags:

Try lifestyle changes and antacids


If there is no improvement and a negative H pylori test, start PPI for 4 weeks
If there is no improvement and a positive H pylori test, start PPI and H pylori eradication
If there is no improvement aer 4 weeks of PPI, start twice-daily PPI for additional 4 weeks
If there is no improvement, do an upper endoscopy

Page - 793
Internal Medicine - Gastroenterology

Question 53/106

Question #53

A 62-year-old male has had recurrent pain in the epigastrium for 2 months. e pain increases aer
food and is better on an empty stomach. What is the most appropriate to perform at this point?

a. Barium swallow
b. Stool antigen test for H pylori
c. Serology test for H pylori
d. Upper endoscopy
e. Start PPI

‫اﻹﺟﺎﺑﺔ ﻋﲆ اﻟﺼﻔﺤﺔ اﻟﺘﺎﻟﻴﺔ‬

Page - 794
Internal Medicine - Gastroenterology - Epigastric pain and dyspepsia

Question 53/106

Question #53

A 62-year-old male has had recurrent pain in the epigastrium for 2 months. e pain increases aer
food and is better on an empty stomach. What is the most appropriate to perform at this point?

a. Barium swallow
b. Stool antigen test for H pylori
c. Serology test for H pylori
d. Upper endoscopy √
e. Start PPI

Description

is patient is older than 55 years, and he has unexplained dyspepsia.

Any patient older than 55 years or those with alarming symptoms should have upper endoscopy
regardless of the results of H pylori testing.

Endoscopy here is indicated to rule out malignant conditions.

Page - 795
Internal Medicine - Gastroenterology

Question 54/106

Question #54

An alcoholic 33-year-old male patient presents with three episodes of bloody vomiting associated
with epigastric pain. He denies weight loss, anorexia, or night sweating. His physical examination is
normal except for epigastric tenderness on deep palpation. His ECG and cardiac enzymes are
normal. What is the next step to do with this patient?

a. Start lansoprazole
b. Test for H pylori
c. Upper endoscopy
d. Repeat ECG and cardiac enzymes
e. Provide liquid antacids

‫اﻹﺟﺎﺑﺔ ﻋﲆ اﻟﺼﻔﺤﺔ اﻟﺘﺎﻟﻴﺔ‬

Page - 796
Internal Medicine - Gastroenterology - Epigastric pain and dyspepsia

Question 54/106

Question #54

An alcoholic 33-year-old male patient presents with three episodes of bloody vomiting associated
with epigastric pain. He denies weight loss, anorexia, or night sweating. His physical examination is
normal except for epigastric tenderness on deep palpation. His ECG and cardiac enzymes are
normal. What is the next step to do with this patient?

a. Start lansoprazole
b. Test for H pylori √
c. Upper endoscopy
d. Repeat ECG and cardiac enzymes
e. Provide liquid antacids

Description

e most likely diagnosis in this patient is gastritis vs. peptic ulcer disease.

Alcoholism is the leading risk factor in this case

Test and treatment strategy for H pylori is the best next step for this patient

Page - 797
Internal Medicine - Gastroenterology

Question 55/106

Question #55

A 50-year-old lady has retrosternal chest pain and dysphagia, which is unpredictable and
intermittent. She feels the food stuck in her chest and has to clear it by drinking water. A barium
swallow shows a corkscrew pattern. What is the most likely diagnosis?

a. Achalasia
b. Esophageal spasm
c. Reflux esophagitis
d. Oropharyngeal dysphagia
e. Esophageal stricture

‫اﻹﺟﺎﺑﺔ ﻋﲆ اﻟﺼﻔﺤﺔ اﻟﺘﺎﻟﻴﺔ‬

Page - 798
Internal Medicine - Gastroenterology - Esophageal spasm

Question 55/106

Question #55

A 50-year-old lady has retrosternal chest pain and dysphagia, which is unpredictable and
intermittent. She feels the food stuck in her chest and has to clear it by drinking water. A barium
swallow shows a corkscrew pattern. What is the most likely diagnosis?

a. Achalasia
b. Esophageal spasm √
c. Reflux esophagitis
d. Oropharyngeal dysphagia
e. Esophageal stricture

Description

Esophageal spasm is a motility disorder of the esophagus resulting in high-pressure peristalsis.

It is associated with dysphagia that is more prominent on drinking cold or hot liquids

Clinical features include chest pain (which may be typical) and dysphagia to solids and liquids

A barium swallow will demonstrate a Corkscrew appearance in the esophagus

Nitrates and calcium channel blockers are the most appropriate medical treatment for this
condition

Page - 799
Internal Medicine - Gastroenterology

Question 56/106

Question #56

A 33-year-old male has had central chest pain for 1 hour related to drinking a cold drink. e patient
is previously healthy, his ECG shows no acute changes, his cardiac enzymes are flat, and his vital
signs are within normal limits. What is the most likely diagnosis?

a. Acute myocardial infarction


b. Acute pericarditis
c. Pneumothorax
d. Esophageal spasm
e. Pulmonary embolism

‫اﻹﺟﺎﺑﺔ ﻋﲆ اﻟﺼﻔﺤﺔ اﻟﺘﺎﻟﻴﺔ‬

Page - 800
Internal Medicine - Gastroenterology - Esophageal spasm

Question 56/106

Question #56

A 33-year-old male has had central chest pain for 1 hour related to drinking a cold drink. e patient
is previously healthy, his ECG shows no acute changes, his cardiac enzymes are flat, and his vital
signs are within normal limits. What is the most likely diagnosis?

a. Acute myocardial infarction


b. Acute pericarditis
c. Pneumothorax
d. Esophageal spasm √
e. Pulmonary embolism

Description

Esophageal spasm is a motility disorder of the esophagus resulting in high-pressure peristalsis.

It is associated with dysphagia that is more prominent on drinking cold or hot liquids

Clinical features include chest pain (which may be typical) and dysphagia to solids and liquids

A barium swallow will demonstrate a Corkscrew appearance in the esophagus

Nitrates and calcium channel blockers are the most appropriate medical treatment for this
condition

Page - 801
Internal Medicine - Gastroenterology

Question 57/106

Question #57

A 50-year-old lady has retrosternal chest pain and dysphagia, which is unpredictable and
intermittent. She feels the food stuck in her chest and has to clear it by drinking water. A barium
meal shows a corkscrew pattern. What is the most appropriate medical treatment to start at this
point?

a. Aspirin
b. Lansoprazole
c. Nifedipine
d. Enoxaparin
e. Amoxicillin

‫اﻹﺟﺎﺑﺔ ﻋﲆ اﻟﺼﻔﺤﺔ اﻟﺘﺎﻟﻴﺔ‬

Page - 802
Internal Medicine - Gastroenterology - Esophageal spasm

Question 57/106

Question #57

A 50-year-old lady has retrosternal chest pain and dysphagia, which is unpredictable and
intermittent. She feels the food stuck in her chest and has to clear it by drinking water. A barium
meal shows a corkscrew pattern. What is the most appropriate medical treatment to start at this
point?

a. Aspirin
b. Lansoprazole
c. Nifedipine √
d. Enoxaparin
e. Amoxicillin

Description

Esophageal spasm is a motility disorder of the esophagus resulting in high-pressure peristalsis.

It is associated with dysphagia that is more prominent on drinking cold or hot liquids

Clinical features include chest pain (which may be typical) and dysphagia to solids and liquids

A barium swallow will demonstrate a Corkscrew appearance in the esophagus

Nitrates and calcium channel blockers are the most appropriate medical treatment for this
condition

Page - 803
Internal Medicine - Gastroenterology

Question 58/106

Question #58

A 33-year-old male patient complains of dysphagia and oral thrush. His medical history is significant
for HIV infection. What is the most appropriate next step?

a. Perform upper endoscopy and esophageal biopsy


b. Start treatment with Fluconazole
c. Start treatment with PPIs
d. Start treatment with Ganciclovir
e. Start treatment with acyclovir

‫اﻹﺟﺎﺑﺔ ﻋﲆ اﻟﺼﻔﺤﺔ اﻟﺘﺎﻟﻴﺔ‬

Page - 804
Internal Medicine - Gastroenterology - Esophagitis

Question 58/106

Question #58

A 33-year-old male patient complains of dysphagia and oral thrush. His medical history is significant
for HIV infection. What is the most appropriate next step?

a. Perform upper endoscopy and esophageal biopsy


b. Start treatment with Fluconazole √
c. Start treatment with PPIs
d. Start treatment with Ganciclovir
e. Start treatment with acyclovir

Description

Candida albicans: is a normal commensal of mouth


It can cause oral thrush and esophagitis in immune-suppressed patients (e.g., DM, Steroid
users, HIV)
In an immunosuppressed patient with oral thrush and odynophagia, treat for candidiasis
without doing an upper endoscopy.
e absence of oral thrush does not rule out esophageal candidiasis
Fluconazole or Itraconazole ae the treatment of choice for candidiasis

Page - 805
Internal Medicine - Gastroenterology

Question 59/106

Question #59

e following agents are known to induce esophagitis except:

a. Tetracycline
b. Alendronate
c. NSAIDs
d. Quinidine
e. Lansoprazole

‫اﻹﺟﺎﺑﺔ ﻋﲆ اﻟﺼﻔﺤﺔ اﻟﺘﺎﻟﻴﺔ‬

Page - 806
Internal Medicine - Gastroenterology - Esophagitis

Question 59/106

Question #59

e following agents are known to induce esophagitis except:

a. Tetracycline
b. Alendronate
c. NSAIDs
d. Quinidine
e. Lansoprazole √

Description

Proton pump inhibitors are not known to cause drug-induced esophagitis

e following agents may cause Drug-induced esophagitis:

Potassium supplements
NSAIDs
Tetracyclines
Bisphosphonates

Page - 807
Internal Medicine - Gastroenterology

Question 60/106

Question #60

A 22-year-old male patient presents to your oce with recurrent dull aching chest pain for 6
months, associated with a bitter taste in his mouth. ere is no exercise intolerance, and his physical
examination and lab investigations are normal. What is the most likely diagnosis?

a. Myocardial ischemia
b. Muscular chest pain
c. Reflux esophagitis
d. Pneumonia
e. Anxiety

‫اﻹﺟﺎﺑﺔ ﻋﲆ اﻟﺼﻔﺤﺔ اﻟﺘﺎﻟﻴﺔ‬

Page - 808
Internal Medicine - Gastroenterology - Esophagitis

Question 60/106

Question #60

A 22-year-old male patient presents to your oce with recurrent dull aching chest pain for 6
months, associated with a bitter taste in his mouth. ere is no exercise intolerance, and his physical
examination and lab investigations are normal. What is the most likely diagnosis?

a. Myocardial ischemia
b. Muscular chest pain
c. Reflux esophagitis √
d. Pneumonia
e. Anxiety

Description

e most common non-cardiac cause of chest pain is GI pain. Note that the bitter taste in his mouth
will support the diagnosis of GERD, and esophageal esophagitis is the most likely diagnosis

Myocardial ischemia presents in older ages with risk factors and with an exertional chest pain

Pneumonia comes with fever, cough, and sputum, along with the chest pain.

Page - 809
Internal Medicine - Gastroenterology

Question 61/106

Question #61

A 22-year-old male patient has had intermittent and dull chest pain for the past 6 months. e pain
usually occurs at night, is associated with a bitter taste, and is unrelated to exercise. Physical
examination is normal. Which one of the following is the most likely diagnosis?

a. Generalized anxiety disorder


b. Transient myocardial ischemia
c. Reflux esophagitis
d. Myocardial infarction
e. Costochondritis

‫اﻹﺟﺎﺑﺔ ﻋﲆ اﻟﺼﻔﺤﺔ اﻟﺘﺎﻟﻴﺔ‬

Page - 810
Internal Medicine - Gastroenterology - Gastroesophageal Reflux Disease (GERD)

Question 61/106

Question #61

A 22-year-old male patient has had intermittent and dull chest pain for the past 6 months. e pain
usually occurs at night, is associated with a bitter taste, and is unrelated to exercise. Physical
examination is normal. Which one of the following is the most likely diagnosis?

a. Generalized anxiety disorder


b. Transient myocardial ischemia
c. Reflux esophagitis √
d. Myocardial infarction
e. Costochondritis

Description

A central chest pain related to food and a bitter taste is most likely due to reflux-induced
esophagitis. CAD chest pain is usually related to exertion. Costochondritis is associated with chest
wall tenderness.

Gastroesophageal reflux disease (GERD):

Reflux disease: reflux of gastric acidity to the esophagus.


It happens due to inappropriate relaxation of the lower esophageal sphincter (LES)
Heartburn is the most common symptom
Gastric content regurgitation, Chest pain, and respiratory symptoms may manifest.
Hoarseness, metallic taste, and dental caries are not uncommon in GERD, but dysphagia is a
late sign.
e typical features and the improvement aer the trial of Proton pump inhibitors are usually
diagnostic, but 24-hour PH Monitoring is the best investigation
Upper endoscopy is indicated if the patient has any alarming symptoms.

Page - 811
Internal Medicine - Gastroenterology

Question 62/106

Question #62

A 32-year-old female presents to your oce with a 6-month history of cough. She denies fever,
weight loss, heartburn, or regurgitation of food. Her physical examination is unremarkable.
However, a trial of Salbutamol and antihistamine doesn’t improve the patient’s condition. What
would you do next?

a. Methacholine challenge test


b. Pulmonary function test
c. Chest high-resolution CT scan
d. A trial of PPI
e. 24-hour PH monitoring of the esophagus

‫اﻹﺟﺎﺑﺔ ﻋﲆ اﻟﺼﻔﺤﺔ اﻟﺘﺎﻟﻴﺔ‬

Page - 812
Internal Medicine - Gastroenterology - Gastroesophageal Reflux Disease (GERD)

Question 62/106

Question #62

A 32-year-old female presents to your oce with a 6-month history of cough. She denies fever,
weight loss, heartburn, or regurgitation of food. Her physical examination is unremarkable.
However, a trial of Salbutamol and antihistamine doesn’t improve the patient’s condition. What
would you do next?

a. Methacholine challenge test


b. Pulmonary function test
c. Chest high-resolution CT scan
d. A trial of PPI √
e. 24-hour PH monitoring of the esophagus

Description

In patients with chronic cough and no suggestive signs or symptoms of respiratory or cardiac
disorder, using a twice-daily PPI for 3 months does demonstrate the causal relationship between
GERD and extraesophageal symptoms

e lack of response to SABA will rule out asthma, so the methacholine challenge test is not
indicated

ere is no suggestive history of lung disease in this patient, making PFT and Chest CT not indicated
at this time

Despite that, it is the most accurate test, but an initial therapeutic trial of proton pump inhibitors is
favored over 24-hour pH monitoring because it is less uncomfortable to the patient and has a better
clinical correlation.

Gastroesophageal reflux disease (GERD):

Reflux disease: reflux of gastric acidity to the esophagus.


It happens due to inappropriate relaxation of the lower esophageal sphincter (LES)
Heartburn is the most common symptom
Gastric content regurgitation, Chest pain, and respiratory symptoms may manifest.
Hoarseness, metallic taste, and dental caries are not uncommon in GERD, but dysphagia is a
late sign. Page - 813
e typical features and the improvement aer the trial of Proton pump inhibitors are usually
diagnostic, but 24-hour PH Monitoring is the best investigation
Upper endoscopy is indicated if the patient has any alarming symptoms.

Page - 814
Internal Medicine - Gastroenterology

Question 63/106

Question #63

A non-smoker 33-year-old male presents to your oce with recurrent wheezing for the past month.
He has no history of asthma or any medical illness. However, on examination, the patient has
hoarseness of voice and a red, inflamed larynx. What is the initial treatment of this patient’s
condition?

a. Antibiotics
b. Proton pump inhibitors
c. Steroids
d. Albuterol inhaler
e. Salmeterol inhaler

‫اﻹﺟﺎﺑﺔ ﻋﲆ اﻟﺼﻔﺤﺔ اﻟﺘﺎﻟﻴﺔ‬

Page - 815
Internal Medicine - Gastroenterology - Gastroesophageal Reflux Disease (GERD)

Question 63/106

Question #63

A non-smoker 33-year-old male presents to your oce with recurrent wheezing for the past month.
He has no history of asthma or any medical illness. However, on examination, the patient has
hoarseness of voice and a red, inflamed larynx. What is the initial treatment of this patient’s
condition?

a. Antibiotics
b. Proton pump inhibitors √
c. Steroids
d. Albuterol inhaler
e. Salmeterol inhaler

Description

e hoarseness of voice, inflamed larynx, and wheezes in the chest are extraesophageal
manifestations of GERD. erefore, the trial of PPI is the best initial treatment for this patient.

Gastroesophageal reflux disease (GERD):

Reflux disease: reflux of gastric acidity to the esophagus.


It happens due to inappropriate relaxation of the lower esophageal sphincter (LES)
Heartburn is the most common symptom
Gastric content regurgitation, Chest pain, and respiratory symptoms may manifest.
Hoarseness, metallic taste, and dental caries are not uncommon in GERD, but dysphagia is a
late sign.
e typical features and the improvement aer the trial of Proton pump inhibitors are usually
diagnostic, but 24-hour PH Monitoring is the best investigation
Upper endoscopy is indicated if the patient has any alarming symptoms.

Page - 816
Internal Medicine - Gastroenterology

Question 64/106

Question #64

A 39-year-old male patient has chronic cough for 6 months. ere is no fever, weight loss, or any
systemic manifestations. Auscultation of the lungs shows no evidence of acute disease. However, a
trial of inhaled bronchodilator doesn’t improve the symptoms. What is the most appropriate next
step?

a. Methacholine challenge test


b. Pulmonary function test
c. Chest CT scan
d. A trial of proton pump inhibitor (PPI)
e. A 24-hours PH monitoring

‫اﻹﺟﺎﺑﺔ ﻋﲆ اﻟﺼﻔﺤﺔ اﻟﺘﺎﻟﻴﺔ‬

Page - 817
Internal Medicine - Gastroenterology - Gastroesophageal Reflux Disease (GERD)

Question 64/106

Question #64

A 39-year-old male patient has chronic cough for 6 months. ere is no fever, weight loss, or any
systemic manifestations. Auscultation of the lungs shows no evidence of acute disease. However, a
trial of inhaled bronchodilator doesn’t improve the symptoms. What is the most appropriate next
step?

a. Methacholine challenge test


b. Pulmonary function test
c. Chest CT scan
d. A trial of proton pump inhibitor (PPI) √
e. A 24-hours PH monitoring

Description

is patient is typical for GERD; you have to try PPI.

A positive response to PPI indicates a higher probability of GERD.

Note that 24-hour PH monitoring is the most accurate test to diagnose GERD.

A pulmonary function test and methacholine challenge test are used when asthma is the most likely
diagnosis. e patient’s unresponsiveness to bronchodilators excludes asthma.

Gastroesophageal reflux disease (GERD):

Reflux disease: reflux of gastric acidity to the esophagus.


It happens due to inappropriate relaxation of the lower esophageal sphincter (LES)
Heartburn is the most common symptom
Gastric content regurgitation, Chest pain, and respiratory symptoms may manifest.
Hoarseness, metallic taste, and dental caries are not uncommon in GERD, but dysphagia is a
late sign.
e typical features and the improvement aer the trial of Proton pump inhibitors are usually
diagnostic, but 24-hour PH Monitoring is the best investigation
Upper endoscopy is indicated if the patient has any alarming symptoms.

Page - 818
Internal Medicine - Gastroenterology

Question 65/106

Question #65

A 62-year-old diabetic male patient presents with nausea, postprandial bloating, and early satiety
for 2 months. ere is no abdominal pain or heartburn. Physical examination and lab investigations
are all unremarkable. Which one of the following would help confirm the most likely diagnosis?

a. 24-hours PH monitoring
b. Abdominal ultrasound
c. Gastric emptying scintigraphy
d. H. pylori serology test
e. Upper endoscopy

‫اﻹﺟﺎﺑﺔ ﻋﲆ اﻟﺼﻔﺤﺔ اﻟﺘﺎﻟﻴﺔ‬

Page - 819
Internal Medicine - Gastroenterology - Gastroparesis

Question 65/106

Question #65

A 62-year-old diabetic male patient presents with nausea, postprandial bloating, and early satiety
for 2 months. ere is no abdominal pain or heartburn. Physical examination and lab investigations
are all unremarkable. Which one of the following would help confirm the most likely diagnosis?

a. 24-hours PH monitoring
b. Abdominal ultrasound
c. Gastric emptying scintigraphy √
d. H. pylori serology test
e. Upper endoscopy

Description

is patient is most likely suering from autonomic diabetic neuropathy causing gastroparesis. It
can be confirmed by a nuclear gastric emptying study

Gastroparesis:

It is a defective gastric emptying but without obstruction.


It can be an inherited or acquired disorder of the gastric pacemaker or an autonomic disorder
Longstanding diabetes leads to gastric dysmotility is the most common cause
e diagnosis is clinical, but a gastric emptying nuclear study is accurate and helpful in
showing reduced gastric emptying in these patients.
e treatment is achieved by drugs that enhance gastric emptying (e.g., erythromycin and
metoclopramide)

Page - 820
Internal Medicine - Gastroenterology

Question 66/106

Question #66

A 62-year-old diabetic male patient presents with nausea, postprandial bloating, and early satiety
for 2 months. ere is no abdominal pain or heartburn. Physical examination and lab investigations
are all unremarkable. What is the next step in the management of this patient?

a. Domperidone
b. Omeprazole
c. H pylori eradication
d. Erythromycin
e. Famotidine

‫اﻹﺟﺎﺑﺔ ﻋﲆ اﻟﺼﻔﺤﺔ اﻟﺘﺎﻟﻴﺔ‬

Page - 821
Internal Medicine - Gastroenterology - Gastroparesis

Question 66/106

Question #66

A 62-year-old diabetic male patient presents with nausea, postprandial bloating, and early satiety
for 2 months. ere is no abdominal pain or heartburn. Physical examination and lab investigations
are all unremarkable. What is the next step in the management of this patient?

a. Domperidone
b. Omeprazole
c. H pylori eradication
d. Erythromycin √
e. Famotidine

Description

is patient is most likely suering from autonomic diabetic neuropathy causing gastroparesis. e
treatment is achieved by increasing gastric emptying by using erythromycin or metoclopramide

Gastroparesis:

It is a defective gastric emptying but without obstruction.


It can be an inherited or acquired disorder of the gastric pacemaker or an autonomic disorder
Longstanding diabetes leads to gastric dysmotility is the most common cause
e diagnosis is clinical, but a gastric emptying nuclear study is accurate and helpful in
showing reduced gastric emptying in these patients.
e treatment is achieved by drugs that enhance gastric emptying (e.g., erythromycin and
metoclopramide)

Page - 822
Internal Medicine - Gastroenterology

Question 67/106

Question #67

A 32-year-old female from Irbid, Jordan, presents to you with dyspepsia. She was diagnosed with a
duodenal ulcer secondary to H pylori. Which of the following would be the preferred treatment
option for her condition?

a. Lansoprazole alone
b. Combined antibiotics and PPI
c. Surgical management
d. Famotidine
e. Liquid aluminum-containing antacids

‫اﻹﺟﺎﺑﺔ ﻋﲆ اﻟﺼﻔﺤﺔ اﻟﺘﺎﻟﻴﺔ‬

Page - 823
Internal Medicine - Gastroenterology - H. pylori Infection

Question 67/106

Question #67

A 32-year-old female from Irbid, Jordan, presents to you with dyspepsia. She was diagnosed with a
duodenal ulcer secondary to H pylori. Which of the following would be the preferred treatment
option for her condition?

a. Lansoprazole alone
b. Combined antibiotics and PPI √
c. Surgical management
d. Famotidine
e. Liquid aluminum-containing antacids

Description

H pylori is a bacterium that is highly resistant to antibiotics. It needs a combination of more than
one antibiotic along with PPI to eradicate it.

Peptic ulcer disease (PUD):

It is an ulceration of the lower esophagus, stomach, or duodenum and can occur in the ileum
adjacent to Meckel’s diverticulum.
e most common site of gastric ulcer (GU) is at the lesser curvature (exactly at the Incisura
angularis)
e most common cause in developing countries is H. Pylori infection, while NSAIDs are
responsible for most cases in developed countries
All patients with peptic ulcer disease should be tested for H. pylori regardless of the use of
NSAIDs
Dyspepsia is the most common symptom, but epigastric pain, GI bleeding, and vomiting may
present.
Upper GI endoscopy is considered the most accurate test
e treatment is achieved by elimination of the cause and the use of PPIs

Page - 824
Internal Medicine - Gastroenterology

Question 68/106

Question #68

A 35-year-old female has H pylori infection and epigastric pain. She has been treated for 14 days with
amoxicillin, clarithromycin, and lansoprazole. She is now asymptomatic. Which of the following is
recommended to test for H pylori eradication?

a. Upper endoscopy and biopsy


b. Rapid ureas test
c. IgG serology test
d. Barium meal
e. Stool antigen test

‫اﻹﺟﺎﺑﺔ ﻋﲆ اﻟﺼﻔﺤﺔ اﻟﺘﺎﻟﻴﺔ‬

Page - 825
Internal Medicine - Gastroenterology - H. pylori Infection

Question 68/106

Question #68

A 35-year-old female has H pylori infection and epigastric pain. She has been treated for 14 days with
amoxicillin, clarithromycin, and lansoprazole. She is now asymptomatic. Which of the following is
recommended to test for H pylori eradication?

a. Upper endoscopy and biopsy


b. Rapid ureas test
c. IgG serology test
d. Barium meal
e. Stool antigen test √

Description

A serology test is not appropriate to test for H pylori eradication because once positive, it will be
positive for a long time and will give a false-positive result.

It is too aggressive to use endoscopic tests. erefore, they are not indicated as there are no alarm
symptoms, and the patient is not elderly.

Barium meal has no rule in evaluating H pylori infection or its follow-up.

Page - 826
Internal Medicine - Gastroenterology

Question 69/106

Question #69

An endoscopy result of a 40-year-old male patient reveals gastritis and H pylori infection. What is
the standard treatment for this condition?

a. Amoxicillin-clavulanate, famotidine, and bismuth


b. Erythromycin, omeprazole, and metronidazole
c. Esomeprazole, Amoxicillin, and Clarithromycin
d. Fluconazole, omeprazole and metronidazole
e. Lansoprazole, omeprazole and metronidazole

‫اﻹﺟﺎﺑﺔ ﻋﲆ اﻟﺼﻔﺤﺔ اﻟﺘﺎﻟﻴﺔ‬

Page - 827
Internal Medicine - Gastroenterology - H. pylori Infection

Question 69/106

Question #69

An endoscopy result of a 40-year-old male patient reveals gastritis and H pylori infection. What is
the standard treatment for this condition?

a. Amoxicillin-clavulanate, famotidine, and bismuth


b. Erythromycin, omeprazole, and metronidazole
c. Esomeprazole, Amoxicillin, and Clarithromycin √
d. Fluconazole, omeprazole and metronidazole
e. Lansoprazole, omeprazole and metronidazole

Description

Treatment of H. pylori:

Triple therapy:
e first line of treatment
PPI bid, amoxicillin 1g bid, and clarithromycin 500 mg bid for 7 – 14 days
Quadruple therapy:
PPI + metronidazole + tetracycline + bismuth.
Indicated in the probability of clarithromycin resistance or penicillin allergy
Levofloxacin-based therapy: PPI bid + amoxicillin 1g bid + levofloxacin 500 mg qday for 10 – 14
days.

Page - 828
Internal Medicine - Gastroenterology

Question 70/106

Question #70

A 32-year-old male patient presents with epigastric pain related to eating. Upper endoscopy showed
a duodenal ulcer, and the test for H pylori was positive. Which is the best treatment option?

a. Metronidazole
b. Lansoprazole
c. Clarithromycin and amoxicillin
d. Metronidazole and amoxicillin
e. Lansoprazole, clarithromycin, and amoxicillin

‫اﻹﺟﺎﺑﺔ ﻋﲆ اﻟﺼﻔﺤﺔ اﻟﺘﺎﻟﻴﺔ‬

Page - 829
Internal Medicine - Gastroenterology - H. pylori Infection

Question 70/106

Question #70

A 32-year-old male patient presents with epigastric pain related to eating. Upper endoscopy showed
a duodenal ulcer, and the test for H pylori was positive. Which is the best treatment option?

a. Metronidazole
b. Lansoprazole
c. Clarithromycin and amoxicillin
d. Metronidazole and amoxicillin
e. Lansoprazole, clarithromycin, and amoxicillin √

Description

e triple therapy of amoxicillin, clarithromycin, and PPIs is the best initial treatment for H pylori
infection

Note that H pylori are resistant bacteria to most antibiotics and should be treated with a
combination of more than one antibiotic.

Treatment of H. pylori:

Triple therapy:
e first line of treatment
PPI bid, amoxicillin 1g bid, and clarithromycin 500 mg bid for 7 – 14 days
Quadruple therapy:
PPI + metronidazole + tetracycline + bismuth.
Indicated in the probability of clarithromycin resistance or penicillin allergy
Levofloxacin-based therapy: PPI bid + amoxicillin 1g bid + levofloxacin 500 mg qday for 10 – 14
days.

Page - 830
Internal Medicine - Gastroenterology

Question 71/106

Question #71

A 32-year-old male patient was treated by triple therapy for H pylori infection 2 months ago, and
now he is asymptomatic. Another H pylori serology test was done yesterday, which came back
positive. What is the best management at this time?

a. Repeat the triple therapy


b. Start quadrable therapy
c. Start Levofloxacin-based therapy
d. Perform an upper endoscopy
e. Reassurance

‫اﻹﺟﺎﺑﺔ ﻋﲆ اﻟﺼﻔﺤﺔ اﻟﺘﺎﻟﻴﺔ‬

Page - 831
Internal Medicine - Gastroenterology - H. pylori Infection

Question 71/106

Question #71

A 32-year-old male patient was treated by triple therapy for H pylori infection 2 months ago, and
now he is asymptomatic. Another H pylori serology test was done yesterday, which came back
positive. What is the best management at this time?

a. Repeat the triple therapy


b. Start quadrable therapy
c. Start Levofloxacin-based therapy
d. Perform an upper endoscopy
e. Reassurance √

Description

Once it becomes positive, the serology test for H pylori will be positive for a long time (maybe for
more than 3 years), even in the absence of the bacteria itself

For follow-up of H pylori eradication, perform a stool antigen test or urea breath test instead of a
serology test.

Treatment of H. pylori:

Triple therapy:
e first line of treatment
PPI bid, amoxicillin 1g bid, and clarithromycin 500 mg bid for 7 – 14 days
Quadruple therapy:
PPI + metronidazole + tetracycline + bismuth.
Indicated in the probability of clarithromycin resistance or penicillin allergy
Levofloxacin-based therapy: PPI bid + amoxicillin 1g bid + levofloxacin 500 mg qday for 10 – 14
days.

Page - 832
Internal Medicine - Gastroenterology

Question 72/106

Question #72

A 33-year-old male presented to you with a complaint of heartburn and reflux. e patient has no
altered bowel habits, dysphagia, night sweating, or weight loss. His history is only significant for H
pylori infection that was eradicated successfully. What is the most appropriate test to ensure the
eradication of the H pylori infection?

a. H pylori antibodies
b. Upper GI endoscopy
c. C13 urea breath test
d. Blood culture for H pylori
e. Symptomatic response to PPI

‫اﻹﺟﺎﺑﺔ ﻋﲆ اﻟﺼﻔﺤﺔ اﻟﺘﺎﻟﻴﺔ‬

Page - 833
Internal Medicine - Gastroenterology - H. pylori Infection

Question 72/106

Question #72

A 33-year-old male presented to you with a complaint of heartburn and reflux. e patient has no
altered bowel habits, dysphagia, night sweating, or weight loss. His history is only significant for H
pylori infection that was eradicated successfully. What is the most appropriate test to ensure the
eradication of the H pylori infection?

a. H pylori antibodies
b. Upper GI endoscopy
c. C13 urea breath test √
d. Blood culture for H pylori
e. Symptomatic response to PPI

Description

e question here is about H pylori eradication. e C13 urea breath test is the most appropriate of
the mentioned choices.

Serum antibodies for H pylori may remain positive for a long time in 50% of cases of H pylori, even
eradicated.

Upper endoscopy is not indicated in this patient because he is younger than 55-year-old, and there
are no alarming symptoms.

Blood culture is not used to diagnose H pylori infection or eradication.

Page - 834
Internal Medicine - Gastroenterology

Question 73/106

Question #73

A 22-year-old female is on sulfasalazine for Ulcerative colitis. She complains of severe diarrhea 7 – 9
times a day, abdominal pain, and fecal urgency. On examination, the abdomen is tender but not
distended and has no palpable masses. In addition, her temperature is 38.5 °C, her heart rate is 103
bpm, and her blood pressure is normal. What is the next step in the management of this patient?

a. Abdominal x-ray
b. Colonoscopy
c. Stool culture and sensitivity
d. Urgent laparotomy
e. Abdominal CT angiography

‫اﻹﺟﺎﺑﺔ ﻋﲆ اﻟﺼﻔﺤﺔ اﻟﺘﺎﻟﻴﺔ‬

Page - 835
Internal Medicine - Gastroenterology - Inflammatory Bowel Diseases (IBD)

Question 73/106

Question #73

A 22-year-old female is on sulfasalazine for Ulcerative colitis. She complains of severe diarrhea 7 – 9
times a day, abdominal pain, and fecal urgency. On examination, the abdomen is tender but not
distended and has no palpable masses. In addition, her temperature is 38.5 °C, her heart rate is 103
bpm, and her blood pressure is normal. What is the next step in the management of this patient?

a. Abdominal x-ray √
b. Colonoscopy
c. Stool culture and sensitivity
d. Urgent laparotomy
e. Abdominal CT angiography

Description

is lady has acute severe Ulcerative colitis (UC) relapse.

Abdominal x-ray is the best next step in investigating the complications of Ulcerative colitis (to rule
out toxic megacolon)

If the question asks about the best treatment to induce remission, intravenous steroids should be
used.

Page - 836
Internal Medicine - Gastroenterology

Question 74/106

Question #74

A patient has been diagnosed with Crohn’s disease. All the following complications are more
common in Crohn’s disease than Ulcerative colitis except:

a. Fistula formation
b. Perianal abscess
c. Malabsorption
d. Intestinal obstruction
e. Toxic megacolon

‫اﻹﺟﺎﺑﺔ ﻋﲆ اﻟﺼﻔﺤﺔ اﻟﺘﺎﻟﻴﺔ‬

Page - 837
Internal Medicine - Gastroenterology - Inflammatory Bowel Diseases (IBD)

Question 74/106

Question #74

A patient has been diagnosed with Crohn’s disease. All the following complications are more
common in Crohn’s disease than Ulcerative colitis except:

a. Fistula formation
b. Perianal abscess
c. Malabsorption
d. Intestinal obstruction
e. Toxic megacolon √

Description

Toxic megacolon is a severe life-threatening complication of IBD seen more commonly in UC,
characterized by dilatation of the colon with a risk of rupture

Clinical features:

Abdominal pain
Abdominal distension
Fever
Shock, tachycardia

Treatment:

Treatment of the cause


Bowel decompression
IV fluid and electrolytes to prevent dehydration
Surgery (Colectomy) if no improvement within 48 – 72 hours

Page - 838
Internal Medicine - Gastroenterology

Question 75/106

Question #75

A 23-year-old female has chronic abdominal pain and diarrhea. Her rectal biopsy results show a
reduced number of goblet cells. What is the most likely diagnosis?

a. Ulcerative colitis
b. Crohn’s disease
c. Colorectal cancer
d. Irritable bowel syndrome
e. Pseudomembranous colitis

‫اﻹﺟﺎﺑﺔ ﻋﲆ اﻟﺼﻔﺤﺔ اﻟﺘﺎﻟﻴﺔ‬

Page - 839
Internal Medicine - Gastroenterology - Inflammatory Bowel Diseases (IBD)

Question 75/106

Question #75

A 23-year-old female has chronic abdominal pain and diarrhea. Her rectal biopsy results show a
reduced number of goblet cells. What is the most likely diagnosis?

a. Ulcerative colitis √
b. Crohn’s disease
c. Colorectal cancer
d. Irritable bowel syndrome
e. Pseudomembranous colitis

Description

e reduced goblet cells is a histological sign of ulcerative colitis.

Goblet cells are increased in Crohn’s disease.

e following table shows the dierence between UC and CD.

Page - 840
Internal Medicine - Gastroenterology

Question 76/106

Question #76

A 33-year-old female complains of intermittent bloody diarrhea, tenesmus, and abdominal pain.
Which of the following is the most likely diagnosis?

a. Irritable bowel syndrome


b. Bowel cancer
c. Inflammatory bowel disease
d. Endometriosis
e. Giardiasis

‫اﻹﺟﺎﺑﺔ ﻋﲆ اﻟﺼﻔﺤﺔ اﻟﺘﺎﻟﻴﺔ‬

Page - 841
Internal Medicine - Gastroenterology - Inflammatory Bowel Diseases (IBD)

Question 76/106

Question #76

A 33-year-old female complains of intermittent bloody diarrhea, tenesmus, and abdominal pain.
Which of the following is the most likely diagnosis?

a. Irritable bowel syndrome


b. Bowel cancer
c. Inflammatory bowel disease √
d. Endometriosis
e. Giardiasis

Description

is history suggests inflammatory bowel disease (IBD) as the most likely diagnosis.

Bowel cancer is unlikely to present in this age group.

Irritable bowel syndrome never presents with bloody diarrhea

Endometriosis presents with infertility, dysmenorrhea, and dyspareunia

Giardiasis presents with watery diarrhea, not bloody diarrhea.

Page - 842
Internal Medicine - Gastroenterology

Question 77/106

Question #77

A 23-year-old female has chronic abdominal pain and diarrhea. Her colonoscopy and biopsy show a
crypt abscess. What is the most likely diagnosis?

a. Ulcerative colitis
b. Crohn’s disease
c. Colorectal cancer
d. Irritable bowel syndrome
e. Pseudomembranous colitis

‫اﻹﺟﺎﺑﺔ ﻋﲆ اﻟﺼﻔﺤﺔ اﻟﺘﺎﻟﻴﺔ‬

Page - 843
Internal Medicine - Gastroenterology - Inflammatory Bowel Diseases (IBD)

Question 77/106

Question #77

A 23-year-old female has chronic abdominal pain and diarrhea. Her colonoscopy and biopsy show a
crypt abscess. What is the most likely diagnosis?

a. Ulcerative colitis √
b. Crohn’s disease
c. Colorectal cancer
d. Irritable bowel syndrome
e. Pseudomembranous colitis

Description

Crypt abscess is typical of Ulcerative colitis.

e following table shows the dierence between UC and CD.

Page - 844
Internal Medicine - Gastroenterology

Question 78/106

Question #78

A 39-year-old male with a known case of Crohn’s disease suers from low back pain. In addition, the
patient reports morning stiness in his back that is relieved aer getting up and doing some
exercises or by having a hot shower. What is the most likely cause of his pain?

a. Spondyloarthritis
b. Osteoarthritis
c. Osteoporosis
d. Avascular necrosis
e. Metastasis of colon cancer

‫اﻹﺟﺎﺑﺔ ﻋﲆ اﻟﺼﻔﺤﺔ اﻟﺘﺎﻟﻴﺔ‬

Page - 845
Internal Medicine - Gastroenterology - Inflammatory Bowel Diseases (IBD)

Question 78/106

Question #78

A 39-year-old male with a known case of Crohn’s disease suers from low back pain. In addition, the
patient reports morning stiness in his back that is relieved aer getting up and doing some
exercises or by having a hot shower. What is the most likely cause of his pain?

a. Spondyloarthritis √
b. Osteoarthritis
c. Osteoporosis
d. Avascular necrosis
e. Metastasis of colon cancer

Description

Seronegative Spondyloarthritis is a well-known extra-intestinal manifestation of Crohn’s disease.

It may be characterized by back pain and limitation of spinal movement.

is kind of arthritis aects the large joints more than the small ones and has a negative RA factor in
the serology tests.

Page - 846
Internal Medicine - Gastroenterology

Question 79/106

Question #79

A 22-year-old female is on sulfasalazine for Ulcerative colitis. She complains of severe diarrhea 7 – 9
times daily, abdominal pain, and fecal urgency. On examination, the abdomen is tender but not
distended and has no palpable masses. In addition, her temperature is 38.5 °C, her heart rate is 103
bpm, and her blood pressure is normal. What is the next step in the management of this patient?

a. Mesalazine enema
b. Increase the dose of sulfasalazine
c. Intravenous hydrocortisone
d. Oral prednisolone
e. intravenous metronidazole

‫اﻹﺟﺎﺑﺔ ﻋﲆ اﻟﺼﻔﺤﺔ اﻟﺘﺎﻟﻴﺔ‬

Page - 847
Internal Medicine - Gastroenterology - Inflammatory Bowel Diseases (IBD)

Question 79/106

Question #79

A 22-year-old female is on sulfasalazine for Ulcerative colitis. She complains of severe diarrhea 7 – 9
times daily, abdominal pain, and fecal urgency. On examination, the abdomen is tender but not
distended and has no palpable masses. In addition, her temperature is 38.5 °C, her heart rate is 103
bpm, and her blood pressure is normal. What is the next step in the management of this patient?

a. Mesalazine enema
b. Increase the dose of sulfasalazine
c. Intravenous hydrocortisone √
d. Oral prednisolone
e. intravenous metronidazole

Description

is lady has acute severe Ulcerative colitis (UC) relapse.

Intravenous steroid here is indicated to induce remission.

e following table shows the treatment options for Ulcerative colitis.

Page - 848
Internal Medicine - Gastroenterology

Question 80/106

Question #80

A 25-year-old male patient has had abdominal pain and diarrhea for 4 months. His lower endoscopy
demonstrates Deep ulcerations and skip lesions. What is the most likely diagnosis?

a. Ulcerative colitis
b. Crohn’s disease
c. Colorectal cancer
d. Irritable bowel syndrome
e. Pseudomembranous colitis

‫اﻹﺟﺎﺑﺔ ﻋﲆ اﻟﺼﻔﺤﺔ اﻟﺘﺎﻟﻴﺔ‬

Page - 849
Internal Medicine - Gastroenterology - Inflammatory Bowel Diseases (IBD)

Question 80/106

Question #80

A 25-year-old male patient has had abdominal pain and diarrhea for 4 months. His lower endoscopy
demonstrates Deep ulcerations and skip lesions. What is the most likely diagnosis?

a. Ulcerative colitis
b. Crohn’s disease √
c. Colorectal cancer
d. Irritable bowel syndrome
e. Pseudomembranous colitis

Description

Crohn’s disease is characterized by transmural inflammation and skip lesions.

Ulcerative colitis is a superficial and continuous disease.

e following table shows the dierence between CD and UC.

Page - 850
Internal Medicine - Gastroenterology

Question 81/106

Question #81

A 25-year-old male patient has had abdominal pain and diarrhea for 4 months. His lower endoscopy
demonstrates a cobblestone appearance. What is the most likely diagnosis?

a. Ulcerative colitis
b. Crohn’s disease
c. Colorectal cancer
d. Irritable bowel syndrome
e. Pseudomembranous colitis

‫اﻹﺟﺎﺑﺔ ﻋﲆ اﻟﺼﻔﺤﺔ اﻟﺘﺎﻟﻴﺔ‬

Page - 851
Internal Medicine - Gastroenterology - Inflammatory Bowel Diseases (IBD)

Question 81/106

Question #81

A 25-year-old male patient has had abdominal pain and diarrhea for 4 months. His lower endoscopy
demonstrates a cobblestone appearance. What is the most likely diagnosis?

a. Ulcerative colitis
b. Crohn’s disease √
c. Colorectal cancer
d. Irritable bowel syndrome
e. Pseudomembranous colitis

Description

is is a direct clinical scenario of Crohn’s disease.

e cobblestone appearance of the mucosa is pathognomonic for Crohn’s disease.

Page - 852
Internal Medicine - Gastroenterology

Question 82/106

Question #82

A 24-year-old female complains of progressive abdominal pain, abdominal distension, and fever for
a few days. However, the patient has suered from abdominal discomfort, chronic diarrhea, and
malaise for several months. An erect abdominal x-ray shows dilated transverse colon and
edematous mucosa. Her lab investigations show high WBC, positive CRP, and mild microcytic
anemia. What is the most likely diagnosis?

a. Crohn’s disease
b. C. dicile diarrhea
c. Ulcerative colitis
d. Ischemia colitis
e. Celiac disease

‫اﻹﺟﺎﺑﺔ ﻋﲆ اﻟﺼﻔﺤﺔ اﻟﺘﺎﻟﻴﺔ‬

Page - 853
Internal Medicine - Gastroenterology - Inflammatory Bowel Diseases (IBD)

Question 82/106

Question #82

A 24-year-old female complains of progressive abdominal pain, abdominal distension, and fever for
a few days. However, the patient has suered from abdominal discomfort, chronic diarrhea, and
malaise for several months. An erect abdominal x-ray shows dilated transverse colon and
edematous mucosa. Her lab investigations show high WBC, positive CRP, and mild microcytic
anemia. What is the most likely diagnosis?

a. Crohn’s disease
b. C. dicile diarrhea
c. Ulcerative colitis √
d. Ischemia colitis
e. Celiac disease

Description

Positive CRP and high WBC indicate an active inflammatory process that is with the clinical scenario,
suggestive of inflammatory bowel disease

is is a classic presentation of Ulcerative colitis complicated by toxic megacolon.

C. dicile diarrhea is usually present aer the use of antibiotics.

Crohn’s disease is less likely to present with toxic megacolon.

Page - 854
Internal Medicine - Gastroenterology

Question 83/106

Question #83

A 22-year-old female is suspected of having ulcerative colitis. Which of the following is the most
likely finding in barium enema?

a. Loss of haustrations
b. Rose thorn ulcer
c. Intestinal fistula
d. Intestinal stricture
e. Cobblestone appearance

‫اﻹﺟﺎﺑﺔ ﻋﲆ اﻟﺼﻔﺤﺔ اﻟﺘﺎﻟﻴﺔ‬

Page - 855
Internal Medicine - Gastroenterology - Inflammatory Bowel Diseases (IBD)

Question 83/106

Question #83

A 22-year-old female is suspected of having ulcerative colitis. Which of the following is the most
likely finding in barium enema?

a. Loss of haustrations √
b. Rose thorn ulcer
c. Intestinal fistula
d. Intestinal stricture
e. Cobblestone appearance

Description

Loss of haustral markings is pathognomonic for Ulcerative colitis

Fistula, stricture, cobblestone appearance, and rose thorn ulcer are seen in the small bowel in
Crohn’s disease, not ulcerative colitis.

Page - 856
Internal Medicine - Gastroenterology

Question 84/106

Question #84

A 32-year-old female complains of abdominal pain and chronic diarrhea. Which of the following
suggests Crohn’s disease rather than ulcerative colitis?

a. Crypt abscess
b. Positive CRP
c. Pseudomembranes
d. Colonic mucosal involvement
e. Non-caseating granuloma

‫اﻹﺟﺎﺑﺔ ﻋﲆ اﻟﺼﻔﺤﺔ اﻟﺘﺎﻟﻴﺔ‬

Page - 857
Internal Medicine - Gastroenterology - Inflammatory Bowel Diseases (IBD)

Question 84/106

Question #84

A 32-year-old female complains of abdominal pain and chronic diarrhea. Which of the following
suggests Crohn’s disease rather than ulcerative colitis?

a. Crypt abscess
b. Positive CRP
c. Pseudomembranes
d. Colonic mucosal involvement
e. Non-caseating granuloma √

Description

Noncaseating granuloma is pathognomonic for Crohn’s disease.

Crypt abscesses are typical of Ulcerative colitis.

Colonic mucosa can be involved in both CD and UC

CRP is not specific and can be elevated in any inflammatory disease

Pseudomembranes suggest the presence of C. dicile infection.

Page - 858
Internal Medicine - Gastroenterology

Question 85/106

Question #85

A 22-year-old male has complained of bloody diarrhea with mucous 4 times a day for 2 months
associated with abdominal pain. However, the patient denies weight loss. Colonoscopy was
performed. What is the most likely to be found in this patient?

a. Ulcerative colitis
b. Crohn’s disease
c. Intestinal TB
d. Intestinal lymphoma
e. Colorectal carcinoma

‫اﻹﺟﺎﺑﺔ ﻋﲆ اﻟﺼﻔﺤﺔ اﻟﺘﺎﻟﻴﺔ‬

Page - 859
Internal Medicine - Gastroenterology - Inflammatory Bowel Diseases (IBD)

Question 85/106

Question #85

A 22-year-old male has complained of bloody diarrhea with mucous 4 times a day for 2 months
associated with abdominal pain. However, the patient denies weight loss. Colonoscopy was
performed. What is the most likely to be found in this patient?

a. Ulcerative colitis √
b. Crohn’s disease
c. Intestinal TB
d. Intestinal lymphoma
e. Colorectal carcinoma

Description

e two main types of inflammatory bowel disease are Crohn’s disease (CD) and Ulcerative colitis
(UC).

Ulcerative colitis is more likely to present with bloody diarrhea and is usually not associated with
weight loss or malabsorption.

e best initial treatment in UC is mesalazine

Page - 860
Internal Medicine - Gastroenterology

Question 86/106

Question #86

A 19-year-old female complains of abdominal pain and bloody diarrhea. In addition, she has a
significant weight loss. However, her upper endoscopy, abdominal x-ray, and ultrasound are
unremarkable except for a thick intestinal wall. What is the most appropriate next step in the
management?

a. Colonoscopy
b. Abdominal CT scan
c. Stool analysis and culture
d. Repeat the upper endoscopy
e. Abdominal CT angiography

‫اﻹﺟﺎﺑﺔ ﻋﲆ اﻟﺼﻔﺤﺔ اﻟﺘﺎﻟﻴﺔ‬

Page - 861
Internal Medicine - Gastroenterology - Inflammatory Bowel Diseases (IBD)

Question 86/106

Question #86

A 19-year-old female complains of abdominal pain and bloody diarrhea. In addition, she has a
significant weight loss. However, her upper endoscopy, abdominal x-ray, and ultrasound are
unremarkable except for a thick intestinal wall. What is the most appropriate next step in the
management?

a. Colonoscopy √
b. Abdominal CT scan
c. Stool analysis and culture
d. Repeat the upper endoscopy
e. Abdominal CT angiography

Description

is patient is suspected of having Crohn’s disease (CD).

CD most commonly aects the terminal ileum and colon but can occur in any part of the GI tract.

Colonoscopy with biopsy and barium contrast studies are used in the diagnosis of CD

A colonoscopy with biopsy is the most accurate and the best next step in this patient.

Page - 862
Internal Medicine - Gastroenterology

Question 87/106

Question #87

A 33-year-old male patient has complained of abdominal pain, weight loss, and diarrhea for 6
months. on examination, you noted clubbing fingers and abdominal tenderness at the right iliac
fossa. However, colonoscopy and biopsy show transmural granulomatous inflammation involving
the ileocecal junction. What is the most likely diagnosis?

a. Crohn’s disease
b. Ulcerative colitis
c. Bowel carcinoma
d. Intestinal TB
e. Intestinal lymphoma

‫اﻹﺟﺎﺑﺔ ﻋﲆ اﻟﺼﻔﺤﺔ اﻟﺘﺎﻟﻴﺔ‬

Page - 863
Internal Medicine - Gastroenterology - Inflammatory Bowel Diseases (IBD)

Question 87/106

Question #87

A 33-year-old male patient has complained of abdominal pain, weight loss, and diarrhea for 6
months. on examination, you noted clubbing fingers and abdominal tenderness at the right iliac
fossa. However, colonoscopy and biopsy show transmural granulomatous inflammation involving
the ileocecal junction. What is the most likely diagnosis?

a. Crohn’s disease √
b. Ulcerative colitis
c. Bowel carcinoma
d. Intestinal TB
e. Intestinal lymphoma

Description

e two main types of inflammatory bowel disease are Crohn’s disease (CD) and Ulcerative colitis
(UC).

Crohn’s disease is a transmural disease that mainly aects the terminal ileum.

Note that non-caseating granuloma is accurate in dierentiating CD from UC.

Non-caseating granuloma means granulomatous lesions without necrosis

e histological findings of Crohn’s disease are as the following:

Right iliac fossa mass


Granulomas are seen in histology
Increased goblet cells
Transmural, skip lesions, cobbles stone appearance
Bowel enema will show: rose thorn ulcers and fistulae

Page - 864
Internal Medicine - Gastroenterology

Question 88/106

Question #88

A 30-year-old lady complains of chronic diarrhea. She has lost 10 kilograms of her weight in the past
2 months. Aer an appropriate workup, you noted perianal and colonic fistulae. What is the most
likely diagnosis?

a. Crohn’s disease
b. Irritable bowel syndrome
c. Ulcerative colitis
d. Celiac disease
e. Diverticulosis of the colon

‫اﻹﺟﺎﺑﺔ ﻋﲆ اﻟﺼﻔﺤﺔ اﻟﺘﺎﻟﻴﺔ‬

Page - 865
Internal Medicine - Gastroenterology - Inflammatory Bowel Diseases (IBD)

Question 88/106

Question #88

A 30-year-old lady complains of chronic diarrhea. She has lost 10 kilograms of her weight in the past
2 months. Aer an appropriate workup, you noted perianal and colonic fistulae. What is the most
likely diagnosis?

a. Crohn’s disease √
b. Irritable bowel syndrome
c. Ulcerative colitis
d. Celiac disease
e. Diverticulosis of the colon

Description

Crohn’s disease is the most likely diagnosis on this patient

Chronic diarrhea and weight loss are suggestive, but the presence of fistulae accurately
dierentiates Ulcerative colitis from Crohn’s disease.

Page - 866
Internal Medicine - Gastroenterology

Question 89/106

Question #89

A 22-year-old male patient has been diagnosed with inflammatory bowel disease (IBD). Which of the
following is not considered an extra-intestinal manifestation of IBD?

a. Spondyloarthritis
b. Uveitis
c. Primary sclerosing cholangitis
d. Erythema nodosum
e. Facial palsy

‫اﻹﺟﺎﺑﺔ ﻋﲆ اﻟﺼﻔﺤﺔ اﻟﺘﺎﻟﻴﺔ‬

Page - 867
Internal Medicine - Gastroenterology - Inflammatory Bowel Diseases (IBD)

Question 89/106

Question #89

A 22-year-old male patient has been diagnosed with inflammatory bowel disease (IBD). Which of the
following is not considered an extra-intestinal manifestation of IBD?

a. Spondyloarthritis
b. Uveitis
c. Primary sclerosing cholangitis
d. Erythema nodosum
e. Facial palsy √

Description

Extra-intestinal manifestations of inflammatory bowel disease:

Eye: Uveitis, Scleritis, Episcleritis


Cutaneous: Erythema nodosum, Pyoderma gangrenosum
Hepatobiliary: Primary sclerosing cholangitis, Liver cirrhosis, Gallstones (Bile acid
malabsorption in CD)
Pancreas: Pancreatitis
Genitourinary: Nephrolithiasis
Pulmonary: Chronic bronchitis, Bronchiectasis
Musculoskeletal: Seronegative arthritis
Hematology: Anemia, Increased thromboembolic events

Page - 868
Internal Medicine - Gastroenterology

Question 90/106

Question #90

A patient has been diagnosed with Crohn’s disease and is suering from an isolated perianal
disease. Which of the following is the best treatment option for his condition?

a. Amoxicillin
b. Metronidazole
c. Methotrexate
d. Doxycycline
e. TMP/SMX

‫اﻹﺟﺎﺑﺔ ﻋﲆ اﻟﺼﻔﺤﺔ اﻟﺘﺎﻟﻴﺔ‬

Page - 869
Internal Medicine - Gastroenterology - Inflammatory Bowel Diseases (IBD)

Question 90/106

Question #90

A patient has been diagnosed with Crohn’s disease and is suering from an isolated perianal
disease. Which of the following is the best treatment option for his condition?

a. Amoxicillin
b. Metronidazole √
c. Methotrexate
d. Doxycycline
e. TMP/SMX

Description

Metronidazole and ciprofloxacin are used for isolated perianal Crohn’s disease.

Amoxicillin, Doxycycline, and TMP/SMX are not helpful in this case scenario.

Page - 870
Internal Medicine - Gastroenterology

Question 91/106

Question #91

A 25-year-old male patient has had abdominal pain and diarrhea for 4 months. His lower endoscopy
demonstrates a transmural cobblestone appearance. What is the most appropriate management?

a. Metronidazole
b. Paracetamol
c. Mesalazine
d. Diclofenac sodium
e. Vancomycin

‫اﻹﺟﺎﺑﺔ ﻋﲆ اﻟﺼﻔﺤﺔ اﻟﺘﺎﻟﻴﺔ‬

Page - 871
Internal Medicine - Gastroenterology - Inflammatory Bowel Diseases (IBD)

Question 91/106

Question #91

A 25-year-old male patient has had abdominal pain and diarrhea for 4 months. His lower endoscopy
demonstrates a transmural cobblestone appearance. What is the most appropriate management?

a. Metronidazole
b. Paracetamol
c. Mesalazine √
d. Diclofenac sodium
e. Vancomycin

Description

is is a direct clinical scenario of Crohn’s disease.

e cobblestone appearance of the mucosa is pathognomonic for Crohn’s disease.

e following table shows the treatment options for Crohn’s disease.

Page - 872
Internal Medicine - Gastroenterology

Question 92/106

Question #92

Incidentally, you found a mass in the cecum of a 22-year-old female while doing an appendectomy.
Laboratory analysis revealed transmural granulomatous lesion necrosis. What is the most likely
diagnosis?

a. Cecal cancer
b. Lymphoma
c. Crohn’s disease
d. Ulcerative colitis
e. Intestinal TB

‫اﻹﺟﺎﺑﺔ ﻋﲆ اﻟﺼﻔﺤﺔ اﻟﺘﺎﻟﻴﺔ‬

Page - 873
Internal Medicine - Gastroenterology - Inflammatory Bowel Diseases (IBD)

Question 92/106

Question #92

Incidentally, you found a mass in the cecum of a 22-year-old female while doing an appendectomy.
Laboratory analysis revealed transmural granulomatous lesion necrosis. What is the most likely
diagnosis?

a. Cecal cancer
b. Lymphoma
c. Crohn’s disease √
d. Ulcerative colitis
e. Intestinal TB

Description

e two main types of inflammatory bowel disease are Crohn’s disease (CD) and Ulcerative colitis
(UC).

Crohn’s disease is a transmural disease that mainly aects the terminal ileum.

Note that non-caseating granuloma is accurate in dierentiating CD from UC.

Non-caseating granuloma means granulomatous lesions without necrosis

e histological findings of Crohn’s disease are as the following:

Right iliac fossa mass


Granulomas are seen on histology
Increased goblet cells
Transmural, skip lesions, cobbles stone appearance
Bowel enema will show: rose thorn ulcers and fistulae

Page - 874
Internal Medicine - Gastroenterology

Question 93/106

Question #93

A 29-year-old female has been diagnosed with Crohn’s disease. e inflammatory changes are most
commonly found in which of the following sites?

a. Stomach
b. Esophagus
c. Duodenum
d. Terminal ileum
e. Rectum

‫اﻹﺟﺎﺑﺔ ﻋﲆ اﻟﺼﻔﺤﺔ اﻟﺘﺎﻟﻴﺔ‬

Page - 875
Internal Medicine - Gastroenterology - Inflammatory Bowel Diseases (IBD)

Question 93/106

Question #93

A 29-year-old female has been diagnosed with Crohn’s disease. e inflammatory changes are most
commonly found in which of the following sites?

a. Stomach
b. Esophagus
c. Duodenum
d. Terminal ileum √
e. Rectum

Description

Crohn’s disease (CD) most commonly aects the terminal ileum, while Ulcerative Colitis (UC) most
commonly aects the rectum.

CD can involve any part of the GI tract (from mouth to anus), while UC only starts at the rectum and
extends proximally variably but never reaches beyond the ileocecal valve.

Page - 876
Internal Medicine - Gastroenterology

Question 94/106

Question #94

Which of the following, if present, excludes the diagnosis of irritable bowel syndrome?

a. Onset at the age of 24 years


b. Abdominal pain and bloating
c. Constipation
d. Bloody diarrhea
e. Relieved with defecation

‫اﻹﺟﺎﺑﺔ ﻋﲆ اﻟﺼﻔﺤﺔ اﻟﺘﺎﻟﻴﺔ‬

Page - 877
Internal Medicine - Gastroenterology - Irritable Bowel syndrome (IBS)

Question 94/106

Question #94

Which of the following, if present, excludes the diagnosis of irritable bowel syndrome?

a. Onset at the age of 24 years


b. Abdominal pain and bloating
c. Constipation
d. Bloody diarrhea √
e. Relieved with defecation

Description

Bloody diarrhea is considered a red flag.

If any red flag manifests, you should consider a diagnosis other than IBS.

e following are the red flags for irritable bowel syndrome (IBS):

- Age of onset > 50 years

- A family history of IBD

- Weight loss, Anemia, Fever

- Blood or pus in the stool

- Nocturnal defecation

- Abnormal gross findings on flexible sigmoidoscopy

Irritable bowel syndrome:

It is a functional disorder of the large bowel, not explained by investigations.


IBS is the most common cause of GI referral
More in young adults, more in females than males
10-15% of the population

Diagnosis is achieved by Rome IV criteria.

Page - 878
Abdominal pain for at least 1 day per week, for at least 3 months in the past 6 months with 2 out of
the following:

Relieved with defecation


Associated with a change in consistency
Associated with a change in frequency

Page - 879
Internal Medicine - Gastroenterology

Question 95/106

Question #95

A 23-year-old female has a 1-year history of diarrhea and abdominal pain relieved by defecation. No
blood in stool and no weight loss. Her physical examination and lab investigations are
unremarkable. What is the best treatment option for this patient?

a. Gluten-free diet
b. Prednisolone
c. Loperamide
d. Colonoscopy and biopsy
e. Upper GI endoscopy

‫اﻹﺟﺎﺑﺔ ﻋﲆ اﻟﺼﻔﺤﺔ اﻟﺘﺎﻟﻴﺔ‬

Page - 880
Internal Medicine - Gastroenterology - Irritable Bowel syndrome (IBS)

Question 95/106

Question #95

A 23-year-old female has a 1-year history of diarrhea and abdominal pain relieved by defecation. No
blood in stool and no weight loss. Her physical examination and lab investigations are
unremarkable. What is the best treatment option for this patient?

a. Gluten-free diet
b. Prednisolone
c. Loperamide √
d. Colonoscopy and biopsy
e. Upper GI endoscopy

Description

is is typical for Irritable Bowel Syndrome (IBS)

Irritable bowel syndrome:

It is a functional disorder of the large bowel, not explained by investigations.


IBS is the most common cause of GI referral
More in young adults, more in females than males
10-15% of the population

Diagnosis is achieved by Rome IV criteria.

Abdominal pain for at least 1 day per week, for at least 3 months in the past 6 months with 2 out of
the following:

Relieved with defecation


Associated with a change in consistency
Associated with a change in frequency

Loperamide is used to treat diarrhea-dominant IBS.

Page - 881
Internal Medicine - Gastroenterology

Question 96/106

Question #96

A 51-year-old male patient presents with abdominal pain and alternating diarrhea and constipation.
e pain is relieved by defecation. Which of the following would indicate further evaluation?

a. e age of onset
b. e presence of diarrhea
c. e presence of constipation
d. the pain relief by defecation
e. non-bloody nature of the diarrhea

‫اﻹﺟﺎﺑﺔ ﻋﲆ اﻟﺼﻔﺤﺔ اﻟﺘﺎﻟﻴﺔ‬

Page - 882
Internal Medicine - Gastroenterology - Irritable Bowel syndrome (IBS)

Question 96/106

Question #96

A 51-year-old male patient presents with abdominal pain and alternating diarrhea and constipation.
e pain is relieved by defecation. Which of the following would indicate further evaluation?

a. e age of onset √
b. e presence of diarrhea
c. e presence of constipation
d. the pain relief by defecation
e. non-bloody nature of the diarrhea

Description

If any red flag manifests, you should think about a diagnosis other than IBS.

e following are the red flags for irritable bowel syndrome (IBS):

- Age of onset > 50 years

- A family history of IBD

- Weight loss, Anemia, Fever

- Blood or pus in the stool

- Nocturnal defecation

- Abnormal gross findings on flexible sigmoidoscopy

Page - 883
Internal Medicine - Gastroenterology

Question 97/106

Question #97

A 25-year-old female has 6 months history of abdominal pain relieved by defecation and associated
with alternating diarrhea and constipation. Which of the following would indicate a further
evaluation in this patient?

a. Symptoms relieved with defecation


b. Change in stool consistency
c. Change in stool frequency
d. Abdominal bloating and flatulence
e. Passage of mucus with the stool

‫اﻹﺟﺎﺑﺔ ﻋﲆ اﻟﺼﻔﺤﺔ اﻟﺘﺎﻟﻴﺔ‬

Page - 884
Internal Medicine - Gastroenterology - Irritable Bowel syndrome (IBS)

Question 97/106

Question #97

A 25-year-old female has 6 months history of abdominal pain relieved by defecation and associated
with alternating diarrhea and constipation. Which of the following would indicate a further
evaluation in this patient?

a. Symptoms relieved with defecation


b. Change in stool consistency
c. Change in stool frequency
d. Abdominal bloating and flatulence
e. Passage of mucus with the stool √

Description

Mucus or blood in stool is considered a red flag.

If any red flag manifests, you should consider a diagnosis other than IBS.

e following are the red flags for irritable bowel syndrome (IBS):

- Age of onset > 50 years

- A family history of IBD

- Weight loss, Anemia, Fever

- Blood or pus in the stool

- Nocturnal defecation

- Abnormal gross findings on flexible sigmoidoscopy

Irritable bowel syndrome:

It is a functional disorder of the large bowel, not explained by investigations.


IBS is the most common cause of GI referral
More in young adults, more in females than males
10-15% of the population
Page - 885
Diagnosis is achieved by Rome IV criteria.

Abdominal pain for at least 1 day per week, for at least 3 months in the past 6 months with 2 out of
the following:

Relieved with defecation


Associated with a change in consistency
Associated with a change in frequency

Page - 886
Internal Medicine - Gastroenterology

Question 98/106

Question #98

A 45-year-old male patient from Amman, Jordan, presents with dyspepsia and epigastric pain
aggravated by food. You suspect a peptic ulcer disease. What is the most likely cause of his
condition?

a. H pylori
b. NSAIDs use
c. Gastrinoma
d. Crohn’s disease
e. Stress ulcer

‫اﻹﺟﺎﺑﺔ ﻋﲆ اﻟﺼﻔﺤﺔ اﻟﺘﺎﻟﻴﺔ‬

Page - 887
Internal Medicine - Gastroenterology - Peptic ulcer disease (PUD)

Question 98/106

Question #98

A 45-year-old male patient from Amman, Jordan, presents with dyspepsia and epigastric pain
aggravated by food. You suspect a peptic ulcer disease. What is the most likely cause of his
condition?

a. H pylori √
b. NSAIDs use
c. Gastrinoma
d. Crohn’s disease
e. Stress ulcer

Description

In developing countries, H pylori infection is the most common cause of peptic ulcer disease, while
NSAID use is the most common cause in developed countries

ere are no suggestive features of Crohn’s, Stress ulcer, or Gastrinoma.

Peptic ulcer disease (PUD):

It is an ulceration of the lower esophagus, stomach, or duodenum and can occur in the ileum
adjacent to Meckel’s diverticulum.
e most common site of gastric ulcer (GU) is at the lesser curvature (exactly at the Incisura
angularis)
e most common cause in developing countries is H. Pylori infection, while NSAIDs are
responsible for most cases in developed countries
All patients with peptic ulcer disease should be tested for H. pylori regardless of the use of
NSAIDs
Dyspepsia is the most common symptom, but epigastric pain, GI bleeding, and vomiting may
present.
Upper GI endoscopy is considered the most accurate test
e treatment is achieved by elimination of the cause and the use of PPIs

Page - 888
Internal Medicine - Gastroenterology

Question 99/106

Question #99

A 45-year-old male patient from America presents with dyspepsia and epigastric pain aggravated by
food. You suspect a peptic ulcer disease. What is the most likely cause of his condition?

a. H pylori
b. NSAIDs use
c. Gastrinoma
d. Crohn’s disease
e. Stress ulcer

‫اﻹﺟﺎﺑﺔ ﻋﲆ اﻟﺼﻔﺤﺔ اﻟﺘﺎﻟﻴﺔ‬

Page - 889
Internal Medicine - Gastroenterology - Peptic ulcer disease (PUD)

Question 99/106

Question #99

A 45-year-old male patient from America presents with dyspepsia and epigastric pain aggravated by
food. You suspect a peptic ulcer disease. What is the most likely cause of his condition?

a. H pylori
b. NSAIDs use √
c. Gastrinoma
d. Crohn’s disease
e. Stress ulcer

Description

In developing countries, H pylori infection is the most common cause of peptic ulcer disease, while
NSAID use is the most common cause in developed countries

ere are no suggestive features of Crohn’s, Stress ulcer, or Gastrinoma.

Peptic ulcer disease (PUD):

It is an ulceration of the lower esophagus, stomach, or duodenum and can occur in the ileum
adjacent to Meckel’s diverticulum.
e most common site of gastric ulcer (GU) is at the lesser curvature (exactly at the Incisura
angularis)
e most common cause in developing countries is H. Pylori infection, while NSAIDs are
responsible for most cases in developed countries
All patients with peptic ulcer disease should be tested for H. pylori regardless of the use of
NSAIDs
Dyspepsia is the most common symptom, but epigastric pain, GI bleeding, and vomiting may
present.
Upper GI endoscopy is considered the most accurate test
e treatment is achieved by elimination of the cause and the use of PPIs

Page - 890
Internal Medicine - Gastroenterology

Question 100/106

Question #100

A patient has an endoscopy-confirmed chronic peptic ulcer disease, and the biopsy showed no
malignancy. Which of the following is the best method for curing his disease?

a. H pylori eradication
b. Proton pump inhibitors
c. H2 blockers
d. Antacids
e. Nissen fundoplication

‫اﻹﺟﺎﺑﺔ ﻋﲆ اﻟﺼﻔﺤﺔ اﻟﺘﺎﻟﻴﺔ‬

Page - 891
Internal Medicine - Gastroenterology - Peptic ulcer disease (PUD)

Question 100/106

Question #100

A patient has an endoscopy-confirmed chronic peptic ulcer disease, and the biopsy showed no
malignancy. Which of the following is the best method for curing his disease?

a. H pylori eradication √
b. Proton pump inhibitors
c. H2 blockers
d. Antacids
e. Nissen fundoplication

Description

e disruption of the gastric mucosal defense mechanism caused by H pylori will make the
treatment dicult without the eradication of the bacteria

Patients with H pylori eradication have a much lower recurrence rate of peptic ulcer disease
compared with those without H pylori eradication.

Peptic ulcer disease (PUD):

It is an ulceration of the lower esophagus, stomach, or duodenum and can occur in the ileum
adjacent to Meckel’s diverticulum.
e most common site of gastric ulcer (GU) is at the lesser curvature (exactly at the Incisura
angularis)
e most common cause in developing countries is H. Pylori infection, while NSAIDs are
responsible for most cases in developed countries
All patients with peptic ulcer disease should be tested for H. pylori regardless of the use of
NSAIDs
Dyspepsia is the most common symptom, but epigastric pain, GI bleeding, and vomiting may
present.
Upper GI endoscopy is considered the most accurate test
e treatment is achieved by elimination of the cause and the use of PPIs

Page - 892
Internal Medicine - Gastroenterology

Question 101/106

Question #101

Which of the following drugs is the most potent in the mean of reducing stomach acidity in the
treatment of gastritis?

a. Vitamin C
b. Famotidine
c. Aluminum containing antacids
d. Itraconazole
e. Pantoprazole

‫اﻹﺟﺎﺑﺔ ﻋﲆ اﻟﺼﻔﺤﺔ اﻟﺘﺎﻟﻴﺔ‬

Page - 893
Internal Medicine - Gastroenterology - Peptic ulcer disease (PUD)

Question 101/106

Question #101

Which of the following drugs is the most potent in the mean of reducing stomach acidity in the
treatment of gastritis?

a. Vitamin C
b. Famotidine
c. Aluminum containing antacids
d. Itraconazole
e. Pantoprazole √

Description

Proton Pump Inhibitors (PPIs) are the most eective in reducing stomach acidity and are the most
appropriate in treating gastritis and peptic ulcer disease.

Note that Itraconazole is an antifungal; it doesn’t reduce acidity. Furthermore, vitamin C (ascorbic
acid) will increase stomach acidity.

e following figure demonstrates the site of action of each drug used to reduce stomach acidity:

Page - 894
Internal Medicine - Gastroenterology

Question 102/106

Question #102

A 32-year-old male patient with a biopsy documented benign gastric ulcer disease was treated orally
for 4 weeks with lansoprazole with no improvement. His upper endoscopy was repeated and
revealed that the ulcer was still there. A second biopsy was sent, and no malignancy or H Pylori was
present. What would you do next in the management of this patient?

a. Stop lansoprazole and start famotidine


b. Continue treatment with lansoprazole for another 4 weeks
c. Endoscopic Pneumatic dilatation
d. Start triple therapy for H pylori
e. Perform a truncal vagotomy

‫اﻹﺟﺎﺑﺔ ﻋﲆ اﻟﺼﻔﺤﺔ اﻟﺘﺎﻟﻴﺔ‬

Page - 895
Internal Medicine - Gastroenterology - Peptic ulcer disease (PUD)

Question 102/106

Question #102

A 32-year-old male patient with a biopsy documented benign gastric ulcer disease was treated orally
for 4 weeks with lansoprazole with no improvement. His upper endoscopy was repeated and
revealed that the ulcer was still there. A second biopsy was sent, and no malignancy or H Pylori was
present. What would you do next in the management of this patient?

a. Stop lansoprazole and start famotidine


b. Continue treatment with lansoprazole for another 4 weeks √
c. Endoscopic Pneumatic dilatation
d. Start triple therapy for H pylori
e. Perform a truncal vagotomy

Description

PPI’s healed duodenal ulcers in more than 95% of patients at 4 weeks and gastric ulcers in 80 – 90%
of patients at 8 weeks.

So, the patient should be given another 4-week chance to get better with no change in the
treatment

Peptic ulcer disease (PUD):

It is an ulceration of the lower esophagus, stomach, or duodenum and can occur in the ileum
adjacent to Meckel’s diverticulum.
e most common site of gastric ulcer (GU) is at the lesser curvature (exactly at the Incisura
angularis)
e most common cause in developing countries is H. Pylori infection, while NSAIDs are
responsible for most cases in developed countries
All patients with peptic ulcer disease should be tested for H. pylori regardless of the use of
NSAIDs
Dyspepsia is the most common symptom, but epigastric pain, GI bleeding, and vomiting may
present.
Upper GI endoscopy is considered the most accurate test
e treatment is achieved by elimination of the cause and the use of PPIs

Page - 896
Internal Medicine - Gastroenterology

Question 103/106

Question #103

A 52-year-old male comes to you with 2 episodes of coee-ground vomiting and epigastric pain. His
history is significant for rheumatoid arthritis, for which he takes NSAIDs. His blood pressure is
130/80, his pulse is 113bpm, and his hemoglobin is 13 g/dL (baseline was 14.5 g/dL). What is the most
appropriate management?

a. Upper endoscopy
b. Intravenous proton pump inhibitor
c. Intravenous fluid
d. Serial HB, KFT, electrolytes
e. All of the above

‫اﻹﺟﺎﺑﺔ ﻋﲆ اﻟﺼﻔﺤﺔ اﻟﺘﺎﻟﻴﺔ‬

Page - 897
Internal Medicine - Gastroenterology - Upper Gastrointestinal bleeding (UGIB)

Question 103/106

Question #103

A 52-year-old male comes to you with 2 episodes of coee-ground vomiting and epigastric pain. His
history is significant for rheumatoid arthritis, for which he takes NSAIDs. His blood pressure is
130/80, his pulse is 113bpm, and his hemoglobin is 13 g/dL (baseline was 14.5 g/dL). What is the most
appropriate management?

a. Upper endoscopy
b. Intravenous proton pump inhibitor
c. Intravenous fluid
d. Serial HB, KFT, electrolytes
e. All of the above √

Description

is patient has active upper GI bleeding. Tachycardia here indicates 15% blood loss; the patient may
have a shock.

e management of upper GI bleeding is as the following:

- Stabilize patient (ABC): NPO, fluid resuscitation

- Blood grouping and cross match and blood transfusion if there is a drop in HB

- Platelet transfusion if the platelet count is less than 50,000

- Vitamin K, FFP is used if there is a coagulopathy

- Serial HB, KFT, electrolytes, vitals monitoring for follow-up of the patient

- NG tube insertion (this will determine UGIB vs. LGIB)

- Treatment of the cause if present

- intravenous proton pump inhibitor

- Octreotide is used to reduce the portal pressure in the case of esophageal varices

- Metoclopramide or erythromycin to increase gastric emptying (used before endoscopy)


Page - 898
- Upper endoscopy (is the most accurate diagnostic and therapeutic procedure in upper GI
bleeding)

- Surgery may have a rule if endoscopic management fails to stop the bleeding

Page - 899
Internal Medicine - Gastroenterology

Question 104/106

Question #104

A 44-year-old female patient has had fatigue and easy tiredness for the past 4 months. e patient
also has painless dysphagia. Her investigations show microcytic hypochromic anemia. In addition,
her upper endoscopy demonstrates an esophageal web at the post-cricoid region. What is the most
likely diagnosis?

a. Plummer-Vinson syndrome
b. Barrett’s esophagus
c. Reflux esophagitis
d. Peptic ulcer disease
e. Esophageal cancer

‫اﻹﺟﺎﺑﺔ ﻋﲆ اﻟﺼﻔﺤﺔ اﻟﺘﺎﻟﻴﺔ‬

Page - 900
Internal Medicine - Gastroenterology - Upper Gastrointestinal bleeding (UGIB)

Question 104/106

Question #104

A 44-year-old female patient has had fatigue and easy tiredness for the past 4 months. e patient
also has painless dysphagia. Her investigations show microcytic hypochromic anemia. In addition,
her upper endoscopy demonstrates an esophageal web at the post-cricoid region. What is the most
likely diagnosis?

a. Plummer-Vinson syndrome √
b. Barrett’s esophagus
c. Reflux esophagitis
d. Peptic ulcer disease
e. Esophageal cancer

Description

Plummer-Vinson syndrome is the presence of esophageal web, dysphagia, and iron deficiency
anemia.

Anemia is due to recurrent bleeding from the web as the food enters through it.

Treatment should include dilating the esophageal web and correcting iron deficiency anemia.

Page - 901
Internal Medicine - Gastroenterology

Question 105/106

Question #105

A 60-year-old male patient with a known H Pylori infection has black stool and dizziness. His blood
pressure is 90/50 mmHg, pulse rate is 129 bpm. e initial investigations demonstrate hemoglobin
of 9.5 g/dL, normal electrolytes, and normal kidney function. e patient received intravenous PPI
and intravenous fluid, and a crossmatch for blood transfusion was sent. What is the next step in the
management of this patient?

a. Surgical intervention
b. Start H Pylori eradication
c. Esophagogastroduodenoscopy
d. Administrate Vitamin K and FFP
e. Administrate intravenous octreotide

‫اﻹﺟﺎﺑﺔ ﻋﲆ اﻟﺼﻔﺤﺔ اﻟﺘﺎﻟﻴﺔ‬

Page - 902
Internal Medicine - Gastroenterology - Upper Gastrointestinal bleeding (UGIB)

Question 105/106

Question #105

A 60-year-old male patient with a known H Pylori infection has black stool and dizziness. His blood
pressure is 90/50 mmHg, pulse rate is 129 bpm. e initial investigations demonstrate hemoglobin
of 9.5 g/dL, normal electrolytes, and normal kidney function. e patient received intravenous PPI
and intravenous fluid, and a crossmatch for blood transfusion was sent. What is the next step in the
management of this patient?

a. Surgical intervention
b. Start H Pylori eradication
c. Esophagogastroduodenoscopy √
d. Administrate Vitamin K and FFP
e. Administrate intravenous octreotide

Description

is patient has active upper GI bleeding. In addition, the patient has tachycardia and hypotension;
this indicates active bleeding and hypovolemic shock.

Vitamin K and FFP are only used when there is coagulopathy

Octreotide is used if the GI bleeding is related to increased portal pressure, e.g., liver cirrhosis and
esophageal varices.

e management of upper GI bleeding is as the following:

- Stabilize patient (ABC): NPO, fluid resuscitation

- Blood grouping and cross match and blood transfusion if there is a drop in HB

- Platelet transfusion if the platelet count is less than 50,000

- Vitamin K, FFP is used if there is a coagulopathy

- Serial HB, KFT, electrolytes, vitals monitoring for follow-up of the patient

- NG tube insertion (this will determine UGIB vs. LGIB)

Page - 903
- Treatment of the cause if present

- intravenous proton pump inhibitor

- Octreotide is used to reduce the portal pressure in the case of esophageal varices

- Metoclopramide or erythromycin to increase gastric emptying (used before endoscopy)

- Upper endoscopy (is the most accurate diagnostic and therapeutic procedure in upper GI
bleeding)

- Surgery may have a rule if endoscopic management fails to stop the bleeding

Page - 904
Internal Medicine - Gastroenterology

Question 106/106

Question #106

A 53-year-old heavily alcoholic male patient presents with night blindness and dry hyperkeratotic
skin. In addition, he has conjunctival dryness and redness. Which of the following supplements
would most likely prevent the patient’s condition?

a. Retinoic acid
b. iamin
c. Cobalamin
d. Folic acid
e. Ascorbic acid

‫اﻹﺟﺎﺑﺔ ﻋﲆ اﻟﺼﻔﺤﺔ اﻟﺘﺎﻟﻴﺔ‬

Page - 905
Internal Medicine - Gastroenterology - Vitamin A Deficiency

Question 106/106

Question #106

A 53-year-old heavily alcoholic male patient presents with night blindness and dry hyperkeratotic
skin. In addition, he has conjunctival dryness and redness. Which of the following supplements
would most likely prevent the patient’s condition?

a. Retinoic acid √
b. iamin
c. Cobalamin
d. Folic acid
e. Ascorbic acid

Description

Given the history of alcoholism, blindness, and hyperkeratosis, vitamin A (retinoic acid) deficiency is
the most likely diagnosis in this patient.

Usually, it occurs in patients with malabsorption or proteinuria, liver disease, alcoholics, and those
receiving total parenteral nutrition (TPN).

Page - 906
Hematology

Page - 907
Hematology

‫ﻣﻮاﺿﻴﻊ اﻷﺳﺌﻠﺔ وأﻋﺪادﻫﺎ‬

1) Anemia classification => 1 Questions


2) Anemia of chronic disease => 2 Questions
3) Anticoagulants => 10 Questions
4) Aplastic Anemia (AA) => 3 Questions
5) Autoimmune hemolytic anemia (AIHA) => 1 Questions
6) Blood transfusion => 4 Questions
7) Deep Vein rombosis (DVT) => 14 Questions
8) Disorders of hemostasis => 2 Questions
9) Disseminated intravascular coagulation (DIC) => 3 Questions
10) Essential rombocythemia (ET) => 1 Questions
11) HIT syndrome => 3 Questions
12) Hematologic investigations => 10 Questions
13) Hemolytic Anemia => 7 Questions
14) Hemophilia => 3 Questions
15) Hereditary Spherocytosis (HS) => 4 Questions
16) Idiopathic rombocytopenic Purpura (ITP) => 2 Questions
17) Iron Deficiency Anemia (IDA) => 9 Questions
18) Lead Poisoning => 2 Questions
19) Leukemia => 7 Questions
20) Lymphoma => 7 Questions
21) Macrocytic megaloblastic anemia => 15 Questions
22) Multiple Myeloma (MM) => 7 Questions
23) Myelofibrosis => 1 Questions
24) Normocytic anemia => 2 Questions
25) Platelet dysfunction => 1 Questions
26) Polycythemia Vera (PV) => 6 Questions
27) Pulmonary Embolism (PE) => 12 Questions
28) Secondary polycythemia => 3 Questions
29) Sickle Cell Anemia (SCA) => 2 Questions
30) alassemia => 5 Questions
31) rombotic rombocytopenic Purpura (TTP) => 2 Questions
32) Tumor lysis syndrome => 1 Questions
33) Von Willebrand Disease (vWD) => 3 Questions

Page - 908
Internal Medicine - Hematology

Question 1/155

Question #1

A 22-year-old female develops anemia with low mean corpuscular volume. Which of the following is
the least likely diagnosis?

a. Iron deficiency anemia


b. alassemia
c. Sideroblastic anemia
d. Anemia of chronic disease
e. Sickle cell anemia

‫اﻹﺟﺎﺑﺔ ﻋﲆ اﻟﺼﻔﺤﺔ اﻟﺘﺎﻟﻴﺔ‬

Page - 909
Internal Medicine - Hematology - Anemia classification

Question 1/155

Question #1

A 22-year-old female develops anemia with low mean corpuscular volume. Which of the following is
the least likely diagnosis?

a. Iron deficiency anemia


b. alassemia
c. Sideroblastic anemia
d. Anemia of chronic disease
e. Sickle cell anemia √

Description

Sickle cell anemia (SCA) is more likely to be associated with normocytic anemia.

Classifications: Anemia is classified according to MCV to

Microcytic anemia: (MCV < 80 fl)

Iron deficiency (the most common cause of anemia)


Anemia of chronic disease (30% of cases)
alassemia
Lead poisoning
Sideroblastic anemia (maybe macrocytic)

Normocytic anemia: (MCV 80-100 fl)

With High Reticulocytes count: Acute bleeding, hemolysis


With low Reticulocytes count
Anemia of chronic disease (most commonly normocytic (70%) but can be microcytic)

Renal/liver disease, Aplastic anemia, Myelofibrosis, Leukemia, Drugs (e.g.,


chemotherapy)

Macrocytic anemia: (MCV > 100 fl)

Megaloblastic: B12 deficiency, Folate deficiency, Drugs that impair DNA synthesis
(methotrexate, sulfa, chemotherapy)
Page - 910
Non-Megaloblastic: Liver disease, Alcoholism, Hypothyroidism, MDs

Page - 911
Internal Medicine - Hematology

Question 2/155

Question #2

A 66-year-old female complains of fatigue and palpitation. She gave a history of chronic kidney
disease and almost daily alcohol consumption. On examination, she has tachycardia, tachypnea,
and pallor. Lab investigations show hemoglobin of 9 g/dL, MCV of 84 fl, and reticulocytes of 0.5%.
What is the most likely diagnosis?

a. Iron deficiency anemia


b. B12 deficiency
c. Alcoholism
d. Autoimmune hemolytic anemia
e. Anemia of chronic disease

‫اﻹﺟﺎﺑﺔ ﻋﲆ اﻟﺼﻔﺤﺔ اﻟﺘﺎﻟﻴﺔ‬

Page - 912
Internal Medicine - Hematology - Anemia of chronic disease

Question 2/155

Question #2

A 66-year-old female complains of fatigue and palpitation. She gave a history of chronic kidney
disease and almost daily alcohol consumption. On examination, she has tachycardia, tachypnea,
and pallor. Lab investigations show hemoglobin of 9 g/dL, MCV of 84 fl, and reticulocytes of 0.5%.
What is the most likely diagnosis?

a. Iron deficiency anemia


b. B12 deficiency
c. Alcoholism
d. Autoimmune hemolytic anemia
e. Anemia of chronic disease √

Description

is patient is most likely suering from anemia of chronic diseases.

e presence of chronic kidney disease and normocytic anemia in an elderly patient strongly
suggests the condition.

Anemia of chronic disease presents in chronic infections, chronic inflammations, and neoplasia (TB,
Osteomyelitis, endocarditis, IBD, Chronic renal failure, SLE, RA, Malignancy).

Iron deficiency anemia will present with microcytic anemia

Alcoholism and B12 deficiency present with macrocytic anemia

Hemolytic anemia presents with normal MCV, but high reticulocyte count, jaundice, and red urine
should be mentioned in the history.

Pathogenesis of anemia of chronic disease:

Decreased release of iron from Bone marrow to developing erythroblast


Decreased erythropoietin production and response
Decreased RBC survival (less than 120 days)

Treatment:

Page - 913
Treatment of the underlying cause
Erythropoietin injection (target Hb should not exceed 11 g/dl)
No response to ferrous treatment
Rarely need a Blood transfusion (usually mild anemia)

Page - 914
Internal Medicine - Hematology

Question 3/155

Question #3

A 60-year-old male patient is known to have rheumatoid arthritis and is on methotrexate. Her lab
investigations show hemoglobin of 8g/dL, MCV of 75 fl, reticulocytes of 1%, and serum ferritin of 70
μmol/L. What is the most likely diagnosis?

a. Folate deficiency anemia


b. Chronic GI bleeding
c. B12 deficiency anemia
d. Lead poisoning
e. Anemia of chronic disease

‫اﻹﺟﺎﺑﺔ ﻋﲆ اﻟﺼﻔﺤﺔ اﻟﺘﺎﻟﻴﺔ‬

Page - 915
Internal Medicine - Hematology - Anemia of chronic disease

Question 3/155

Question #3

A 60-year-old male patient is known to have rheumatoid arthritis and is on methotrexate. Her lab
investigations show hemoglobin of 8g/dL, MCV of 75 fl, reticulocytes of 1%, and serum ferritin of 70
μmol/L. What is the most likely diagnosis?

a. Folate deficiency anemia


b. Chronic GI bleeding
c. B12 deficiency anemia
d. Lead poisoning
e. Anemia of chronic disease √

Description

is patient is most likely suering from anemia of chronic disease.

e presence of chronic disease (Rheumatoid arthritis) and normocytic anemia in an elderly patient
strongly suggests the condition.

Note that anemia of chronic disease is 70% normocytic and 30% microcytic.

Anemia of chronic disease presents in chronic infections, chronic inflammations, and neoplasia (TB,
Osteomyelitis, endocarditis, IBD, Chronic renal failure, SLE, RA, Malignancy).

Iron deficiency anemia will present with a low ferritin level.

e patient is taking methotrexate and is at risk of developing folic acid deficiency anemia. However,
folic acid deficiency will present with macrocytic rather than microcytic anemia.

Pathogenesis of anemia of chronic disease:

Decreased release of iron from Bone marrow to developing erythroblast


Decreased erythropoietin production and response
Decreased RBC survival (less than 120 days)

Treatment:

Treatment of the underlying cause


Page - 916
Erythropoietin injection (target Hb should not exceed 11 g/dl)
No response to ferrous treatment
Rarely need a Blood transfusion (usually mild anemia)

Page - 917
Internal Medicine - Hematology

Question 4/155

Question #4

In which of the following conditions would you consider stopping warfarin and administration of
oral vitamin K for warfarin toxicity?

a. An INR of 5.5 without bleeding


b. An INR of 7.5 with intracranial bleeding
c. An INR of 9 without bleeding
d. An INR of 10 with minor bleeding
e. An INR of 12 with minor bleeding

‫اﻹﺟﺎﺑﺔ ﻋﲆ اﻟﺼﻔﺤﺔ اﻟﺘﺎﻟﻴﺔ‬

Page - 918
Internal Medicine - Hematology - Anticoagulants

Question 4/155

Question #4

In which of the following conditions would you consider stopping warfarin and administration of
oral vitamin K for warfarin toxicity?

a. An INR of 5.5 without bleeding


b. An INR of 7.5 with intracranial bleeding
c. An INR of 9 without bleeding √
d. An INR of 10 with minor bleeding
e. An INR of 12 with minor bleeding

Description

Patients with INR of 5 – 8 without bleeding should be treated with oral vitamin K, while intravenous
vitamin K is indicated when bleeding is present.

e following points are the up-to-date protocol in the treatment of warfarin overdose.

Treatment of warfarin overdose if no bleeding:

INR 5 – 8:
Hold warfarin for 2 doses
Restart warfarin in lower dose when INR < 5
INR > 8:
Stop warfarin
Vitamin K 1 – 5 mg PO
Repeat Vitamin K dose if INR still > 8 for > 24 hours
Restart warfarin in lower dose when INR < 5

Treatment of warfarin overdose with minor bleeding:

Stop warfarin
Vitamin K 1 – 3 mg intravenously
Repeat Vitamin K dose if INR still > 8 for > 24 hours
Restart warfarin in lower dose when INR < 5

Treatment of warfarin overdose with major bleeding:


Page - 919
Stop warfarin
Vitamin K 5 mg intravenously
Provide Prothrombin complex concentrate
Provide FFP is Prothrombin complex concentrate is not available

Page - 920
Internal Medicine - Hematology

Question 5/155

Question #5

A 32-year-old female presents to the emergency department with a suicidal attempt by ingestion of
40 tablets of warfarin. Which of the following is not indicated in the management of this patient?

a. Vitamin K administration
b. Provide Prothrombin complex concentrate
c. Gastric lavage
d. Fresh Frozen Plasma (FFP)
e. Salicylate to prevent skin necrosis

‫اﻹﺟﺎﺑﺔ ﻋﲆ اﻟﺼﻔﺤﺔ اﻟﺘﺎﻟﻴﺔ‬

Page - 921
Internal Medicine - Hematology - Anticoagulants

Question 5/155

Question #5

A 32-year-old female presents to the emergency department with a suicidal attempt by ingestion of
40 tablets of warfarin. Which of the following is not indicated in the management of this patient?

a. Vitamin K administration
b. Provide Prothrombin complex concentrate
c. Gastric lavage
d. Fresh Frozen Plasma (FFP)
e. Salicylate to prevent skin necrosis √

Description

Salicylate inhibits the CYP450 enzyme, reducing metabolization and enhancing the warfarin eect.
erefore, it increased the risk of bleeding and contraindicated in this situation.

Drug overdose should be treated by gastric lavage, activated charcoal, and providing antidote.

Treatment of warfarin overdose if no bleeding:

INR 5 – 8:
Hold warfarin for 2 doses
Restart warfarin in lower dose when INR < 5
INR > 8:
Stop warfarin
Vitamin K 1 – 5 mg PO
Repeat Vitamin K dose if INR still > 8 for > 24 hours
Restart warfarin in lower dose when INR < 5

Treatment of warfarin overdose with minor bleeding:

Stop warfarin
Vitamin K 1 – 3 mg intravenously
Repeat Vitamin K dose if INR still > 8 for > 24 hours
Restart warfarin in lower dose when INR < 5

Treatment of warfarin overdose with major bleeding:


Page - 922
Stop warfarin
Vitamin K 5 mg intravenously
Provide Prothrombin complex concentrate
Provide FFP is Prothrombin complex concentrate is not available

Page - 923
Internal Medicine - Hematology

Question 6/155

Question #6

Which of the following is the best to rapidly restore the normal coagulation in a patient with
warfarin overdose and brain hemorrhage?

a. Vitamin K
b. Fresh Frozen Plasma
c. Prothrombin complex concentrate
d. Tissue plasminogen activator
e. Packed RBC transfusion

‫اﻹﺟﺎﺑﺔ ﻋﲆ اﻟﺼﻔﺤﺔ اﻟﺘﺎﻟﻴﺔ‬

Page - 924
Internal Medicine - Hematology - Anticoagulants

Question 6/155

Question #6

Which of the following is the best to rapidly restore the normal coagulation in a patient with
warfarin overdose and brain hemorrhage?

a. Vitamin K
b. Fresh Frozen Plasma
c. Prothrombin complex concentrate √
d. Tissue plasminogen activator
e. Packed RBC transfusion

Description

Prothrombin complex concentrate is superior to fresh frozen plasma in reducing the INR and
returning the coagulability state.

e following points are the up-to-date protocol in the treatment of warfarin overdose.

Treatment of warfarin overdose if no bleeding:

INR 5 – 8:
Hold warfarin for 2 doses
Restart warfarin in lower dose when INR < 5
INR > 8:
Stop warfarin
Vitamin K 1 – 5 mg PO
Repeat Vitamin K dose if INR still > 8 for > 24 hours
Restart warfarin in lower dose when INR < 5

Treatment of warfarin overdose with minor bleeding:

Stop warfarin
Vitamin K 1 – 3 mg intravenously
Repeat Vitamin K dose if INR still > 8 for > 24 hours
Restart warfarin in lower dose when INR < 5

Treatment of warfarin overdose with major bleeding:


Page - 925
Stop warfarin
Vitamin K 5 mg intravenously
Provide Prothrombin complex concentrate
Provide FFP is Prothrombin complex concentrate is not available

Page - 926
Internal Medicine - Hematology

Question 7/155

Question #7

A 52-year-old female patient has been started on warfarin for mitral valve replacement. Which of
the following is recommended for the monitoring of her anticoagulation status?

a. International normalized ratio (INR)


b. Partial thromboplastin time (PTT)
c. Protein C level
d. Plasma factor Xa levels
e. No need for monitoring

‫اﻹﺟﺎﺑﺔ ﻋﲆ اﻟﺼﻔﺤﺔ اﻟﺘﺎﻟﻴﺔ‬

Page - 927
Internal Medicine - Hematology - Anticoagulants

Question 7/155

Question #7

A 52-year-old female patient has been started on warfarin for mitral valve replacement. Which of
the following is recommended for the monitoring of her anticoagulation status?

a. International normalized ratio (INR) √


b. Partial thromboplastin time (PTT)
c. Protein C level
d. Plasma factor Xa levels
e. No need for monitoring

Description

Monitoring of the Warfarin eect is best done by the international normalized ratio (INR)

INR is the ratio of the prothrombin time (PT) for the patient over the average PT.

e target INR in patients on warfarin:

Unprovoked DVT/PE: 2 – 3 for 6 months


Provoked DVT/PE: 2 – 3 for 3 months
Recurrent DVT/PE: 2 – 3 for life
Atrial fibrillation with high CHA2DS2VAS score: 2 – 3 lifelong
Aortic valve replacement: 2 – 3 lifelong
Mitral valve replacement: 2.5 – 3.5 lifelong

Page - 928
Internal Medicine - Hematology

Question 8/155

Question #8

Which of the following is the most accurate in describing the mechanism of action of warfarin?

a. Interacts with the coagulation factors I and VI


b. Antagonizes the vitamin E production
c. Antagonizes vitamin K
d. Increases the level of protein C
e. Increase the level of vitamin K dependent clotting factors

‫اﻹﺟﺎﺑﺔ ﻋﲆ اﻟﺼﻔﺤﺔ اﻟﺘﺎﻟﻴﺔ‬

Page - 929
Internal Medicine - Hematology - Anticoagulants

Question 8/155

Question #8

Which of the following is the most accurate in describing the mechanism of action of warfarin?

a. Interacts with the coagulation factors I and VI


b. Antagonizes the vitamin E production
c. Antagonizes vitamin K √
d. Increases the level of protein C
e. Increase the level of vitamin K dependent clotting factors

Description

Mechanism of action of warfarin:

Warfarin is a vitamin K antagonist (VKA)


It inhibits clotting factors II, VII, IX, X, and protein C

Page - 930
Internal Medicine - Hematology

Question 9/155

Question #9

A patient who is known case of valve replacement is on warfarin therapy. He recently started on
rifampicin for meningococcal prophylaxis as he was in direct contact with a patient. Which of the
following is the expected Drug-Drug interaction between warfarin and rifampicin?

a. Increased eect of warfarin causing bleeding


b. Reduced eect of warfarin causing thrombosis
c. Increased eect of rifampicin
d. Reduced eect of rifampicin
e. No known interaction

‫اﻹﺟﺎﺑﺔ ﻋﲆ اﻟﺼﻔﺤﺔ اﻟﺘﺎﻟﻴﺔ‬

Page - 931
Internal Medicine - Hematology - Anticoagulants

Question 9/155

Question #9

A patient who is known case of valve replacement is on warfarin therapy. He recently started on
rifampicin for meningococcal prophylaxis as he was in direct contact with a patient. Which of the
following is the expected Drug-Drug interaction between warfarin and rifampicin?

a. Increased eect of warfarin causing bleeding


b. Reduced eect of warfarin causing thrombosis √
c. Increased eect of rifampicin
d. Reduced eect of rifampicin
e. No known interaction

Description

Warfarin is metabolized by the Cytochrome P 450 enzyme system in the liver.

Many drugs and substances aect this enzyme system, leading to an alteration of warfarin eects.

Rifampicin is known to enhance the action of the CYP450 enzyme and increase the washout of
warfarin, leading to a reduction of its anticoagulation eect and lower INR.

e following table shows the most important interactants with warfarin:

Page - 932
Internal Medicine - Hematology

Question 10/155

Question #10

A patient has mitral valve replacement with a prosthetic mechanical valve. Which of the following is
the most appropriate in his management?

a. Anticoagulation with warfarin at target INR 2 – 3


b. Anticoagulation with warfarin at target INR 2.5 – 3.5
c. Anticoagulation with warfarin at target INR 1 – 2
d. Low molecular weight heparin for 6 months
e. No anticoagulation required

‫اﻹﺟﺎﺑﺔ ﻋﲆ اﻟﺼﻔﺤﺔ اﻟﺘﺎﻟﻴﺔ‬

Page - 933
Internal Medicine - Hematology - Anticoagulants

Question 10/155

Question #10

A patient has mitral valve replacement with a prosthetic mechanical valve. Which of the following is
the most appropriate in his management?

a. Anticoagulation with warfarin at target INR 2 – 3


b. Anticoagulation with warfarin at target INR 2.5 – 3.5 √
c. Anticoagulation with warfarin at target INR 1 – 2
d. Low molecular weight heparin for 6 months
e. No anticoagulation required

Description

is patient carries a high risk of thrombosis and CVA; therefore, he needs anticoagulation (warfarin
is the first line anticoagulant of valvular heart disease)

e target INR in patients on warfarin:

Mitral valve replacement: 2.5 – 3.5 lifelong


Unprovoked DVT/PE: 2 – 3 for 6 months
Provoked DVT/PE: 2 – 3 for 3 months
Recurrent DVT/PE: 2 – 3 for life
Atrial fibrillation with high CHA2DS2VAS score: 2 – 3 lifelong
Aortic valve replacement: 2 – 3 lifelong

Page - 934
Internal Medicine - Hematology

Question 11/155

Question #11

A 74-year-old female patient is on warfarin because of a history of atrial fibrillation. She presents
with an INR level of 7, but she denies bleeding. What is the most appropriate management?

a. Hold warfarin for 2 doses


b. Provide 5 mg of oral vitamin K
c. Provide 3 mg IV vitamin K
d. Provide 5 mg IV vitamin K
e. Provide Fresh Frozen Plasma (FFP)

‫اﻹﺟﺎﺑﺔ ﻋﲆ اﻟﺼﻔﺤﺔ اﻟﺘﺎﻟﻴﺔ‬

Page - 935
Internal Medicine - Hematology - Anticoagulants

Question 11/155

Question #11

A 74-year-old female patient is on warfarin because of a history of atrial fibrillation. She presents
with an INR level of 7, but she denies bleeding. What is the most appropriate management?

a. Hold warfarin for 2 doses √


b. Provide 5 mg of oral vitamin K
c. Provide 3 mg IV vitamin K
d. Provide 5 mg IV vitamin K
e. Provide Fresh Frozen Plasma (FFP)

Description

A patient warfarin overdose and an INR level of 5 – 8 is not an indication of warfarin or FFP
administration.

e following points are the up-to-date protocol in the treatment of warfarin overdose.

Treatment of warfarin overdose if no bleeding:

INR 5 – 8:
Hold warfarin for 2 doses
Restart warfarin in lower dose when INR < 5
INR > 8:
Stop warfarin
Vitamin K 1 – 5 mg PO
Repeat Vitamin K dose if INR still > 8 for > 24 hours
Restart warfarin in lower dose when INR < 5

Treatment of warfarin overdose with minor bleeding:

Stop warfarin
Vitamin K 1 – 3 mg intravenously
Repeat Vitamin K dose if INR still > 8 for > 24 hours
Restart warfarin in lower dose when INR < 5

Treatment of warfarin overdose with major bleeding:


Page - 936
Stop warfarin
Vitamin K 5 mg intravenously
Provide Prothrombin complex concentrate
Provide FFP is Prothrombin complex concentrate is not available

Page - 937
Internal Medicine - Hematology

Question 12/155

Question #12

A pregnant 29-year-old female has been prescribed low molecular weight heparin for prophylaxis
against venous thrombosis. Which of the following is recommended for the monitoring of her
anticoagulation status?

a. International normalized ratio (INR)


b. Partial thromboplastin time (PTT)
c. Prothrombin time (PT)
d. Plasma factor Xa levels
e. No need for monitoring

‫اﻹﺟﺎﺑﺔ ﻋﲆ اﻟﺼﻔﺤﺔ اﻟﺘﺎﻟﻴﺔ‬

Page - 938
Internal Medicine - Hematology - Anticoagulants

Question 12/155

Question #12

A pregnant 29-year-old female has been prescribed low molecular weight heparin for prophylaxis
against venous thrombosis. Which of the following is recommended for the monitoring of her
anticoagulation status?

a. International normalized ratio (INR)


b. Partial thromboplastin time (PTT)
c. Prothrombin time (PT)
d. Plasma factor Xa levels
e. No need for monitoring √

Description

ere is no needed monitoring for LMWH because the anticoagulation eect in LMWH is
predictable.

e monitoring indicated for warfarin eect by INR

e monitoring is needed for unfractionated heparin by PTT

Page - 939
Internal Medicine - Hematology

Question 13/155

Question #13

A 66-year-old male patient is on warfarin to reduce the risk of stroke because he has atrial
fibrillation. However, in follow-up labs, he is found to have an INR level of 10. What is the most
appropriate treatment at this time if the patient has no bleeding?

a. Intravenous vitamin K
b. Oral vitamin K
c. Fresh Frozen plasma
d. Desmopressin (DDAVP)
e. Stop warfarin without additional treatment

‫اﻹﺟﺎﺑﺔ ﻋﲆ اﻟﺼﻔﺤﺔ اﻟﺘﺎﻟﻴﺔ‬

Page - 940
Internal Medicine - Hematology - Anticoagulants

Question 13/155

Question #13

A 66-year-old male patient is on warfarin to reduce the risk of stroke because he has atrial
fibrillation. However, in follow-up labs, he is found to have an INR level of 10. What is the most
appropriate treatment at this time if the patient has no bleeding?

a. Intravenous vitamin K
b. Oral vitamin K √
c. Fresh Frozen plasma
d. Desmopressin (DDAVP)
e. Stop warfarin without additional treatment

Description

is patient has an INR level of > 8 and no bleeding, so he needs to be treated with oral vitamin K.

e following points are the up-to-date protocol in the treatment of warfarin overdose.

Treatment of warfarin overdose if no bleeding:

INR 5 – 8:
Hold warfarin for 2 doses
Restart warfarin in lower dose when INR < 5
INR > 8:
Stop warfarin
Vitamin K 1 – 5 mg PO
Repeat Vitamin K dose if INR still > 8 for > 24 hours
Restart warfarin in lower dose when INR < 5

Treatment of warfarin overdose with minor bleeding:

Stop warfarin
Vitamin K 1 – 3 mg intravenously
Repeat Vitamin K dose if INR still > 8 for > 24 hours
Restart warfarin in lower dose when INR < 5

Treatment of warfarin overdose with major bleeding:


Page - 941
Stop warfarin
Vitamin K 5 mg intravenously
Provide Prothrombin complex concentrate
Provide FFP is Prothrombin complex concentrate is not available

Page - 942
Internal Medicine - Hematology

Question 14/155

Question #14

A previously healthy 19-year-old lady presents with bruises and petechial rashes. On examination,
she is pale, has a petechial rash, and has ecchymosis at her extensor surfaces. Lab investigations
show a hemoglobin level of 8 g/dL, WBC of 1.8*10^9/L, and platelet count of 22*10^9/L. Her blood
film shows a reduced platelet count, normocytic anemia, and absent reticulocytes. Her bone marrow
is hypocellular. What is the most likely diagnosis?

a. B12 deficiency anemia


b. Myelodysplastic syndrome
c. Aplastic anemia
d. Acute myeloid leukemia
e. Multiple myeloma

‫اﻹﺟﺎﺑﺔ ﻋﲆ اﻟﺼﻔﺤﺔ اﻟﺘﺎﻟﻴﺔ‬

Page - 943
Internal Medicine - Hematology - Aplastic Anemia (AA)

Question 14/155

Question #14

A previously healthy 19-year-old lady presents with bruises and petechial rashes. On examination,
she is pale, has a petechial rash, and has ecchymosis at her extensor surfaces. Lab investigations
show a hemoglobin level of 8 g/dL, WBC of 1.8*10^9/L, and platelet count of 22*10^9/L. Her blood
film shows a reduced platelet count, normocytic anemia, and absent reticulocytes. Her bone marrow
is hypocellular. What is the most likely diagnosis?

a. B12 deficiency anemia


b. Myelodysplastic syndrome
c. Aplastic anemia √
d. Acute myeloid leukemia
e. Multiple myeloma

Description

In aplastic anemia, the patient develops pancytopenia, reduced or absence of the reticulocytes and
hypocellular bone marrow

Note that MDs and leukemia will show increased blast cells in the bone marrow

In Multiple Myeloma (MM), the bone marrow will show plasma cells of > 10%

B12 deficiency anemia may present with pancytopenia, and megaloblasts are in the bone marrow
but not hypocellular.

Page - 944
Internal Medicine - Hematology

Question 15/155

Question #15

A 29-year-old female complains of fatigue and recurrent epistaxis. On examination, the patient is
pale and has a petechial rash on her extensor surfaces. e spleen and lymph nodes are not palpable.
Her CBC shows pancytopenia. A biopsy from which of the following tissues would be most helpful in
the diagnosis?

a. Lymph nodes
b. e liver
c. e bone marrow
d. Kidney
e. e spleen

‫اﻹﺟﺎﺑﺔ ﻋﲆ اﻟﺼﻔﺤﺔ اﻟﺘﺎﻟﻴﺔ‬

Page - 945
Internal Medicine - Hematology - Aplastic Anemia (AA)

Question 15/155

Question #15

A 29-year-old female complains of fatigue and recurrent epistaxis. On examination, the patient is
pale and has a petechial rash on her extensor surfaces. e spleen and lymph nodes are not palpable.
Her CBC shows pancytopenia. A biopsy from which of the following tissues would be most helpful in
the diagnosis?

a. Lymph nodes
b. e liver
c. e bone marrow √
d. Kidney
e. e spleen

Description

is lady is suspected of having aplastic anemia. You should perform a bone marrow biopsy to
confirm the diagnosis.

Hypocellular bone marrow, absent reticulocytes on the blood film, and pancytopenia in CBC are
classic presentations for Aplastic anemia.

Page - 946
Internal Medicine - Hematology

Question 16/155

Question #16

A 29-year-old male patient was admitted with a case of pneumonia and started to receive
antibiotics. His further investigations demonstrate hemoglobin of 7 g/dL, WBC of 1*10^9/L, platelet
count of 15*10^9/L, and reticulocytes of 0.4%. In addition, bone marrow examination shows reduced
hematopoietic stem cells. What is the most likely diagnosis?

a. Pernicious anemia
b. Acute leukemia
c. Chronic leukemia
d. Multiple Myeloma
e. Aplastic Anemia

‫اﻹﺟﺎﺑﺔ ﻋﲆ اﻟﺼﻔﺤﺔ اﻟﺘﺎﻟﻴﺔ‬

Page - 947
Internal Medicine - Hematology - Aplastic Anemia (AA)

Question 16/155

Question #16

A 29-year-old male patient was admitted with a case of pneumonia and started to receive
antibiotics. His further investigations demonstrate hemoglobin of 7 g/dL, WBC of 1*10^9/L, platelet
count of 15*10^9/L, and reticulocytes of 0.4%. In addition, bone marrow examination shows reduced
hematopoietic stem cells. What is the most likely diagnosis?

a. Pernicious anemia
b. Acute leukemia
c. Chronic leukemia
d. Multiple Myeloma
e. Aplastic Anemia √

Description

In aplastic anemia, the patient develops pancytopenia, reduced or absence of the reticulocytes and
hypocellular bone marrow

Note that MDs and leukemia will show increased blast cells in the bone marrow

In Multiple Myeloma (MM), the bone marrow will show plasma cells of > 10%

B12 deficiency anemia may present with pancytopenia, and megaloblasts are in the bone marrow
but not hypocellular.

Page - 948
Internal Medicine - Hematology

Question 17/155

Question #17

A 46-year-old male patient was diagnosed with autoimmune hemolytic anemia. All the following
are true except:

a. Cold autoimmune hemolytic anemia is associated with mycoplasma pneumonia


b. Cold AIHA is mediated by IgM antibodies
c. Warm AIHA is extravascular
d. Coombs test is positive
e. Cold AIHA is treated by steroids or splenectomy

‫اﻹﺟﺎﺑﺔ ﻋﲆ اﻟﺼﻔﺤﺔ اﻟﺘﺎﻟﻴﺔ‬

Page - 949
Internal Medicine - Hematology - Autoimmune hemolytic anemia (AIHA)

Question 17/155

Question #17

A 46-year-old male patient was diagnosed with autoimmune hemolytic anemia. All the following
are true except:

a. Cold autoimmune hemolytic anemia is associated with mycoplasma pneumonia


b. Cold AIHA is mediated by IgM antibodies
c. Warm AIHA is extravascular
d. Coombs test is positive
e. Cold AIHA is treated by steroids or splenectomy √

Description

Steroid and splenectomy do not have a role in cold Agglutinin Hemolytic anemia

Autoimmune hemolytic anemia (AIHA) is an RBC destruction by autoantibodies. It is divided into


‘warm’ and ‘cold’ types (according to which temperature the antibodies best cause hemolysis)

Both types of AIHA are characterized by a positive DAT (Coombs test).

Worm AIHA is IgG-mediated destruction of RBCs. It is extravascular and treated by steroids or


splenectomy.

Cold AIHA is IgM mediated, intravascular, and occurs at a temperature of 4 °C or less.

Page - 950
Internal Medicine - Hematology

Question 18/155

Question #18

A 22-year-old male patient is donating blood for his sister, who has severe antepartum hemorrhage.
While donating blood, the patient suddenly passed out. Which of the following is not appropriate in
this scenario?

a. Intravenous fluid replacement with continuing blood donation


b. Ensure adequate hydration of the patient
c. Elevate the donor’s legs, as this is most commonly a vasovagal attack
d. Ensure that the donor meets the minimum required hematocrit level for blood donation
e. Monitoring of the donor’s blood pressure.

‫اﻹﺟﺎﺑﺔ ﻋﲆ اﻟﺼﻔﺤﺔ اﻟﺘﺎﻟﻴﺔ‬

Page - 951
Internal Medicine - Hematology - Blood transfusion

Question 18/155

Question #18

A 22-year-old male patient is donating blood for his sister, who has severe antepartum hemorrhage.
While donating blood, the patient suddenly passed out. Which of the following is not appropriate in
this scenario?

a. Intravenous fluid replacement with continuing blood donation √


b. Ensure adequate hydration of the patient
c. Elevate the donor’s legs, as this is most commonly a vasovagal attack
d. Ensure that the donor meets the minimum required hematocrit level for blood donation
e. Monitoring of the donor’s blood pressure.

Description

You should never continue blood donation while the adverse eect is happening.

e vasovagal attack during blood donation is common; elevation of the patient’s legs will enhance
the blood perfusion in the brain (gravity eect)

Blood donors should be well hydrated and eat well before donation

People should not donate blood until you ensure that hemoglobin is above the minimum allowed
level.

Page - 952
Internal Medicine - Hematology

Question 19/155

Question #19

While receiving a blood transfusion, a patient has a fever and shivering. Which of the following is
the most appropriate action?

a. Stop blood transfusion and provide paracetamol


b. Stop blood transfusion and provide mannitol
c. Stop blood transfusion and start IV fluid
d. Continue blood transfusion and provide paracetamol
e. Continue blood transfusion and ignore the complaint

‫اﻹﺟﺎﺑﺔ ﻋﲆ اﻟﺼﻔﺤﺔ اﻟﺘﺎﻟﻴﺔ‬

Page - 953
Internal Medicine - Hematology - Blood transfusion

Question 19/155

Question #19

While receiving a blood transfusion, a patient has a fever and shivering. Which of the following is
the most appropriate action?

a. Stop blood transfusion and provide paracetamol


b. Stop blood transfusion and provide mannitol
c. Stop blood transfusion and start IV fluid
d. Continue blood transfusion and provide paracetamol √
e. Continue blood transfusion and ignore the complaint

Description

e non-hemolytic febrile reaction is a common complaint during blood transfusions.

It occurs due to the presence of WBCs in the transfused blood.

Blood transfusion can be continued, and paracetamol should provide to control fever.

If the patient has a history of febrile reaction in a previous blood transfusion, giving filtered blood
without WBCs will reduce the recurrence.

Page - 954
Internal Medicine - Hematology

Question 20/155

Question #20

A 22-year-old male patient has upper gastrointestinal bleeding and a rapid drop in hemoglobin. You
ordered to prepare and provide 4 units of blood. Which is the most likely test involved in preparing
the blood transfusion?

a. Indirect Coombs test


b. Direct Coombs test
c. Blood film
d. Osmotic fragility test
e. Glucose-6-phosphate dehydrogenase enzyme assay

‫اﻹﺟﺎﺑﺔ ﻋﲆ اﻟﺼﻔﺤﺔ اﻟﺘﺎﻟﻴﺔ‬

Page - 955
Internal Medicine - Hematology - Blood transfusion

Question 20/155

Question #20

A 22-year-old male patient has upper gastrointestinal bleeding and a rapid drop in hemoglobin. You
ordered to prepare and provide 4 units of blood. Which is the most likely test involved in preparing
the blood transfusion?

a. Indirect Coombs test √


b. Direct Coombs test
c. Blood film
d. Osmotic fragility test
e. Glucose-6-phosphate dehydrogenase enzyme assay

Description

An indirect Coombs test (also known as an indirect antiglobulin test) would test the patient’s serum
(not the RBCs).

Indirect Antiglobulin Test (IAT) detects antibodies in plasma

Stage 1: Plasma mixed with RBCs with known antigen

Stage 2: then mixed with antibodies to human globulin

If agglutination → Plasma contains antibodies against that RBC’s

Page - 956
Internal Medicine - Hematology

Question 21/155

Question #21

A 60-year-old female with aplastic anemia and recurrent blood transfusions had developed a febrile
reaction during her last blood transfusion. You ordered the subsequent transfusion through a filter
to prevent the reaction. Which mechanisms are most likely to explain the filter’s eect?

a. Reducing bacterial transmission


b. Reducing viral transmission
c. Reducing parasite transmission
d. Reducing leukocyte transfusion
e. Reducing reticulocyte transfusion

‫اﻹﺟﺎﺑﺔ ﻋﲆ اﻟﺼﻔﺤﺔ اﻟﺘﺎﻟﻴﺔ‬

Page - 957
Internal Medicine - Hematology - Blood transfusion

Question 21/155

Question #21

A 60-year-old female with aplastic anemia and recurrent blood transfusions had developed a febrile
reaction during her last blood transfusion. You ordered the subsequent transfusion through a filter
to prevent the reaction. Which mechanisms are most likely to explain the filter’s eect?

a. Reducing bacterial transmission


b. Reducing viral transmission
c. Reducing parasite transmission
d. Reducing leukocyte transfusion √
e. Reducing reticulocyte transfusion

Description

e non-hemolytic febrile reaction is a common complaint during a blood transfusion.

It occurs due to the presence of WBCs in the transfused blood.

Blood transfusion can be continued, and paracetamol should provide to control fever.

If the patient has a history of febrile reaction in a previous blood transfusion, giving filtered blood
without WBCs will reduce the recurrence.

Page - 958
Internal Medicine - Hematology

Question 22/155

Question #22

A 36-year-old female patient has recurrent deep vein thrombosis. Which of the following is the most
common abnormality?

a. Antiphospholipid antibody syndrome


b. Anti-thrombin III deficiency
c. Factor V Leiden mutation
d. Protein C deficiency
e. Protein S deficiency

‫اﻹﺟﺎﺑﺔ ﻋﲆ اﻟﺼﻔﺤﺔ اﻟﺘﺎﻟﻴﺔ‬

Page - 959
Internal Medicine - Hematology - Deep Vein rombosis (DVT)

Question 22/155

Question #22

A 36-year-old female patient has recurrent deep vein thrombosis. Which of the following is the most
common abnormality?

a. Antiphospholipid antibody syndrome


b. Anti-thrombin III deficiency
c. Factor V Leiden mutation √
d. Protein C deficiency
e. Protein S deficiency

Description

e mentioned choices are known to cause a hypercoagulable state and may result in DVT.

Factor V Leiden mutation is responsible for the most common hypercoagulable state

Additional notes:

Protein C, protein S, and antithrombin III are coagulation-inhibiting proteins


e levels of all three coagulation-inhibiting proteins can be reduced during the acute
thrombotic state.
Heparin suppresses the Antithrombin III
Warfarin suppresses protein C and protein S levels
Pregnancy and OCP use suppress protein S levels

Page - 960
Internal Medicine - Hematology

Question 23/155

Question #23

A 39-year-old male patient develops le lower limb swelling, hotness, redness, and pain. His clinical
scenario started aer returning from his vacation in Aqaba. His CBC, CRP, and ESR are normal. What
is the most likely diagnosis?

a. Lower limb cellulitis


b. Erysipelas
c. Deep vein thrombosis
d. Cauda equina syndrome
e. None of the above

‫اﻹﺟﺎﺑﺔ ﻋﲆ اﻟﺼﻔﺤﺔ اﻟﺘﺎﻟﻴﺔ‬

Page - 961
Internal Medicine - Hematology - Deep Vein rombosis (DVT)

Question 23/155

Question #23

A 39-year-old male patient develops le lower limb swelling, hotness, redness, and pain. His clinical
scenario started aer returning from his vacation in Aqaba. His CBC, CRP, and ESR are normal. What
is the most likely diagnosis?

a. Lower limb cellulitis


b. Erysipelas
c. Deep vein thrombosis √
d. Cauda equina syndrome
e. None of the above

Description

e history of travel and the presence of hotness, redness, tenderness, and swelling in the lower
limb strongly suggests DVT.

e presence of normal CRP, WBC, and ESR will exclude cellulitis

Erysipelas is an infection of the superficial skin layers; it also presents with elevated acute phase
reactants.

Cauda equina syndrome is a rare condition in which there is compression on the lower spinal roots

Page - 962
Internal Medicine - Hematology

Question 24/155

Question #24

A 66-year-old male patient has a 1-week right leg pain and swelling history. His records were
significant for a previous DVT two years ago, and he was treated with warfarin for 6 months. His D
dimer is positive, and his Doppler and duplex ultrasound demonstrate features of DVT again. Which
of the following is the most appropriate duration of anticoagulation therapy in his condition?

a. One month
b. ree months
c. Six months
d. 12 months
e. Indefinite

‫اﻹﺟﺎﺑﺔ ﻋﲆ اﻟﺼﻔﺤﺔ اﻟﺘﺎﻟﻴﺔ‬

Page - 963
Internal Medicine - Hematology - Deep Vein rombosis (DVT)

Question 24/155

Question #24

A 66-year-old male patient has a 1-week right leg pain and swelling history. His records were
significant for a previous DVT two years ago, and he was treated with warfarin for 6 months. His D
dimer is positive, and his Doppler and duplex ultrasound demonstrate features of DVT again. Which
of the following is the most appropriate duration of anticoagulation therapy in his condition?

a. One month
b. ree months
c. Six months
d. 12 months
e. Indefinite √

Description

Treatment of DVT:

DOAC (Apixaban or Rivaroxaban):


It should be started once the diagnosis is suspected
e first-line treatment for patients who do not have contraindications
In active malignancy patients, DOAC became preferred over LMWH.
Warfarin:
e second line treatment aer DOAC
Used in severe renal impairment (GFR < 15 mL/min).
Used as first-line if the patient has antiphospholipid syndrome.
LMWH:
Used as a bridge for warfarin (to prevent skin necrosis)
Used as monotherapy for 6 months in malignancy or pregnant patients (second line
aer DOAC)
Inferior vena cava filter indications:
Contraindications to anticoagulants
Recurrent emboli while on adequate anticoagulation
Right ventricular heart failure

Length of anticoagulation:
Page - 964
3 months: for provoked DVT/PE (known risk factor)
6 months: for unprovoked cases (unknown risk factor)
3 – 6 months: for people with active cancer
Lifelong: for recurrent DVT/PE

Page - 965
Internal Medicine - Hematology

Question 25/155

Question #25

A 66-year-old male patient recently arrived from his trip to America and started to have calf pain
and swelling. His D dimer is positive, and his Doppler ultrasound confirms the presence of popliteal
vein DVT. However, he denies shortness of breath or chest pain. What is the most appropriate
treatment?

a. Inpatient warfarin and unfractionated heparin


b. Inpatient leg elevation and warfarin
c. Inpatient warfarin and low molecular weight heparin
d. Outpatient compression stockings and Apixaban
e. Outpatient unfractionated heparin and warfarin

‫اﻹﺟﺎﺑﺔ ﻋﲆ اﻟﺼﻔﺤﺔ اﻟﺘﺎﻟﻴﺔ‬

Page - 966
Internal Medicine - Hematology - Deep Vein rombosis (DVT)

Question 25/155

Question #25

A 66-year-old male patient recently arrived from his trip to America and started to have calf pain
and swelling. His D dimer is positive, and his Doppler ultrasound confirms the presence of popliteal
vein DVT. However, he denies shortness of breath or chest pain. What is the most appropriate
treatment?

a. Inpatient warfarin and unfractionated heparin


b. Inpatient leg elevation and warfarin
c. Inpatient warfarin and low molecular weight heparin
d. Outpatient compression stockings and Apixaban √
e. Outpatient unfractionated heparin and warfarin

Description

is patient requires Apixaban therapy for 3 months.

Compression stockings reduce the development of hyperpigmentation secondary to DVT. However,


this patient doesn’t need admission.

Unfractionated heparin is not suitable for outpatient use.

e indications for admission to DVT are:

Massive DVT (e.g., iliofemoral DVT)


Presence of pulmonary embolism
High risk of bleeding with anticoagulation
Comorbid conditions requiring admission

Treatment of DVT:

DOAC (Apixaban or Rivaroxaban):


It should be started once the diagnosis is suspected
e first-line treatment for patients who do not have contraindications
In active malignancy patients, DOAC became preferred over LMWH.
Warfarin:
e second line treatment aer DOAC
Page - 967
Used in severe renal impairment (GFR < 15 mL/min).
Used as first-line if the patient has antiphospholipid syndrome.
LMWH:
Used as a bridge for warfarin (to prevent skin necrosis)
Used as monotherapy for 6 months in malignancy or pregnant patients (second line
aer DOAC)
Inferior vena cava filter indications:
Contraindications to anticoagulants
Recurrent emboli while on adequate anticoagulation
Right ventricular heart failure

Length of anticoagulation:

3 months: for provoked DVT/PE (known risk factor)


6 months: for unprovoked cases (unknown risk factor)
3 – 6 months: for people with active cancer
Lifelong: for recurrent DVT/PE

Page - 968
Internal Medicine - Hematology

Question 26/155

Question #26

A previously healthy 66-year-old female is started on warfarin for deep vein thrombosis acquired
aer a lower limb fracture. Which of the following is the recommended target INR in this condition?

a. 2 – 3 for 3 months
b. 2 – 3 for 6 months
c. 2.5 – 3.5 for 6 months
d. 3 – 4 for 3 months
e. No need for monitoring

‫اﻹﺟﺎﺑﺔ ﻋﲆ اﻟﺼﻔﺤﺔ اﻟﺘﺎﻟﻴﺔ‬

Page - 969
Internal Medicine - Hematology - Deep Vein rombosis (DVT)

Question 26/155

Question #26

A previously healthy 66-year-old female is started on warfarin for deep vein thrombosis acquired
aer a lower limb fracture. Which of the following is the recommended target INR in this condition?

a. 2 – 3 for 3 months √
b. 2 – 3 for 6 months
c. 2.5 – 3.5 for 6 months
d. 3 – 4 for 3 months
e. No need for monitoring

Description

Patients who receive warfarin for DVT should have their target INR of 2 – 3 for 3 months in provoked
DVT and 6 months for unprovoked DVT.

Treatment of DVT:

DOAC (Apixaban or Rivaroxaban):


It should be started once the diagnosis is suspected
e first-line treatment for patients who do not have contraindications
In active malignancy patients, DOAC became preferred over LMWH.
Warfarin:
e second line treatment aer DOAC
Used in severe renal impairment (GFR < 15 mL/min).
Used as first-line if the patient has antiphospholipid syndrome.
LMWH:
Used as a bridge for warfarin (to prevent skin necrosis)
Used as monotherapy for 6 months in malignancy or pregnant patients (second line
aer DOAC)
Inferior vena cava filter indications:
Contraindications to anticoagulants
Recurrent emboli while on adequate anticoagulation
Right ventricular heart failure

Length of anticoagulation:
Page - 970
3 months: for provoked DVT/PE (known risk factor)
6 months: for unprovoked cases (unknown risk factor)
3 – 6 months: for people with active cancer
Lifelong: for recurrent DVT/PE

Page - 971
Internal Medicine - Hematology

Question 27/155

Question #27

A 55-year-old female is started on warfarin for deep vein thrombosis. Two days later, she developed
a skin condition shown in the picture below. Which of the following is the most likely explanation?

a. Warfarin side eect


b. Embolization DVT
c. e patient is already hypercoagulable
d. Infection
e. e patient has sickle cell disease

‫اﻹﺟﺎﺑﺔ ﻋﲆ اﻟﺼﻔﺤﺔ اﻟﺘﺎﻟﻴﺔ‬

Page - 972
Internal Medicine - Hematology - Deep Vein rombosis (DVT)

Question 27/155

Question #27

A 55-year-old female is started on warfarin for deep vein thrombosis. Two days later, she developed
a skin condition shown in the picture below. Which of the following is the most likely explanation?

a. Warfarin side eect √


b. Embolization DVT
c. e patient is already hypercoagulable
d. Infection
e. e patient has sickle cell disease

Description

Warfarin-induced skin necrosis is rare due to acquired protein C deficiency following treatment with
anti-vitamin K anticoagulants.

is condition is treated by Fresh Frozen Plasma (FFP), pure activated protein C, and surgical
management of the necrotic area may be needed.

Page - 973
Internal Medicine - Hematology

Question 28/155

Question #28

A 66-year-old female is started on Rivaroxaban for deep vein thrombosis acquired aer a lower limb
fracture. Which of the following is the recommended target INR in this condition?

a. 2 – 3 for 3 months
b. 2 – 3 for 6 months
c. 2.5 – 3.5 for 6 months
d. 3 – 4 for 3 months
e. No need for monitoring

‫اﻹﺟﺎﺑﺔ ﻋﲆ اﻟﺼﻔﺤﺔ اﻟﺘﺎﻟﻴﺔ‬

Page - 974
Internal Medicine - Hematology - Deep Vein rombosis (DVT)

Question 28/155

Question #28

A 66-year-old female is started on Rivaroxaban for deep vein thrombosis acquired aer a lower limb
fracture. Which of the following is the recommended target INR in this condition?

a. 2 – 3 for 3 months
b. 2 – 3 for 6 months
c. 2.5 – 3.5 for 6 months
d. 3 – 4 for 3 months
e. No need for monitoring √

Description

DOACs have become the first-line treatment of DVT over warfarin.

Compared to warfarin, DOACs carry a low risk of bleeding and do not need any monitoring.

Treatment of DVT:

DOAC (Apixaban or Rivaroxaban):


It should be started once the diagnosis is suspected
e first-line treatment for patients who do not have contraindications
In active malignancy patients, DOAC became preferred over LMWH.
Warfarin:
e second line treatment aer DOAC
Used in severe renal impairment (GFR < 15 mL/min).
Used as first-line if the patient has antiphospholipid syndrome.
LMWH:
Used as a bridge for warfarin (to prevent skin necrosis)
Used as monotherapy for 6 months in malignancy or pregnant patients (second line
aer DOAC)
Inferior vena cava filter indications:
Contraindications to anticoagulants
Recurrent emboli while on adequate anticoagulation
Right ventricular heart failure

Page - 975
Length of anticoagulation:

3 months: for provoked DVT/PE (known risk factor)


6 months: for unprovoked cases (unknown risk factor)
3 – 6 months: for people with active cancer
Lifelong: for recurrent DVT/PE

Page - 976
Internal Medicine - Hematology

Question 29/155

Question #29

Which of the following isn’t considered a risk factor for venous thrombosis?

a. Combined oral contraceptive use


b. Lower limb fracture
c. Prostate cancer
d. Anemia
e. Traveling

‫اﻹﺟﺎﺑﺔ ﻋﲆ اﻟﺼﻔﺤﺔ اﻟﺘﺎﻟﻴﺔ‬

Page - 977
Internal Medicine - Hematology - Deep Vein rombosis (DVT)

Question 29/155

Question #29

Which of the following isn’t considered a risk factor for venous thrombosis?

a. Combined oral contraceptive use


b. Lower limb fracture
c. Prostate cancer
d. Anemia √
e. Traveling

Description

Virchow’s Triad is critically important in the development of venous thromboembolism.

Venous stasis: (major surgery, prolonged travel, pregnancy, lower limb injury)
Endothelial damage (i.e., trauma)
Hypercoagulability state:
History of DVT/PE
Hormone replacement therapy (e.g., estrogen)
Malignancy (of any kind)
Nephrotic syndrome (due to antithrombin III loss)
Coagulopathies (e.g., factor V Leiden)

Page - 978
Internal Medicine - Hematology

Question 30/155

Question #30

A 35-year-old male patient with a history of spinal cord injury 3 weeks ago started to have right calf
swelling, pain, and tenderness. e diameter of his right calf is higher than the le one, and passive
movements cause pain. What is the most likely diagnosis?

a. Cellulitis
b. Deep vein thrombosis
c. Peripheral arterial disease
d. Superficial thrombophlebitis
e. Lower limb edema

‫اﻹﺟﺎﺑﺔ ﻋﲆ اﻟﺼﻔﺤﺔ اﻟﺘﺎﻟﻴﺔ‬

Page - 979
Internal Medicine - Hematology - Deep Vein rombosis (DVT)

Question 30/155

Question #30

A 35-year-old male patient with a history of spinal cord injury 3 weeks ago started to have right calf
swelling, pain, and tenderness. e diameter of his right calf is higher than the le one, and passive
movements cause pain. What is the most likely diagnosis?

a. Cellulitis
b. Deep vein thrombosis √
c. Peripheral arterial disease
d. Superficial thrombophlebitis
e. Lower limb edema

Description

Any element of Virchow’s Triad and leg swelling strongly suggests DVT.

is patient has venous stasis secondary to spinal cord injury and immobility; therefore, he is
hypercoagulable.

e calf having a larger diameter than the other is a known sign of a DVT.

Virchow’s Triad (Risk factors for DVT/PE):

Venous stasis: (major surgery, prolonged travel, pregnancy, lower limb injury)
Endothelial damage (i.e., trauma)
Hypercoagulability state:
History of DVT/PE
Hormone replacement therapy (e.g., estrogen)
Malignancy (of any kind)
Nephrotic syndrome (due to antithrombin III loss)
Coagulopathies (e.g., factor V Leiden)

Page - 980
Internal Medicine - Hematology

Question 31/155

Question #31

A 39-year-old male patient develops swollen le lower limb swelling, hotness, redness, and pain. His
clinical scenario started aer returning from his vacation in Aqaba. His CBC, CRP, and ESR are
normal. However, Doppler ultrasound confirms incompressibility and echogenic material in the
femoral vein. What is the most appropriate management?

a. Elastic compression stockings and repeat ultrasound in 48 hours


b. Heparin bridge to and warfarin for 3 months
c. Insertion of inferior vena cava filter
d. Aspirin for 6 months duration
e. Start Apixaban

‫اﻹﺟﺎﺑﺔ ﻋﲆ اﻟﺼﻔﺤﺔ اﻟﺘﺎﻟﻴﺔ‬

Page - 981
Internal Medicine - Hematology - Deep Vein rombosis (DVT)

Question 31/155

Question #31

A 39-year-old male patient develops swollen le lower limb swelling, hotness, redness, and pain. His
clinical scenario started aer returning from his vacation in Aqaba. His CBC, CRP, and ESR are
normal. However, Doppler ultrasound confirms incompressibility and echogenic material in the
femoral vein. What is the most appropriate management?

a. Elastic compression stockings and repeat ultrasound in 48 hours


b. Heparin bridge to and warfarin for 3 months
c. Insertion of inferior vena cava filter
d. Aspirin for 6 months duration
e. Start Apixaban √

Description

Warfarin is no longer the first-line treatment in this case; Direct oral anticoagulants should be
started whenever the diagnosis is suspected.

Treatment of DVT:

DOAC (Apixaban or Rivaroxaban):


It should be started once the diagnosis is suspected
e first-line treatment for patients who do not have contraindications
In active malignancy patients, DOAC became preferred over LMWH.
Warfarin:
e second line treatment aer DOAC
Used in severe renal impairment (GFR < 15 mL/min).
Used as first-line if the patient has antiphospholipid syndrome.
LMWH:
Used as a bridge for warfarin (to prevent skin necrosis)
Used as monotherapy for 6 months in malignancy or pregnant patients (second line
aer DOAC)
Inferior vena cava filter indications:
Contraindications to anticoagulants
Recurrent emboli while on adequate anticoagulation
Page - 982
Right ventricular heart failure

Length of anticoagulation:

3 months: for provoked DVT/PE (known risk factor)


6 months: for unprovoked cases (unknown risk factor)
3 – 6 months: for people with active cancer
Lifelong: for recurrent DVT/PE

Page - 983
Internal Medicine - Hematology

Question 32/155

Question #32

A 66-year-old male patient was admitted to the ICU as a case of interventricular hemorrhage. On
day 8 aer admission, he develops right lower limb swelling and pain. However, his imaging studies
confirm the presence of iliofemoral DVT. What is the most appropriate at this time?

a. Start LMWH and warfarin


b. Start Apixaban alone
c. Start unfractionated heparin and warfarin
d. Insert an inferior vena cava filter
e. Leg elevation and elastic compression stockings

‫اﻹﺟﺎﺑﺔ ﻋﲆ اﻟﺼﻔﺤﺔ اﻟﺘﺎﻟﻴﺔ‬

Page - 984
Internal Medicine - Hematology - Deep Vein rombosis (DVT)

Question 32/155

Question #32

A 66-year-old male patient was admitted to the ICU as a case of interventricular hemorrhage. On
day 8 aer admission, he develops right lower limb swelling and pain. However, his imaging studies
confirm the presence of iliofemoral DVT. What is the most appropriate at this time?

a. Start LMWH and warfarin


b. Start Apixaban alone
c. Start unfractionated heparin and warfarin
d. Insert an inferior vena cava filter √
e. Leg elevation and elastic compression stockings

Description

is patient suers from massive DVT, but he is contraindicated to have anticoagulation because of
a brain hemorrhage.

An inferior vena cava filter in his condition will prevent the development of pulmonary embolism.

Leg elevation and elastic compression stockings alone will not reduce the risk of PE in this patient.

Page - 985
Internal Medicine - Hematology

Question 33/155

Question #33

A pregnant 32-year-old lady develops right leg DVT. Which of the following is the most appropriate
treatment?

a. Low molecular weight heparin and warfarin for 6 months


b. Low molecular weight heparin and warfarin for 3 months
c. Warfarin alone
d. Inferior vena cava filter insertion
e. Low molecular weight heparin for 6 months

‫اﻹﺟﺎﺑﺔ ﻋﲆ اﻟﺼﻔﺤﺔ اﻟﺘﺎﻟﻴﺔ‬

Page - 986
Internal Medicine - Hematology - Deep Vein rombosis (DVT)

Question 33/155

Question #33

A pregnant 32-year-old lady develops right leg DVT. Which of the following is the most appropriate
treatment?

a. Low molecular weight heparin and warfarin for 6 months


b. Low molecular weight heparin and warfarin for 3 months
c. Warfarin alone
d. Inferior vena cava filter insertion
e. Low molecular weight heparin for 6 months √

Description

Pregnant ladies with DVT should be treated with low molecular weight heparin alone for 6 months.

Warfarin can cross the placenta; it is contraindicated in pregnancy.

Page - 987
Internal Medicine - Hematology

Question 34/155

Question #34

A 66-year-old male patient with prostatic cancer developed swelling and pain in his le leg. His
Doppler ultrasound demonstrates monophasic blood flow, incompressibility, and echogenic
material in the femoral vein. What is the most appropriate treatment at this time?

a. Low molecular weight heparin and warfarin for 6 months


b. Warfarin alone
c. Inferior vena cava filter insertion
d. Low molecular weight heparin for 6 months
e. Apixaban for 6 months

‫اﻹﺟﺎﺑﺔ ﻋﲆ اﻟﺼﻔﺤﺔ اﻟﺘﺎﻟﻴﺔ‬

Page - 988
Internal Medicine - Hematology - Deep Vein rombosis (DVT)

Question 34/155

Question #34

A 66-year-old male patient with prostatic cancer developed swelling and pain in his le leg. His
Doppler ultrasound demonstrates monophasic blood flow, incompressibility, and echogenic
material in the femoral vein. What is the most appropriate treatment at this time?

a. Low molecular weight heparin and warfarin for 6 months


b. Warfarin alone
c. Inferior vena cava filter insertion
d. Low molecular weight heparin for 6 months
e. Apixaban for 6 months √

Description

In the past, LMWH for 6 months was the first line treatment in active malignancy patients, but
today DOAC became preferred over LMWH.

Treatment of DVT:

DOAC (Apixaban or Rivaroxaban):


It should be started once the diagnosis is suspected
e first-line treatment for patients who do not have contraindications
In active malignancy patients, DOAC became preferred over LMWH.
Warfarin:
e second line treatment aer DOAC
Used in severe renal impairment (GFR < 15 mL/min).
Used as first-line if the patient has antiphospholipid syndrome.
LMWH:
Used as a bridge for warfarin (to prevent skin necrosis)
Used as monotherapy for 6 months in malignancy or pregnant patients (second line
aer DOAC)
Inferior vena cava filter indications:
Contraindications to anticoagulants
Recurrent emboli while on adequate anticoagulation
Right ventricular heart failure
Page - 989
Length of anticoagulation:

3 months: for provoked DVT/PE (known risk factor)


6 months: for unprovoked cases (unknown risk factor)
3 – 6 months: for people with active cancer
Lifelong: for recurrent DVT/PE

Page - 990
Internal Medicine - Hematology

Question 35/155

Question #35

An 87-year-old male patient is planning to travel on a long airline flight. Which of the following
should be considered in this situation?

a. Risk of hypoxia during flight


b. Risk of dehydration
c. Risk of thrombosis
d. Alcohol withdrawal symptoms
e. Risk of hyperglycemia

‫اﻹﺟﺎﺑﺔ ﻋﲆ اﻟﺼﻔﺤﺔ اﻟﺘﺎﻟﻴﺔ‬

Page - 991
Internal Medicine - Hematology - Deep Vein rombosis (DVT)

Question 35/155

Question #35

An 87-year-old male patient is planning to travel on a long airline flight. Which of the following
should be considered in this situation?

a. Risk of hypoxia during flight


b. Risk of dehydration
c. Risk of thrombosis √
d. Alcohol withdrawal symptoms
e. Risk of hyperglycemia

Description

DVT associated with long airline flights is a real concern in this situation.

Prophylactic use of ambulation and exercises during long flights is essential in high-risk patients to
prevent DVT and PE.

Venous stasis during long travels is a part of Virchow’s Triad; it increases the risk of DVT and PE.

Page - 992
Internal Medicine - Hematology

Question 36/155

Question #36

A 29-year-old female patient complains of a 1-month duration of rash-like skin bruises and gum
bleeding aer brushing her teeth. e following investigations are appropriate at this time except:

a. Bleeding time (BT)


b. Complete blood count
c. Kidney function test
d. PT, PTT, and INR
e. D Dimer

‫اﻹﺟﺎﺑﺔ ﻋﲆ اﻟﺼﻔﺤﺔ اﻟﺘﺎﻟﻴﺔ‬

Page - 993
Internal Medicine - Hematology - Disorders of hemostasis

Question 36/155

Question #36

A 29-year-old female patient complains of a 1-month duration of rash-like skin bruises and gum
bleeding aer brushing her teeth. e following investigations are appropriate at this time except:

a. Bleeding time (BT)


b. Complete blood count
c. Kidney function test
d. PT, PTT, and INR
e. D Dimer √

Description

is patient suers from a bleeding disorder due to an issue in her hemostasis. Bleeding time will
provide information about the primary hemostasis, while PT, PTT, And INR will be informative about
the secondary hemostasis. In addition, a kidney function test may be beneficial to exclude uremia-
induced platelet dysfunction.

D Dimer is a fibrinogen degradation product (FDP) that is elevated in the case of thrombosis, not a
bleeding tendency.

Page - 994
Internal Medicine - Hematology

Question 37/155

Question #37

Which of the following drugs causes irreversible platelet dysfunction?

a. Aspirin
b. Ibuprofen
c. Warfarin
d. Heparin
e. Dipyridamole

‫اﻹﺟﺎﺑﺔ ﻋﲆ اﻟﺼﻔﺤﺔ اﻟﺘﺎﻟﻴﺔ‬

Page - 995
Internal Medicine - Hematology - Disorders of hemostasis

Question 37/155

Question #37

Which of the following drugs causes irreversible platelet dysfunction?

a. Aspirin √
b. Ibuprofen
c. Warfarin
d. Heparin
e. Dipyridamole

Description

Several drugs inhibit the platelet function, but aspirin is the only one that inhibits the platelet
function for the life of the platelet.

Aspirin (Acetylsalicylic acid) is a non-selective and irreversible inhibitor of the cyclooxygenase


enzyme. is will inhibit the formation of prostaglandin, which causes pain. In addition, the
inhibition of prostaglandin will prevent the conversion of arachidonic acid to thromboxane A2, a
potent inducer of platelet aggregation.

Page - 996
Internal Medicine - Hematology

Question 38/155

Question #38

A 38-year-old male patient developed sepsis and was admitted to the ICU for intravenous
management. On day 2 aer admission, he developed a rash, coee ground vomiting, elevated PT,
PTT, INR, D Dimer, and low platelet count. What is the most likely diagnosis?

a. Acute hepatic failure


b. Peptic ulcer disease
c. Disseminated intravascular coagulation
d. Aplastic anemia
e. rombotic thrombocytopenic purpura

‫اﻹﺟﺎﺑﺔ ﻋﲆ اﻟﺼﻔﺤﺔ اﻟﺘﺎﻟﻴﺔ‬

Page - 997
Internal Medicine - Hematology - Disseminated intravascular coagulation (DIC)

Question 38/155

Question #38

A 38-year-old male patient developed sepsis and was admitted to the ICU for intravenous
management. On day 2 aer admission, he developed a rash, coee ground vomiting, elevated PT,
PTT, INR, D Dimer, and low platelet count. What is the most likely diagnosis?

a. Acute hepatic failure


b. Peptic ulcer disease
c. Disseminated intravascular coagulation √
d. Aplastic anemia
e. rombotic thrombocytopenic purpura

Description

is patient is suering from DIC. e risk factor here is sepsis, and the lab investigations are typical
of DIC

Risk factors:

Sepsis, malignancy, drug toxicity


Burns, snake bites
Cancer
Obstetric problems (placental abruption, missed abortion, amniotic fluid embolism)

Pathogenesis:

Tissue factor released from destructed tissues causes widespread activation of coagulation →
consumption of coagulation factors and platelets → thrombocytopenia, and depletion of plasma
coagulation factors and fibrinogen, which cause bleeding.

Bleeding into organs and microvascular thromboses may cause dysfunction and failure in multiple
organs. In addition, delayed dissolution of fibrin polymers by fibrinolysis may result in the
mechanical disruption of RBCs, producing mild intravascular hemolysis.

Page - 998
Internal Medicine - Hematology

Question 39/155

Question #39

A 40-year-old female patient develops massive bleeding from a venipuncture site. Her lab
investigations show hemoglobin of 10 g/dL, WBC of 10*10^9, platelet count of 35 *10^9/L, prolonged
PT, PTT, and BT. In addition, her fibrin level is reduced, and the fibrinogen degradation products are
elevated. What is the most likely diagnosis?

a. Acute hepatic failure


b. Peptic ulcer disease
c. Disseminated intravascular coagulation
d. Aplastic anemia
e. rombotic thrombocytopenic purpura

‫اﻹﺟﺎﺑﺔ ﻋﲆ اﻟﺼﻔﺤﺔ اﻟﺘﺎﻟﻴﺔ‬

Page - 999
Internal Medicine - Hematology - Disseminated intravascular coagulation (DIC)

Question 39/155

Question #39

A 40-year-old female patient develops massive bleeding from a venipuncture site. Her lab
investigations show hemoglobin of 10 g/dL, WBC of 10*10^9, platelet count of 35 *10^9/L, prolonged
PT, PTT, and BT. In addition, her fibrin level is reduced, and the fibrinogen degradation products are
elevated. What is the most likely diagnosis?

a. Acute hepatic failure


b. Peptic ulcer disease
c. Disseminated intravascular coagulation √
d. Aplastic anemia
e. rombotic thrombocytopenic purpura

Description

e patient’s presentation and lab investigations are typical of DIC

Risk factors:

Sepsis, malignancy, drug toxicity


Burns, snake bites
Cancer
Obstetric problems (placental abruption, missed abortion, amniotic fluid embolism)

Pathogenesis:

Tissue factor released from destructed tissues causes widespread activation of coagulation →
consumption of coagulation factors and platelets → thrombocytopenia, and depletion of plasma
coagulation factors and fibrinogen, which cause bleeding.

Bleeding into organs and microvascular thromboses may cause dysfunction and failure in multiple
organs. In addition, delayed dissolution of fibrin polymers by fibrinolysis may result in the
mechanical disruption of RBCs, producing mild intravascular hemolysis.

Page - 1000
Internal Medicine - Hematology

Question 40/155

Question #40

A 38-year-old pregnant woman develops missed abortion. She was admitted to the emergency unit
with purpura over her legs and arms. Her lab investigations show hemoglobin of 10g/dL, WBC of
16*10^9/L, platelet count of 31*10^9/L, Prolonged PT, PTT, and positive D Dimer. What is the most
likely diagnosis?

a. Septic abortion and septicemia


b. Pulmonary embolism
c. Disseminated intravascular coagulation
d. Factor V Leiden mutation
e. rombotic thrombocytopenic purpura

‫اﻹﺟﺎﺑﺔ ﻋﲆ اﻟﺼﻔﺤﺔ اﻟﺘﺎﻟﻴﺔ‬

Page - 1001
Internal Medicine - Hematology - Disseminated intravascular coagulation (DIC)

Question 40/155

Question #40

A 38-year-old pregnant woman develops missed abortion. She was admitted to the emergency unit
with purpura over her legs and arms. Her lab investigations show hemoglobin of 10g/dL, WBC of
16*10^9/L, platelet count of 31*10^9/L, Prolonged PT, PTT, and positive D Dimer. What is the most
likely diagnosis?

a. Septic abortion and septicemia


b. Pulmonary embolism
c. Disseminated intravascular coagulation √
d. Factor V Leiden mutation
e. rombotic thrombocytopenic purpura

Description

is patient is suering from DIC. e risk factor here is missed abortion, and the lab investigations
are typical of DIC

Risk factors:

Sepsis, malignancy, drug toxicity


Burns, snake bites
Cancer
Obstetric problems (placental abruption, missed abortion, amniotic fluid embolism)

Pathogenesis:

Tissue factor released from destructed tissues causes widespread activation of coagulation →
consumption of coagulation factors and platelets → thrombocytopenia, and depletion of plasma
coagulation factors and fibrinogen, which cause bleeding.

Bleeding into organs and microvascular thromboses may cause dysfunction and failure in multiple
organs. In addition, delayed dissolution of fibrin polymers by fibrinolysis may result in the
mechanical disruption of RBCs, producing mild intravascular hemolysis.

Page - 1002
Internal Medicine - Hematology

Question 41/155

Question #41

Erythromelalgia is painful erythema on the hand that is relieved by aspirin. It is most likely found in:

a. Acute leukemia
b. Polycythemia Vera
c. Myelofibrosis
d. Essential thrombocythemia (ET)
e. Myelodysplastic syndrome

‫اﻹﺟﺎﺑﺔ ﻋﲆ اﻟﺼﻔﺤﺔ اﻟﺘﺎﻟﻴﺔ‬

Page - 1003
Internal Medicine - Hematology - Essential rombocythemia (ET)

Question 41/155

Question #41

Erythromelalgia is painful erythema on the hand that is relieved by aspirin. It is most likely found in:

a. Acute leukemia
b. Polycythemia Vera
c. Myelofibrosis
d. Essential thrombocythemia (ET) √
e. Myelodysplastic syndrome

Description

ET is a myeloproliferative disease characterized by the proliferation of megakaryocytes in the bone


marrow, leading to increased platelet production. It is more common in females and patients aged
50 – 60 years.

Erythromelalgia (painful erythema on the hand that is relieved by aspirin)

Page - 1004
Internal Medicine - Hematology

Question 42/155

Question #42

A 49-year-old male patient was admitted to the CCU with acute non-ST elevation myocardial
infarction. On hospitalization day 7, he complained of le arm pain, paresthesia, and absent pulse.
His lab investigations show a platelet count of 30*10^9/L. What is the most likely diagnosis?

a. Heparin-induced thrombocytopenic purpura type 1


b. Heparin-induced thrombocytopenic purpura type 2
c. Brachial artery vasospasm
d. Hypercoagulable state due to immobilization
e. Embolization from infective endocarditis

‫اﻹﺟﺎﺑﺔ ﻋﲆ اﻟﺼﻔﺤﺔ اﻟﺘﺎﻟﻴﺔ‬

Page - 1005
Internal Medicine - Hematology - HIT syndrome

Question 42/155

Question #42

A 49-year-old male patient was admitted to the CCU with acute non-ST elevation myocardial
infarction. On hospitalization day 7, he complained of le arm pain, paresthesia, and absent pulse.
His lab investigations show a platelet count of 30*10^9/L. What is the most likely diagnosis?

a. Heparin-induced thrombocytopenic purpura type 1


b. Heparin-induced thrombocytopenic purpura type 2 √
c. Brachial artery vasospasm
d. Hypercoagulable state due to immobilization
e. Embolization from infective endocarditis

Description

e patient received heparin to treat NSTEMI and developed low platelet and thrombosis (this is
typical for HIT syndrome)

Heparin-induced thrombocytopenia (HIT) syndrome is the platelet count drop≥ 50% aer heparin
therapy.

e following table shows the dierence between type 1 and type 2 HIT syndrome.

Page - 1006
Internal Medicine - Hematology

Question 43/155

Question #43

A 66-year-old female patient undergoes elective knee replacement surgery and has received
prophylactic enoxaparin. As a result, 1 week later, she developed a pulmonary embolism. However,
her platelet count has dropped from 340 to 150*10^9/L in one week. What is the most appropriate at
this point?

a. Keep on the same dose of prophylactic enoxaparin


b. Increase heparin to a therapeutic dose and add warfarin
c. Stop heparin and start warfarin
d. Stop enoxaparin and start intravenous heparin
e. Stop enoxaparin and start argatroban

‫اﻹﺟﺎﺑﺔ ﻋﲆ اﻟﺼﻔﺤﺔ اﻟﺘﺎﻟﻴﺔ‬

Page - 1007
Internal Medicine - Hematology - HIT syndrome

Question 43/155

Question #43

A 66-year-old female patient undergoes elective knee replacement surgery and has received
prophylactic enoxaparin. As a result, 1 week later, she developed a pulmonary embolism. However,
her platelet count has dropped from 340 to 150*10^9/L in one week. What is the most appropriate at
this point?

a. Keep on the same dose of prophylactic enoxaparin


b. Increase heparin to a therapeutic dose and add warfarin
c. Stop heparin and start warfarin
d. Stop enoxaparin and start intravenous heparin
e. Stop enoxaparin and start argatroban √

Description

is is a classic scenario of Heparin-induced thrombocytopenia (HIT) syndrome.

A drop of platelet count ≥ 50% aer starting heparin therapy


is condition is not dose-dependent (can occur aer administration of any amount of
heparin)
Both types of heparins can cause HIT syndrome, but in LMWH, it is less likely

Treatment of HIT syndrome:

Stop heparin and use direct thrombin inhibitor (such as argatroban or lepirudin)
Because of 90% cross-reactivity, LMWH should not be substituted

Page - 1008
Internal Medicine - Hematology

Question 44/155

Question #44

A 30-year-old female was started on low molecular weight heparin (LMWH) and warfarin to treat
deep venous thrombosis. On day 6 aer admission, she develops a drop in her platelet count of more
than 50% of the baseline. What is the most appropriate action at this time?

a. Stop LMWH and continue warfarin


b. Stop warfarin and continue LMWH
c. Switch to unfractionated heparin
d. Stop heparin and start argatroban
e. Stop anticoagulation and start aspirin

‫اﻹﺟﺎﺑﺔ ﻋﲆ اﻟﺼﻔﺤﺔ اﻟﺘﺎﻟﻴﺔ‬

Page - 1009
Internal Medicine - Hematology - HIT syndrome

Question 44/155

Question #44

A 30-year-old female was started on low molecular weight heparin (LMWH) and warfarin to treat
deep venous thrombosis. On day 6 aer admission, she develops a drop in her platelet count of more
than 50% of the baseline. What is the most appropriate action at this time?

a. Stop LMWH and continue warfarin


b. Stop warfarin and continue LMWH
c. Switch to unfractionated heparin
d. Stop heparin and start argatroban √
e. Stop anticoagulation and start aspirin

Description

is is a classic scenario of Heparin-induced thrombocytopenia (HIT) syndrome.

A drop of platelet count ≥ 50% aer starting heparin therapy


is condition is not dose-dependent (can occur aer administration of any amount of
heparin)
Both types of heparins can cause HIT syndrome, but in LMWH, it is less likely

Treatment of HIT syndrome:

Stop heparin and use direct thrombin inhibitor (such as argatroban or lepirudin)
Because of 90% cross-reactivity, LMWH should not be substituted

Page - 1010
Internal Medicine - Hematology

Question 45/155

Question #45

A 62-year-old male patient develops pallor and fatigue. What is the best initial test to perform at
this time?

a. Full blood count


b. PT, PTT, and INR
c. HB electrophoresis
d. Serum ferritin level
e. Bone marrow examination

‫اﻹﺟﺎﺑﺔ ﻋﲆ اﻟﺼﻔﺤﺔ اﻟﺘﺎﻟﻴﺔ‬

Page - 1011
Internal Medicine - Hematology - Hematologic investigations

Question 45/155

Question #45

A 62-year-old male patient develops pallor and fatigue. What is the best initial test to perform at
this time?

a. Full blood count √


b. PT, PTT, and INR
c. HB electrophoresis
d. Serum ferritin level
e. Bone marrow examination

Description

e first logical diagnosis in the presence of fatigue and pallor is anemia

CBC is the best, cheapest, and easiest way to know the hematocrit level and diagnose anemia

Serum ferritin, HB electrophoresis, and bone marrow examination may help dierentiate dierent
causes of anemia.

PT, PTT, and INR would help detect the cause of bleeding tendency in particular patients.

Page - 1012
Internal Medicine - Hematology

Question 46/155

Question #46

A 17-year-old female develops a urinary tract infection. Her CBC show a WBC count of 13*10^9/L and
platelet count of 640*10^9/L, and her C-reactive protein is elevated. She was prescribed oral
antibiotics for her UTI. What would you do regarding her thrombocytosis?

a. Blood film
b. Bone marrow aspiration
c. Bone marrow biopsy
d. Repeat CBC in 4 weeks
e. Start aspirin therapy

‫اﻹﺟﺎﺑﺔ ﻋﲆ اﻟﺼﻔﺤﺔ اﻟﺘﺎﻟﻴﺔ‬

Page - 1013
Internal Medicine - Hematology - Hematologic investigations

Question 46/155

Question #46

A 17-year-old female develops a urinary tract infection. Her CBC show a WBC count of 13*10^9/L and
platelet count of 640*10^9/L, and her C-reactive protein is elevated. She was prescribed oral
antibiotics for her UTI. What would you do regarding her thrombocytosis?

a. Blood film
b. Bone marrow aspiration
c. Bone marrow biopsy
d. Repeat CBC in 4 weeks √
e. Start aspirin therapy

Description

Platelets are acute-phase reactants that increase during infections and other inflammatory
conditions (reactive thrombocytosis).

e platelet count should be repeated a few weeks later; at that time, you will mostly find an
average Platelet count if reactive thrombocytosis is the diagnosis.

Examples of acute-phase reactants: are CRP, ESR, WBC, ferritin, and platelet count

Page - 1014
Internal Medicine - Hematology

Question 47/155

Question #47

A 23-year-old male patient started to have a fever 4 days ago. His temperature is 38.9 °C, and he has
no localizing symptoms. His CBC show an elevated WBC with absolute lymphocytosis. Which is the
most likely diagnosis?

a. Bacterial infection
b. Viral infection
c. Parasitic infection
d. Connective tissue disease
e. Lymphoma

‫اﻹﺟﺎﺑﺔ ﻋﲆ اﻟﺼﻔﺤﺔ اﻟﺘﺎﻟﻴﺔ‬

Page - 1015
Internal Medicine - Hematology - Hematologic investigations

Question 47/155

Question #47

A 23-year-old male patient started to have a fever 4 days ago. His temperature is 38.9 °C, and he has
no localizing symptoms. His CBC show an elevated WBC with absolute lymphocytosis. Which is the
most likely diagnosis?

a. Bacterial infection
b. Viral infection √
c. Parasitic infection
d. Connective tissue disease
e. Lymphoma

Description

e absolute lymphocyte count equals the total number of white blood cells multiplied by the
percentage of lymphocytes.

e most common cause of absolute lymphocytosis is a viral infection.

Connective tissue disorders infrequently cause reactive lymphocytosis.

Neoplastic hyperproliferation (e.g., Lymphoma) is a possible less common cause of lymphocytosis

Bacterial infections cause neutrophilia, not lymphocytosis, except Bordetella pertussis, which has
been known to cause absolute lymphocyte counts of up to 70,000/L

Page - 1016
Internal Medicine - Hematology

Question 48/155

Question #48

e following figure shows a blood film. What is the finding you see?

a. Schistocytes
b. Target cells
c. Spherocytosis
d. Teardrop cells
e. Burr cells

‫اﻹﺟﺎﺑﺔ ﻋﲆ اﻟﺼﻔﺤﺔ اﻟﺘﺎﻟﻴﺔ‬

Page - 1017
Internal Medicine - Hematology - Hematologic investigations

Question 48/155

Question #48

e following figure shows a blood film. What is the finding you see?

a. Schistocytes
b. Target cells √
c. Spherocytosis
d. Teardrop cells
e. Burr cells

Description

e normal RBC shape is a biconcave disk; it will typically appear under the microscope with a
central pallor

e figure shows target cells that may be found in the following diseases;

SCA/alassemia
Page - 1018
IDA
Hyposplenism
Liver disease

Page - 1019
Internal Medicine - Hematology

Question 49/155

Question #49

A 22-year-old female has an accidental finding in her complete blood count; her platelet count is
30*10^9/L. e platelet is found to be clumped under the microscope. What would you do next?

a. Repeat CBC aer 2 weeks


b. Repeat CBC now with an alternative anticoagulant
c. Refer to the hematology clinic
d. HIV testing
e. Tell the patient that she has an ITP

‫اﻹﺟﺎﺑﺔ ﻋﲆ اﻟﺼﻔﺤﺔ اﻟﺘﺎﻟﻴﺔ‬

Page - 1020
Internal Medicine - Hematology - Hematologic investigations

Question 49/155

Question #49

A 22-year-old female has an accidental finding in her complete blood count; her platelet count is
30*10^9/L. e platelet is found to be clumped under the microscope. What would you do next?

a. Repeat CBC aer 2 weeks


b. Repeat CBC now with an alternative anticoagulant √
c. Refer to the hematology clinic
d. HIV testing
e. Tell the patient that she has an ITP

Description

Clumped platelets are an artifact due to antibodies to the anticoagulant used in the CBC tube.

e CBC automated machine calculates the clumped group of platelets as one platelet.

You should repeat the test in an alternative heparin tube or citrate tube, and the test will
demonstrate the accurate platelet count.

Page - 1021
Internal Medicine - Hematology

Question 50/155

Question #50

A patient’s blood film result shows a significant number of schistocytes. Which of the following is
not a likely diagnosis?

a. Disseminated intravascular coagulation (DIC)


b. Hemolytic uremic syndrome (HUS)
c. rombotic thrombocytopenic purpura (TTP)
d. Microangiopathic hemolytic anemia (MAHA)
e. Alpha thalassemia

‫اﻹﺟﺎﺑﺔ ﻋﲆ اﻟﺼﻔﺤﺔ اﻟﺘﺎﻟﻴﺔ‬

Page - 1022
Internal Medicine - Hematology - Hematologic investigations

Question 50/155

Question #50

A patient’s blood film result shows a significant number of schistocytes. Which of the following is
not a likely diagnosis?

a. Disseminated intravascular coagulation (DIC)


b. Hemolytic uremic syndrome (HUS)
c. rombotic thrombocytopenic purpura (TTP)
d. Microangiopathic hemolytic anemia (MAHA)
e. Alpha thalassemia √

Description

Schistocytes (also known as Helmet cells or Fragmented RBCs) are expected to be found in
intravascular hemolysis.

Alpha thalassemia usually presents with Heinz bodies.

Page - 1023
Internal Medicine - Hematology

Question 51/155

Question #51

A 22-year-old male patient develops severe diarrhea aer eating in a restaurant. His CBC shows
eosinophilia. What is the best next step in management?

a. Stool bacterial culture


b. Stool for ova or parasites
c. Stool WBC
d. Gastric lavage
e. Colonoscopy and biopsy

‫اﻹﺟﺎﺑﺔ ﻋﲆ اﻟﺼﻔﺤﺔ اﻟﺘﺎﻟﻴﺔ‬

Page - 1024
Internal Medicine - Hematology - Hematologic investigations

Question 51/155

Question #51

A 22-year-old male patient develops severe diarrhea aer eating in a restaurant. His CBC shows
eosinophilia. What is the best next step in management?

a. Stool bacterial culture


b. Stool for ova or parasites √
c. Stool WBC
d. Gastric lavage
e. Colonoscopy and biopsy

Description

Eosinophilia indicates an allergic reaction or parasitic infection.

e clinical scenario fits with a parasitic infection acquired orally in the restaurant and manifested
as diarrhea.

Page - 1025
Internal Medicine - Hematology

Question 52/155

Question #52

e following figure shows a blood film. What is the finding you see?

a. Schistocytes
b. Target cells
c. Spherocytosis
d. Teardrop cells
e. Burr cells

‫اﻹﺟﺎﺑﺔ ﻋﲆ اﻟﺼﻔﺤﺔ اﻟﺘﺎﻟﻴﺔ‬

Page - 1026
Internal Medicine - Hematology - Hematologic investigations

Question 52/155

Question #52

e following figure shows a blood film. What is the finding you see?

a. Schistocytes
b. Target cells
c. Spherocytosis
d. Teardrop cells √
e. Burr cells

Description

e normal RBC shape is a biconcave disk; it will typically appear under the microscope with a
central pallor

e figure shows teardrop cells that may be found in myelofibrosis

Page - 1027
Page - 1028
Internal Medicine - Hematology

Question 53/155

Question #53

A 27-year-old female complains of menorrhagia and recurrent epistaxis. Her CBC shows a
hemoglobin of 12.1 g/dL, WBC of 7*10^9/L, and platelet count of 70*10^9/L. What is the most likely
diagnosis?

a. Polycythemia
b. rombocytosis
c. rombocytopenia
d. Leukocytosis
e. Leukopenia

‫اﻹﺟﺎﺑﺔ ﻋﲆ اﻟﺼﻔﺤﺔ اﻟﺘﺎﻟﻴﺔ‬

Page - 1029
Internal Medicine - Hematology - Hematologic investigations

Question 53/155

Question #53

A 27-year-old female complains of menorrhagia and recurrent epistaxis. Her CBC shows a
hemoglobin of 12.1 g/dL, WBC of 7*10^9/L, and platelet count of 70*10^9/L. What is the most likely
diagnosis?

a. Polycythemia
b. rombocytosis
c. rombocytopenia √
d. Leukocytosis
e. Leukopenia

Description

rombocytopenia is defined as the presence of a low platelet count.

It may result in bleeding and elevation of the bleeding time.

Polycythemia is the presence of high hemoglobin level

rombocytosis is the presence of a high platelet count

Leukocytosis is the presence of an elevated WBC count, while leukopenia is the presence of a low
WBC count.

Page - 1030
Internal Medicine - Hematology

Question 54/155

Question #54

e following figure shows a blood film. What is the finding you see?

a. Schistocytes
b. Target cells
c. Spherocytosis
d. Teardrop cells
e. Burr cells

‫اﻹﺟﺎﺑﺔ ﻋﲆ اﻟﺼﻔﺤﺔ اﻟﺘﺎﻟﻴﺔ‬

Page - 1031
Internal Medicine - Hematology - Hematologic investigations

Question 54/155

Question #54

e following figure shows a blood film. What is the finding you see?

a. Schistocytes
b. Target cells
c. Spherocytosis √
d. Teardrop cells
e. Burr cells

Description

e normal RBC shape is a biconcave disk; it will typically appear under the microscope with a
central pallor

e figure shows spherocytes which may be found in hereditary spherocytosis and autoimmune
hemolytic anemia.
Page - 1032
Page - 1033
Internal Medicine - Hematology

Question 55/155

Question #55

Which of the following is the least likely diagnosis in a patient with hemolytic anemia and a positive
direct antiglobulin test?

a. Cold agglutinin hemolytic anemia


b. Warm agglutinin hemolytic anemia
c. Immediate blood transfusion reaction
d. Delayed blood transfusion reaction
e. Microangiopathic hemolytic anemia

‫اﻹﺟﺎﺑﺔ ﻋﲆ اﻟﺼﻔﺤﺔ اﻟﺘﺎﻟﻴﺔ‬

Page - 1034
Internal Medicine - Hematology - Hemolytic Anemia

Question 55/155

Question #55

Which of the following is the least likely diagnosis in a patient with hemolytic anemia and a positive
direct antiglobulin test?

a. Cold agglutinin hemolytic anemia


b. Warm agglutinin hemolytic anemia
c. Immediate blood transfusion reaction
d. Delayed blood transfusion reaction
e. Microangiopathic hemolytic anemia √

Description

Positive Coombs test indicates immune-mediated destruction of the blood cells (autoimmune or
alloimmune).

Microangiopathic hemolytic anemia is not an immune-mediated disorder. Nevertheless, it is caused


by various conditions like (TTP, DIC, HUS, etc.)

Classifications according to the cause:

Congenital:

RBC membrane (Spherocytosis, Elliptocytosis)


RBC hemoglobin (thalassemia, sickle cell disease)
Metabolic (G6PD deficiency)

Acquired

Alloimmune:
Hemolytic transfusion reaction
Hemolytic disease of the newborn
Autoimmune:
Cold antibody
Warm antibody
Non-immune:
Paroxysmal nocturnal hemoglobinuria (PNH)
Page - 1035
Microangiopathic hemolytic anemia (MAHA)
Mechanical (prosthetic valve, burn, etc.)
Hypersplenism
March hemoglobinuria
Infections (malaria, sepsis, etc.)
Drugs and chemicals
Systemic diseases (renal failure, liver failure)

Page - 1036
Internal Medicine - Hematology

Question 56/155

Question #56

e following disorders can cause intravascular hemolytic anemia except:

a. Microangiopathic hemolytic anemia


b. Paroxysmal nocturnal hemoglobinuria (PNH)
c. Cold agglutinin hemolytic anemia
d. Glucose-6-phosphate dehydrogenase enzyme deficiency
e. Hereditary spherocytosis

‫اﻹﺟﺎﺑﺔ ﻋﲆ اﻟﺼﻔﺤﺔ اﻟﺘﺎﻟﻴﺔ‬

Page - 1037
Internal Medicine - Hematology - Hemolytic Anemia

Question 56/155

Question #56

e following disorders can cause intravascular hemolytic anemia except:

a. Microangiopathic hemolytic anemia


b. Paroxysmal nocturnal hemoglobinuria (PNH)
c. Cold agglutinin hemolytic anemia
d. Glucose-6-phosphate dehydrogenase enzyme deficiency
e. Hereditary spherocytosis √

Description

Classification according to the site of hemolysis:

Extravascular: RBC removed by liver and spleen prematurely


Intravascular: RBC lysis in circulation

Hereditary spherocytosis is a known cause of extravascular hemolysis, not intravascular.

Note that G6PD deficiency can cause intra- and extravascular hemolysis, but the intravascular part
is more prominent.

e following table shows the hemolytic anemias' classification according to the hemolysis site.

Page - 1038
Internal Medicine - Hematology

Question 57/155

Question #57

A previously healthy 32-year-old female presents with fatigue, jaundice, and pallor. Her lab tests
show normocytic anemia, high reticulocyte count, and indirect hyperbilirubinemia. In addition, her
liver enzymes are normal. What is the best next step in the management of this patient?

a. Bone marrow aspiration


b. Direct antiglobulin test
c. Vitamin B12 level
d. Iron studies
e. Biliary ultrasound

‫اﻹﺟﺎﺑﺔ ﻋﲆ اﻟﺼﻔﺤﺔ اﻟﺘﺎﻟﻴﺔ‬

Page - 1039
Internal Medicine - Hematology - Hemolytic Anemia

Question 57/155

Question #57

A previously healthy 32-year-old female presents with fatigue, jaundice, and pallor. Her lab tests
show normocytic anemia, high reticulocyte count, and indirect hyperbilirubinemia. In addition, her
liver enzymes are normal. What is the best next step in the management of this patient?

a. Bone marrow aspiration


b. Direct antiglobulin test √
c. Vitamin B12 level
d. Iron studies
e. Biliary ultrasound

Description

is patient is suspected of having hemolytic anemia.

e first step in diagnosing the cause of hemolytic anemia is to do Coombs testing.

Direct antiglobulin test (Direct Coombs test) is used to detect antibodies bound to RBCs

It will be positive in case of autoimmune Hemolytic anemia, transfusion reaction, and hemolytic
disease of the newborn.

Blood sample mixed with antibodies to human globulin:

If agglutination → Positive DAT

If no agglutination → negative DAT

Page - 1040
Internal Medicine - Hematology

Question 58/155

Question #58

A 30-year-old female presents with jaundice and fatigue. She told you that she had an upper
respiratory infection 10 days ago. However, her lab investigations show hemoglobin of 9 g/dL, MCV
of 97fl, bilirubin of 2mg/dL, AST 20u/L, ALT 22u/L, and Alkaline phosphatase of 73 U/L. In addition,
her blood film shows spherocytes and immature RBCs. What is the best next step in the
management of this patient?

a. Indirect Coombs test


b. Direct Coombs test
c. Bone marrow biopsy
d. Abdominal ultrasound
e. Osmotic fragility test

‫اﻹﺟﺎﺑﺔ ﻋﲆ اﻟﺼﻔﺤﺔ اﻟﺘﺎﻟﻴﺔ‬

Page - 1041
Internal Medicine - Hematology - Hemolytic Anemia

Question 58/155

Question #58

A 30-year-old female presents with jaundice and fatigue. She told you that she had an upper
respiratory infection 10 days ago. However, her lab investigations show hemoglobin of 9 g/dL, MCV
of 97fl, bilirubin of 2mg/dL, AST 20u/L, ALT 22u/L, and Alkaline phosphatase of 73 U/L. In addition,
her blood film shows spherocytes and immature RBCs. What is the best next step in the
management of this patient?

a. Indirect Coombs test


b. Direct Coombs test √
c. Bone marrow biopsy
d. Abdominal ultrasound
e. Osmotic fragility test

Description

is patient is suspected of having hemolytic anemia.

e first step in diagnosing the cause of hemolytic anemia is to do Coombs testing.

Direct antiglobulin test (Direct Coombs test) is used to detect antibodies bound to RBCs

It will be positive in case of autoimmune Hemolytic anemia, transfusion reaction, and hemolytic
disease of the newborn.

Blood sample mixed with antibodies to human globulin:

If agglutination → Positive DAT


If no agglutination → negative DAT

Page - 1042
Internal Medicine - Hematology

Question 59/155

Question #59

A 22-year-old female presents with jaundice, anemia, and high reticulocytes count. What is the most
appropriate at this time?

a. G6PD enzyme assay


b. Direct Coombs test
c. Indirect Coombs test
d. Bone marrow biopsy
e. Osmotic fragility test

‫اﻹﺟﺎﺑﺔ ﻋﲆ اﻟﺼﻔﺤﺔ اﻟﺘﺎﻟﻴﺔ‬

Page - 1043
Internal Medicine - Hematology - Hemolytic Anemia

Question 59/155

Question #59

A 22-year-old female presents with jaundice, anemia, and high reticulocytes count. What is the most
appropriate at this time?

a. G6PD enzyme assay


b. Direct Coombs test √
c. Indirect Coombs test
d. Bone marrow biopsy
e. Osmotic fragility test

Description

is patient is suspected of having hemolytic anemia.

e first step in diagnosing the cause of hemolytic anemia is to do Coombs testing.

Direct antiglobulin test (Direct Coombs test) is used to detect antibodies bound to RBCs

It will be positive in case of autoimmune Hemolytic anemia, transfusion reaction, and hemolytic
disease of the newborn.

Blood sample mixed with antibodies to human globulin:

If agglutination → Positive DAT


If no agglutination → negative DAT

Page - 1044
Internal Medicine - Hematology

Question 60/155

Question #60

A 35-year-old female is admitted to the hematology ward with hemolytic anemia. Which of the
following is most characteristic of her condition?

a. Reduced LDH
b. Conjugated hyperbilirubinemia
c. rombocytopenia
d. Reticulocytosis
e. Elevated serum haptoglobin

‫اﻹﺟﺎﺑﺔ ﻋﲆ اﻟﺼﻔﺤﺔ اﻟﺘﺎﻟﻴﺔ‬

Page - 1045
Internal Medicine - Hematology - Hemolytic Anemia

Question 60/155

Question #60

A 35-year-old female is admitted to the hematology ward with hemolytic anemia. Which of the
following is most characteristic of her condition?

a. Reduced LDH
b. Conjugated hyperbilirubinemia
c. rombocytopenia
d. Reticulocytosis √
e. Elevated serum haptoglobin

Description

A high reticulocyte count indicates either hemolytic anemia or acute bleeding.

Features of hemolytic anemia are:

Increase of Free hemoglobin, reticulocyte count (> 2%), LDH, Indirect bilirubin (unconjugated
hyperbilirubinemia), Urinary urobilinogen, Urine hemoglobin, and Urine hemosiderin.

Decrease of hemoglobin, Serum Haptoglobin, and RBC survival

Page - 1046
Internal Medicine - Hematology

Question 61/155

Question #61

A 48-year-old male patient has features suggestive of hemolytic anemia. Which of the following
most likely fits with this diagnosis?

a. Elevated LDH, low haptoglobin, normal Indirect bilirubin


b. Elevated LDH, Elevated Haptoglobin, low Indirect bilirubin
c. Low LDH, Elevated Haptoglobin, Elevated Indirect bilirubin
d. Elevated LDH, low Haptoglobin, Elevated Indirect bilirubin
e. Low LDH, low Haptoglobin, Elevated Indirect bilirubin

‫اﻹﺟﺎﺑﺔ ﻋﲆ اﻟﺼﻔﺤﺔ اﻟﺘﺎﻟﻴﺔ‬

Page - 1047
Internal Medicine - Hematology - Hemolytic Anemia

Question 61/155

Question #61

A 48-year-old male patient has features suggestive of hemolytic anemia. Which of the following
most likely fits with this diagnosis?

a. Elevated LDH, low haptoglobin, normal Indirect bilirubin


b. Elevated LDH, Elevated Haptoglobin, low Indirect bilirubin
c. Low LDH, Elevated Haptoglobin, Elevated Indirect bilirubin
d. Elevated LDH, low Haptoglobin, Elevated Indirect bilirubin √
e. Low LDH, low Haptoglobin, Elevated Indirect bilirubin

Description

High LDH, low haptoglobin, and indirect hyperbilirubinemia are the most likely lab results in
hemolytic anemia.

Features of hemolytic anemia are:

Increase of Free hemoglobin, reticulocyte count (> 2%), LDH, Indirect bilirubin (unconjugated
hyperbilirubinemia), Urinary urobilinogen, Urine hemoglobin, and Urine hemosiderin.

Decrease of hemoglobin, Serum Haptoglobin, and RBC survival

Page - 1048
Internal Medicine - Hematology

Question 62/155

Question #62

A male patient with a known case of hemophilia A presents to you for genetic counseling. However,
his wife has no family history of hemophilia. Which of the following is the most factual statement?

a. Half of his daughters will be carriers of the disease


b. Half of his sons will be aected
c. All his sons will be aected
d. All his daughters will be carriers
e. Half of his children will be aected regardless of the gender

‫اﻹﺟﺎﺑﺔ ﻋﲆ اﻟﺼﻔﺤﺔ اﻟﺘﺎﻟﻴﺔ‬

Page - 1049
Internal Medicine - Hematology - Hemophilia

Question 62/155

Question #62

A male patient with a known case of hemophilia A presents to you for genetic counseling. However,
his wife has no family history of hemophilia. Which of the following is the most factual statement?

a. Half of his daughters will be carriers of the disease


b. Half of his sons will be aected
c. All his sons will be aected
d. All his daughters will be carriers √
e. Half of his children will be aected regardless of the gender

Description

Hemophilia is an X-linked recessive disease (almost exclusively in males).

His sons will be XY (the X from the mother and the Y from the father), so their X chromosome will
not contain the abnormal gene.

His daughters will take the aected X chromosome from him and a normal X chromosome from
their mother. As a result, they will be in a carrier status.

So, his sons will not be aected, and all his daughters will be in a carrier status.

Page - 1050
Internal Medicine - Hematology

Question 63/155

Question #63

A 25-year-old female patient presents with easy bruises and bleeding tendencies. However, she has
no family history of bleeding tendency. Based on this limited history, which diagnosis is the least
likely?

a. B12 deficiency
b. Leukemia
c. Hemophilia
d. Von Willebrand disease
e. Vitamin K deficiency

‫اﻹﺟﺎﺑﺔ ﻋﲆ اﻟﺼﻔﺤﺔ اﻟﺘﺎﻟﻴﺔ‬

Page - 1051
Internal Medicine - Hematology - Hemophilia

Question 63/155

Question #63

A 25-year-old female patient presents with easy bruises and bleeding tendencies. However, she has
no family history of bleeding tendency. Based on this limited history, which diagnosis is the least
likely?

a. B12 deficiency
b. Leukemia
c. Hemophilia √
d. Von Willebrand disease
e. Vitamin K deficiency

Description

Being a female will make hemophilia less likely. Hemophilia is an X-linked recessive disease (almost
exclusively in males).

Hemophilia A is Factor VIII deficiency, while hemophilia B is factor IX deficiency (type B is known as
Christmas disease)

Patients with hemophilia present with bleeding tendency and prolonged aPTT.

e diagnosis is confirmed by testing for factor VIII level.

Page - 1052
Internal Medicine - Hematology

Question 64/155

Question #64

In the routine preoperative assessment, A 22-year-old male patient is found to have prolonged aPTT.
His family history is unremarkable, and his lab investigations show normal PT, INR, and BT. What is
the most likely diagnosis?

a. Warfarin overdose
b. Von Willebrand’s disease
c. Factor VIII deficiency
d. Factor XIII deficiency
e. rombocytopenia

‫اﻹﺟﺎﺑﺔ ﻋﲆ اﻟﺼﻔﺤﺔ اﻟﺘﺎﻟﻴﺔ‬

Page - 1053
Internal Medicine - Hematology - Hemophilia

Question 64/155

Question #64

In the routine preoperative assessment, A 22-year-old male patient is found to have prolonged aPTT.
His family history is unremarkable, and his lab investigations show normal PT, INR, and BT. What is
the most likely diagnosis?

a. Warfarin overdose
b. Von Willebrand’s disease
c. Factor VIII deficiency √
d. Factor XIII deficiency
e. rombocytopenia

Description

Being a male with elevated aPTT is suggestive of hemophilia A

Note that vWD presents with abnormal BT

Warfarin overdose and vitamin K deficiency present with prolonged PT and INR

Factor XIII deficiency presents with bleeding tendency and normal PTT, PT, and INR

rombocytopenia is simply diagnosed on CBC and presents with prolonged BT.

Hemophilia is an X-linked recessive disease (almost exclusively in males). Hemophilia A is


Factor VIII deficiency, while hemophilia B is factor IX deficiency (type B is known as Christmas
disease)
e patient presents with bleeding tendency and prolonged aPTT.
e diagnosis is confirmed by testing for factor VIII level.

Page - 1054
Internal Medicine - Hematology

Question 65/155

Question #65

A 22-year-old male patient is suspected of having hereditary spherocytosis. Which is the most
accurate test to help detect the condition?

a. Indirect Coombs test


b. Direct Coombs test
c. Flow cytometric EMA (eosin-5-maleimide) binding test
d. Osmotic fragility test
e. Laparoscopy

‫اﻹﺟﺎﺑﺔ ﻋﲆ اﻟﺼﻔﺤﺔ اﻟﺘﺎﻟﻴﺔ‬

Page - 1055
Internal Medicine - Hematology - Hereditary Spherocytosis (HS)

Question 65/155

Question #65

A 22-year-old male patient is suspected of having hereditary spherocytosis. Which is the most
accurate test to help detect the condition?

a. Indirect Coombs test


b. Direct Coombs test
c. Flow cytometric EMA (eosin-5-maleimide) binding test √
d. Osmotic fragility test
e. Laparoscopy

Description

In Hereditary Spherocytosis (HS), the most common abnormality is the deficiency of membrane
proteins (Beta-spectrin or ankyrin), leading to reduced RBC elasticity and destruction of the RBCs
when passing through the spleen.

Diagnosis of HS:

Hb: 6 – 10 g/dL (depends on the degree of compensation)


Reticulocytosis (usually 6 – 20%)
High indirect bilirubin and LDH
Blood film: Spherocytes (small round hyperchromic RBCs without central pallor)
Negative Coombs test
e osmotic fragility test is no longer recommended
Flow cytometric EMA (eosin-5-maleimide) binding test (the most accurate test)

Page - 1056
Internal Medicine - Hematology

Question 66/155

Question #66

A 23-year-old male patient presents to your clinic with right upper quadrant recurrent abdominal
pain. His family history includes a father with a splenectomy at a young age. His lab investigations
show anemia, low MCV, and high MCH. In addition, abdominal ultrasound demonstrates
gallbladder stones. What is the most likely diagnosis?

a. G6PD deficiency
b. Beta thalassemia
c. Alpha thalassemia
d. Hereditary spherocytosis
e. Lead poisoning

‫اﻹﺟﺎﺑﺔ ﻋﲆ اﻟﺼﻔﺤﺔ اﻟﺘﺎﻟﻴﺔ‬

Page - 1057
Internal Medicine - Hematology - Hereditary Spherocytosis (HS)

Question 66/155

Question #66

A 23-year-old male patient presents to your clinic with right upper quadrant recurrent abdominal
pain. His family history includes a father with a splenectomy at a young age. His lab investigations
show anemia, low MCV, and high MCH. In addition, abdominal ultrasound demonstrates
gallbladder stones. What is the most likely diagnosis?

a. G6PD deficiency
b. Beta thalassemia
c. Alpha thalassemia
d. Hereditary spherocytosis √
e. Lead poisoning

Description

e patient is suspected of having hereditary spherocytosis.

e family history of hemolytic anemia, microcytic anemia, high MCH, and gallstones strongly
suggest hereditary spherocytosis.

G6PD presents with episodic hemolysis aer exposure to oxidants.

In Hereditary Spherocytosis (HS), the most common abnormality is the deficiency of membrane
proteins (Beta-spectrin or ankyrin), leading to reduced RBC elasticity and destruction of the RBCs
when passing through the spleen.

Page - 1058
Internal Medicine - Hematology

Question 67/155

Question #67

A 23-year-old female with a history of hereditary spherocytosis presents with fatigue and pallor. Her
blood tests show anemia of 5.1g/dL, leukopenia, thrombocytopenia, low MCV, and a reticulocyte
count of 0.2%. What is the most likely explanation?

a. Hemolytic crisis
b. Parvovirus B19 infection
c. Sequestration crisis
d. Autoimmune hemolytic anemia
e. Megaloblastic crisis

‫اﻹﺟﺎﺑﺔ ﻋﲆ اﻟﺼﻔﺤﺔ اﻟﺘﺎﻟﻴﺔ‬

Page - 1059
Internal Medicine - Hematology - Hereditary Spherocytosis (HS)

Question 67/155

Question #67

A 23-year-old female with a history of hereditary spherocytosis presents with fatigue and pallor. Her
blood tests show anemia of 5.1g/dL, leukopenia, thrombocytopenia, low MCV, and a reticulocyte
count of 0.2%. What is the most likely explanation?

a. Hemolytic crisis
b. Parvovirus B19 infection √
c. Sequestration crisis
d. Autoimmune hemolytic anemia
e. Megaloblastic crisis

Description

Aplastic crisis is associated with parvovirus B19 infection.

It manifests as low HB, WBC, Platelet, and reticulocytes.

Hemolytic crisis presents with elevated reticulocytes not reduced.

Megaloblastic crisis will present with macrocytic anemia (not Low MCV)

Page - 1060
Internal Medicine - Hematology

Question 68/155

Question #68

A 22-year-old male patient is suspected of having hereditary spherocytosis. Which of the following is
the most common mood inheritance for this condition?

a. Autosomal dominant
b. Autosomal recessive
c. X linked recessive
d. X linked dominant
e. Y linked

‫اﻹﺟﺎﺑﺔ ﻋﲆ اﻟﺼﻔﺤﺔ اﻟﺘﺎﻟﻴﺔ‬

Page - 1061
Internal Medicine - Hematology - Hereditary Spherocytosis (HS)

Question 68/155

Question #68

A 22-year-old male patient is suspected of having hereditary spherocytosis. Which of the following is
the most common mood inheritance for this condition?

a. Autosomal dominant √
b. Autosomal recessive
c. X linked recessive
d. X linked dominant
e. Y linked

Description

In Hereditary Spherocytosis (HS), the most common abnormality is the deficiency of membrane
proteins (Beta-spectrin or ankyrin), leading to reduced RBC elasticity and destruction of the RBCs
when passing through the spleen.

Approximately 75% of hereditary spherocytosis cases have an autosomal dominant inheritance


pattern; 25% are recessive forms and de novo mutations.

Page - 1062
Internal Medicine - Hematology

Question 69/155

Question #69

A 22-year-old female presents with menorrhagia, bruises, and petechiae for a week. Her lab
investigations show a platelet count of 30*10^9/L and elevated bleeding time. What is the most
likely diagnosis?

a. Cytomegalovirus infection
b. HIV infection
c. Systemic lupus erythematosus
d. Idiopathic thrombocytopenic purpura
e. Acute leukemia

‫اﻹﺟﺎﺑﺔ ﻋﲆ اﻟﺼﻔﺤﺔ اﻟﺘﺎﻟﻴﺔ‬

Page - 1063
Internal Medicine - Hematology - Idiopathic rombocytopenic Purpura (ITP)

Question 69/155

Question #69

A 22-year-old female presents with menorrhagia, bruises, and petechiae for a week. Her lab
investigations show a platelet count of 30*10^9/L and elevated bleeding time. What is the most
likely diagnosis?

a. Cytomegalovirus infection
b. HIV infection
c. Systemic lupus erythematosus
d. Idiopathic thrombocytopenic purpura √
e. Acute leukemia

Description

Idiopathic rombocytopenic Purpura (ITP) is a disorder of primary hemostasis in which


autoantibodies against platelets lead to platelet destruction, thrombocytopenia, and bleeding.

e isolated thrombocytopenia in healthy persons strongly suggests the diagnosis of ITP.

Possible symptoms are: easy bruising, epistaxis, menorrhagia, and petechial rash

It is a diagnosis of exclusion. However, bone marrow examination is indicated in elderly patients to


exclude malignancy.

Page - 1064
Internal Medicine - Hematology

Question 70/155

Question #70

A previously healthy 28-year-old male is brought to you by his friend, who noted a skin rash on his
neck. On examination, you noted a purpuric rash extending along with his necklace. His blood count
shows hemoglobin of 14.5 g/dL, WBC of 5*10^9/L, and platelet count of 3*10^9. What is the most
likely diagnosis?

a. Henoch Schönlein purpura


b. rombotic thrombocytopenic purpura
c. Idiopathic thrombocytopenic purpura (ITP)
d. Von Willebrand’s disease
e. Systemic lupus erythematosus

‫اﻹﺟﺎﺑﺔ ﻋﲆ اﻟﺼﻔﺤﺔ اﻟﺘﺎﻟﻴﺔ‬

Page - 1065
Internal Medicine - Hematology - Idiopathic rombocytopenic Purpura (ITP)

Question 70/155

Question #70

A previously healthy 28-year-old male is brought to you by his friend, who noted a skin rash on his
neck. On examination, you noted a purpuric rash extending along with his necklace. His blood count
shows hemoglobin of 14.5 g/dL, WBC of 5*10^9/L, and platelet count of 3*10^9. What is the most
likely diagnosis?

a. Henoch Schönlein purpura


b. rombotic thrombocytopenic purpura
c. Idiopathic thrombocytopenic purpura (ITP) √
d. Von Willebrand’s disease
e. Systemic lupus erythematosus

Description

Idiopathic rombocytopenic Purpura (ITP) is a disorder of primary hemostasis in which


autoantibodies against platelets lead to platelet destruction, thrombocytopenia, and bleeding.

e isolated thrombocytopenia in healthy persons strongly suggests the diagnosis of ITP.

Possible symptoms are: easy bruising, epistaxis, menorrhagia, and petechial rash

It is a diagnosis of exclusion. However, bone marrow examination is indicated in elderly patients to


exclude malignancy.

Page - 1066
Internal Medicine - Hematology

Question 71/155

Question #71

A 48-year-old male patient was diagnosed with anemia. Which of the following most likely indicates
iron deficiency?

a. Reduced total iron-binding capacity


b. Elevated serum ferritin level
c. Elevated transferrin saturation
d. Reticulocytosis 1 week aer initiating oral iron
e. Reduced RDW

‫اﻹﺟﺎﺑﺔ ﻋﲆ اﻟﺼﻔﺤﺔ اﻟﺘﺎﻟﻴﺔ‬

Page - 1067
Internal Medicine - Hematology - Iron Deficiency Anemia (IDA)

Question 71/155

Question #71

A 48-year-old male patient was diagnosed with anemia. Which of the following most likely indicates
iron deficiency?

a. Reduced total iron-binding capacity


b. Elevated serum ferritin level
c. Elevated transferrin saturation
d. Reticulocytosis 1 week aer initiating oral iron √
e. Reduced RDW

Description

In a patient with iron deficiency anemia (IDA), the diagnosis can be confirmed by providing an oral
iron and then one week later performing a reticulocyte count

Reticulocytosis peaks on the 7th – 10th day aer starting the therapy.

Iron deficiency anemia (IDA) is generally the most common cause of anemia.

It is characterized by microcytic hypochromic anemia.

Its causes include:

Poor intake
Decrease absorption (e.g., celiac disease, gastrectomy
Increased Iron demands (pregnancy, growing adolescent)
Chronic Blood loss (most common cause)

Laboratory investigations will show:

CBC: Low HB, MCV, MCH, and high RDW


Blood film: Poikilocytosis (Variable RBC shapes), Anisocytosis (Variable RBC sizes), and Target
cells
Serology: Low serum ferritin, Iron, Transferrin saturation, High transferrin level, and Total Iron
Binding Capacity (TIBC)

Specific features of IDA:


Page - 1068
Brittle nails and nail cracking
Koilonychia (flattening or concavity of the nails)
Angular stomatitis
Sore tongue
Pica (eating unusual substances like soil, ice, etc.)

Page - 1069
Internal Medicine - Hematology

Question 72/155

Question #72

A 40-year-old woman complains of fatigue. Her complete blood count shows hemoglobin of 9.8
g/dL, MCV of 74 fl, serum ferritin of 8 mg/L, and total iron-binding capacity of 75 umol/L. What is the
most likely diagnosis?

a. alassemia major
b. alassemia minor
c. Iron deficiency anemia
d. Anemia of chronic disease
e. B12 deficiency

‫اﻹﺟﺎﺑﺔ ﻋﲆ اﻟﺼﻔﺤﺔ اﻟﺘﺎﻟﻴﺔ‬

Page - 1070
Internal Medicine - Hematology - Iron Deficiency Anemia (IDA)

Question 72/155

Question #72

A 40-year-old woman complains of fatigue. Her complete blood count shows hemoglobin of 9.8
g/dL, MCV of 74 fl, serum ferritin of 8 mg/L, and total iron-binding capacity of 75 umol/L. What is the
most likely diagnosis?

a. alassemia major
b. alassemia minor
c. Iron deficiency anemia √
d. Anemia of chronic disease
e. B12 deficiency

Description

Iron deficiency anemia (IDA) is generally the most common cause of anemia.

It is characterized by microcytic hypochromic anemia.

Its causes include:

Poor intake
Decrease absorption (e.g., celiac disease, gastrectomy
Increased Iron demands (pregnancy, growing adolescent)
Chronic Blood loss (most common cause)

Laboratory investigations will show:

CBC: Low HB, MCV, MCH, and high RDW


Blood film: Poikilocytosis (Variable RBC shapes), Anisocytosis (Variable RBC sizes), and Target
cells
Serology: Low serum ferritin, Iron, Transferrin saturation, High transferrin level, and Total Iron
Binding Capacity (TIBC)

Specific features of IDA:

Brittle nails and nail cracking


Koilonychia (flattening or concavity of the nails)
Page - 1071
Angular stomatitis
Sore tongue
Pica (eating unusual substances like soil, ice, etc.)

Page - 1072
Internal Medicine - Hematology

Question 73/155

Question #73

A 32-year -old male patient develops easy fatigability and pallor for 3 months. His CBC shows
hemoglobin of 8.9 g/dL, MCV of 69 fl, and serum ferritin of 2 mg/L. What is the most likely
diagnosis?

a. B12 deficiency
b. Folic acid deficiency
c. Iron deficiency
d. Sideroblastic anemia
e. alassemia

‫اﻹﺟﺎﺑﺔ ﻋﲆ اﻟﺼﻔﺤﺔ اﻟﺘﺎﻟﻴﺔ‬

Page - 1073
Internal Medicine - Hematology - Iron Deficiency Anemia (IDA)

Question 73/155

Question #73

A 32-year -old male patient develops easy fatigability and pallor for 3 months. His CBC shows
hemoglobin of 8.9 g/dL, MCV of 69 fl, and serum ferritin of 2 mg/L. What is the most likely
diagnosis?

a. B12 deficiency
b. Folic acid deficiency
c. Iron deficiency √
d. Sideroblastic anemia
e. alassemia

Description

Iron deficiency anemia (IDA) is generally the most common cause of anemia.

It is characterized by microcytic hypochromic anemia.

Its causes include:

Poor intake
Decrease absorption (e.g., celiac disease, gastrectomy
Increased Iron demands (pregnancy, growing adolescent)
Chronic Blood loss (most common cause)

Laboratory investigations will show:

CBC: Low HB, MCV, MCH, and high RDW


Blood film: Poikilocytosis (Variable RBC shapes), Anisocytosis (Variable RBC sizes), and Target
cells
Serology: Low serum ferritin, Iron, Transferrin saturation, High transferrin level, and Total Iron
Binding Capacity (TIBC)

Specific features of IDA:

Brittle nails and nail cracking


Koilonychia (flattening or concavity of the nails)
Page - 1074
Angular stomatitis
Sore tongue
Pica (eating unusual substances like soil, ice, etc.)

Page - 1075
Internal Medicine - Hematology

Question 74/155

Question #74

Which of the following statements is true about iron absorption?

a. Iron is primarily absorbed in the terminal ileum


b. Diarrhea is a common complication of oral iron
c. It is absorbed better if taken aer meals
d. Proton pump inhibitors do not aect its absorption
e. Patients with celiac disease may develop iron deficiency

‫اﻹﺟﺎﺑﺔ ﻋﲆ اﻟﺼﻔﺤﺔ اﻟﺘﺎﻟﻴﺔ‬

Page - 1076
Internal Medicine - Hematology - Iron Deficiency Anemia (IDA)

Question 74/155

Question #74

Which of the following statements is true about iron absorption?

a. Iron is primarily absorbed in the terminal ileum


b. Diarrhea is a common complication of oral iron
c. It is absorbed better if taken aer meals
d. Proton pump inhibitors do not aect its absorption
e. Patients with celiac disease may develop iron deficiency √

Description

Patients with celiac disease will develop atrophic villi, which are more significant in the proximal
intestine (at the site of iron absorption). So, patients with celiac disease are likely to develop iron
deficiency secondary to malabsorption.

Factors that increase iron absorption:

Increased acidity

Alcohol

Use of vitamin C supplement

When iron is complemented with lactose

e iron of animal source (ferrous) is more absorbable than plant source (ferric)

Factors that decrease iron absorption:

Tea drinking

e use of PPIs or any acid suppressant

Plant-source iron

Page - 1077
Internal Medicine - Hematology

Question 75/155

Question #75

A 42-year-old female is suering from iron deficiency anemia secondary to menorrhagia. Which of
the following laboratory findings fits with the diagnosis?

a. Ferritin low, TIBC high, MCV low, MCH high


b. Ferritin low, TIBC high, MCV high, MCH low
c. Ferritin high, TIBC low, MCV low, MCH low
d. Ferritin low, TIBC high, MCV low, MCH low
e. Ferritin low, TIBC low, MCV low, MCH low

‫اﻹﺟﺎﺑﺔ ﻋﲆ اﻟﺼﻔﺤﺔ اﻟﺘﺎﻟﻴﺔ‬

Page - 1078
Internal Medicine - Hematology - Iron Deficiency Anemia (IDA)

Question 75/155

Question #75

A 42-year-old female is suering from iron deficiency anemia secondary to menorrhagia. Which of
the following laboratory findings fits with the diagnosis?

a. Ferritin low, TIBC high, MCV low, MCH high


b. Ferritin low, TIBC high, MCV high, MCH low
c. Ferritin high, TIBC low, MCV low, MCH low
d. Ferritin low, TIBC high, MCV low, MCH low √
e. Ferritin low, TIBC low, MCV low, MCH low

Description

Laboratory investigations in iron deficiency anemia will show the following:

CBC: Low HB, MCV, MCH, and high RDW


Blood film: Poikilocytosis (Variable RBC shapes), Anisocytosis (Variable RBC sizes), and Target
cells
Serology: Low serum ferritin, Iron, Transferrin saturation, High transferrin level, and Total Iron
Binding Capacity (TIBC)

Iron deficiency anemia (IDA) is generally the most common cause of anemia.

It is characterized by microcytic hypochromic anemia.

Its causes include:

Poor intake
Decrease absorption (e.g., celiac disease, gastrectomy
Increased Iron demands (pregnancy, growing adolescent)
Chronic Blood loss (most common cause)

Specific features of IDA:

Brittle nails and nail cracking


Koilonychia (flattening or concavity of the nails)
Angular stomatitis
Page - 1079
Sore tongue
Pica (eating unusual substances like soil, ice, etc.)

Page - 1080
Internal Medicine - Hematology

Question 76/155

Question #76

A 30-year-old female presents with fatigue and pallor. Which of the following is the most reliable
indicator of iron deficiency anemia?

a. How Total Iron Binding Capacity (TIBC)


b. Low serum iron
c. Low MCV
d. Low serum ferritin
e. Low MCH

‫اﻹﺟﺎﺑﺔ ﻋﲆ اﻟﺼﻔﺤﺔ اﻟﺘﺎﻟﻴﺔ‬

Page - 1081
Internal Medicine - Hematology - Iron Deficiency Anemia (IDA)

Question 76/155

Question #76

A 30-year-old female presents with fatigue and pallor. Which of the following is the most reliable
indicator of iron deficiency anemia?

a. How Total Iron Binding Capacity (TIBC)


b. Low serum iron
c. Low MCV
d. Low serum ferritin √
e. Low MCH

Description

Low serum ferritin level is diagnostic of iron deficiency anemia even in its early phase.

Iron deficiency anemia (IDA) is generally the most common cause of anemia.

It is characterized by microcytic hypochromic anemia.

Its causes include:

Poor intake
Decrease absorption (e.g., celiac disease, gastrectomy
Increased Iron demands (pregnancy, growing adolescent)
Chronic Blood loss (most common cause)

Laboratory investigations will show:

CBC: Low HB, MCV, MCH, and high RDW


Blood film: Poikilocytosis (Variable RBC shapes), Anisocytosis (Variable RBC sizes), and Target
cells
Serology: Low serum ferritin, Iron, Transferrin saturation, High transferrin level, and Total Iron
Binding Capacity (TIBC)

Specific features of IDA:

Brittle nails and nail cracking


Koilonychia (flattening or concavity of the nails) Page - 1082
Angular stomatitis
Sore tongue
Pica (eating unusual substances like soil, ice, etc.)

Page - 1083
Internal Medicine - Hematology

Question 77/155

Question #77

A 48-year-old male patient presents with fatigue and shortness of breath. His complaints have been
progressive over the past 2 months. On examination, there are a red sore tongue, angular stomatitis,
and Koilonychia. His lab investigations are significant for a hematocrit of 24% and MCV of 66 fl.
What is the most likely diagnosis?

a. B12 deficiency
b. Folic acid deficiency
c. Iron deficiency
d. Sideroblastic anemia
e. alassemia

‫اﻹﺟﺎﺑﺔ ﻋﲆ اﻟﺼﻔﺤﺔ اﻟﺘﺎﻟﻴﺔ‬

Page - 1084
Internal Medicine - Hematology - Iron Deficiency Anemia (IDA)

Question 77/155

Question #77

A 48-year-old male patient presents with fatigue and shortness of breath. His complaints have been
progressive over the past 2 months. On examination, there are a red sore tongue, angular stomatitis,
and Koilonychia. His lab investigations are significant for a hematocrit of 24% and MCV of 66 fl.
What is the most likely diagnosis?

a. B12 deficiency
b. Folic acid deficiency
c. Iron deficiency √
d. Sideroblastic anemia
e. alassemia

Description

Iron deficiency anemia (IDA) is generally the most common cause of anemia.

It is characterized by microcytic hypochromic anemia.

Its causes include:

Poor intake
Decrease absorption (e.g., celiac disease, gastrectomy
Increased Iron demands (pregnancy, growing adolescent)
Chronic Blood loss (most common cause)

Laboratory investigations will show:

CBC: Low HB, MCV, MCH, and high RDW


Blood film: Poikilocytosis (Variable RBC shapes), Anisocytosis (Variable RBC sizes), and Target
cells
Serology: Low serum ferritin, Iron, Transferrin saturation, High transferrin level, and Total Iron
Binding Capacity (TIBC)

Specific features of IDA:

Brittle nails and nail cracking


Page - 1085
Koilonychia (flattening or concavity of the nails)
Angular stomatitis
Sore tongue
Pica (eating unusual substances like soil, ice, etc.)

Page - 1086
Internal Medicine - Hematology

Question 78/155

Question #78

A 22-year-old female developed iron deficiency anemia. She started on oral iron one week ago.
Which of the following is appropriate to assess the response of oral iron at this time?

a. Repeat hemoglobin level


b. Perform hemoglobin electrophoresis
c. Check for the elevation of MCV
d. Check for reticulocyte count
e. Check for serum iron level

‫اﻹﺟﺎﺑﺔ ﻋﲆ اﻟﺼﻔﺤﺔ اﻟﺘﺎﻟﻴﺔ‬

Page - 1087
Internal Medicine - Hematology - Iron Deficiency Anemia (IDA)

Question 78/155

Question #78

A 22-year-old female developed iron deficiency anemia. She started on oral iron one week ago.
Which of the following is appropriate to assess the response of oral iron at this time?

a. Repeat hemoglobin level


b. Perform hemoglobin electrophoresis
c. Check for the elevation of MCV
d. Check for reticulocyte count √
e. Check for serum iron level

Description

In a patient with iron deficiency anemia (IDA), the diagnosis can be confirmed by providing an oral
iron and then one week later performing a reticulocyte count

Reticulocytosis peaks on the 7th – 10th day aer starting the therapy.

e hemoglobin level will not change for at least 2 weeks and can take 2 months to return to its
normal level.

Serum iron level is diet-sensitive, but it is not an indicator of response to the treatment. However, its
elevation aer oral iron intake indicates appropriate absorption of iron.

Page - 1088
Internal Medicine - Hematology

Question 79/155

Question #79

A 52-year-old female is on methotrexate and diclofenac sodium for rheumatoid arthritis. Recently
she developed epigastric pain, especially aer eating, palpitation, and pallor. Her ECG and cardiac
enzymes are unremarkable. Her rest of the lab investigations show hemoglobin of 8.9 g/dL, MCV of
73 fl, and a serum ferritin level of 5 mg/L. What is the most likely diagnosis?

a. Chronic gastrointestinal bleeding


b. Folic acid deficiency
c. Chronic hemolytic anemia
d. Aplastic anemia
e. Anemia of chronic disease

‫اﻹﺟﺎﺑﺔ ﻋﲆ اﻟﺼﻔﺤﺔ اﻟﺘﺎﻟﻴﺔ‬

Page - 1089
Internal Medicine - Hematology - Iron Deficiency Anemia (IDA)

Question 79/155

Question #79

A 52-year-old female is on methotrexate and diclofenac sodium for rheumatoid arthritis. Recently
she developed epigastric pain, especially aer eating, palpitation, and pallor. Her ECG and cardiac
enzymes are unremarkable. Her rest of the lab investigations show hemoglobin of 8.9 g/dL, MCV of
73 fl, and a serum ferritin level of 5 mg/L. What is the most likely diagnosis?

a. Chronic gastrointestinal bleeding √


b. Folic acid deficiency
c. Chronic hemolytic anemia
d. Aplastic anemia
e. Anemia of chronic disease

Description

Iron deficiency anemia (IDA) is generally the most common cause of anemia.

It is characterized by microcytic hypochromic anemia.

Its causes include:

Poor intake
Decrease absorption (e.g., celiac disease, gastrectomy
Increased Iron demands (pregnancy, growing adolescent)
Chronic Blood loss (most common cause)

Laboratory investigations will show:

CBC: Low HB, MCV, MCH, and high RDW


Blood film: Poikilocytosis (Variable RBC shapes), Anisocytosis (Variable RBC sizes), and Target
cells
Serology: Low serum ferritin, Iron, Transferrin saturation, High transferrin level, and Total Iron
Binding Capacity (TIBC)

Specific features of IDA:

Brittle nails and nail cracking


Page - 1090
Koilonychia (flattening or concavity of the nails)
Angular stomatitis
Sore tongue
Pica (eating unusual substances like soil, ice, etc.)

Page - 1091
Internal Medicine - Hematology

Question 80/155

Question #80

A 32-year-old male patient complains of fatigue, non-localized abdominal pain, tremor, and
insomnia. His symptoms started 6 weeks ago and are now getting worse. He works in a company
that manufactures bullets. On examination, the patient has a noticeable tremor and a wrist drop.
His lab investigations show microcytic anemia and high ferritin level. Which of the following is most
likely to present in his blood film examination?

a. Target cells
b. Schist cytosis
c. Spherocytosis
d. Teardrop cells
e. Basophilic stippling

‫اﻹﺟﺎﺑﺔ ﻋﲆ اﻟﺼﻔﺤﺔ اﻟﺘﺎﻟﻴﺔ‬

Page - 1092
Internal Medicine - Hematology - Lead Poisoning

Question 80/155

Question #80

A 32-year-old male patient complains of fatigue, non-localized abdominal pain, tremor, and
insomnia. His symptoms started 6 weeks ago and are now getting worse. He works in a company
that manufactures bullets. On examination, the patient has a noticeable tremor and a wrist drop.
His lab investigations show microcytic anemia and high ferritin level. Which of the following is most
likely to present in his blood film examination?

a. Target cells
b. Schist cytosis
c. Spherocytosis
d. Teardrop cells
e. Basophilic stippling √

Description

Basophilic stippling: aggregation of RNA in RBC due to defect in enzyme pyrimidine 5-nucleotidase
caused by lead poisoning

is clinical scenario demonstrated a patient who develops anemia and has occupational exposure
to lead.

Lead poisoning may present with the following features that are typical for this case scenario:

Microcytic anemia
Autonomic neuropathy
Motor neuropathy
Abdominal pain

Consider lead poisoning in a patient with a microcytic anemia with normal or high ferritin
associated with neuropathy and abdominal pain when he is at risk of exposure to lead.

Page - 1093
Internal Medicine - Hematology

Question 81/155

Question #81

A 32-year-old male patient complains of fatigue, non-localized abdominal pain, tremor, and
insomnia. His symptoms started 6 weeks ago and are now getting worse. He works in a company
that manufactures bullets. On examination, the patient has a noticeable tremor and a wrist drop.
His lab investigations show:

Hemoglobin: 9 g/dL

MCV: 65 fl

WBC: 5000 *10^9/L

Ferritin: 880 mic/L

What is the most likely diagnosis?

a. Iron deficiency anemia


b. Anemia of chronic disease
c. Lead poisoning
d. alassemia minor
e. Folate deficiency

‫اﻹﺟﺎﺑﺔ ﻋﲆ اﻟﺼﻔﺤﺔ اﻟﺘﺎﻟﻴﺔ‬

Page - 1094
Internal Medicine - Hematology - Lead Poisoning

Question 81/155

Question #81

A 32-year-old male patient complains of fatigue, non-localized abdominal pain, tremor, and
insomnia. His symptoms started 6 weeks ago and are now getting worse. He works in a company
that manufactures bullets. On examination, the patient has a noticeable tremor and a wrist drop.
His lab investigations show:

Hemoglobin: 9 g/dL

MCV: 65 fl

WBC: 5000 *10^9/L

Ferritin: 880 mic/L

What is the most likely diagnosis?

a. Iron deficiency anemia


b. Anemia of chronic disease
c. Lead poisoning √
d. alassemia minor
e. Folate deficiency

Description

is clinical scenario demonstrated a patient who develops anemia and has occupational exposure
to lead.

Lead poisoning may present with the following features that are typical for this case scenario:

Microcytic anemia
Autonomic neuropathy
Motor neuropathy
Abdominal pain

Consider lead poisoning in a patient with a microcytic anemia with normal or high ferritin
associated with neuropathy and abdominal pain when he is at risk of exposure to lead.

Page - 1095
Internal Medicine - Hematology

Question 82/155

Question #82

A 50-year-old male patient has anemia, thrombocytopenia, leukocytosis, and splenomegaly. His
Philadelphia chromosome is positive. What is the most likely diagnosis?

a. Acute lymphoblastic leukemia


b. Acute myeloid leukemia
c. Chronic lymphoid leukemia
d. Chronic myeloid leukemia
e. Non-Hodgkin lymphoma

‫اﻹﺟﺎﺑﺔ ﻋﲆ اﻟﺼﻔﺤﺔ اﻟﺘﺎﻟﻴﺔ‬

Page - 1096
Internal Medicine - Hematology - Leukemia

Question 82/155

Question #82

A 50-year-old male patient has anemia, thrombocytopenia, leukocytosis, and splenomegaly. His
Philadelphia chromosome is positive. What is the most likely diagnosis?

a. Acute lymphoblastic leukemia


b. Acute myeloid leukemia
c. Chronic lymphoid leukemia
d. Chronic myeloid leukemia √
e. Non-Hodgkin lymphoma

Description

About 90% of CML patients have gene mutation t(9;22) called Philadelphia chromosome → leads to
the production of the BCR-ABL gene, which encodes for BCR-ABL protein with a tyrosine kinase
activity → abnormal cell dierentiation.

Page - 1097
Internal Medicine - Hematology

Question 83/155

Question #83

A 19-year-old male patient with recurrent pneumonia presents to you with tiredness and low
energy. His complete blood count shows hemoglobin of 7 g/dL, WBC of 4*10^9/L, and platelet count
of 82*10^9/L. In addition, blood film shows numerous blast cells. What is the most likely diagnosis?

a. Chronic leukemia
b. Lymphoma
c. Acute leukemia
d. Multiple myeloma
e. HIV infection

‫اﻹﺟﺎﺑﺔ ﻋﲆ اﻟﺼﻔﺤﺔ اﻟﺘﺎﻟﻴﺔ‬

Page - 1098
Internal Medicine - Hematology - Leukemia

Question 83/155

Question #83

A 19-year-old male patient with recurrent pneumonia presents to you with tiredness and low
energy. His complete blood count shows hemoglobin of 7 g/dL, WBC of 4*10^9/L, and platelet count
of 82*10^9/L. In addition, blood film shows numerous blast cells. What is the most likely diagnosis?

a. Chronic leukemia
b. Lymphoma
c. Acute leukemia √
d. Multiple myeloma
e. HIV infection

Description

Leukemia results from the proliferation of Myeloblasts.

On lab investigations, the total WBC count is oen high. However, it may also be normal or even
low.

Bone marrow biopsy is the most accurate (will show > 20% blast cells).

Page - 1099
Internal Medicine - Hematology

Question 84/155

Question #84

A 72-year-old male patient presents with cervical lymph node enlargements and recurrent infection.
His lab investigations show anemia, very high WBC, and smudge cells. What is the most likely
diagnosis?

a. Chronic myeloid leukemia


b. Acute myeloid leukemia
c. Chronic lymphoid leukemia
d. Acute lymphoid leukemia
e. Infectious mononucleosis

‫اﻹﺟﺎﺑﺔ ﻋﲆ اﻟﺼﻔﺤﺔ اﻟﺘﺎﻟﻴﺔ‬

Page - 1100
Internal Medicine - Hematology - Leukemia

Question 84/155

Question #84

A 72-year-old male patient presents with cervical lymph node enlargements and recurrent infection.
His lab investigations show anemia, very high WBC, and smudge cells. What is the most likely
diagnosis?

a. Chronic myeloid leukemia


b. Acute myeloid leukemia
c. Chronic lymphoid leukemia √
d. Acute lymphoid leukemia
e. Infectious mononucleosis

Description

In CLL, there will be:

High WBC count > 20,000 in CLL (predominantly lymphocytes)


50% of patients have hypogammaglobulinemia
Smudge cells: lab artifact (the fragile nucleus is crushed by the coverslip)

Page - 1101
Internal Medicine - Hematology

Question 85/155

Question #85

A 29-year-old male with recurrent infections develops severe bone pain, gum bleeding, and low
energy. Physical examination demonstrates hepatosplenomegaly. His lab investigations show a
hemoglobin level of 8 g/dL, WBC of 103*10^9/L, and numerous blast cells in the blood film and bone
marrow biopsy. What is the most likely diagnosis?

a. Infectious mononucleosis
b. Lymphoma
c. Acute myeloid leukemia
d. Multiple myeloma
e. HIV infection

‫اﻹﺟﺎﺑﺔ ﻋﲆ اﻟﺼﻔﺤﺔ اﻟﺘﺎﻟﻴﺔ‬

Page - 1102
Internal Medicine - Hematology - Leukemia

Question 85/155

Question #85

A 29-year-old male with recurrent infections develops severe bone pain, gum bleeding, and low
energy. Physical examination demonstrates hepatosplenomegaly. His lab investigations show a
hemoglobin level of 8 g/dL, WBC of 103*10^9/L, and numerous blast cells in the blood film and bone
marrow biopsy. What is the most likely diagnosis?

a. Infectious mononucleosis
b. Lymphoma
c. Acute myeloid leukemia √
d. Multiple myeloma
e. HIV infection

Description

High leukocyte count associated with bone pain and gum bleeding is common in AML (gum
bleeding can also occur in ALL).

AML is the most common leukemia in adults. It results from the proliferation of Myeloblasts.

On lab investigations, the total WBC count is oen high. However, it may also be normal or even
low.

Bone marrow biopsy is the most accurate (will show > 20% blast cells).

Page - 1103
Internal Medicine - Hematology

Question 86/155

Question #86

Which of the following is the least likely cause of massive splenomegaly?

a. Chronic malaria
b. CML
c. Myelofibrosis
d. Kala-azar
e. Polycythemia Vera

‫اﻹﺟﺎﺑﺔ ﻋﲆ اﻟﺼﻔﺤﺔ اﻟﺘﺎﻟﻴﺔ‬

Page - 1104
Internal Medicine - Hematology - Leukemia

Question 86/155

Question #86

Which of the following is the least likely cause of massive splenomegaly?

a. Chronic malaria
b. CML
c. Myelofibrosis
d. Kala-azar
e. Polycythemia Vera √

Description

Polycythemia Vera presents with mild splenomegaly (not massive)

Causes of massive splenomegaly are:

CML
Myelofibrosis
Lymphoma
Kala-azar (visceral leishmaniasis)
Malaria (chronic)
Gaucher’s syndrome

Page - 1105
Internal Medicine - Hematology

Question 87/155

Question #87

A 54-year-old male patient complains of weight loss, fatigue, and abdominal discomfort. Physical
examination demonstrates an enlarged spleen. Lab investigations show hemoglobin of 10g/dL, WBC
170*10^9/L (88% neutrophils), and the Philadelphia chromosome is positive. What is the most likely
diagnosis?

a. Chronic lymphoblastic leukemia


b. Chronic myeloid leukemia
c. Acute myeloid leukemia
d. Hodgkin lymphoma
e. Multiple myeloma

‫اﻹﺟﺎﺑﺔ ﻋﲆ اﻟﺼﻔﺤﺔ اﻟﺘﺎﻟﻴﺔ‬

Page - 1106
Internal Medicine - Hematology - Leukemia

Question 87/155

Question #87

A 54-year-old male patient complains of weight loss, fatigue, and abdominal discomfort. Physical
examination demonstrates an enlarged spleen. Lab investigations show hemoglobin of 10g/dL, WBC
170*10^9/L (88% neutrophils), and the Philadelphia chromosome is positive. What is the most likely
diagnosis?

a. Chronic lymphoblastic leukemia


b. Chronic myeloid leukemia √
c. Acute myeloid leukemia
d. Hodgkin lymphoma
e. Multiple myeloma

Description

e patient has very high WBC with a predominance of neutrophils and features suggestive of
leukemia. is scenario strongly suggests CML as the most likely diagnosis.

About 90% of CML patients have gene mutation t(9;22) called Philadelphia chromosome → leads to
the production of the BCR-ABL gene, which encodes for BCR-ABL protein with a tyrosine kinase
activity → abnormal cell dierentiation.

Page - 1107
Internal Medicine - Hematology

Question 88/155

Question #88

Which of the following is the most characteristic of acute promyelocytic leukemia?

a. Reduced leukocyte alkaline phosphatase score


b. Translocation (9:22)
c. Disseminated intravascular coagulation
d. Positive nuclear TdT staining
e. It is the M2 type of AML

‫اﻹﺟﺎﺑﺔ ﻋﲆ اﻟﺼﻔﺤﺔ اﻟﺘﺎﻟﻴﺔ‬

Page - 1108
Internal Medicine - Hematology - Leukemia

Question 88/155

Question #88

Which of the following is the most characteristic of acute promyelocytic leukemia?

a. Reduced leukocyte alkaline phosphatase score


b. Translocation (9:22)
c. Disseminated intravascular coagulation √
d. Positive nuclear TdT staining
e. It is the M2 type of AML

Description

Acute Promyelocytic Leukemia (APML):

It is the (M3) subtype of AML


Due to t(15:17) mutation
A high number of Auer rods and increased risk of coagulation
Can present with DIC
Treatment by vitamin A derivative (ATRA)

Page - 1109
Internal Medicine - Hematology

Question 89/155

Question #89

A patient is suspected of having Hodgkin’s lymphoma. What is the most common initial clinical
feature of this condition?

a. Fever of unknown origin


b. Painless lymphadenopathy
c. Night sweats
d. Loss of weight
e. Anorexia

‫اﻹﺟﺎﺑﺔ ﻋﲆ اﻟﺼﻔﺤﺔ اﻟﺘﺎﻟﻴﺔ‬

Page - 1110
Internal Medicine - Hematology - Lymphoma

Question 89/155

Question #89

A patient is suspected of having Hodgkin’s lymphoma. What is the most common initial clinical
feature of this condition?

a. Fever of unknown origin


b. Painless lymphadenopathy √
c. Night sweats
d. Loss of weight
e. Anorexia

Description

e most common presenting symptom of Hodgkin’s lymphoma is painless lymphadenopathy.

B symptoms present in 30% of patients with Hodgkin’s lymphoma

Page - 1111
Internal Medicine - Hematology

Question 90/155

Question #90

A 60-year-old male patient complained of night sweats, weight loss, and anorexia for 6 months. on
examination, you found a palpable rubbery neck mass. Excisional LN biopsy demonstrates Hodgkin’s
lymphoma. What type of cells are most likely to be associated with this condition?

a. Plasma cells
b. Auer rods containing blast cells
c. Blast cells with positive nuclear TdT staining
d. Reed Sternberg cells
e. T-Lymphocytes

‫اﻹﺟﺎﺑﺔ ﻋﲆ اﻟﺼﻔﺤﺔ اﻟﺘﺎﻟﻴﺔ‬

Page - 1112
Internal Medicine - Hematology - Lymphoma

Question 90/155

Question #90

A 60-year-old male patient complained of night sweats, weight loss, and anorexia for 6 months. on
examination, you found a palpable rubbery neck mass. Excisional LN biopsy demonstrates Hodgkin’s
lymphoma. What type of cells are most likely to be associated with this condition?

a. Plasma cells
b. Auer rods containing blast cells
c. Blast cells with positive nuclear TdT staining
d. Reed Sternberg cells √
e. T-Lymphocytes

Description

Histological evaluation of lymph node biopsy demonstrating Reed Sternberg cells is the most
accurate in confirming the diagnosis of Hodgkin’s lymphoma.

Plasma cells > 10% in bone marrow is characteristic of Multiple Myeloma

Blast cells with Auer rods in the bone marrow of > 20% are characteristic of acute myeloid leukemia

Positive TdT staining is present in lymphoblasts, so it is found in acute lymphoblastic leukemia.

Page - 1113
Internal Medicine - Hematology

Question 91/155

Question #91

A 19-year-old female complains of non-tender neck mass, night sweats, weight loss, splenomegaly,
and fever. What is the most likely diagnosis?

a. Tuberculosis
b. Lipoma
c. Toxoplasmosis
d. yroglossal cyst
e. Hodgkin lymphoma

‫اﻹﺟﺎﺑﺔ ﻋﲆ اﻟﺼﻔﺤﺔ اﻟﺘﺎﻟﻴﺔ‬

Page - 1114
Internal Medicine - Hematology - Lymphoma

Question 91/155

Question #91

A 19-year-old female complains of non-tender neck mass, night sweats, weight loss, splenomegaly,
and fever. What is the most likely diagnosis?

a. Tuberculosis
b. Lipoma
c. Toxoplasmosis
d. yroglossal cyst
e. Hodgkin lymphoma √

Description

e combination of cervical adenopathy and splenomegaly makes lymphoma the most likely in this
patient.

Despite it can also cause the same picture, in toxoplasmosis, weight loss is less prominent

Page - 1115
Internal Medicine - Hematology

Question 92/155

Question #92

A 22-year-old male patient presents with painless, rubbery, and non-tender enlarged cervical lymph
nodes and hepatosplenomegaly. His lab investigations show anemia, leukocytosis, and normal
platelet count. Which of the following is the most important investigation to establish or exclude
the diagnosis of Hodgkin’s lymphoma?

a. Chest, Abdominal, and pelvic CT scans


b. Bone marrow aspiration
c. Excisional biopsy of the lymph nodes
d. Leukocyte alkaline phosphatase score
e. ESR and CRP

‫اﻹﺟﺎﺑﺔ ﻋﲆ اﻟﺼﻔﺤﺔ اﻟﺘﺎﻟﻴﺔ‬

Page - 1116
Internal Medicine - Hematology - Lymphoma

Question 92/155

Question #92

A 22-year-old male patient presents with painless, rubbery, and non-tender enlarged cervical lymph
nodes and hepatosplenomegaly. His lab investigations show anemia, leukocytosis, and normal
platelet count. Which of the following is the most important investigation to establish or exclude
the diagnosis of Hodgkin’s lymphoma?

a. Chest, Abdominal, and pelvic CT scans


b. Bone marrow aspiration
c. Excisional biopsy of the lymph nodes √
d. Leukocyte alkaline phosphatase score
e. ESR and CRP

Description

Histological evaluation of lymph node biopsy demonstrating Reed Sternberg cells is the most
accurate in confirming the diagnosis of Hodgkin’s lymphoma.

CT scan is used for staging of lymphoma according to Ann Arbor staging system.

Leukocyte alkaline phosphatase score is used to dierentiate between leukemia and leukemoid
reaction.

ESR and CRP are acute phase reactants and are not specific investigations.

Page - 1117
Internal Medicine - Hematology

Question 93/155

Question #93

A 60-year-old male patient complained of night sweats, weight loss, and anorexia for 6 months. on
examination, you found a palpable rubbery neck mass. Excisional LN biopsy demonstrates Reed
Sternberg cells. What is the most likely diagnosis?

a. Hodgkin’s lymphoma
b. Burkitt lymphoma
c. Immunoblastic lymphomas
d. Chronic lymphoid leukemia
e. Infectious mononucleosis

‫اﻹﺟﺎﺑﺔ ﻋﲆ اﻟﺼﻔﺤﺔ اﻟﺘﺎﻟﻴﺔ‬

Page - 1118
Internal Medicine - Hematology - Lymphoma

Question 93/155

Question #93

A 60-year-old male patient complained of night sweats, weight loss, and anorexia for 6 months. on
examination, you found a palpable rubbery neck mass. Excisional LN biopsy demonstrates Reed
Sternberg cells. What is the most likely diagnosis?

a. Hodgkin’s lymphoma √
b. Burkitt lymphoma
c. Immunoblastic lymphomas
d. Chronic lymphoid leukemia
e. Infectious mononucleosis

Description

Histological evaluation of lymph node biopsy demonstrating Reed Sternberg cells is the most
accurate in confirming the diagnosis of Hodgkin’s lymphoma.

Burkitt lymphoma and Immunoblastic lymphomas are non-Hodgkin’s lymphoma types that do not
present with Reed Sternberg cells on LN biopsy.

Page - 1119
Internal Medicine - Hematology

Question 94/155

Question #94

A 22-year-old male patient complains of shortness of breath, and a mediastinal mass is shown on his
chest x-ray. His physical examination shows cervical lymph node enlargement. What is the most
appropriate investigation?

a. Bronchoalveolar lavage
b. Barium swallow
c. Lymph node biopsy
d. CT Pulmonary Angiography
e. Laryngoscopy

‫اﻹﺟﺎﺑﺔ ﻋﲆ اﻟﺼﻔﺤﺔ اﻟﺘﺎﻟﻴﺔ‬

Page - 1120
Internal Medicine - Hematology - Lymphoma

Question 94/155

Question #94

A 22-year-old male patient complains of shortness of breath, and a mediastinal mass is shown on his
chest x-ray. His physical examination shows cervical lymph node enlargement. What is the most
appropriate investigation?

a. Bronchoalveolar lavage
b. Barium swallow
c. Lymph node biopsy √
d. CT Pulmonary Angiography
e. Laryngoscopy

Description

is patient is most likely to have lymphoma.

A lymph node biopsy is the most accurate test to confirm or exclude the diagnosis.

Page - 1121
Internal Medicine - Hematology

Question 95/155

Question #95

A 40-year-old male patient complains of general weakness, cervical adenopathy, and splenomegaly.
In addition, he lost 15 kilograms in the past 6 weeks. What is the most likely diagnosis?

a. Pneumococcal infection
b. H influenza infection
c. Lymphoma
d. Tuberculosis
e. Toxoplasmosis

‫اﻹﺟﺎﺑﺔ ﻋﲆ اﻟﺼﻔﺤﺔ اﻟﺘﺎﻟﻴﺔ‬

Page - 1122
Internal Medicine - Hematology - Lymphoma

Question 95/155

Question #95

A 40-year-old male patient complains of general weakness, cervical adenopathy, and splenomegaly.
In addition, he lost 15 kilograms in the past 6 weeks. What is the most likely diagnosis?

a. Pneumococcal infection
b. H influenza infection
c. Lymphoma √
d. Tuberculosis
e. Toxoplasmosis

Description

e combination of cervical adenopathy and splenomegaly makes lymphoma the most likely in this
patient.

Despite it can also cause the same picture, in toxoplasmosis, weight loss is less prominent

Page - 1123
Internal Medicine - Hematology

Question 96/155

Question #96

A 45-year-old male patient presents with signs of anemia, easy bruising, and peripheral numbness.
CBC shows Hemoglobin: 9.5 g/dl, Platelets 83*10^9/L, WBCs19*10^9/L, and MCV 125 fl. What would
be the most sensitive investigation of the most likely diagnosis?

a. Bone marrow aspiration


b. Serum B12 level
c. Methylmalonic acid level
d. Low haptoglobin
e. Low homocysteine

‫اﻹﺟﺎﺑﺔ ﻋﲆ اﻟﺼﻔﺤﺔ اﻟﺘﺎﻟﻴﺔ‬

Page - 1124
Internal Medicine - Hematology - Macrocytic megaloblastic anemia

Question 96/155

Question #96

A 45-year-old male patient presents with signs of anemia, easy bruising, and peripheral numbness.
CBC shows Hemoglobin: 9.5 g/dl, Platelets 83*10^9/L, WBCs19*10^9/L, and MCV 125 fl. What would
be the most sensitive investigation of the most likely diagnosis?

a. Bone marrow aspiration


b. Serum B12 level
c. Methylmalonic acid level √
d. Low haptoglobin
e. Low homocysteine

Description

Vitamin B12 is a cofactor in synthesizing methionine and succinyl coenzyme A, and vitamin B12
deficiency leads to the accumulation of methylmalonic acid and homocysteine, the precursors of
these compounds. An elevated level of these substances is, therefore, more sensitive than a low
vitamin B12 level for vitamin B12 deficiency

Page - 1125
Internal Medicine - Hematology

Question 97/155

Question #97

An alcoholic 62-year-old male complains of fatigue. His blood tests show hemoglobin of 8g/dL, and
his MCV is 108 fl. Which of the following is the most likely diagnosis?

a. alassemia
b. Iron deficiency anemia
c. Folic acid deficiency anemia
d. Anemia of chronic disease
e. Sickle cell anemia

‫اﻹﺟﺎﺑﺔ ﻋﲆ اﻟﺼﻔﺤﺔ اﻟﺘﺎﻟﻴﺔ‬

Page - 1126
Internal Medicine - Hematology - Macrocytic megaloblastic anemia

Question 97/155

Question #97

An alcoholic 62-year-old male complains of fatigue. His blood tests show hemoglobin of 8g/dL, and
his MCV is 108 fl. Which of the following is the most likely diagnosis?

a. alassemia
b. Iron deficiency anemia
c. Folic acid deficiency anemia √
d. Anemia of chronic disease
e. Sickle cell anemia

Description

is is a case of macrocytic anemia. B12 and folate deficiency most commonly cause it.

Since the patient is alcoholic, folic acid deficiency is more common to cause this condition.

Folate deficiency is the most common cause of megaloblastic anemia in chronic alcoholics

Page - 1127
Internal Medicine - Hematology

Question 98/155

Question #98

You are performing an anemia workup for a 54-year-old male patient. His blood film demonstrates
oval macrocytes, Hypersegmented neutrophils, and reduced platelet count. What is the most likely
diagnosis?

a. Iron deficiency anemia


b. alassemia
c. B12 deficiency
d. Sickle cell anemia
e. Chronic liver disease

‫اﻹﺟﺎﺑﺔ ﻋﲆ اﻟﺼﻔﺤﺔ اﻟﺘﺎﻟﻴﺔ‬

Page - 1128
Internal Medicine - Hematology - Macrocytic megaloblastic anemia

Question 98/155

Question #98

You are performing an anemia workup for a 54-year-old male patient. His blood film demonstrates
oval macrocytes, Hypersegmented neutrophils, and reduced platelet count. What is the most likely
diagnosis?

a. Iron deficiency anemia


b. alassemia
c. B12 deficiency √
d. Sickle cell anemia
e. Chronic liver disease

Description

is is a case of macrocytic anemia, hypersegmented neutrophils, and reduced platelets. ese
findings strongly suggest B12 deficiency.

In B12 deficiency, the involvement of neutrophils and platelet may present and can cause:

Macrocytic anemia (Normocytic if co-existent iron deficiency or thalassemia)

Pancytopenia

Hyper-segmented nuclei of neutrophils (6 lobes)

Hypercellularity and megaloblasts in bone marrow

Page - 1129
Internal Medicine - Hematology

Question 99/155

Question #99

A 45-year-old female refuses to take injectable B12 to treat her pernicious anemia. Which of the
following is the main reason her disease does not respond to oral B12?

a. Bacteria in the ileum prevent B12 absorption


b. Chronic pancreatic insuciency
c. Intrinsic factor deficiency
d. Increased gastric acidity
e. Reduced gastric acidity

‫اﻹﺟﺎﺑﺔ ﻋﲆ اﻟﺼﻔﺤﺔ اﻟﺘﺎﻟﻴﺔ‬

Page - 1130
Internal Medicine - Hematology - Macrocytic megaloblastic anemia

Question 99/155

Question #99

A 45-year-old female refuses to take injectable B12 to treat her pernicious anemia. Which of the
following is the main reason her disease does not respond to oral B12?

a. Bacteria in the ileum prevent B12 absorption


b. Chronic pancreatic insuciency
c. Intrinsic factor deficiency √
d. Increased gastric acidity
e. Reduced gastric acidity

Description

Pernicious anemia is an autoimmune disease characterized by atrophy of gastric mucosa and failure
of the secretion of the intrinsic factor. is will lead to B12 malabsorption and megaloblastic anemia

Intrinsic factor antibodies in 50% or Anti-parietal cell antibodies can be detected

In pernicious anemia, the absence of the intrinsic factor is the primary mechanism for developing
anemia. In addition, it is the main cause that oral B12 will not be absorbed by the traditional intrinsic
factor terminal ilium pathway.

e non-intrinsic factor-terminal ilium pathway needs a high dose of oral vitamin B12 (1000
micrograms) to absorb B12.

Page - 1131
Internal Medicine - Hematology

Question 100/155

Question #100

A 60-year-old female suers from fatigue and confusion. Her husband reports gradual confusion
over the past 6 months. Her lab investigations demonstrate macrocytic megaloblastic anemia, low
b12, and low folic acid levels. Which is the most appropriate course of treatment for this patient?

a. Start oral folate, then injectable B12 aer folic acid returns to normal
b. Start injectable B12, then oral folic acid aer B12 returns to normal
c. Start oral folic acid only
d. Start injectable B12 only
e. Refer to a hematologist

‫اﻹﺟﺎﺑﺔ ﻋﲆ اﻟﺼﻔﺤﺔ اﻟﺘﺎﻟﻴﺔ‬

Page - 1132
Internal Medicine - Hematology - Macrocytic megaloblastic anemia

Question 100/155

Question #100

A 60-year-old female suers from fatigue and confusion. Her husband reports gradual confusion
over the past 6 months. Her lab investigations demonstrate macrocytic megaloblastic anemia, low
b12, and low folic acid levels. Which is the most appropriate course of treatment for this patient?

a. Start oral folate, then injectable B12 aer folic acid returns to normal
b. Start injectable B12, then oral folic acid aer B12 returns to normal √
c. Start oral folic acid only
d. Start injectable B12 only
e. Refer to a hematologist

Description

Replacement of folate in a patient with B12 deficiency before correcting B12 will aggravate
neuropathy.

You should correct the B12 level first, then provide folic acid to avoid precipitating subacute
combined degeneration of the cord.

Page - 1133
Internal Medicine - Hematology

Question 101/155

Question #101

A 22-year-old female decided to be a strict vegan. She watches the labels closely of everything before
she eats. Assuming her B12 level and methylmalonic acid today are normal, aer how long is she
expected to develop B12 deficiency?

a. Within days
b. Within months
c. Aer 1 year
d. Aer 2 years
e. Aer 3 years

‫اﻹﺟﺎﺑﺔ ﻋﲆ اﻟﺼﻔﺤﺔ اﻟﺘﺎﻟﻴﺔ‬

Page - 1134
Internal Medicine - Hematology - Macrocytic megaloblastic anemia

Question 101/155

Question #101

A 22-year-old female decided to be a strict vegan. She watches the labels closely of everything before
she eats. Assuming her B12 level and methylmalonic acid today are normal, aer how long is she
expected to develop B12 deficiency?

a. Within days
b. Within months
c. Aer 1 year
d. Aer 2 years
e. Aer 3 years √

Description

Vitamin B12 is naturally present in foods of animal origin.

Vegetarian and vegan persons are at risk of B12 deficiency. However, the liver stores B12 enough for 3
years; therefore, B12 deficiency takes 3 years to manifest

Page - 1135
Internal Medicine - Hematology

Question 102/155

Question #102

A strict vegan 23-year-old female complains of fatigue and dyspnea. Her complete blood picture
shows hemoglobin of 9 g/dL, platelet count of 80 *10^9/l, WBC 2*10^9, and MCV of 107 fl. Which is
the most appropriate next step in managing this patient?

a. Iron study
b. Serum B12 level
c. Bone marrow aspiration
d. Hb electrophoresis
e. Coombs test

‫اﻹﺟﺎﺑﺔ ﻋﲆ اﻟﺼﻔﺤﺔ اﻟﺘﺎﻟﻴﺔ‬

Page - 1136
Internal Medicine - Hematology - Macrocytic megaloblastic anemia

Question 102/155

Question #102

A strict vegan 23-year-old female complains of fatigue and dyspnea. Her complete blood picture
shows hemoglobin of 9 g/dL, platelet count of 80 *10^9/l, WBC 2*10^9, and MCV of 107 fl. Which is
the most appropriate next step in managing this patient?

a. Iron study
b. Serum B12 level √
c. Bone marrow aspiration
d. Hb electrophoresis
e. Coombs test

Description

Pancytopenia and high MCV strongly suggest the presence of B12 deficiency

Vegan patients are at risk of B12 deficiency because vitamin B12 is naturally present in foods of
animal origin.

Involvement of neutrophils and platelet may present and can cause:

Macrocytic anemia (Normocytic if co-existent iron deficiency or thalassemia)


Pancytopenia
Hyper-segmented nuclei of neutrophils (6 lobes)
Hypercellularity and megaloblasts in bone marrow

Page - 1137
Internal Medicine - Hematology

Question 103/155

Question #103

A 49-year-old male patient complains of fatigue and pallor. His history is significant for gastrectomy
1 year ago due to recurrent peptic ulcer disease. Lab investigations show hemoglobin of 8 g/dL, WBC
of 8*10^9/L, platelet of 160*10^9/L, and MCV of 121 fl. What is the most likely finding in his blood
film?

a. Hypochromic microcytic anemia


b. Numerous blast cells
c. Schistocytes
d. Target cells
e. Hypersegmented neutrophils

‫اﻹﺟﺎﺑﺔ ﻋﲆ اﻟﺼﻔﺤﺔ اﻟﺘﺎﻟﻴﺔ‬

Page - 1138
Internal Medicine - Hematology - Macrocytic megaloblastic anemia

Question 103/155

Question #103

A 49-year-old male patient complains of fatigue and pallor. His history is significant for gastrectomy
1 year ago due to recurrent peptic ulcer disease. Lab investigations show hemoglobin of 8 g/dL, WBC
of 8*10^9/L, platelet of 160*10^9/L, and MCV of 121 fl. What is the most likely finding in his blood
film?

a. Hypochromic microcytic anemia


b. Numerous blast cells
c. Schistocytes
d. Target cells
e. Hypersegmented neutrophils √

Description

Given the fact that B12 is absorbed by the intrinsic factor terminal ileum pathway, patients with
gastrectomy or terminal ileum resection are at risk of B12 malabsorption and B12 deficiency.

B12 deficiency anemia in blood film will show macrocytosis, hypersegmented neutrophils, and
reduced platelet number

Page - 1139
Internal Medicine - Hematology

Question 104/155

Question #104

A 22-year-old male patient presents with fatigue and palpitations. His history is significant for
epilepsy, for which he takes phenytoin. His lab investigations show hemoglobin of 9g/dL, the mean
corpuscular volume of 110 fl. Which of the following is the most likely diagnosis?

a. Sickle cell anemia


b. Folic acid deficiency
c. alassemia
d. Liver disease
e. Iron deficiency anemia

‫اﻹﺟﺎﺑﺔ ﻋﲆ اﻟﺼﻔﺤﺔ اﻟﺘﺎﻟﻴﺔ‬

Page - 1140
Internal Medicine - Hematology - Macrocytic megaloblastic anemia

Question 104/155

Question #104

A 22-year-old male patient presents with fatigue and palpitations. His history is significant for
epilepsy, for which he takes phenytoin. His lab investigations show hemoglobin of 9g/dL, the mean
corpuscular volume of 110 fl. Which of the following is the most likely diagnosis?

a. Sickle cell anemia


b. Folic acid deficiency √
c. alassemia
d. Liver disease
e. Iron deficiency anemia

Description

Phenytoin intake here is the crucial point.

Phenytoin reduces folic acid levels leading to folic acid deficiency anemia

Folic acid deficiency anemia is characterized by macrocytic megaloblastic anemia.

Patients taking phenytoin should receive folic acid supplements to reduce the risk of folate
deficiency.

Pregnant women taking phenytoin should receive folic acid because they are at risk of folate
deficiency and neural tube defects.

Page - 1141
Internal Medicine - Hematology

Question 105/155

Question #105

A 50-year-old female patient develops fatigue and palpitations. Her blood count demonstrates
macrocytic anemia, and her physical examination is significant for pallor and vitiligo. Which of the
following is the most likely diagnosis?

a. Folic acid deficiency


b. Dietary B12 deficiency
c. Pernicious anemia
d. Anemia of chronic disease
e. Liver disease

‫اﻹﺟﺎﺑﺔ ﻋﲆ اﻟﺼﻔﺤﺔ اﻟﺘﺎﻟﻴﺔ‬

Page - 1142
Internal Medicine - Hematology - Macrocytic megaloblastic anemia

Question 105/155

Question #105

A 50-year-old female patient develops fatigue and palpitations. Her blood count demonstrates
macrocytic anemia, and her physical examination is significant for pallor and vitiligo. Which of the
following is the most likely diagnosis?

a. Folic acid deficiency


b. Dietary B12 deficiency
c. Pernicious anemia √
d. Anemia of chronic disease
e. Liver disease

Description

is is typical for pernicious anemia.

Look for a female patient with associated autoimmune disorders and features of B12 deficiency.

Female patients are more likely to have autoimmune diseases

Autoimmune diseases are more likely to occur in association with other autoimmune diseases.

Page - 1143
Internal Medicine - Hematology

Question 106/155

Question #106

A 32-year-old vegan female started to have paresthesia and fatigue for 3 months. Her physical
examination reveals angular stomatitis and a sore red tongue. Laboratory investigations were done,
and they demonstrated anemia. Which of the following is the most likely finding in blood film?

a. Numerous blast cells


b. Macrocytic RBCs
c. Microcytic hypochromic RBCs
d. Schistocytes
e. Spherocytes

‫اﻹﺟﺎﺑﺔ ﻋﲆ اﻟﺼﻔﺤﺔ اﻟﺘﺎﻟﻴﺔ‬

Page - 1144
Internal Medicine - Hematology - Macrocytic megaloblastic anemia

Question 106/155

Question #106

A 32-year-old vegan female started to have paresthesia and fatigue for 3 months. Her physical
examination reveals angular stomatitis and a sore red tongue. Laboratory investigations were done,
and they demonstrated anemia. Which of the following is the most likely finding in blood film?

a. Numerous blast cells


b. Macrocytic RBCs √
c. Microcytic hypochromic RBCs
d. Schistocytes
e. Spherocytes

Description

Because B12 is acquired from animal diet sources, vegan people are at risk of B12 deficiency.

B12 deficiency anemia in blood film will show macrocytosis, hypersegmented neutrophils, and
reduced platelet number

Note that angular stomatitis is found in iron deficiency anemia. However, it sometimes manifests in
B12 deficiency anemia.

e tongue’s appearance in vitamin B12 deficiency is described as “beefy” or “fiery red and sore”.

Page - 1145
Internal Medicine - Hematology

Question 107/155

Question #107

A 32-year-old female started to have paresthesia and fatigue for 3 months. Her medical history is
significant for repeated attacks of pancreatitis. Laboratory investigations were done, and they
demonstrated anemia. What is the most appropriate management?

a. Provide vitamin B12


b. Provide vitamin B1
c. Provide vitamin B6
d. Provide folic acid
e. Refer to a neurologist

‫اﻹﺟﺎﺑﺔ ﻋﲆ اﻟﺼﻔﺤﺔ اﻟﺘﺎﻟﻴﺔ‬

Page - 1146
Internal Medicine - Hematology - Macrocytic megaloblastic anemia

Question 107/155

Question #107

A 32-year-old female started to have paresthesia and fatigue for 3 months. Her medical history is
significant for repeated attacks of pancreatitis. Laboratory investigations were done, and they
demonstrated anemia. What is the most appropriate management?

a. Provide vitamin B12 √


b. Provide vitamin B1
c. Provide vitamin B6
d. Provide folic acid
e. Refer to a neurologist

Description

B12 deficiency can be caused by pancreatic insuciency since pancreatic secretions usually free B12
from carrier proteins.

e liver stores B12 enough for 3 years (B12 deficiency takes years to manifest)

B12 deficiency leads to:

Neurological disease (Gloves and stockings neuropathy, subacute combined degeneration of the
spinal cord – ataxia, dementia, optic atrophy)

Hematological disease (Megaloblastic anemia)

Page - 1147
Internal Medicine - Hematology

Question 108/155

Question #108

A 49-year-old male patient complains of fatigue and pallor. His history is significant for terminal
ilium resection 1 year ago. Lab investigations show hemoglobin of 8 g/dL, WBC of 8*10^9/L, platelet
of 160*10^9/L, and MCV of 121 fl. What is the most likely finding in his blood film?

a. Hypochromic microcytic anemia


b. Numerous blast cells
c. Schistocytes
d. Target cells
e. Hypersegmented neutrophils

‫اﻹﺟﺎﺑﺔ ﻋﲆ اﻟﺼﻔﺤﺔ اﻟﺘﺎﻟﻴﺔ‬

Page - 1148
Internal Medicine - Hematology - Macrocytic megaloblastic anemia

Question 108/155

Question #108

A 49-year-old male patient complains of fatigue and pallor. His history is significant for terminal
ilium resection 1 year ago. Lab investigations show hemoglobin of 8 g/dL, WBC of 8*10^9/L, platelet
of 160*10^9/L, and MCV of 121 fl. What is the most likely finding in his blood film?

a. Hypochromic microcytic anemia


b. Numerous blast cells
c. Schistocytes
d. Target cells
e. Hypersegmented neutrophils √

Description

Given the fact that B12 is absorbed by the intrinsic factor terminal ileum pathway, patients with
gastrectomy or terminal ileum resection are at risk of B12 malabsorption and B12 deficiency.

B12 deficiency anemia in blood film will show macrocytosis, hypersegmented neutrophils, and
reduced platelet number

Page - 1149
Internal Medicine - Hematology

Question 109/155

Question #109

A 29-year-old female with hypothyroidism develops pins and needles sensation in her hands,
fatigue, and palpitation. In addition, her complete blood count shows macrocytic anemia. Which of
the following is the most likely cause of her anemia?

a. Pernicious anemia
b. Hypothyroidism
c. Iron deficiency anemia
d. Anemia of chronic disease
e. Crohn’s disease

‫اﻹﺟﺎﺑﺔ ﻋﲆ اﻟﺼﻔﺤﺔ اﻟﺘﺎﻟﻴﺔ‬

Page - 1150
Internal Medicine - Hematology - Macrocytic megaloblastic anemia

Question 109/155

Question #109

A 29-year-old female with hypothyroidism develops pins and needles sensation in her hands,
fatigue, and palpitation. In addition, her complete blood count shows macrocytic anemia. Which of
the following is the most likely cause of her anemia?

a. Pernicious anemia √
b. Hypothyroidism
c. Iron deficiency anemia
d. Anemia of chronic disease
e. Crohn’s disease

Description

is is typical for pernicious anemia.

Look for a female patient with associated autoimmune disorders and features of B12 deficiency.

Female patients are more likely to have autoimmune diseases

Autoimmune diseases are more likely to occur in association with other autoimmune diseases

Page - 1151
Internal Medicine - Hematology

Question 110/155

Question #110

A 54-year-old male patient complains of fatigue and palpitation. His history is significant
rheumatoid arthritis which is controlled by medications. However, his lab investigations show
macrocytic megaloblastic anemia. Which of the following is the most likely responsible for his
anemia?

a. Non-steroidal anti-inflammatory drugs (NSAIDs)


b. Steroids
c. e chronic disease
d. Methotrexate
e. Chronic blood loss

‫اﻹﺟﺎﺑﺔ ﻋﲆ اﻟﺼﻔﺤﺔ اﻟﺘﺎﻟﻴﺔ‬

Page - 1152
Internal Medicine - Hematology - Macrocytic megaloblastic anemia

Question 110/155

Question #110

A 54-year-old male patient complains of fatigue and palpitation. His history is significant
rheumatoid arthritis which is controlled by medications. However, his lab investigations show
macrocytic megaloblastic anemia. Which of the following is the most likely responsible for his
anemia?

a. Non-steroidal anti-inflammatory drugs (NSAIDs)


b. Steroids
c. e chronic disease
d. Methotrexate √
e. Chronic blood loss

Description

Methotrexate is a folate antagonist used in the treatment of rheumatoid arthritis. is drug can
cause folic acid deficiency, manifesting as macrocytic megaloblastic anemia.

NSAIDs may cause a peptic ulcer or gastritis with chronic bleeding and iron deficiency anemia with
low MCV (which is not the case here)

Rheumatoid arthritis, a chronic disease, may cause anemia of chronic disease, but it is associated
with normal MCV in 70% and low MCV in 30% of cases.

Page - 1153
Internal Medicine - Hematology

Question 111/155

Question #111

A 64-year-old male patient presents with lower back pain and fatigue. His symptoms have been
progressive over the past 6 months. His lab investigations show mild anemia, elevated urea and
creatinine, and hypercalcemia. What is the most accurate test to confirm the most likely diagnosis?

a. Blood film
b. Bone marrow biopsy
c. Full blood picture
d. Urinary protein level
e. Skull x-ray

‫اﻹﺟﺎﺑﺔ ﻋﲆ اﻟﺼﻔﺤﺔ اﻟﺘﺎﻟﻴﺔ‬

Page - 1154
Internal Medicine - Hematology - Multiple Myeloma (MM)

Question 111/155

Question #111

A 64-year-old male patient presents with lower back pain and fatigue. His symptoms have been
progressive over the past 6 months. His lab investigations show mild anemia, elevated urea and
creatinine, and hypercalcemia. What is the most accurate test to confirm the most likely diagnosis?

a. Blood film
b. Bone marrow biopsy √
c. Full blood picture
d. Urinary protein level
e. Skull x-ray

Description

e most likely diagnosis here is Multiple Myeloma.

A bone marrow biopsy showing >10% plasma cells is the most accurate test.

ese abnormal Immunoglobulins in Multiple Myeloma (MM) lead to:

Infection
Renal deposition and renal failure
Bence-Jones proteinuria (detected by urine immunoelectrophoresis)
Hyperviscosity syndrome
Activation of osteolytic activity of the bone, which leads to hypercalcemia

Page - 1155
Internal Medicine - Hematology

Question 112/155

Question #112

A 66-year-old male patient presents with polydipsia and polyuria, and His lab investigations show
hypercalcemia. On further questioning, the patient has back pain and fatigue for 6 weeks. e rest of
the lab investigations demonstrate hemoglobin of 9g/dL, alkaline phosphatase of 110 u/L, serum
creatinine of 2.3mg/dL, and ESR of 90mm/Hour. Which of the following are the most likely cells
found elevated in bone marrow biopsy?

a. Myeloblast cells
b. Lymphoblast cells
c. Megakaryocytes
d. Plasma cells
e. Bence-Johnse protein

‫اﻹﺟﺎﺑﺔ ﻋﲆ اﻟﺼﻔﺤﺔ اﻟﺘﺎﻟﻴﺔ‬

Page - 1156
Internal Medicine - Hematology - Multiple Myeloma (MM)

Question 112/155

Question #112

A 66-year-old male patient presents with polydipsia and polyuria, and His lab investigations show
hypercalcemia. On further questioning, the patient has back pain and fatigue for 6 weeks. e rest of
the lab investigations demonstrate hemoglobin of 9g/dL, alkaline phosphatase of 110 u/L, serum
creatinine of 2.3mg/dL, and ESR of 90mm/Hour. Which of the following are the most likely cells
found elevated in bone marrow biopsy?

a. Myeloblast cells
b. Lymphoblast cells
c. Megakaryocytes
d. Plasma cells √
e. Bence-Johnse protein

Description

is is a classic presentation of Multiple Myeloma (MM).

A bone marrow biopsy showing >10% plasma cells is the most accurate test.

ese abnormal Immunoglobulins in Multiple Myeloma (MM) lead to:

Infection
Renal deposition and renal failure
Bence-Jones proteinuria (detected by urine immunoelectrophoresis)
Hyperviscosity syndrome
Activation of osteolytic activity of the bone, which leads to hypercalcemia

Elevated myeloblasts in the bone marrow are typical for myeloid leukemia

Elevated lymphoblasts are typical for lymphoid leukemia

Megakaryocytes elevation is typical for essential thrombocythemia

Bence-Johnse proteins are not cells; they are paraproteins detected in the plasma or urine in MM.

Page - 1157
Internal Medicine - Hematology

Question 113/155

Question #113

A 64-year-old male patient presents with lower back pain and fatigue. His symptoms have been
progressive over the past 6 months. His lab investigations show mild anemia, elevated urea and
creatinine, and hypercalcemia. What is the most likely diagnosis?

a. Acute leukemia
b. Chronic leukemia
c. Lymphoma
d. Multiple Myeloma
e. rombotic rombocytopenic Purpura

‫اﻹﺟﺎﺑﺔ ﻋﲆ اﻟﺼﻔﺤﺔ اﻟﺘﺎﻟﻴﺔ‬

Page - 1158
Internal Medicine - Hematology - Multiple Myeloma (MM)

Question 113/155

Question #113

A 64-year-old male patient presents with lower back pain and fatigue. His symptoms have been
progressive over the past 6 months. His lab investigations show mild anemia, elevated urea and
creatinine, and hypercalcemia. What is the most likely diagnosis?

a. Acute leukemia
b. Chronic leukemia
c. Lymphoma
d. Multiple Myeloma √
e. rombotic rombocytopenic Purpura

Description

ese abnormal Immunoglobulins in Multiple Myeloma (MM) lead to:

Infection
Renal deposition and renal failure
Bence-Jones proteinuria (detected by urine immunoelectrophoresis)
Hyperviscosity syndrome
Activation of osteolytic activity of the bone, which leads to hypercalcemia

Page - 1159
Internal Medicine - Hematology

Question 114/155

Question #114

A previously healthy 57-year-old male patient is diagnosed with Multiple Myeloma, but he is
asymptomatic and has no organ damage. What is the most appropriate treatment at this time?

a. Start chemotherapy
b. Start radiotherapy
c. Bone marrow transplantation
d. Imatinib
e. No treatment

‫اﻹﺟﺎﺑﺔ ﻋﲆ اﻟﺼﻔﺤﺔ اﻟﺘﺎﻟﻴﺔ‬

Page - 1160
Internal Medicine - Hematology - Multiple Myeloma (MM)

Question 114/155

Question #114

A previously healthy 57-year-old male patient is diagnosed with Multiple Myeloma, but he is
asymptomatic and has no organ damage. What is the most appropriate treatment at this time?

a. Start chemotherapy
b. Start radiotherapy
c. Bone marrow transplantation
d. Imatinib
e. No treatment √

Description

Treatment of Multiple Myeloma is as the following:

If asymptomatic → no Treatment
-Supportive therapy: high fluid intake, analgesia, allopurinol, protein plasmapheresis for
hyperviscosity
Specific treatment:
alidomide + melphalan + prednisolone: for older patients
Chemotherapy followed by HSCT: improves the quality of life but does not cure the
disease
Bisphosphonates: decrease bone pain and prevent bone fracture

Page - 1161
Internal Medicine - Hematology

Question 115/155

Question #115

e following features are known to be found in Multiple Myeloma (MM) except:

a. Recurrent pneumonia
b. Osteoclastic activation
c. High serum alkaline phosphatase
d. Hypercalcemia
e. Light chain renal deposition

‫اﻹﺟﺎﺑﺔ ﻋﲆ اﻟﺼﻔﺤﺔ اﻟﺘﺎﻟﻴﺔ‬

Page - 1162
Internal Medicine - Hematology - Multiple Myeloma (MM)

Question 115/155

Question #115

e following features are known to be found in Multiple Myeloma (MM) except:

a. Recurrent pneumonia
b. Osteoclastic activation
c. High serum alkaline phosphatase √
d. Hypercalcemia
e. Light chain renal deposition

Description

Normal serum alkaline phosphatase level is expected in Multiple Myeloma.

ese abnormal Immunoglobulins in Multiple Myeloma (MM) lead to:

Infection
Renal deposition and renal failure
Bence-Jones proteinuria (detected by urine immunoelectrophoresis)
Hyperviscosity syndrome
Activation of osteolytic activity of the bone, which leads to hypercalcemia

Page - 1163
Internal Medicine - Hematology

Question 116/155

Question #116

A 62-year-old female is a known case of multiple myeloma. However, she is confused, and her labs
show a corrected calcium level of three mmol/l. Which one of the following is the most significant
cause of the raised calcium level?

a. A side eect of the treatment


b. Osteoclastic activation
c. Osteoblastic activation
d. Deteriorating renal function
e. Vitamin D overactivation

‫اﻹﺟﺎﺑﺔ ﻋﲆ اﻟﺼﻔﺤﺔ اﻟﺘﺎﻟﻴﺔ‬

Page - 1164
Internal Medicine - Hematology - Multiple Myeloma (MM)

Question 116/155

Question #116

A 62-year-old female is a known case of multiple myeloma. However, she is confused, and her labs
show a corrected calcium level of three mmol/l. Which one of the following is the most significant
cause of the raised calcium level?

a. A side eect of the treatment


b. Osteoclastic activation √
c. Osteoblastic activation
d. Deteriorating renal function
e. Vitamin D overactivation

Description

In Multiple Myeloma, the activation of osteolytic activity leads to lytic lesions of the bone and
hypercalcemia

Page - 1165
Internal Medicine - Hematology

Question 117/155

Question #117

A 72-year-old male patient develops back pain, confusion, and polyuria. Her lab investigations show
hemoglobin of 10 g/dL, serum creatinine of 2 mg/dL, and serum calcium of 3.1 mmol/L. In addition,
you ordered a lumbar spine lateral view x-ray, which is significant for lytic lesions. What is the most
likely diagnosis?

a. Sarcoidosis
b. Multiple myeloma
c. Diabetes insipidus
d. Primary hyperparathyroidism
e. Vitamin D intoxication

‫اﻹﺟﺎﺑﺔ ﻋﲆ اﻟﺼﻔﺤﺔ اﻟﺘﺎﻟﻴﺔ‬

Page - 1166
Internal Medicine - Hematology - Multiple Myeloma (MM)

Question 117/155

Question #117

A 72-year-old male patient develops back pain, confusion, and polyuria. Her lab investigations show
hemoglobin of 10 g/dL, serum creatinine of 2 mg/dL, and serum calcium of 3.1 mmol/L. In addition,
you ordered a lumbar spine lateral view x-ray, which is significant for lytic lesions. What is the most
likely diagnosis?

a. Sarcoidosis
b. Multiple myeloma √
c. Diabetes insipidus
d. Primary hyperparathyroidism
e. Vitamin D intoxication

Description

is is a classic presentation of Multiple Myeloma (MM).

A bone marrow biopsy showing >10% plasma cells is the most accurate test.

ese abnormal Immunoglobulins in Multiple Myeloma (MM) lead to:

Infection
Renal deposition and renal failure
Bence-Jones proteinuria (detected by urine immunoelectrophoresis)
Hyperviscosity syndrome
Activation of osteolytic activity of the bone, which leads to hypercalcemia

Page - 1167
Internal Medicine - Hematology

Question 118/155

Question #118

A 60-year-old male patient complains of night sweating, weight loss, easy bruises, and fever. His
abdominal examination demonstrates massive splenomegaly, and his lab investigations show
pancytopenia, bone marrow hypocellularity, and replacement of normal tissue with fibrous tissue.
What is the most likely diagnosis?

a. Polycythemia Vera
b. Paroxysmal nocturnal hemoglobinuria
c. Essential thrombocythemia
d. Myelofibrosis
e. Myelodysplastic syndrome

‫اﻹﺟﺎﺑﺔ ﻋﲆ اﻟﺼﻔﺤﺔ اﻟﺘﺎﻟﻴﺔ‬

Page - 1168
Internal Medicine - Hematology - Myelofibrosis

Question 118/155

Question #118

A 60-year-old male patient complains of night sweating, weight loss, easy bruises, and fever. His
abdominal examination demonstrates massive splenomegaly, and his lab investigations show
pancytopenia, bone marrow hypocellularity, and replacement of normal tissue with fibrous tissue.
What is the most likely diagnosis?

a. Polycythemia Vera
b. Paroxysmal nocturnal hemoglobinuria
c. Essential thrombocythemia
d. Myelofibrosis √
e. Myelodysplastic syndrome

Description

Myelofibrosis is a bone marrow fibrosis due to the increased production of fibroblasts. It usually
presents in patients > 50 years old.

Clinical features:

Weight loss, night sweat


Hepatomegaly, Massive splenomegaly
Features of anemia

Lab tests:

Pancytopenia (Platelets, WBC can be high early in the disease)


Teardrop poikilocytosis in blood film
Bone marrow aspiration and biopsy (replacement of normal tissue with fibrous tissue)
JAK-2 mutation
Others: Folate deficiency, High uric acid level

Treatment:

Supportive: blood transfusion, folic acid


Hydroxyurea: reduce splenomegaly and reduce WBC
Splenectomy may be required (reduce the risk of splenic rupture)
Page - 1169
HSCT (definitive treatment)

Page - 1170
Internal Medicine - Hematology

Question 119/155

Question #119

A patient is diagnosed with normochromic anemia. e following statements are true except:

a. It may result from blood loss


b. e MCV is 80 – 100 fl
c. Renal disease is a possible cause
d. May result from folate deficiency
e. Hematocrit is reduced

‫اﻹﺟﺎﺑﺔ ﻋﲆ اﻟﺼﻔﺤﺔ اﻟﺘﺎﻟﻴﺔ‬

Page - 1171
Internal Medicine - Hematology - Normocytic anemia

Question 119/155

Question #119

A patient is diagnosed with normochromic anemia. e following statements are true except:

a. It may result from blood loss


b. e MCV is 80 – 100 fl
c. Renal disease is a possible cause
d. May result from folate deficiency √
e. Hematocrit is reduced

Description

Normocytic anemia is the presence of low hematocrit and normal MCV (80 – 100 fl)

e following conditions can cause normocytic anemia:

Normocytic anemia with High Reticulocytes count:

Acute bleeding
Hemolysis

Normocytic anemia with low Reticulocytes count:

Anemia of chronic disease (most commonly normocytic (70%) but can be microcytic)
Renal/liver disease
Aplastic anemia
Myelofibrosis
Leukemia
Drugs (e.g., chemotherapy)

Page - 1172
Internal Medicine - Hematology

Question 120/155

Question #120

A patient has low hematocrit, high reticulocyte count, and his MCV is normal. What is the most
likely diagnosis?

a. Alcoholism
b. Liver disease
c. Iron deficiency
d. Acute bleeding
e. Chronic bleeding

‫اﻹﺟﺎﺑﺔ ﻋﲆ اﻟﺼﻔﺤﺔ اﻟﺘﺎﻟﻴﺔ‬

Page - 1173
Internal Medicine - Hematology - Normocytic anemia

Question 120/155

Question #120

A patient has low hematocrit, high reticulocyte count, and his MCV is normal. What is the most
likely diagnosis?

a. Alcoholism
b. Liver disease
c. Iron deficiency
d. Acute bleeding √
e. Chronic bleeding

Description

Normocytic anemia with a high reticulocyte count indicates either acute bleeding or acute
hemolytic anemia.

Alcoholism and liver disease usually cause macrocytic non-megaloblastic anemia

Chronic bleeding causes an Iron deficiency; they present with microcytic hypochromic anemia.

Page - 1174
Internal Medicine - Hematology

Question 121/155

Question #121

A 43-year-old male patient is known to have ESRD and is on hemodialysis. He presents to you with
uncontrolled epistaxis. His lab investigations show a normal CBC, serum creatinine of 10 mg/dL,
urea of 100mg/dL, and prolonged bleeding time. What is the most appropriate next step in the
management of this patient?

a. Fresh frozen plasma


b. Cryoprecipitate
c. Factor 8
d. Activated factor 7
e. Hemodialysis

‫اﻹﺟﺎﺑﺔ ﻋﲆ اﻟﺼﻔﺤﺔ اﻟﺘﺎﻟﻴﺔ‬

Page - 1175
Internal Medicine - Hematology - Platelet dysfunction

Question 121/155

Question #121

A 43-year-old male patient is known to have ESRD and is on hemodialysis. He presents to you with
uncontrolled epistaxis. His lab investigations show a normal CBC, serum creatinine of 10 mg/dL,
urea of 100mg/dL, and prolonged bleeding time. What is the most appropriate next step in the
management of this patient?

a. Fresh frozen plasma


b. Cryoprecipitate
c. Factor 8
d. Activated factor 7
e. Hemodialysis √

Description

is is a case of dysfunctional platelet secondary to uremia (uremic platelet dysfunction)

Uremia prevents the normal function of platelets

Look for a patient with uremia and platelet-type bleeding

BT is prolonged, but PT, PTT, INR, factor 8, and vWF are normal

Treatment: DDAVP, dialysis, and estrogen

Note that estrogen shortens the bleeding time in uremic platelet dysfunction; it may provide a more
sustained hemostatic eect over desmopressin.

Page - 1176
Internal Medicine - Hematology

Question 122/155

Question #122

A 54-year-old male complains of headaches and blurry vision. He has a history of hypertension. In
addition, the patient reports pruritus and a burning sensation in his fingers, especially aer hot
baths. Abdominal examination demonstrates palpable spleen. His lab investigations show
polycythemia, thrombocytosis, leukocytosis, and low erythropoietin level. What is the SINGLE most
appropriate management?

a. Splenectomy
b. Indomethacin
c. Phlebotomy
d. Heparin
e. Warfarin

‫اﻹﺟﺎﺑﺔ ﻋﲆ اﻟﺼﻔﺤﺔ اﻟﺘﺎﻟﻴﺔ‬

Page - 1177
Internal Medicine - Hematology - Polycythemia Vera (PV)

Question 122/155

Question #122

A 54-year-old male complains of headaches and blurry vision. He has a history of hypertension. In
addition, the patient reports pruritus and a burning sensation in his fingers, especially aer hot
baths. Abdominal examination demonstrates palpable spleen. His lab investigations show
polycythemia, thrombocytosis, leukocytosis, and low erythropoietin level. What is the SINGLE most
appropriate management?

a. Splenectomy
b. Indomethacin
c. Phlebotomy √
d. Heparin
e. Warfarin

Description

In primary polycythemia, there will be reduced erythropoietin secondary to increased bone


marrow activity. is condition is caused by JAK 2 mutation
In secondary polycythemia, elevated erythropoietin will lead to elevated bone marrow activity.

Other findings in PV:

Mild splenomegaly
rombocytosis (> 400 * 10^9/L)
Leukocytosis (> 12 * 10^9/L)
Leukocyte alkaline phosphatase (> 100U but no fever or infection)

Treatment of polycythemia Vera includes:

Aspirin; to prevent peripheral arterial or cerebrovascular diseases


Venesection: to maintain Hb less than 15 g/dl, decrease symptoms of Hyperviscosity
Hydroxyurea, interferon-alpha to suppress myeloproliferation
Radioactive phosphate for the older patient (decrease transformation to acute leukemias)
Antihistamine (symptomatic treatment)
Allopurinol (protect against high uric acid)

Page - 1178
Internal Medicine - Hematology

Question 123/155

Question #123

A 68-year-old male patient presented to your oce complaining of a hot, red, and inflamed first
metatarsal joint. e patient is not an alcoholic and never smoked. Upon further questioning, he
admitted to itching aer hot baths. Labs show hemoglobin 18.9, hematocrit 57%, and high uric acid
level. e bone marrow biopsy of this patient is most likely to show one of the following:

a. Normal examination
b. Blast cells 15%
c. Blast cells 25%
d. Hypercellular bone marrow
e. Hypocellular bone marrow

‫اﻹﺟﺎﺑﺔ ﻋﲆ اﻟﺼﻔﺤﺔ اﻟﺘﺎﻟﻴﺔ‬

Page - 1179
Internal Medicine - Hematology - Polycythemia Vera (PV)

Question 123/155

Question #123

A 68-year-old male patient presented to your oce complaining of a hot, red, and inflamed first
metatarsal joint. e patient is not an alcoholic and never smoked. Upon further questioning, he
admitted to itching aer hot baths. Labs show hemoglobin 18.9, hematocrit 57%, and high uric acid
level. e bone marrow biopsy of this patient is most likely to show one of the following:

a. Normal examination
b. Blast cells 15%
c. Blast cells 25%
d. Hypercellular bone marrow √
e. Hypocellular bone marrow

Description

is patient complains of acute gouty arthritis.

Polycythemia, hyperuricemia, and pruritus strongly suggest polycythemia Vera

In polycythemia Vera, bone marrow biopsy will show hypercellularity with prominent erythroid,
granulocytic, and megakaryocytic proliferation

Page - 1180
Internal Medicine - Hematology

Question 124/155

Question #124

A previously healthy 58-year-old male patient complains of pruritus, especially aer a hot bath and
blurry vision. His hemoglobin level is 19.8 g/dL, and his erythropoietin level is below normal. What is
the most likely to be associated with this condition?

a. Reduced platelet count


b. Hypoxia
c. JAK2 mutation
d. Massive splenomegaly
e. Blast cells >20% in the bone marrow

‫اﻹﺟﺎﺑﺔ ﻋﲆ اﻟﺼﻔﺤﺔ اﻟﺘﺎﻟﻴﺔ‬

Page - 1181
Internal Medicine - Hematology - Polycythemia Vera (PV)

Question 124/155

Question #124

A previously healthy 58-year-old male patient complains of pruritus, especially aer a hot bath and
blurry vision. His hemoglobin level is 19.8 g/dL, and his erythropoietin level is below normal. What is
the most likely to be associated with this condition?

a. Reduced platelet count


b. Hypoxia
c. JAK2 mutation √
d. Massive splenomegaly
e. Blast cells >20% in the bone marrow

Description

Polycythemia Vera (PV) usually, in the age above 40 years

Results from a genetic problem caused by JAK2 mutation

is disorder can transform into acute leukemia or myelofibrosis in 25% of cases

e mutation will increase bone marrow activity and erythropoiesis despite a low erythropoietin
level.

Other findings in PV:

Mild splenomegaly
rombocytosis (> 400 * 10^9/L)
Leukocytosis (> 12 * 10^9/L)
Leukocyte alkaline phosphatase (> 100U but no fever or infection)

Polycythemia Vera (PV) requires two major and one minor or the first major criterion together with
two minor criteria.

Major criteria:

HB > 18.5 g/dL in males or > 16.5 in females or elevated RBC mass > 25% above mean normal
predicted value
Presence of JAK2 mutation Page - 1182
Minor criteria:

BM biopsy with hypercellularity with prominent erythroid, granulocytic, and megakaryocytic


proliferation
Low serum erythropoietin
Endogenous erythroid colony formation in vitro

Page - 1183
Internal Medicine - Hematology

Question 125/155

Question #125

A 52-year-old male patient complains of headaches. His history and physical examination are
unremarkable, but his lab investigations show a hematocrit level of 59%. In addition, his
erythropoietin level is low. What is the most likely diagnosis?

a. e patient lives in high altitude


b. e patient has COPD
c. e patient has renal cell carcinoma
d. e patient has Polycythemia Vera
e. e patient has lung fibrosis

‫اﻹﺟﺎﺑﺔ ﻋﲆ اﻟﺼﻔﺤﺔ اﻟﺘﺎﻟﻴﺔ‬

Page - 1184
Internal Medicine - Hematology - Polycythemia Vera (PV)

Question 125/155

Question #125

A 52-year-old male patient complains of headaches. His history and physical examination are
unremarkable, but his lab investigations show a hematocrit level of 59%. In addition, his
erythropoietin level is low. What is the most likely diagnosis?

a. e patient lives in high altitude


b. e patient has COPD
c. e patient has renal cell carcinoma
d. e patient has Polycythemia Vera √
e. e patient has lung fibrosis

Description

In the case of polycythemia, you should identify primary vs. secondary causes.

In primary polycythemia, there will be reduced erythropoietin secondary to increased bone marrow
activity. is condition is caused by JAK2 mutation.

In secondary polycythemia, elevated erythropoietin will lead to elevated bone marrow activity.

Patients with COPD, lung fibrosis, or those living at a high altitude will develop a high erythropoietin
level due to hypoxia

Renal cell carcinoma may cause a paraneoplastic feature of polycythemia due to ectopic
erythropoietin secretion, but it also presents with renal impairment, hematuria, and other features
suggestive of the disease.

Page - 1185
Internal Medicine - Hematology

Question 126/155

Question #126

A 54-year-old male complains of headaches and blurry vision. He has a history of hypertension. In
addition, the patient reports pruritus and a burning sensation in his fingers, especially aer taking a
hot bath. Abdominal examination demonstrates palpable spleen. His lab investigations show
polycythemia, thrombocytosis, leukocytosis, and low erythropoietin level. What is the most likely
diagnosis?

a. Myelofibrosis
b. Polycythemia Vera
c. Essential thrombocythemia
d. Acute leukemia
e. Chronic leukemia

‫اﻹﺟﺎﺑﺔ ﻋﲆ اﻟﺼﻔﺤﺔ اﻟﺘﺎﻟﻴﺔ‬

Page - 1186
Internal Medicine - Hematology - Polycythemia Vera (PV)

Question 126/155

Question #126

A 54-year-old male complains of headaches and blurry vision. He has a history of hypertension. In
addition, the patient reports pruritus and a burning sensation in his fingers, especially aer taking a
hot bath. Abdominal examination demonstrates palpable spleen. His lab investigations show
polycythemia, thrombocytosis, leukocytosis, and low erythropoietin level. What is the most likely
diagnosis?

a. Myelofibrosis
b. Polycythemia Vera √
c. Essential thrombocythemia
d. Acute leukemia
e. Chronic leukemia

Description

In primary polycythemia, there will be reduced erythropoietin secondary to increased bone marrow
activity. is condition is caused by JAK2 mutation.

In secondary polycythemia, elevated erythropoietin will lead to elevated bone marrow activity.

Other findings in PV:

Mild splenomegaly
rombocytosis (> 400 * 10^9/L)
Leukocytosis (> 12 * 10^9/L)
Leukocyte alkaline phosphatase (> 100U but no fever or infection)

Polycythemia Vera (PV) requires two major and one minor or the first major criterion together with
two minor criteria.

Major criteria:

HB > 18.5 g/dL in males or > 16.5 in females or elevated RBC mass > 25% above mean normal
predicted value
Presence of JAK2 mutation

Page - 1187
Minor criteria:

BM biopsy with hypercellularity with prominent erythroid, granulocytic, and megakaryocytic


proliferation
Low serum erythropoietin
Endogenous erythroid colony formation in vitro

Page - 1188
Internal Medicine - Hematology

Question 127/155

Question #127

A previously healthy 55-year-old male patient presents with typical chest pain and is diagnosed with
inferior STEMI. His lab investigations show positive cardiac enzymes, hematocrit of 60%, normal
kidney function test, and low erythropoietin levels. What is the most likely cause of his Myocardial
infarction?

a. Primary polycythemia
b. Secondary polycythemia
c. Ruptured plaque
d. Congenital vascular malformation
e. Embolization from a distant focus

‫اﻹﺟﺎﺑﺔ ﻋﲆ اﻟﺼﻔﺤﺔ اﻟﺘﺎﻟﻴﺔ‬

Page - 1189
Internal Medicine - Hematology - Polycythemia Vera (PV)

Question 127/155

Question #127

A previously healthy 55-year-old male patient presents with typical chest pain and is diagnosed with
inferior STEMI. His lab investigations show positive cardiac enzymes, hematocrit of 60%, normal
kidney function test, and low erythropoietin levels. What is the most likely cause of his Myocardial
infarction?

a. Primary polycythemia √
b. Secondary polycythemia
c. Ruptured plaque
d. Congenital vascular malformation
e. Embolization from a distant focus

Description

A hematocrit of 60% and a low erythropoietin level indicate polycythemia Vera (primary
polycythemia).

In primary polycythemia, there will be reduced erythropoietin secondary to increased bone marrow
activity. is condition is caused by JAK2 mutation.

In secondary polycythemia, elevated erythropoietin will lead to elevated bone marrow activity.

Polycythemia Vera increases the risk of thromboembolism; therefore, patients with PV should
receive aspirin therapy and be treated with phlebotomy to reduce the hematocrit to normal.

Page - 1190
Internal Medicine - Hematology

Question 128/155

Question #128

A 23-year-old male with a known case of SLE presents with sudden onset of hemoptysis, pleuritic
chest pain, and tachycardia. Physical examination reveals a clear chest and a swollen erythematous
hot right lower limb. His chest x-ray and lab investigations are normal. What is the most appropriate
at this time?

a. Computed tomography pulmonary angiography


b. Ventilation-perfusion scan of the lung
c. Arterial blood gases
d. D-Dimer
e. Doppler and duplex scan of the leg

‫اﻹﺟﺎﺑﺔ ﻋﲆ اﻟﺼﻔﺤﺔ اﻟﺘﺎﻟﻴﺔ‬

Page - 1191
Internal Medicine - Hematology - Pulmonary Embolism (PE)

Question 128/155

Question #128

A 23-year-old male with a known case of SLE presents with sudden onset of hemoptysis, pleuritic
chest pain, and tachycardia. Physical examination reveals a clear chest and a swollen erythematous
hot right lower limb. His chest x-ray and lab investigations are normal. What is the most appropriate
at this time?

a. Computed tomography pulmonary angiography √


b. Ventilation-perfusion scan of the lung
c. Arterial blood gases
d. D-Dimer
e. Doppler and duplex scan of the leg

Description

According to the well’s criteria, this patient will get 1 point for hemoptysis, 1.5 points for tachycardia,
3 points for suspected DVT, and 3 points for the absence of an alternative diagnosis. erefore, the
total score is 8.5, which indicates a high probability of pulmonary embolism.

Unless contraindicated, the best test to confirm the diagnosis, in this case, is CTPA.

Page - 1192
Internal Medicine - Hematology

Question 129/155

Question #129

An elderly man traveled to Jordan from America. Aer 5 days, he develops pleuritic chest pain,
dyspnea, and tachycardia. What is the most likely diagnosis?

a. Pulmonary embolism
b. Pneumonia
c. Myocardial infarction
d. Aortic dissection
e. Muscular spasm

‫اﻹﺟﺎﺑﺔ ﻋﲆ اﻟﺼﻔﺤﺔ اﻟﺘﺎﻟﻴﺔ‬

Page - 1193
Internal Medicine - Hematology - Pulmonary Embolism (PE)

Question 129/155

Question #129

An elderly man traveled to Jordan from America. Aer 5 days, he develops pleuritic chest pain,
dyspnea, and tachycardia. What is the most likely diagnosis?

a. Pulmonary embolism √
b. Pneumonia
c. Myocardial infarction
d. Aortic dissection
e. Muscular spasm

Description

Pulmonary embolism is most likely due to the traveling history, pleuritic chest pain, and tachycardia.

PE is an occlusion of the pulmonary artery or its branches by an embolus


Virchow’s triad is a group of risk factors for venous thromboembolism (VTE).
Clot migration from the DVT site to lodge in the pulmonary artery
Less than 30% have clinical evidence of DVT

Page - 1194
Internal Medicine - Hematology

Question 130/155

Question #130

A known case of malignancy 67-year-old male had surgery 10 days ago, and the surgery went
uneventfully. Today he presents with severe sudden dyspnea, pleuritic chest pain, diaphoresis, and
feeling panic. e chest is clear to examination, and the chest x-ray is normal. Which of the following
is the most likely diagnosis?

a. Atelectasis
b. Pulmonary edema
c. Pulmonary embolism
d. Pleural eusion
e. Lung abscess

‫اﻹﺟﺎﺑﺔ ﻋﲆ اﻟﺼﻔﺤﺔ اﻟﺘﺎﻟﻴﺔ‬

Page - 1195
Internal Medicine - Hematology - Pulmonary Embolism (PE)

Question 130/155

Question #130

A known case of malignancy 67-year-old male had surgery 10 days ago, and the surgery went
uneventfully. Today he presents with severe sudden dyspnea, pleuritic chest pain, diaphoresis, and
feeling panic. e chest is clear to examination, and the chest x-ray is normal. Which of the following
is the most likely diagnosis?

a. Atelectasis
b. Pulmonary edema
c. Pulmonary embolism √
d. Pleural eusion
e. Lung abscess

Description

Malignancy and the history of recent surgery are the main risk factors for pulmonary embolism as
the most likely diagnosis in this patient.

Atelectasis usually presents with postoperative fever on the first day

Pleural eusion, lung abscess, and pulmonary edema will not present with a clear chest and normal
chest x-ray.

Page - 1196
Internal Medicine - Hematology

Question 131/155

Question #131

A 39-year-old male came with a 2-day history of pleuritic right-side sharp chest pain of sudden
onset. He was previously healthy and had no record of travel. Physical examination is only
remarkable for pleural friction rub at the right middle zone. Vital signs, ECG, cardiac enzymes,
Oxygen saturation, and D Dimer levels are normal. What is the best initial test to do at this time?

a. No further testing
b. Chest x-ray
c. Repeat D Dimer level
d. CT pulmonary angiography
e. ANA and Anti-dsDNA

‫اﻹﺟﺎﺑﺔ ﻋﲆ اﻟﺼﻔﺤﺔ اﻟﺘﺎﻟﻴﺔ‬

Page - 1197
Internal Medicine - Hematology - Pulmonary Embolism (PE)

Question 131/155

Question #131

A 39-year-old male came with a 2-day history of pleuritic right-side sharp chest pain of sudden
onset. He was previously healthy and had no record of travel. Physical examination is only
remarkable for pleural friction rub at the right middle zone. Vital signs, ECG, cardiac enzymes,
Oxygen saturation, and D Dimer levels are normal. What is the best initial test to do at this time?

a. No further testing
b. Chest x-ray √
c. Repeat D Dimer level
d. CT pulmonary angiography
e. ANA and Anti-dsDNA

Description

In the case of pleurisy, you must exclude life-threatening conditions like myocardial infarction,
pneumothorax, and pulmonary embolism.

e best initial test is a chest x-ray before going forward with the investigations.

Page - 1198
Internal Medicine - Hematology

Question 132/155

Question #132

A 43-year-old male with a known case of CKD and SLE presents with sudden onset of hemoptysis,
pleuritic chest pain, and tachycardia. Physical examination reveals a clear chest and a swollen
erythematous hot right lower limb. His chest x-ray and lab investigations are normal. Which of the
following is the best study to confirm the diagnosis of his condition?

a. Conventional CT scan
b. VQ scan
c. Serum troponin level
d. Coronary angiography
e. Echocardiogram

‫اﻹﺟﺎﺑﺔ ﻋﲆ اﻟﺼﻔﺤﺔ اﻟﺘﺎﻟﻴﺔ‬

Page - 1199
Internal Medicine - Hematology - Pulmonary Embolism (PE)

Question 132/155

Question #132

A 43-year-old male with a known case of CKD and SLE presents with sudden onset of hemoptysis,
pleuritic chest pain, and tachycardia. Physical examination reveals a clear chest and a swollen
erythematous hot right lower limb. His chest x-ray and lab investigations are normal. Which of the
following is the best study to confirm the diagnosis of his condition?

a. Conventional CT scan
b. VQ scan √
c. Serum troponin level
d. Coronary angiography
e. Echocardiogram

Description

Of the above choices, the VQ scan is the best study to confirm the diagnosis of Pulmonary embolism

e conventional CT scan is not beneficial in pulmonary embolism, but CT pulmonary angiography,


which uses higher dose contrast and proper timing, is the imaging of choice unless contraindicated.

Page - 1200
Internal Medicine - Hematology

Question 133/155

Question #133

A 45-year-old female presents to you with pleuritic chest pain. Her well’s score was 1.5 only for
tachycardia. Which of the following is the best next step for evaluating this patient?

a. CTPA
b. D Dimer
c. VQ scan
d. Troponin level
e. BNP

‫اﻹﺟﺎﺑﺔ ﻋﲆ اﻟﺼﻔﺤﺔ اﻟﺘﺎﻟﻴﺔ‬

Page - 1201
Internal Medicine - Hematology - Pulmonary Embolism (PE)

Question 133/155

Question #133

A 45-year-old female presents to you with pleuritic chest pain. Her well’s score was 1.5 only for
tachycardia. Which of the following is the best next step for evaluating this patient?

a. CTPA
b. D Dimer √
c. VQ scan
d. Troponin level
e. BNP

Description

Low probability patients (score 4 or less on well’s criteria) should be investigated for D Dimer level
and then order CTPA if the D Dimer is elevated.

VQ scan used if the CTPA was contraindicated

Troponin level is used to evaluate whether the patient has MI and in case of massive PE

BNP is used at the ER to dierentiate cardiac vs. respiratory sources of dyspnea.

Page - 1202
Internal Medicine - Hematology

Question 134/155

Question #134

A 33-year-old male with a history of complete immobilization for 4 weeks due to a lower limb
fracture presents to you with sudden onset right upper sharp pleuritic chest pain. e presence of
pleural friction rub in this patient suggests one of the following:

a. Pericarditis
b. Pneumothorax
c. Pneumonia
d. Pulmonary embolism with infarction
e. Pancreatitis

‫اﻹﺟﺎﺑﺔ ﻋﲆ اﻟﺼﻔﺤﺔ اﻟﺘﺎﻟﻴﺔ‬

Page - 1203
Internal Medicine - Hematology - Pulmonary Embolism (PE)

Question 134/155

Question #134

A 33-year-old male with a history of complete immobilization for 4 weeks due to a lower limb
fracture presents to you with sudden onset right upper sharp pleuritic chest pain. e presence of
pleural friction rub in this patient suggests one of the following:

a. Pericarditis
b. Pneumothorax
c. Pneumonia
d. Pulmonary embolism with infarction √
e. Pancreatitis

Description

e pleural frictional rub is almost diagnostic of infarction of the overlying pleura.

Pericardial rub presents in case of pericarditis, while pneumonia presents with consolidation, fever,
and high WBC count

Pneumothorax and pulmonary embolism without infarction do not typically cause pleural friction
rub.

Page - 1204
Internal Medicine - Hematology

Question 135/155

Question #135

A 45-year-old presents with sharp pleuritic chest pain for 3 hours with tachycardia and diaphoresis.
His ECG, chest x-ray, and lab investigations are normal, while his ABGs reveal PH 7.48, PaCO2 21
mmHg, and HCO3 of 26 mg/dL. Which of the following is false regarding this condition?

a. Pulmonary embolism is one of the dierential diagnoses


b. e patient has respiratory alkalosis
c. Myocardial infarction should be suspected in case of pleuritic chest pain
d. Hypotension indicates life-threatening pulmonary embolism
e. e chest x-ray is diagnostic in case of pulmonary embolism.

‫اﻹﺟﺎﺑﺔ ﻋﲆ اﻟﺼﻔﺤﺔ اﻟﺘﺎﻟﻴﺔ‬

Page - 1205
Internal Medicine - Hematology - Pulmonary Embolism (PE)

Question 135/155

Question #135

A 45-year-old presents with sharp pleuritic chest pain for 3 hours with tachycardia and diaphoresis.
His ECG, chest x-ray, and lab investigations are normal, while his ABGs reveal PH 7.48, PaCO2 21
mmHg, and HCO3 of 26 mg/dL. Which of the following is false regarding this condition?

a. Pulmonary embolism is one of the dierential diagnoses


b. e patient has respiratory alkalosis
c. Myocardial infarction should be suspected in case of pleuritic chest pain
d. Hypotension indicates life-threatening pulmonary embolism
e. e chest x-ray is diagnostic in case of pulmonary embolism. √

Description

e chest x-ray is not specific or sensitive in PE patients. It is most commonly normal, but atelectasis,
oligemia, or Hampton hump sign may present.

5% of atypical chest pain is due to myocardial infarction, so ECG should be done in those who
develop pleuritic chest pain

Page - 1206
Internal Medicine - Hematology

Question 136/155

Question #136

A 43-year-old male with a known case of CKD and SLE presents with sudden onset of hemoptysis,
pleuritic chest pain, and tachycardia. Physical examination reveals a clear chest and a swollen
erythematous hot right lower limb. His chest x-ray and lab investigations are normal. What is the
most appropriate at this time?

a. Computed tomography pulmonary angiography


b. Ventilation-perfusion scan of the lung
c. Arterial blood gases
d. D-Dimer
e. Doppler and duplex scan of the leg

‫اﻹﺟﺎﺑﺔ ﻋﲆ اﻟﺼﻔﺤﺔ اﻟﺘﺎﻟﻴﺔ‬

Page - 1207
Internal Medicine - Hematology - Pulmonary Embolism (PE)

Question 136/155

Question #136

A 43-year-old male with a known case of CKD and SLE presents with sudden onset of hemoptysis,
pleuritic chest pain, and tachycardia. Physical examination reveals a clear chest and a swollen
erythematous hot right lower limb. His chest x-ray and lab investigations are normal. What is the
most appropriate at this time?

a. Computed tomography pulmonary angiography


b. Ventilation-perfusion scan of the lung √
c. Arterial blood gases
d. D-Dimer
e. Doppler and duplex scan of the leg

Description

According to the well’s criteria, this patient will get 1 point for hemoptysis, 1.5 points for tachycardia,
3 points for suspected DVT, and 3 points for the absence of an alternative diagnosis. erefore, the
total score is 8.5, which indicates a high probability of pulmonary embolism.

CTPA is contraindicated due to CKD, so a VQ scan is the best test to confirm the diagnosis.

Page - 1208
Internal Medicine - Hematology

Question 137/155

Question #137

A 43-year-old male with a known case of CKD and SLE presents with sudden onset of hemoptysis,
pleuritic chest pain, and tachycardia. Physical examination reveals a clear chest and a swollen
erythematous hot right lower limb. His chest x-ray and lab investigations are normal. Which of the
following ABGs findings suggests the diagnosis of pulmonary embolism?

a. Decreased A-a gradient


b. Decreased PaCO2
c. Decreased PH
d. Decreased Bicarbonate level
e. Normal PaO2

‫اﻹﺟﺎﺑﺔ ﻋﲆ اﻟﺼﻔﺤﺔ اﻟﺘﺎﻟﻴﺔ‬

Page - 1209
Internal Medicine - Hematology - Pulmonary Embolism (PE)

Question 137/155

Question #137

A 43-year-old male with a known case of CKD and SLE presents with sudden onset of hemoptysis,
pleuritic chest pain, and tachycardia. Physical examination reveals a clear chest and a swollen
erythematous hot right lower limb. His chest x-ray and lab investigations are normal. Which of the
following ABGs findings suggests the diagnosis of pulmonary embolism?

a. Decreased A-a gradient


b. Decreased PaCO2 √
c. Decreased PH
d. Decreased Bicarbonate level
e. Normal PaO2

Description

Hypoxia, CO2 wash (reduced PaCO2), and respiratory alkalosis are the main features of pulmonary
embolism in ABGs.

Page - 1210
Internal Medicine - Hematology

Question 138/155

Question #138

A known case of malignancy, a 55-year-old male has just been diagnosed with pulmonary embolism.
He has a blood pressure of 130/80 mmHg and a pulse rate of 130 bpm. Which of the following is the
best treatment option for this patient?

a. Apixaban for 6 months


b. Low molecular weight heparin for 6 months
c. Unfractionated heparin for 3 months
d. rombolytics
e. Warfarin for 3 months

‫اﻹﺟﺎﺑﺔ ﻋﲆ اﻟﺼﻔﺤﺔ اﻟﺘﺎﻟﻴﺔ‬

Page - 1211
Internal Medicine - Hematology - Pulmonary Embolism (PE)

Question 138/155

Question #138

A known case of malignancy, a 55-year-old male has just been diagnosed with pulmonary embolism.
He has a blood pressure of 130/80 mmHg and a pulse rate of 130 bpm. Which of the following is the
best treatment option for this patient?

a. Apixaban for 6 months √


b. Low molecular weight heparin for 6 months
c. Unfractionated heparin for 3 months
d. rombolytics
e. Warfarin for 3 months

Description

LMWH for 6 months was the first line treatment of DVT and PE in patients with active malignancy.
However, the guidelines have recently changed, and the DOACs are now the first line.

Treatment of DVT/PE:

DOAC (Apixaban or Rivaroxaban):


It should be started once the diagnosis is suspected
e first-line treatment for patients who do not have contraindications
In active malignancy patients, DOAC became preferred over LMWH.
Warfarin:
e second line treatment aer DOAC
Used in severe renal impairment (GFR < 15 mL/min).
Used as first-line if the patient has antiphospholipid syndrome.
LMWH:
Used as a bridge for warfarin (to prevent skin necrosis)
Used as monotherapy for 6 months in malignancy or pregnant patients (second line
aer DOAC)
Inferior vena cava filter indications:
Contraindications to anticoagulants
Recurrent emboli while on adequate anticoagulation
Right ventricular heart failure
Page - 1212
Length of anticoagulation:

3 months: for provoked DVT/PE (known risk factor)


6 months: for unprovoked cases (unknown risk factor)
3 – 6 months: for people with active cancer
Lifelong: for recurrent DVT/PE

Page - 1213
Internal Medicine - Hematology

Question 139/155

Question #139

A 55-year-old male just diagnosed with pulmonary embolism has a blood pressure of 70/50 mmHg
and a pulse rate of 130 bpm. Which of the following is the best treatment option for this patient?

a. Apixaban
b. Low molecular weight heparin
c. Unfractionated heparin
d. rombolytics
e. Warfarin

‫اﻹﺟﺎﺑﺔ ﻋﲆ اﻟﺼﻔﺤﺔ اﻟﺘﺎﻟﻴﺔ‬

Page - 1214
Internal Medicine - Hematology - Pulmonary Embolism (PE)

Question 139/155

Question #139

A 55-year-old male just diagnosed with pulmonary embolism has a blood pressure of 70/50 mmHg
and a pulse rate of 130 bpm. Which of the following is the best treatment option for this patient?

a. Apixaban
b. Low molecular weight heparin
c. Unfractionated heparin
d. rombolytics √
e. Warfarin

Description

Hypotension is a sign of a massive pulmonary embolism. Unless contraindicated, it should be


treated with thrombolytic therapy.

Aspirin has no rule in the treatment of PE, while heparin and warfarin can be used in pulmonary
embolism according to the patient’s condition

Page - 1215
Internal Medicine - Hematology

Question 140/155

Question #140

A heavy smoker 66-year-old male patient is found to have polycythemia on routine complete blood
count. However, the rest of the CBC findings are normal. What is the most likely diagnosis?

a. Polycythemia Vera
b. Essential thrombocythemia
c. Myelofibrosis
d. Chronic leukemia
e. Secondary polycythemia

‫اﻹﺟﺎﺑﺔ ﻋﲆ اﻟﺼﻔﺤﺔ اﻟﺘﺎﻟﻴﺔ‬

Page - 1216
Internal Medicine - Hematology - Secondary polycythemia

Question 140/155

Question #140

A heavy smoker 66-year-old male patient is found to have polycythemia on routine complete blood
count. However, the rest of the CBC findings are normal. What is the most likely diagnosis?

a. Polycythemia Vera
b. Essential thrombocythemia
c. Myelofibrosis
d. Chronic leukemia
e. Secondary polycythemia √

Description

In the case of polycythemia, you should identify primary vs. secondary causes.

Heavy smoking is one of the most common causes of polycythemia.

If the patient is not a smoker and no cause of polycythemia is identified, you may think about
Polycythemia Vera.

In polycythemia Vera, bone marrow hyperactivity is due to JAK2 mutation leading to polycythemia.
However, the reduced erythropoietin level will dierentiate it from secondary polycythemia.

Page - 1217
Internal Medicine - Hematology

Question 141/155

Question #141

A 55-year-old male patient has polycythemia and elevated erythropoietin. His blood pressure is
120/80, his pulse is 80 bpm, and his oxygen saturation is 89%. Which of the following is the most
likely diagnosis?

a. Polycythemia Vera
b. Pure red cell aplasia (PRCA)
c. Renal adenocarcinoma
d. Chronic obstructive pulmonary disease
e. Bone marrow lymphoma

‫اﻹﺟﺎﺑﺔ ﻋﲆ اﻟﺼﻔﺤﺔ اﻟﺘﺎﻟﻴﺔ‬

Page - 1218
Internal Medicine - Hematology - Secondary polycythemia

Question 141/155

Question #141

A 55-year-old male patient has polycythemia and elevated erythropoietin. His blood pressure is
120/80, his pulse is 80 bpm, and his oxygen saturation is 89%. Which of the following is the most
likely diagnosis?

a. Polycythemia Vera
b. Pure red cell aplasia (PRCA)
c. Renal adenocarcinoma
d. Chronic obstructive pulmonary disease √
e. Bone marrow lymphoma

Description

In the case of polycythemia, you should identify primary vs. secondary causes.

In primary polycythemia, there will be reduced erythropoietin secondary to increased bone marrow
activity. is condition is caused by JAK 2 mutation.

In secondary polycythemia, elevated erythropoietin will lead to elevated bone marrow activity.

Our patient, in this case, has elevated erythropoietin, indicating secondary polycythemia.

COPD will explain the presence of hypoxia, so it is the most likely diagnosis.

So, COPD will lead to hypoxia and hypoxia, resulting in elevated erythropoietin and, thus,
polycythemia.

Polycythemia Vera will present with low erythropoietin

Renal cell carcinoma will not explain hypoxia

PRCA will present with anemia and BM hypoactivity

Page - 1219
Internal Medicine - Hematology

Question 142/155

Question #142

A 52-year-old male patient presents with tiredness, headache, shortness of breath, and persistent
cough. He drinks alcohol socially and has smoked 30 cigarettes daily for the past 30 years. His lab
investigations show hemoglobin of 19.9 mg/dL, WBC of 11*10^9/L, and platelet count of 240*10^9/L.
Which of the following will help establish the cause of polycythemia in this patient?

a. Dexamethasone suppression test


b. Growth hormone suppression test
c. Erythropoietin level
d. yroid function test
e. Blood film

‫اﻹﺟﺎﺑﺔ ﻋﲆ اﻟﺼﻔﺤﺔ اﻟﺘﺎﻟﻴﺔ‬

Page - 1220
Internal Medicine - Hematology - Secondary polycythemia

Question 142/155

Question #142

A 52-year-old male patient presents with tiredness, headache, shortness of breath, and persistent
cough. He drinks alcohol socially and has smoked 30 cigarettes daily for the past 30 years. His lab
investigations show hemoglobin of 19.9 mg/dL, WBC of 11*10^9/L, and platelet count of 240*10^9/L.
Which of the following will help establish the cause of polycythemia in this patient?

a. Dexamethasone suppression test


b. Growth hormone suppression test
c. Erythropoietin level √
d. yroid function test
e. Blood film

Description

In the case of polycythemia, you should identify primary vs. secondary causes.

Heavy smoking is one of the most common causes of polycythemia.

If the patient is not a smoker and no cause of polycythemia, you may think about Polycythemia Vera.

In polycythemia Vera, bone marrow hyperactivity is due to JAK2 mutation leading to polycythemia.
However, the reduced erythropoietin level will dierentiate it from secondary polycythemia.

Page - 1221
Internal Medicine - Hematology

Question 143/155

Question #143

A 21-year-old male patient with sickle cell anemia presents with fatigue, pallor, and flu-like
symptoms. His lab tests show hemoglobin of 5 g/dL and positive specific parvovirus B19 IgM
antibodies. What is the most likely diagnosis?

a. Hemolytic crisis
b. Aplastic crisis
c. Splenic sequestration crisis
d. Vaso-occlusive crisis
e. Acute chest syndrome

‫اﻹﺟﺎﺑﺔ ﻋﲆ اﻟﺼﻔﺤﺔ اﻟﺘﺎﻟﻴﺔ‬

Page - 1222
Internal Medicine - Hematology - Sickle Cell Anemia (SCA)

Question 143/155

Question #143

A 21-year-old male patient with sickle cell anemia presents with fatigue, pallor, and flu-like
symptoms. His lab tests show hemoglobin of 5 g/dL and positive specific parvovirus B19 IgM
antibodies. What is the most likely diagnosis?

a. Hemolytic crisis
b. Aplastic crisis √
c. Splenic sequestration crisis
d. Vaso-occlusive crisis
e. Acute chest syndrome

Description

is patient complains of acutely low hemoglobin and parvovirus B19 infection consisting of an
aplastic crisis.

In aplastic crisis, the patient develops pancytopenia and low reticulocyte count related to parvovirus
infection.

Any question mentions sickle cell anemia and parvovirus B19 infection, think about aplastic crisis

Types of crises in sickle cell anemia:

Vaso-occlusive (thrombotic) crisis

It is the most common crisis in sickle cell anemia


It is precipitated by infection, cold, or dehydration
Obstructed microcirculation leads to ischemia and may commonly manifest as:
Severe bone pain
Painful infarction of the spleen (leading to autosplenectomy)
Papillary necrosis and renal impairment
Mesenteric infarction (mimicking abdominal pain)
Chest Pain (Bone involvement or pulmonary infarction)
Penile involvement (Priapism: persistent painful erection of the penis)
Avascular necrosis of the femoral head

Page - 1223
Pain crisis:

Acute severe pain caused by vaso-occlusion


It may aect any part of the body according to the site of vaso-occlusion

Aplastic crisis:

Transient cessation of erythropoiesis caused by parvovirus B19 infection


Hemoglobin, reticulocytes, WBC, and Platelets are low

Hemolytic crisis:

Increased rate of hemolysis


RBCs count will be low, but high reticulocytes count

Page - 1224
Internal Medicine - Hematology

Question 144/155

Question #144

A 19-year-old male patient with sickle cell anemia presents with severe abdominal pain and high
temperature. You started intravenous fluids and obtained lab tests, including cultures. What is the
best next step in this situation?

a. Oxygen supplementation
b. Parenteral opiate
c. Hydroxyurea
d. Blood transfusion
e. Intravenous paracetamol

‫اﻹﺟﺎﺑﺔ ﻋﲆ اﻟﺼﻔﺤﺔ اﻟﺘﺎﻟﻴﺔ‬

Page - 1225
Internal Medicine - Hematology - Sickle Cell Anemia (SCA)

Question 144/155

Question #144

A 19-year-old male patient with sickle cell anemia presents with severe abdominal pain and high
temperature. You started intravenous fluids and obtained lab tests, including cultures. What is the
best next step in this situation?

a. Oxygen supplementation
b. Parenteral opiate √
c. Hydroxyurea
d. Blood transfusion
e. Intravenous paracetamol

Description

To avoid developing chronic pain syndrome in patients with sickle cell anemia, you should treat the
acute pain crisis appropriately.

Pain crisis is treated by the following steps:

Parenteral opiate (started at fixed intervals, not as needed)


Oral analgesia aer tapering the intravenous opiate
Intravenous fluid
Treatment of the precipitant
Incentive spirometry
For frequent and severe pain, long-term hydroxyurea

Hypoxia is only indicated if the patient is hypoxic.

Types of crises in sickle cell anemia:

Vaso-occlusive (thrombotic) crisis

It is the most common crisis in sickle cell anemia


It is precipitated by infection, cold, or dehydration
Obstructed microcirculation leads to ischemia and may commonly manifest as:
Severe bone pain
Painful infarction of the spleen (leading to autosplenectomy)
Page - 1226
Papillary necrosis and renal impairment
Mesenteric infarction (mimicking abdominal pain)
Chest Pain (Bone involvement or pulmonary infarction)
Penile involvement (Priapism: persistent painful erection of the penis)
Avascular necrosis of the femoral head

Pain crisis:

Acute severe pain caused by vaso-occlusion


It may aect any part of the body according to the site of vaso-occlusion

Aplastic crisis:

Transient cessation of erythropoiesis caused by parvovirus B19 infection


Hemoglobin, reticulocytes, WBC, and Platelets are low

Hemolytic crisis:

Increased rate of hemolysis


RBCs count will be low, but high reticulocytes count

Page - 1227
Internal Medicine - Hematology

Question 145/155

Question #145

A 19-year-old male with a Hemoglobin concentration of 9.9 g/dl and microcytosis, serum iron, TIBC,
serum ferritin, and Hb-electrophoresis are normal. Which of the following is the most likely
diagnosis?

a. Iron deficiency anemia


b. Sideroblastic anemia
c. Beta-thalassemia
d. Alpha-thalassemia trait
e. Anemia of chronic disease

‫اﻹﺟﺎﺑﺔ ﻋﲆ اﻟﺼﻔﺤﺔ اﻟﺘﺎﻟﻴﺔ‬

Page - 1228
Internal Medicine - Hematology - alassemia

Question 145/155

Question #145

A 19-year-old male with a Hemoglobin concentration of 9.9 g/dl and microcytosis, serum iron, TIBC,
serum ferritin, and Hb-electrophoresis are normal. Which of the following is the most likely
diagnosis?

a. Iron deficiency anemia


b. Sideroblastic anemia
c. Beta-thalassemia
d. Alpha-thalassemia trait √
e. Anemia of chronic disease

Description

Alpha thalassemia trait is the only cause of thalassemia presenting normal HB electrophoresis.

In Iron deficiency anemia, serum iron and ferritin will be low, and the TIBC will be high.

In Beta-thalassemia, the HB electrophoresis will be abnormal

In sideroblastic anemia, serum iron and ferritin will be elevated.

ere is no evidence of chronic disease in this young man.

Page - 1229
Internal Medicine - Hematology

Question 146/155

Question #146

A 19-year-old female was accidentally found to have microcytic anemia, her iron study is normal, but
her Hb electrophoresis shows elevated hemoglobin A2. What is the most likely diagnosis?

a. Alpha-thalassemia
b. Beta-thalassemia
c. Sickle cell trait
d. Hereditary spherocytosis
e. Iron deficiency anemia

‫اﻹﺟﺎﺑﺔ ﻋﲆ اﻟﺼﻔﺤﺔ اﻟﺘﺎﻟﻴﺔ‬

Page - 1230
Internal Medicine - Hematology - alassemia

Question 146/155

Question #146

A 19-year-old female was accidentally found to have microcytic anemia, her iron study is normal, but
her Hb electrophoresis shows elevated hemoglobin A2. What is the most likely diagnosis?

a. Alpha-thalassemia
b. Beta-thalassemia √
c. Sickle cell trait
d. Hereditary spherocytosis
e. Iron deficiency anemia

Description

Beta-thalassemia is a genetic disorder of hemoglobin synthesis leading to abnormal hemoglobin,


membrane damage, and decreased RBC survival

In beta-alassemia, there is a reduced beta chains synthesis

Hb electrophoresis is the most accurate test in beta-thalassemia

In thalassemia minor (Hb A: 80-95%, Hb A2 4-8%, and Hb F: 1-5%)


In thalassemia major (Hb A: 0%, Hb A2: 4-10%, and Hb F: 90-96%)

Once the iron study is normal, this excludes iron deficiency anemia.

Page - 1231
Internal Medicine - Hematology

Question 147/155

Question #147

A 21-year-old female was tested for a complete blood count during her routine examination. She is
found to have a hemoglobin level of 10.5 g/dL and an RDW of 14. Based on these limited lab results,
what is the most likely diagnosis?

a. Iron deficiency anemia


b. B12 deficiency anemia
c. Anemia of chronic disease
d. Lead poisoning
e. alassemia

‫اﻹﺟﺎﺑﺔ ﻋﲆ اﻟﺼﻔﺤﺔ اﻟﺘﺎﻟﻴﺔ‬

Page - 1232
Internal Medicine - Hematology - alassemia

Question 147/155

Question #147

A 21-year-old female was tested for a complete blood count during her routine examination. She is
found to have a hemoglobin level of 10.5 g/dL and an RDW of 14. Based on these limited lab results,
what is the most likely diagnosis?

a. Iron deficiency anemia


b. B12 deficiency anemia
c. Anemia of chronic disease
d. Lead poisoning
e. alassemia √

Description

e causes of microcytic anemia are:

Iron deficiency anemia


alassemia
Anemia of chronic disease (but mostly normocytic)
Lead poisoning
Sideroblastic anemia

Causes of high RDW:

Iron deficiency anemia (low MCV)


Sideroblastic anemia (Low MCV)
Mixed Iron and B12 deficiency anemia (Normal MCV)
Folate and B12 deficiency (high MCV)
Recent hemorrhage (Normal MCV)

e RDW will be normal in thalassemia and elevated in iron deficiency and sideroblastic anemia.

So, thalassemia is the most likely diagnosis in this scenario.

Page - 1233
Internal Medicine - Hematology

Question 148/155

Question #148

A 17-year-old male is complaining of jaundice, pallor, and skull bossing. He has been dependent on
blood transfusion since childhood. What is the most likely diagnosis?

a. Hereditary spherocytosis
b. Iron deficiency anemia
c. Beta alassemia major
d. Beta thalassemia minor
e. Sideroblastic anemia

‫اﻹﺟﺎﺑﺔ ﻋﲆ اﻟﺼﻔﺤﺔ اﻟﺘﺎﻟﻴﺔ‬

Page - 1234
Internal Medicine - Hematology - alassemia

Question 148/155

Question #148

A 17-year-old male is complaining of jaundice, pallor, and skull bossing. He has been dependent on
blood transfusion since childhood. What is the most likely diagnosis?

a. Hereditary spherocytosis
b. Iron deficiency anemia
c. Beta alassemia major √
d. Beta thalassemia minor
e. Sideroblastic anemia

Description

Beta-thalassemia is a genetic disorder of hemoglobin synthesis leading to abnormal hemoglobin,


membrane damage, and decreased RBC survival

In beta-alassemia, there is a reduced beta chains synthesis

e clinical picture of beta-thalassemia major:

Severe anemia requiring transfusion in the first year of life


Intermittent infections
Bone marrow hyperplasia causing bossing of the head and prominent malar eminence
(alassemic face)
Splenomegaly (early), Hepatomegaly (slow)
Iron overload in a transfusion-dependent patient (usually aer 20 units of blood transfusion) →
resulting in features of hemochromatosis

Beta-thalassemia minor is usually asymptomatic and does not require a blood transfusion.

Page - 1235
Internal Medicine - Hematology

Question 149/155

Question #149

A 30-year-old male patient is a known case of beta-thalassemia major. He presents to you with lung
crepitations and gray skin. His lab investigations show hemoglobin of 7.3 g/dL and fasting glucose of
230mg/dL. What is the most appropriate at this time?

a. Blood transfusion
b. Phlebotomy
c. Oral iron and vitamin C
d. Hydroxyurea
e. Erythropoietin injection

‫اﻹﺟﺎﺑﺔ ﻋﲆ اﻟﺼﻔﺤﺔ اﻟﺘﺎﻟﻴﺔ‬

Page - 1236
Internal Medicine - Hematology - alassemia

Question 149/155

Question #149

A 30-year-old male patient is a known case of beta-thalassemia major. He presents to you with lung
crepitations and gray skin. His lab investigations show hemoglobin of 7.3 g/dL and fasting glucose of
230mg/dL. What is the most appropriate at this time?

a. Blood transfusion
b. Phlebotomy
c. Oral iron and vitamin C
d. Hydroxyurea √
e. Erythropoietin injection

Description

e patient is a known case of beta-thalassemia major that indicates recurrent blood transfusion. As
a result, those patients are at risk of iron overload.

Iron overload may aect several organs, including the heart, liver, pancreas, and skin. So, the
presence of features of liver failure, heart failure, gray skin, and hyperglycemia in a patient with
recurrent blood transfusion strongly suggests iron overload in this patient.

In this case, hydroxyurea will increase the formation of hemoglobin F, reducing the need for blood
transfusion and, thus, the risk of iron overload.

Phlebotomy is indicated in the case of hemochromatosis when the hemoglobin is elevated, but it is
contraindicated because it will worsen anemia.

Blood transfusion, oral iron, and vitamin C will increase the iron load and worsen the condition

Erythropoietin is used in anemia of chronic disease, not in thalassemia.

Page - 1237
Internal Medicine - Hematology

Question 150/155

Question #150

A 33-year-old male presents with headache, red urine, fatigue, and jaundice. His lab investigations
show prolonged bleeding time, low platelet, low hemoglobin, renal impairment, and indirect
hyperbilirubinemia. However, his PT, PTT, INR, and D Dimer are normal. What is the most likely
diagnosis?

a. Idiopathic thrombocytopenic purpura


b. Von Willebrand’s disease
c. Systemic lupus erythematosus
d. rombotic thrombocytopenic purpura
e. Acute liver disease

‫اﻹﺟﺎﺑﺔ ﻋﲆ اﻟﺼﻔﺤﺔ اﻟﺘﺎﻟﻴﺔ‬

Page - 1238
Internal Medicine - Hematology - rombotic rombocytopenic Purpura (TTP)

Question 150/155

Question #150

A 33-year-old male presents with headache, red urine, fatigue, and jaundice. His lab investigations
show prolonged bleeding time, low platelet, low hemoglobin, renal impairment, and indirect
hyperbilirubinemia. However, his PT, PTT, INR, and D Dimer are normal. What is the most likely
diagnosis?

a. Idiopathic thrombocytopenic purpura


b. Von Willebrand’s disease
c. Systemic lupus erythematosus
d. rombotic thrombocytopenic purpura √
e. Acute liver disease

Description

is is a typical scenario of rombotic rombocytopenic purpura (TTP)

PT, aPTT, D-dimer, and fibrinogen levels are normal in TTP-HUS and abnormal in DIC.

TTP:

Both TTP and HUS are dierent versions of the same basic disease (deficiency of
metalloproteinase ADAMTS 13)
Hemolytic Uremic Syndrome (HUS) is associated with E. coli O157:H7 → more frequent in
children (see pediatrics)

Pathogenesis:

TTP patient has an ADAMTS13 (vWF-cleaving protease) deficiency


TTP can occur in patients with cancer, transplant recipients, HIV, SLE, and those who receive
quinine, clopidogrel, ticlopidine, or cyclosporine.
Platelets and fibrin adhesion deposited in arterioles lead to the destruction of RBCs when
passing through these arterioles (Microangiopathic hemolytic anemia - MAHA).

Clinical presentation and diagnosis:

Fever
Page - 1239
Neurological manifestations (headache, confusion, ataxia, seizures, and mental status and
focal abnormalities)
Microangiopathic hemolytic anemia
Fragmented RBCs (schistocytes)
High reticulocytes and LDH levels
Indirect hyperbilirubinemia
Other features of intravascular hemolysis
Coombs test is negative
rombocytopenia
Renal impairment

Treatment:

Immediate discontinuation of the causative drug.


Emergency plasma exchange

Page - 1240
Internal Medicine - Hematology

Question 151/155

Question #151

You are seeing a 40-year-old female patient in the emergency department. She has a fever and
decreased level of consciousness. On examination, you note a petechial rash, mild jaundice, and
splenomegaly. Her lab investigations show hemoglobin of 8.6 g/dL, reticulocytes of 11%, platelet
count of 15*10^9/L, creatinine of 2.9 mg/dL, urea of 40 mg/dL, normal PT and PTT, indirect
hyperbilirubinemia, and LDH of 860 u/L. In addition, her blood film shows numerous schistocytes.
What is the most appropriate next step in the management of this patient?

a. Renal dialysis
b. Start oral aspirin
c. Platelet transfusion
d. Plasmapheresis and fresh frozen plasma
e. Blood transfusion

‫اﻹﺟﺎﺑﺔ ﻋﲆ اﻟﺼﻔﺤﺔ اﻟﺘﺎﻟﻴﺔ‬

Page - 1241
Internal Medicine - Hematology - rombotic rombocytopenic Purpura (TTP)

Question 151/155

Question #151

You are seeing a 40-year-old female patient in the emergency department. She has a fever and
decreased level of consciousness. On examination, you note a petechial rash, mild jaundice, and
splenomegaly. Her lab investigations show hemoglobin of 8.6 g/dL, reticulocytes of 11%, platelet
count of 15*10^9/L, creatinine of 2.9 mg/dL, urea of 40 mg/dL, normal PT and PTT, indirect
hyperbilirubinemia, and LDH of 860 u/L. In addition, her blood film shows numerous schistocytes.
What is the most appropriate next step in the management of this patient?

a. Renal dialysis
b. Start oral aspirin
c. Platelet transfusion
d. Plasmapheresis and fresh frozen plasma √
e. Blood transfusion

Description

is is a typical scenario of rombotic rombocytopenic purpura (TTP)

Treatment of TTP is by emergency plasma exchange.

Platelet transfusion in TTP-HUS can exacerbate microvascular occlusion.

PT, aPTT, D-dimer, and fibrinogen levels are normal in TTP-HUS and abnormal in DIC.

TTP:

Both TTP and HUS are dierent versions of the same basic disease (deficiency of
metalloproteinase ADAMTS 13)
Hemolytic Uremic Syndrome (HUS) is associated with E. coli O157:H7 → more frequent in
children (see pediatrics)

Pathogenesis:

TTP patient has an ADAMTS13 (vWF-cleaving protease) deficiency


TTP can occur in patients with cancer, transplant recipients, HIV, SLE, and those who receive
quinine, clopidogrel, ticlopidine, or cyclosporine.
Page - 1242
Platelets and fibrin adhesion deposited in arterioles lead to the destruction of RBCs when
passing through these arterioles (Microangiopathic hemolytic anemia - MAHA).

Clinical presentation and diagnosis:

Fever
Neurological manifestations (headache, confusion, ataxia, seizures, and mental status and
focal abnormalities)
Microangiopathic hemolytic anemia
Fragmented RBCs (schistocytes)
High reticulocytes and LDH levels
Indirect hyperbilirubinemia
Other features of intravascular hemolysis
Coombs test is negative
rombocytopenia
Renal impairment

Page - 1243
Internal Medicine - Hematology

Question 152/155

Question #152

A 30-year-old male patient with Burkitt’s lymphoma is undergoing chemotherapy. As a result, he


develops tumor lysis syndrome. All the following are expected manifestations except:

a. Hyperuricemia
b. Hyperkalemia
c. Hypercalcemia
d. Hyperphosphatemia
e. Arrhythmias and sudden death

‫اﻹﺟﺎﺑﺔ ﻋﲆ اﻟﺼﻔﺤﺔ اﻟﺘﺎﻟﻴﺔ‬

Page - 1244
Internal Medicine - Hematology - Tumor lysis syndrome

Question 152/155

Question #152

A 30-year-old male patient with Burkitt’s lymphoma is undergoing chemotherapy. As a result, he


develops tumor lysis syndrome. All the following are expected manifestations except:

a. Hyperuricemia
b. Hyperkalemia
c. Hypercalcemia √
d. Hyperphosphatemia
e. Arrhythmias and sudden death

Description

Hypocalcemia, not hypercalcemia, is expected to manifest in tumor lysis syndrome.

Calcium acts as a chelating agent for phosphate leading to reduced calcium level

Tumor lysis syndrome is a side eect of chemotherapy due to the rapid destruction of a large
number of cells

Clinical features include:

Hyperkalemia
Hyperphosphatemia
Hyperuricemia
Hypocalcemia
Acute renal failure

Page - 1245
Internal Medicine - Hematology

Question 153/155

Question #153

A 19-year-old female develops prolonged bleeding aer an appendectomy. Her father and parental
uncle have experienced the same condition. What is the most likely mode of inheritance of this
disorder?

a. Autosomal dominant
b. Autosomal recessive
c. X Linked recessive
d. Y linked
e. Mitochondrial gene defect

‫اﻹﺟﺎﺑﺔ ﻋﲆ اﻟﺼﻔﺤﺔ اﻟﺘﺎﻟﻴﺔ‬

Page - 1246
Internal Medicine - Hematology - Von Willebrand Disease (vWD)

Question 153/155

Question #153

A 19-year-old female develops prolonged bleeding aer an appendectomy. Her father and parental
uncle have experienced the same condition. What is the most likely mode of inheritance of this
disorder?

a. Autosomal dominant √
b. Autosomal recessive
c. X Linked recessive
d. Y linked
e. Mitochondrial gene defect

Description

vWD is the most common inherited bleeding disorder resulting from an autosomal dominant
mutation. As a result, there will be a decreased level of functioning vWF and bleeding tendency.

vWD has 3 main types. Type 1 is the most common (80% of cases) and is inherited autosomal
dominant, while types 2 and 3 are mainly inherited in an autosomal recessive manner. Type 3 is the
most severe form.

Type 1: reduced vWF (autosomal dominant)

Type 2: abnormal quality of vWF (certain subtypes are autosomal recessive)

Type 3: absent vWF (the most severe) (autosomal recessive)

vWF is a protein that has two functions:

Carrier for factor VIII (if deficient vWF → decreased factor VIII in plasma)
Binding platelets to sub-endothelial collagen (primary hemostasis)

e coagulation profile of vWD will be as the following:

High BT
High PTT (50% of cases)
Low Factor 8
Low vWF
Page - 1247
Normal platelet count and PT

Treatment of vWD:

Mild bleeding: Tranexamic acid


Desmopressin (DDAVP) increases the level of vWF
Severe bleeding: Cryoprecipitate or Factor VIII

Page - 1248
Internal Medicine - Hematology

Question 154/155

Question #154

A 22-year-old male patient started to have heavy bleeding aer a tooth extraction. His medical
history reveals easy bruising and gum bleeding aer brushing his teeth. In addition, a family history
of a similar condition in his father is present. What is the most likely diagnosis?

a. Hemophilia A
b. Hemophilia B
c. Von Willebrand’s disease
d. Factor 12 deficiency
e. Factor 13 deficiency

‫اﻹﺟﺎﺑﺔ ﻋﲆ اﻟﺼﻔﺤﺔ اﻟﺘﺎﻟﻴﺔ‬

Page - 1249
Internal Medicine - Hematology - Von Willebrand Disease (vWD)

Question 154/155

Question #154

A 22-year-old male patient started to have heavy bleeding aer a tooth extraction. His medical
history reveals easy bruising and gum bleeding aer brushing his teeth. In addition, a family history
of a similar condition in his father is present. What is the most likely diagnosis?

a. Hemophilia A
b. Hemophilia B
c. Von Willebrand’s disease √
d. Factor 12 deficiency
e. Factor 13 deficiency

Description

Patients with bleeding tendency and positive family history are most likely suering from Von
Willebrand’s disease (vWD)

vWD is the most common inherited bleeding disorder resulting from an autosomal dominant
mutation. As a result, there will be a decreased level of functioning vWF and bleeding tendency.

vWF is a protein that has two functions:

Carrier for factor VIII (if deficient vWF → decreased factor VIII in plasma)
Binding platelets to sub-endothelial collagen (primary hemostasis)

e coagulation profile of vWD will be as the following:

High BT
High PTT (50% of cases)
Low Factor 8
Low vWF
Normal platelet count and PT

Treatment of vWD:

Mild bleeding: Tranexamic acid


Desmopressin (DDAVP) increases the level of vWF
Page - 1250
Severe bleeding: Cryoprecipitate or Factor VIII

Page - 1251
Internal Medicine - Hematology

Question 155/155

Question #155

A 19-year-old female was diagnosed with recurrent prolonged bleeding and ecchymosis secondary
to Von Willebrand’s disease. Which of the following is the most appropriate at this time?

a. Vitamin K
b. Fresh frozen plasma
c. Desmopressin
d. Platelet transfusion
e. Protamine sulfate

‫اﻹﺟﺎﺑﺔ ﻋﲆ اﻟﺼﻔﺤﺔ اﻟﺘﺎﻟﻴﺔ‬

Page - 1252
Internal Medicine - Hematology - Von Willebrand Disease (vWD)

Question 155/155

Question #155

A 19-year-old female was diagnosed with recurrent prolonged bleeding and ecchymosis secondary
to Von Willebrand’s disease. Which of the following is the most appropriate at this time?

a. Vitamin K
b. Fresh frozen plasma
c. Desmopressin √
d. Platelet transfusion
e. Protamine sulfate

Description

vWD is the most common inherited bleeding disorder resulting from an autosomal dominant
mutation. As a result, there will be a decreased level of functioning vWF and bleeding tendency.

vWF is a protein that has two functions:

Carrier for factor VIII (if deficient vWF → decreased factor VIII in plasma)
Binding platelets to sub-endothelial collagen (primary hemostasis)

e coagulation profile of vWD will be as the following:

High BT
High PTT (50% of cases)
Low Factor 8
Low vWF
Normal platelet count and PT

Treatment of vWD:

Mild bleeding: Tranexamic acid


Desmopressin (DDAVP) increases the level of vWF
Severe bleeding: Cryoprecipitate or Factor VIII

Page - 1253
Hepatology

Page - 1254
Hepatology

‫ﻣﻮاﺿﻴﻊ اﻷﺳﺌﻠﺔ وأﻋﺪادﻫﺎ‬

1) Alcoholic liver disease => 1 Questions


2) Ascites => 2 Questions
3) Autoimmune hepatitis => 2 Questions
4) Hemochromatosis => 12 Questions
5) Hepatic Encephalopathy => 3 Questions
6) Hepatitis => 13 Questions
7) Jaundice => 5 Questions
8) Liver Cirrhosis => 3 Questions
9) Liver function tests (LFT) => 3 Questions
10) Non-alcoholic Fatty Liver Disease (NAFLD) => 2 Questions
11) Paracetamol poisoning => 2 Questions
12) Portal HTN and esophageal varices => 6 Questions
13) Primary Biliary Cholangitis (PBC) => 2 Questions
14) Primary Sclerosing Cholangitis (PSC) => 2 Questions
15) Spontaneous Bacterial Peritonitis (SBP) => 3 Questions
16) Wilson Disease => 5 Questions

Page - 1255
Internal Medicine - Hepatology

Question 1/66

Question #1

A 36-year-old alcoholic male patient was referred to you by a family physician because he suspected
alcoholic liver disease. Which of the following is the most sensitive in evaluating this patient?

a. e history of alcoholism
b. Low platelet and high MCV count
c. Alkaline phosphatase level
d. Aspartate to Alanine aminotransferase ratio
e. Direct to total bilirubin level

‫اﻹﺟﺎﺑﺔ ﻋﲆ اﻟﺼﻔﺤﺔ اﻟﺘﺎﻟﻴﺔ‬

Page - 1256
Internal Medicine - Hepatology - Alcoholic liver disease

Question 1/66

Question #1

A 36-year-old alcoholic male patient was referred to you by a family physician because he suspected
alcoholic liver disease. Which of the following is the most sensitive in evaluating this patient?

a. e history of alcoholism
b. Low platelet and high MCV count
c. Alkaline phosphatase level
d. Aspartate to Alanine aminotransferase ratio √
e. Direct to total bilirubin level

Description

Alcoholic liver diseases are 3 conditions: alcoholic fatty liver, alcoholic hepatitis, and alcoholic liver
cirrhosis.

If AST > ALT and AST/ALT ratio > 2, then the alcoholic liver disease is the diagnosis

High MCV level is a less sensitive indicator for alcoholic liver diseases

Page - 1257
Internal Medicine - Hepatology

Question 2/66

Question #2

A 62-year-old female has been heavily alcoholic for 30 years. She presents to your oce complaining
of abdominal distention, shiing dullness, and positive fluid thrill in the abdomen. Which of the
following is false?

a. Sodium restriction is the most important initial treatment


b. Paracentesis is the most accurate test
c. Spironolactone is the most eective diuretic in the treatment
d. Liver cirrhosis presents with high SAAG
e. Spontaneous bacterial peritonitis never presents without abdominal pain

‫اﻹﺟﺎﺑﺔ ﻋﲆ اﻟﺼﻔﺤﺔ اﻟﺘﺎﻟﻴﺔ‬

Page - 1258
Internal Medicine - Hepatology - Ascites

Question 2/66

Question #2

A 62-year-old female has been heavily alcoholic for 30 years. She presents to your oce complaining
of abdominal distention, shiing dullness, and positive fluid thrill in the abdomen. Which of the
following is false?

a. Sodium restriction is the most important initial treatment


b. Paracentesis is the most accurate test
c. Spironolactone is the most eective diuretic in the treatment
d. Liver cirrhosis presents with high SAAG
e. Spontaneous bacterial peritonitis never presents without abdominal pain √

Description

All patients with ascites should have paracentesis to rule out SBP (30% of patients are
asymptomatic)

High SAAG indicates liver cirrhosis, CHF, or Budd-Chiari syndrome, while low SAAG indicates
nephrotic syndrome, malignancy, pancreatitis, or tuberculosis.

Treatment of ascites:

Stop ACE-I, ARB, and NSAIDs in patients with ascites


Treatment of the cause
Sodium restriction
Diuretics (spironolactone is the best, but furosemide may be used)
Paracentesis
TIPSS (Trans-jugular intrahepatic portosystemic shunt)

Page - 1259
Internal Medicine - Hepatology

Question 3/66

Question #3

You ordered Serum Ascites Albumin Gradient (SAAG) for a patient with ascites, the SAAG was 1.3,
and the ascitic protein level was 1.9 g/dL. What is the most likely cause of his ascites?

a. Liver cirrhosis
b. Nephrotic syndrome
c. Congestive heart failure
d. Pancreatitis
e. Peritoneal tuberculosis

‫اﻹﺟﺎﺑﺔ ﻋﲆ اﻟﺼﻔﺤﺔ اﻟﺘﺎﻟﻴﺔ‬

Page - 1260
Internal Medicine - Hepatology - Ascites

Question 3/66

Question #3

You ordered Serum Ascites Albumin Gradient (SAAG) for a patient with ascites, the SAAG was 1.3,
and the ascitic protein level was 1.9 g/dL. What is the most likely cause of his ascites?

a. Liver cirrhosis √
b. Nephrotic syndrome
c. Congestive heart failure
d. Pancreatitis
e. Peritoneal tuberculosis

Description

e above choices can cause ascites, but SAAG and ascitic protein levels would dierentiate the
cause.

e following table shows the dierential diagnosis of ascites according to SAAG and ascitic protein
level.

Page - 1261
Internal Medicine - Hepatology

Question 4/66

Question #4

A 29-year-old female develops secondary amenorrhea for 6 months. Her pregnancy test is negative.
Serum bilirubin is 2.8 mg/dL, ALT is 120 iu/L, AST is 83 iu/L, and Alp is 190 iu/L. What is the most likely
diagnosis?

a. Primary sclerosing cholangitis (PSC)


b. Primary biliary cholangitis (PBC)
c. Gilbert syndrome
d. Autoimmune hepatitis
e. Dubin-Johnson Syndrome

‫اﻹﺟﺎﺑﺔ ﻋﲆ اﻟﺼﻔﺤﺔ اﻟﺘﺎﻟﻴﺔ‬

Page - 1262
Internal Medicine - Hepatology - Autoimmune hepatitis

Question 4/66

Question #4

A 29-year-old female develops secondary amenorrhea for 6 months. Her pregnancy test is negative.
Serum bilirubin is 2.8 mg/dL, ALT is 120 iu/L, AST is 83 iu/L, and Alp is 190 iu/L. What is the most likely
diagnosis?

a. Primary sclerosing cholangitis (PSC)


b. Primary biliary cholangitis (PBC)
c. Gilbert syndrome
d. Autoimmune hepatitis √
e. Dubin-Johnson Syndrome

Description

Consider autoimmune hepatitis in a young woman with hepatitis or hepatic failure who is serology
negative to hepatitis viruses and has other associated autoimmune conditions.

e serum alkaline phosphatase should be normal or minimally elevated in autoimmune hepatitis. A


more than two-fold elevation suggests an alternative or additional diagnosis.

PSC and PBC are associated with higher alkaline phosphatase level

Gilbert syndrome never aects the AST and ALT levels (only isolated unconjugated
hyperbilirubinemia)

Page - 1263
Internal Medicine - Hepatology

Question 5/66

Question #5

A 23-year-old female patient is known to have hypothyroidism and is receiving thyroxine


replacement. She presents with fever, fatigue, and jaundice. Her hepatitis serology test is negative,
serum bilirubin is 2.8 mg/dL, ALT is 105 iu/L, ALP is 200 iu/L, and her PT and INR are prolonged. What
is the most likely diagnosis?

a. Drug-induced hepatitis
b. Autoimmune hepatitis
c. Acute liver failure
d. Alcoholic liver disease
e. Hyperacute liver failure

‫اﻹﺟﺎﺑﺔ ﻋﲆ اﻟﺼﻔﺤﺔ اﻟﺘﺎﻟﻴﺔ‬

Page - 1264
Internal Medicine - Hepatology - Autoimmune hepatitis

Question 5/66

Question #5

A 23-year-old female patient is known to have hypothyroidism and is receiving thyroxine


replacement. She presents with fever, fatigue, and jaundice. Her hepatitis serology test is negative,
serum bilirubin is 2.8 mg/dL, ALT is 105 iu/L, ALP is 200 iu/L, and her PT and INR are prolonged. What
is the most likely diagnosis?

a. Drug-induced hepatitis
b. Autoimmune hepatitis √
c. Acute liver failure
d. Alcoholic liver disease
e. Hyperacute liver failure

Description

Consider autoimmune hepatitis in a young woman with hepatitis or hepatic failure who is serology
negative to hepatitis viruses and has other associated autoimmune conditions.

yroid disease here is the associated autoimmune disease.

e serum alkaline phosphatase should be normal or minimally elevated in autoimmune hepatitis. A


more than two-fold elevation suggests an alternative or additional diagnosis.

Page - 1265
Internal Medicine - Hepatology

Question 6/66

Question #6

A patient is suspected of having hereditary hemochromatosis. Which of the following is the best
initial test in his management?

a. Serum ferritin
b. Serum transferrin saturation
c. Serum ceruloplasmin level
d. serum Iron level
e. HFE gene mutation

‫اﻹﺟﺎﺑﺔ ﻋﲆ اﻟﺼﻔﺤﺔ اﻟﺘﺎﻟﻴﺔ‬

Page - 1266
Internal Medicine - Hepatology - Hemochromatosis

Question 6/66

Question #6

A patient is suspected of having hereditary hemochromatosis. Which of the following is the best
initial test in his management?

a. Serum ferritin
b. Serum transferrin saturation √
c. Serum ceruloplasmin level
d. serum Iron level
e. HFE gene mutation

Description

e most appropriate screening test for hereditary hemochromatosis is fasting serum transferrin
saturation

Hemochromatosis (also known as bronze diabetes) is an autosomal recessive disorder; HFE gene
mutation leads to increased iron absorption.

e iron will accumulate in the body and deposit in the liver, heart, skin, pancreas, testes, and
endocrine glands.

Symptoms usually start at the age of 40 years and will show:

Fatigue and Arthropathy (early symptoms)


DM, HF, arrhythmias, Impotence, loss of libido, testicular atrophy
Destructive Arthropathy of the second MCP joints with Hook-like osteophytes
Unusual sites Osteoarthritis (e.g., ankle, shoulder, etc.)
Liver cirrhosis may be complicated by Hepatocellular carcinoma
Gray skin pigmentation (bronzed diabetes)

Page - 1267
Internal Medicine - Hepatology

Question 7/66

Question #7

A 40-year-old male presents with fatigue and darkening of his skin, his liver is enlarged, and his
urine dipstick shows glucosuria. His father died 12 years ago of liver cirrhosis. Which of the following
would help in the diagnosis?

a. Serum ceruloplasmin level


b. Alpha-1-antitrypsin level
c. Ferritin level
d. Schilling test
e. Karyotyping

‫اﻹﺟﺎﺑﺔ ﻋﲆ اﻟﺼﻔﺤﺔ اﻟﺘﺎﻟﻴﺔ‬

Page - 1268
Internal Medicine - Hepatology - Hemochromatosis

Question 7/66

Question #7

A 40-year-old male presents with fatigue and darkening of his skin, his liver is enlarged, and his
urine dipstick shows glucosuria. His father died 12 years ago of liver cirrhosis. Which of the following
would help in the diagnosis?

a. Serum ceruloplasmin level


b. Alpha-1-antitrypsin level
c. Ferritin level √
d. Schilling test
e. Karyotyping

Description

e presence of dark skin and the family history of liver diseases strongly suggest hemochromatosis
as the most likely diagnosis.

Symptoms usually start at the age of 40 years and will show:

Fatigue and Arthropathy (early symptoms)


DM, HF, arrhythmias, Impotence, loss of libido, testicular atrophy
Destructive Arthropathy of the second MCP joints with Hook-like osteophytes
Unusual sites Osteoarthritis (e.g., ankle, shoulder, etc.)
Liver cirrhosis may be complicated by Hepatocellular carcinoma
Gray skin pigmentation (bronzed diabetes)

Page - 1269
Internal Medicine - Hepatology

Question 8/66

Question #8

A 42-year-old female develops polyuria, polyphagia, and polydipsia. His father died due to liver
cirrhosis at the age of 55 years. On examination, the patient has dark skin and hepatomegaly. What
is the most likely diagnosis?

a. Wilson disease
b. Hemochromatosis
c. Addison disease
d. Alpha-1-antitrypsin deficiency
e. Hepatocellular carcinoma

‫اﻹﺟﺎﺑﺔ ﻋﲆ اﻟﺼﻔﺤﺔ اﻟﺘﺎﻟﻴﺔ‬

Page - 1270
Internal Medicine - Hepatology - Hemochromatosis

Question 8/66

Question #8

A 42-year-old female develops polyuria, polyphagia, and polydipsia. His father died due to liver
cirrhosis at the age of 55 years. On examination, the patient has dark skin and hepatomegaly. What
is the most likely diagnosis?

a. Wilson disease
b. Hemochromatosis √
c. Addison disease
d. Alpha-1-antitrypsin deficiency
e. Hepatocellular carcinoma

Description

Hemochromatosis (also known as bronze diabetes) is an autosomal recessive disorder; HFE gene
mutation leads to increased iron absorption.

e iron will accumulate in the body and deposit in the liver, heart, skin, pancreas, testes, and
endocrine glands.

Symptoms usually start at the age of 40 years and will show:

Fatigue and Arthropathy (early symptoms)


DM, HF, arrhythmias, Impotence, loss of libido, testicular atrophy
Destructive Arthropathy of the second MCP joints with Hook-like osteophytes
Unusual sites Osteoarthritis (e.g., ankle, shoulder, etc.)
Liver cirrhosis may be complicated by Hepatocellular carcinoma
Gray skin pigmentation (bronzed diabetes)

Page - 1271
Internal Medicine - Hepatology

Question 9/66

Question #9

A 43-year-old male patient develops gynecomastia and fatigue. On examination, he has dark skin,
hepatomegaly, and palmar erythema. His lab tests show an HbA1c level of 8%. What is the most
likely diagnosis?

a. Idiopathic gynecomastia
b. Hemochromatosis
c. Wilson disease
d. Hepatocellular carcinoma
e. Chronic hepatitis

‫اﻹﺟﺎﺑﺔ ﻋﲆ اﻟﺼﻔﺤﺔ اﻟﺘﺎﻟﻴﺔ‬

Page - 1272
Internal Medicine - Hepatology - Hemochromatosis

Question 9/66

Question #9

A 43-year-old male patient develops gynecomastia and fatigue. On examination, he has dark skin,
hepatomegaly, and palmar erythema. His lab tests show an HbA1c level of 8%. What is the most
likely diagnosis?

a. Idiopathic gynecomastia
b. Hemochromatosis √
c. Wilson disease
d. Hepatocellular carcinoma
e. Chronic hepatitis

Description

Gynecomastia, hepatomegaly, and palmar erythema suggest chronic liver disease.

Chronic liver disease, DM, and bronze skin strongly suggest hemochromatosis

Hemochromatosis (also known as bronze diabetes) is an autosomal recessive disorder; HFE gene
mutation leads to increased iron absorption.

e iron will accumulate in the body and deposit in the liver, heart, skin, pancreas, testes, and
endocrine glands.

Symptoms usually start at the age of 40 years and will show:

Fatigue and Arthropathy (early symptoms)


DM, HF, arrhythmias, Impotence, loss of libido, testicular atrophy
Destructive Arthropathy of the second MCP joints with Hook-like osteophytes
Unusual sites Osteoarthritis (e.g., ankle, shoulder, etc.)
Liver cirrhosis may be complicated by Hepatocellular carcinoma
Gray skin pigmentation (bronzed diabetes)

Page - 1273
Internal Medicine - Hepatology

Question 10/66

Question #10

A 41-year-old male patient has DM, heart failure, chronic liver disease, and bronze skin. He was
diagnosed with hemochromatosis. He asked you about the chance that his children may have the
condition. e most appropriate answer is:

a. All of his sons will get the disease


b. All of his daughters will get the disease
c. All his sons and daughters will be carriers of the disease
d. 50% of his children will get the disease
e. 25% of his children will get the disease

‫اﻹﺟﺎﺑﺔ ﻋﲆ اﻟﺼﻔﺤﺔ اﻟﺘﺎﻟﻴﺔ‬

Page - 1274
Internal Medicine - Hepatology - Hemochromatosis

Question 10/66

Question #10

A 41-year-old male patient has DM, heart failure, chronic liver disease, and bronze skin. He was
diagnosed with hemochromatosis. He asked you about the chance that his children may have the
condition. e most appropriate answer is:

a. All of his sons will get the disease


b. All of his daughters will get the disease
c. All his sons and daughters will be carriers of the disease √
d. 50% of his children will get the disease
e. 25% of his children will get the disease

Description

Knowing that hemochromatosis is an autosomal recessive condition, the patient will have (hh)
genotype.

Assuming his mother is a normal (HH) genotype, all his children will be (Hh).

(Hh) genotype is heterozygous; it indicates carrier status.

Page - 1275
Internal Medicine - Hepatology

Question 11/66

Question #11

A 45-year-old male patient develops arthralgia, fatigue, and erectile dysfunction. His physical
examination reveals hepatomegaly, and his lab investigations show elevated transaminases. What
is the most likely diagnosis?

a. Wilson disease
b. Hemochromatosis
c. Addison disease
d. Alpha-1-antitrypsin deficiency
e. Hepatocellular carcinoma

‫اﻹﺟﺎﺑﺔ ﻋﲆ اﻟﺼﻔﺤﺔ اﻟﺘﺎﻟﻴﺔ‬

Page - 1276
Internal Medicine - Hepatology - Hemochromatosis

Question 11/66

Question #11

A 45-year-old male patient develops arthralgia, fatigue, and erectile dysfunction. His physical
examination reveals hepatomegaly, and his lab investigations show elevated transaminases. What
is the most likely diagnosis?

a. Wilson disease
b. Hemochromatosis √
c. Addison disease
d. Alpha-1-antitrypsin deficiency
e. Hepatocellular carcinoma

Description

Hemochromatosis (also known as bronze diabetes) is an autosomal recessive disorder; HFE gene
mutation leads to increased iron absorption.

e iron will accumulate in the body and deposit in the liver, heart, skin, pancreas, testes, and
endocrine glands.

Symptoms usually start at the age of 40 years and will show:

Fatigue and Arthropathy (early symptoms)


DM, HF, arrhythmias, Impotence, loss of libido, testicular atrophy
Destructive Arthropathy of the second MCP joints with Hook-like osteophytes
Unusual sites Osteoarthritis (e.g., ankle, shoulder, etc.)
Liver cirrhosis may be complicated by Hepatocellular carcinoma
Gray skin pigmentation (bronzed diabetes)

Page - 1277
Internal Medicine - Hepatology

Question 12/66

Question #12

A 40-year-old man develops orthopnea and fatigue. Recently, he was diagnosed with diabetes, for
which he received insulin therapy. On examination, the patient has lung crepitation, hepatomegaly,
and bronze skin. In addition, his blood tests show a high ferritin level. What is the most likely
diagnosis?

a. Ischemic heart disease


b. Wilson disease
c. Hemochromatosis
d. Addison disease
e. Alpha-1-antitrypsin deficiency

‫اﻹﺟﺎﺑﺔ ﻋﲆ اﻟﺼﻔﺤﺔ اﻟﺘﺎﻟﻴﺔ‬

Page - 1278
Internal Medicine - Hepatology - Hemochromatosis

Question 12/66

Question #12

A 40-year-old man develops orthopnea and fatigue. Recently, he was diagnosed with diabetes, for
which he received insulin therapy. On examination, the patient has lung crepitation, hepatomegaly,
and bronze skin. In addition, his blood tests show a high ferritin level. What is the most likely
diagnosis?

a. Ischemic heart disease


b. Wilson disease
c. Hemochromatosis √
d. Addison disease
e. Alpha-1-antitrypsin deficiency

Description

Hemochromatosis (also known as bronze diabetes) is an autosomal recessive disorder; HFE gene
mutation leads to increased iron absorption.

e iron will accumulate in the body and deposit in the liver, heart, skin, pancreas, testes, and
endocrine glands.

Symptoms usually start at the age of 40 years and will show:

Fatigue and Arthropathy (early symptoms)


DM, HF, arrhythmias, Impotence, loss of libido, testicular atrophy
Destructive Arthropathy of the second MCP joints with Hook-like osteophytes
Unusual sites Osteoarthritis (e.g., ankle, shoulder, etc.)
Liver cirrhosis may be complicated by Hepatocellular carcinoma
Gray skin pigmentation (bronzed diabetes)

Page - 1279
Internal Medicine - Hepatology

Question 13/66

Question #13

A 40-year-old male patient presents with a darkening of his skin. His history is significant for
paternal death with liver cirrhosis, and his physical examination shows hepatomegaly. What is the
most likely diagnosis?

a. Alpha-1-antitrypsin deficiency
b. Hemochromatosis
c. Hepatocellular carcinoma
d. Liver cirrhosis
e. Wilson disease

‫اﻹﺟﺎﺑﺔ ﻋﲆ اﻟﺼﻔﺤﺔ اﻟﺘﺎﻟﻴﺔ‬

Page - 1280
Internal Medicine - Hepatology - Hemochromatosis

Question 13/66

Question #13

A 40-year-old male patient presents with a darkening of his skin. His history is significant for
paternal death with liver cirrhosis, and his physical examination shows hepatomegaly. What is the
most likely diagnosis?

a. Alpha-1-antitrypsin deficiency
b. Hemochromatosis √
c. Hepatocellular carcinoma
d. Liver cirrhosis
e. Wilson disease

Description

e presence of dark skin and the family history of liver diseases strongly suggest hemochromatosis
as the most likely diagnosis.

Hemochromatosis (also known as bronze diabetes) is an autosomal recessive disorder; HFE gene
mutation leads to increased iron absorption.

e iron will accumulate in the body and deposit in the liver, heart, skin, pancreas, testes, and
endocrine glands.

Symptoms usually start at the age of 40 years and will show:

Fatigue and Arthropathy (early symptoms)


DM, HF, arrhythmias, Impotence, loss of libido, testicular atrophy
Destructive Arthropathy of the second MCP joints with Hook-like osteophytes
Unusual sites Osteoarthritis (e.g., ankle, shoulder, etc.)
Liver cirrhosis may be complicated by Hepatocellular carcinoma
Gray skin pigmentation (bronzed diabetes)

Page - 1281
Internal Medicine - Hepatology

Question 14/66

Question #14

A 41-year-old male patient has DM, heart failure, chronic liver disease, and bronze skin. As a result,
he was diagnosed with hemochromatosis. He asked you about the chance that his children may
have the disease. What is the treatment of choice for this condition?

a. Blood transfusion
b. A chelating agent (penicillamine)
c. Recurrent phlebotomy
d. Only insulin and digoxin
e. Ferrous sulfate oral tablets

‫اﻹﺟﺎﺑﺔ ﻋﲆ اﻟﺼﻔﺤﺔ اﻟﺘﺎﻟﻴﺔ‬

Page - 1282
Internal Medicine - Hepatology - Hemochromatosis

Question 14/66

Question #14

A 41-year-old male patient has DM, heart failure, chronic liver disease, and bronze skin. As a result,
he was diagnosed with hemochromatosis. He asked you about the chance that his children may
have the disease. What is the treatment of choice for this condition?

a. Blood transfusion
b. A chelating agent (penicillamine)
c. Recurrent phlebotomy √
d. Only insulin and digoxin
e. Ferrous sulfate oral tablets

Description

Hemochromatosis (also known as bronze diabetes) is an autosomal recessive disorder; HFE gene
mutation leads to increased iron absorption.

e management of hemochromatosis is by recurrent phlebotomy (venesection) weekly 500 ml (250


mg iron) – until serum iron is normal, then as required

Aer treatment, the liver and heart will improve, but DM is irreversible

Blood transfusion and ferrous sulfate will exacerbate the condition and cause more iron load.

Penicillamine is a cooper chelating agent used in Wilson disease, not hemochromatosis.

Insulin and digoxin will control heart failure and DM, but without reducing the body’s iron, they will
not control the patient’s condition

Page - 1283
Internal Medicine - Hepatology

Question 15/66

Question #15

A 45-year-old female is found to have abnormal liver function tests. Her liver biopsy demonstrates a
considerable amount of iron in the hepatocytes. What is the most likely diagnosis?

a. Wilsons disease
b. Alpha-1-antitrypsin deficiency
c. Hemochromatosis
d. McArdle’s disease
e. Autoimmune hepatitis

‫اﻹﺟﺎﺑﺔ ﻋﲆ اﻟﺼﻔﺤﺔ اﻟﺘﺎﻟﻴﺔ‬

Page - 1284
Internal Medicine - Hepatology - Hemochromatosis

Question 15/66

Question #15

A 45-year-old female is found to have abnormal liver function tests. Her liver biopsy demonstrates a
considerable amount of iron in the hepatocytes. What is the most likely diagnosis?

a. Wilsons disease
b. Alpha-1-antitrypsin deficiency
c. Hemochromatosis √
d. McArdle’s disease
e. Autoimmune hepatitis

Description

e presence of high iron content in Hepatocytes (the liver cells) is diagnostic of hemochromatosis.

Hemochromatosis (also known as bronze diabetes) is an autosomal recessive disorder; HFE gene
mutation leads to increased iron absorption.

e iron will accumulate in the body and deposit in the liver, heart, skin, pancreas, testes, and
endocrine glands.

Symptoms usually start at the age of 40 years and will show:

Fatigue and Arthropathy (early symptoms)


DM, HF, arrhythmias, Impotence, loss of libido, testicular atrophy
Destructive Arthropathy of the second MCP joints with Hook-like osteophytes
Unusual sites Osteoarthritis (e.g., ankle, shoulder, etc.)
Liver cirrhosis may be complicated by Hepatocellular carcinoma
Gray skin pigmentation (bronzed diabetes)

Page - 1285
Internal Medicine - Hepatology

Question 16/66

Question #16

A 45-year-old male patient is a known case of hemochromatosis. He presents with hematemesis. On


examination, the patient is confused and drowsy. His blood pressure is 112/80, the pulse is 113 bpm,
and the spleen is palpable. Which of the following is true?

a. e patient has liver cirrhosis


b. e patient mostly has portal HTN
c. e patient mostly has bleeding esophageal varices
d. e patient has hepatic encephalopathy
e. All of the above

‫اﻹﺟﺎﺑﺔ ﻋﲆ اﻟﺼﻔﺤﺔ اﻟﺘﺎﻟﻴﺔ‬

Page - 1286
Internal Medicine - Hepatology - Hemochromatosis

Question 16/66

Question #16

A 45-year-old male patient is a known case of hemochromatosis. He presents with hematemesis. On


examination, the patient is confused and drowsy. His blood pressure is 112/80, the pulse is 113 bpm,
and the spleen is palpable. Which of the following is true?

a. e patient has liver cirrhosis


b. e patient mostly has portal HTN
c. e patient mostly has bleeding esophageal varices
d. e patient has hepatic encephalopathy
e. All of the above √

Description

Splenomegaly suggests portal HTN

e presence of hemochromatosis in this scenario makes liver cirrhosis more likely.

e presence of hematemesis in a patient with liver cirrhosis suggests bleeding esophageal varices

e presence of confusion and upper GI bleeding in a patient with liver cirrhosis suggest hepatic
encephalopathy.

Note that shock is a possible cause of confusion, but confusion results aer the drop in blood
pressure (unlike this case).

Page - 1287
Internal Medicine - Hepatology

Question 17/66

Question #17

A 40-year-old man develops orthopnea and fatigue. Recently, he was diagnosed with diabetes, for
which he received insulin therapy. On examination, the patient has lung crepitation, hepatomegaly,
and bronze skin. His blood tests show a high ferritin level. However, the patient refuses any
treatment. Which of the following organs would most likely develop cancer in this patient?

a. e pancreas
b. e heart
c. e liver
d. e testes
e. e skin

‫اﻹﺟﺎﺑﺔ ﻋﲆ اﻟﺼﻔﺤﺔ اﻟﺘﺎﻟﻴﺔ‬

Page - 1288
Internal Medicine - Hepatology - Hemochromatosis

Question 17/66

Question #17

A 40-year-old man develops orthopnea and fatigue. Recently, he was diagnosed with diabetes, for
which he received insulin therapy. On examination, the patient has lung crepitation, hepatomegaly,
and bronze skin. His blood tests show a high ferritin level. However, the patient refuses any
treatment. Which of the following organs would most likely develop cancer in this patient?

a. e pancreas
b. e heart
c. e liver √
d. e testes
e. e skin

Description

Hemochromatosis is a known cause of liver cirrhosis and liver cancer.

Hemochromatosis (also known as bronze diabetes) is an autosomal recessive disorder; HFE gene
mutation leads to increased iron absorption.

e iron will accumulate in the body and deposit in the liver, heart, skin, pancreas, testes, and
endocrine glands.

Symptoms usually start at the age of 40 years and will show:

Fatigue and Arthropathy (early symptoms)


DM, HF, arrhythmias, Impotence, loss of libido, testicular atrophy
Destructive Arthropathy of the second MCP joints with Hook-like osteophytes
Unusual sites Osteoarthritis (e.g., ankle, shoulder, etc.)
Liver cirrhosis may be complicated by Hepatocellular carcinoma
Gray skin pigmentation (bronzed diabetes)

Page - 1289
Internal Medicine - Hepatology

Question 18/66

Question #18

A 55-year-old male patient comes to the emergency department with bloody vomiting and
confusion. His history includes hemochromatosis and diabetes. On examination, the patient is
flushed and has a blood pressure of 120/80 mmHg. e pulse is 110 bpm, the abdominal wall shows a
venous pattern, and the patient is drowsy and confused with no neurological deficit. Which of the
following is not used in the treatment of this patient?

a. Oral lactulose and lactulose enema


b. Intravenous antibiotics (Rifaximin)
c. Intravenous fluid
d. Keep the patient NPO
e. Endoscopic banding of the esophageal varices

‫اﻹﺟﺎﺑﺔ ﻋﲆ اﻟﺼﻔﺤﺔ اﻟﺘﺎﻟﻴﺔ‬

Page - 1290
Internal Medicine - Hepatology - Hepatic Encephalopathy

Question 18/66

Question #18

A 55-year-old male patient comes to the emergency department with bloody vomiting and
confusion. His history includes hemochromatosis and diabetes. On examination, the patient is
flushed and has a blood pressure of 120/80 mmHg. e pulse is 110 bpm, the abdominal wall shows a
venous pattern, and the patient is drowsy and confused with no neurological deficit. Which of the
following is not used in the treatment of this patient?

a. Oral lactulose and lactulose enema


b. Intravenous antibiotics (Rifaximin) √
c. Intravenous fluid
d. Keep the patient NPO
e. Endoscopic banding of the esophageal varices

Description

Rifaximin is a non-absorbable antibiotic used in cases like hepatic encephalopathy, traveler’s


diarrhea, and IBS with diarrhea.

It is administrated orally, no intravenously

is patient should be treated for both upper GI bleeding and hepatic encephalopathy.

IV fluids and the endoscopic band are used to control bleeding (mostly due to varices in this
patient).
NPO, lactulose, and Rifaximin are used to treat hepatic encephalopathy.

e following is a summary of the treatment of hepatic encephalopathy:

Treatment of the precipitating cause


Lactulose (laxative)
Decrease colon PH → decrease ammonia absorption
Decrease protein content in the GI tract
Antibiotics (Rifaximin 400 mg TID), Non-absorbable antibiotic, Decrease GUT bacteria → less
ammonia

Page - 1291
Internal Medicine - Hepatology

Question 19/66

Question #19

A patient with liver cirrhosis had a heavy protein meal yesterday. Today, he has developed
drowsiness and aggressive behavior. Which of the following is false about this condition?

a. Plasma ammonia level is elevated in this patient


b. EEG will show slow alpha waves
c. Loperamide should be administrated in this patient
d. is patient has grade 2 encephalopathy
e. Lactulose and antibiotics are required for the treatment

‫اﻹﺟﺎﺑﺔ ﻋﲆ اﻟﺼﻔﺤﺔ اﻟﺘﺎﻟﻴﺔ‬

Page - 1292
Internal Medicine - Hepatology - Hepatic Encephalopathy

Question 19/66

Question #19

A patient with liver cirrhosis had a heavy protein meal yesterday. Today, he has developed
drowsiness and aggressive behavior. Which of the following is false about this condition?

a. Plasma ammonia level is elevated in this patient


b. EEG will show slow alpha waves
c. Loperamide should be administrated in this patient √
d. is patient has grade 2 encephalopathy
e. Lactulose and antibiotics are required for the treatment

Description

is patient has hepatic encephalopathy.

Loperamide is an antidiarrheal agent that is not appropriate in this case.

Loperamide will increase constipation and the protein’s length of stay in the gut, leading to more
ammonia production.

e following is a summary of the treatment of hepatic encephalopathy:

Treatment of the precipitating cause


Lactulose (laxative)
Decrease colon PH → decrease ammonia absorption
Decrease protein content in the GI tract
Antibiotics (Rifaximin 400 mg TID), Non-absorbable antibiotic, Decrease GUT bacteria → less
ammonia

Page - 1293
Internal Medicine - Hepatology

Question 20/66

Question #20

A 55-year-old male patient comes to the emergency department with bloody vomiting and
confusion. His history includes hemochromatosis and diabetes. On examination, the patient is
flushed and has a blood pressure of 120/80 mmHg. e pulse is 110 bpm, the abdominal wall shows a
venous pattern, and the patient is drowsy and confused with no neurological deficit. What is the
most likely diagnosis?

a. Stroke
b. Subarachnoid hemorrhage
c. Hepatic encephalopathy
d. Hypoglycemia
e. Vitamin B12 deficiency

‫اﻹﺟﺎﺑﺔ ﻋﲆ اﻟﺼﻔﺤﺔ اﻟﺘﺎﻟﻴﺔ‬

Page - 1294
Internal Medicine - Hepatology - Hepatic Encephalopathy

Question 20/66

Question #20

A 55-year-old male patient comes to the emergency department with bloody vomiting and
confusion. His history includes hemochromatosis and diabetes. On examination, the patient is
flushed and has a blood pressure of 120/80 mmHg. e pulse is 110 bpm, the abdominal wall shows a
venous pattern, and the patient is drowsy and confused with no neurological deficit. What is the
most likely diagnosis?

a. Stroke
b. Subarachnoid hemorrhage
c. Hepatic encephalopathy √
d. Hypoglycemia
e. Vitamin B12 deficiency

Description

e history of hemochromatosis and the physical finding of abdominal wall venous pattern (caput-
medusae) strongly suggest liver cirrhosis in this patient.

Bloody vomiting is suggestive of upper GI bleeding.

e GI bleeding will increase the protein content in the intestine and increase ammonia production
by the intestinal bacteria.

e ammonia level in the blood will increase due to two mechanisms:

e cirrhotic liver can’t process ammonia


e blood is shunted from the portal to systemic circulation.

As a result, ammonia will accumulate in the brain and cause neurological symptoms (confusion and
drowsiness).

Page - 1295
Internal Medicine - Hepatology

Question 21/66

Question #21

A 35-year-old female was diagnosed with latent TB and started INH treatment 3 months ago. Today,
her liver function test shows significantly elevated serum AST, ALT, and minimally elevated Alkaline
phosphatase. Which of the following is the most likely diagnosis?

a. Gilbert syndrome
b. Viral hepatitis
c. Drug-induced hepatitis
d. Liver cirrhosis
e. Obstructive liver disease

‫اﻹﺟﺎﺑﺔ ﻋﲆ اﻟﺼﻔﺤﺔ اﻟﺘﺎﻟﻴﺔ‬

Page - 1296
Internal Medicine - Hepatology - Hepatitis

Question 21/66

Question #21

A 35-year-old female was diagnosed with latent TB and started INH treatment 3 months ago. Today,
her liver function test shows significantly elevated serum AST, ALT, and minimally elevated Alkaline
phosphatase. Which of the following is the most likely diagnosis?

a. Gilbert syndrome
b. Viral hepatitis
c. Drug-induced hepatitis √
d. Liver cirrhosis
e. Obstructive liver disease

Description

e significantly elevated AST and ALT, along with minimal Alkaline phosphatase elevation, indicate
hepatitis.

Note that the timing here is important, and the presence of hepatitis features aer INH
administration is a clue.

INH is neurotoxic and hepatotoxic

Gilbert syndrome will present with normal AST, ALT, and Alkaline phosphatase (only minimally
elevated indirect bilirubin level)

Viral hepatitis is still a possibility here, but the presence of hepatotoxic drug history here is
important

e obstructive liver disease presents with minimal elevation of AST and ALT and significant
elevation of the Alkaline phosphatase.

Liver cirrhosis is a chronic condition, and no signs of chronic liver disease here to suspect this
diagnosis

Page - 1297
Internal Medicine - Hepatology

Question 22/66

Question #22

A 32-year-old female with multiple sex partners was investigated for general fatigue and anorexia.
e blood tests show the following: negative HBsAg, positive HBcAb, positive Anti-HBs, positive
Anti-HCV, and HCV RNA is detected. In addition, liver biopsy demonstrated moderate inflammation
with steatosis. What is the most likely cause for these findings?

a. Current HBV infection


b. Current HCV infection
c. Concomitant HBV and HCV infection
d. HIV infection
e. CMV hepatitis

‫اﻹﺟﺎﺑﺔ ﻋﲆ اﻟﺼﻔﺤﺔ اﻟﺘﺎﻟﻴﺔ‬

Page - 1298
Internal Medicine - Hepatology - Hepatitis

Question 22/66

Question #22

A 32-year-old female with multiple sex partners was investigated for general fatigue and anorexia.
e blood tests show the following: negative HBsAg, positive HBcAb, positive Anti-HBs, positive
Anti-HCV, and HCV RNA is detected. In addition, liver biopsy demonstrated moderate inflammation
with steatosis. What is the most likely cause for these findings?

a. Current HBV infection


b. Current HCV infection √
c. Concomitant HBV and HCV infection
d. HIV infection
e. CMV hepatitis

Description

e patient has recovered from hepatitis B (the anti-HBs and anti-HBc indicate a previous infection)

Hepatitis C antibodies and RNA indicate active infection with the hepatitis C virus

Anti-HCV indicates infection, while HCV RNA indicates active disease

e following table shows dierent laboratory findings in HRV infection and immunity and their
results.

Page - 1299
Internal Medicine - Hepatology

Question 23/66

Question #23

A previously healthy male patient presents for follow-up. On examination, he is afebrile with normal
vital signs, and his body mass index is 42 Kg/M2. His AST and ALT today are 75 and 103 U/L,
respectively. Which of the following is the best next step in management?

a. Liver biopsy
b. Liver CT scan
c. Liver Ultrasound
d. Testing for viral hepatitis
e. Repeat AST and ALT aer 6 months

‫اﻹﺟﺎﺑﺔ ﻋﲆ اﻟﺼﻔﺤﺔ اﻟﺘﺎﻟﻴﺔ‬

Page - 1300
Internal Medicine - Hepatology - Hepatitis

Question 23/66

Question #23

A previously healthy male patient presents for follow-up. On examination, he is afebrile with normal
vital signs, and his body mass index is 42 Kg/M2. His AST and ALT today are 75 and 103 U/L,
respectively. Which of the following is the best next step in management?

a. Liver biopsy
b. Liver CT scan
c. Liver Ultrasound
d. Testing for viral hepatitis √
e. Repeat AST and ALT aer 6 months

Description

Even though the most likely diagnosis is a non-alcoholic fatty liver disease (NAFLD), hepatitis B and C
infection should be ruled out first

A liver ultrasound should be done if hepatitis B and C tests are negative, it will show increased
echogenicity of the liver if there is a fatty liver disease

A liver biopsy is not appropriate initially at this time

A Liver CT scan is appropriate if a mass is found during ultrasonography.

Page - 1301
Internal Medicine - Hepatology

Question 24/66

Question #24

A 32-year-old nurse presents with fatigue and anorexia for several days; his lab investigations show
an ALT of 1438 U/L. You performed a hepatitis profile which showed positive HBsAg, Positive anti-
HBc IgM. What is the most likely diagnosis?

a. Acute HBV infection


b. Chronic HBV infection
c. e patient has a previous HBV infection
d. e patient is recovering from an HBV infection
e. e patient is vaccinated against HBV

‫اﻹﺟﺎﺑﺔ ﻋﲆ اﻟﺼﻔﺤﺔ اﻟﺘﺎﻟﻴﺔ‬

Page - 1302
Internal Medicine - Hepatology - Hepatitis

Question 24/66

Question #24

A 32-year-old nurse presents with fatigue and anorexia for several days; his lab investigations show
an ALT of 1438 U/L. You performed a hepatitis profile which showed positive HBsAg, Positive anti-
HBc IgM. What is the most likely diagnosis?

a. Acute HBV infection √


b. Chronic HBV infection
c. e patient has a previous HBV infection
d. e patient is recovering from an HBV infection
e. e patient is vaccinated against HBV

Description

HBsAg means infection, while IgM anti-HBc means acute.

e following table shows dierent laboratory findings in HRV infection and immunity and their
results.

Page - 1303
Internal Medicine - Hepatology

Question 25/66

Question #25

Which of the following laboratory tests will first become abnormal in the case of acute hepatitis B
infection?

a. ALT and AST elevation


b. Anti-HBc ab
c. Anti-Hbe ab
d. HBsAg
e. Serum bilirubin level

‫اﻹﺟﺎﺑﺔ ﻋﲆ اﻟﺼﻔﺤﺔ اﻟﺘﺎﻟﻴﺔ‬

Page - 1304
Internal Medicine - Hepatology - Hepatitis

Question 25/66

Question #25

Which of the following laboratory tests will first become abnormal in the case of acute hepatitis B
infection?

a. ALT and AST elevation


b. Anti-HBc ab
c. Anti-Hbe ab
d. HBsAg √
e. Serum bilirubin level

Description

e first abnormal serology test that occurs in Hepatitis B infection is HBsAg.

HBeAg and HBV DNA appear soon aer HBsAg.

ALT and Anti-HBc (IgM) appear simultaneously aer the onset of symptoms.

Page - 1305
Internal Medicine - Hepatology

Question 26/66

Question #26

Aer his blood donation, a 22-year-old male patient was informed that his routine blood screening
showed positive HBsAg and Anti-HBc with negative anti-HBs antibodies. He presents to you for
consultation regarding these results. What would you tell him?

a. He has an acute infection of HBV


b. He has an acute or chronic infection of HBV
c. He is immune to HBV and can donate his blood again
d. He is susceptible to HBV infection
e. e patient should be vaccinated against HBV immediately

‫اﻹﺟﺎﺑﺔ ﻋﲆ اﻟﺼﻔﺤﺔ اﻟﺘﺎﻟﻴﺔ‬

Page - 1306
Internal Medicine - Hepatology - Hepatitis

Question 26/66

Question #26

Aer his blood donation, a 22-year-old male patient was informed that his routine blood screening
showed positive HBsAg and Anti-HBc with negative anti-HBs antibodies. He presents to you for
consultation regarding these results. What would you tell him?

a. He has an acute infection of HBV


b. He has an acute or chronic infection of HBV √
c. He is immune to HBV and can donate his blood again
d. He is susceptible to HBV infection
e. e patient should be vaccinated against HBV immediately

Description

is patient has acute or chronic HBV infection and should never donate his blood.

Vaccination will do nothing aer the patient’s infection, but it is recommended aer a needle stick,
blood reception, or sexual intercourse with someone who already has an active HBV infection. In
addition, it is recommended for babies of HBV mothers aer delivery.

Page - 1307
Internal Medicine - Hepatology

Question 27/66

Question #27

A 17-year-old male patient is suspected of having acute hepatitis A infection. What is the most
important test to be done for diagnosis?

a. Anti-HAV IgG antibody


b. Serum bilirubin level
c. Anti-HAV IgM antibody
d. AST, ALT, and Alkaline phosphatase
e. PT and INR

‫اﻹﺟﺎﺑﺔ ﻋﲆ اﻟﺼﻔﺤﺔ اﻟﺘﺎﻟﻴﺔ‬

Page - 1308
Internal Medicine - Hepatology - Hepatitis

Question 27/66

Question #27

A 17-year-old male patient is suspected of having acute hepatitis A infection. What is the most
important test to be done for diagnosis?

a. Anti-HAV IgG antibody


b. Serum bilirubin level
c. Anti-HAV IgM antibody √
d. AST, ALT, and Alkaline phosphatase
e. PT and INR

Description

Anti-HAV IgM antibody titer is the best way to diagnose hepatitis A infection

A high titer of IgG antibodies against HAV indicates a previous infection

AST, ALT, Alkaline phosphatase, PT, INR, and bilirubin level are all parts of LFT but are not specific to
hepatitis A infection

Page - 1309
Internal Medicine - Hepatology

Question 28/66

Question #28

A patient received an HBV vaccination 6 months ago. Which of the following is the best indicator of
acquired immunity?

a. HBsAg
b. HBeAg
c. Anti-HBc (IgG)
d. Anti-HBs
e. HBV DNA

‫اﻹﺟﺎﺑﺔ ﻋﲆ اﻟﺼﻔﺤﺔ اﻟﺘﺎﻟﻴﺔ‬

Page - 1310
Internal Medicine - Hepatology - Hepatitis

Question 28/66

Question #28

A patient received an HBV vaccination 6 months ago. Which of the following is the best indicator of
acquired immunity?

a. HBsAg
b. HBeAg
c. Anti-HBc (IgG)
d. Anti-HBs √
e. HBV DNA

Description

Anti-HBs antibody is the best indicator of immunity in patients who have received vaccinations or
been exposed to HBV infection previously

e following table shows dierent laboratory findings in HRV infection and immunity and their
results.

Page - 1311
Internal Medicine - Hepatology

Question 29/66

Question #29

A patient’s hepatitis serology results are positive for HBsAg, HBeAg, and Anti-HBc (IgG). You can tell
him that he has:

a. Chronic infective HBV


b. Acute infective HBV
c. Chronic inactive HBV
d. Recovered from HBV
e. Immune against HBV

‫اﻹﺟﺎﺑﺔ ﻋﲆ اﻟﺼﻔﺤﺔ اﻟﺘﺎﻟﻴﺔ‬

Page - 1312
Internal Medicine - Hepatology - Hepatitis

Question 29/66

Question #29

A patient’s hepatitis serology results are positive for HBsAg, HBeAg, and Anti-HBc (IgG). You can tell
him that he has:

a. Chronic infective HBV √


b. Acute infective HBV
c. Chronic inactive HBV
d. Recovered from HBV
e. Immune against HBV

Description

HBsAg means infection, IgM anti-HBc means acute, HBeAg means active or infective

e following table shows dierent laboratory findings in HRV infection and immunity and their
results.

Page - 1313
Internal Medicine - Hepatology

Question 30/66

Question #30

A 34-year-old nurse presents for routine antenatal care, she has received two doses of her hepatitis B
vaccine, but her pregnancy test was positive before taking the 3rd dose. Which of the following
indicates vaccine immunization in this lady?

a. Negative HBsAg, negative Anti-HBc, negative Anti-HBs


b. Negative HBsAg, negative Anti-HBc, positive Anti-HBs
c. Negative HBsAg, positive Anti-HBc, positive Anti-HBs
d. Positive HBsAg, positive Anti-HBc, negative Anti-HBs
e. Positive HBsAg, positive Anti-HBc, positive Anti-HBs

‫اﻹﺟﺎﺑﺔ ﻋﲆ اﻟﺼﻔﺤﺔ اﻟﺘﺎﻟﻴﺔ‬

Page - 1314
Internal Medicine - Hepatology - Hepatitis

Question 30/66

Question #30

A 34-year-old nurse presents for routine antenatal care, she has received two doses of her hepatitis B
vaccine, but her pregnancy test was positive before taking the 3rd dose. Which of the following
indicates vaccine immunization in this lady?

a. Negative HBsAg, negative Anti-HBc, negative Anti-HBs


b. Negative HBsAg, negative Anti-HBc, positive Anti-HBs √
c. Negative HBsAg, positive Anti-HBc, positive Anti-HBs
d. Positive HBsAg, positive Anti-HBc, negative Anti-HBs
e. Positive HBsAg, positive Anti-HBc, positive Anti-HBs

Description

e HBV vaccine contains the HBsAg only, so it will enhance the production of Anti-HBs by the
immune system but never trigger the production of anti-HBc or Anti-Hbe antibodies

e following table shows dierent laboratory findings in HRV infection and immunity and their
results.

Page - 1315
Internal Medicine - Hepatology

Question 31/66

Question #31

A male patient had a history of hepatitis A infection 5 years ago. Today on a routine visit, his physical
examination is normal, but his AST level is 93 U/L. You ordered further investigations that show the
following: positive Anti-HAV, negative Anti-HBs, Positive HBsAg, and Positive Anti-HBc IgM. Which
of the following is the best description for this patient?

a. He has both HAV and HBV infection


b. He is immune to HBV
c. He has a chronic hepatitis B infection
d. is patient has chronic hepatitis A infection
e. HBsAg appears aer infection and 1 – 6 weeks before the symptoms develop.

‫اﻹﺟﺎﺑﺔ ﻋﲆ اﻟﺼﻔﺤﺔ اﻟﺘﺎﻟﻴﺔ‬

Page - 1316
Internal Medicine - Hepatology - Hepatitis

Question 31/66

Question #31

A male patient had a history of hepatitis A infection 5 years ago. Today on a routine visit, his physical
examination is normal, but his AST level is 93 U/L. You ordered further investigations that show the
following: positive Anti-HAV, negative Anti-HBs, Positive HBsAg, and Positive Anti-HBc IgM. Which
of the following is the best description for this patient?

a. He has both HAV and HBV infection


b. He is immune to HBV
c. He has a chronic hepatitis B infection
d. is patient has chronic hepatitis A infection
e. HBsAg appears aer infection and 1 – 6 weeks before the symptoms develop. √

Description

e presence of Anti-HAV indicates a previous infection in this patient

Positive HBsAg and Anti-HBc IgM mean that there is an acute infection with HBV in this patient

Aer infection and 1 to 6 weeks before symptoms occur, HBsAg appears.

Page - 1317
Internal Medicine - Hepatology

Question 32/66

Question #32

A 43-year-old male patient has a chronic hepatitis infection. Which of the following indicates that
the infection is active?

a. Detectable levels of HBeAb


b. Detectable levels of Anti-HBe
c. High levels of AST and ALT
d. Undetectable levels of HBV DNA
e. Normal liver biopsy

‫اﻹﺟﺎﺑﺔ ﻋﲆ اﻟﺼﻔﺤﺔ اﻟﺘﺎﻟﻴﺔ‬

Page - 1318
Internal Medicine - Hepatology - Hepatitis

Question 32/66

Question #32

A 43-year-old male patient has a chronic hepatitis infection. Which of the following indicates that
the infection is active?

a. Detectable levels of HBeAb


b. Detectable levels of Anti-HBe
c. High levels of AST and ALT √
d. Undetectable levels of HBV DNA
e. Normal liver biopsy

Description

HBeAg, not HBeAb, is an indication of active HBV infection

Note that the signs of active infection in the case of HBV are evidenced by elevated liver enzymes,
indicating liver inflammation, and the presence of HBeAg, indicating high levels of HBV DNA

Page - 1319
Internal Medicine - Hepatology

Question 33/66

Question #33

A 30-year-old male patient has positive Anti-HCV. Which of the following would be most beneficial
to determine the stage of this disease?

a. Hepatic transaminases
b. PT and INR
c. Liver biopsy
d. Liver CT scan
e. Serum bilirubin level

‫اﻹﺟﺎﺑﺔ ﻋﲆ اﻟﺼﻔﺤﺔ اﻟﺘﺎﻟﻴﺔ‬

Page - 1320
Internal Medicine - Hepatology - Hepatitis

Question 33/66

Question #33

A 30-year-old male patient has positive Anti-HCV. Which of the following would be most beneficial
to determine the stage of this disease?

a. Hepatic transaminases
b. PT and INR
c. Liver biopsy √
d. Liver CT scan
e. Serum bilirubin level

Description

A liver biopsy is the best way to determine the stage of HCV disease.

Hepatitis C infection:

HCV is the most common hepatitis transmitted by a blood transfusion


Acute HCV is asymptomatic, and patients usually present with signs and symptoms of chronic
liver disease
ALT and AST are normal in 40% of HCV infection cases
Order Anti-HCV to diagnose its presence
Order HCV RNA (if positive, indicates active disease)
HCV genotyping should be performed at the time of diagnosis to help to choose the treatment
regimen

Page - 1321
Internal Medicine - Hepatology

Question 34/66

Question #34

A patient develops a yellowish discoloration of her sclera and mucous membranes. Which of the
following is not accurate about this condition?

a. Carotenemia is one of the dierentials


b. Hemolytic anemia presents with conjugated hyperbilirubinemia
c. Glucuronyltransferase enzyme inactivity is one of the possible causes
d. Conjugated bilirubin is water-soluble
e. Hepatitis serology tests can help in detecting the cause

‫اﻹﺟﺎﺑﺔ ﻋﲆ اﻟﺼﻔﺤﺔ اﻟﺘﺎﻟﻴﺔ‬

Page - 1322
Internal Medicine - Hepatology - Jaundice

Question 34/66

Question #34

A patient develops a yellowish discoloration of her sclera and mucous membranes. Which of the
following is not accurate about this condition?

a. Carotenemia is one of the dierentials


b. Hemolytic anemia presents with conjugated hyperbilirubinemia √
c. Glucuronyltransferase enzyme inactivity is one of the possible causes
d. Conjugated bilirubin is water-soluble
e. Hepatitis serology tests can help in detecting the cause

Description

Hemolytic anemia manifests as unconjugated (indirect) hyperbilirubinemia.

Increased beta-carotene levels in the blood (carotenemia) can cause a yellowish discoloration of the
skin and mucous membranes (mimicking jaundice)

Glucuronyltransferase enzyme in the liver is responsible for the conjugation of bilirubin (converts it
to water-soluble conjugated form)

Page - 1323
Internal Medicine - Hepatology

Question 35/66

Question #35

A 35-year-old male patient has been found to have intermittent jaundice for several years. His lab
investigations always show isolated mild, indirect hyperbilirubinemia and normal CBC. His physical
examination is normal except for mild jaundice. Which of the following is the most likely diagnosis?

a. Chronic hepatitis C infection


b. Hemolytic anemia
c. Gilbert syndrome
d. Obstructive jaundice
e. Wilson’s disease

‫اﻹﺟﺎﺑﺔ ﻋﲆ اﻟﺼﻔﺤﺔ اﻟﺘﺎﻟﻴﺔ‬

Page - 1324
Internal Medicine - Hepatology - Jaundice

Question 35/66

Question #35

A 35-year-old male patient has been found to have intermittent jaundice for several years. His lab
investigations always show isolated mild, indirect hyperbilirubinemia and normal CBC. His physical
examination is normal except for mild jaundice. Which of the following is the most likely diagnosis?

a. Chronic hepatitis C infection


b. Hemolytic anemia
c. Gilbert syndrome √
d. Obstructive jaundice
e. Wilson’s disease

Description

Isolated hyperbilirubinemia in an otherwise healthy young man is Gilbert’s syndrome until proven
otherwise.

Gilbert syndrome:

An autosomal dominant familial condition


Decreased UDP-glucuronosyltransferase enzyme
Mild jaundice (< 3 mg/dl unconjugated hyperbilirubinemia), more prominent if dehydration or
stress occurs
Normal AST and ALT and absence of hemolysis along with indirect hyperbilirubinemia is
enough for diagnosis (without extensive testing)
Patient otherwise asymptomatic
No need for treatment

Page - 1325
Internal Medicine - Hepatology

Question 36/66

Question #36

A 26-year-old male recently diagnosed with multiple sclerosis has a routine liver function test before
starting on the disease-modifying drug “Fingolimod”. His LFT shows an isolated mild elevation of
the indirect bilirubin and normal AST, ALT, and ALP levels. Physical examination is unremarkable,
and the rest of the lab investigations are normal. What is the most likely cause of this patient’s
hyperbilirubinemia?

a. Hereditary spherocytosis
b. Non-alcoholic fatty liver
c. Hepatitis A infection
d. Obstructive liver disease
e. Gilbert syndrome

‫اﻹﺟﺎﺑﺔ ﻋﲆ اﻟﺼﻔﺤﺔ اﻟﺘﺎﻟﻴﺔ‬

Page - 1326
Internal Medicine - Hepatology - Jaundice

Question 36/66

Question #36

A 26-year-old male recently diagnosed with multiple sclerosis has a routine liver function test before
starting on the disease-modifying drug “Fingolimod”. His LFT shows an isolated mild elevation of
the indirect bilirubin and normal AST, ALT, and ALP levels. Physical examination is unremarkable,
and the rest of the lab investigations are normal. What is the most likely cause of this patient’s
hyperbilirubinemia?

a. Hereditary spherocytosis
b. Non-alcoholic fatty liver
c. Hepatitis A infection
d. Obstructive liver disease
e. Gilbert syndrome √

Description

Isolated hyperbilirubinemia in an otherwise healthy young man is Gilbert’s syndrome until proven
otherwise.

Gilbert syndrome:

An autosomal dominant familial condition


Decreased UDP-glucuronosyltransferase enzyme
Mild jaundice (< 3 mg/dl unconjugated hyperbilirubinemia), more prominent if dehydration or
stress occurs
Normal AST and ALT and absence of hemolysis along with indirect hyperbilirubinemia is
enough for diagnosis (without extensive testing)
Patient otherwise asymptomatic
No need for treatment

Page - 1327
Internal Medicine - Hepatology

Question 37/66

Question #37

During a routine examination, an alcoholic 35-year-old male is found to have a total bilirubin level of
2.7mg/dL and direct bilirubin of 0.5mg/dL. His physical examination, CBC, AST, ALT, and ALP are all
within normal limits. Which of the following is the most likely diagnosis?

a. Alcoholic liver disease


b. Non-alcoholic fatty liver
c. Hepatitis A infection
d. Obstructive liver disease
e. Gilbert syndrome

‫اﻹﺟﺎﺑﺔ ﻋﲆ اﻟﺼﻔﺤﺔ اﻟﺘﺎﻟﻴﺔ‬

Page - 1328
Internal Medicine - Hepatology - Jaundice

Question 37/66

Question #37

During a routine examination, an alcoholic 35-year-old male is found to have a total bilirubin level of
2.7mg/dL and direct bilirubin of 0.5mg/dL. His physical examination, CBC, AST, ALT, and ALP are all
within normal limits. Which of the following is the most likely diagnosis?

a. Alcoholic liver disease


b. Non-alcoholic fatty liver
c. Hepatitis A infection
d. Obstructive liver disease
e. Gilbert syndrome √

Description

Isolated hyperbilirubinemia in an otherwise healthy young man is Gilbert’s syndrome until proven
otherwise.

Gilbert syndrome:

An autosomal dominant familial condition


Decreased UDP-glucuronosyltransferase enzyme
Mild jaundice (< 3 mg/dl unconjugated hyperbilirubinemia), more prominent if dehydration or
stress occurs
Normal AST and ALT and absence of hemolysis along with indirect hyperbilirubinemia is
enough for diagnosis (without extensive testing)
Patient otherwise asymptomatic
No need for treatment

Page - 1329
Internal Medicine - Hepatology

Question 38/66

Question #38

A 21-year-old male patient complains of flu-like symptoms and dry coughs. Physical examination is
normal except for mild scleral yellowish discoloration. His total bilirubin level is 2mg/dL, and the
direct potion is 0.4 mg/dL. His ALT, AST, ALP, and urine dipstick are normal. What is the most likely
diagnosis?

a. Acute viral hepatitis


b. Dubin Johnson Syndrome
c. G6PD deficiency
d. Gilbert syndrome
e. Obstructive jaundice

‫اﻹﺟﺎﺑﺔ ﻋﲆ اﻟﺼﻔﺤﺔ اﻟﺘﺎﻟﻴﺔ‬

Page - 1330
Internal Medicine - Hepatology - Jaundice

Question 38/66

Question #38

A 21-year-old male patient complains of flu-like symptoms and dry coughs. Physical examination is
normal except for mild scleral yellowish discoloration. His total bilirubin level is 2mg/dL, and the
direct potion is 0.4 mg/dL. His ALT, AST, ALP, and urine dipstick are normal. What is the most likely
diagnosis?

a. Acute viral hepatitis


b. Dubin Johnson Syndrome
c. G6PD deficiency
d. Gilbert syndrome √
e. Obstructive jaundice

Description

It is not uncommon to diagnose Gilbert syndrome for the first time during an acute illness.

In this disease, jaundice is exacerbated by acute infections of dehydration.

e presence of isolated indirect hyperbilirubinemia here is the clue.

Isolated indirect hyperbilirubinemia means high indirect bilirubin in otherwise normal liver function
tests.

Gilbert syndrome:

An autosomal dominant familial condition


Decreased UDP-glucuronosyltransferase enzyme
Mild jaundice (< 3 mg/dl unconjugated hyperbilirubinemia), more prominent if dehydration or
stress occurs
Normal AST and ALT and absence of hemolysis along with indirect hyperbilirubinemia is
enough for diagnosis (without extensive testing)
Patient otherwise asymptomatic
No need for treatment

Page - 1331
Internal Medicine - Hepatology

Question 39/66

Question #39

A 43-year-old male patient presents with gradual onset lethargy and weakness, and you diagnose
him with liver cirrhosis. All the following are possible causes for his condition except:

a. Hepatitis A
b. Hepatitis C
c. Hepatitis B
d. Alcohol abuse
e. Alpha 1 antitrypsin deficiency

‫اﻹﺟﺎﺑﺔ ﻋﲆ اﻟﺼﻔﺤﺔ اﻟﺘﺎﻟﻴﺔ‬

Page - 1332
Internal Medicine - Hepatology - Liver Cirrhosis

Question 39/66

Question #39

A 43-year-old male patient presents with gradual onset lethargy and weakness, and you diagnose
him with liver cirrhosis. All the following are possible causes for his condition except:

a. Hepatitis A √
b. Hepatitis C
c. Hepatitis B
d. Alcohol abuse
e. Alpha 1 antitrypsin deficiency

Description

Liver cirrhosis is a chronic liver disease that is usually occurring due to chronic liver disease

Note that Hepatitis A infection is always acute and never be chronic

e causes of liver cirrhosis are the following:

Chronic hepatitis (B, C, B+D) BUT NOT hepatitis A and E


Alcoholic and Non-alcoholic fatty liver disease
Autoimmune hepatitis
Alpha-1-antitrypsin deficiency (10% cause liver disease)
Drug-induced liver cirrhosis.
Right-side Heart failure (chronic liver congestion and cardiac cirrhosis)

Page - 1333
Internal Medicine - Hepatology

Question 40/66

Question #40

One of the following diseases is not a cause of liver cirrhosis:

a. Hepatitis A
b. Hepatitis B
c. Hepatitis C
d. Wilson disease
e. Hemochromatosis

‫اﻹﺟﺎﺑﺔ ﻋﲆ اﻟﺼﻔﺤﺔ اﻟﺘﺎﻟﻴﺔ‬

Page - 1334
Internal Medicine - Hepatology - Liver Cirrhosis

Question 40/66

Question #40

One of the following diseases is not a cause of liver cirrhosis:

a. Hepatitis A √
b. Hepatitis B
c. Hepatitis C
d. Wilson disease
e. Hemochromatosis

Description

HAV is the most common hepatotropic virus. It can only cause acute hepatitis (no chronicity and no
risk of liver cirrhosis).

Hepatitis B and C cause chronic hepatitis, liver cirrhosis, and hepatocellular carcinoma.

Wilson disease is the accumulation of copper, while hemochromatosis is the iron accumulation in
the liver and other organs, increasing the risk of liver cirrhosis.

Page - 1335
Internal Medicine - Hepatology

Question 41/66

Question #41

A 43-year-old male patient presents with gradual onset lethargy and weakness, and you diagnose
him with liver cirrhosis. All the following are poor prognostic factors for this patient except:

a. e presence of ascites
b. High bilirubin level
c. High albumin level
d. Prolonged PT and INR
e. e presence of encephalopathy

‫اﻹﺟﺎﺑﺔ ﻋﲆ اﻟﺼﻔﺤﺔ اﻟﺘﺎﻟﻴﺔ‬

Page - 1336
Internal Medicine - Hepatology - Liver Cirrhosis

Question 41/66

Question #41

A 43-year-old male patient presents with gradual onset lethargy and weakness, and you diagnose
him with liver cirrhosis. All the following are poor prognostic factors for this patient except:

a. e presence of ascites
b. High bilirubin level
c. High albumin level √
d. Prolonged PT and INR
e. e presence of encephalopathy

Description

e poor prognostic factors of liver cirrhosis are summarized in the child-pugh score to interpret the
mortality rate in these patients.

Page - 1337
Internal Medicine - Hepatology

Question 42/66

Question #42

A 35-year-old male patient has elevated AST and ALT levels for the second time during the last 8
months. e patient is asymptomatic and has a normal physical examination. Which of the
following is the next step in managing this patient?

a. Liver biopsy
b. Liver CT scan
c. Abdominal U/S with doppler
d. Positive emission tomography (PET) scan
e. Abdominal MRI

‫اﻹﺟﺎﺑﺔ ﻋﲆ اﻟﺼﻔﺤﺔ اﻟﺘﺎﻟﻴﺔ‬

Page - 1338
Internal Medicine - Hepatology - Liver function tests (LFT)

Question 42/66

Question #42

A 35-year-old male patient has elevated AST and ALT levels for the second time during the last 8
months. e patient is asymptomatic and has a normal physical examination. Which of the
following is the next step in managing this patient?

a. Liver biopsy
b. Liver CT scan
c. Abdominal U/S with doppler √
d. Positive emission tomography (PET) scan
e. Abdominal MRI

Description

Ultrasound is the next step because of the following reasons:

Easy to be reached (widely available)


No radiation exposure
No need for contrast media
Inexpensive
Provides helpful information regarding liver morphology and portal and hepatic flow

CT scan is beneficial to assess the hepatic nodules (if revealed in ultrasound), whether benign or
malignant

Liver biopsy is not the initial study for this case but can be used to know the type of tissue in any
mass (benign or malignant)

PET scans can detect the presence of liver metastases from certain cancers but are not used as an
initial test for evaluating liver disease.

MRI can assess the changes in the appearance or size of any liver mass, not initially in this case

Page - 1339
Internal Medicine - Hepatology

Question 43/66

Question #43

About the liver function test, all the following are true except:

a. Prothrombin time reflects the ability of the liver to synthesize protein


b. Serum albumin is a specific test to determine the liver function
c. AST is more elevated than ALT in alcoholic liver diseases
d. Alkaline phosphatase will be markedly elevated in obstructive liver disease
e. In acute viral hepatitis, AST and ALT may be as high as 1000 u/l

‫اﻹﺟﺎﺑﺔ ﻋﲆ اﻟﺼﻔﺤﺔ اﻟﺘﺎﻟﻴﺔ‬

Page - 1340
Internal Medicine - Hepatology - Liver function tests (LFT)

Question 43/66

Question #43

About the liver function test, all the following are true except:

a. Prothrombin time reflects the ability of the liver to synthesize protein


b. Serum albumin is a specific test to determine the liver function √
c. AST is more elevated than ALT in alcoholic liver diseases
d. Alkaline phosphatase will be markedly elevated in obstructive liver disease
e. In acute viral hepatitis, AST and ALT may be as high as 1000 u/l

Description

Serum albumin is not specific to liver diseases. However, it can be reduced in the case of
malnutrition, nephrotic syndrome, protein-losing enteropathy, sepsis, and other conditions not
related to the liver.

Page - 1341
Internal Medicine - Hepatology

Question 44/66

Question #44

A female patient presents with jaundice, dark urine, bilirubinuria, and pale stool. Her biliary
ultrasound demonstrates a dilated common bile duct. Which of the following would support the
most likely diagnosis?

a. High Reticulocyte count


b. High AST and ALT levels
c. Low Serum albumin level
d. High Alkaline phosphatase level
e. Low Hemoglobin level

‫اﻹﺟﺎﺑﺔ ﻋﲆ اﻟﺼﻔﺤﺔ اﻟﺘﺎﻟﻴﺔ‬

Page - 1342
Internal Medicine - Hepatology - Liver function tests (LFT)

Question 44/66

Question #44

A female patient presents with jaundice, dark urine, bilirubinuria, and pale stool. Her biliary
ultrasound demonstrates a dilated common bile duct. Which of the following would support the
most likely diagnosis?

a. High Reticulocyte count


b. High AST and ALT levels
c. Low Serum albumin level
d. High Alkaline phosphatase level √
e. Low Hemoglobin level

Description

Jaundice, dark urine, and pale stool indicate obstructive (post-hepatic) jaundice.

Alkaline phosphatase (ALP) is an indicator of obstructive liver disease if very high with less
prominent elevated AST and ALT (Confirm obstructive liver disease by elevated GGT)

Additional notes:

If isolated high ALP is present, consider a bone disease


ALP can be produced by the placenta and is elevated in the case of pregnancy.

Page - 1343
Internal Medicine - Hepatology

Question 45/66

Question #45

A 49-year-old alcoholic male patient with DM and HTN presents for a routine examination. His BMI
is 37 kg/m2. His blood pressure is 140/89. e pulse is 89. His lab tests show an A1C level of 9.5%, AST
of 94 U/mL, and ALT of 150 u/mL. What is the most likely diagnosis?

a. Alcoholic fatty liver disease


b. Non-alcoholic fatty liver disease
c. Acute Viral hepatitis
d. Normal liver function test
e. Autoimmune hepatitis

‫اﻹﺟﺎﺑﺔ ﻋﲆ اﻟﺼﻔﺤﺔ اﻟﺘﺎﻟﻴﺔ‬

Page - 1344
Internal Medicine - Hepatology - Non-alcoholic Fatty Liver Disease (NAFLD)

Question 45/66

Question #45

A 49-year-old alcoholic male patient with DM and HTN presents for a routine examination. His BMI
is 37 kg/m2. His blood pressure is 140/89. e pulse is 89. His lab tests show an A1C level of 9.5%, AST
of 94 U/mL, and ALT of 150 u/mL. What is the most likely diagnosis?

a. Alcoholic fatty liver disease


b. Non-alcoholic fatty liver disease √
c. Acute Viral hepatitis
d. Normal liver function test
e. Autoimmune hepatitis

Description

Minimal ALT and AST elevation in patients with risk factors for fatty liver (DM, dyslipidemia, high
BMI) indicate non-alcoholic fatty liver disease.

Alcoholic fatty liver diseases present with high ALT and AST with an AST/ALT ratio of > 2:1

Page - 1345
Internal Medicine - Hepatology

Question 46/66

Question #46

A previously healthy male patient presents for his AST and ALT follow-up. On examination, he is
afebrile with normal vital signs, and his body mass index is 42 Kg/m2. His AST and ALT today are 75
and 103 U/L, respectively. Which of the following is the most likely diagnosis?

a. Viral hepatitis
b. Alcoholic fatty liver
c. Non-alcoholic fatty liver
d. Liver cirrhosis
e. Hepatocellular carcinoma

‫اﻹﺟﺎﺑﺔ ﻋﲆ اﻟﺼﻔﺤﺔ اﻟﺘﺎﻟﻴﺔ‬

Page - 1346
Internal Medicine - Hepatology - Non-alcoholic Fatty Liver Disease (NAFLD)

Question 46/66

Question #46

A previously healthy male patient presents for his AST and ALT follow-up. On examination, he is
afebrile with normal vital signs, and his body mass index is 42 Kg/m2. His AST and ALT today are 75
and 103 U/L, respectively. Which of the following is the most likely diagnosis?

a. Viral hepatitis
b. Alcoholic fatty liver
c. Non-alcoholic fatty liver √
d. Liver cirrhosis
e. Hepatocellular carcinoma

Description

Elevated ALT, more than AST, minimally indicates non-alcoholic fatty liver disease (NAFLD) as the
most likely diagnosis

Alcoholic fatty liver is unlikely because the AST is not more than double the ALT

Viral hepatitis is still a possibility here and should be ruled out, but NAFLD is the most likely

Page - 1347
Internal Medicine - Hepatology

Question 47/66

Question #47

A 55-year-old male patient has chronic back pain, usually relieved by paracetamol oral tablets.
However, he is concerned that he may tack too much and asked you for his maximum daily
recommended dose of paracetamol. What would be your appropriate answer?

a. 7 grams
b. 6 grams
c. 4 grams
d. 3 grams
e. 2 grams

‫اﻹﺟﺎﺑﺔ ﻋﲆ اﻟﺼﻔﺤﺔ اﻟﺘﺎﻟﻴﺔ‬

Page - 1348
Internal Medicine - Hepatology - Paracetamol poisoning

Question 47/66

Question #47

A 55-year-old male patient has chronic back pain, usually relieved by paracetamol oral tablets.
However, he is concerned that he may tack too much and asked you for his maximum daily
recommended dose of paracetamol. What would be your appropriate answer?

a. 7 grams
b. 6 grams
c. 4 grams √
d. 3 grams
e. 2 grams

Description

Four grams of paracetamol is the adult’s maximum recommended dose of paracetamol. Note that
amount above 7 grams or 140mg/Kg is considered hepatotoxic

Page - 1349
Internal Medicine - Hepatology

Question 48/66

Question #48

Which of the following is considered the toxic dose for paracetamol for a 78 kilograms previously
healthy adult?

a. 7 grams
b. 6 grams
c. 4 grams
d. 3 grams
e. 2 grams

‫اﻹﺟﺎﺑﺔ ﻋﲆ اﻟﺼﻔﺤﺔ اﻟﺘﺎﻟﻴﺔ‬

Page - 1350
Internal Medicine - Hepatology - Paracetamol poisoning

Question 48/66

Question #48

Which of the following is considered the toxic dose for paracetamol for a 78 kilograms previously
healthy adult?

a. 7 grams √
b. 6 grams
c. 4 grams
d. 3 grams
e. 2 grams

Description

A dose of more than 7 grams or 140mg/Kg is considered hepatotoxic

If the patient is alcoholic or has liver disease, the toxic dose may be lower than that

e maximum allowed dose to be taken daily for the treatment of pain is 4 grams

Page - 1351
Internal Medicine - Hepatology

Question 49/66

Question #49

A 50-year-old female with a biopsy documented liver cirrhosis presents with vomiting of bright red
blood. Her blood pressure is 80/30 mmHg, the pulse is 130 bpm, and gastric aspiration contains
blood. Which of the following is the most appropriate at this time?

a. Octreotide and noradrenaline


b. Terlipressin and Intravenous saline
c. Endoscopic sclerotherapy
d. Administration of omeprazole and propranolol
e. Test for H pylori infection

‫اﻹﺟﺎﺑﺔ ﻋﲆ اﻟﺼﻔﺤﺔ اﻟﺘﺎﻟﻴﺔ‬

Page - 1352
Internal Medicine - Hepatology - Portal HTN and esophageal varices

Question 49/66

Question #49

A 50-year-old female with a biopsy documented liver cirrhosis presents with vomiting of bright red
blood. Her blood pressure is 80/30 mmHg, the pulse is 130 bpm, and gastric aspiration contains
blood. Which of the following is the most appropriate at this time?

a. Octreotide and noradrenaline


b. Terlipressin and Intravenous saline √
c. Endoscopic sclerotherapy
d. Administration of omeprazole and propranolol
e. Test for H pylori infection

Description

e priority in actively bleeding patients is to control the bleeding and stabilize the patient as soon
as possible.

e first line here is to introduce 2 large-bore cannulae and start intravenous fluid.

e source of bleeding is most likely due to esophageal varices secondary to liver cirrhosis.

In this case, you should use Terlipressin to reduce portal pressure and control the bleeding.

Crossmatch and blood transfusion

Endoscopy is both diagnostic and therapeutic in this case.

Endoscopic sclerotherapy is not done in actively bleeding patients.

Page - 1353
Internal Medicine - Hepatology

Question 50/66

Question #50

A 45-year-old alcoholic man comes into the emergency room with upper gastrointestinal bleeding.
Urgent endoscopy shows dilated esophageal veins. Which of the following is the best treatment
option?

a. Start Propranolol
b. Nissen fundoplication
c. Endoscopic ligation
d. Vancomycin and lactulose
e. Trans-jugular intrahepatic portosystemic shunt (TIPSS)

‫اﻹﺟﺎﺑﺔ ﻋﲆ اﻟﺼﻔﺤﺔ اﻟﺘﺎﻟﻴﺔ‬

Page - 1354
Internal Medicine - Hepatology - Portal HTN and esophageal varices

Question 50/66

Question #50

A 45-year-old alcoholic man comes into the emergency room with upper gastrointestinal bleeding.
Urgent endoscopy shows dilated esophageal veins. Which of the following is the best treatment
option?

a. Start Propranolol
b. Nissen fundoplication
c. Endoscopic ligation √
d. Vancomycin and lactulose
e. Trans-jugular intrahepatic portosystemic shunt (TIPSS)

Description

e patient has esophageal varices confirmed by upper endoscopy.

e treatment of choice is to perform endoscopic ligation.

Propranolol is not used in acute variceal bleeding but to control portal HTN in non-bleeding patients.

Nissen fundoplication is used in the treatment of GERD to strengthen the lower esophageal
sphincter

Vancomycin and lactulose are used in the treatment of hepatic encephalopathy.

TIPSS is reserved for recurrent esophageal bleeding with end-stage liver disease.

Page - 1355
Internal Medicine - Hepatology

Question 51/66

Question #51

A 45-year-old alcoholic man comes into the emergency room with upper gastrointestinal bleeding.
Urgent endoscopy is shown in the picture below. Which of the following is the most likely diagnosis?

a. Esophageal varices
b. Esophageal carcinoma
c. A foreign body in the esophagus
d. Esophageal stricture
e. Barrett’s esophagus

‫اﻹﺟﺎﺑﺔ ﻋﲆ اﻟﺼﻔﺤﺔ اﻟﺘﺎﻟﻴﺔ‬

Page - 1356
Internal Medicine - Hepatology - Portal HTN and esophageal varices

Question 51/66

Question #51

A 45-year-old alcoholic man comes into the emergency room with upper gastrointestinal bleeding.
Urgent endoscopy is shown in the picture below. Which of the following is the most likely diagnosis?

a. Esophageal varices √
b. Esophageal carcinoma
c. A foreign body in the esophagus
d. Esophageal stricture
e. Barrett’s esophagus

Description

e patient has esophageal varices confirmed by upper endoscopy.

Alcohol intake is the risk factor in this case.

Page - 1357
Internal Medicine - Hepatology

Question 52/66

Question #52

A 62-year-old female with liver cirrhosis presents with bright red vomits and black stools. Her vital
signs show tachycardia and orthostatic hypotension. Which of the following is the best diagnostic
and therapeutic procedure in this clinical scenario?

a. Abdominal CT scan
b. Upper gastrointestinal series
c. Standing chest x-ray
d. Esophagogastroduodenoscopy
e. Trans-Jugular intrahepatic Portosystemic Shunt (TIPSS)

‫اﻹﺟﺎﺑﺔ ﻋﲆ اﻟﺼﻔﺤﺔ اﻟﺘﺎﻟﻴﺔ‬

Page - 1358
Internal Medicine - Hepatology - Portal HTN and esophageal varices

Question 52/66

Question #52

A 62-year-old female with liver cirrhosis presents with bright red vomits and black stools. Her vital
signs show tachycardia and orthostatic hypotension. Which of the following is the best diagnostic
and therapeutic procedure in this clinical scenario?

a. Abdominal CT scan
b. Upper gastrointestinal series
c. Standing chest x-ray
d. Esophagogastroduodenoscopy √
e. Trans-Jugular intrahepatic Portosystemic Shunt (TIPSS)

Description

is clinical scenario strongly suggests esophageal variceal bleeding.

is condition is characterized by dilated veins in the distal esophagus or proximal stomach.

e history of liver cirrhosis and upper GI bleeding is suggestive.

Upper gastrointestinal endoscopy (also known as Esophagogastroduodenoscopy) is the best


diagnostic and therapeutic procedure to diagnose this condition

Endoscopic banding is the primary treatment for this condition

Octreotide or Terlipressin may be used to reduce the portal pressure in acutely bleeding patients

Intravenous fluid and blood transfusion should be started to restore circulation and treat shock.

TIPSS should be reserved for recurrent Upper GI bleedings secondary to esophageal varices in a
patient with end-stage liver disease; it can exacerbate hepatic encephalopathy due to shunting
blood from the portal to the systemic circulation.

Page - 1359
Internal Medicine - Hepatology

Question 53/66

Question #53

A 47-year-old alcoholic male patient had bloody vomiting this morning. On examination, his blood
pressure is 99/70 mmHg, his pulse is 109 bpm, his abdomen is positive for shiing dullness, and he
has spider nevi on his chest. What is the most likely diagnosis?

a. Peptic ulcer disease


b. Stomach cancer
c. Esophageal cancer
d. Esophageal varices
e. Gastritis

‫اﻹﺟﺎﺑﺔ ﻋﲆ اﻟﺼﻔﺤﺔ اﻟﺘﺎﻟﻴﺔ‬

Page - 1360
Internal Medicine - Hepatology - Portal HTN and esophageal varices

Question 53/66

Question #53

A 47-year-old alcoholic male patient had bloody vomiting this morning. On examination, his blood
pressure is 99/70 mmHg, his pulse is 109 bpm, his abdomen is positive for shiing dullness, and he
has spider nevi on his chest. What is the most likely diagnosis?

a. Peptic ulcer disease


b. Stomach cancer
c. Esophageal cancer
d. Esophageal varices √
e. Gastritis

Description

e presence of chronic liver disease and hematemesis strongly suggests esophageal varices.

e patient has features suggestive of liver cirrhosis (alcoholism, spider nevi, and ascites).

e presence of liver cirrhosis will result in portal HTN.

When portal pressure is high, → collaterals will appear to shunt blood to the systemic circulation →
esophageal Variceal, Caput medusa, rectal varices

Page - 1361
Internal Medicine - Hepatology

Question 54/66

Question #54

A 55-year-old male heavy drinker presents to you with hematemesis and orthostatic hypotension.
Which of the following is the most eective treatment, at the point of endoscopy, for esophageal
varices?

a. Endoscopic sclerotherapy
b. Tranexamic acid
c. Endoscopic banding
d. Adrenalin injection
e. Proton pump inhibitor

‫اﻹﺟﺎﺑﺔ ﻋﲆ اﻟﺼﻔﺤﺔ اﻟﺘﺎﻟﻴﺔ‬

Page - 1362
Internal Medicine - Hepatology - Portal HTN and esophageal varices

Question 54/66

Question #54

A 55-year-old male heavy drinker presents to you with hematemesis and orthostatic hypotension.
Which of the following is the most eective treatment, at the point of endoscopy, for esophageal
varices?

a. Endoscopic sclerotherapy
b. Tranexamic acid
c. Endoscopic banding √
d. Adrenalin injection
e. Proton pump inhibitor

Description

Endoscopic banding is the most eective treatment to stop bleeding varices.

Treatment of Upper GI bleeding due to esophageal varices:

e priority is to restore circulation with IV fluids, Blood, and plasma


e hemoglobin transfusion goal is 7 g/dl (increasing Hb more than 7 will lead to increased
portal pressure and thus more bleeding)
All should receive prophylactic antibiotics, “ciprofloxacin,” to prevent hepatic encephalopathy
and SBP.
Terlipressin or Octreotide:
Vasopressin analog reduces the portal pressure
Terlipressin reduces mortality in Variceal bleeding
Endoscopy:
Diagnostic and therapeutic
Banding can be used to stop bleeding

Balloon tamponade:

Sengstaken-Blakemore tube can be inserted and inflated to stop bleeding. It should be


deflated 10 minutes every 3 hours to avoid esophageal necrosis
Transjugular intrahepatic portosystemic shunt (TIPSS):
A stent inserted between the portal and hepatic vein
Page - 1363
It increases the risk of hepatic encephalopathy

Page - 1364
Internal Medicine - Hepatology

Question 55/66

Question #55

A 42-year-old female has had jaundice and pruritus for a long history. Her history is significant for
Sjögren syndrome, and her lab tests show elevated alkaline phosphatase. What is the most
appropriate test to perform at this time?

a. Rheumatoid factor (RA)


b. Anti-nuclear antibody (ANA)
c. Anti-smooth muscle antibodies (ASMA)
d. Anti-mitochondrial antibodies (AMA)
e. Anti-neutrophil cytoplasmic antibodies (ANCA)

‫اﻹﺟﺎﺑﺔ ﻋﲆ اﻟﺼﻔﺤﺔ اﻟﺘﺎﻟﻴﺔ‬

Page - 1365
Internal Medicine - Hepatology - Primary Biliary Cholangitis (PBC)

Question 55/66

Question #55

A 42-year-old female has had jaundice and pruritus for a long history. Her history is significant for
Sjögren syndrome, and her lab tests show elevated alkaline phosphatase. What is the most
appropriate test to perform at this time?

a. Rheumatoid factor (RA)


b. Anti-nuclear antibody (ANA)
c. Anti-smooth muscle antibodies (ASMA)
d. Anti-mitochondrial antibodies (AMA) √
e. Anti-neutrophil cytoplasmic antibodies (ANCA)

Description

is is a direct question pointing to Primary Biliary Cholangitis (PBC).

Anti-mitochondrial antibodies should be done to support the diagnosis further.

e classic presentation contains the following:

Middle-aged woman
Oen asymptomatic, but the itching and jaundice may present
High alkaline phosphatase level
Positive anti-mitochondrial antibodies
Association with Sjögren syndrome is common

e following table shows the dierence between PBC and PSC:

Page - 1366
Internal Medicine - Hepatology

Question 56/66

Question #56

A 38-year-old lady is known to have Sjögren syndrome. She develops jaundice, high alkaline
phosphatase, and positive anti-mitochondrial antibodies. What is the most likely diagnosis?

a. Common bile duct stone


b. Hepatitis B infection
c. Hepatitis C infection
d. Primary biliary cholangitis
e. Primary sclerosing cholangitis

‫اﻹﺟﺎﺑﺔ ﻋﲆ اﻟﺼﻔﺤﺔ اﻟﺘﺎﻟﻴﺔ‬

Page - 1367
Internal Medicine - Hepatology - Primary Biliary Cholangitis (PBC)

Question 56/66

Question #56

A 38-year-old lady is known to have Sjögren syndrome. She develops jaundice, high alkaline
phosphatase, and positive anti-mitochondrial antibodies. What is the most likely diagnosis?

a. Common bile duct stone


b. Hepatitis B infection
c. Hepatitis C infection
d. Primary biliary cholangitis √
e. Primary sclerosing cholangitis

Description

is is a direct question pointing to Primary Biliary Cholangitis (PBC).

e classic presentation contains the following:

Middle-aged woman
Oen asymptomatic, but the itching and jaundice may present
High alkaline phosphatase level
Positive anti-mitochondrial antibodies
Association with Sjögren syndrome is common

e following table shows the dierence between PBC and PSC:

Page - 1368
Internal Medicine - Hepatology

Question 57/66

Question #57

A 46-year-old female complains of abdominal pain, itching, and yellowish discoloration of the skin
and sclera. Her history includes ulcerative Colitis. On examination, she is jaundiced, and abdominal
tenderness on deep palpation is noted at the right upper and lower quadrants, but her vital signs
and the rest of the physical examination are normal. Her alkaline phosphatase is 420 u/L, and her
bilirubin is 2.9 mg/dL. What is the most likely diagnosis?

a. Primary sclerosing cholangitis


b. Primary biliary cholangitis
c. Autoimmune hepatitis
d. Common bile duct stone
e. Liver cirrhosis

‫اﻹﺟﺎﺑﺔ ﻋﲆ اﻟﺼﻔﺤﺔ اﻟﺘﺎﻟﻴﺔ‬

Page - 1369
Internal Medicine - Hepatology - Primary Sclerosing Cholangitis (PSC)

Question 57/66

Question #57

A 46-year-old female complains of abdominal pain, itching, and yellowish discoloration of the skin
and sclera. Her history includes ulcerative Colitis. On examination, she is jaundiced, and abdominal
tenderness on deep palpation is noted at the right upper and lower quadrants, but her vital signs
and the rest of the physical examination are normal. Her alkaline phosphatase is 420 u/L, and her
bilirubin is 2.9 mg/dL. What is the most likely diagnosis?

a. Primary sclerosing cholangitis √


b. Primary biliary cholangitis
c. Autoimmune hepatitis
d. Common bile duct stone
e. Liver cirrhosis

Description

Ulcerative Colitis, elevated alkaline phosphatase, and jaundice are typical for primary sclerosing
cholangitis (PSC).

PSC is the most common chronic liver disease in patients with Ulcerative Colitis.

PSC is characterized by progressive cholestasis with inflammation and fibrosis of the intrahepatic
and extrahepatic bile ducts

Page - 1370
Internal Medicine - Hepatology

Question 58/66

Question #58

A 46-year-old female complains of abdominal pain, itching, and yellowish discoloration of the skin
and sclera. Her history includes ulcerative Colitis. On examination, she is jaundiced, and abdominal
tenderness on deep palpation is noted at the right upper and lower quadrants, but her vital signs
and the rest of the physical examination are normal. Her alkaline phosphatase is 420 u/L, and her
bilirubin is 2.9 mg/dL. What is the most appropriate investigation to confirm the diagnosis?

a. Abdominal ultrasonography
b. Abdominal CT scan
c. Magnetic resonance cholangiopancreatography
d. Endoscopic retrograde cholangiopancreatography
e. Upper endoscopy

‫اﻹﺟﺎﺑﺔ ﻋﲆ اﻟﺼﻔﺤﺔ اﻟﺘﺎﻟﻴﺔ‬

Page - 1371
Internal Medicine - Hepatology - Primary Sclerosing Cholangitis (PSC)

Question 58/66

Question #58

A 46-year-old female complains of abdominal pain, itching, and yellowish discoloration of the skin
and sclera. Her history includes ulcerative Colitis. On examination, she is jaundiced, and abdominal
tenderness on deep palpation is noted at the right upper and lower quadrants, but her vital signs
and the rest of the physical examination are normal. Her alkaline phosphatase is 420 u/L, and her
bilirubin is 2.9 mg/dL. What is the most appropriate investigation to confirm the diagnosis?

a. Abdominal ultrasonography
b. Abdominal CT scan
c. Magnetic resonance cholangiopancreatography √
d. Endoscopic retrograde cholangiopancreatography
e. Upper endoscopy

Description

e presence of Ulcerative Colitis, elevated alkaline phosphatase, and jaundice is a typical scenario
for primary sclerosing cholangitis (PSC).

PSC is the most common chronic liver disease in patients with Ulcerative Colitis.

PSC is characterized by progressive cholestasis with inflammation and fibrosis of the intrahepatic
and extrahepatic bile ducts

ERCP and MRCP are both eective in the diagnosis of PSC, but ERCP has a higher rate of
complications

Page - 1372
Internal Medicine - Hepatology

Question 59/66

Question #59

A 59-year-old male with liver cirrhosis is complaining of abdominal discomfort and fever. His history
started 2 weeks ago. His medications include spironolactone and furosemide. His examination
reveals a fever of 38.0 °C, vague abdominal tenderness, and shiing dullness. What is the most
appropriate to establish the diagnosis of Spontaneous Bacterial Peritonitis (SBP)?

a. Ascitic PH < 7.2


b. Bloody looking ascitic fluid
c. Absolute Neutrophil Count (ANC) > 250*10^6 cells/L
d. Total protein of 2 g/dL
e. History of abdominal pain and fever

‫اﻹﺟﺎﺑﺔ ﻋﲆ اﻟﺼﻔﺤﺔ اﻟﺘﺎﻟﻴﺔ‬

Page - 1373
Internal Medicine - Hepatology - Spontaneous Bacterial Peritonitis (SBP)

Question 59/66

Question #59

A 59-year-old male with liver cirrhosis is complaining of abdominal discomfort and fever. His history
started 2 weeks ago. His medications include spironolactone and furosemide. His examination
reveals a fever of 38.0 °C, vague abdominal tenderness, and shiing dullness. What is the most
appropriate to establish the diagnosis of Spontaneous Bacterial Peritonitis (SBP)?

a. Ascitic PH < 7.2


b. Bloody looking ascitic fluid
c. Absolute Neutrophil Count (ANC) > 250*10^6 cells/L √
d. Total protein of 2 g/dL
e. History of abdominal pain and fever

Description

Spontaneous bacterial peritonitis (SBP) is diagnosed by the following findings:

High ANC (Absolute neutrophil count) in ascites (best initial test)


Culture 80% sensitivity (usually not needed for diagnosis) (most accurate test)

Note that the ascitic PH doesn’t go down until the ANC is elevated, so it is less reliable than ANC

Bloody-looking ascites may be associated with malignancy or traumatic paracentesis, but it is not
specific to SBP.

e history of abdominal pain and fever is not highly specific and may be absent in 30% of PBP cases.

Page - 1374
Internal Medicine - Hepatology

Question 60/66

Question #60

A 53-year-old male complains of fever and confusion. On examination, ascites is easily verified, and
the liver edge is firm and irregular. Serology tests show serum albumin of 23g/L and a Total protein
of 45 g/L. Paracentesis is done and shows cloudy-looking fluid with albumin of 6 g/L, total protein of
10 g/L, absolute neutrophil count of 540*10^6 cells/L, and the cytology is negative for malignant
cells. Ascitic fluid culture is positive for gram-negative bacilli. What is the most likely diagnosis?

a. Pancreatitis related ascites


b. Tuberculous peritonitis
c. Intestinal perforation and peritonitis
d. Spontaneous bacterial peritonitis
e. Peritoneal cancer

‫اﻹﺟﺎﺑﺔ ﻋﲆ اﻟﺼﻔﺤﺔ اﻟﺘﺎﻟﻴﺔ‬

Page - 1375
Internal Medicine - Hepatology - Spontaneous Bacterial Peritonitis (SBP)

Question 60/66

Question #60

A 53-year-old male complains of fever and confusion. On examination, ascites is easily verified, and
the liver edge is firm and irregular. Serology tests show serum albumin of 23g/L and a Total protein
of 45 g/L. Paracentesis is done and shows cloudy-looking fluid with albumin of 6 g/L, total protein of
10 g/L, absolute neutrophil count of 540*10^6 cells/L, and the cytology is negative for malignant
cells. Ascitic fluid culture is positive for gram-negative bacilli. What is the most likely diagnosis?

a. Pancreatitis related ascites


b. Tuberculous peritonitis
c. Intestinal perforation and peritonitis
d. Spontaneous bacterial peritonitis √
e. Peritoneal cancer

Description

Serum ascites albumin gradient (SAAG) = 23 – 6 = 17 g/L = 1.7 g/dL

e total ascitic protein is 10g/L = 1g/dL

SAAG is more than 1.1 g/dL, and total protein is less than 2.5 g/dL.

So, the diagnosis is liver cirrhosis.

e ANC and the presence of gram-negative bacilli (E. coli) strongly suggest spontaneous bacterial
peritonitis

ere are no features suggestive of intestinal perforation.

Malignancy is unlikely because the ascitic fluid is negative for malignant cells.

e following table shows the dierential diagnosis of ascites according to SAAG and total ascitic
protein level.

Page - 1376
Page - 1377
Internal Medicine - Hepatology

Question 61/66

Question #61

A patient with liver cirrhosis and ascites has abdominal discomfort and mild fever. Which of the
following is not consistent with spontaneous bacterial peritonitis?

a. Abdominal discomfort
b. e temperature of 37.8 °C
c. Multiple organisms in ascitic fluid culture
d. Ascitic neutrophil count of 400*10^6 cells/L
e. WBC count in the ascitic fluid of 650*10^6 cell/L

‫اﻹﺟﺎﺑﺔ ﻋﲆ اﻟﺼﻔﺤﺔ اﻟﺘﺎﻟﻴﺔ‬

Page - 1378
Internal Medicine - Hepatology - Spontaneous Bacterial Peritonitis (SBP)

Question 61/66

Question #61

A patient with liver cirrhosis and ascites has abdominal discomfort and mild fever. Which of the
following is not consistent with spontaneous bacterial peritonitis?

a. Abdominal discomfort
b. e temperature of 37.8 °C
c. Multiple organisms in ascitic fluid culture √
d. Ascitic neutrophil count of 400*10^6 cells/L
e. WBC count in the ascitic fluid of 650*10^6 cell/L

Description

Spontaneous bacterial peritonitis (SBP) is a bacterial infection in patients with ascites but without
bowel perforation.

It is common in cirrhotic ascites, especially in alcoholics.

SBP is more common in patients with GI bleeding.

e most common bacteria causing SBP is E. coli

Streptococcus pneumonia is another possible cause.

Usually, a single organism causes SBP (polymicrobial infection is rare)

Page - 1379
Internal Medicine - Hepatology

Question 62/66

Question #62

A 30-year-old female is diagnosed with Wilson disease and started on a copper chelating agent.
Which of the following would be the best to monitor the treatment response?

a. Serum ceruloplasmin
b. Abdominal CT scan
c. Liver function test
d. Urinary copper excretion
e. No need to monitor

‫اﻹﺟﺎﺑﺔ ﻋﲆ اﻟﺼﻔﺤﺔ اﻟﺘﺎﻟﻴﺔ‬

Page - 1380
Internal Medicine - Hepatology - Wilson Disease

Question 62/66

Question #62

A 30-year-old female is diagnosed with Wilson disease and started on a copper chelating agent.
Which of the following would be the best to monitor the treatment response?

a. Serum ceruloplasmin
b. Abdominal CT scan
c. Liver function test
d. Urinary copper excretion √
e. No need to monitor

Description

Wilson disease is an autosomal recessive disorder of copper metabolism on chromosome 13


Copper is normally absorbed in the stomach and proximal small intestine, → taken to the liver, →
stored → incorporated into ceruloplasmin → secreted to the blood
In Wilsons disease: no synthesis of ceruloplasmin → copper accumulation

Treatment:

Low copper diet


A chelating agent (penicillamine for life)
Liver transplant, if indicated

e best test to monitor the treatment response is to measure the urinary excretion of copper.

Normal urinary copper excretion is less than 40 micrograms/day

In patients with Wilson disease, urinary copper excretion is above 100

microgram/day.

e use of copper chelating agents (D-penicillamine) promotes the 24-hour excretion of


approximately 2 mg of copper in the urine to induce a negative copper balance.

Page - 1381
Internal Medicine - Hepatology

Question 63/66

Question #63

A 21-year-old woman presents to the clinic with new symptoms of tremors and incoordination for 2
months. Symptoms have progressively worsened to the point that she fell 1 week ago. An
ophthalmologic examination reveals a brownish-pigmented ring at the corneal margin. Which of
the following is the treatment of choice for this condition?

a. Deferoxamine
b. Edetate-Disodium
c. Penicillin
d. Penicillamine
e. Phlebotomy

‫اﻹﺟﺎﺑﺔ ﻋﲆ اﻟﺼﻔﺤﺔ اﻟﺘﺎﻟﻴﺔ‬

Page - 1382
Internal Medicine - Hepatology - Wilson Disease

Question 63/66

Question #63

A 21-year-old woman presents to the clinic with new symptoms of tremors and incoordination for 2
months. Symptoms have progressively worsened to the point that she fell 1 week ago. An
ophthalmologic examination reveals a brownish-pigmented ring at the corneal margin. Which of
the following is the treatment of choice for this condition?

a. Deferoxamine
b. Edetate-Disodium
c. Penicillin
d. Penicillamine √
e. Phlebotomy

Description

e neurological symptoms and Kayser-Fleischer ring in the eye strongly suggest Wilson’s disease.

It is an autosomal recessive disorder of copper metabolism on chromosome 13


Copper is normally absorbed in the stomach and proximal small intestine, → taken to the liver, →
stored → incorporated into ceruloplasmin → secreted to the blood
In Wilsons disease: no synthesis of ceruloplasmin → copper accumulation

Treatment:

Low copper diet


A chelating agent (penicillamine for life)
Liver transplant, if indicated

Page - 1383
Internal Medicine - Hepatology

Question 64/66

Question #64

A 22-year-old male patient develops memory changes and diculty in speaking. His family history is
significant for a paternal uncle who had a similar condition and died at 45. On examination, you
noted hepatomegaly. In addition, the slit lamp examination of the eye shows a hyperpigmented ring
around the cornea. What is the most accurate test to establish the diagnosis?

a. Serum ceruloplasmin
b. Urinary copper level
c. Serum copper level
d. Liver biopsy
e. Liver function test

‫اﻹﺟﺎﺑﺔ ﻋﲆ اﻟﺼﻔﺤﺔ اﻟﺘﺎﻟﻴﺔ‬

Page - 1384
Internal Medicine - Hepatology - Wilson Disease

Question 64/66

Question #64

A 22-year-old male patient develops memory changes and diculty in speaking. His family history is
significant for a paternal uncle who had a similar condition and died at 45. On examination, you
noted hepatomegaly. In addition, the slit lamp examination of the eye shows a hyperpigmented ring
around the cornea. What is the most accurate test to establish the diagnosis?

a. Serum ceruloplasmin
b. Urinary copper level
c. Serum copper level
d. Liver biopsy √
e. Liver function test

Description

Liver disease, neurological symptoms, and Kayser-Fleischer ring in the eye strongly suggest Wilson’s
disease.

It is an autosomal recessive disorder of copper metabolism on chromosome 13


Copper is normally absorbed in the stomach and proximal small intestine, → taken to the liver, →
stored → incorporated into ceruloplasmin → secreted to the blood
In Wilsons disease: no synthesis of ceruloplasmin → copper accumulation

Diagnosis of Wilson disease:

Low serum ceruloplasmin (best lab for diagnosis)


High free serum copper
High urine copper
Very high hepatic copper (biopsy) (the best)

Page - 1385
Internal Medicine - Hepatology

Question 65/66

Question #65

A patient develops dementia and is diagnosed with Wilson disease. Which of the following is
accurate about this condition?

a. e main aected organ is the heart


b. It is a disease of copper overabsorption
c. Low serum ceruloplasmin level is characteristic
d. It is an autosomal dominant condition
e. e chelating agent Edetate-disodium is used to treat the condition

‫اﻹﺟﺎﺑﺔ ﻋﲆ اﻟﺼﻔﺤﺔ اﻟﺘﺎﻟﻴﺔ‬

Page - 1386
Internal Medicine - Hepatology - Wilson Disease

Question 65/66

Question #65

A patient develops dementia and is diagnosed with Wilson disease. Which of the following is
accurate about this condition?

a. e main aected organ is the heart


b. It is a disease of copper overabsorption
c. Low serum ceruloplasmin level is characteristic √
d. It is an autosomal dominant condition
e. e chelating agent Edetate-disodium is used to treat the condition

Description

Low serum ceruloplasmin is the best lab for diagnosis of Wilson’s disease

Wilson disease is an autosomal recessive “Not dominant” disorder of copper metabolism on


chromosome 13
e main aected organs are the Liver, Basal ganglia of the brain, Eye, Kidney, and Skeleton

Pathogenesis:

Copper is normally absorbed in the stomach and proximal small intestine, → taken to the liver, →
stored → incorporated into ceruloplasmin → secreted to the blood
In Wilsons disease: no synthesis of ceruloplasmin → copper accumulation

Treatment:

Low copper diet


A chelating agent (penicillamine for life)
Liver transplant, if indicated

Page - 1387
Internal Medicine - Hepatology

Question 66/66

Question #66

A 22-year-old male patient develops memory changes and diculty in speaking. His family history is
significant for a paternal uncle who had a similar condition and died at 45. On examination, you
noted hepatomegaly. In addition, the slit lamp examination of the eye shows a hyperpigmented ring
around the cornea. What is the most likely diagnosis?

a. Hemochromatosis
b. Wilson disease
c. Sarcoidosis
d. Ischemic stroke
e. Subarachnoid hemorrhage

‫اﻹﺟﺎﺑﺔ ﻋﲆ اﻟﺼﻔﺤﺔ اﻟﺘﺎﻟﻴﺔ‬

Page - 1388
Internal Medicine - Hepatology - Wilson Disease

Question 66/66

Question #66

A 22-year-old male patient develops memory changes and diculty in speaking. His family history is
significant for a paternal uncle who had a similar condition and died at 45. On examination, you
noted hepatomegaly. In addition, the slit lamp examination of the eye shows a hyperpigmented ring
around the cornea. What is the most likely diagnosis?

a. Hemochromatosis
b. Wilson disease √
c. Sarcoidosis
d. Ischemic stroke
e. Subarachnoid hemorrhage

Description

Liver disease, neurological symptoms, and Kayser-Fleischer ring in the eye strongly suggest Wilson’s
disease.

It is an autosomal recessive disorder of copper metabolism on chromosome 13


Copper is normally absorbed in the stomach and proximal small intestine, → taken to the liver, →
stored → incorporated into ceruloplasmin → secreted to the blood
In Wilsons disease: no synthesis of ceruloplasmin → copper accumulation

Page - 1389
Infectious diseases

Page - 1390
Infectious diseases

‫ﻣﻮاﺿﻴﻊ اﻷﺳﺌﻠﺔ وأﻋﺪادﻫﺎ‬

1) Brucellosis => 1 Questions


2) Chlamydia => 2 Questions
3) Gonorrhea => 3 Questions
4) HIV Infection => 6 Questions
5) Herpes Simplex virus => 3 Questions
6) Herpes Zoster infection => 5 Questions
7) Infectious Mononucleosis => 2 Questions
8) Lyme Disease => 3 Questions
9) Malaria => 2 Questions
10) Rabies => 1 Questions
11) Sepsis => 3 Questions
12) Staphylococcus infection => 2 Questions
13) Syphilis => 2 Questions
14) Tetanus => 2 Questions
15) e Antibiotics => 5 Questions
16) Typhoid fever (Enteric fever) => 2 Questions

Page - 1391
Internal Medicine - Infectious diseases

Question 1/44

Question #1

A 22-year-old male patient drank un-boiled milk yesterday. He came to your oce and asked which
diseases are transmitted by milk. Your answer will be all the following except:

a. Brucellosis
b. Tuberculosis
c. Staphylococcus food poisoning
d. Toxoplasmosis
e. Infectious mononucleosis

‫اﻹﺟﺎﺑﺔ ﻋﲆ اﻟﺼﻔﺤﺔ اﻟﺘﺎﻟﻴﺔ‬

Page - 1392
Internal Medicine - Infectious diseases - Brucellosis

Question 1/44

Question #1

A 22-year-old male patient drank un-boiled milk yesterday. He came to your oce and asked which
diseases are transmitted by milk. Your answer will be all the following except:

a. Brucellosis
b. Tuberculosis
c. Staphylococcus food poisoning
d. Toxoplasmosis √
e. Infectious mononucleosis

Description

Brucellosis is also known as undulant fever or Malta fever


e cause: Brucella organism (gram-negative coccobacilli)
Rout of infection: Ingestion of raw milk from infected cattle or goats
e bacteria travel in the lymphatic system and infect lymph nodes, then spread by the
hematogenous route to localize in the reticuloendothelial system
e most common cause of death is endocarditis

Page - 1393
Internal Medicine - Infectious diseases

Question 2/44

Question #2

A 22-year-old male presented with purulent urethral discharge and was treated with ceriaxone
aer being diagnosed with gonorrhea. One week later, he came with dysuria and urethral discharge.
What would you do next?

a. Repeat the treatment with ceriaxone


b. Give Azithromycin 500mg orally – single dose
c. Give a single dose of benzylpenicillin intramuscular
d. Give ciprofloxacin orally 500mg bid of r7 days
e. Start treatment with nitrofurantoin

‫اﻹﺟﺎﺑﺔ ﻋﲆ اﻟﺼﻔﺤﺔ اﻟﺘﺎﻟﻴﺔ‬

Page - 1394
Internal Medicine - Infectious diseases - Chlamydia

Question 2/44

Question #2

A 22-year-old male presented with purulent urethral discharge and was treated with ceriaxone
aer being diagnosed with gonorrhea. One week later, he came with dysuria and urethral discharge.
What would you do next?

a. Repeat the treatment with ceriaxone


b. Give Azithromycin 500mg orally – single dose √
c. Give a single dose of benzylpenicillin intramuscular
d. Give ciprofloxacin orally 500mg bid of r7 days
e. Start treatment with nitrofurantoin

Description

Chlamydia is suspected in a patient who is not improving on gonorrhea treatment

Chlamydia infection:

It is a bacterial infection that is transmitted similarly to gonorrhea


Clinically also similar to gonorrhea
In men, urethral symptoms are mild, and < 50% of cases
In women, the most common sites are the cervix and urethra
Chlamydia is treated by Azithromycin 1g PO single dose (first line), or Doxycycline 100mg bd for
7 days

Page - 1395
Internal Medicine - Infectious diseases

Question 3/44

Question #3

A 22-year-old male presented with purulent urethral discharge and was treated with ceriaxone
aer being diagnosed with gonorrhea. One week later, he came with dysuria and urethral discharge.
What would you do next?

a. Repeat the treatment with ceriaxone


b. Give doxycycline 100 mg Bid for 7 days
c. Give a single dose of benzylpenicillin intramuscular
d. Give ciprofloxacin orally 500mg bid of r7 days
e. Start treatment with nitrofurantoin

‫اﻹﺟﺎﺑﺔ ﻋﲆ اﻟﺼﻔﺤﺔ اﻟﺘﺎﻟﻴﺔ‬

Page - 1396
Internal Medicine - Infectious diseases - Chlamydia

Question 3/44

Question #3

A 22-year-old male presented with purulent urethral discharge and was treated with ceriaxone
aer being diagnosed with gonorrhea. One week later, he came with dysuria and urethral discharge.
What would you do next?

a. Repeat the treatment with ceriaxone


b. Give doxycycline 100 mg Bid for 7 days √
c. Give a single dose of benzylpenicillin intramuscular
d. Give ciprofloxacin orally 500mg bid of r7 days
e. Start treatment with nitrofurantoin

Description

Chlamydia is suspected in a patient who is not improving on gonorrhea treatment

Chlamydia infection:

It is a bacterial infection that is transmitted similarly to gonorrhea


Clinically also similar to gonorrhea
In men, urethral symptoms are mild, and < 50% of cases
In women, the most common sites are the cervix and urethra
Chlamydia is treated by Azithromycin 1g PO single dose (first line), or Doxycycline 100mg bd for
7 days

Page - 1397
Internal Medicine - Infectious diseases

Question 4/44

Question #4

Which of the following is the most eective for treating a 20-year-old male patient with purulent
urethral discharge who is confirmed to have gonorrhea?

a. Once-daily amoxicillin of 3 grams dose


b. One dose of ciprofloxacin 500 mg orally
c. One dose of ceriaxone 500 mg intramuscular
d. 3 days course of twice daily 100 mg doxycycline
e. One dose of Azithromycin 1 gram orally

‫اﻹﺟﺎﺑﺔ ﻋﲆ اﻟﺼﻔﺤﺔ اﻟﺘﺎﻟﻴﺔ‬

Page - 1398
Internal Medicine - Infectious diseases - Gonorrhea

Question 4/44

Question #4

Which of the following is the most eective for treating a 20-year-old male patient with purulent
urethral discharge who is confirmed to have gonorrhea?

a. Once-daily amoxicillin of 3 grams dose


b. One dose of ciprofloxacin 500 mg orally
c. One dose of ceriaxone 500 mg intramuscular √
d. 3 days course of twice daily 100 mg doxycycline
e. One dose of Azithromycin 1 gram orally

Description

One dose of ceriaxone 500 mg intramuscular is the eective treatment of gonorrhea

Page - 1399
Internal Medicine - Infectious diseases

Question 5/44

Question #5

A 23-year-old female patient with multiple sex partners is suspected of having gonorrhea. What is
the treatment of choice for her condition?

a. Ceriaxone for her and all her sex partners


b. Azithromycin for her and all her sex partners
c. Ceriaxone for her and her symptomatic sex partners
d. Azithromycin for her and her symptomatic sex partners
e. Penicillin for her and her symptomatic sex partners

‫اﻹﺟﺎﺑﺔ ﻋﲆ اﻟﺼﻔﺤﺔ اﻟﺘﺎﻟﻴﺔ‬

Page - 1400
Internal Medicine - Infectious diseases - Gonorrhea

Question 5/44

Question #5

A 23-year-old female patient with multiple sex partners is suspected of having gonorrhea. What is
the treatment of choice for her condition?

a. Ceriaxone for her and all her sex partners √


b. Azithromycin for her and all her sex partners
c. Ceriaxone for her and her symptomatic sex partners
d. Azithromycin for her and her symptomatic sex partners
e. Penicillin for her and her symptomatic sex partners

Description

e patient and all the sex partners should receive treatment for gonorrhea regardless of the
symptoms.

Page - 1401
Internal Medicine - Infectious diseases

Question 6/44

Question #6

A 29-year-old female with multiple sex partners presents with migratory arthropathy, papular rash,
and tenosynovitis. What is the treatment of choice for this condition?

a. Once-daily amoxicillin of 3 grams dose


b. One dose of ciprofloxacin 500 mg orally
c. One dose of ceriaxone 500 mg intramuscular
d. 3 days course of twice daily 100 mg doxycycline
e. One dose of Azithromycin 1 gram orally

‫اﻹﺟﺎﺑﺔ ﻋﲆ اﻟﺼﻔﺤﺔ اﻟﺘﺎﻟﻴﺔ‬

Page - 1402
Internal Medicine - Infectious diseases - Gonorrhea

Question 6/44

Question #6

A 29-year-old female with multiple sex partners presents with migratory arthropathy, papular rash,
and tenosynovitis. What is the treatment of choice for this condition?

a. Once-daily amoxicillin of 3 grams dose


b. One dose of ciprofloxacin 500 mg orally
c. One dose of ceriaxone 500 mg intramuscular √
d. 3 days course of twice daily 100 mg doxycycline
e. One dose of Azithromycin 1 gram orally

Description

One dose of ceriaxone 500 mg intramuscular is the eective treatment of gonorrhea

Page - 1403
Internal Medicine - Infectious diseases

Question 7/44

Question #7

A 40-year-old male patient is a known case of HIV and latent TB. What is the most appropriate
treatment regimen for an HIV-positive patient with latent TB?

a. INH for 9 months


b. INH, Rifampicin, pyrazinamide, and ethambutol
c. Rifampicin for 6 moths
d. Rifampin plus pyrazinamide daily for 4 months
e. Streptomycin for 4 months duration

‫اﻹﺟﺎﺑﺔ ﻋﲆ اﻟﺼﻔﺤﺔ اﻟﺘﺎﻟﻴﺔ‬

Page - 1404
Internal Medicine - Infectious diseases - HIV Infection

Question 7/44

Question #7

A 40-year-old male patient is a known case of HIV and latent TB. What is the most appropriate
treatment regimen for an HIV-positive patient with latent TB?

a. INH for 9 months √


b. INH, Rifampicin, pyrazinamide, and ethambutol
c. Rifampicin for 6 moths
d. Rifampin plus pyrazinamide daily for 4 months
e. Streptomycin for 4 months duration

Description

Isoniazid monotherapy is the treatment of choice for patients with latent TB.

Page - 1405
Internal Medicine - Infectious diseases

Question 8/44

Question #8

In a patient with HIV, which organism can cause retinitis, encephalitis, enterocolitis, and interstitial
pneumonia?

a. Mycobacterium avium-intercellulare
b. Pneumocystis carinii
c. CMV
d. EBV
e. Mycoplasma pneumonia

‫اﻹﺟﺎﺑﺔ ﻋﲆ اﻟﺼﻔﺤﺔ اﻟﺘﺎﻟﻴﺔ‬

Page - 1406
Internal Medicine - Infectious diseases - HIV Infection

Question 8/44

Question #8

In a patient with HIV, which organism can cause retinitis, encephalitis, enterocolitis, and interstitial
pneumonia?

a. Mycobacterium avium-intercellulare
b. Pneumocystis carinii
c. CMV √
d. EBV
e. Mycoplasma pneumonia

Description

CMV can cause those conditions in an HIV patient with CD4 less than 50 cells/mm3

e most important opportunistic infections in HIV are shown in the following table:

Page - 1407
Internal Medicine - Infectious diseases

Question 9/44

Question #9

A 22-year-old female is a known case of HIV. Despite taking antiviral therapy, her CD4 count is 35
cells/mm3. Which of the following is the most appropriate prophylactic agent to be added to this
patient’s treatment at this point?

a. Azithromycin
b. TMP/SMX
c. Ceriaxone
d. Ciprofloxacin
e. Amphotericin B

‫اﻹﺟﺎﺑﺔ ﻋﲆ اﻟﺼﻔﺤﺔ اﻟﺘﺎﻟﻴﺔ‬

Page - 1408
Internal Medicine - Infectious diseases - HIV Infection

Question 9/44

Question #9

A 22-year-old female is a known case of HIV. Despite taking antiviral therapy, her CD4 count is 35
cells/mm3. Which of the following is the most appropriate prophylactic agent to be added to this
patient’s treatment at this point?

a. Azithromycin √
b. TMP/SMX
c. Ceriaxone
d. Ciprofloxacin
e. Amphotericin B

Description

Any HIV patient with CD4 less than 50 cells/mm3 should receive Azithromycin as prophylaxis for
Mycobacterium avium-intercellulare

e most important opportunistic infections in HIV are shown in the following table:

Page - 1409
Internal Medicine - Infectious diseases

Question 10/44

Question #10

A 36-year-old male patient with a known case of HIV is suspected of having an opportunistic
organism. His CD4 count was ordered, and we are waiting for the results. What is the least likely
cause?

a. Candida albicans
b. Aspergillus fumigatus
c. Cytomegalovirus
d. Pneumocystis carinii
e. Mycoplasma pneumonia

‫اﻹﺟﺎﺑﺔ ﻋﲆ اﻟﺼﻔﺤﺔ اﻟﺘﺎﻟﻴﺔ‬

Page - 1410
Internal Medicine - Infectious diseases - HIV Infection

Question 10/44

Question #10

A 36-year-old male patient with a known case of HIV is suspected of having an opportunistic
organism. His CD4 count was ordered, and we are waiting for the results. What is the least likely
cause?

a. Candida albicans
b. Aspergillus fumigatus
c. Cytomegalovirus
d. Pneumocystis carinii
e. Mycoplasma pneumonia √

Description

Mycoplasma pneumonia is not one of the opportunistic organisms in HIV patients.

e most important opportunistic infections in HIV are shown in the following table:

Page - 1411
Internal Medicine - Infectious diseases

Question 11/44

Question #11

A 34-year-old IV drug user presents with fever, SOB, cough with sputum, and is diagnosed with
tuberculosis. Which of the following conditions is the most likely associated with this illness?

a. Beta-thalassemia
b. Alcoholism
c. Drug abuse
d. HIV infection
e. Lymphoma

‫اﻹﺟﺎﺑﺔ ﻋﲆ اﻟﺼﻔﺤﺔ اﻟﺘﺎﻟﻴﺔ‬

Page - 1412
Internal Medicine - Infectious diseases - HIV Infection

Question 11/44

Question #11

A 34-year-old IV drug user presents with fever, SOB, cough with sputum, and is diagnosed with
tuberculosis. Which of the following conditions is the most likely associated with this illness?

a. Beta-thalassemia
b. Alcoholism
c. Drug abuse
d. HIV infection √
e. Lymphoma

Description

Tuberculosis and HIV have been strongly associated with each other

Page - 1413
Internal Medicine - Infectious diseases

Question 12/44

Question #12

An HIV 33-year-old male patient presented with oral thrush and dysphagia. What is the most likely
diagnosis?

a. Candida esophagitis
b. Viral glossitis
c. Vitamin A deficiency
d. Achalasia
e. Esophageal cancer

‫اﻹﺟﺎﺑﺔ ﻋﲆ اﻟﺼﻔﺤﺔ اﻟﺘﺎﻟﻴﺔ‬

Page - 1414
Internal Medicine - Infectious diseases - HIV Infection

Question 12/44

Question #12

An HIV 33-year-old male patient presented with oral thrush and dysphagia. What is the most likely
diagnosis?

a. Candida esophagitis √
b. Viral glossitis
c. Vitamin A deficiency
d. Achalasia
e. Esophageal cancer

Description

Oral thrush and dysphagia in an immunocompromised patient are the classic presentation of
Esophageal candidiasis

e most important opportunistic infections in HIV are shown in the following table:

Page - 1415
Internal Medicine - Infectious diseases

Question 13/44

Question #13

A 22-year-old female patient who was previously healthy presented with a skin condition on her lips
(see the picture). What is the most likely diagnosis?

a. Angioedema
b. Herpes simplex virus
c. Impetigo
d. Basal cell carcinoma
e. Squamous cell carcinoma

‫اﻹﺟﺎﺑﺔ ﻋﲆ اﻟﺼﻔﺤﺔ اﻟﺘﺎﻟﻴﺔ‬

Page - 1416
Internal Medicine - Infectious diseases - Herpes Simplex virus

Question 13/44

Question #13

A 22-year-old female patient who was previously healthy presented with a skin condition on her lips
(see the picture). What is the most likely diagnosis?

a. Angioedema
b. Herpes simplex virus √
c. Impetigo
d. Basal cell carcinoma
e. Squamous cell carcinoma

Description

is is an HSV infection aecting the lips.

Herps simple virus (HSV):

Organism: Herpesvirus type 1 and type 2

It can be a primary or recurrent infection


Primary: ulcerative stomatitis, Keratitis, Finger infection, Vulvovaginitis, Balanitis, encephalitis
Recurrent infection: commonest at Lips (herpes labialis) or genital lesions.
Usually, no need for treatment, but acyclovir can be used in severe or systemic diseases.

Page - 1417
Internal Medicine - Infectious diseases

Question 14/44

Question #14

A 30-year-old dentist presented with painful blisters at his distal phalanx of the right index finger,
shown in the following picture. In addition, he has a low-grade fever and tender enlarged right
axillary lymph nodes. Which one of the following is the most likely diagnosis?

a. Herpetic whitlow
b. Traumatic lesion
c. Felon
d. Tenosynovitis
e. Basal cell carcinoma

‫اﻹﺟﺎﺑﺔ ﻋﲆ اﻟﺼﻔﺤﺔ اﻟﺘﺎﻟﻴﺔ‬

Page - 1418
Internal Medicine - Infectious diseases - Herpes Simplex virus

Question 14/44

Question #14

A 30-year-old dentist presented with painful blisters at his distal phalanx of the right index finger,
shown in the following picture. In addition, he has a low-grade fever and tender enlarged right
axillary lymph nodes. Which one of the following is the most likely diagnosis?

a. Herpetic whitlow √
b. Traumatic lesion
c. Felon
d. Tenosynovitis
e. Basal cell carcinoma

Description

Herpetic whitlow results from inoculation of type 1 or type 2 herpes simplex virus into broken skin

Herpes simplex virus (HSV):

Organism: Herpesvirus type 1 and type 2

It can be a primary or recurrent infection


Primary: ulcerative stomatitis, Keratitis, Finger infection, Vulvovaginitis, Balanitis, encephalitis
Recurrent infection: commonest at Lips (herpes labialis) or genital lesions.
Usually, no need for treatment, but acyclovir can be used in severe or systemic diseases.
Page - 1419
Page - 1420
Internal Medicine - Infectious diseases

Question 15/44

Question #15

A 22-year-old pregnant female presents at her 37-week gestation with labor pain. Her examination
shows vaginal blisters consistent with active herpes simplex lesions. What would you do regarding
her delivery?

a. Vaginal delivery without additional intervention


b. Vaginal delivery with the use of acyclovir cream before delivery
c. Give tocolytics and treat the lesion, then proceed with vaginal delivery
d. Cesarean section delivery
e. Vaginal delivery and administration of acyclovir to the newborn

‫اﻹﺟﺎﺑﺔ ﻋﲆ اﻟﺼﻔﺤﺔ اﻟﺘﺎﻟﻴﺔ‬

Page - 1421
Internal Medicine - Infectious diseases - Herpes Simplex virus

Question 15/44

Question #15

A 22-year-old pregnant female presents at her 37-week gestation with labor pain. Her examination
shows vaginal blisters consistent with active herpes simplex lesions. What would you do regarding
her delivery?

a. Vaginal delivery without additional intervention


b. Vaginal delivery with the use of acyclovir cream before delivery
c. Give tocolytics and treat the lesion, then proceed with vaginal delivery
d. Cesarean section delivery √
e. Vaginal delivery and administration of acyclovir to the newborn

Description

Active herpes lesion at the genitalia is a contraindication for vaginal delivery.

Herpes simplex virus (HSV):

Organism: Herpesvirus type 1 and type 2

It can be a primary or recurrent infection


Primary: ulcerative stomatitis, Keratitis, Finger infection, Vulvovaginitis, Balanitis, encephalitis
Recurrent infection: commonest at Lips (herpes labialis) or genital lesions.
Usually, no need for treatment, but acyclovir can be used in severe or systemic diseases.

Page - 1422
Internal Medicine - Infectious diseases

Question 16/44

Question #16

A 55-year-old male patient developed a burning pain followed by a vesicular rash on the le side of
the chest. What is the best treatment for his condition?

a. Only pain killer


b. Oral acyclovir
c. Systemic steroids
d. Topical acyclovir
e. Topical steroids

‫اﻹﺟﺎﺑﺔ ﻋﲆ اﻟﺼﻔﺤﺔ اﻟﺘﺎﻟﻴﺔ‬

Page - 1423
Internal Medicine - Infectious diseases - Herpes Zoster infection

Question 16/44

Question #16

A 55-year-old male patient developed a burning pain followed by a vesicular rash on the le side of
the chest. What is the best treatment for his condition?

a. Only pain killer


b. Oral acyclovir √
c. Systemic steroids
d. Topical acyclovir
e. Topical steroids

Description

e treatment of choice for acute herpes zoster is oral acyclovir, valacyclovir, or famciclovir for 7
days.

Shingles:

is virus may become latent aer primary infection, then later in adulthood, will reactivate
and present with (Shingles)
Present with severe burning pain at a specific dermatome site with a vesicular rash that does
not cross the Medline of the body
Shingles are not contagious
It may be followed by neuropathic pain aer resolution (Gabapentin or Pregabalin is the drug
of choice for neuropathic pain)
Treatment: Analgesia and antiviral (acyclovir, valacyclovir, famciclovir)

Page - 1424
Internal Medicine - Infectious diseases

Question 17/44

Question #17

A 56-year-old male patient presents with a skin rash on his right thigh (on a specific dermatome)
consistent with shingles. What is the most appropriate treatment?

a. Only symptomatic and painkillers


b. Acyclovir oral tablets 800 mg five times a day for 7 days
c. Topical acyclovir 5 days duration for 7 days
d. Oral prednisolone 30mg for 2 1 week
e. Ganciclovir 1000 mg tid for a 1-week duration

‫اﻹﺟﺎﺑﺔ ﻋﲆ اﻟﺼﻔﺤﺔ اﻟﺘﺎﻟﻴﺔ‬

Page - 1425
Internal Medicine - Infectious diseases - Herpes Zoster infection

Question 17/44

Question #17

A 56-year-old male patient presents with a skin rash on his right thigh (on a specific dermatome)
consistent with shingles. What is the most appropriate treatment?

a. Only symptomatic and painkillers


b. Acyclovir oral tablets 800 mg five times a day for 7 days √
c. Topical acyclovir 5 days duration for 7 days
d. Oral prednisolone 30mg for 2 1 week
e. Ganciclovir 1000 mg tid for a 1-week duration

Description

e treatment of choice for acute herpes zoster is oral acyclovir, valacyclovir, or famciclovir for 7
days.

Shingles:

is virus may become latent aer primary infection, then later in adulthood, will reactivate
and present with (Shingles)
Present with severe burning pain at a specific dermatome site with a vesicular rash that does
not cross the Medline of the body
Shingles are not contagious
It may be followed by neuropathic pain aer resolution (Gabapentin or Pregabalin is the drug
of choice for neuropathic pain)
Treatment: Analgesia and antiviral (acyclovir, valacyclovir, famciclovir)

Page - 1426
Internal Medicine - Infectious diseases

Question 18/44

Question #18

A 55-year-old male patient develops atypical right-side chest burning pain in his skin provoked by
touching or when the area is rubbed by his shirt. In addition, the patient had shingles about 3 weeks
ago. Which is the best treatment for the patient’s condition?

a. Oral Acyclovir
b. Topical acyclovir
c. Prednisolone
d. Amitriptyline
e. Ibuprofen

‫اﻹﺟﺎﺑﺔ ﻋﲆ اﻟﺼﻔﺤﺔ اﻟﺘﺎﻟﻴﺔ‬

Page - 1427
Internal Medicine - Infectious diseases - Herpes Zoster infection

Question 18/44

Question #18

A 55-year-old male patient develops atypical right-side chest burning pain in his skin provoked by
touching or when the area is rubbed by his shirt. In addition, the patient had shingles about 3 weeks
ago. Which is the best treatment for the patient’s condition?

a. Oral Acyclovir
b. Topical acyclovir
c. Prednisolone
d. Amitriptyline √
e. Ibuprofen

Description

Tricyclic antidepressants are used to treat post-herpetic neuralgia.

Shingles:

is virus may become latent aer primary infection, then later in adulthood, will reactivate
and present with (Shingles)
Present with severe burning pain at a specific dermatome site with a vesicular rash that does
not cross the Medline of the body
Shingles are not contagious
It may be followed by neuropathic pain aer resolution (Gabapentin or Pregabalin is the drug
of choice for neuropathic pain)
Treatment: Analgesia and antiviral (acyclovir, valacyclovir, famciclovir)

Page - 1428
Internal Medicine - Infectious diseases

Question 19/44

Question #19

A 52-year-old male presents with le frontal headaches for 6 days. His frontal area and nose show
blisters on a specific dermatome (see picture). In addition to your treatment for his condition, you
should refer the patient to which of the following specialties?

a. Allergist
b. General internist
c. Neurologist
d. Ophthalmologist
e. Otolaryngologist

‫اﻹﺟﺎﺑﺔ ﻋﲆ اﻟﺼﻔﺤﺔ اﻟﺘﺎﻟﻴﺔ‬

Page - 1429
Internal Medicine - Infectious diseases - Herpes Zoster infection

Question 19/44

Question #19

A 52-year-old male presents with le frontal headaches for 6 days. His frontal area and nose show
blisters on a specific dermatome (see picture). In addition to your treatment for his condition, you
should refer the patient to which of the following specialties?

a. Allergist
b. General internist
c. Neurologist
d. Ophthalmologist √
e. Otolaryngologist

Description

is is a case of herpes zoster ophthalmicus. e possibility of corneal involvement warrants an


ophthalmologist consultation

Shingles:

is virus may become latent aer primary infection, then later in adulthood, will reactivate
Page - 1430
and present with (Shingles)
Present with severe burning pain at a specific dermatome site with a vesicular rash that does
not cross the Medline of the body
Shingles are not contagious
It may be followed by neuropathic pain aer resolution (Gabapentin or Pregabalin is the drug
of choice for neuropathic pain)
Treatment: Analgesia and antiviral (acyclovir, valacyclovir, famciclovir)

Page - 1431
Internal Medicine - Infectious diseases

Question 20/44

Question #20

A 32-year-old female patient presents with burning blisters extending from the epigastrium to the
midline of her back. e most likely diagnosis is:

a. Contact dermatitis
b. Cholecystitis
c. Herpes zoster
d. Herpes simplex
e. Acute pancreatitis

‫اﻹﺟﺎﺑﺔ ﻋﲆ اﻟﺼﻔﺤﺔ اﻟﺘﺎﻟﻴﺔ‬

Page - 1432
Internal Medicine - Infectious diseases - Herpes Zoster infection

Question 20/44

Question #20

A 32-year-old female patient presents with burning blisters extending from the epigastrium to the
midline of her back. e most likely diagnosis is:

a. Contact dermatitis
b. Cholecystitis
c. Herpes zoster √
d. Herpes simplex
e. Acute pancreatitis

Description

Band-like painful blisters on a specific dermatome are a classic presentation for shingles.

Shingles:

is virus may become latent aer primary infection, then later in adulthood, will reactivate
and present with (Shingles)
Present with severe burning pain at a specific dermatome site with a vesicular rash that does
not cross the Medline of the body
Shingles are not contagious
Page - 1433
It may be followed by neuropathic pain aer resolution (Gabapentin or Pregabalin is the drug
of choice for neuropathic pain)
Treatment: Analgesia and antiviral (acyclovir, valacyclovir, famciclovir)

Page - 1434
Internal Medicine - Infectious diseases

Question 21/44

Question #21

A 19-year-old male patient developed a fever and sore throat associated with fatigue and cervical
lymphadenopathy. What is the most common cause of his condition if the rapid streptococcal
antigen test was negative?

a. Streptococcal pharyngitis
b. Infectious mononucleosis
c. Lymphoma
d. Adenovirus infection
e. Acute sinusitis

‫اﻹﺟﺎﺑﺔ ﻋﲆ اﻟﺼﻔﺤﺔ اﻟﺘﺎﻟﻴﺔ‬

Page - 1435
Internal Medicine - Infectious diseases - Infectious Mononucleosis

Question 21/44

Question #21

A 19-year-old male patient developed a fever and sore throat associated with fatigue and cervical
lymphadenopathy. What is the most common cause of his condition if the rapid streptococcal
antigen test was negative?

a. Streptococcal pharyngitis
b. Infectious mononucleosis √
c. Lymphoma
d. Adenovirus infection
e. Acute sinusitis

Description

Fever, fatigue, and lymphadenopathy is the classic presentation of EBV infection

Infectious mononucleosis (Glandular fever):

Organism: EBV (DNA virus)


e virus spreads via infected droplets but is rarely blood-borne (It is also known as Kissing
disease)
Incubation period: 30-50 days

Page - 1436
Internal Medicine - Infectious diseases

Question 22/44

Question #22

A 19-year-old male football player presents with fever, fatigue, and a sore throat. Physical
examination shows posterior cervical lymphadenitis and splenomegaly. e monospot test is
positive. He asked you, “when can I play football again?”. e most appropriate answer would be?

a. When the WBC level returns to normal


b. When the physical examination is normal
c. When symptoms disappear
d. When the Monospot test is negative
e. Aer 4 weeks

‫اﻹﺟﺎﺑﺔ ﻋﲆ اﻟﺼﻔﺤﺔ اﻟﺘﺎﻟﻴﺔ‬

Page - 1437
Internal Medicine - Infectious diseases - Infectious Mononucleosis

Question 22/44

Question #22

A 19-year-old male football player presents with fever, fatigue, and a sore throat. Physical
examination shows posterior cervical lymphadenitis and splenomegaly. e monospot test is
positive. He asked you, “when can I play football again?”. e most appropriate answer would be?

a. When the WBC level returns to normal


b. When the physical examination is normal √
c. When symptoms disappear
d. When the Monospot test is negative
e. Aer 4 weeks

Description

To avoid splenic rupture, he can play contact sports only when the splenomegaly resolves.

Infectious mononucleosis (Glandular fever):

Organism: EBV (DNA virus)


e virus spreads via infected droplets but is rarely blood-borne (It is also known as Kissing
disease)
Incubation period: 30-50 days

Page - 1438
Internal Medicine - Infectious diseases

Question 23/44

Question #23

A 32-year-old male patient developed a fever, myalgia, and a skin rash (see the picture below!).
About 1 week ago, he went on a hike, and the skin rash was noted 2 days later. What is the first-line
treatment for this patient?

a. Azithromycin
b. Doxycycline
c. Clindamycin
d. Clarithromycin
e. Erythromycin

‫اﻹﺟﺎﺑﺔ ﻋﲆ اﻟﺼﻔﺤﺔ اﻟﺘﺎﻟﻴﺔ‬

Page - 1439
Internal Medicine - Infectious diseases - Lyme Disease

Question 23/44

Question #23

A 32-year-old male patient developed a fever, myalgia, and a skin rash (see the picture below!).
About 1 week ago, he went on a hike, and the skin rash was noted 2 days later. What is the first-line
treatment for this patient?

a. Azithromycin
b. Doxycycline √
c. Clindamycin
d. Clarithromycin
e. Erythromycin

Description

is picture shows erythema migrans which is associated with Lyme disease

Lyme disease is caused by the bacteria Borrelia burgdorferi. However, it is transmitted to humans via
Page - 1440
an infected tick bite (Ixodes).

Clinical diagnosis is important with a history of travel to specific areas

Lyme disease is treated with doxycycline or Amoxicillin for 14 days.

Page - 1441
Internal Medicine - Infectious diseases

Question 24/44

Question #24

A 32-year-old male patient developed a fever, myalgia, and a skin rash (see the picture below!).
About 1 week ago, he went on a hike, and the skin rash was noted 2 days later. What is this skin rash
called?

a. Erythema migrans
b. Erythema marginatum
c. Erythema infectiosum
d. Erythema nodosum
e. Erythema multiform

‫اﻹﺟﺎﺑﺔ ﻋﲆ اﻟﺼﻔﺤﺔ اﻟﺘﺎﻟﻴﺔ‬

Page - 1442
Internal Medicine - Infectious diseases - Lyme Disease

Question 24/44

Question #24

A 32-year-old male patient developed a fever, myalgia, and a skin rash (see the picture below!).
About 1 week ago, he went on a hike, and the skin rash was noted 2 days later. What is this skin rash
called?

a. Erythema migrans √
b. Erythema marginatum
c. Erythema infectiosum
d. Erythema nodosum
e. Erythema multiform

Description

is picture shows erythema migrans which is associated with Lyme disease.

Lyme disease is caused by the bacteria Borrelia burgdorferi. However, it is transmitted to humans via
Page - 1443
an infected tick bite (Ixodes).

Clinical diagnosis is important with a history of travel to specific areas

Lyme disease is treated with doxycycline or Amoxicillin for 14 days.

Page - 1444
Internal Medicine - Infectious diseases

Question 25/44

Question #25

A 32-year-old male patient developed a fever, myalgia, and a skin rash (see the picture below!).
About 1 week ago, he went on a hike, and the skin rash was noted 2 days later. What is the best
treatment for this lady?

a. Azithromycin
b. Clarithromycin
c. Clindamycin
d. Amoxicillin
e. Erythromycin

‫اﻹﺟﺎﺑﺔ ﻋﲆ اﻟﺼﻔﺤﺔ اﻟﺘﺎﻟﻴﺔ‬

Page - 1445
Internal Medicine - Infectious diseases - Lyme Disease

Question 25/44

Question #25

A 32-year-old male patient developed a fever, myalgia, and a skin rash (see the picture below!).
About 1 week ago, he went on a hike, and the skin rash was noted 2 days later. What is the best
treatment for this lady?

a. Azithromycin
b. Clarithromycin
c. Clindamycin
d. Amoxicillin √
e. Erythromycin

Description

is picture shows erythema migrans which is associated with Lyme disease

Lyme disease is caused by the bacteria Borrelia burgdorferi. However, it is transmitted to humans via
Page - 1446
an infected tick bite (Ixodes).

Clinical diagnosis is important with a history of travel to specific areas

Lyme disease is treated with doxycycline or Amoxicillin for 14 days.

Page - 1447
Internal Medicine - Infectious diseases

Question 26/44

Question #26

A 22-year-old businessman who traveled to Africa 2 months ago presents with a headache and cyclic
fever for a 1-week duration. Which of the following diagnosis is most likely?

a. Meningococcal meningitis
b. Tuberculosis
c. Malaria
d. HIV
e. Syphilis

‫اﻹﺟﺎﺑﺔ ﻋﲆ اﻟﺼﻔﺤﺔ اﻟﺘﺎﻟﻴﺔ‬

Page - 1448
Internal Medicine - Infectious diseases - Malaria

Question 26/44

Question #26

A 22-year-old businessman who traveled to Africa 2 months ago presents with a headache and cyclic
fever for a 1-week duration. Which of the following diagnosis is most likely?

a. Meningococcal meningitis
b. Tuberculosis
c. Malaria √
d. HIV
e. Syphilis

Description

History of travel and cyclic fever are suggestive of malaria

Malaria:

A parasitic infection transmitted by Mosquito (Anopheles) aects blood and liver cells
Patients with sickle cell anemia are protected against malaria
e mosquito has the plasmodium in their saliva → and injects it into the human bloodstream →
Sporozoites reach the liver and engage in Asexual reproduction (this is called the extra-
erythrocytic phase) → erythrocytic phase: parasite enters the RBCs → asexual replication and
hemolytic anemia

Page - 1449
Internal Medicine - Infectious diseases

Question 27/44

Question #27

e antimalarial agent Primaquine is indicated in which one of the following?

a. Plasmodium falciparum
b. Plasmodium vivax
c. Plasmodium malariae
d. Plasmodium knowlesi
e. None of the above

‫اﻹﺟﺎﺑﺔ ﻋﲆ اﻟﺼﻔﺤﺔ اﻟﺘﺎﻟﻴﺔ‬

Page - 1450
Internal Medicine - Infectious diseases - Malaria

Question 27/44

Question #27

e antimalarial agent Primaquine is indicated in which one of the following?

a. Plasmodium falciparum
b. Plasmodium vivax √
c. Plasmodium malariae
d. Plasmodium knowlesi
e. None of the above

Description

Primaquine is used in the treatment of Ovale or Vivax malaria (to destroy liver hypnozoite and
prevent relapse)

Malaria:

A parasitic infection transmitted by Mosquito (Anopheles) aects blood and liver cells
Patients with sickle cell anemia are protected against malaria
e mosquito has the plasmodium in their saliva → and injects it into the human bloodstream →
Sporozoites reach the liver and engage in Asexual reproduction (this is called the extra-
erythrocytic phase) → erythrocytic phase: parasite enters the RBCs → asexual replication and
hemolytic anemia

Page - 1451
Internal Medicine - Infectious diseases

Question 28/44

Question #28

A 22-year-old male came aer a dog bite on his leg. Regarding the postexposure immunization
against rabies, it is given in 6 doses as which of the following?

a. At 0, 5, 9, 14, 60, 90 days


b. At 0, 5, 14, 21, 30, 120 days
c. At 0, 3, 7, 14, 30, 90 days
d. At 0, 3, 10, 21, 60,120 days
e. At 0, 3, 30, 60, 90,120 days

‫اﻹﺟﺎﺑﺔ ﻋﲆ اﻟﺼﻔﺤﺔ اﻟﺘﺎﻟﻴﺔ‬

Page - 1452
Internal Medicine - Infectious diseases - Rabies

Question 28/44

Question #28

A 22-year-old male came aer a dog bite on his leg. Regarding the postexposure immunization
against rabies, it is given in 6 doses as which of the following?

a. At 0, 5, 9, 14, 60, 90 days


b. At 0, 5, 14, 21, 30, 120 days
c. At 0, 3, 7, 14, 30, 90 days √
d. At 0, 3, 10, 21, 60,120 days
e. At 0, 3, 30, 60, 90,120 days

Description

e anti-rabies serum is given in 6 doses at 0, 3, 7, 14, 30, and 90 days.

Rabies:

It is a viral infection that can cause brain infection


Rabies is a DNA rhabdovirus
Most transmitted by a dog bite then, the virus travels up at nerve axons to CNS
e incubation period is 1-2 months

Page - 1453
Internal Medicine - Infectious diseases

Question 29/44

Question #29

A 70-year-old male develops septicemia due to gram-negative bacteria secondary to UTI. You started
IV fluid therapy, but the patient is still hypotensive. e blood pressure now is 65/30. What would
you do next?

a. Vasopressin
b. Adrenalin
c. Noradrenalin
d. Dopamine
e. Dobutamine

‫اﻹﺟﺎﺑﺔ ﻋﲆ اﻟﺼﻔﺤﺔ اﻟﺘﺎﻟﻴﺔ‬

Page - 1454
Internal Medicine - Infectious diseases - Sepsis

Question 29/44

Question #29

A 70-year-old male develops septicemia due to gram-negative bacteria secondary to UTI. You started
IV fluid therapy, but the patient is still hypotensive. e blood pressure now is 65/30. What would
you do next?

a. Vasopressin
b. Adrenalin
c. Noradrenalin √
d. Dopamine
e. Dobutamine

Description

If there is no response to fluid in a septic shock patient, start vasopressors.

Noradrenalin here is the best choice

Dobutamine is better for cardiogenic shock

Dopamine can be used, but it is arrhythmogenic

Page - 1455
Internal Medicine - Infectious diseases

Question 30/44

Question #30

A 49-year-old male patient presents with fever and weakness. Aer an appropriate workup, he is
diagnosed with sepsis. Which of the following is expected to be decreased?

a. Ferritin level
b. C-reactive protein
c. Erythrocyte sedimentation rate
d. Serum albumin
e. Fibrin degradation products

‫اﻹﺟﺎﺑﺔ ﻋﲆ اﻟﺼﻔﺤﺔ اﻟﺘﺎﻟﻴﺔ‬

Page - 1456
Internal Medicine - Infectious diseases - Sepsis

Question 30/44

Question #30

A 49-year-old male patient presents with fever and weakness. Aer an appropriate workup, he is
diagnosed with sepsis. Which of the following is expected to be decreased?

a. Ferritin level
b. C-reactive protein
c. Erythrocyte sedimentation rate
d. Serum albumin √
e. Fibrin degradation products

Description

e synthesis of acute-phase proteins by hepatocytes is altered, leading to decreased serum levels of


several proteins, including albumin and transferrin.

Note that ferritin, ESR, and CRP are acute-phase reactants and are expected to be high

D Dimer and other FDPs are expected to be high in sepsis

Page - 1457
Internal Medicine - Infectious diseases

Question 31/44

Question #31

A 78-year-old male patient developed flank pain, dysuria, and diculty voiding. However, 1 day aer
starting treatment, he developed lethargy, fever, hypotension, and tachycardia. What is the most
likely cause of his recent symptoms?

a. Urinary retention
b. Respiratory infection
c. Septic shock
d. Pulmonary edema
e. Pulmonary embolism

‫اﻹﺟﺎﺑﺔ ﻋﲆ اﻟﺼﻔﺤﺔ اﻟﺘﺎﻟﻴﺔ‬

Page - 1458
Internal Medicine - Infectious diseases - Sepsis

Question 31/44

Question #31

A 78-year-old male patient developed flank pain, dysuria, and diculty voiding. However, 1 day aer
starting treatment, he developed lethargy, fever, hypotension, and tachycardia. What is the most
likely cause of his recent symptoms?

a. Urinary retention
b. Respiratory infection
c. Septic shock √
d. Pulmonary edema
e. Pulmonary embolism

Description

e presence of hypotension and evidence of infection is suggestive of septic shock as the most
likely diagnosis

is is a classic presentation of septic shock secondary to UTI

Page - 1459
Internal Medicine - Infectious diseases

Question 32/44

Question #32

A 20-year-old IV drug user presents to you with severe infective arthritis. His temperature is 39°C, his
blood pressure is 85/50 mmHg, and his pulse is 119 bpm. You started IV fluids. What is the best
empirical antibiotic to use in this condition?

a. IV Linezolid and Gentamycin


b. IV Meropenem
c. IV Oxacillin and Clindamycin
d. IV Vancomycin and Ceriaxone
e. PO Vancomycin and IV Ceriaxone

‫اﻹﺟﺎﺑﺔ ﻋﲆ اﻟﺼﻔﺤﺔ اﻟﺘﺎﻟﻴﺔ‬

Page - 1460
Internal Medicine - Infectious diseases - Staphylococcus infection

Question 32/44

Question #32

A 20-year-old IV drug user presents to you with severe infective arthritis. His temperature is 39°C, his
blood pressure is 85/50 mmHg, and his pulse is 119 bpm. You started IV fluids. What is the best
empirical antibiotic to use in this condition?

a. IV Linezolid and Gentamycin


b. IV Meropenem
c. IV Oxacillin and Clindamycin
d. IV Vancomycin and Ceriaxone √
e. PO Vancomycin and IV Ceriaxone

Description

IV Vancomycin to cover MRASA

Oral vancomycin can’t be absorbed by the gut

Linezolid is reserved for MRSA that become resistant to vancomycin

Page - 1461
Internal Medicine - Infectious diseases

Question 33/44

Question #33

A 50-year-old male patient came with a new-onset murmur and fever; his blood culture result
shows gram-positive cocci in clusters. Antibiotic sensitivity is still not known. e most appropriate
antibiotic treatment would be:

a. Amoxicillin
b. Nafcillin
c. Streptomycin and penicillin
d. Ceriaxone
e. Vancomycin and gentamicin

‫اﻹﺟﺎﺑﺔ ﻋﲆ اﻟﺼﻔﺤﺔ اﻟﺘﺎﻟﻴﺔ‬

Page - 1462
Internal Medicine - Infectious diseases - Staphylococcus infection

Question 33/44

Question #33

A 50-year-old male patient came with a new-onset murmur and fever; his blood culture result
shows gram-positive cocci in clusters. Antibiotic sensitivity is still not known. e most appropriate
antibiotic treatment would be:

a. Amoxicillin
b. Nafcillin
c. Streptomycin and penicillin
d. Ceriaxone
e. Vancomycin and gentamicin √

Description

e patient is most likely suering from infective endocarditis. It is most commonly caused by staph
aureus bacteria, so MRSA should be suspected.

Major infections with MRSA should be treated with vancomycin intravenously

Page - 1463
Internal Medicine - Infectious diseases

Question 34/44

Question #34

A sexually active 29-year-old male presents with a painless 1*1 cm ulcer in his penis. It is associated
with painless lymphadenopathy. e rest of the physical examination is normal. What is the best
diagnostic test for this patient?

a. Seminal analysis
b. Urine culture
c. Blood culture
d. Darkfield microscopy
e. Lymph node biopsy

‫اﻹﺟﺎﺑﺔ ﻋﲆ اﻟﺼﻔﺤﺔ اﻟﺘﺎﻟﻴﺔ‬

Page - 1464
Internal Medicine - Infectious diseases - Syphilis

Question 34/44

Question #34

A sexually active 29-year-old male presents with a painless 1*1 cm ulcer in his penis. It is associated
with painless lymphadenopathy. e rest of the physical examination is normal. What is the best
diagnostic test for this patient?

a. Seminal analysis
b. Urine culture
c. Blood culture
d. Darkfield microscopy √
e. Lymph node biopsy

Description

In the case of Syphilis, Darkfield microscopy of the lesion scraping shows the spirochetes with their
characteristic corkscrew appearance.

Syphilis:

It is mainly a sexually transmitted disease


Transmitted by sex, blood transfusion, vertical (transplacental)
Organism: the spirochete Treponema Pallidum, incubation period: between 9-90 days

Page - 1465
Internal Medicine - Infectious diseases

Question 35/44

Question #35

A 25-year-old male presents with a rash involving the chest, abdomen, palms, and soles. He has
painless penile ulceration with indurated base. His inguinal lymph nodes are painlessly enlarged. A
patchy hair loss on his scalp is noted. Darkfield microscopy confirms the diagnosis. What is the most
appropriate treatment?

a. Penicillin
b. Doxycycline
c. Topical steroids
d. Itraconazole
e. Systemic steroids

‫اﻹﺟﺎﺑﺔ ﻋﲆ اﻟﺼﻔﺤﺔ اﻟﺘﺎﻟﻴﺔ‬

Page - 1466
Internal Medicine - Infectious diseases - Syphilis

Question 35/44

Question #35

A 25-year-old male presents with a rash involving the chest, abdomen, palms, and soles. He has
painless penile ulceration with indurated base. His inguinal lymph nodes are painlessly enlarged. A
patchy hair loss on his scalp is noted. Darkfield microscopy confirms the diagnosis. What is the most
appropriate treatment?

a. Penicillin √
b. Doxycycline
c. Topical steroids
d. Itraconazole
e. Systemic steroids

Description

Penicillin is the drug of choice for syphilis.

Syphilis:

It is mainly a sexually transmitted disease


Transmitted by sex, blood transfusion, vertical (transplacental)
Organism: the spirochete Treponema Pallidum, incubation period: between 9-90 days

Page - 1467
Internal Medicine - Infectious diseases

Question 36/44

Question #36

A 15-year-old male patient presents with a cut wound in his forearm during a quarrel. e wound is
contaminated with soil. All the following about tetanus are true except:

a. Trismus is feature
b. Mild tetanus has less than 1% mortality
c. Cephalic tetanus has a short incubation period
d. Temperature is very high
e. Laryngeal spasm may occur, requiring intubation

‫اﻹﺟﺎﺑﺔ ﻋﲆ اﻟﺼﻔﺤﺔ اﻟﺘﺎﻟﻴﺔ‬

Page - 1468
Internal Medicine - Infectious diseases - Tetanus

Question 36/44

Question #36

A 15-year-old male patient presents with a cut wound in his forearm during a quarrel. e wound is
contaminated with soil. All the following about tetanus are true except:

a. Trismus is feature
b. Mild tetanus has less than 1% mortality
c. Cephalic tetanus has a short incubation period
d. Temperature is very high √
e. Laryngeal spasm may occur, requiring intubation

Description

Fever is not a feature of Tetanus infection.

Tetanus:

Clostridium tetani is a gram-positive spore-forming anaerobic bacteria


It is excreted by animal feces to contaminate soil, dust, water, and wounds, including the
umbilical stump. Vaccination site... etc.
Aer contamination, the spores germinate and proliferate locally and produce 2 toxins
(tetanospasmin and tetanolysin) which travel along the nerve trunks and bloodstream to CNS
and redistribute to the spinal cord, brain, and motor system
e Incubation period is 1-14 days, and maybe longer

Page - 1469
Internal Medicine - Infectious diseases

Question 37/44

Question #37

A 24-year-old male patient developed diculty opening his mouth due to tetanus aer wound
contamination 2 weeks ago. What is the cause of death in a patient with established tetanus?

a. Septic shock
b. Heart failure
c. Renal failure
d. Liver failure
e. Respiratory failure

‫اﻹﺟﺎﺑﺔ ﻋﲆ اﻟﺼﻔﺤﺔ اﻟﺘﺎﻟﻴﺔ‬

Page - 1470
Internal Medicine - Infectious diseases - Tetanus

Question 37/44

Question #37

A 24-year-old male patient developed diculty opening his mouth due to tetanus aer wound
contamination 2 weeks ago. What is the cause of death in a patient with established tetanus?

a. Septic shock
b. Heart failure
c. Renal failure
d. Liver failure
e. Respiratory failure √

Description

Respiratory muscle involvement and respiratory failure are the most likely to cause death in tetanus.

Tetanus:

Clostridium tetani is a gram-positive spore-forming anaerobic bacteria


It is excreted by animal feces to contaminate soil, dust, water, and wounds, including the
umbilical stump. Vaccination site, etc.
Aer contamination, the spores germinate and proliferate locally and produce 2 toxins
(tetanospasmin and tetanolysin) which travel along the nerve trunks and bloodstream to CNS
and redistribute to the spinal cord, brain, and motor system
e Incubation period is 1-14 days, and maybe longer

Page - 1471
Internal Medicine - Infectious diseases

Question 38/44

Question #38

A 48-year-old male patient is treated for sepsis with vancomycin and ceriaxone. What would you
do to prevent red man syndrome?

a. Give only half the calculated dose of vancomycin


b. Slow infusion of ceriaxone
c. Provide ceriaxone in the morning and vancomycin in the evening
d. Administrate vancomycin over 90 minutes
e. Give hydrocortisone before vancomycin infusion

‫اﻹﺟﺎﺑﺔ ﻋﲆ اﻟﺼﻔﺤﺔ اﻟﺘﺎﻟﻴﺔ‬

Page - 1472
Internal Medicine - Infectious diseases - e Antibiotics

Question 38/44

Question #38

A 48-year-old male patient is treated for sepsis with vancomycin and ceriaxone. What would you
do to prevent red man syndrome?

a. Give only half the calculated dose of vancomycin


b. Slow infusion of ceriaxone
c. Provide ceriaxone in the morning and vancomycin in the evening
d. Administrate vancomycin over 90 minutes √
e. Give hydrocortisone before vancomycin infusion

Description

Red-man syndrome: a side eect of vancomycin will occur if vancomycin is given as a rapid infusion.
Vancomycin must be given over 90 minutes to avoid this syndrome.

Page - 1473
Internal Medicine - Infectious diseases

Question 39/44

Question #39

A 67-year-old male patient was admitted to the ICU as a case of septic shock. He was started on
empirical antibiotic therapy. e blood culture result is available three days later and shows
Methicillin-Resistant Staph aureus. Of the following, which is the most eective drug to be used?

a. Methicillin
b. Amoxicillin
c. Vancomycin
d. Amoxicillin with clavulanic acid
e. TMP/SMX

‫اﻹﺟﺎﺑﺔ ﻋﲆ اﻟﺼﻔﺤﺔ اﻟﺘﺎﻟﻴﺔ‬

Page - 1474
Internal Medicine - Infectious diseases - e Antibiotics

Question 39/44

Question #39

A 67-year-old male patient was admitted to the ICU as a case of septic shock. He was started on
empirical antibiotic therapy. e blood culture result is available three days later and shows
Methicillin-Resistant Staph aureus. Of the following, which is the most eective drug to be used?

a. Methicillin
b. Amoxicillin
c. Vancomycin √
d. Amoxicillin with clavulanic acid
e. TMP/SMX

Description

MRSA: Methicillin-resistant Staph. aureus:

Minor infection with MRSA treated with: TMP/SMX, doxycycline, clindamycin, or Linezolid

Major infection treated with vancomycin

Page - 1475
Internal Medicine - Infectious diseases

Question 40/44

Question #40

A 67-year-old male patient was admitted to the ICU as a case of septic shock. He was started on
empirical antibiotic therapy. ree days later, the blood culture result is available and shows
pseudomonas. Which of the following drugs is not used to treat pseudomonas?

a. Cefepime
b. Gentamycin
c. Levofloxacin
d. Ertapenem
e. Meropenem

‫اﻹﺟﺎﺑﺔ ﻋﲆ اﻟﺼﻔﺤﺔ اﻟﺘﺎﻟﻴﺔ‬

Page - 1476
Internal Medicine - Infectious diseases - e Antibiotics

Question 40/44

Question #40

A 67-year-old male patient was admitted to the ICU as a case of septic shock. He was started on
empirical antibiotic therapy. ree days later, the blood culture result is available and shows
pseudomonas. Which of the following drugs is not used to treat pseudomonas?

a. Cefepime
b. Gentamycin
c. Levofloxacin
d. Ertapenem √
e. Meropenem

Description

All Carbapenems are eective against pseudomonas, except Ertapenem

Some antibiotics that are active against pseudomonas:

Carbapenems (except Ertapenem) (the first line)


Tigecycline
Cefepime (4th generation cephalosporin)
Aminoglycosides (gentamycin, Amikacin)
Fluoroquinolones (Except Moxifloxacin)

Page - 1477
Internal Medicine - Infectious diseases

Question 41/44

Question #41

e following antibiotics act by inhibiting protein synthesis, except:

a. Penicillin
b. Gentamycin
c. Clarithromycin
d. Tetracycline
e. Neomycin

‫اﻹﺟﺎﺑﺔ ﻋﲆ اﻟﺼﻔﺤﺔ اﻟﺘﺎﻟﻴﺔ‬

Page - 1478
Internal Medicine - Infectious diseases - e Antibiotics

Question 41/44

Question #41

e following antibiotics act by inhibiting protein synthesis, except:

a. Penicillin √
b. Gentamycin
c. Clarithromycin
d. Tetracycline
e. Neomycin

Description

Antibiotics act on dierent target sites in the targeted bacteria.

Penicillin inhibits cell wall formation.

e following table demonstrates the site of action of the most used antibiotics in clinical practice.

Page - 1479
Internal Medicine - Infectious diseases

Question 42/44

Question #42

All the following are bacteriostatic antibiotics except:

a. Azithromycin
b. Chloramphenicol
c. Isoniazid.
d. Trimethoprim
e. Tetracycline

‫اﻹﺟﺎﺑﺔ ﻋﲆ اﻟﺼﻔﺤﺔ اﻟﺘﺎﻟﻴﺔ‬

Page - 1480
Internal Medicine - Infectious diseases - e Antibiotics

Question 42/44

Question #42

All the following are bacteriostatic antibiotics except:

a. Azithromycin
b. Chloramphenicol
c. Isoniazid. √
d. Trimethoprim
e. Tetracycline

Description

Antibiotics are classified into cidal and static types according to their mode of action. Bactericidal
antibiotics kill the bacteria, while bacteriostatic antibiotics suppress the bacterial growth.

Isoniazid is a bactericidal antibiotic.

e following table demonstrates examples of these types.

Page - 1481
Internal Medicine - Infectious diseases

Question 43/44

Question #43

A 23-year-old man presents with 5 days history of fever, headache, and constipation. Which of the
following tests is the best to confirm a diagnosis of typhoid fever?

a. Blood culture
b. Urine culture
c. Stool culture
d. History and Clinical examination
e. CSF culture

‫اﻹﺟﺎﺑﺔ ﻋﲆ اﻟﺼﻔﺤﺔ اﻟﺘﺎﻟﻴﺔ‬

Page - 1482
Internal Medicine - Infectious diseases - Typhoid fever (Enteric fever)

Question 43/44

Question #43

A 23-year-old man presents with 5 days history of fever, headache, and constipation. Which of the
following tests is the best to confirm a diagnosis of typhoid fever?

a. Blood culture √
b. Urine culture
c. Stool culture
d. History and Clinical examination
e. CSF culture

Description

In the case of typhoid fever, blood culture is positive in 40 – 60 % in the first week.

Diagnosis of typhoid fever:

CBC: anemia and leucopenia (due to toxic depression of the bone marrow)
Positive blood culture (40 – 60 % in the 1st week)
In the 2nd week:
Positive stool culture
Positive Widal test (not specific)
In the 3rd week: urine culture
PCR
Bone marrow cells culture (if aected)

Page - 1483
Internal Medicine - Infectious diseases

Question 44/44

Question #44

e following facts are true about typhoid fever except:

a. It is caused by Salmonella typhi


b. It is associated with leucopenia
c. Blood culture is positive in 40 – 60^ in the first week
d. Stool culture is better than urine culture for the diagnosis in the second week
e. Diarrhea is a prominent symptom

‫اﻹﺟﺎﺑﺔ ﻋﲆ اﻟﺼﻔﺤﺔ اﻟﺘﺎﻟﻴﺔ‬

Page - 1484
Internal Medicine - Infectious diseases - Typhoid fever (Enteric fever)

Question 44/44

Question #44

e following facts are true about typhoid fever except:

a. It is caused by Salmonella typhi


b. It is associated with leucopenia
c. Blood culture is positive in 40 – 60^ in the first week
d. Stool culture is better than urine culture for the diagnosis in the second week
e. Diarrhea is a prominent symptom √

Description

Constipation, not diarrhea, is a feature of typhoid fever

Typhoid fever (Enteric fever):

Organism: salmonella typhi and paratyphi (A, B, C), gram-negative bacilli


Humans are only reservoirs
Route: oral-fecal transmission
Bacteria proliferate in Peyer's patches in the small intestine, then cause primary bacteremia
and redistribute to the reticuloendothelial system.

Page - 1485
Nephrology

Page - 1486
Nephrology

‫ﻣﻮاﺿﻴﻊ اﻷﺳﺌﻠﺔ وأﻋﺪادﻫﺎ‬

1) Acid-Base Disorders => 9 Questions


2) Acute Kidney Injury (AKI) => 15 Questions
3) Chronic Kidney disease (CKD) => 6 Questions
4) Kidney function test. => 4 Questions
5) Nephrotic Syndrome => 4 Questions
6) Renal Artery Stenosis => 6 Questions
7) Renal glomerular diseases => 12 Questions
8) Cystic diseases of the kidney => 4 Questions
9) Electrolytes disturbances => 17 Questions
10) Hypertension (HTN) => 19 Questions

Page - 1487
Internal Medicine - Nephrology

Question 1/96

Question #1

Based on limited data from the following ABGs results: Ph 7.30, PaCO2 66 mmHg, PaO2 50 mmHg.
What is the most likely diagnosis?

a. Pulmonary embolism
b. Chronic obstructive pulmonary disease
c. Mild asthma exacerbation
d. Renal impairment
e. Salicylate overdose

‫اﻹﺟﺎﺑﺔ ﻋﲆ اﻟﺼﻔﺤﺔ اﻟﺘﺎﻟﻴﺔ‬

Page - 1488
Internal Medicine - Nephrology - Acid-Base Disorders

Question 1/96

Question #1

Based on limited data from the following ABGs results: Ph 7.30, PaCO2 66 mmHg, PaO2 50 mmHg.
What is the most likely diagnosis?

a. Pulmonary embolism
b. Chronic obstructive pulmonary disease √
c. Mild asthma exacerbation
d. Renal impairment
e. Salicylate overdose

Description

is is respiratory acidosis and hypoxia that, from the above choices, fits with the diagnosis of COPD.

Mild asthma and pulmonary embolism present with respiratory alkalosis

Renal impairment and salicylate poisoning present with high anion gap metabolic acidosis.

Page - 1489
Internal Medicine - Nephrology

Question 2/96

Question #2

A 22-year-old male patient presents with severe persistent vomiting. His arterial blood gas results
are as the following: PH 7.42, HCO3 24mmol/L, PaCO2 40 mmHg, serum sodium 140 mEq/L, serum
potassium 4 mEq/L, serum Cl 93 mEq/L. What is the correct interpretation of arterial blood gas?

a. Normal ABGs result


b. Mixed metabolic acidosis with respiratory acidosis
c. Mixed metabolic acidosis with respiratory alkalosis
d. Mixed metabolic acidosis with metabolic alkalosis
e. Mixed respiratory alkalosis and respiratory acidosis

‫اﻹﺟﺎﺑﺔ ﻋﲆ اﻟﺼﻔﺤﺔ اﻟﺘﺎﻟﻴﺔ‬

Page - 1490
Internal Medicine - Nephrology - Acid-Base Disorders

Question 2/96

Question #2

A 22-year-old male patient presents with severe persistent vomiting. His arterial blood gas results
are as the following: PH 7.42, HCO3 24mmol/L, PaCO2 40 mmHg, serum sodium 140 mEq/L, serum
potassium 4 mEq/L, serum Cl 93 mEq/L. What is the correct interpretation of arterial blood gas?

a. Normal ABGs result


b. Mixed metabolic acidosis with respiratory acidosis
c. Mixed metabolic acidosis with respiratory alkalosis
d. Mixed metabolic acidosis with metabolic alkalosis √
e. Mixed respiratory alkalosis and respiratory acidosis

Description

Apparently, the ABGs here are normal, but here is how to solve this issue.

On calculating anion gap, AG = Na – (Cl + HCO3) = 140 – (93 + 24) = 23.

high plasma anion gap means that there are unmeasured anions indicating high anion gap
metabolic acidosis.

e presence of metabolic alkalosis is considered for two reasons: the first evidence is the presence
of severe recurrent vomiting. e second evidence is the presence of normal PH and HCO3 in the
presence of metabolic acidosis.

Page - 1491
Internal Medicine - Nephrology

Question 3/96

Question #3

A patient with intestinal obstruction develops severe persistent vomiting. What is the most likely
ABGs finding in this case?

a. Ph 7.49, HCO3 30 mmol/L, K 3 mEq/L, Cl 87 mEq/L


b. Ph 7.32, HCO3 12 mmol/L, K 5.7 mEq/L, Cl 90 mEq/L
c. Ph 7.25, HCO3 27 mmol/L, K 2.9 mEq/l, Cl 111 mEq/L
d. Ph 7.44, HCO3 13 mmol/L, K 3.2 mEq/L, Cl 100 mEq/L
e. Ph 7.56, HCO3 15 mmol/L, K 4mEq/L, Cl 92 mEq/L

‫اﻹﺟﺎﺑﺔ ﻋﲆ اﻟﺼﻔﺤﺔ اﻟﺘﺎﻟﻴﺔ‬

Page - 1492
Internal Medicine - Nephrology - Acid-Base Disorders

Question 3/96

Question #3

A patient with intestinal obstruction develops severe persistent vomiting. What is the most likely
ABGs finding in this case?

a. Ph 7.49, HCO3 30 mmol/L, K 3 mEq/L, Cl 87 mEq/L √


b. Ph 7.32, HCO3 12 mmol/L, K 5.7 mEq/L, Cl 90 mEq/L
c. Ph 7.25, HCO3 27 mmol/L, K 2.9 mEq/l, Cl 111 mEq/L
d. Ph 7.44, HCO3 13 mmol/L, K 3.2 mEq/L, Cl 100 mEq/L
e. Ph 7.56, HCO3 15 mmol/L, K 4mEq/L, Cl 92 mEq/L

Description

Vomiting will lead to loss of potassium, chloride, and H ions (Acid)

is will lead to hypokalemia, hypochloremic, metabolic Alkalosis.

Choice A is typical for this finding.

Page - 1493
Internal Medicine - Nephrology

Question 4/96

Question #4

An arterial blood gas results are as the following: PH 7.32, HCO3 15mmol/L, PaCO2 29 mmHg, serum
sodium 135 mEq/L, serum potassium 4 mEq/L, serum Cl 110 mEq/L. Which of the following is the
most likely diagnosis?

a. Diabetic ketoacidosis
b. Renal impairment
c. Starvation
d. Diarrhea
e. Aspirin overdose

‫اﻹﺟﺎﺑﺔ ﻋﲆ اﻟﺼﻔﺤﺔ اﻟﺘﺎﻟﻴﺔ‬

Page - 1494
Internal Medicine - Nephrology - Acid-Base Disorders

Question 4/96

Question #4

An arterial blood gas results are as the following: PH 7.32, HCO3 15mmol/L, PaCO2 29 mmHg, serum
sodium 135 mEq/L, serum potassium 4 mEq/L, serum Cl 110 mEq/L. Which of the following is the
most likely diagnosis?

a. Diabetic ketoacidosis
b. Renal impairment
c. Starvation
d. Diarrhea √
e. Aspirin overdose

Description

e most important causes of non-anion gap metabolic acidosis are diarrhea and renal tubular
acidosis.

e rest of the options cause high anion gap metabolic acidosis

Causes of metabolic acidosis with wide anion gap (MUD PILS):

Methanol overdose
Uremia: renal failure
DKA
Phosphate, paraldehyde, propylene glycol
Ischemia
Lactate: hypotension, hypoperfusion
Starvation, Salicylate overdose

Causes of metabolic acidosis with a normal anion gap (HARDASS):

Hyperalimentation
Addison’s disease
Renal tubular acidosis (RTA)
Diarrhea, ileostomies, fistula.
Acetazolamide
Page - 1495
Spironolactone
Saline infusion

Page - 1496
Internal Medicine - Nephrology

Question 5/96

Question #5

A 72-year-old female sustained a femoral neck fracture 10 days ago. She presents today with
shortness of breath, tachypnea, and tachycardia. In addition, her right calf is swollen, tender, red,
and hot. What is the most likely ABGs finding in this situation?

a. Ph 7.30, PaCO2 15 mmHg, PaO2 100 mmHg


b. Ph 7.32, PaCO2 40 mmHg, PaO2 70mmHg
c. Ph 7.47, PaCO2 20 mmHg, PaO2 140 mmHg
d. Ph 7.30, PaCO2 60 mmHg, PaO2 150 mmHg
e. Ph 7.56, PaCO2 17 mmHg, PaO2 60 mmHg

‫اﻹﺟﺎﺑﺔ ﻋﲆ اﻟﺼﻔﺤﺔ اﻟﺘﺎﻟﻴﺔ‬

Page - 1497
Internal Medicine - Nephrology - Acid-Base Disorders

Question 5/96

Question #5

A 72-year-old female sustained a femoral neck fracture 10 days ago. She presents today with
shortness of breath, tachypnea, and tachycardia. In addition, her right calf is swollen, tender, red,
and hot. What is the most likely ABGs finding in this situation?

a. Ph 7.30, PaCO2 15 mmHg, PaO2 100 mmHg


b. Ph 7.32, PaCO2 40 mmHg, PaO2 70mmHg
c. Ph 7.47, PaCO2 20 mmHg, PaO2 140 mmHg
d. Ph 7.30, PaCO2 60 mmHg, PaO2 150 mmHg
e. Ph 7.56, PaCO2 17 mmHg, PaO2 60 mmHg √

Description

e patient has a high probability of pulmonary embolism, which manifests as respiratory alkalosis
and hypoxia. erefore, choice (e) fits well with the diagnosis.

Page - 1498
Internal Medicine - Nephrology

Question 6/96

Question #6

e following diseases are matched correctly with their ABGs finding except:

a. Severe asthma and respiratory acidosis


b. Mild asthma and respiratory alkalosis
c. DKA and non-anion gap metabolic acidosis
d. Vomiting and metabolic alkalosis
e. Obstructive sleep apnea and respiratory acidosis

‫اﻹﺟﺎﺑﺔ ﻋﲆ اﻟﺼﻔﺤﺔ اﻟﺘﺎﻟﻴﺔ‬

Page - 1499
Internal Medicine - Nephrology - Acid-Base Disorders

Question 6/96

Question #6

e following diseases are matched correctly with their ABGs finding except:

a. Severe asthma and respiratory acidosis


b. Mild asthma and respiratory alkalosis
c. DKA and non-anion gap metabolic acidosis √
d. Vomiting and metabolic alkalosis
e. Obstructive sleep apnea and respiratory acidosis

Description

Diabetic ketoacidosis (DKA) manifests as high anion gap metabolic acidosis.

Causes of metabolic acidosis with wide anion gap (MUD PILS):

Methanol overdose
Uremia: renal failure
DKA
Phosphate, paraldehyde, propylene glycol
Ischemia
Lactate: hypotension, hypoperfusion
Starvation, Salicylate overdose

Causes of metabolic acidosis with a normal anion gap (HARDASS):

Hyperalimentation
Addison’s disease
Renal tubular acidosis (RTA)
Diarrhea, ileostomies, fistula.
Acetazolamide
Spironolactone
Saline infusion

Causes of metabolic alkalosis:

Page - 1500
GI loss of acid: vomiting, NG tube suction
Increased aldosterone: Conn’s, Cushing’s
Diuretics and Hypokalemia
Milk-alkali syndrome: high-volume liquid antacids

Causes of respiratory alkalosis:

Anemia
Anxiety and Pain
Fever
Interstitial lung disease
Pulmonary embolism
Mild Asthma

Causes of respiratory acidosis:

COPD
Drowning
Opiate overdose
Kyphoscoliosis
Sleep apnea
Life-threatening Asthma

Page - 1501
Internal Medicine - Nephrology

Question 7/96

Question #7

A 29-year-old man with type 1 diabetes presented to the emergency room complaining of abdominal
pain and vomiting for several hours. Has arterial blood gas been PH 7.35, PaCO2 23 mmHg, HCO3 10
mEq/L. what is the acid base condition he has?

a. Compensated metabolic acidosis


b. Uncompensated metabolic acidosis
c. Compensated metabolic alkalosis
d. Uncompensated respiratory acidosis
e. Compensated respiratory alkalosis

‫اﻹﺟﺎﺑﺔ ﻋﲆ اﻟﺼﻔﺤﺔ اﻟﺘﺎﻟﻴﺔ‬

Page - 1502
Internal Medicine - Nephrology - Acid-Base Disorders

Question 7/96

Question #7

A 29-year-old man with type 1 diabetes presented to the emergency room complaining of abdominal
pain and vomiting for several hours. Has arterial blood gas been PH 7.35, PaCO2 23 mmHg, HCO3 10
mEq/L. what is the acid base condition he has?

a. Compensated metabolic acidosis √


b. Uncompensated metabolic acidosis
c. Compensated metabolic alkalosis
d. Uncompensated respiratory acidosis
e. Compensated respiratory alkalosis

Description

is patient is most likely complaining of DKA.

Regardless of his condition, the question is focusing on the ABGs interpretation.

Low HCO3 here is the primary abnormality making the answer is metabolic acidosis.

e PH in compensated cases will return to near normal results and in this case it is 7.35 which
makes is compensated metabolic acidosis.

e expected PaCO2 here is (1.5*10 + 8) = 23.

Page - 1503
Internal Medicine - Nephrology

Question 8/96

Question #8

A 75-year-old male patient with shortness of breath and general weakness was investigated at the
ER. His ABGs results are pH 7.16, HCO3 16 mmol/L, PaCO2 32 mmHg, PO2 90 mmHg, serum urea 90
mg/dL, creatinine 4 mg/dL. What is the most likely interpretation?

a. Metabolic acidosis
b. Respiratory acidosis
c. Metabolic acidosis and respiratory acidosis
d. Normal ABGs
e. No adequate information

‫اﻹﺟﺎﺑﺔ ﻋﲆ اﻟﺼﻔﺤﺔ اﻟﺘﺎﻟﻴﺔ‬

Page - 1504
Internal Medicine - Nephrology - Acid-Base Disorders

Question 8/96

Question #8

A 75-year-old male patient with shortness of breath and general weakness was investigated at the
ER. His ABGs results are pH 7.16, HCO3 16 mmol/L, PaCO2 32 mmHg, PO2 90 mmHg, serum urea 90
mg/dL, creatinine 4 mg/dL. What is the most likely interpretation?

a. Metabolic acidosis √
b. Respiratory acidosis
c. Metabolic acidosis and respiratory acidosis
d. Normal ABGs
e. No adequate information

Description

is is a case of metabolic acidosis

PH shows acidosis

e PaCO2 (same direction) – metabolic

e estimated PaCO2 = the actual PaCO2

Page - 1505
Internal Medicine - Nephrology

Question 9/96

Question #9

A lady develops a severe anxiety attack aer a stressful event. What is the most likely ABGs finding
in this scenario?

a. Ph 7.30, PaCO2 15 mmHg, PaO2 100 mmHg


b. Ph 7.32, PaCO2 40 mmHg, PaO2 70mmHg
c. Ph 7.47, PaCO2 20 mmHg, PaO2 140 mmHg
d. Ph 7.30, PaCO2 60 mmHg, PaO2 150 mmHg
e. Ph 7.56, PaCO2 17 mmHg, PaO2 60 mmHg

‫اﻹﺟﺎﺑﺔ ﻋﲆ اﻟﺼﻔﺤﺔ اﻟﺘﺎﻟﻴﺔ‬

Page - 1506
Internal Medicine - Nephrology - Acid-Base Disorders

Question 9/96

Question #9

A lady develops a severe anxiety attack aer a stressful event. What is the most likely ABGs finding
in this scenario?

a. Ph 7.30, PaCO2 15 mmHg, PaO2 100 mmHg


b. Ph 7.32, PaCO2 40 mmHg, PaO2 70mmHg
c. Ph 7.47, PaCO2 20 mmHg, PaO2 140 mmHg √
d. Ph 7.30, PaCO2 60 mmHg, PaO2 150 mmHg
e. Ph 7.56, PaCO2 17 mmHg, PaO2 60 mmHg

Description

is patient has hyperventilation syndrome, resulting in respiratory alkalosis without hypoxia or
even high PaO2. erefore, choice (c) fits with the diagnosis.

Page - 1507
Internal Medicine - Nephrology

Question 10/96

Question #10

You are reviewing a patient with acute kidney injury. His lab results show the following: elevated
urea and creatinine, serum sodium of 150 mEq/L, serum potassium of 5.1 mEq/L, FENa is 0.70%,
specific gravity is 1030, and urine osmolality is 850 mOsm/kg. What is the most likely cause of AKI in
this patient?

a. Drug-induced
b. Rhabdomyolysis
c. Hemolytic anemia
d. Post-renal obstruction
e. Renal hypoperfusion

‫اﻹﺟﺎﺑﺔ ﻋﲆ اﻟﺼﻔﺤﺔ اﻟﺘﺎﻟﻴﺔ‬

Page - 1508
Internal Medicine - Nephrology - Acute Kidney Injury (AKI)

Question 10/96

Question #10

You are reviewing a patient with acute kidney injury. His lab results show the following: elevated
urea and creatinine, serum sodium of 150 mEq/L, serum potassium of 5.1 mEq/L, FENa is 0.70%,
specific gravity is 1030, and urine osmolality is 850 mOsm/kg. What is the most likely cause of AKI in
this patient?

a. Drug-induced
b. Rhabdomyolysis
c. Hemolytic anemia
d. Post-renal obstruction
e. Renal hypoperfusion √

Description

e presence of low FENa (<1%), high urine osmolality, and specific gravity strongly suggest pre-renal
etiology of the acute kidney injury.

e following table dierentiates pre-renal vs. acute tubular necrosis according to urine chemistry.

Page - 1509
Internal Medicine - Nephrology

Question 11/96

Question #11

An epileptic 35-year-old male patient was admitted to the hospital for generalized weakness,
muscle pains, and red urine. However, the day before admission he had an episode of convulsions.
Unfortunately, he developed elevated kidney function. What is the most likely finding in his urine
chemistry in this situation?

a. Urine osmolality of 65m mOsm/L


b. Specific gravity of 1035
c. Urine sodium of 15 mEq/L
d. Fractional sodium excretion of 4%
e. Fractional urea excretion of 25%

‫اﻹﺟﺎﺑﺔ ﻋﲆ اﻟﺼﻔﺤﺔ اﻟﺘﺎﻟﻴﺔ‬

Page - 1510
Internal Medicine - Nephrology - Acute Kidney Injury (AKI)

Question 11/96

Question #11

An epileptic 35-year-old male patient was admitted to the hospital for generalized weakness,
muscle pains, and red urine. However, the day before admission he had an episode of convulsions.
Unfortunately, he developed elevated kidney function. What is the most likely finding in his urine
chemistry in this situation?

a. Urine osmolality of 65m mOsm/L


b. Specific gravity of 1035
c. Urine sodium of 15 mEq/L
d. Fractional sodium excretion of 4% √
e. Fractional urea excretion of 25%

Description

is is a typical scenario of rhabdomyolysis complicated by acute tubular necrosis (ATN).

In ATN the urinary osmolality and specific gravity will be reduced and equal to that of the patient’s
plasma (isosthenuria)

In ATN, the urinary sodium and the fractional excretion of sodium and the fractional excretion of
sodium and urea will be high due to loss of reabsorption function of the necrotic tubular cells.

e following table demonstrates the dierence between prerenal acute kidney injury and Acute
tubular necrosis.

Page - 1511
Internal Medicine - Nephrology

Question 12/96

Question #12

A 30-year-old male patient presents to you with macular skin rash, arthralgia, and fever. He thinks
his condition started 6 days ago with a streptococcal throat infection confirmed by culture and
treated with penicillin. His lab tests show deterioration of renal function, eosinophilia, proteinuria,
and eosinophiluria. However, his urinary protein excretion is 700 mg/day, and no RBC casts are in his
urine. What is the most likely diagnosis?

a. Post-streptococcal glomerulonephritis
b. IgA nephropathy
c. Membranous glomerulonephritis
d. Drug-induced interstitial nephritis
e. Polyarteritis nodosa

‫اﻹﺟﺎﺑﺔ ﻋﲆ اﻟﺼﻔﺤﺔ اﻟﺘﺎﻟﻴﺔ‬

Page - 1512
Internal Medicine - Nephrology - Acute Kidney Injury (AKI)

Question 12/96

Question #12

A 30-year-old male patient presents to you with macular skin rash, arthralgia, and fever. He thinks
his condition started 6 days ago with a streptococcal throat infection confirmed by culture and
treated with penicillin. His lab tests show deterioration of renal function, eosinophilia, proteinuria,
and eosinophiluria. However, his urinary protein excretion is 700 mg/day, and no RBC casts are in his
urine. What is the most likely diagnosis?

a. Post-streptococcal glomerulonephritis
b. IgA nephropathy
c. Membranous glomerulonephritis
d. Drug-induced interstitial nephritis √
e. Polyarteritis nodosa

Description

Penicillin is one of the causes of acute interstitial nephritis (AIN).

e presence of arthralgia, skin rash, fever, renal impairment, eosinophilia, and eosinophiluria are
typical of AIN

70% of AIN cases are caused by hypersensitivity to a medication

Causes of AIN:

Drugs (70%) – (Penicillin, PPI’s, Phenytoin, Cephalosporin, Sulfa drugs, Rifampicin, NSAIDs,
Furosemide, Quinolones, streptomycin, Allopurinol).
Systemic diseases (SLE, sarcoidosis, multiple myeloma, Sjögren’s syndrome)
Infections (streptococci, TB, CMV)

Page - 1513
Internal Medicine - Nephrology

Question 13/96

Question #13

A 44-year-old male patient came with recurrent severe diarrhea and dehydration. His lab
investigations demonstrate serum creatinine of 3.0 mg/dL, urine sodium of 45 mEq/L, urine
osmolality of 270 mOsm/L, and fractional sodium excretion of 3%. What is the most likely
diagnosis?

a. Prerenal acute kidney injury


b. Acute tubular necrosis
c. Acute interstitial nephritis
d. IgA nephropathy
e. Membranous glomerulopathy

‫اﻹﺟﺎﺑﺔ ﻋﲆ اﻟﺼﻔﺤﺔ اﻟﺘﺎﻟﻴﺔ‬

Page - 1514
Internal Medicine - Nephrology - Acute Kidney Injury (AKI)

Question 13/96

Question #13

A 44-year-old male patient came with recurrent severe diarrhea and dehydration. His lab
investigations demonstrate serum creatinine of 3.0 mg/dL, urine sodium of 45 mEq/L, urine
osmolality of 270 mOsm/L, and fractional sodium excretion of 3%. What is the most likely
diagnosis?

a. Prerenal acute kidney injury


b. Acute tubular necrosis √
c. Acute interstitial nephritis
d. IgA nephropathy
e. Membranous glomerulopathy

Description

For the first impression, the history of recurrent diarrhea and dehydration may lead to the diagnosis
of prerenal acute kidney injury, but the urine chemistry do not fit with the diagnosis.

e presence of untreated prerenal AKI may lead to intra-renal acute tubular necrosis (ischemic
type).

e following table demonstrates the dierence between prerenal acute kidney injury and Acute
tubular necrosis:

Page - 1515
Internal Medicine - Nephrology

Question 14/96

Question #14

An athletic female develops muscle aches and weakness aer a day of vigorous training. Her urine is
positive for myoglobin, and her creatinine kinase level is significantly elevated. In addition, her
kidney function is deteriorating. What is the most likely cause of elevated creatinine kinase in this
patient?

a. Renal impairment
b. Renal tubular cell damage
c. Muscle destruction
d. Urinary tract infection
e. Myocardial infarction

‫اﻹﺟﺎﺑﺔ ﻋﲆ اﻟﺼﻔﺤﺔ اﻟﺘﺎﻟﻴﺔ‬

Page - 1516
Internal Medicine - Nephrology - Acute Kidney Injury (AKI)

Question 14/96

Question #14

An athletic female develops muscle aches and weakness aer a day of vigorous training. Her urine is
positive for myoglobin, and her creatinine kinase level is significantly elevated. In addition, her
kidney function is deteriorating. What is the most likely cause of elevated creatinine kinase in this
patient?

a. Renal impairment
b. Renal tubular cell damage
c. Muscle destruction √
d. Urinary tract infection
e. Myocardial infarction

Description

Rhabdomyolysis occurs due to severe destruction or necrosis of the muscles, which leads to a release
of a large amount of myoglobin which has a toxic eect on renal tubules

Vigorous exercise in this patient is a precipitant.

In a patient with known precipitant, renal impairment and myoglobinuria indicate the disease.

e muscle cells contain a significant amount of creatinine kinase (CK); therefore, CK is elevated
because the muscle destruction

Page - 1517
Internal Medicine - Nephrology

Question 15/96

Question #15

Aer a day of vigorous training, a tennis player develops weakness, muscle pain, and red urine. His
serum creatinine kinase was significantly elevated, his creatinine level was 1.9mg/dL, and his serum
potassium was 6.9 mEq/L. His ECG shows wide QRS, peaked T wave, and small P. What is the most
important initial management in this patient?

a. Intravenous normal saline


b. Intravenous insulin and glucose
c. Intravenous calcium gluconate
d. Hemodialysis
e. Pain relief with diclofenac sodium

‫اﻹﺟﺎﺑﺔ ﻋﲆ اﻟﺼﻔﺤﺔ اﻟﺘﺎﻟﻴﺔ‬

Page - 1518
Internal Medicine - Nephrology - Acute Kidney Injury (AKI)

Question 15/96

Question #15

Aer a day of vigorous training, a tennis player develops weakness, muscle pain, and red urine. His
serum creatinine kinase was significantly elevated, his creatinine level was 1.9mg/dL, and his serum
potassium was 6.9 mEq/L. His ECG shows wide QRS, peaked T wave, and small P. What is the most
important initial management in this patient?

a. Intravenous normal saline


b. Intravenous insulin and glucose
c. Intravenous calcium gluconate √
d. Hemodialysis
e. Pain relief with diclofenac sodium

Description

Rhabdomyolysis occurs due to severe destruction or necrosis of the muscles, which leads to a release
of a large amount of myoglobin which has a toxic eect on renal tubules

Vigorous exercise in this patient is a precipitant.

In a patient with known precipitant, renal impairment and myoglobinuria indicate the disease.

ECG is the most urgent step to do (to rule out arrhythmia secondary to hyperkalemia).

is patient has confirmed hyperkalemia with ECG changes that may lead to arrhythmia and death.
So, the patient should receive calcium gluconate as soon as possible to stabilize the cell membrane
and reduce the risk of arrhythmia.

Intravenous normal saline is indicated to reduce the contact time of myoglobin with the renal
tubular cells to reduce nephrotoxicity.

Hemodialysis is indicated only in hyperkalemia ≥ 7, which is refractory to treatment.

Insulin is the most eective step to reduce serum potassium but calcium gluconate, to prevent
arrhythmia, is a more important step.

Diclofenac sodium should be avoided because it exacerbates nephrotoxicity and worsens this
condition.
Page - 1519
Page - 1520
Internal Medicine - Nephrology

Question 16/96

Question #16

A diabetic 66-year-old female is suspected of pulmonary embolism, and you would like to order CT
pulmonary angiography. What is the most appropriate to prevent contrast-induced nephropathy in
this patient?

a. Diabetic control with metformin and insulin before imaging


b. Administrate furosemide intravenously
c. intravenous steroids
d. Intravenous 0.9% saline
e. Intravenous mannitol

‫اﻹﺟﺎﺑﺔ ﻋﲆ اﻟﺼﻔﺤﺔ اﻟﺘﺎﻟﻴﺔ‬

Page - 1521
Internal Medicine - Nephrology - Acute Kidney Injury (AKI)

Question 16/96

Question #16

A diabetic 66-year-old female is suspected of pulmonary embolism, and you would like to order CT
pulmonary angiography. What is the most appropriate to prevent contrast-induced nephropathy in
this patient?

a. Diabetic control with metformin and insulin before imaging


b. Administrate furosemide intravenously
c. intravenous steroids
d. Intravenous 0.9% saline √
e. Intravenous mannitol

Description

Contrast-induced nephropathy is preventable by good hydration because the condition is more


likely to develop in dehydrated patients.

e best practice is to administer normal saline 0.9% 12 before the contrast administration and
continue for 24 hours at a rate of 1ml/kg/hour.

N-Acetylcysteine is another option used in medical practice.

Page - 1522
Internal Medicine - Nephrology

Question 17/96

Question #17

Which of the following drugs should be held before intravenous contrast media administration?

a. Metformin
b. Paracetamol
c. Insulin
d. Glyburide
e. Pioglitazone

‫اﻹﺟﺎﺑﺔ ﻋﲆ اﻟﺼﻔﺤﺔ اﻟﺘﺎﻟﻴﺔ‬

Page - 1523
Internal Medicine - Nephrology - Acute Kidney Injury (AKI)

Question 17/96

Question #17

Which of the following drugs should be held before intravenous contrast media administration?

a. Metformin √
b. Paracetamol
c. Insulin
d. Glyburide
e. Pioglitazone

Description

e elimination of metformin primarily occurs in the kidneys.

It should be held before intravenous contrast media administration because of the risk of
exacerbation of renal impairment and lactic acidosis.

Page - 1524
Internal Medicine - Nephrology

Question 18/96

Question #18

A 60-year-old male patient was admitted with a case of acute kidney injury. His renal biopsy
confirms the presence of acute interstitial nephritis (AIN). What is the most common cause of this
condition?

a. Diabetes mellitus
b. Hypertension
c. Renal hypoperfusion
d. Hypersensitivity to medication
e. Infectious process

‫اﻹﺟﺎﺑﺔ ﻋﲆ اﻟﺼﻔﺤﺔ اﻟﺘﺎﻟﻴﺔ‬

Page - 1525
Internal Medicine - Nephrology - Acute Kidney Injury (AKI)

Question 18/96

Question #18

A 60-year-old male patient was admitted with a case of acute kidney injury. His renal biopsy
confirms the presence of acute interstitial nephritis (AIN). What is the most common cause of this
condition?

a. Diabetes mellitus
b. Hypertension
c. Renal hypoperfusion
d. Hypersensitivity to medication √
e. Infectious process

Description

Although infection and systemic disorders like SLE, sarcoidosis, and Sjögren’s syndrome can cause
acute interstitial nephritis, 70% of the cases result from hypersensitivity to drugs.

Causes of AIN:

Drugs (70%) – (Penicillin, PPI’s, Phenytoin, Cephalosporin, Sulfa drugs, Rifampicin, NSAIDs,
Furosemide, Quinolones, streptomycin, Allopurinol).
Systemic diseases (SLE, sarcoidosis, multiple myeloma, Sjögren’s syndrome)
Infections (streptococci, TB, CMV)

Page - 1526
Internal Medicine - Nephrology

Question 19/96

Question #19

A previously healthy 32-year-old lady developed an antepartum hemorrhage 3 days ago, requiring a
massive blood transfusion. Today, her serum creatinine is 4.1 mg/dL, and her serum potassium is 5
mEq/L. What is the most likely diagnosis?

a. Rhabdomyolysis
b. Acute tubular necrosis
c. Acute interstitial nephritis
d. Addison crisis
e. Sheehan syndrome

‫اﻹﺟﺎﺑﺔ ﻋﲆ اﻟﺼﻔﺤﺔ اﻟﺘﺎﻟﻴﺔ‬

Page - 1527
Internal Medicine - Nephrology - Acute Kidney Injury (AKI)

Question 19/96

Question #19

A previously healthy 32-year-old lady developed an antepartum hemorrhage 3 days ago, requiring a
massive blood transfusion. Today, her serum creatinine is 4.1 mg/dL, and her serum potassium is 5
mEq/L. What is the most likely diagnosis?

a. Rhabdomyolysis
b. Acute tubular necrosis √
c. Acute interstitial nephritis
d. Addison crisis
e. Sheehan syndrome

Description

e elevated creatinine in a previously healthy patient is a characteristic of acute renal failure

is patient had massive bleeding and renal hypoperfusion, which later resulted in ischemic acute
tubular necrosis (ATN)

Page - 1528
Internal Medicine - Nephrology

Question 20/96

Question #20

An alcoholic 22-year-old male patient was found unconscious on the floor and brought to the
emergency department. He was very confused and disoriented; his blood pressure was 110/70, and
his pulse was 90bpm. His lab investigations demonstrate a creatinine level of 10 mg/dL, serum
potassium of 6 mEq/L, and the urine dipstick shows hematuria. What is the best investigation to
perform next?

a. Complete blood count


b. Creatinine kinase level
c. Renal ultrasound
d. Renal biopsy
e. Serum amylase level

‫اﻹﺟﺎﺑﺔ ﻋﲆ اﻟﺼﻔﺤﺔ اﻟﺘﺎﻟﻴﺔ‬

Page - 1529
Internal Medicine - Nephrology - Acute Kidney Injury (AKI)

Question 20/96

Question #20

An alcoholic 22-year-old male patient was found unconscious on the floor and brought to the
emergency department. He was very confused and disoriented; his blood pressure was 110/70, and
his pulse was 90bpm. His lab investigations demonstrate a creatinine level of 10 mg/dL, serum
potassium of 6 mEq/L, and the urine dipstick shows hematuria. What is the best investigation to
perform next?

a. Complete blood count


b. Creatinine kinase level √
c. Renal ultrasound
d. Renal biopsy
e. Serum amylase level

Description

Rhabdomyolysis occurs due to severe destruction or necrosis of the muscles, which leads to a release
of a large amount of myoglobin which has a toxic eect on renal tubules

Prolonged immobility in this patient is a precipitant.

In a patient with known precipitant, renal impairment and myoglobinuria indicate the disease.

Note that myoglobin has heme that results in a positive dipstick for RBCs, but microscopy will fail to
show any RBCs in the urine in the case of myoglobinuria

In the case of rhabdomyolysis, creatinine kinase level will be elevated to more than three times the
normal upper limit.

Page - 1530
Internal Medicine - Nephrology

Question 21/96

Question #21

A 30-year-old male patient was rescued aer being trapped under heavy rocks for 12 hours aer a
building had fallen in an earthquake. His urine dipstick shows red urine with numerous RBCs, but
urine microscopy shows no erythrocytes. In addition, his creatinine is 1.5 mg/dL, but he has normal
serum electrolyte levels. What is the most likely cause of his elevated creatinine?

a. Pre-renal acute renal failure


b. Rhabdomyolysis
c. Postrenal obstruction
d. Direct renal injury
e. Normal kidney function

‫اﻹﺟﺎﺑﺔ ﻋﲆ اﻟﺼﻔﺤﺔ اﻟﺘﺎﻟﻴﺔ‬

Page - 1531
Internal Medicine - Nephrology - Acute Kidney Injury (AKI)

Question 21/96

Question #21

A 30-year-old male patient was rescued aer being trapped under heavy rocks for 12 hours aer a
building had fallen in an earthquake. His urine dipstick shows red urine with numerous RBCs, but
urine microscopy shows no erythrocytes. In addition, his creatinine is 1.5 mg/dL, but he has normal
serum electrolyte levels. What is the most likely cause of his elevated creatinine?

a. Pre-renal acute renal failure


b. Rhabdomyolysis √
c. Postrenal obstruction
d. Direct renal injury
e. Normal kidney function

Description

In crush injury, reperfusion injury may occur to any limb aer being released leading to more
damage and rhabdomyolysis.

Myoglobin released from the damaged muscles is spelled into the circulation. Note that myoglobin
has heme that results in a positive dipstick for RBCs, but microscopy will fail to show any RBCs in the
urine in the case of myoglobinuria.

Myoglobin is nephrotoxic and can lead to acute tubular necrosis, renal impairment, electrolyte
disturbances, and maybe death.

Page - 1532
Internal Medicine - Nephrology

Question 22/96

Question #22

A 39-year-old female had gastric sleeve surgery 1 week ago. She has recurrent vomiting, which
occurs almost aer eating or drinking anything. Today she presents to you with dizziness when
standing and general fatigue. Her physical examination is significant for moderate dehydration,
blood pressure of 90/49 mmHg, and pulse of 105 bpm. Her lab results show serum creatinine of 2.3
mg/dL, urea of 90 mg/dL, and normal electrolytes. What is the most appropriate initial therapy at
this time?

a. Intravenous furosemide
b. Blood transfusion
c. Start dopamine (renal dose)
d. Intravenous fluid
e. Hemodialysis

‫اﻹﺟﺎﺑﺔ ﻋﲆ اﻟﺼﻔﺤﺔ اﻟﺘﺎﻟﻴﺔ‬

Page - 1533
Internal Medicine - Nephrology - Acute Kidney Injury (AKI)

Question 22/96

Question #22

A 39-year-old female had gastric sleeve surgery 1 week ago. She has recurrent vomiting, which
occurs almost aer eating or drinking anything. Today she presents to you with dizziness when
standing and general fatigue. Her physical examination is significant for moderate dehydration,
blood pressure of 90/49 mmHg, and pulse of 105 bpm. Her lab results show serum creatinine of 2.3
mg/dL, urea of 90 mg/dL, and normal electrolytes. What is the most appropriate initial therapy at
this time?

a. Intravenous furosemide
b. Blood transfusion
c. Start dopamine (renal dose)
d. Intravenous fluid √
e. Hemodialysis

Description

is is a clear condition of pre-renal acute kidney injury (AKI)

e presence of vomiting, dehydration, and hypotension followed by renal failure is a typical


scenario of AKI.

e priority is to restore circulation and renal perfusion by intravenous fluid.

e patient is not a candidate for hemodialysis at this time (no indication)

Blood transfusion is indicated when the cause is bleeding (hypovolemic shock), which is not the
scenario in this case.

Page - 1534
Internal Medicine - Nephrology

Question 23/96

Question #23

A 55-year-old diabetic male patient presents with typical chest pain, ST elevation on ECG, and
positive cardiac enzymes. You transferred him to the cardiac unit for PCI. What is the most
appropriate measure to prevent renal impairment secondary to contrast use in PCI?

a. Diabetic control with metformin and insulin before imaging


b. Administrate furosemide intravenously
c. intravenous steroids
d. Intravenous 0.9% saline
e. N-acetylcysteine

‫اﻹﺟﺎﺑﺔ ﻋﲆ اﻟﺼﻔﺤﺔ اﻟﺘﺎﻟﻴﺔ‬

Page - 1535
Internal Medicine - Nephrology - Acute Kidney Injury (AKI)

Question 23/96

Question #23

A 55-year-old diabetic male patient presents with typical chest pain, ST elevation on ECG, and
positive cardiac enzymes. You transferred him to the cardiac unit for PCI. What is the most
appropriate measure to prevent renal impairment secondary to contrast use in PCI?

a. Diabetic control with metformin and insulin before imaging


b. Administrate furosemide intravenously
c. intravenous steroids
d. Intravenous 0.9% saline √
e. N-acetylcysteine

Description

Contrast-induced nephropathy is preventable by good hydration because the condition is more


likely to develop in dehydrated patients.

e best practice is to administer normal saline 0.9% 12 before the contrast administration and
continue for 24 hours at a rate of 1ml/kg/hour.

N-Acetylcysteine is another option used in medical practice.

Page - 1536
Internal Medicine - Nephrology

Question 24/96

Question #24

A diabetic 66-year-old female is suspected of having a pulmonary embolism, and you would like to
order CT pulmonary angiography. What is the most appropriate to prevent contrast-induced
nephropathy in this patient?

a. Reduce the dose of contrast


b. Administrate furosemide intravenously
c. ACE inhibitors
d. Plenty of fluid
e. Intravenous dextrose

‫اﻹﺟﺎﺑﺔ ﻋﲆ اﻟﺼﻔﺤﺔ اﻟﺘﺎﻟﻴﺔ‬

Page - 1537
Internal Medicine - Nephrology - Acute Kidney Injury (AKI)

Question 24/96

Question #24

A diabetic 66-year-old female is suspected of having a pulmonary embolism, and you would like to
order CT pulmonary angiography. What is the most appropriate to prevent contrast-induced
nephropathy in this patient?

a. Reduce the dose of contrast


b. Administrate furosemide intravenously
c. ACE inhibitors
d. Plenty of fluid √
e. Intravenous dextrose

Description

Contrast-induced nephropathy is preventable by good hydration because the condition is more


likely to develop in dehydrated patients.

e best practice is to administer normal saline 0.9% 12 before the contrast administration and
continue for 24 hours at a rate of 1ml/kg/hour.

N-Acetylcysteine is another option used in medical practice.

Page - 1538
Internal Medicine - Nephrology

Question 25/96

Question #25

During the routine annual screening of a diabetic 55-year-old male patient, you found an
albumin:creatinine ratio of 3.2. However, his blood pressure is 115/65. What is the most appropriate
treatment?

a. Start oral enalapril


b. Tight glycemic control only
c. Start oral nifedipine
d. Start oral carvedilol
e. Reassurance and follow-up aer 1 year

‫اﻹﺟﺎﺑﺔ ﻋﲆ اﻟﺼﻔﺤﺔ اﻟﺘﺎﻟﻴﺔ‬

Page - 1539
Internal Medicine - Nephrology - Chronic Kidney disease (CKD)

Question 25/96

Question #25

During the routine annual screening of a diabetic 55-year-old male patient, you found an
albumin:creatinine ratio of 3.2. However, his blood pressure is 115/65. What is the most appropriate
treatment?

a. Start oral enalapril √


b. Tight glycemic control only
c. Start oral nifedipine
d. Start oral carvedilol
e. Reassurance and follow-up aer 1 year

Description

Persistent albuminuria of 30 – 299 mg/day (previously called microalbuminuria) is an early sign of


diabetic nephropathy

Albumin:creatinine ratio of more than 2.5 indicates microalbuminuria.

Screening: by albumin creatinine ratio (ACR):

It should be done annually for all patients


It should be done by the early morning specimen
ACR > 2.5 is considered microalbuminuria

Management:

Diabetic control is the most important step in the management


Strict blood pressure control (< 140/90)
ACE inhibitors or ARBs are the treatments of choice to reduce the progression of the disease
and to reduce the level of microalbuminuria
e strict control of dyslipidemia

Page - 1540
Internal Medicine - Nephrology

Question 26/96

Question #26

A 55-year-old male patient with Type 2 DM presents for a routine checkup. Which of the following is
the best test to screen for diabetic nephropathy?

a. Urinalysis for proteinuria


b. Renal ultrasound
c. Creatinine and glomerular filtration rate (GFR)
d. Urinary albumin:creatinine ratio
e. HbA1C level

‫اﻹﺟﺎﺑﺔ ﻋﲆ اﻟﺼﻔﺤﺔ اﻟﺘﺎﻟﻴﺔ‬

Page - 1541
Internal Medicine - Nephrology - Chronic Kidney disease (CKD)

Question 26/96

Question #26

A 55-year-old male patient with Type 2 DM presents for a routine checkup. Which of the following is
the best test to screen for diabetic nephropathy?

a. Urinalysis for proteinuria


b. Renal ultrasound
c. Creatinine and glomerular filtration rate (GFR)
d. Urinary albumin:creatinine ratio √
e. HbA1C level

Description

Persistent albuminuria of 30 – 299 mg/day (previously called microalbuminuria) is an early sign of


diabetic nephropathy

Albumin:creatinine ratio of more than 2.5 indicates microalbuminuria.

Screening: by albumin creatinine ratio (ACR):

It should be done annually for all patients


It should be done by the early morning specimen
ACR > 2.5 is considered microalbuminuria

Note that GFR will be elevated in the early stages of diabetic nephropathy (hyperfiltration), so it is
not a good predictor in this case

Urinalysis can’t detect microalbuminuria and is not used for the screening of this patient

Renal ultrasound may show enlarged kidneys, but this is not specific and has no value in screening
DM nephropathy.

Page - 1542
Internal Medicine - Nephrology

Question 27/96

Question #27

About diabetic nephropathy, all the following are true except:

a. e rate of progression of renal impairment can be reduced with the use of ACE inhibitors or ARBs
b. Microalbuminuria is defined as persistent proteinuria of 30 – 299 mg/day
c. Hyperfiltration is the earliest sign of diabetic nephropathy
d. e disease primarily aects the glomeruli
e. Annual screening for microalbuminuria is done by urinalysis

‫اﻹﺟﺎﺑﺔ ﻋﲆ اﻟﺼﻔﺤﺔ اﻟﺘﺎﻟﻴﺔ‬

Page - 1543
Internal Medicine - Nephrology - Chronic Kidney disease (CKD)

Question 27/96

Question #27

About diabetic nephropathy, all the following are true except:

a. e rate of progression of renal impairment can be reduced with the use of ACE inhibitors or ARBs
b. Microalbuminuria is defined as persistent proteinuria of 30 – 299 mg/day
c. Hyperfiltration is the earliest sign of diabetic nephropathy
d. e disease primarily aects the glomeruli
e. Annual screening for microalbuminuria is done by urinalysis √

Description

Persistent albuminuria 30–299 mg/day (Microalbuminuria)

Occurs aer 5 – 10 years aer DM onset


May progress to macroalbuminuria (≥ 300 mg/day)
It can’t be detected by urinalysis

Screening for diabetic nephropathy is recommended by albumin creatinine ratio (ACR):

It should be done annually for all patients


It should be done by the early morning specimen
ACR > 2.5 is considered microalbuminuria

Page - 1544
Internal Medicine - Nephrology

Question 28/96

Question #28

A 55-year-old male patient has been a known case of chronic kidney disease (CKD) for a long time. He
presents with fatigue, and his physical examination shows pallor. His anemia workup demonstrates
normal serum iron and ferritin, normocytic anemia of 8g/dL, and normal B12 and folate. What is the
most eective treatment in this scenario?

a. Folate supplementation
b. B12 supplementation
c. Iron supplementation
d. Erythropoietin administration
e. Hemodialysis

‫اﻹﺟﺎﺑﺔ ﻋﲆ اﻟﺼﻔﺤﺔ اﻟﺘﺎﻟﻴﺔ‬

Page - 1545
Internal Medicine - Nephrology - Chronic Kidney disease (CKD)

Question 28/96

Question #28

A 55-year-old male patient has been a known case of chronic kidney disease (CKD) for a long time. He
presents with fatigue, and his physical examination shows pallor. His anemia workup demonstrates
normal serum iron and ferritin, normocytic anemia of 8g/dL, and normal B12 and folate. What is the
most eective treatment in this scenario?

a. Folate supplementation
b. B12 supplementation
c. Iron supplementation
d. Erythropoietin administration √
e. Hemodialysis

Description

In patients with CKD, the endocrine function of the kidney is impaired, and erythropoietin (EPO)
production is reduced. is will result in normocytic anemia, called anemia of chronic disease.

e treatment of choice for anemia of chronic disease in the presence of normal ferritin level is
erythropoietin (EPO) administration.

Iron supplementation should be given before EPO administration if low serum ferritin is present.

Page - 1546
Internal Medicine - Nephrology

Question 29/96

Question #29

A 48-year-old male patient with a history of chronic kidney disease presents with bone pain. Aer an
appropriate workup, you diagnosed him with hypocalcemia and vitamin D deficiency. Which of the
following is the main cause of vitamin D deficiency in CKD patients?

a. Reduced vitamin D absorption from the gut


b. Reduced 25(OH)D3 level
c. Reduced 1,25(OH)D3 activity
d. Reduced transformation of dehydrocholesterol to calciferol in the skin
e. Excessive vitamin D loss in urine

‫اﻹﺟﺎﺑﺔ ﻋﲆ اﻟﺼﻔﺤﺔ اﻟﺘﺎﻟﻴﺔ‬

Page - 1547
Internal Medicine - Nephrology - Chronic Kidney disease (CKD)

Question 29/96

Question #29

A 48-year-old male patient with a history of chronic kidney disease presents with bone pain. Aer an
appropriate workup, you diagnosed him with hypocalcemia and vitamin D deficiency. Which of the
following is the main cause of vitamin D deficiency in CKD patients?

a. Reduced vitamin D absorption from the gut


b. Reduced 25(OH)D3 level
c. Reduced 1,25(OH)D3 activity √
d. Reduced transformation of dehydrocholesterol to calciferol in the skin
e. Excessive vitamin D loss in urine

Description

e enzyme 1-alpha-hydroxylase in the kidneys is responsible for the hydroxylation of 25(oh)D3 to


become the active form of vitamin D “1.25(OH)D3”.

In the case of CKD, this enzyme is reduced, leading to reduced levels of the active form of vitamin D
“1,25(OH)D3”.

In the case of CKD, vitamin D replacement with active 1,25(OH)D3 or a related analog, along with
calcium supplementation, are required to prevent hypocalcemia and bone pain.

Page - 1548
Internal Medicine - Nephrology

Question 30/96

Question #30

A 55-year-old male patient has a history of chronic kidney disease and DM. he presents with loss of
sensation in his fingers and muscle cramping. His lab investigations show serum sodium of 139
mEq/L, serum potassium of 5.8 mEq/L, serum calcium of 1.5 mmol/L, normal PTH, and alkaline
phosphatase. What is the most likely cause of his symptoms?

a. Hyperkalemia
b. 1,25 (OH)D3 deficiency
c. Metabolic acidosis
d. Hyperphosphatemia
e. Hypernatremia

‫اﻹﺟﺎﺑﺔ ﻋﲆ اﻟﺼﻔﺤﺔ اﻟﺘﺎﻟﻴﺔ‬

Page - 1549
Internal Medicine - Nephrology - Chronic Kidney disease (CKD)

Question 30/96

Question #30

A 55-year-old male patient has a history of chronic kidney disease and DM. he presents with loss of
sensation in his fingers and muscle cramping. His lab investigations show serum sodium of 139
mEq/L, serum potassium of 5.8 mEq/L, serum calcium of 1.5 mmol/L, normal PTH, and alkaline
phosphatase. What is the most likely cause of his symptoms?

a. Hyperkalemia
b. 1,25 (OH)D3 deficiency √
c. Metabolic acidosis
d. Hyperphosphatemia
e. Hypernatremia

Description

is is a typical feature of hypocalcemia.

e enzyme 1-alpha-hydroxylase in the kidneys is responsible for the hydroxylation of 25(oh)D3 to


become the active form of vitamin D “1.25(OH)D3”.

In the case of CKD, this enzyme is reduced, leading to reduced levels of the active form of vitamin D
“1,25(OH)D3” and thus hypocalcemia.

Page - 1550
Internal Medicine - Nephrology

Question 31/96

Question #31

A 40-year-old male patient is found to have microscopic hematuria of 7 RBCs/hpf. His history and
physical examination are unremarkable. What is the next step in management?

a. Perform renal biopsy


b. 24-hour urinary collection for protein
c. Repeat the test 1 – 2 weeks later
d. Urine culture
e. Cystoscopy

‫اﻹﺟﺎﺑﺔ ﻋﲆ اﻟﺼﻔﺤﺔ اﻟﺘﺎﻟﻴﺔ‬

Page - 1551
Internal Medicine - Nephrology - Kidney function test.

Question 31/96

Question #31

A 40-year-old male patient is found to have microscopic hematuria of 7 RBCs/hpf. His history and
physical examination are unremarkable. What is the next step in management?

a. Perform renal biopsy


b. 24-hour urinary collection for protein
c. Repeat the test 1 – 2 weeks later √
d. Urine culture
e. Cystoscopy

Description

Description:

When you encounter microscopic hematuria in an otherwise asymptomatic and healthy patient,
you should confirm the diagnosis by repeating the urinalysis 1 – 2 weeks later. If the repeated test
returns as normal, nothing to do.

If the repeat comes back positive for hematuria again, you should identify the cause.

e most common cause is an infection, so urine culture should be done for all hematuria patients.

Note that glomerulonephritis, cancers, trauma, stones, and other pathologies could cause
hematuria, so you should perform your investigations according to your patient’s associated
symptoms and signs.

Page - 1552
Internal Medicine - Nephrology

Question 32/96

Question #32

A 33-year-old male patient has +1 proteinuria on routine urine dipstick analysis. He has no
symptoms, his vital signs are normal, and he was previously healthy. What is the most appropriate
next step in the management of this patient?

a. Refer to a nephrologist
b. Start Enalapril
c. Perform 24-hour urine collection for protein
d. Perform the same test again
e. No further evaluation

‫اﻹﺟﺎﺑﺔ ﻋﲆ اﻟﺼﻔﺤﺔ اﻟﺘﺎﻟﻴﺔ‬

Page - 1553
Internal Medicine - Nephrology - Kidney function test.

Question 32/96

Question #32

A 33-year-old male patient has +1 proteinuria on routine urine dipstick analysis. He has no
symptoms, his vital signs are normal, and he was previously healthy. What is the most appropriate
next step in the management of this patient?

a. Refer to a nephrologist
b. Start Enalapril
c. Perform 24-hour urine collection for protein
d. Perform the same test again √
e. No further evaluation

Description

You should repeat the test if the urine dipstick shows trace to +1 proteinuria without other evidence
of renal disease. is is because transient proteinuria and false-positive proteinuria are common.

Transient proteinuria is common in young patients; the following are possible causes of false-
positive or transient proteinuria:

Alkaline urine
Semen, vaginal secretion, or pus in the urine
If the dipstick is immersed for too long
Exercises

If you repeat the test and it is again positive for proteinuria, the next step will be 24-hours urine
collection for protein or protein creatinine ratio.

ACE inhibitors or ARBs are used in diabetic patients with proteinuria.

Referral to a nephrologist is indicated that 24-hour urinary protein is high.

Page - 1554
Internal Medicine - Nephrology

Question 33/96

Question #33

A 40-year-old male patient is found to have microscopic hematuria of 7 RBCs/hpf. His history and
physical examination are unremarkable. However, you decided to repeat the test 1 week later, and
the result returned positive again. What is the next step in management?

a. Perform renal biopsy


b. 24-hour urinary collection for protein
c. Repeat the test 1 – 2 weeks later
d. Urine culture
e. Cystoscopy

‫اﻹﺟﺎﺑﺔ ﻋﲆ اﻟﺼﻔﺤﺔ اﻟﺘﺎﻟﻴﺔ‬

Page - 1555
Internal Medicine - Nephrology - Kidney function test.

Question 33/96

Question #33

A 40-year-old male patient is found to have microscopic hematuria of 7 RBCs/hpf. His history and
physical examination are unremarkable. However, you decided to repeat the test 1 week later, and
the result returned positive again. What is the next step in management?

a. Perform renal biopsy


b. 24-hour urinary collection for protein
c. Repeat the test 1 – 2 weeks later
d. Urine culture √
e. Cystoscopy

Description

When you encounter microscopic hematuria in an otherwise asymptomatic and healthy patient,
you should confirm the diagnosis by repeating the urinalysis 1 – 2 weeks later. If the repeated test
returns as normal, then nothing to do.

If the repeat comes back positive for hematuria again, you should identify the cause.

e most common cause is an infection, so urine culture should be done for all hematuria patients.

Note that glomerulonephritis, cancers, trauma, stones, and other pathologies could cause
hematuria, so you should perform your investigations according to your patient’s associated
symptoms and signs.

Page - 1556
Internal Medicine - Nephrology

Question 34/96

Question #34

On routine examination, a 22-year-old male patient was found to have proteinuria of +2, which was
later confirmed 2 times to be present. You ordered a 24-hour urinary protein split collection, and the
result was 1.9 g/day (Total), normal 8 hours of urine protein excretion at night, and elevated 16 hours
of daytime protein excretion. In addition, creatinine clearance was normal. What is the most likely
diagnosis?

a. Nephrotic syndrome
b. Acute renal impairment
c. Nephritic syndrome
d. Orthostatic proteinuria
e. IgA nephropathy

‫اﻹﺟﺎﺑﺔ ﻋﲆ اﻟﺼﻔﺤﺔ اﻟﺘﺎﻟﻴﺔ‬

Page - 1557
Internal Medicine - Nephrology - Kidney function test.

Question 34/96

Question #34

On routine examination, a 22-year-old male patient was found to have proteinuria of +2, which was
later confirmed 2 times to be present. You ordered a 24-hour urinary protein split collection, and the
result was 1.9 g/day (Total), normal 8 hours of urine protein excretion at night, and elevated 16 hours
of daytime protein excretion. In addition, creatinine clearance was normal. What is the most likely
diagnosis?

a. Nephrotic syndrome
b. Acute renal impairment
c. Nephritic syndrome
d. Orthostatic proteinuria √
e. IgA nephropathy

Description

You should assess for orthostatic proteinuria for any < 30-year-old patient with < 2g/day proteinuria
and normal creatinine clearance.

Orthostatic proteinuria is a benign condition that occurs in adolescents and young adults.

It is characterized by increased protein excretion during the upright position but normal protein
excretion at the supine position.

Page - 1558
Internal Medicine - Nephrology

Question 35/96

Question #35

A 40-year-old male presents with generalized edema, hypoalbuminemia, and proteinuria. What is
the most accurate test to reach the final diagnosis?

a. Protein creatinine ratio


b. Urine culture
c. Abdominal CT scan with contrast
d. Renal biopsy
e. Serum total protein level

‫اﻹﺟﺎﺑﺔ ﻋﲆ اﻟﺼﻔﺤﺔ اﻟﺘﺎﻟﻴﺔ‬

Page - 1559
Internal Medicine - Nephrology - Nephrotic Syndrome

Question 35/96

Question #35

A 40-year-old male presents with generalized edema, hypoalbuminemia, and proteinuria. What is
the most accurate test to reach the final diagnosis?

a. Protein creatinine ratio


b. Urine culture
c. Abdominal CT scan with contrast
d. Renal biopsy √
e. Serum total protein level

Description

Based on this limited data, the patient is most likely suering from nephrotic syndrome.

To address the cause of Nephrotic Syndrome, the most accurate test to be done is renal biopsy.

Page - 1560
Internal Medicine - Nephrology

Question 36/96

Question #36

About nephrotic syndrome, the following statements are true except:

a. Diabetes Mellitus is a possible cause


b. Renal vein thrombosis is a known complication
c. Proteinuria is usually > 3.5 g/day
d. Hypoalbuminemia will lead to generalized edema
e. RBC casts and dysmorphic RBCs are characteristic

‫اﻹﺟﺎﺑﺔ ﻋﲆ اﻟﺼﻔﺤﺔ اﻟﺘﺎﻟﻴﺔ‬

Page - 1561
Internal Medicine - Nephrology - Nephrotic Syndrome

Question 36/96

Question #36

About nephrotic syndrome, the following statements are true except:

a. Diabetes Mellitus is a possible cause


b. Renal vein thrombosis is a known complication
c. Proteinuria is usually > 3.5 g/day
d. Hypoalbuminemia will lead to generalized edema
e. RBC casts and dysmorphic RBCs are characteristic √

Description

RBC casts and dysmorphic RBCs are characteristic findings in nephritic (not nephrotic) syndrome

Features of nephrotic syndrome include the following:

Proteinuria > 3.5 grams/day


Hypoalbuminemia leads to edema
Hypercholesterolemia (due to liver compensation)
Decreased renal perfusion due to reduced plasma volume → activation of the renin-angiotensin
system → sodium water retention
Increased risk of infection (due to protein loss)
Hypercoagulability due to:
Loss of anti-thrombin III in urine
Altered levels of protein C and S
Hyperfibrinogenemia (increased hepatic secretion)
Increased platelet tendency to aggregate

Page - 1562
Internal Medicine - Nephrology

Question 37/96

Question #37

A 70-year-old male patient presents with bilateral lower limb swelling and frothy looking urine. His
medical history is significant for lung cancer for which he receives palliative chemotherapy. On
examination, his blood pressure is 145/80 mmHg, and his pulse is 90 bpm. In addition, his lab
investigations demonstrate hemoglobin of 12g/dL, platelet count of 300 *10^9, WBC of 5 * 10^9,
serum albumin of 19g/L, serum creatinine of 3mg/dL, serum urea of 66 mg/dL, and 24-hour urine
collection for protein is 5 grams. What is the most likely diagnosis?

a. Post-streptococcal Glomerulonephritis
b. IgA nephropathy
c. Membranous glomerulonephritis
d. Focal segmental glomerulosclerosis
e. Minimal change disease

‫اﻹﺟﺎﺑﺔ ﻋﲆ اﻟﺼﻔﺤﺔ اﻟﺘﺎﻟﻴﺔ‬

Page - 1563
Internal Medicine - Nephrology - Nephrotic Syndrome

Question 37/96

Question #37

A 70-year-old male patient presents with bilateral lower limb swelling and frothy looking urine. His
medical history is significant for lung cancer for which he receives palliative chemotherapy. On
examination, his blood pressure is 145/80 mmHg, and his pulse is 90 bpm. In addition, his lab
investigations demonstrate hemoglobin of 12g/dL, platelet count of 300 *10^9, WBC of 5 * 10^9,
serum albumin of 19g/L, serum creatinine of 3mg/dL, serum urea of 66 mg/dL, and 24-hour urine
collection for protein is 5 grams. What is the most likely diagnosis?

a. Post-streptococcal Glomerulonephritis
b. IgA nephropathy
c. Membranous glomerulonephritis √
d. Focal segmental glomerulosclerosis
e. Minimal change disease

Description

e patient has nephrotic range proteinuria, lower limb swellings, and hypoalbuminemia. erefore,
he has a nephrotic syndrome.

e indication for the diagnosis of membranous glomerulonephritis here is the presence of


malignancy.

Malignancy can cause secondary membranous nephropathy, and the presence of nephrotic
syndrome in elderly patients increases the likelihood of malignancy.

Page - 1564
Internal Medicine - Nephrology

Question 38/96

Question #38

Which is considered the most common cause of secondary nephrotic syndrome?

a. Amyloidosis
b. Multiple myeloma
c. Diabetes mellitus
d. Hepatitis C
e. Post-infectious state

‫اﻹﺟﺎﺑﺔ ﻋﲆ اﻟﺼﻔﺤﺔ اﻟﺘﺎﻟﻴﺔ‬

Page - 1565
Internal Medicine - Nephrology - Nephrotic Syndrome

Question 38/96

Question #38

Which is considered the most common cause of secondary nephrotic syndrome?

a. Amyloidosis
b. Multiple myeloma
c. Diabetes mellitus √
d. Hepatitis C
e. Post-infectious state

Description

DM is the most common cause of Nephrotic Syndrome

Features of nephrotic syndrome include the following:

Proteinuria > 3.5 grams/day


Hypoalbuminemia → leads to edemas
Hypercholesterolemia (due to liver compensation)
Decreased renal perfusion due to reduced plasma volume → activation of the renin-angiotensin
system → sodium water retention
Increased risk of infection (due to protein loss)
Hypercoagulability due to:
Loss of anti-thrombin III in urine
Altered levels of protein C and S
Hyperfibrinogenemia (increased hepatic secretion)

Increased platelet tendency to aggregate

Page - 1566
Internal Medicine - Nephrology

Question 39/96

Question #39

A 24-year-old male patient with a history of chronic kidney disease was recently diagnosed with
HTN and was started on hydrochlorothiazide. What is the best next step in the management of this
patient?

a. Urinary metanephrine level


b. Dexamethasone suppression test
c. Insulin-like growth factor
d. Renal doppler ultrasound
e. No further evaluation

‫اﻹﺟﺎﺑﺔ ﻋﲆ اﻟﺼﻔﺤﺔ اﻟﺘﺎﻟﻴﺔ‬

Page - 1567
Internal Medicine - Nephrology - Renal Artery Stenosis

Question 39/96

Question #39

A 24-year-old male patient with a history of chronic kidney disease was recently diagnosed with
HTN and was started on hydrochlorothiazide. What is the best next step in the management of this
patient?

a. Urinary metanephrine level


b. Dexamethasone suppression test
c. Insulin-like growth factor
d. Renal doppler ultrasound √
e. No further evaluation

Description

is patient is most commonly suering from renal artery stenosis and secondary HTN. Consider
renal artery stenosis in a young patient with HTN

Renal artery stenosis:

e most common cause in the elderly is atherosclerosis,


e most common cause in young patients is fibromuscular dysplasia
Suspect RAS if deteriorating renal function (> 30% increase in creatinine) in a patient with HTN
and newly put on ACEI/ARBs
Diagnosis: abdominal bruit (O/E), Doppler U/S, angiography (best)
Treatment: angioplasty with stenting
ACEI is contraindicated in bilateral disease (they lead to an increase in creatinine)

Page - 1568
Internal Medicine - Nephrology

Question 40/96

Question #40

In a patient with renal artery stenosis, azotemia can develop due to taking which of the following
medications?

a. Nifedipine
b. Doxazocin
c. Valsartan
d. Metoprolol
e. Indapamide

‫اﻹﺟﺎﺑﺔ ﻋﲆ اﻟﺼﻔﺤﺔ اﻟﺘﺎﻟﻴﺔ‬

Page - 1569
Internal Medicine - Nephrology - Renal Artery Stenosis

Question 40/96

Question #40

In a patient with renal artery stenosis, azotemia can develop due to taking which of the following
medications?

a. Nifedipine
b. Doxazocin
c. Valsartan √
d. Metoprolol
e. Indapamide

Description

ACE inhibitors and ARBs can induce renal insuciency in the presence of renal artery stenosis.
Elevation of creatinine > 30% above the baseline aer 1 week of ACE inhibitors or ARBs use should
raise the suspicion of renal artery stenosis.

Renal artery stenosis (RAS):

e most common cause in the elderly is atherosclerosis,


e most common cause in young patients is fibromuscular dysplasia
Suspect RAS if deteriorating renal function (> 30% increase in creatinine) in a patient with HTN
and newly put on ACEI/ARBs
Diagnosis: abdominal bruit (O/E), Doppler U/S, angiography (best)
Treatment: angioplasty with stenting
ACEI is contraindicated in bilateral disease (they lead to an increase in creatinine)

Page - 1570
Internal Medicine - Nephrology

Question 41/96

Question #41

A 32-year-old female is seen for refractory HTN. Her physical examination demonstrates abdominal
bruits bilaterally. What is the most likely diagnosis?

a. Pheochromocytoma
b. Primary hyperaldosteronism
c. Cushing syndrome
d. Renal artery stenosis
e. Liddle syndrome

‫اﻹﺟﺎﺑﺔ ﻋﲆ اﻟﺼﻔﺤﺔ اﻟﺘﺎﻟﻴﺔ‬

Page - 1571
Internal Medicine - Nephrology - Renal Artery Stenosis

Question 41/96

Question #41

A 32-year-old female is seen for refractory HTN. Her physical examination demonstrates abdominal
bruits bilaterally. What is the most likely diagnosis?

a. Pheochromocytoma
b. Primary hyperaldosteronism
c. Cushing syndrome
d. Renal artery stenosis √
e. Liddle syndrome

Description

is patient is most commonly suering from renal artery stenosis and secondary HTN

Consider renal artery stenosis in a young patient with HTN

Renal artery stenosis:

e most common cause in the elderly is atherosclerosis,


e most common cause in young patients is fibromuscular dysplasia
Suspect RAS if deteriorating renal function (> 30% increase in creatinine) in a patient with HTN
and newly put on ACEI/ARBs
Diagnosis: abdominal bruit (O/E), Doppler U/S, angiography (best)
Treatment: angioplasty with stenting
ACEI is contraindicated in bilateral disease (they lead to an increase in creatinine)

Page - 1572
Internal Medicine - Nephrology

Question 42/96

Question #42

In a 40-year-old male patient with severe HTN and renal failure, which of the following is the most
appropriate to reach the diagnosis?

a. Renal ultrasonography
b. Brain CT scan with contrast
c. Renal doppler scan
d. Renal biopsy
e. Urinary metanephrine levels

‫اﻹﺟﺎﺑﺔ ﻋﲆ اﻟﺼﻔﺤﺔ اﻟﺘﺎﻟﻴﺔ‬

Page - 1573
Internal Medicine - Nephrology - Renal Artery Stenosis

Question 42/96

Question #42

In a 40-year-old male patient with severe HTN and renal failure, which of the following is the most
appropriate to reach the diagnosis?

a. Renal ultrasonography
b. Brain CT scan with contrast
c. Renal doppler scan √
d. Renal biopsy
e. Urinary metanephrine levels

Description

Suspect renal artery stenosis in a young patient with HTN and renal impairment.

Renal artery stenosis:

e most common cause in the elderly is atherosclerosis,


e most common cause in young patients is fibromuscular dysplasia
Suspect RAS if deteriorating renal function (> 30% increase in creatinine) in a patient with HTN
and newly put on ACEI/ARBs
Diagnosis: abdominal bruit (O/E), Doppler U/S, angiography (best)
Treatment: angioplasty with stenting
ACEI is contraindicated in bilateral disease (they lead to an increase in creatinine)

Page - 1574
Internal Medicine - Nephrology

Question 43/96

Question #43

Which of the following options best states the pathophysiology of renal artery stenosis?

a. Renin ↑, Angiotensin II ↓, Aldosterone ↑, Sodium excretion ↓, Renal blood flow ↓


b. Renin ↓, Angiotensin II ↑, Aldosterone ↓, Sodium excretion ↓, Renal blood flow ↓
c. Renin ↑, Angiotensin II ↑, Aldosterone ↑, Sodium excretion ↓, Renal blood flow ↓
d. Renin ↑, Angiotensin II ↑, Aldosterone ↑, Sodium excretion ↑, Renal blood flow ↓
e. Renin ↑, Angiotensin II ↑, Aldosterone ↑, Sodium excretion ↓, Renal blood flow ↑

‫اﻹﺟﺎﺑﺔ ﻋﲆ اﻟﺼﻔﺤﺔ اﻟﺘﺎﻟﻴﺔ‬

Page - 1575
Internal Medicine - Nephrology - Renal Artery Stenosis

Question 43/96

Question #43

Which of the following options best states the pathophysiology of renal artery stenosis?

a. Renin ↑, Angiotensin II ↓, Aldosterone ↑, Sodium excretion ↓, Renal blood flow ↓


b. Renin ↓, Angiotensin II ↑, Aldosterone ↓, Sodium excretion ↓, Renal blood flow ↓
c. Renin ↑, Angiotensin II ↑, Aldosterone ↑, Sodium excretion ↓, Renal blood flow ↓ √
d. Renin ↑, Angiotensin II ↑, Aldosterone ↑, Sodium excretion ↑, Renal blood flow ↓
e. Renin ↑, Angiotensin II ↑, Aldosterone ↑, Sodium excretion ↓, Renal blood flow ↑

Description

In renal artery stenosis, there will be a reduction in the blood supply to the kidney due to the
stenosed artery (reduced renal flow)

e reduction in the renal flow will result in renin-angiotensin-aldosterone system activation; this
means high renin, high angiotensin, and high aldosterone levels

e high aldosterone level will lead to sodium and water retention, i.e., reduced-sodium urinary
excretion.

GFR is maintained in the early phase of the disease.

Page - 1576
Internal Medicine - Nephrology

Question 44/96

Question #44

A 72-year-old male patient with chronic kidney disease (baseline creatinine is 1.4 mg/dL) and HTN
was started on Enalapril 10 mg for uncontrolled blood pressure. 2 weeks later, his blood pressure
was found to be 145/92, and his serum creatinine level was found to be 4 mg/dL. What is the most
appropriate step in the management of this patient?

a. Stop Enalapril and start valsartan


b. Stop enalapril and perform renal doppler ultrasound
c. Increase the dose of enalapril
d. No change in his medications
e. Hemodialysis

‫اﻹﺟﺎﺑﺔ ﻋﲆ اﻟﺼﻔﺤﺔ اﻟﺘﺎﻟﻴﺔ‬

Page - 1577
Internal Medicine - Nephrology - Renal Artery Stenosis

Question 44/96

Question #44

A 72-year-old male patient with chronic kidney disease (baseline creatinine is 1.4 mg/dL) and HTN
was started on Enalapril 10 mg for uncontrolled blood pressure. 2 weeks later, his blood pressure
was found to be 145/92, and his serum creatinine level was found to be 4 mg/dL. What is the most
appropriate step in the management of this patient?

a. Stop Enalapril and start valsartan


b. Stop enalapril and perform renal doppler ultrasound √
c. Increase the dose of enalapril
d. No change in his medications
e. Hemodialysis

Description

Suspect renal artery stenosis when deteriorating renal function (> 30% increase in creatinine aer 1
week) in a patient with HTN and newly put on ACEI/ARB

Renal artery stenosis:

e most common cause in the elderly is atherosclerosis,


e most common cause in young patients is fibromuscular dysplasia
Diagnosis: abdominal bruit (O/E), Doppler U/S, angiography (best)
Treatment: angioplasty with stenting
ACEI is contraindicated in bilateral disease (they lead to an increase in creatinine)

Page - 1578
Internal Medicine - Nephrology

Question 45/96

Question #45

A 32-year-old male patient has recurrent hematuria related to an upper respiratory infection. Today,
he presented with a fever and sore throat for 2 days, and was again associated with hematuria. His
blood pressure is 175/99 mmHg. His urine test shows numerous RBCs, heavy proteinuria, and RBC
casts. What is the most likely diagnosis?

a. Post-streptococcal glomerulonephritis
b. Acute interstitial nephritis
c. IgA nephropathy
d. Urinary tract infection
e. Membranous glomerulonephritis

‫اﻹﺟﺎﺑﺔ ﻋﲆ اﻟﺼﻔﺤﺔ اﻟﺘﺎﻟﻴﺔ‬

Page - 1579
Internal Medicine - Nephrology - Renal glomerular diseases

Question 45/96

Question #45

A 32-year-old male patient has recurrent hematuria related to an upper respiratory infection. Today,
he presented with a fever and sore throat for 2 days, and was again associated with hematuria. His
blood pressure is 175/99 mmHg. His urine test shows numerous RBCs, heavy proteinuria, and RBC
casts. What is the most likely diagnosis?

a. Post-streptococcal glomerulonephritis
b. Acute interstitial nephritis
c. IgA nephropathy √
d. Urinary tract infection
e. Membranous glomerulonephritis

Description

IgA nephropathy results from IgA deposition in the mesangial tissue of the glomeruli. It is the most
common cause of glomerulonephritis in the world.

IgA nephropathy usually occurs in relation to upper respiratory infections with a time interval of 1 –
2 days aer the infection.

Note that Post-streptococcal glomerulonephritis occurs 1 – 2 weeks aer a throat infection and is
associated with low complement proteins C3 and C4.

Interstitial nephritis is common aer streptococcal infection as well but usually presents with skin
rash, fever, arthralgia, and eosinophilia.

e following table describes the dierence between IgA nephropathy and PSGN:

Page - 1580
Internal Medicine - Nephrology

Question 46/96

Question #46

A 22-year-old female develops red urine without dysuria or abdominal pain. During the last 2 days,
she has been suering from fever and sore throat. Her lab tests today show 17 RBCs /hpf and
otherwise normal urinalysis. Which of the following is the most accurate test to confirm the
diagnosis of this lady?

a. ASO titer
b. roat swap and culture
c. Renal biopsy
d. Renal ultrasound
e. Kidney function test

‫اﻹﺟﺎﺑﺔ ﻋﲆ اﻟﺼﻔﺤﺔ اﻟﺘﺎﻟﻴﺔ‬

Page - 1581
Internal Medicine - Nephrology - Renal glomerular diseases

Question 46/96

Question #46

A 22-year-old female develops red urine without dysuria or abdominal pain. During the last 2 days,
she has been suering from fever and sore throat. Her lab tests today show 17 RBCs /hpf and
otherwise normal urinalysis. Which of the following is the most accurate test to confirm the
diagnosis of this lady?

a. ASO titer
b. roat swap and culture
c. Renal biopsy √
d. Renal ultrasound
e. Kidney function test

Description

is lady is most likely suering from IgA nephropathy.

e most accurate test to confirm the diagnosis is renal biopsy

Berger’s disease is also known as IgA nephropathy.

IgA nephropathy results from IgA deposition in the mesangial tissue of the glomeruli. It is the most
common cause of glomerulonephritis in the world.

IgA nephropathy usually occurs in relation to upper respiratory infections with a time interval of 1 –
2 days aer the infection.

Note that Post-streptococcal glomerulonephritis occurs 1 – 2 weeks aer a throat infection and is
associated with low complement proteins C3 and C4.

Page - 1582
Internal Medicine - Nephrology

Question 47/96

Question #47

A 25-year-old male patient complains of tea-colored urine for a 1-day duration. His history is only
significant for sore throat and fever 2 weeks ago. However, his lab tests show positive ASO titer.
What is the most likely diagnosis?

a. Focal segmental glomerulosclerosis


b. Minimal change disease
c. IgA nephropathy
d. Post-streptococcal glomerulonephritis
e. Granulomatosis with polyangiitis

‫اﻹﺟﺎﺑﺔ ﻋﲆ اﻟﺼﻔﺤﺔ اﻟﺘﺎﻟﻴﺔ‬

Page - 1583
Internal Medicine - Nephrology - Renal glomerular diseases

Question 47/96

Question #47

A 25-year-old male patient complains of tea-colored urine for a 1-day duration. His history is only
significant for sore throat and fever 2 weeks ago. However, his lab tests show positive ASO titer.
What is the most likely diagnosis?

a. Focal segmental glomerulosclerosis


b. Minimal change disease
c. IgA nephropathy
d. Post-streptococcal glomerulonephritis √
e. Granulomatosis with polyangiitis

Description

is patient is most likely suering from Post streptococcal glomerulonephritis (PSGN).

e presence of sore throat and fever 2 weeks ago, followed by hematuria and positive ASO titer,
suggests the disease.

e following table describes the dierence between IgA nephropathy and PSGN:

Page - 1584
Internal Medicine - Nephrology

Question 48/96

Question #48

A 21-year-old male patient presents with generalized edema and frothy-looking urine. His urine tests
show 5g/day proteinuria, hypercholesterolemia, and hypoalbuminemia. A renal biopsy is done and
shows a normal examination under the light microscope. However, the electronic microscope shows
eacement of epithelial foot processes. What is the most likely diagnosis?

a. Minimal change disease


b. Membranous glomerulonephritis
c. Membranoproliferative glomerulonephritis
d. Focal segmental glomerulosclerosis
e. Diuse proliferative glomerulonephritis

‫اﻹﺟﺎﺑﺔ ﻋﲆ اﻟﺼﻔﺤﺔ اﻟﺘﺎﻟﻴﺔ‬

Page - 1585
Internal Medicine - Nephrology - Renal glomerular diseases

Question 48/96

Question #48

A 21-year-old male patient presents with generalized edema and frothy-looking urine. His urine tests
show 5g/day proteinuria, hypercholesterolemia, and hypoalbuminemia. A renal biopsy is done and
shows a normal examination under the light microscope. However, the electronic microscope shows
eacement of epithelial foot processes. What is the most likely diagnosis?

a. Minimal change disease √


b. Membranous glomerulonephritis
c. Membranoproliferative glomerulonephritis
d. Focal segmental glomerulosclerosis
e. Diuse proliferative glomerulonephritis

Description

Minimal change disease is the most common cause of primary Nephrotic syndrome in children.

It is characterized by eacement of podocytes foot processes seen by the electronic microscope but
no changes seen on the light microscope.

Minimal change disease is the most responsive to steroids.

Page - 1586
Internal Medicine - Nephrology

Question 49/96

Question #49

A 22-year-old male patient is suspected of having acute glomerulonephritis. Which of the following
would most strongly support the diagnosis?

a. RBCs in urine
b. WBCs in urine
c. Positive myoglobinuria
d. Urinary eosinophilia
e. Erythrocyte cast in urine

‫اﻹﺟﺎﺑﺔ ﻋﲆ اﻟﺼﻔﺤﺔ اﻟﺘﺎﻟﻴﺔ‬

Page - 1587
Internal Medicine - Nephrology - Renal glomerular diseases

Question 49/96

Question #49

A 22-year-old male patient is suspected of having acute glomerulonephritis. Which of the following
would most strongly support the diagnosis?

a. RBCs in urine
b. WBCs in urine
c. Positive myoglobinuria
d. Urinary eosinophilia
e. Erythrocyte cast in urine √

Description

e presence of RBC casts indicates glomerular disease.

Acute glomerulonephritis usually presents with hypertension, edema, and hematuria.

RBCs in urine (Hematuria) is not specific and may occur in urinary tract conditions like trauma,
tumors, stones, injuries, and infections.

WBCs in the urine suggest infection, while eosinophiluria may present with acute interstitial
nephritis.

Myoglobinuria presents with rhabdomyolysis that may lead to acute tubular necrosis.

Page - 1588
Internal Medicine - Nephrology

Question 50/96

Question #50

About 48 hours aer an upper respiratory illness, a 19-year-old male patient develops hematuria. His
urine dipstick shows proteinuria and red blood cells. Dysmorphic RBCs and RBCs cast are shown in
urine microscopy. What is the most likely diagnosis?

a. Post streptococcal glomerulonephritis


b. Berger’s disease
c. Minimal change disease
d. Henoch-Schönlein purpura
e. Acute interstitial nephritis

‫اﻹﺟﺎﺑﺔ ﻋﲆ اﻟﺼﻔﺤﺔ اﻟﺘﺎﻟﻴﺔ‬

Page - 1589
Internal Medicine - Nephrology - Renal glomerular diseases

Question 50/96

Question #50

About 48 hours aer an upper respiratory illness, a 19-year-old male patient develops hematuria. His
urine dipstick shows proteinuria and red blood cells. Dysmorphic RBCs and RBCs cast are shown in
urine microscopy. What is the most likely diagnosis?

a. Post streptococcal glomerulonephritis


b. Berger’s disease √
c. Minimal change disease
d. Henoch-Schönlein purpura
e. Acute interstitial nephritis

Description

Berger’s disease is also known as IgA nephropathy.

IgA nephropathy results from IgA deposition in the mesangial tissue of the glomeruli. It is the most
common cause of glomerulonephritis in the world.

IgA nephropathy usually occurs in relation to upper respiratory infections with a time interval of 1 –
2 days aer the infection.

Note that Post-streptococcal glomerulonephritis occurs 1 – 2 weeks aer a throat infection and is
associated with low complement proteins C3 and C4.

Interstitial nephritis is common aer streptococcal infection as well but usually presents with skin
rash, fever, arthralgia, and eosinophilia.

e following table describes the dierence between IgA nephropathy and PSGN:

Page - 1590
Internal Medicine - Nephrology

Question 51/96

Question #51

A 55-year-old male patient develops pedal edema, nephrotic range proteinuria,


hypercholesterolemia, and hypoalbuminemia. His renal biopsy was done, but the result has not
come back yet. What is the most likely cause of his nephrotic syndrome?

a. Minimal change disease


b. Membranous glomerulonephritis
c. Membranoproliferative glomerulonephritis
d. Focal segmental glomerulonephritis
e. IgA nephropathy

‫اﻹﺟﺎﺑﺔ ﻋﲆ اﻟﺼﻔﺤﺔ اﻟﺘﺎﻟﻴﺔ‬

Page - 1591
Internal Medicine - Nephrology - Renal glomerular diseases

Question 51/96

Question #51

A 55-year-old male patient develops pedal edema, nephrotic range proteinuria,


hypercholesterolemia, and hypoalbuminemia. His renal biopsy was done, but the result has not
come back yet. What is the most likely cause of his nephrotic syndrome?

a. Minimal change disease


b. Membranous glomerulonephritis √
c. Membranoproliferative glomerulonephritis
d. Focal segmental glomerulonephritis
e. IgA nephropathy

Description

Membranous glomerulonephritis is the most common form of primary nephrotic syndrome in


adults

Minimal change disease is the most common in children, while focal segmental glomerulosclerosis
is more in immunosuppressed patients (HIV patients)

Page - 1592
Internal Medicine - Nephrology

Question 52/96

Question #52

A 32-year-old male patient has recurrent hematuria related to an upper respiratory infection. Today,
he presented with a fever and sore throat for 2 days which was again associated with hematuria. His
blood pressure is 175/99 mmHg. His urine test shows numerous RBCs, heavy proteinuria, and
dysmorphic RBCs with RBC casts. e presence of dysmorphic RBCs is suggestive of:

a. Urinary bladder mucosal bleeding


b. Glomerular bleeding
c. Ureteral stone
d. Renal malignancy
e. Urinary tract infection

‫اﻹﺟﺎﺑﺔ ﻋﲆ اﻟﺼﻔﺤﺔ اﻟﺘﺎﻟﻴﺔ‬

Page - 1593
Internal Medicine - Nephrology - Renal glomerular diseases

Question 52/96

Question #52

A 32-year-old male patient has recurrent hematuria related to an upper respiratory infection. Today,
he presented with a fever and sore throat for 2 days which was again associated with hematuria. His
blood pressure is 175/99 mmHg. His urine test shows numerous RBCs, heavy proteinuria, and
dysmorphic RBCs with RBC casts. e presence of dysmorphic RBCs is suggestive of:

a. Urinary bladder mucosal bleeding


b. Glomerular bleeding √
c. Ureteral stone
d. Renal malignancy
e. Urinary tract infection

Description

e presence of dysmorphic RBCs suggests glomerular hematuria.

e patient most likely has IgA nephropathy.

IgA nephropathy results from IgA deposition in the mesangial tissue of the glomeruli. It is the most
common cause of glomerulonephritis in the world.

IgA nephropathy usually occurs in relation to upper respiratory infections with a time interval of 1 –
2 days aer the infection.

Note that Post-streptococcal glomerulonephritis occurs 1 – 2 weeks aer a throat infection and is
associated with low complement proteins C3 and C4.

Interstitial nephritis is common aer streptococcal infection as well but usually presents with skin
rash, fever, arthralgia, and eosinophilia.

e following table describes the dierence between IgA nephropathy and PSGN:

Page - 1594
Page - 1595
Internal Medicine - Nephrology

Question 53/96

Question #53

A 35-year-old male patient develops coughs and hemoptysis for 2 months. His lab tests show
elevated urea and creatinine levels, urinalysis shows red urine that is positive for numerous RBCs,
and his arterial blood gases are normal. Which of the following is the most helpful to confirm the
diagnosis?

a. Anti-mitochondrial antibodies
b. Anti-GBM antibodies
c. Anti-neutrophil cytoplasmic antibodies
d. Anti-smooth muscle antibodies
e. Anti-nuclear antibodies

‫اﻹﺟﺎﺑﺔ ﻋﲆ اﻟﺼﻔﺤﺔ اﻟﺘﺎﻟﻴﺔ‬

Page - 1596
Internal Medicine - Nephrology - Renal glomerular diseases

Question 53/96

Question #53

A 35-year-old male patient develops coughs and hemoptysis for 2 months. His lab tests show
elevated urea and creatinine levels, urinalysis shows red urine that is positive for numerous RBCs,
and his arterial blood gases are normal. Which of the following is the most helpful to confirm the
diagnosis?

a. Anti-mitochondrial antibodies
b. Anti-GBM antibodies √
c. Anti-neutrophil cytoplasmic antibodies
d. Anti-smooth muscle antibodies
e. Anti-nuclear antibodies

Description

is is a typical case scenario of Goodpasture syndrome.

Goodpasture syndrome is a clinical syndrome that comprises glomerulonephritis and pulmonary


hemorrhage due to injury mediated by Anti-glomerular basement membrane antibody (Anti-GBM)

e presence of hemoptysis and hematuria without upper respiratory involvement strongly


suggests the disease

If upper respiratory involvement is present, you should consider granulomatosis with polyangiitis as
the most likely diagnosis.

Page - 1597
Internal Medicine - Nephrology

Question 54/96

Question #54

A 33-year-old male patient presents with 2 weeks history of hemoptysis. In the last 48 hours, he has
developed hematuria, breathlessness, and oliguria. His chest x-ray shows patchy interstitial
infiltrates aecting mainly the lower zones. His lab investigations show renal impairment and mild
hyperkalemia. What is the most appropriate management at this point?

a. Anti-glomerular basement membrane antibodies test


b. Abdominal x-ray
c. Abdominal ultrasound
d. Abdominal and chest CT scan
e. Cystoscopy and bronchoscopy

‫اﻹﺟﺎﺑﺔ ﻋﲆ اﻟﺼﻔﺤﺔ اﻟﺘﺎﻟﻴﺔ‬

Page - 1598
Internal Medicine - Nephrology - Renal glomerular diseases

Question 54/96

Question #54

A 33-year-old male patient presents with 2 weeks history of hemoptysis. In the last 48 hours, he has
developed hematuria, breathlessness, and oliguria. His chest x-ray shows patchy interstitial
infiltrates aecting mainly the lower zones. His lab investigations show renal impairment and mild
hyperkalemia. What is the most appropriate management at this point?

a. Anti-glomerular basement membrane antibodies test √


b. Abdominal x-ray
c. Abdominal ultrasound
d. Abdominal and chest CT scan
e. Cystoscopy and bronchoscopy

Description

is is a typical case scenario of Goodpasture syndrome.

Goodpasture syndrome is a clinical syndrome that comprises glomerulonephritis and pulmonary


hemorrhage due to injury mediated by Anti-glomerular basement membrane antibody (Anti-GBM)

e presence of hemoptysis and hematuria without upper respiratory involvement strongly


suggests the disease

If upper respiratory involvement is present, you should consider granulomatosis with polyangiitis as
the most likely diagnosis.

Page - 1599
Internal Medicine - Nephrology

Question 55/96

Question #55

A 30-year-old male patient presents with generalized edema. His lab tests show hypoalbuminemia,
heavy proteinuria, and hypercholesterolemia. His past medical history includes advanced HIV
infection. However, his blood pressure is 140/90, his pulse is 90bpm, and he is afebrile. What is the
most likely cause of his condition?

a. Minimal change disease


b. Membranous glomerulonephritis
c. Membranoproliferative glomerulonephritis
d. Focal segmental glomerulosclerosis
e. Diuse proliferative glomerulonephritis

‫اﻹﺟﺎﺑﺔ ﻋﲆ اﻟﺼﻔﺤﺔ اﻟﺘﺎﻟﻴﺔ‬

Page - 1600
Internal Medicine - Nephrology - Renal glomerular diseases

Question 55/96

Question #55

A 30-year-old male patient presents with generalized edema. His lab tests show hypoalbuminemia,
heavy proteinuria, and hypercholesterolemia. His past medical history includes advanced HIV
infection. However, his blood pressure is 140/90, his pulse is 90bpm, and he is afebrile. What is the
most likely cause of his condition?

a. Minimal change disease


b. Membranous glomerulonephritis
c. Membranoproliferative glomerulonephritis
d. Focal segmental glomerulosclerosis √
e. Diuse proliferative glomerulonephritis

Description

Focal segmental glomerulosclerosis is the most common form of idiopathic nephrotic syndrome in
HIV-positive patients.

Minimal change disease is more common in children, while membranous glomerulonephritis is


more common in adults.

Page - 1601
Internal Medicine - Nephrology

Question 56/96

Question #56

Which of the following is considered the most responsive to steroid therapy?

a. Minimal change disease


b. Membranous glomerulonephritis
c. Membranoproliferative glomerulonephritis
d. Focal segmental glomerulosclerosis
e. Diuse proliferative glomerulonephritis

‫اﻹﺟﺎﺑﺔ ﻋﲆ اﻟﺼﻔﺤﺔ اﻟﺘﺎﻟﻴﺔ‬

Page - 1602
Internal Medicine - Nephrology - Renal glomerular diseases

Question 56/96

Question #56

Which of the following is considered the most responsive to steroid therapy?

a. Minimal change disease √


b. Membranous glomerulonephritis
c. Membranoproliferative glomerulonephritis
d. Focal segmental glomerulosclerosis
e. Diuse proliferative glomerulonephritis

Description

Minimal change disease is the most common cause of primary Nephrotic syndrome in children.

It is characterized by eacement of podocytes foot processes seen by the electronic microscope but
no changes seen on the light microscope.

Minimal change disease is the most responsive to steroids.

Page - 1603
Internal Medicine - Nephrology

Question 57/96

Question #57

A 39-year-old male patient is diagnosed with adult polycystic kidney disease. However, his brother
recently died due to a brain insult. What is the most likely cause of his brother’s death?

a. Ruptured berry aneurysm


b. Subdural hematoma
c. Epidural hematoma
d. Intracerebral hemorrhage
e. Ischemic CVA

‫اﻹﺟﺎﺑﺔ ﻋﲆ اﻟﺼﻔﺤﺔ اﻟﺘﺎﻟﻴﺔ‬

Page - 1604
Internal Medicine - Nephrology - Cystic diseases of the kidney

Question 57/96

Question #57

A 39-year-old male patient is diagnosed with adult polycystic kidney disease. However, his brother
recently died due to a brain insult. What is the most likely cause of his brother’s death?

a. Ruptured berry aneurysm √


b. Subdural hematoma
c. Epidural hematoma
d. Intracerebral hemorrhage
e. Ischemic CVA

Description

Autosomal dominant polycystic kidney disease is associated with berry aneurysms that may rupture
and lead to subarachnoid hemorrhage.

A family history of ADPKD is a sucient clue in the question to raise the suspicion of berry aneurysm

Page - 1605
Internal Medicine - Nephrology

Question 58/96

Question #58

A 35-year-old male patient complains of HTN, painless hematuria, flank pain, and recurrent
headache. What is the most likely diagnosis?

a. Transitional cell carcinoma of the urinary bladder


b. Urinary tract infection
c. Hemolytic uremic syndrome
d. Adult polycystic kidney disease
e. Prostatic cancer

‫اﻹﺟﺎﺑﺔ ﻋﲆ اﻟﺼﻔﺤﺔ اﻟﺘﺎﻟﻴﺔ‬

Page - 1606
Internal Medicine - Nephrology - Cystic diseases of the kidney

Question 58/96

Question #58

A 35-year-old male patient complains of HTN, painless hematuria, flank pain, and recurrent
headache. What is the most likely diagnosis?

a. Transitional cell carcinoma of the urinary bladder


b. Urinary tract infection
c. Hemolytic uremic syndrome
d. Adult polycystic kidney disease √
e. Prostatic cancer

Description

is is a typical presentation of adult-onset autosomal dominant polycystic kidney disease (ADPKD)

ADPKD results from two types of gene mutations

PKD1 (more severe and early onset) (code for polycystin1)


PKD2 (less severe and later onset) (code for polycystin2)

Recurrent headaches are due to the association between ADPKD and berry aneurysms, which may
rupture and cause SAH.

Page - 1607
Internal Medicine - Nephrology

Question 59/96

Question #59

A 36-year-old male patient has a history of HTN and subarachnoid hemorrhage. In addition, he
presents with recurrent hematuria and UTI. However, His physical examination demonstrates
enlarged kidneys, and his family history is positive for a grandfather with renal impairment at the
age of 40. What is the most likely diagnosis?

a. Autosomal dominant polycystic kidney disease


b. Autosomal recessive polycystic kidney disease
c. Alport syndrome
d. IgA nephropathy
e. Ascending bacterial infection of the urinary tract

‫اﻹﺟﺎﺑﺔ ﻋﲆ اﻟﺼﻔﺤﺔ اﻟﺘﺎﻟﻴﺔ‬

Page - 1608
Internal Medicine - Nephrology - Cystic diseases of the kidney

Question 59/96

Question #59

A 36-year-old male patient has a history of HTN and subarachnoid hemorrhage. In addition, he
presents with recurrent hematuria and UTI. However, His physical examination demonstrates
enlarged kidneys, and his family history is positive for a grandfather with renal impairment at the
age of 40. What is the most likely diagnosis?

a. Autosomal dominant polycystic kidney disease √


b. Autosomal recessive polycystic kidney disease
c. Alport syndrome
d. IgA nephropathy
e. Ascending bacterial infection of the urinary tract

Description

Description:

is is a typical presentation of adult-onset autosomal dominant polycystic kidney disease (ADPKD)

ADPKD results from two types of gene mutations

PKD1 (more severe and early onset) (code for polycystin1)


PKD2 (less severe and later onset) (code for polycystin2)

e presence of subarachnoid hemorrhage (SAH) is due to the presence of an association between


ADPKD and berry aneurysm, which may rupture and cause SAH

Page - 1609
Internal Medicine - Nephrology

Question 60/96

Question #60

A 43-year-old male patient was found to have an accidental finding of unilateral simple renal cyst.
Lab results and physical examination are unremarkable. What is the next step in the management
of this patient?

a. Reassurance
b. Perform renal biopsy
c. Perform nephrectomy
d. Genetic study for PKD1 and PKD2 genes
e. Perform Pan-CT scan

‫اﻹﺟﺎﺑﺔ ﻋﲆ اﻟﺼﻔﺤﺔ اﻟﺘﺎﻟﻴﺔ‬

Page - 1610
Internal Medicine - Nephrology - Cystic diseases of the kidney

Question 60/96

Question #60

A 43-year-old male patient was found to have an accidental finding of unilateral simple renal cyst.
Lab results and physical examination are unremarkable. What is the next step in the management
of this patient?

a. Reassurance √
b. Perform renal biopsy
c. Perform nephrectomy
d. Genetic study for PKD1 and PKD2 genes
e. Perform Pan-CT scan

Description

A simple renal cyst is a common finding encountered incidentally in an ultrasound done for another
purpose.

ere is no need to take any action in this situation, only follow-up and expectant management

Page - 1611
Internal Medicine - Nephrology

Question 61/96

Question #61

e following ECG changes are considered typical hyperkalemia changes except:

a. Flat P wave
b. Wide QRS complex
c. Peaked T wave
d. Sine wave
e. U wave

‫اﻹﺟﺎﺑﺔ ﻋﲆ اﻟﺼﻔﺤﺔ اﻟﺘﺎﻟﻴﺔ‬

Page - 1612
Internal Medicine - Nephrology - Electrolytes disturbances

Question 61/96

Question #61

e following ECG changes are considered typical hyperkalemia changes except:

a. Flat P wave
b. Wide QRS complex
c. Peaked T wave
d. Sine wave
e. U wave √

Description

U wave is an ECG finding of hypokalemia, not hyperkalemia.

Hyperkalemia-related cardiac conduction abnormalities on ECG are:

Low or absent P, Prolonged PR, Wide QRS, peaked T wave


Sine wave (severe hyperkalemia)

Cardiac arrhythmias

Page - 1613
Internal Medicine - Nephrology

Question 62/96

Question #62

Which of the following is the main cause of osmotic demyelination?

a. Rapid correction of hypocalcemia


b. Rapid correction of hypernatremia
c. Rapid correction of hyponatremia
d. Rapid correction of hypomagnesemia
e. Rapid correction of hyperkalemia

‫اﻹﺟﺎﺑﺔ ﻋﲆ اﻟﺼﻔﺤﺔ اﻟﺘﺎﻟﻴﺔ‬

Page - 1614
Internal Medicine - Nephrology - Electrolytes disturbances

Question 62/96

Question #62

Which of the following is the main cause of osmotic demyelination?

a. Rapid correction of hypocalcemia


b. Rapid correction of hypernatremia
c. Rapid correction of hyponatremia √
d. Rapid correction of hypomagnesemia
e. Rapid correction of hyperkalemia

Description

When hyponatremia is corrected too rapidly, the brain will have no time to recapture the organic
osmolytes; this will lead to osmotic demyelination of the brain.

e accurate correction rate in hyponatremia is no more than 8 – 12 mEq/l daily (slower correction “5
– 8 mEq/l daily” is indicated in chronic hyponatremia).

Page - 1615
Internal Medicine - Nephrology

Question 63/96

Question #63

A patient with heart failure is on spironolactone, beta-blockers, furosemide, and ACE inhibitors. e
patient has minimal symptoms at night and on exertion. His kidney function test is normal, his
serum potassium level is 5 mEq/L, and his blood pressure is 119/79. What is the most appropriate
action at this time?

a. Increase the dose of furosemide


b. Increase the dose of spironolactone
c. Increase the dose of ACE inhibitor
d. Reduce the dose of furosemide
e. Reduce the dose of beta-blockers

‫اﻹﺟﺎﺑﺔ ﻋﲆ اﻟﺼﻔﺤﺔ اﻟﺘﺎﻟﻴﺔ‬

Page - 1616
Internal Medicine - Nephrology - Electrolytes disturbances

Question 63/96

Question #63

A patient with heart failure is on spironolactone, beta-blockers, furosemide, and ACE inhibitors. e
patient has minimal symptoms at night and on exertion. His kidney function test is normal, his
serum potassium level is 5 mEq/L, and his blood pressure is 119/79. What is the most appropriate
action at this time?

a. Increase the dose of furosemide √


b. Increase the dose of spironolactone
c. Increase the dose of ACE inhibitor
d. Reduce the dose of furosemide
e. Reduce the dose of beta-blockers

Description

In the case of heart failure, it is common to control and balance serum potassium by using
spironolactone and furosemide dose adjustments

e patient’s potassium is more than 4.5 mEq/L, so increasing the dose of furosemide will reduce the
symptoms of heart failure and reduce the serum potassium to a safer level.

Increasing ACE inhibitors or spironolactone may lead to hyperkalemia in this case.

Page - 1617
Internal Medicine - Nephrology

Question 64/96

Question #64

You encountered a case of elevated serum potassium (7.3 mEq/L). What is the best next step in the
management of this patient?

a. Insulin and glucose


b. Hemodialysis
c. Calcium gluconate
d. Furosemide
e. Nebulized salbutamol

‫اﻹﺟﺎﺑﺔ ﻋﲆ اﻟﺼﻔﺤﺔ اﻟﺘﺎﻟﻴﺔ‬

Page - 1618
Internal Medicine - Nephrology - Electrolytes disturbances

Question 64/96

Question #64

You encountered a case of elevated serum potassium (7.3 mEq/L). What is the best next step in the
management of this patient?

a. Insulin and glucose


b. Hemodialysis
c. Calcium gluconate √
d. Furosemide
e. Nebulized salbutamol

Description

e priority here is to stabilize the cell membrane and prevent arrhythmia.

e following table demonstrates the anti-hyperkalemia protocol:

Page - 1619
Internal Medicine - Nephrology

Question 65/96

Question #65

On routine investigations, a patient is found to have serum potassium of 5.7 mEq/L. However, the
rest of his investigations and physical examination are normal. What is the most appropriate action
to take at this time?

a. Hemodialysis
b. Provide calcium gluconate
c. Provide insulin and dextrose
d. Start nebulized salbutamol
e. Repeat the blood sample without Torniquate

‫اﻹﺟﺎﺑﺔ ﻋﲆ اﻟﺼﻔﺤﺔ اﻟﺘﺎﻟﻴﺔ‬

Page - 1620
Internal Medicine - Nephrology - Electrolytes disturbances

Question 65/96

Question #65

On routine investigations, a patient is found to have serum potassium of 5.7 mEq/L. However, the
rest of his investigations and physical examination are normal. What is the most appropriate action
to take at this time?

a. Hemodialysis
b. Provide calcium gluconate
c. Provide insulin and dextrose
d. Start nebulized salbutamol
e. Repeat the blood sample without Torniquate √

Description

is patient is found to have hyperkalemia

e first step in this scenario is to repeat the blood sample without Torniquate to exclude the
possibility of pseudohyperkalemia.

Hemodialysis is indicated in refractory hyperkalemia of more than 7 mEq/L

Anti-hyperkalemia protocol should be initiated once hyperkalemia is confirmed or once ECG


changes of hyperkalemia are noted.

Page - 1621
Internal Medicine - Nephrology

Question 66/96

Question #66

A patient was admitted to the medical ward with severe vomiting and dehydration. His lab
investigations show serum sodium of 155mEq/L. What is the most appropriate fluid therapy at this
time?

a. Normal saline
b. Ringer lactate
c. Dextrose 5% water
d. Dextrose 10% water
e. Water restriction

‫اﻹﺟﺎﺑﺔ ﻋﲆ اﻟﺼﻔﺤﺔ اﻟﺘﺎﻟﻴﺔ‬

Page - 1622
Internal Medicine - Nephrology - Electrolytes disturbances

Question 66/96

Question #66

A patient was admitted to the medical ward with severe vomiting and dehydration. His lab
investigations show serum sodium of 155mEq/L. What is the most appropriate fluid therapy at this
time?

a. Normal saline √
b. Ringer lactate
c. Dextrose 5% water
d. Dextrose 10% water
e. Water restriction

Description

In the case of hypovolemic hypernatremia, you should start the treatment with isotonic 0.9% NaCl
to correct the water deficit, and then you can switch to hypotonic fluid (e.g., D5%W or 0.45% NS.

Ringer lactate is not a good choice in this condition as it will aggravate the metabolic alkalosis
resulting from the vomiting.

Page - 1623
Internal Medicine - Nephrology

Question 67/96

Question #67

In a patient with hyperkalemia, the anti-hyperkalemia protocol fails to achieve a response despite
being repeated appropriately. Which of the following is the most appropriate treatment?

a. Exchange transfusion
b. Check for magnesium level
c. Continuous anti-hyperkalemia protocol
d. Hemodialysis
e. Check for calcium level

‫اﻹﺟﺎﺑﺔ ﻋﲆ اﻟﺼﻔﺤﺔ اﻟﺘﺎﻟﻴﺔ‬

Page - 1624
Internal Medicine - Nephrology - Electrolytes disturbances

Question 67/96

Question #67

In a patient with hyperkalemia, the anti-hyperkalemia protocol fails to achieve a response despite
being repeated appropriately. Which of the following is the most appropriate treatment?

a. Exchange transfusion
b. Check for magnesium level
c. Continuous anti-hyperkalemia protocol
d. Hemodialysis √
e. Check for calcium level

Description

Hemodialysis is the treatment of choice for refractory hyperkalemia.

Serum magnesium levels should be checked for refractory hypocalcemia and refractory
hypokalemia (not hyperkalemia).

Page - 1625
Internal Medicine - Nephrology

Question 68/96

Question #68

Which of the following agents doesn’t used to reduce serum potassium in hyperkalemia?

a. Calcium gluconate
b. Insulin
c. Salbutamol
d. Calcium resonium
e. Furosemide

‫اﻹﺟﺎﺑﺔ ﻋﲆ اﻟﺼﻔﺤﺔ اﻟﺘﺎﻟﻴﺔ‬

Page - 1626
Internal Medicine - Nephrology - Electrolytes disturbances

Question 68/96

Question #68

Which of the following agents doesn’t used to reduce serum potassium in hyperkalemia?

a. Calcium gluconate √
b. Insulin
c. Salbutamol
d. Calcium resonium
e. Furosemide

Description

Calcium gluconate only stabilizes the cell membrane and prevents arrhythmia. It doesn’t reduce the
serum potassium level. Calcium gluconate is the first-line treatment for hyperkalemia.

e following table demonstrates the anti-hyperkalemia protocol:

Page - 1627
Internal Medicine - Nephrology

Question 69/96

Question #69

Which drug is considered the most eective in reducing serum potassium levels?

a. Furosemide
b. Calcium gluconate
c. Insulin
d. Calcium resonium
e. Sodium polystyrene sulfate

‫اﻹﺟﺎﺑﺔ ﻋﲆ اﻟﺼﻔﺤﺔ اﻟﺘﺎﻟﻴﺔ‬

Page - 1628
Internal Medicine - Nephrology - Electrolytes disturbances

Question 69/96

Question #69

Which drug is considered the most eective in reducing serum potassium levels?

a. Furosemide
b. Calcium gluconate
c. Insulin √
d. Calcium resonium
e. Sodium polystyrene sulfate

Description

Insulin is the strongest anti hyperkalemia to reduce the serum potassium level

It shis the extracellular potassium into the cells

e following table demonstrates the anti-hyperkalemia protocol:

Page - 1629
Internal Medicine - Nephrology

Question 70/96

Question #70

A 56-year-old woman presents to the emergency department with recurrent abdominal pain and
diarrhea. Laboratory results showed serum potassium is 2.5 mEq/L. An Electrocardiogram is
expected to be one of these:

a. Tented T wave
b. Reduced P wave amplitude
c. Prominent U wave
d. Short QT interval
e. Wide QRS complex

‫اﻹﺟﺎﺑﺔ ﻋﲆ اﻟﺼﻔﺤﺔ اﻟﺘﺎﻟﻴﺔ‬

Page - 1630
Internal Medicine - Nephrology - Electrolytes disturbances

Question 70/96

Question #70

A 56-year-old woman presents to the emergency department with recurrent abdominal pain and
diarrhea. Laboratory results showed serum potassium is 2.5 mEq/L. An Electrocardiogram is
expected to be one of these:

a. Tented T wave
b. Reduced P wave amplitude
c. Prominent U wave √
d. Short QT interval
e. Wide QRS complex

Description

is is a case of hypokalemia secondary to recurrent diarrhea. Regardeless the cause of diarrhea, the
question is focusing about the ECG changes that are related to hypokalemia.

ECG changes in hypokalemia include high P wave, Depressed ST, broad flat T wave, and prominent U
wave.

Page - 1631
Internal Medicine - Nephrology

Question 71/96

Question #71

Despite a high dose KCl through a central line, a 55-year-old male patient has hypokalemia that fails
to rise. What is the most appropriate action?

a. Hemodialysis
b. Continue KCl until serum potassium returns to normal
c. Check for serum magnesium level
d. Administrate combined oral and intravenous potassium
e. Check for urinary potassium loss

‫اﻹﺟﺎﺑﺔ ﻋﲆ اﻟﺼﻔﺤﺔ اﻟﺘﺎﻟﻴﺔ‬

Page - 1632
Internal Medicine - Nephrology - Electrolytes disturbances

Question 71/96

Question #71

Despite a high dose KCl through a central line, a 55-year-old male patient has hypokalemia that fails
to rise. What is the most appropriate action?

a. Hemodialysis
b. Continue KCl until serum potassium returns to normal
c. Check for serum magnesium level √
d. Administrate combined oral and intravenous potassium
e. Check for urinary potassium loss

Description

ere are magnesium-dependent potassium channels that open and spell potassium in urine in case
of hypomagnesemia.

Always check for and correct magnesium levels in the case of refractory hypokalemia.

Page - 1633
Internal Medicine - Nephrology

Question 72/96

Question #72

You are the doctor in charge at the emergency department, and the laboratory calls you to report
that the serum potassium level is 7.6 mEq/L. What is the most urgent step to perform at this time?

a. Urea and creatinine level


b. Repeat the sample
c. Electrocardiogram
d. Serum glucose level
e. Arterial blood gas

‫اﻹﺟﺎﺑﺔ ﻋﲆ اﻟﺼﻔﺤﺔ اﻟﺘﺎﻟﻴﺔ‬

Page - 1634
Internal Medicine - Nephrology - Electrolytes disturbances

Question 72/96

Question #72

You are the doctor in charge at the emergency department, and the laboratory calls you to report
that the serum potassium level is 7.6 mEq/L. What is the most urgent step to perform at this time?

a. Urea and creatinine level


b. Repeat the sample
c. Electrocardiogram √
d. Serum glucose level
e. Arterial blood gas

Description

ECG is the most urgent step to take in the case of hyperkalemia. If there are ECG changes of wide
QRS, peaked T wave, flat P, or even sine wave, you should start calcium gluconate and other anti-
hyperkalemia protocol.

Repeating the sample to exclude pseudohyperkalemia, checking for kidney function, assessing ABGs
to look for associated acidosis, and serum glucose levels are all indicated, but they are not as urgent
as ECG.

Page - 1635
Internal Medicine - Nephrology

Question 73/96

Question #73

Which of the following is the expected electrolyte disturbance in diabetes insipidus?

a. Hypernatremia
b. Hyponatremia
c. Hypermagnesemia
d. Hyperkalemia
e. Hypocalcemia

‫اﻹﺟﺎﺑﺔ ﻋﲆ اﻟﺼﻔﺤﺔ اﻟﺘﺎﻟﻴﺔ‬

Page - 1636
Internal Medicine - Nephrology - Electrolytes disturbances

Question 73/96

Question #73

Which of the following is the expected electrolyte disturbance in diabetes insipidus?

a. Hypernatremia √
b. Hyponatremia
c. Hypermagnesemia
d. Hyperkalemia
e. Hypocalcemia

Description

In diabetes insipidus (DI), there is a decreased secretion of antidiuretic hormone (ADH) from the
pituitary (cranial DI) or an insensitivity to antidiuretic hormone (nephrogenic DI)

In this case, the kidneys are unable to concentrate urine and reabsorb water.

is will lead to water loss and hypernatremia

Hypocalcemia and hypokalemia are possible causes of DI.

Page - 1637
Internal Medicine - Nephrology

Question 74/96

Question #74

e rapid correction of hypernatremia will result in which of the following condition?

a. Osmotic demyelination
b. Brain edema and nerve cell swelling
c. Arrhythmia
d. Rhabdomyolysis
e. Renal impairment

‫اﻹﺟﺎﺑﺔ ﻋﲆ اﻟﺼﻔﺤﺔ اﻟﺘﺎﻟﻴﺔ‬

Page - 1638
Internal Medicine - Nephrology - Electrolytes disturbances

Question 74/96

Question #74

e rapid correction of hypernatremia will result in which of the following condition?

a. Osmotic demyelination
b. Brain edema and nerve cell swelling √
c. Arrhythmia
d. Rhabdomyolysis
e. Renal impairment

Description

Hypernatremia should be corrected at a rate not more than 0.5 mEq/l per hour.

If hypernatremia is corrected rapidly, the brain tissue will have no time to lose organic osmolytes,
leading to brain edema and cellular swelling.

Page - 1639
Internal Medicine - Nephrology

Question 75/96

Question #75

A patient with heart failure was admitted as a case of digoxin toxicity. His lab investigations show
serum potassium of 7.2 mEq/L. What is the first-line treatment in this situation?

a. Calcium gluconate
b. Insulin and dextrose
c. Hemodialysis
d. Furosemide
e. Calcium resonium

‫اﻹﺟﺎﺑﺔ ﻋﲆ اﻟﺼﻔﺤﺔ اﻟﺘﺎﻟﻴﺔ‬

Page - 1640
Internal Medicine - Nephrology - Electrolytes disturbances

Question 75/96

Question #75

A patient with heart failure was admitted as a case of digoxin toxicity. His lab investigations show
serum potassium of 7.2 mEq/L. What is the first-line treatment in this situation?

a. Calcium gluconate
b. Insulin and dextrose √
c. Hemodialysis
d. Furosemide
e. Calcium resonium

Description

In the case of digoxin toxicity, it is better to avoid calcium gluconate because it may precipitate
arrhythmias.

Insulin is the most eective drug in lowering serum potassium.

e following table demonstrates the anti-hyperkalemia protocol:

Page - 1641
Internal Medicine - Nephrology

Question 76/96

Question #76

A 66-year-old male patient is treated for sepsis at the intensive care unit. On his 4th day aer
admission, you found serum sodium of 127 mEq/L, serum potassium of 4 mEq/L, and normal urea
and creatinine levels. What is the most likely cause of his hyponatremia?

a. Hypotension
b. Hypertension
c. Iatrogenic
d. Liver cirrhosis
e. Acute renal impairment

‫اﻹﺟﺎﺑﺔ ﻋﲆ اﻟﺼﻔﺤﺔ اﻟﺘﺎﻟﻴﺔ‬

Page - 1642
Internal Medicine - Nephrology - Electrolytes disturbances

Question 76/96

Question #76

A 66-year-old male patient is treated for sepsis at the intensive care unit. On his 4th day aer
admission, you found serum sodium of 127 mEq/L, serum potassium of 4 mEq/L, and normal urea
and creatinine levels. What is the most likely cause of his hyponatremia?

a. Hypotension
b. Hypertension
c. Iatrogenic √
d. Liver cirrhosis
e. Acute renal impairment

Description

is is a case of iatrogenic hyponatremia resulting from overtreatment with hypotonic saline

In the case of sepsis, the preferred fluid is normal saline 0.9% or Hartmann’s solution; they will
restore the fluid volume without iatrogenic dilution.

Page - 1643
Internal Medicine - Nephrology

Question 77/96

Question #77

A 55-year-old male patient was investigated for polyuria. His lab results show a serum glucose level
of 580 mg/dL. His serum sodium was 130 mEq/L. However, the patient is euvolemic and has a serum
osmolarity of 300 mOsm/L. You started insulin therapy for hyperglycemia. Which of the following is
the most appropriate treatment for hyponatremia?

a. Intravenous normal saline


b. Intravenous hypertonic saline
c. Water restriction
d. Intravenous Ringer Lactate
e. No need for treatment

‫اﻹﺟﺎﺑﺔ ﻋﲆ اﻟﺼﻔﺤﺔ اﻟﺘﺎﻟﻴﺔ‬

Page - 1644
Internal Medicine - Nephrology - Electrolytes disturbances

Question 77/96

Question #77

A 55-year-old male patient was investigated for polyuria. His lab results show a serum glucose level
of 580 mg/dL. His serum sodium was 130 mEq/L. However, the patient is euvolemic and has a serum
osmolarity of 300 mOsm/L. You started insulin therapy for hyperglycemia. Which of the following is
the most appropriate treatment for hyponatremia?

a. Intravenous normal saline


b. Intravenous hypertonic saline
c. Water restriction
d. Intravenous Ringer Lactate
e. No need for treatment √

Description

is is a case of pseudohyponatremia

e corrected sodium, in this case, is 138 mEq/L.

In this scenario, there is no need to treat hyponatremia, but you should correct the hyperglycemia
and the fluid state if disturbed.

Page - 1645
Internal Medicine - Nephrology

Question 78/96

Question #78

A 76-year-old diabetic patient has a blood pressure of 159/94. You exclude the presence of secondary
HTN. What is the best drug to be used for this patient?

a. Valsartan
b. Amlodipine
c. Metoprolol
d. Verapamil
e. iazide diuretics

‫اﻹﺟﺎﺑﺔ ﻋﲆ اﻟﺼﻔﺤﺔ اﻟﺘﺎﻟﻴﺔ‬

Page - 1646
Internal Medicine - Nephrology - Hypertension (HTN)

Question 78/96

Question #78

A 76-year-old diabetic patient has a blood pressure of 159/94. You exclude the presence of secondary
HTN. What is the best drug to be used for this patient?

a. Valsartan √
b. Amlodipine
c. Metoprolol
d. Verapamil
e. iazide diuretics

Description

For patients with DM and HTN, the best initial treatment is to start ACE inhibitors or ARBs because
they can protect the kidney from diabetic nephropathy

Page - 1647
Internal Medicine - Nephrology

Question 79/96

Question #79

A 55-year-old Afro-Caribbean female was diagnosed with HTN. Which of the following is considered
the most successful monotherapy for her hypertension?

a. Captopril
b. Hydrochlorothiazide
c. Metoprolol
d. Diltiazem
e. Valsartan

‫اﻹﺟﺎﺑﺔ ﻋﲆ اﻟﺼﻔﺤﺔ اﻟﺘﺎﻟﻴﺔ‬

Page - 1648
Internal Medicine - Nephrology - Hypertension (HTN)

Question 79/96

Question #79

A 55-year-old Afro-Caribbean female was diagnosed with HTN. Which of the following is considered
the most successful monotherapy for her hypertension?

a. Captopril
b. Hydrochlorothiazide
c. Metoprolol
d. Diltiazem √
e. Valsartan

Description

In the black Afro-Caribbean race, calcium channel blockers are the most eective in controlling HTN

It has been suggested that hypertension in blacks is not as angiotensin II-dependent as it appears to
be in Caucasians

Page - 1649
Internal Medicine - Nephrology

Question 80/96

Question #80

A 66-year-old male patient with DM, first-degree AV block, asthma, impotence, and gout has been
diagnosed recently with HTN. Which of the following is the best treatment option for his
condition?

a. Metoprolol
b. Verapamil
c. Hydrochlorothiazide
d. Valsartan
e. Carvedilol

‫اﻹﺟﺎﺑﺔ ﻋﲆ اﻟﺼﻔﺤﺔ اﻟﺘﺎﻟﻴﺔ‬

Page - 1650
Internal Medicine - Nephrology - Hypertension (HTN)

Question 80/96

Question #80

A 66-year-old male patient with DM, first-degree AV block, asthma, impotence, and gout has been
diagnosed recently with HTN. Which of the following is the best treatment option for his
condition?

a. Metoprolol
b. Verapamil
c. Hydrochlorothiazide
d. Valsartan √
e. Carvedilol

Description

ARBs (Valsartan) are safe and eective in this patient to control blood pressure, and it is also
eective in protecting the kidney and delaying adverse diabetic nephropathy.

e point here in this question is to combine the benefit of the blood pressure-lowering eect and
the safety of the drugs on the patient.

Beta-blockers (metoprolol and carvedilol) should be avoided here in this patient because of the first-
degree AV block and asthma

Verapamil should be avoided here because of the first-degree AV block

Hydrochlorothiazide will exacerbate acute gouty arthritis in patients with gout, so it should be
avoided in this patient

Page - 1651
Internal Medicine - Nephrology

Question 81/96

Question #81

A 66-year-old male patient known to have DM, gout, and IHD presents with elevated blood pressure.
Which of the following anti-hypertensive medications should be avoided in this patient?

a. Beta-blockers
b. ACE Inhibitors
c. Calcium channel blockers
d. iazide diuretics
e. Alpha-blockers

‫اﻹﺟﺎﺑﺔ ﻋﲆ اﻟﺼﻔﺤﺔ اﻟﺘﺎﻟﻴﺔ‬

Page - 1652
Internal Medicine - Nephrology - Hypertension (HTN)

Question 81/96

Question #81

A 66-year-old male patient known to have DM, gout, and IHD presents with elevated blood pressure.
Which of the following anti-hypertensive medications should be avoided in this patient?

a. Beta-blockers
b. ACE Inhibitors
c. Calcium channel blockers
d. iazide diuretics √
e. Alpha-blockers

Description

iazide diuretics will cause an elevation in the serum uric acid.

iazide and loop diuretics increase the reabsorption of uric acid in the proximal renal tubules. is
will exacerbate acute gouty arthritis.

e anti-hypertensive medication of choice in case of gout is losartan

Page - 1653
Internal Medicine - Nephrology

Question 82/96

Question #82

A 35-year-old male patient with HTN was started on Enalapril 10mg for 2 weeks. On the follow-up
visit, you noted an elevated creatinine level, his serum potassium is 5.6 mEq/L, and his blood
pressure is 145/90. What is the most likely diagnosis?

a. Essential HTN
b. Renal artery stenosis
c. Primary hyperaldosteronism
d. Pheochromocytoma
e. Cushing’s syndrome

‫اﻹﺟﺎﺑﺔ ﻋﲆ اﻟﺼﻔﺤﺔ اﻟﺘﺎﻟﻴﺔ‬

Page - 1654
Internal Medicine - Nephrology - Hypertension (HTN)

Question 82/96

Question #82

A 35-year-old male patient with HTN was started on Enalapril 10mg for 2 weeks. On the follow-up
visit, you noted an elevated creatinine level, his serum potassium is 5.6 mEq/L, and his blood
pressure is 145/90. What is the most likely diagnosis?

a. Essential HTN
b. Renal artery stenosis √
c. Primary hyperaldosteronism
d. Pheochromocytoma
e. Cushing’s syndrome

Description

Any patient with HTN and an elevated creatinine level of more than 30% aer one week of
administering the ACE inhibitor should be evaluated for renal artery stenosis.

Essential HTN is unlikely in young patients

Primary hyperaldosteronism presents with HTN and hypokalemia

Cushing’s syndrome presents with hypokalemia, central obesity, stria, hirsutism, HTN, and other
specific morphological features.

Pheochromocytoma usually presents with episodic HTN, sweating, and tremor.

Page - 1655
Internal Medicine - Nephrology

Question 83/96

Question #83

A 55-year-old male patient was diagnosed with essential HTN 3 months ago. His blood pressure was
141/90, and he has been given Enalapril 5mg daily since then. Today he is asymptomatic, and his
blood pressure is 119/79 mmHg. What would you do next?

a. Continue same treatment


b. Switch to another anti-HTN agent
c. Stop his medications as he is now free of HTN
d. Order an echocardiography
e. Reduce the dose of Enalapril by 50%

‫اﻹﺟﺎﺑﺔ ﻋﲆ اﻟﺼﻔﺤﺔ اﻟﺘﺎﻟﻴﺔ‬

Page - 1656
Internal Medicine - Nephrology - Hypertension (HTN)

Question 83/96

Question #83

A 55-year-old male patient was diagnosed with essential HTN 3 months ago. His blood pressure was
141/90, and he has been given Enalapril 5mg daily since then. Today he is asymptomatic, and his
blood pressure is 119/79 mmHg. What would you do next?

a. Continue same treatment √


b. Switch to another anti-HTN agent
c. Stop his medications as he is now free of HTN
d. Order an echocardiography
e. Reduce the dose of Enalapril by 50%

Description

is patient has essential HTN, and his blood pressure is now controlled with ACE inhibitors with no
side eects and no new complaints. So, his medication should be kept at the same dose.

Reduction, stopping, or changing the drug may lead to an uncontrolled blood pressure

Echocardiogram is not indicated at this time

Page - 1657
Internal Medicine - Nephrology

Question 84/96

Question #84

A 76-year-old male patient has a blood pressure of 159/94. He is on Amlodipine 10mg daily but still
uncontrolled. What is the best drug to be used for this patient?

a. Enalapril
b. Verapamil
c. Metoprolol
d. Spironolactone
e. iazide diuretics

‫اﻹﺟﺎﺑﺔ ﻋﲆ اﻟﺼﻔﺤﺔ اﻟﺘﺎﻟﻴﺔ‬

Page - 1658
Internal Medicine - Nephrology - Hypertension (HTN)

Question 84/96

Question #84

A 76-year-old male patient has a blood pressure of 159/94. He is on Amlodipine 10mg daily but still
uncontrolled. What is the best drug to be used for this patient?

a. Enalapril √
b. Verapamil
c. Metoprolol
d. Spironolactone
e. iazide diuretics

Description

e following picture shows the treatment approach for patients with HTN:

Page - 1659
Internal Medicine - Nephrology

Question 85/96

Question #85

A 53-year-old male patient was diagnosed with HTN. He is a known case of DM and asthma. Which
medications are not allowed to be used in this patient?

a. Atenolol
b. Metoprolol
c. Esmolol
d. Bisoprolol
e. Labetalol

‫اﻹﺟﺎﺑﺔ ﻋﲆ اﻟﺼﻔﺤﺔ اﻟﺘﺎﻟﻴﺔ‬

Page - 1660
Internal Medicine - Nephrology - Hypertension (HTN)

Question 85/96

Question #85

A 53-year-old male patient was diagnosed with HTN. He is a known case of DM and asthma. Which
medications are not allowed to be used in this patient?

a. Atenolol
b. Metoprolol
c. Esmolol
d. Bisoprolol
e. Labetalol √

Description

Beta-blockers are divided into two types, selective beta 1 (only block the receptors on the heart) and
non-selective (block beta receptors in the heart and lungs, leading to bradycardia and
bronchospasms)

Non-selective beta-blockers are not used in asthma as they exacerbate symptoms.

Examples of selective beta-blockers are: Acebutolol, Atenolol, Betaxolol, Bisoprolol, Esmolol,


Metoprolol, and Nebivolol

Labetalol is non-selective and is contraindicated to be used in asthma patients

Page - 1661
Internal Medicine - Nephrology

Question 86/96

Question #86

A 56-year-old male patient has had a persistent cough for the last 4 months. His physical
examination and lab investigations are unremarkable except for a blood pressure of 150/95 mmHg.
he is taking enalapril 5mg daily. Which of the following is the most appropriate at this time?

a. Increase the dose of Enalapril


b. Add calcium channel blockers
c. Add ARB
d. Switch to ARBs
e. Switch to another ACE inhibitor

‫اﻹﺟﺎﺑﺔ ﻋﲆ اﻟﺼﻔﺤﺔ اﻟﺘﺎﻟﻴﺔ‬

Page - 1662
Internal Medicine - Nephrology - Hypertension (HTN)

Question 86/96

Question #86

A 56-year-old male patient has had a persistent cough for the last 4 months. His physical
examination and lab investigations are unremarkable except for a blood pressure of 150/95 mmHg.
he is taking enalapril 5mg daily. Which of the following is the most appropriate at this time?

a. Increase the dose of Enalapril


b. Add calcium channel blockers
c. Add ARB
d. Switch to ARBs √
e. Switch to another ACE inhibitor

Description

Because of high levels of bradykinin, cough is a common side eect of ACE inhibitors.

Patients who develop coughs due to ACE inhibitors should receive an ARB instead because ARBs
don’t cause coughs.

Page - 1663
Internal Medicine - Nephrology

Question 87/96

Question #87

A 76-year-old male patient has a blood pressure of 159/94. You exclude the presence of secondary
HTN. What is the best drug to be used for this patient?

a. Enalapril
b. Amlodipine
c. Metoprolol
d. Spironolactone
e. iazide diuretics

‫اﻹﺟﺎﺑﺔ ﻋﲆ اﻟﺼﻔﺤﺔ اﻟﺘﺎﻟﻴﺔ‬

Page - 1664
Internal Medicine - Nephrology - Hypertension (HTN)

Question 87/96

Question #87

A 76-year-old male patient has a blood pressure of 159/94. You exclude the presence of secondary
HTN. What is the best drug to be used for this patient?

a. Enalapril
b. Amlodipine √
c. Metoprolol
d. Spironolactone
e. iazide diuretics

Description

Amlodipine is the best treatment for HTN in elderly and black patients

Other drugs can be used as well, but CCBs are the best for this age group

Page - 1665
Internal Medicine - Nephrology

Question 88/96

Question #88

According to the last JNC guidelines, the target blood pressure for a diabetic 65-year-old patient
with essential HTN is less than:

a. 130/80 mmHg
b. 135/85 mmHg
c. 120/80 mmHg
d. 140/90 mmHg
e. 150/90 mmHg

‫اﻹﺟﺎﺑﺔ ﻋﲆ اﻟﺼﻔﺤﺔ اﻟﺘﺎﻟﻴﺔ‬

Page - 1666
Internal Medicine - Nephrology - Hypertension (HTN)

Question 88/96

Question #88

According to the last JNC guidelines, the target blood pressure for a diabetic 65-year-old patient
with essential HTN is less than:

a. 130/80 mmHg
b. 135/85 mmHg
c. 120/80 mmHg
d. 140/90 mmHg √
e. 150/90 mmHg

Description

According to the very last JNC guidelines for HTN, for any hypertensive patient who is less than 60-
year-old, the target blood pressure is less than 140/90 mmHg

For patients who are more than 60-year-old:

If the patient has DM or CKD, the target will be < 140/90

If the patient has no DM or DCD, the target is < 150/90

Page - 1667
Internal Medicine - Nephrology

Question 89/96

Question #89

A 76-year-old male patient has a blood pressure of 159/94. He is on Amlodipine 10mg daily, Natrilix
1.5 mg daily, and Enalapril 20mg bid but still uncontrolled. His potassium level is 4.2 mEq/L. What is
the best drug to be used for this patient?

a. Valsartan
b. Increase the dose of iazide like diuretic
c. Metoprolol
d. Spironolactone
e. Loop diuretic

‫اﻹﺟﺎﺑﺔ ﻋﲆ اﻟﺼﻔﺤﺔ اﻟﺘﺎﻟﻴﺔ‬

Page - 1668
Internal Medicine - Nephrology - Hypertension (HTN)

Question 89/96

Question #89

A 76-year-old male patient has a blood pressure of 159/94. He is on Amlodipine 10mg daily, Natrilix
1.5 mg daily, and Enalapril 20mg bid but still uncontrolled. His potassium level is 4.2 mEq/L. What is
the best drug to be used for this patient?

a. Valsartan
b. Increase the dose of iazide like diuretic
c. Metoprolol
d. Spironolactone √
e. Loop diuretic

Description

e following picture shows the treatment approach for patients with HTN:

Page - 1669
Internal Medicine - Nephrology

Question 90/96

Question #90

A 70-year-old male patient was admitted to the hospital due to an ischemic stroke. His blood
pressure is 191/115 mmHg. What should you do regarding his blood pressure?

a. Only monitoring
b. Start nitroprusside
c. Start labetalol
d. Start nifedipine
e. Start hydralazine

‫اﻹﺟﺎﺑﺔ ﻋﲆ اﻟﺼﻔﺤﺔ اﻟﺘﺎﻟﻴﺔ‬

Page - 1670
Internal Medicine - Nephrology - Hypertension (HTN)

Question 90/96

Question #90

A 70-year-old male patient was admitted to the hospital due to an ischemic stroke. His blood
pressure is 191/115 mmHg. What should you do regarding his blood pressure?

a. Only monitoring √
b. Start nitroprusside
c. Start labetalol
d. Start nifedipine
e. Start hydralazine

Description

Systolic blood pressure of less than 200 mmHg and diastolic one of less than 120 mmHg indicates
only monitoring of the patient without intervention

High blood pressure in stroke patients is thought to be a protective mechanism that increases
cerebral perfusion.

Reduction of blood pressure, in this case, will exacerbate the damage to the brain and increase
mortality.

Page - 1671
Internal Medicine - Nephrology

Question 91/96

Question #91

A previously healthy 25-year-old male patient presents with a blood pressure of 160/92 on routine
examination. What would you do next?

a. Start ACE inhibitors


b. Start Calcium channel blockers
c. Investigate for secondary HTN
d. Obtain additional BP reading
e. Reassurance

‫اﻹﺟﺎﺑﺔ ﻋﲆ اﻟﺼﻔﺤﺔ اﻟﺘﺎﻟﻴﺔ‬

Page - 1672
Internal Medicine - Nephrology - Hypertension (HTN)

Question 91/96

Question #91

A previously healthy 25-year-old male patient presents with a blood pressure of 160/92 on routine
examination. What would you do next?

a. Start ACE inhibitors


b. Start Calcium channel blockers
c. Investigate for secondary HTN
d. Obtain additional BP reading √
e. Reassurance

Description

Two readings of ≥ 14/90 are required to make the diagnosis of HTN

is is a young patient who is unlikely to have chronic HTN, so you have to obtain another reading to
confirm the diagnosis

Page - 1673
Internal Medicine - Nephrology

Question 92/96

Question #92

A 65-year-old male patient was found to have a blood pressure ranging from 145/90 to 154/99 for
the last month. His medications include full dose valsartan, hydrochlorothiazide, and amlodipine.
e following are possible diagnoses except:

a. Noncompliance with medications


b. High sodium diet
c. Presence of secondary HTN
d. Expired medications
e. Valsartan overdose

‫اﻹﺟﺎﺑﺔ ﻋﲆ اﻟﺼﻔﺤﺔ اﻟﺘﺎﻟﻴﺔ‬

Page - 1674
Internal Medicine - Nephrology - Hypertension (HTN)

Question 92/96

Question #92

A 65-year-old male patient was found to have a blood pressure ranging from 145/90 to 154/99 for
the last month. His medications include full dose valsartan, hydrochlorothiazide, and amlodipine.
e following are possible diagnoses except:

a. Noncompliance with medications


b. High sodium diet
c. Presence of secondary HTN
d. Expired medications
e. Valsartan overdose √

Description

Resistant or refractory hypertension is defined as a blood pressure of 140/90mmHg, despite


adherence to treatment with full doses of at least three anti-hypertensive medications, including a
diuretic.

Valsartan overdose will cause hypotension, not hypertension

e other mentioned choices in the question are well-known causes of refractory HTN.

Page - 1675
Internal Medicine - Nephrology

Question 93/96

Question #93

A 55-year-old male patient has a history of mildly elevated blood pressure recorded several times at
home during the past month. He is asymptomatic, and his blood pressure today is 181/93 mmHg. On
examination, there is a displaced cardiac apex to the le with heaving apex beats. What would you
do next?

a. Repeat blood pressure on a separate day


b. Get lab tests for the patient
c. Perform an exercise stress test
d. Start daily oral furosemide
e. Reassurance

‫اﻹﺟﺎﺑﺔ ﻋﲆ اﻟﺼﻔﺤﺔ اﻟﺘﺎﻟﻴﺔ‬

Page - 1676
Internal Medicine - Nephrology - Hypertension (HTN)

Question 93/96

Question #93

A 55-year-old male patient has a history of mildly elevated blood pressure recorded several times at
home during the past month. He is asymptomatic, and his blood pressure today is 181/93 mmHg. On
examination, there is a displaced cardiac apex to the le with heaving apex beats. What would you
do next?

a. Repeat blood pressure on a separate day


b. Get lab tests for the patient √
c. Perform an exercise stress test
d. Start daily oral furosemide
e. Reassurance

Description

is patient is suspected of having chronic HTN

e displaced and heaving apex beat is suggestive of le ventricular hypertrophy.

Lab investigations, ECG, and echocardiogram should be ordered, and the patient should start on
lifestyle modifications and anti-hypertensive medications.

Requesting an additional reading on this patient is inappropriate because he has severe HTN and
signs of LV hypertrophy.

Page - 1677
Internal Medicine - Nephrology

Question 94/96

Question #94

A 76-year-old male patient has a blood pressure of 159/94. He is on Amlodipine 10mg daily, Enalapril
20mg bid but still uncontrolled. What is the best drug to be used for this patient?

a. Valsartan
b. iazide like diuretic
c. Metoprolol
d. Spironolactone
e. Loop diuretic

‫اﻹﺟﺎﺑﺔ ﻋﲆ اﻟﺼﻔﺤﺔ اﻟﺘﺎﻟﻴﺔ‬

Page - 1678
Internal Medicine - Nephrology - Hypertension (HTN)

Question 94/96

Question #94

A 76-year-old male patient has a blood pressure of 159/94. He is on Amlodipine 10mg daily, Enalapril
20mg bid but still uncontrolled. What is the best drug to be used for this patient?

a. Valsartan
b. iazide like diuretic √
c. Metoprolol
d. Spironolactone
e. Loop diuretic

Description

e following picture shows the treatment approach for patients with HTN:

Page - 1679
Internal Medicine - Nephrology

Question 95/96

Question #95

A 66-year-old male presents with stage I HTN. He has obesity, dyslipidemia, and diabetes. What is
the best medication to prescribe for this patient?

a. iazide diuretics
b. Metoprolol
c. Enalapril
d. Methyldopa
e. Spironolactone

‫اﻹﺟﺎﺑﺔ ﻋﲆ اﻟﺼﻔﺤﺔ اﻟﺘﺎﻟﻴﺔ‬

Page - 1680
Internal Medicine - Nephrology - Hypertension (HTN)

Question 95/96

Question #95

A 66-year-old male presents with stage I HTN. He has obesity, dyslipidemia, and diabetes. What is
the best medication to prescribe for this patient?

a. iazide diuretics
b. Metoprolol
c. Enalapril √
d. Methyldopa
e. Spironolactone

Description

ACE inhibitors are the first line used in patients with HTN and DM.

ey inhibit the angiotensin-converting enzyme, which converts angiotensin I to angiotensin II.

is mechanism will reduce the pressure inside the nephron leading to a protective mechanism
against diabetic nephropathy.

Page - 1681
Internal Medicine - Nephrology

Question 96/96

Question #96

Which of the following recommendations is accurate for measuring blood pressure in adults?

a. Resting for at least 5 minutes will reduce the false high readings
b. A small cu can cause a false high reading
c. e patient should avoid smoking and coee drinking ½ hour before blood pressure measurement
d. Two readings in each arm should be obtained
e. All of the above

‫اﻹﺟﺎﺑﺔ ﻋﲆ اﻟﺼﻔﺤﺔ اﻟﺘﺎﻟﻴﺔ‬

Page - 1682
Internal Medicine - Nephrology - Hypertension (HTN)

Question 96/96

Question #96

Which of the following recommendations is accurate for measuring blood pressure in adults?

a. Resting for at least 5 minutes will reduce the false high readings
b. A small cu can cause a false high reading
c. e patient should avoid smoking and coee drinking ½ hour before blood pressure measurement
d. Two readings in each arm should be obtained
e. All of the above √

Description

To accurately measure blood pressure in adults:

e patient should sit on a chair for at least 5 minutes to reduce the incidence of false high
readings
Always choose an accurately sized cu (a small cu can cause false high reading, while a large
cu can cause false low reading)
e patient should not smoke or drink coee for ½ hour before blood pressure measurement

Obtain readings from both arms, two readings for each arm

Page - 1683
Neurology

Page - 1684
Neurology

‫ﻣﻮاﺿﻴﻊ اﻷﺳﺌﻠﺔ وأﻋﺪادﻫﺎ‬

1) Acute stroke => 3 Questions


2) Alcoholic Encephalopathies => 2 Questions
3) Aphasia => 1 Questions
4) Bell’s palsy => 2 Questions
5) Benign Paroxysmal Positional Vertigo (BPPV) => 2 Questions
6) CNS infections => 6 Questions
7) Cluster headache => 3 Questions
8) Dementia => 15 Questions
9) Epilepsy, Seizure, and Status epilepticus => 10 Questions
10) Essential Tremor => 2 Questions
11) Guillain-Barré syndrome (GBS) => 2 Questions
12) Headaches => 1 Questions
13) Horner syndrome => 3 Questions
14) Introduction to Neurology => 1 Questions
15) Meniere’s Disease => 2 Questions
16) Migraine Headache => 3 Questions
17) Multiple Sclerosis (MS) => 3 Questions
18) Myasthenia Gravis (MG) => 3 Questions
19) Organophosphate poisoning => 2 Questions
20) Parkinson’s Disease (PD) => 6 Questions
21) Postherpetic Neuralgia => 1 Questions
22) Pseudotumor Cerebri => 1 Questions
23) Tension-type headache => 1 Questions
24) e Cranial Nerves => 1 Questions
25) underclap headache => 3 Questions
26) Trigeminal Neuralgia => 2 Questions
27) Upper vs. Lower Neuron lesion => 1 Questions

Page - 1685
Internal Medicine - Neurology

Question 1/82

Question #1

A 68-year-old male patient presented aer 1 week of a brief attack of le-side weakness that lasted
for 1 hour. Today the patent is asymptomatic, with a normal neurological examination. His vital
signs are normal, and his ECG shows a normal sinus rhythm. What is the most important
investigation to do at this time?

a. Brain MRI
b. Echocardiogram
c. Cerebral angiography
d. Chest x-ray
e. Carotid artery ultrasound

‫اﻹﺟﺎﺑﺔ ﻋﲆ اﻟﺼﻔﺤﺔ اﻟﺘﺎﻟﻴﺔ‬

Page - 1686
Internal Medicine - Neurology - Acute stroke

Question 1/82

Question #1

A 68-year-old male patient presented aer 1 week of a brief attack of le-side weakness that lasted
for 1 hour. Today the patent is asymptomatic, with a normal neurological examination. His vital
signs are normal, and his ECG shows a normal sinus rhythm. What is the most important
investigation to do at this time?

a. Brain MRI
b. Echocardiogram
c. Cerebral angiography
d. Chest x-ray
e. Carotid artery ultrasound √

Description

is is a case of TIA; a carotid artery stenosis should be ruled out by ultrasound

If blockage of the blood vessel is reversed while neurons are still viable (< 24 hours), this condition is
called TIA, but if the blockage is prolonged (> 24 hours), infarction and permanent damage with
resulting permanent symptoms can occur

Page - 1687
Internal Medicine - Neurology

Question 2/82

Question #2

A 53-year-old male is a known case of paroxysmal atrial fibrillation. He has no history of HTN, DM,
HF, TIA, or stroke. What is the most appropriate management for this patient at this point?

a. Observation
b. Aspirin
c. Warfarin, target INR of 1.5-2.5
d. Warfarin, target INR of 2.0-3.0
e. Warfarin, target INR of 2.5-3.5

‫اﻹﺟﺎﺑﺔ ﻋﲆ اﻟﺼﻔﺤﺔ اﻟﺘﺎﻟﻴﺔ‬

Page - 1688
Internal Medicine - Neurology - Acute stroke

Question 2/82

Question #2

A 53-year-old male is a known case of paroxysmal atrial fibrillation. He has no history of HTN, DM,
HF, TIA, or stroke. What is the most appropriate management for this patient at this point?

a. Observation √
b. Aspirin
c. Warfarin, target INR of 1.5-2.5
d. Warfarin, target INR of 2.0-3.0
e. Warfarin, target INR of 2.5-3.5

Description

CHA2DS2VAS score is the most appropriate tool to assess the risk of stroke in atrial fibrillation
patients.

Our patient’s score is zero, so he needs no anticoagulation.

e following table demonstrates the CHA2DS2VAS and its interpretation.

Page - 1689
Internal Medicine - Neurology

Question 3/82

Question #3

A 55-year-old male patient with a known case of HTN and DM presented with right-side weakness in
his upper and lower limbs. His reflexes are exaggerated, and his power is 2/5 on the same side. In
addition, reduced temperature and vibratory sensation are present. What is the best next step in
managing this patient if he is conscious and has stable vital signs?

a. Start Aspirin
b. Observation
c. Get a CT scan without contrast
d. Start IV heparin
e. rombolysis

‫اﻹﺟﺎﺑﺔ ﻋﲆ اﻟﺼﻔﺤﺔ اﻟﺘﺎﻟﻴﺔ‬

Page - 1690
Internal Medicine - Neurology - Acute stroke

Question 3/82

Question #3

A 55-year-old male patient with a known case of HTN and DM presented with right-side weakness in
his upper and lower limbs. His reflexes are exaggerated, and his power is 2/5 on the same side. In
addition, reduced temperature and vibratory sensation are present. What is the best next step in
managing this patient if he is conscious and has stable vital signs?

a. Start Aspirin
b. Observation
c. Get a CT scan without contrast √
d. Start IV heparin
e. rombolysis

Description

e first step is to get a CT scan to exclude intracranial hemorrhage.

Acute stroke:

Acute stroke is a rapid appearance of a neurological deficit of brain function due to the death
of brain tissue
Ischemic strokes are more common than hemorrhagic (85% of cases)
A brain CT scan is the most appropriate tool to quickly dierentiate ischemic from
hemorrhagic types of strokes.
Ischemic stroke less than 3 – 4.5 hours since onset is treated with thrombolysis (can increase
the risk of hemorrhagic transformation)

Page - 1691
Internal Medicine - Neurology

Question 4/82

Question #4

A 46-year-old man with a long history of alcohol use presented with confusion, ophthalmoplegia,
and horizontal nystagmus. What is the initial treatment of choice for this condition?

a. Start Aspirin
b. iamin injection
c. Dextrose infusion
d. Naloxone IV
e. Refer to a neurologist

‫اﻹﺟﺎﺑﺔ ﻋﲆ اﻟﺼﻔﺤﺔ اﻟﺘﺎﻟﻴﺔ‬

Page - 1692
Internal Medicine - Neurology - Alcoholic Encephalopathies

Question 4/82

Question #4

A 46-year-old man with a long history of alcohol use presented with confusion, ophthalmoplegia,
and horizontal nystagmus. What is the initial treatment of choice for this condition?

a. Start Aspirin
b. iamin injection √
c. Dextrose infusion
d. Naloxone IV
e. Refer to a neurologist

Description

iamin (Vitamin B1) is the initial treatment of choice for Wernicke’s encephalopathy.

Wernicke's encephalopathy:

Seen in alcoholism due to iamine (Vitamin B1) deficiency


Symptoms precipitated by glucose infusion in a patient with alcoholism (B1 is responsible for
glucose metabolism)
Symptoms include confusion, ataxia, ophthalmoplegia, memory impairment
It is treated by thiamine injection; avoid glucose infusion before thiamine

Page - 1693
Internal Medicine - Neurology

Question 5/82

Question #5

A 46-year-old man with a long history of alcohol use presented with confusion, ophthalmoplegia,
and horizontal nystagmus. Which is the most likely diagnosis?

a. Alcohol intoxication
b. Delirium tremens
c. Wernicke’s encephalopathy
d. Hysterical conversion disorder
e. Cerebrovascular accident (CVA)

‫اﻹﺟﺎﺑﺔ ﻋﲆ اﻟﺼﻔﺤﺔ اﻟﺘﺎﻟﻴﺔ‬

Page - 1694
Internal Medicine - Neurology - Alcoholic Encephalopathies

Question 5/82

Question #5

A 46-year-old man with a long history of alcohol use presented with confusion, ophthalmoplegia,
and horizontal nystagmus. Which is the most likely diagnosis?

a. Alcohol intoxication
b. Delirium tremens
c. Wernicke’s encephalopathy √
d. Hysterical conversion disorder
e. Cerebrovascular accident (CVA)

Description

Acute onset of confusion, nystagmus, partial ophthalmoplegia and ataxia are the main presentation
of Wernicke’s encephalopathy

Wernicke's encephalopathy:

Seen in alcoholism due to iamine (Vitamin B1) deficiency


Symptoms precipitated by glucose infusion in a patient with alcoholism (B1 is responsible for
glucose metabolism)
Symptoms include confusion, ataxia, ophthalmoplegia, memory impairment
It is treated by thiamine injection; avoid glucose infusion before thiamine

Page - 1695
Internal Medicine - Neurology

Question 6/82

Question #6

A 67-year-old male patient with a known case of DM, HTN, presented with right-side weakness and
inability to speak. e patient can obey commands but can’t repeat phrases aer you. What is the
best description of his condition?

a. Receptive aphasia
b. Expressive aphasia
c. Global aphasia
d. Conductive aphasia
e. Anomic aphasia

‫اﻹﺟﺎﺑﺔ ﻋﲆ اﻟﺼﻔﺤﺔ اﻟﺘﺎﻟﻴﺔ‬

Page - 1696
Internal Medicine - Neurology - Aphasia

Question 6/82

Question #6

A 67-year-old male patient with a known case of DM, HTN, presented with right-side weakness and
inability to speak. e patient can obey commands but can’t repeat phrases aer you. What is the
best description of his condition?

a. Receptive aphasia
b. Expressive aphasia √
c. Global aphasia
d. Conductive aphasia
e. Anomic aphasia

Description

Broca’s area is the most likely aected in this case

Types of aphasia:

Wernicke’s (receptive) aphasia: Lesions result in sentences that make no sense and word
substitution, but speech remains fluent
Broca’s (expressive) aphasia: Speech is non-fluent and labored
Conduction aphasia: Speech is fluent, but repetition is poor. Aware of the errors they are
making
Global aphasia: large lesion aecting all 3 of the above areas resulting in severe expressive and
receptive aphasia

Page - 1697
Internal Medicine - Neurology

Question 7/82

Question #7

Two weeks aer a viral illness, a 30-year-old man developed a dropping of his mouth and
incomplete closure of his le eye. His smile is asymmetrical, and his examination is otherwise
normal. What is the most likely diagnosis?

a. Bell’s palsy.
b. Cerebrovascular accident (CVA)
c. Trigeminal nerve palsy
d. ird nerve palsy
e. Horner’s syndrome

‫اﻹﺟﺎﺑﺔ ﻋﲆ اﻟﺼﻔﺤﺔ اﻟﺘﺎﻟﻴﺔ‬

Page - 1698
Internal Medicine - Neurology - Bell’s palsy

Question 7/82

Question #7

Two weeks aer a viral illness, a 30-year-old man developed a dropping of his mouth and
incomplete closure of his le eye. His smile is asymmetrical, and his examination is otherwise
normal. What is the most likely diagnosis?

a. Bell’s palsy. √
b. Cerebrovascular accident (CVA)
c. Trigeminal nerve palsy
d. ird nerve palsy
e. Horner’s syndrome

Description

e weakness aecting the face’s upper and lower parts is characteristic of Bell’s palsy.

Seventh nerve palsy (Bell’s palsy):

Most are idiopathic, but Lyme disease, viral infection, sarcoidosis, and tumors may be a cause

Clinical features include paralysis of the entire side of the face (upper and lower parts),
diculty closing eyes, absent wrinkles on the same side of the forehead, Hyperacusis, and
taste disturbances
e diagnosis is clinical, and no need for special testing but NCS and EMG are the most
accurate
e best initial therapy is prednisolone, but 60% recover fully without treatment

Eye care and physiotherapy should be considered during the recovery.

Page - 1699
Internal Medicine - Neurology

Question 8/82

Question #8

A 22-year-old male patient presented with ocular irritation and a weird unilateral smile noted by his
friend. In addition, the patient cannot pu up the right cheek or frown his face. What is the most
appropriate next step?

a. Oral prednisolone, artificial tears, and follow-up.


b. Acyclovir, Vitamin B12 injections, and prednisolone
c. Brain CT scan
d. Schirmer blotting test
e. Start oral aspirin and Statin

‫اﻹﺟﺎﺑﺔ ﻋﲆ اﻟﺼﻔﺤﺔ اﻟﺘﺎﻟﻴﺔ‬

Page - 1700
Internal Medicine - Neurology - Bell’s palsy

Question 8/82

Question #8

A 22-year-old male patient presented with ocular irritation and a weird unilateral smile noted by his
friend. In addition, the patient cannot pu up the right cheek or frown his face. What is the most
appropriate next step?

a. Oral prednisolone, artificial tears, and follow-up. √


b. Acyclovir, Vitamin B12 injections, and prednisolone
c. Brain CT scan
d. Schirmer blotting test
e. Start oral aspirin and Statin

Description

In Bell’s palsy oral prednisolone is the primary treatment. In addition, artificial tears are used to
protect the eye.

Seventh nerve palsy (Bell’s palsy):

Most are idiopathic, but Lyme disease, viral infection, sarcoidosis, and tumors may be a cause

Clinical features include paralysis of the entire side of the face (upper and lower parts),
diculty closing eyes, absent wrinkles on the same side of the forehead, Hyperacusis, and
taste disturbances
e diagnosis is clinical, and no need for special testing but NCS and EMG are the most
accurate
e best initial therapy is prednisolone, but 60% recover fully without treatment

Eye care and physiotherapy should be considered during the recovery.

Page - 1701
Internal Medicine - Neurology

Question 9/82

Question #9

A previously healthy 45-year-old male came with a spinning sensation upon changes in his position.
Which one of the following is characteristic of benign positional vertigo (BPPV)?

a. Associated sensorineural hearing loss


b. Presence of tinnitus
c. Aggravated by turning the head
d. Preceded by headache
e. Vertigo lasts for hours

‫اﻹﺟﺎﺑﺔ ﻋﲆ اﻟﺼﻔﺤﺔ اﻟﺘﺎﻟﻴﺔ‬

Page - 1702
Internal Medicine - Neurology - Benign Paroxysmal Positional Vertigo (BPPV)

Question 9/82

Question #9

A previously healthy 45-year-old male came with a spinning sensation upon changes in his position.
Which one of the following is characteristic of benign positional vertigo (BPPV)?

a. Associated sensorineural hearing loss


b. Presence of tinnitus
c. Aggravated by turning the head √
d. Preceded by headache
e. Vertigo lasts for hours

Description

In BPPV, no headache, no tinnitus, no hearing loss, and the attack lasts for 10-20 seconds only

Benign Paroxysmal Positional Vertigo (BPPV):

It is a spinning sensation upon changes in the position of the head.


Each episode lasts less than 1 minute
Vertigo (spinning dizziness) is the main feature; it is:
Paroxysmal (Sudden with a short duration)
Positional: (Induced by a change in head position
Associated with nausea and vomiting

Page - 1703
Internal Medicine - Neurology

Question 10/82

Question #10

Dix–Hallpike test is used in the diagnosis of:

a. Meniere’s disease
b. Benign positional vertigo
c. Sensorineural hearing loss
d. Conductive hearing loss
e. Visual field defect

‫اﻹﺟﺎﺑﺔ ﻋﲆ اﻟﺼﻔﺤﺔ اﻟﺘﺎﻟﻴﺔ‬

Page - 1704
Internal Medicine - Neurology - Benign Paroxysmal Positional Vertigo (BPPV)

Question 10/82

Question #10

Dix–Hallpike test is used in the diagnosis of:

a. Meniere’s disease
b. Benign positional vertigo √
c. Sensorineural hearing loss
d. Conductive hearing loss
e. Visual field defect

Description

It is used in BPPV to assess the posterior semicircular canal

Dix–Hallpike test:

A test used to assess the posterior semicircular canal.


Lower your patient quickly to a supine position with the neck extended
Nystagmus will start if the test is positive

Benign Paroxysmal Positional Vertigo (BPPV):

It is a spinning sensation upon changes in the position of the head.


Each episode lasts less than 1 minute
Vertigo (spinning dizziness) is the main feature; it is:
Paroxysmal (Sudden with a short duration)
Positional: (Induced by a change in head position
Associated with nausea and vomiting

Page - 1705
Internal Medicine - Neurology

Question 11/82

Question #11

A 23-year-old male patient presented with headache, fever, hallucinations, and bizarre behaviors
noted by his sister. e patient is confused and disoriented, his temperature is 39.3 °C, and the
meningeal signs are negative. His brain CT is normal. Which of the following is the most likely
diagnosis of this patient?

a. Brain abscess
b. Herpes encephalitis
c. Meningitis
d. Substance abuse
e. Brain hemorrhage

‫اﻹﺟﺎﺑﺔ ﻋﲆ اﻟﺼﻔﺤﺔ اﻟﺘﺎﻟﻴﺔ‬

Page - 1706
Internal Medicine - Neurology - CNS infections

Question 11/82

Question #11

A 23-year-old male patient presented with headache, fever, hallucinations, and bizarre behaviors
noted by his sister. e patient is confused and disoriented, his temperature is 39.3 °C, and the
meningeal signs are negative. His brain CT is normal. Which of the following is the most likely
diagnosis of this patient?

a. Brain abscess
b. Herpes encephalitis √
c. Meningitis
d. Substance abuse
e. Brain hemorrhage

Description

Headache, fever, and confusion in the absence of meningeal signs are the classic presentation of
encephalitis.

Encephalitis:

Encephalitis is an acute inflammation of the brain due to either a viral infection or an


autoimmune process
e most common cause is a herpes infection
e patient presents with headache and fever of acute onset associated with confusion
Photophobia and a sti neck may present
It is treated by acyclovir intravenously

Page - 1707
Internal Medicine - Neurology

Question 12/82

Question #12

A 22-year-old male patient presented with a decreased level of consciousness, fever, nuchal rigidity,
and petechial rash on his legs and trunk. Which of the following is not consistent with this patient’s
condition?

a. Cloudy CSF appearance


b. CSF glucose of 56 mg/dL
c. CSF protein of 65 mg/dL
d. CSF opening pressure of 28 mmH2O
e. Gram negative diplococci in CSF

‫اﻹﺟﺎﺑﺔ ﻋﲆ اﻟﺼﻔﺤﺔ اﻟﺘﺎﻟﻴﺔ‬

Page - 1708
Internal Medicine - Neurology - CNS infections

Question 12/82

Question #12

A 22-year-old male patient presented with a decreased level of consciousness, fever, nuchal rigidity,
and petechial rash on his legs and trunk. Which of the following is not consistent with this patient’s
condition?

a. Cloudy CSF appearance


b. CSF glucose of 56 mg/dL √
c. CSF protein of 65 mg/dL
d. CSF opening pressure of 28 mmH2O
e. Gram negative diplococci in CSF

Description

is is suspected bacterial meningitis (Neisseria meningitides) because of the characteristic rash.

CSF glucose is less than 40 mg/dL in meningococcal meningitis.

Meningitis:

It is the inflammation of the meninges that presents with headache, fever, and meningism
e classic clinical features include headache, fever, and neck rigidity
Antibiotics should be started immediately, do not wait for culture results

Page - 1709
Internal Medicine - Neurology

Question 13/82

Question #13

A 23-year-old male patient presented with headache, fever, hallucinations, and bizarre behaviors
noted by his sister. e patient is confused and disoriented, his temperature is 39.3 °C, and the
meningeal signs are negative. Which of the following is best to confirm the diagnosis of
encephalitis?

a. Brain CT scan
b. Brain MRI
c. Complete blood count
d. PCR for HSV in the CSF.
e. Appearance and cytology of the CSF

‫اﻹﺟﺎﺑﺔ ﻋﲆ اﻟﺼﻔﺤﺔ اﻟﺘﺎﻟﻴﺔ‬

Page - 1710
Internal Medicine - Neurology - CNS infections

Question 13/82

Question #13

A 23-year-old male patient presented with headache, fever, hallucinations, and bizarre behaviors
noted by his sister. e patient is confused and disoriented, his temperature is 39.3 °C, and the
meningeal signs are negative. Which of the following is best to confirm the diagnosis of
encephalitis?

a. Brain CT scan
b. Brain MRI
c. Complete blood count
d. PCR for HSV in the CSF. √
e. Appearance and cytology of the CSF

Description

is is a typical scenario of encephalitis. e diagnosis is confirmed by PCR for HSV in the CSF.

Encephalitis:

Encephalitis is an acute inflammation of the brain due to either a viral infection or an


autoimmune process
e most common cause is a herpes infection
e patient presents with headache and fever of acute onset associated with confusion
Photophobia and a sti neck may present
It is treated by acyclovir intravenously

Page - 1711
Internal Medicine - Neurology

Question 14/82

Question #14

A 55-year-old male patient was found unconscious in the street. His temperature was 39.5 °C, and
marked nuchal rigidity was noted. LP is done and sent to the lab. Which drugs will cover the likely
oending organism and have good penetration into the CSF?

a. Ceriaxone
b. Ciprofloxacin
c. Dexamethasone
d. IV penicillin
e. Oral Rifaximin

‫اﻹﺟﺎﺑﺔ ﻋﲆ اﻟﺼﻔﺤﺔ اﻟﺘﺎﻟﻴﺔ‬

Page - 1712
Internal Medicine - Neurology - CNS infections

Question 14/82

Question #14

A 55-year-old male patient was found unconscious in the street. His temperature was 39.5 °C, and
marked nuchal rigidity was noted. LP is done and sent to the lab. Which drugs will cover the likely
oending organism and have good penetration into the CSF?

a. Ceriaxone √
b. Ciprofloxacin
c. Dexamethasone
d. IV penicillin
e. Oral Rifaximin

Description

Neck rigidity, fever, and decreased level of consciousness suggest meningitis (or maybe
meningoencephalitis) in this patient.

e most common cause of meningitis in this age group is streptococcus pneumonia. Making
ceriaxone “a 3rd generation cephalosporine” meet both criteria.

Meningitis:

It is the inflammation of the meninges that presents with headache, fever, and meningism
e classic clinical features include headache, fever, and neck rigidity
Antibiotics should be started immediately, do not wait for culture results

Page - 1713
Internal Medicine - Neurology

Question 15/82

Question #15

A 22-year-old male patient presented with a decreased level of consciousness, fever, nuchal rigidity,
and petechial rash on his legs and trunk. In addition to treating this patient, which of the following
is recommended for prevention in his contacts?

a. TMP/SMX
b. Metronidazole
c. Isoniazid
d. Pneumococcal vaccination
e. Rifampicin

‫اﻹﺟﺎﺑﺔ ﻋﲆ اﻟﺼﻔﺤﺔ اﻟﺘﺎﻟﻴﺔ‬

Page - 1714
Internal Medicine - Neurology - CNS infections

Question 15/82

Question #15

A 22-year-old male patient presented with a decreased level of consciousness, fever, nuchal rigidity,
and petechial rash on his legs and trunk. In addition to treating this patient, which of the following
is recommended for prevention in his contacts?

a. TMP/SMX
b. Metronidazole
c. Isoniazid
d. Pneumococcal vaccination
e. Rifampicin √

Description

Rifampicin, ciprofloxacin, or ceriaxone are indicated for prophylaxis in meningococcal meningitis

Neisseria meningitidis (meningococcal meningitis):

Look for a young patient with asplenia who present with a sudden high fever and signs of
meningitis plus characteristic rash (reddish or purplish rash)
It is a rare but serious infection.
It needs isolation, and prevention for close contact (rifampicin, ciprofloxacin, or ceriaxone)
No need to give prophylaxis to nurses who take care of a meningococcal patient, only persons
with kissing or another type of saliva-type contact need prophylaxis (e.g., who share cups,
kissing)

Page - 1715
Internal Medicine - Neurology

Question 16/82

Question #16

In a patient who is present with headache, fever, and vomiting, you noted nuchal rigidity. e most
appropriate workup is to:

a. Brain CT scan
b. Start antibiotics and do LP
c. Coagulation profile
d. Start Aspirin therapy
e. Fundoscopic examination

‫اﻹﺟﺎﺑﺔ ﻋﲆ اﻟﺼﻔﺤﺔ اﻟﺘﺎﻟﻴﺔ‬

Page - 1716
Internal Medicine - Neurology - CNS infections

Question 16/82

Question #16

In a patient who is present with headache, fever, and vomiting, you noted nuchal rigidity. e most
appropriate workup is to:

a. Brain CT scan
b. Start antibiotics and do LP √
c. Coagulation profile
d. Start Aspirin therapy
e. Fundoscopic examination

Description

LP is the gold standard for diagnosing meningitis, but antibiotic administration should not be
delayed.

Meningitis:

It is the inflammation of the meninges that presents with headache, fever, and meningism
e classic clinical features include headache, fever, and neck rigidity
Antibiotics should be started immediately, do not wait for culture results

Page - 1717
Internal Medicine - Neurology

Question 17/82

Question #17

A 43-year-old male presents with recent, moderate to severe, bilateral, frontal, and non-throbbing,
recurrent headaches without aura. He has had similar episodes in the past. Based on this limited
history, which type of headaches can be eliminated from the dierential diagnosis?

a. Cluster headache
b. Headache of intracranial neoplasm
c. Migraine headache
d. Sinus headache
e. Tension-type headache

‫اﻹﺟﺎﺑﺔ ﻋﲆ اﻟﺼﻔﺤﺔ اﻟﺘﺎﻟﻴﺔ‬

Page - 1718
Internal Medicine - Neurology - Cluster headache

Question 17/82

Question #17

A 43-year-old male presents with recent, moderate to severe, bilateral, frontal, and non-throbbing,
recurrent headaches without aura. He has had similar episodes in the past. Based on this limited
history, which type of headaches can be eliminated from the dierential diagnosis?

a. Cluster headache √
b. Headache of intracranial neoplasm
c. Migraine headache
d. Sinus headache
e. Tension-type headache

Description

Cluster headache is always unilateral.

Cluster headaches:

It is one of the most painful types of headaches


More common in males than females
It presents with sudden unilateral severe pain around the eye that may awaken the patient
from sleep, associated with conjunctival injection, tearing, miosis, and ptosis.
It occurs in cyclical patterns or clusters, once or twice a day, each episode lasting 15 minutes to
2 hours, and the clusters may last 4-12 weeks

e acute headaches are treated with 100% oxygen for 15 minutes, but the attacks are
prevented with prophylactic verapamil.

Page - 1719
Internal Medicine - Neurology

Question 18/82

Question #18

A 35-year-old male presents with a 7-year history of episodic severe unilateral headaches. e
headaches occurred most days for 2 months during the most recent episode and lasted about 1 hour.
e most likely diagnosis is:

a. Migraine headache
b. Cluster headache
c. Temporal arteritis
d. Trigeminal neuralgia
e. Post-herpetic neuralgia

‫اﻹﺟﺎﺑﺔ ﻋﲆ اﻟﺼﻔﺤﺔ اﻟﺘﺎﻟﻴﺔ‬

Page - 1720
Internal Medicine - Neurology - Cluster headache

Question 18/82

Question #18

A 35-year-old male presents with a 7-year history of episodic severe unilateral headaches. e
headaches occurred most days for 2 months during the most recent episode and lasted about 1 hour.
e most likely diagnosis is:

a. Migraine headache
b. Cluster headache √
c. Temporal arteritis
d. Trigeminal neuralgia
e. Post-herpetic neuralgia

Description

e nature, frequency, and duration of headaches in this patient strongly suggest cluster headaches.

Cluster headaches are common in males, occur in cycles, and are unilateral.

Cluster headaches:

It is one of the most painful types of headaches


More common in males than females
It presents with sudden unilateral severe pain around the eye that may awaken the patient
from sleep, associated with conjunctival injection, tearing, miosis, and ptosis.
It occurs in cyclical patterns or clusters, once or twice a day, each episode lasting 15 minutes to
2 hours, and the clusters may last 4-12 weeks

e acute headaches are treated with 100% oxygen for 15 minutes, but the attacks are
prevented with prophylactic verapamil.

Page - 1721
Internal Medicine - Neurology

Question 19/82

Question #19

A 35-year-old male developed daily, severe, continuous headaches behind his le eye lasting for 1 – 2
hours and relieved by oxygen. It is associated with tearing from the le eye and a nasal discharge
from the le naris. Neurologic examination is unremarkable. What is the most appropriate at this
time?

a. Brain MRI
b. Start carbamazepine
c. Refer to neurologist
d. Only NSAIDs as needed with no additional treatment
e. Daily use of verapamil

‫اﻹﺟﺎﺑﺔ ﻋﲆ اﻟﺼﻔﺤﺔ اﻟﺘﺎﻟﻴﺔ‬

Page - 1722
Internal Medicine - Neurology - Cluster headache

Question 19/82

Question #19

A 35-year-old male developed daily, severe, continuous headaches behind his le eye lasting for 1 – 2
hours and relieved by oxygen. It is associated with tearing from the le eye and a nasal discharge
from the le naris. Neurologic examination is unremarkable. What is the most appropriate at this
time?

a. Brain MRI
b. Start carbamazepine
c. Refer to neurologist
d. Only NSAIDs as needed with no additional treatment
e. Daily use of verapamil √

Description

Verapamil is used for the prevention of cluster headaches.

Cluster headaches:

It is one of the most painful types of headaches


More common in males than females
It presents with sudden unilateral severe pain around the eye that may awaken the patient
from sleep, associated with conjunctival injection, tearing, miosis, and ptosis.
It occurs in cyclical patterns or clusters, once or twice a day, each episode lasting 15 minutes to
2 hours, and the clusters may last 4-12 weeks

e acute headaches are treated with 100% oxygen for 15 minutes, but the attacks are
prevented with prophylactic verapamil.

e following table demonstrates a comparison between Cluster and Migraine headaches:

Page - 1723
Page - 1724
Internal Medicine - Neurology

Question 20/82

Question #20

A 68-year-old woman presents to your oce with a decline in her recent memory over the last 8
years. is was associated with dysphasia. Which of the following is the most likely diagnosis?

a. CVA
b. Hydrocephalus
c. Alzheimer’s disease
d. Creutzfeldt Jakob disease (CJD)
e. Huntington’s disease

‫اﻹﺟﺎﺑﺔ ﻋﲆ اﻟﺼﻔﺤﺔ اﻟﺘﺎﻟﻴﺔ‬

Page - 1725
Internal Medicine - Neurology - Dementia

Question 20/82

Question #20

A 68-year-old woman presents to your oce with a decline in her recent memory over the last 8
years. is was associated with dysphasia. Which of the following is the most likely diagnosis?

a. CVA
b. Hydrocephalus
c. Alzheimer’s disease √
d. Creutzfeldt Jakob disease (CJD)
e. Huntington’s disease

Description

Alzheimer’s disease (AD) is a slowly progressive memory loss in older patients > 65 years old; it is the
most common cause of dementia

Page - 1726
Internal Medicine - Neurology

Question 21/82

Question #21

A 79-year-old female patient presented with declined memory and urinary incontinence. Her brain
MRI shows wide ventricles but no other pathologies. Which of the following would provide
additional evidence of the diagnosis of NPH?

a. Internuclear ophthalmoplegia
b. Recurrent headaches
c. Right leg weakness
d. Seventh nerve palsy
e. Gait disturbance

‫اﻹﺟﺎﺑﺔ ﻋﲆ اﻟﺼﻔﺤﺔ اﻟﺘﺎﻟﻴﺔ‬

Page - 1727
Internal Medicine - Neurology - Dementia

Question 21/82

Question #21

A 79-year-old female patient presented with declined memory and urinary incontinence. Her brain
MRI shows wide ventricles but no other pathologies. Which of the following would provide
additional evidence of the diagnosis of NPH?

a. Internuclear ophthalmoplegia
b. Recurrent headaches
c. Right leg weakness
d. Seventh nerve palsy
e. Gait disturbance √

Description

Normal-pressure hydrocephalus (NPH), also known as Hakim’s syndrome. It is caused by an


abnormal accumulation of CSF in the brain’s ventricles.

A triad of dementia, urinary incontinence, and gait disturbance is a classic presentation of NPH.

CT scan and MRI to rule out mass lesions in the brain should be done, and LP will show normal
pressure. NPH is treated by ventriculoperitoneal shunt.

Page - 1728
Internal Medicine - Neurology

Question 22/82

Question #22

A 66-year-old male patient presented with memory issues and is suspected of having Alzheimer’s
disease. e best initial workup is to do all the following except:

a. B12 level
b. TSH and T4
c. Brain CT scan
d. Carotid doppler U/S
e. History and physical examination

‫اﻹﺟﺎﺑﺔ ﻋﲆ اﻟﺼﻔﺤﺔ اﻟﺘﺎﻟﻴﺔ‬

Page - 1729
Internal Medicine - Neurology - Dementia

Question 22/82

Question #22

A 66-year-old male patient presented with memory issues and is suspected of having Alzheimer’s
disease. e best initial workup is to do all the following except:

a. B12 level
b. TSH and T4
c. Brain CT scan
d. Carotid doppler U/S √
e. History and physical examination

Description

All patients with memory disturbances should be tested for B12 level, Brain CT scan, and yroid
function test. Occult hyper- or hypothyroidism can imitate the symptoms of Alzheimer’s disease.

Page - 1730
Internal Medicine - Neurology

Question 23/82

Question #23

A 72-year-old male patient presented with slowly progressive memory loss associated with urinary
incontinence and gait disturbance. e examination is otherwise normal, and his lab investigations
are normal. Which one of the following should you do next?

a. Brain MRI
b. Start treatment with SSRIs
c. Prescribe carbidopa/levodopa
d. Refer to physiotherapist
e. Order a brainstem auditory evoked potential

‫اﻹﺟﺎﺑﺔ ﻋﲆ اﻟﺼﻔﺤﺔ اﻟﺘﺎﻟﻴﺔ‬

Page - 1731
Internal Medicine - Neurology - Dementia

Question 23/82

Question #23

A 72-year-old male patient presented with slowly progressive memory loss associated with urinary
incontinence and gait disturbance. e examination is otherwise normal, and his lab investigations
are normal. Which one of the following should you do next?

a. Brain MRI √
b. Start treatment with SSRIs
c. Prescribe carbidopa/levodopa
d. Refer to physiotherapist
e. Order a brainstem auditory evoked potential

Description

MRI to rule out other conditions and to check NPH changes (enlargement of the ventricles with
preservation of cerebral parenchyma)

Normal-pressure hydrocephalus (NPH), also known as Hakim’s syndrome. It is caused by an


abnormal accumulation of CSF in the brain’s ventricles.

A triad of dementia, urinary incontinence, and gait disturbance is a classic presentation of NPH.

CT scan and MRI to rule out mass lesions in the brain should be done, and LP will show normal
pressure. NPH is treated by ventriculoperitoneal shunt.

Page - 1732
Internal Medicine - Neurology

Question 24/82

Question #24

A 75-year-old male patient was found wandering in the street and brought by police to the hospital.
He states that he has been having visual hallucinations, and his wife told you that he has a
deteriorating memory. In addition, his movements have been slowing over the past 6 months. On
examination, the patient has a shuing gait, and coarse resting tremor of the hands is noted
bilaterally. What is the most likely diagnosis?

a. Alzheimer’s disease
b. Frontotemporal dementia
c. Lewy body dementia
d. Normal-pressure hydrocephalus
e. Parkinson’s disease

‫اﻹﺟﺎﺑﺔ ﻋﲆ اﻟﺼﻔﺤﺔ اﻟﺘﺎﻟﻴﺔ‬

Page - 1733
Internal Medicine - Neurology - Dementia

Question 24/82

Question #24

A 75-year-old male patient was found wandering in the street and brought by police to the hospital.
He states that he has been having visual hallucinations, and his wife told you that he has a
deteriorating memory. In addition, his movements have been slowing over the past 6 months. On
examination, the patient has a shuing gait, and coarse resting tremor of the hands is noted
bilaterally. What is the most likely diagnosis?

a. Alzheimer’s disease
b. Frontotemporal dementia
c. Lewy body dementia √
d. Normal-pressure hydrocephalus
e. Parkinson’s disease

Description

Hallucinations, parkinsonism, and dementia are the characteristic presentation of Lewy body
dementia

Lewy body dementia presents with parkinsonism and dementia. Vividly detailed hallucinations may
manifest

Page - 1734
Internal Medicine - Neurology

Question 25/82

Question #25

An 82-year-old female patient with progressive forgetfulness over the last 3 years, her physical
examination and laboratory workup are normal. However, a CT scan of the brain reveals diuse
atrophy. Which of the following is the most likely cause of the patient’s memory problem?

a. Alzheimer’s disease
b. Depression induced dementia
c. Lewy body dementia
d. Multiple brain infarctions dementia
e. Normal aging process

‫اﻹﺟﺎﺑﺔ ﻋﲆ اﻟﺼﻔﺤﺔ اﻟﺘﺎﻟﻴﺔ‬

Page - 1735
Internal Medicine - Neurology - Dementia

Question 25/82

Question #25

An 82-year-old female patient with progressive forgetfulness over the last 3 years, her physical
examination and laboratory workup are normal. However, a CT scan of the brain reveals diuse
atrophy. Which of the following is the most likely cause of the patient’s memory problem?

a. Alzheimer’s disease √
b. Depression induced dementia
c. Lewy body dementia
d. Multiple brain infarctions dementia
e. Normal aging process

Description

is is a classic case of Alzheimer’s disease. is patient has no depression, no infarctions, no
Parkinsonism.

Alzheimer’s disease (AD) is a slowly progressive memory loss in older patients > 65 years old; it is the
most common cause of dementia

Page - 1736
Internal Medicine - Neurology

Question 26/82

Question #26

e following are possible causes of dementia. Which of the following will not benefit from a
specific treatment?

a. Alzheimer’s disease
b. B12 deficiency
c. Creutzfeldt Jakob disease
d. Normal-pressure hydrocephalus
e. Sarcoidosis

‫اﻹﺟﺎﺑﺔ ﻋﲆ اﻟﺼﻔﺤﺔ اﻟﺘﺎﻟﻴﺔ‬

Page - 1737
Internal Medicine - Neurology - Dementia

Question 26/82

Question #26

e following are possible causes of dementia. Which of the following will not benefit from a
specific treatment?

a. Alzheimer’s disease
b. B12 deficiency
c. Creutzfeldt Jakob disease √
d. Normal-pressure hydrocephalus
e. Sarcoidosis

Description

Dementia: is a decline in memory or other thinking skills severe enough to reduce a person’s
ability to perform everyday activities.
No cure for Creutzfeldt Jakob disease (CJD). It is usually fatal within months to a few years
Alzheimer’s disease is treated by Donepezil, Rivastigmine, Galantamine
B12 deficiency is treated by B12 replacement
Normal pressure hydrocephalus (NPH) is treated by ventriculoperitoneal shunt
Sarcoidosis will benefit from steroid therapy in certain conditions

Page - 1738
Internal Medicine - Neurology

Question 27/82

Question #27

Which of the following disorders may benefit from the use of Ginkgo Biloba?

a. Benign prostatic hyperplasia


b. Dementia
c. Intermittent claudication
d. Tinnitus
e. Coronary heart disease

‫اﻹﺟﺎﺑﺔ ﻋﲆ اﻟﺼﻔﺤﺔ اﻟﺘﺎﻟﻴﺔ‬

Page - 1739
Internal Medicine - Neurology - Dementia

Question 27/82

Question #27

Which of the following disorders may benefit from the use of Ginkgo Biloba?

a. Benign prostatic hyperplasia


b. Dementia √
c. Intermittent claudication
d. Tinnitus
e. Coronary heart disease

Description

Dementia is a decline in memory or other thinking skills severe enough to reduce a person’s ability
to perform everyday activities

Patients with dementia demonstrate that ginkgo is more eective than a placebo

Page - 1740
Internal Medicine - Neurology

Question 28/82

Question #28

A 72-year-old male patient presented with slowly progressive memory loss associated with urinary
incontinence and gait disturbance. What is the gold standard diagnostic modality for this patient’s
condition?

a. e clinical presentation only


b. Brain CT scan
c. Intrathecal infusion test
d. CSF removal
e. Fundoscopy

‫اﻹﺟﺎﺑﺔ ﻋﲆ اﻟﺼﻔﺤﺔ اﻟﺘﺎﻟﻴﺔ‬

Page - 1741
Internal Medicine - Neurology - Dementia

Question 28/82

Question #28

A 72-year-old male patient presented with slowly progressive memory loss associated with urinary
incontinence and gait disturbance. What is the gold standard diagnostic modality for this patient’s
condition?

a. e clinical presentation only


b. Brain CT scan
c. Intrathecal infusion test
d. CSF removal √
e. Fundoscopy

Description

Large volume lumbar puncture, 30 ml to 40 mL, is the gold standard diagnostic modality.

Normal-pressure hydrocephalus (NPH), also known as Hakim’s syndrome. It is caused by an


abnormal accumulation of CSF in the brain’s ventricles.

A triad of dementia, urinary incontinence, and gait disturbance is a classic presentation of NPH.

CT scan and MRI to rule out mass lesions in the brain should be done, and LP will show normal
pressure. NPH is treated by ventriculoperitoneal shunt.

Page - 1742
Internal Medicine - Neurology

Question 29/82

Question #29

A 45-year-old male patient presents with progressive mental impairment, irritability, and
involuntary dancing hand movements. His father had the same condition at the age of 42. He wants
to know if his children may eventually develop a similar situation. However, his wife has no history
or family history of the same condition. Which would be your most appropriate answer?

a. All his children will inherit this disease


b. Each child has a 25% risk of developing the condition
c. Each child has a 50% risk of developing the condition
d. His male children will be only at risk
e. Only one child will be at risk

‫اﻹﺟﺎﺑﺔ ﻋﲆ اﻟﺼﻔﺤﺔ اﻟﺘﺎﻟﻴﺔ‬

Page - 1743
Internal Medicine - Neurology - Dementia

Question 29/82

Question #29

A 45-year-old male patient presents with progressive mental impairment, irritability, and
involuntary dancing hand movements. His father had the same condition at the age of 42. He wants
to know if his children may eventually develop a similar situation. However, his wife has no history
or family history of the same condition. Which would be your most appropriate answer?

a. All his children will inherit this disease


b. Each child has a 25% risk of developing the condition
c. Each child has a 50% risk of developing the condition √
d. His male children will be only at risk
e. Only one child will be at risk

Description

Huntington’s disease is an autosomal dominant condition. erefore, if one parent is aected, the
risk of inheritance will be 50%.

Huntington's disease/chorea:

Age aected: 30 – 50 years


Family history is common
CAG trinucleotide repeats on chromosome 4
Presence of dementia + chorea + psychiatric disturbance and personality changes
Diagnosis: by genetic testing
Treatment: tetrabenazine (for chorea) and antipsychotics

Page - 1744
Internal Medicine - Neurology

Question 30/82

Question #30

A 53-year-old female patient came to you with her husband, who noted a memory issue with her.
For the past 6 months, she has complained of poor appetite, insomnia, lack of energy, and inability
to do routine housework. Her short-term recall seems impaired, but a more detailed examination
indicates that her memory is good. Which one of the following is the most likely diagnosis?

a. Alzheimer’s disease
b. Frontotemporal dementia
c. Lewy body dementia
d. Creutzfeldt-Jakob disease
e. Pseudodementia

‫اﻹﺟﺎﺑﺔ ﻋﲆ اﻟﺼﻔﺤﺔ اﻟﺘﺎﻟﻴﺔ‬

Page - 1745
Internal Medicine - Neurology - Dementia

Question 30/82

Question #30

A 53-year-old female patient came to you with her husband, who noted a memory issue with her.
For the past 6 months, she has complained of poor appetite, insomnia, lack of energy, and inability
to do routine housework. Her short-term recall seems impaired, but a more detailed examination
indicates that her memory is good. Which one of the following is the most likely diagnosis?

a. Alzheimer’s disease
b. Frontotemporal dementia
c. Lewy body dementia
d. Creutzfeldt-Jakob disease
e. Pseudodementia √

Description

In pseudodementia of depression, the memory is usually intact when enough time is taken to
evaluate the patient carefully.

Page - 1746
Internal Medicine - Neurology

Question 31/82

Question #31

Which one of the following is more predictive regarding the progression of Alzheimer’s disease?

a. Memory deficit reported by a family member


b. Self-reported memory deficit
c. Absence of Hippocampal atrophy on brain MRI
d. e lack of the apolipoprotein E4 gene
e. e development of abnormal body movement

‫اﻹﺟﺎﺑﺔ ﻋﲆ اﻟﺼﻔﺤﺔ اﻟﺘﺎﻟﻴﺔ‬

Page - 1747
Internal Medicine - Neurology - Dementia

Question 31/82

Question #31

Which one of the following is more predictive regarding the progression of Alzheimer’s disease?

a. Memory deficit reported by a family member √


b. Self-reported memory deficit
c. Absence of Hippocampal atrophy on brain MRI
d. e lack of the apolipoprotein E4 gene
e. e development of abnormal body movement

Description

Alzheimer’s disease (AD) is a slowly progressive memory loss in older patients > 65 years old; it is the
most common cause of dementia.

Indications for Parkinson’s disease progression:

Hippocampal atrophy on MRI


e presence of the E4 allele
Memory deficit reported by a family member

Page - 1748
Internal Medicine - Neurology

Question 32/82

Question #32

A 72-year-old male patient presented with slowly progressive memory loss associated with urinary
incontinence and gait disturbance. What is the most likely diagnosis?

a. Alzheimer’s disease
b. Normal-pressure hydrocephalus
c. Parkinson’s disease
d. Lewy body dementia
e. Creutzfeldt Jakob disease (CJD)

‫اﻹﺟﺎﺑﺔ ﻋﲆ اﻟﺼﻔﺤﺔ اﻟﺘﺎﻟﻴﺔ‬

Page - 1749
Internal Medicine - Neurology - Dementia

Question 32/82

Question #32

A 72-year-old male patient presented with slowly progressive memory loss associated with urinary
incontinence and gait disturbance. What is the most likely diagnosis?

a. Alzheimer’s disease
b. Normal-pressure hydrocephalus √
c. Parkinson’s disease
d. Lewy body dementia
e. Creutzfeldt Jakob disease (CJD)

Description

Normal-pressure hydrocephalus (NPH), also known as Hakim’s syndrome. It is caused by an


abnormal accumulation of CSF in the brain’s ventricles.

A triad of dementia, urinary incontinence, and gait disturbance is a classic presentation of NPH.

CT scan and MRI to rule out mass lesions in the brain should be done, and LP will show normal
pressure. NPH is treated by ventriculoperitoneal shunt.

Page - 1750
Internal Medicine - Neurology

Question 33/82

Question #33

Regarding the risk for Alzheimer’s disease, which is false?

a. Trisomy 18 has an increased risk of early Alzheimer’s disease


b. Advanced age has more risk of the disease
c. A family history of Alzheimer’s disease increases the risk
d. Head trauma is associated with a higher incidence
e. A homozygous genotype of apolipoprotein E4

‫اﻹﺟﺎﺑﺔ ﻋﲆ اﻟﺼﻔﺤﺔ اﻟﺘﺎﻟﻴﺔ‬

Page - 1751
Internal Medicine - Neurology - Dementia

Question 33/82

Question #33

Regarding the risk for Alzheimer’s disease, which is false?

a. Trisomy 18 has an increased risk of early Alzheimer’s disease √


b. Advanced age has more risk of the disease
c. A family history of Alzheimer’s disease increases the risk
d. Head trauma is associated with a higher incidence
e. A homozygous genotype of apolipoprotein E4

Description

Alzheimer’s disease (AD) is a slowly progressive memory loss in older patients > 65 years old; it is the
most common cause of dementia.

Trisomy 21, not 18, is associated with early Alzheimer’s disease (usually < 40 years old).

Page - 1752
Internal Medicine - Neurology

Question 34/82

Question #34

A 62-year-old male presents with behavioral changes in the last year and memory disturbances. He
is suspected of having Pick’s disease. What would likely be found on an MRI of this patient’s brain?

a. Periventricular plaques
b. Atrophy of frontal and anterior temporal lobes
c. Corpus callosum lesion
d. Severe hippocampal, medial temporal lobe, and generalized atrophy
e. Peripheral micro-hemorrhages

‫اﻹﺟﺎﺑﺔ ﻋﲆ اﻟﺼﻔﺤﺔ اﻟﺘﺎﻟﻴﺔ‬

Page - 1753
Internal Medicine - Neurology - Dementia

Question 34/82

Question #34

A 62-year-old male presents with behavioral changes in the last year and memory disturbances. He
is suspected of having Pick’s disease. What would likely be found on an MRI of this patient’s brain?

a. Periventricular plaques
b. Atrophy of frontal and anterior temporal lobes √
c. Corpus callosum lesion
d. Severe hippocampal, medial temporal lobe, and generalized atrophy
e. Peripheral micro-hemorrhages

Description

Frontal and temporal lobe atrophy in brain imaging is characteristic of Pick’s disease.

Picks disease (frontotemporal dementia):

Abnormal behavior and personality changes precede other symptoms


Memory loss
No movement disorders
Frontal and temporal lobe atrophy in brain imaging
Treatment is the same as Alzheimer’s disease

Page - 1754
Internal Medicine - Neurology

Question 35/82

Question #35

A 23-year-old male patient presents with recurrent loss of consciousness and tonic-clonic muscular
contractions. His tongue fell back into the throat, and he choked. What is the most likely diagnosis?

a. Petit mal seizure


b. Grand mal seizure
c. Myoclonic seizure
d. Atonic seizure
e. Focal to generalized seizure

‫اﻹﺟﺎﺑﺔ ﻋﲆ اﻟﺼﻔﺤﺔ اﻟﺘﺎﻟﻴﺔ‬

Page - 1755
Internal Medicine - Neurology - Epilepsy, Seizure, and Status epilepticus

Question 35/82

Question #35

A 23-year-old male patient presents with recurrent loss of consciousness and tonic-clonic muscular
contractions. His tongue fell back into the throat, and he choked. What is the most likely diagnosis?

a. Petit mal seizure


b. Grand mal seizure √
c. Myoclonic seizure
d. Atonic seizure
e. Focal to generalized seizure

Description

Grand mal seizure (tonic-clonic seizure) is characterized by a sudden loss of consciousness followed
by tonic and clonic muscular contractions.

Types of seizures:

Focal seizures: (no loss of consciousness)

Simple sensory: e.g., déjà vu


Simple motor: e.g., Jacksonian march
Complex: e.g., Automatism

Generalized seizure: (associated with loss of consciousness)

Tonic-clonic (grand mal)


Absence (petit mal)
Tonic, Clonic, Myoclonic, and Atonic types.

Focal to generalized seizure: (starts as focal, then becomes generalized)

Page - 1756
Internal Medicine - Neurology

Question 36/82

Question #36

A 23-year-old male patient came to you because of recurrent loss of consciousness and tonic-clonic
muscular contractions. During the events, his tongue fell back into his throat, and he choked. What
is the drug of choice for this patient?

a. Carbamazepine
b. Phenobarbital
c. Phenytoin
d. Tiagabine
e. Valproic acid

‫اﻹﺟﺎﺑﺔ ﻋﲆ اﻟﺼﻔﺤﺔ اﻟﺘﺎﻟﻴﺔ‬

Page - 1757
Internal Medicine - Neurology - Epilepsy, Seizure, and Status epilepticus

Question 36/82

Question #36

A 23-year-old male patient came to you because of recurrent loss of consciousness and tonic-clonic
muscular contractions. During the events, his tongue fell back into his throat, and he choked. What
is the drug of choice for this patient?

a. Carbamazepine
b. Phenobarbital
c. Phenytoin
d. Tiagabine
e. Valproic acid √

Description

Grand mal seizure (tonic-clonic) is characterized by a sudden loss of consciousness followed by tonic
and clonic muscular contractions.

Valproic acid is the first-choice agent for treating grand-mal (tonic-clonic) seizures.

Page - 1758
Internal Medicine - Neurology

Question 37/82

Question #37

A 28-year-old male patent presented with recurrent true seizure attacks for the last 15 minutes
without returning consciousness between attacks. What is the best initial treatment for his
condition?

a. Lorazepam
b. Phenytoin
c. Phenobarbital
d. Carbamazepine
e. Valproic acid

‫اﻹﺟﺎﺑﺔ ﻋﲆ اﻟﺼﻔﺤﺔ اﻟﺘﺎﻟﻴﺔ‬

Page - 1759
Internal Medicine - Neurology - Epilepsy, Seizure, and Status epilepticus

Question 37/82

Question #37

A 28-year-old male patent presented with recurrent true seizure attacks for the last 15 minutes
without returning consciousness between attacks. What is the best initial treatment for his
condition?

a. Lorazepam √
b. Phenytoin
c. Phenobarbital
d. Carbamazepine
e. Valproic acid

Description

Lorazepam is the preferred agent in the treatment of Status epilepticus.

Approach to treat status epilepticus:

1st line: Lorazepam/ Diazepam


2nd line: Phenytoin/ Fosphenytoin
3rd line: Phenobarbital
4th line: ICU transfer, intubation, ventilation with general anesthesia

Page - 1760
Internal Medicine - Neurology

Question 38/82

Question #38

Which of the following is the treatment of choice for absent seizures?

a. Valproic acid
b. Lamotrigine
c. Ethosuximide
d. Phenytoin
e. Levetiracetam

‫اﻹﺟﺎﺑﺔ ﻋﲆ اﻟﺼﻔﺤﺔ اﻟﺘﺎﻟﻴﺔ‬

Page - 1761
Internal Medicine - Neurology - Epilepsy, Seizure, and Status epilepticus

Question 38/82

Question #38

Which of the following is the treatment of choice for absent seizures?

a. Valproic acid
b. Lamotrigine
c. Ethosuximide √
d. Phenytoin
e. Levetiracetam

Description

Ethosuximide is the drug of choice for absent (Petit mal) seizures.

e treatment of epilepsy is summarized in the following table:

Page - 1762
Internal Medicine - Neurology

Question 39/82

Question #39

A 25-year-old male patient with epilepsy is on treatment with carbamazepine. In the last 2 weeks, he
had 3 episodes of convulsions. His carbamazepine level in his blood is within the therapeutic range.
What would you do next?

a. Add another agent


b. Brain CT scan and EEG
c. Check carbamazepine level in 2 weeks
d. Observation
e. Order CBC, KFT, U&Es, T4, TSH

‫اﻹﺟﺎﺑﺔ ﻋﲆ اﻟﺼﻔﺤﺔ اﻟﺘﺎﻟﻴﺔ‬

Page - 1763
Internal Medicine - Neurology - Epilepsy, Seizure, and Status epilepticus

Question 39/82

Question #39

A 25-year-old male patient with epilepsy is on treatment with carbamazepine. In the last 2 weeks, he
had 3 episodes of convulsions. His carbamazepine level in his blood is within the therapeutic range.
What would you do next?

a. Add another agent √


b. Brain CT scan and EEG
c. Check carbamazepine level in 2 weeks
d. Observation
e. Order CBC, KFT, U&Es, T4, TSH

Description

Seizure is a clinical syndrome caused by an abnormal electrical discharge in the brain, while epilepsy
is a tendency to have a seizure; a single seizure is not epilepsy, But an indication for investigations.

e patient has uncontrolled epilepsy despite normal blood carbamazepine level. So, another agent
should be added.

Page - 1764
Internal Medicine - Neurology

Question 40/82

Question #40

A 23-year-old male patient came to you because of recurrent loss of consciousness and tonic-clonic
muscular contractions. During the events, his tongue fell back into his throat, and he choked. What
is the best diagnostic test to do for this patient?

a. Chest x-ray
b. Brain CT scan
c. EEG
d. Reassurance
e. Brain MRI

‫اﻹﺟﺎﺑﺔ ﻋﲆ اﻟﺼﻔﺤﺔ اﻟﺘﺎﻟﻴﺔ‬

Page - 1765
Internal Medicine - Neurology - Epilepsy, Seizure, and Status epilepticus

Question 40/82

Question #40

A 23-year-old male patient came to you because of recurrent loss of consciousness and tonic-clonic
muscular contractions. During the events, his tongue fell back into his throat, and he choked. What
is the best diagnostic test to do for this patient?

a. Chest x-ray
b. Brain CT scan
c. EEG √
d. Reassurance
e. Brain MRI

Description

Grand mal seizure (tonic-clonic) is characterized by a sudden loss of consciousness followed by tonic
and clonic muscular contractions.

EEG is the appropriate diagnostic test to be done for the identification and assessment of grand mal
seizures.

Page - 1766
Internal Medicine - Neurology

Question 41/82

Question #41

A 23-year-old male patient experienced an episode of jerky movements and loss of consciousness.
Which of the following would make a true seizure more likely than pseudoseizures?

a. Eyes closed during the attack


b. Post-event confusion
c. History of chronic pain or fibromyalgia
d. Rapid breathing and tachycardia with carpopedal spasm
e. Carpal spasms

‫اﻹﺟﺎﺑﺔ ﻋﲆ اﻟﺼﻔﺤﺔ اﻟﺘﺎﻟﻴﺔ‬

Page - 1767
Internal Medicine - Neurology - Epilepsy, Seizure, and Status epilepticus

Question 41/82

Question #41

A 23-year-old male patient experienced an episode of jerky movements and loss of consciousness.
Which of the following would make a true seizure more likely than pseudoseizures?

a. Eyes closed during the attack


b. Post-event confusion √
c. History of chronic pain or fibromyalgia
d. Rapid breathing and tachycardia with carpopedal spasm
e. Carpal spasms

Description

Eye closure is uncommon in true seizures. In comparison, carpal and carpopedal spasms may present
with hyperventilation syndrome.

Features suggestive of seizure activity include:

Tongue biting
Pre-ictal aura
Post-ictal confusion
Focal neurologic signs.

Page - 1768
Internal Medicine - Neurology

Question 42/82

Question #42

What is the most frequent cause of seizures in elderly patients?

a. Alcohol withdrawal
b. Stroke
c. Head trauma
d. Hypoglycemia
e. Primary epilepsy

‫اﻹﺟﺎﺑﺔ ﻋﲆ اﻟﺼﻔﺤﺔ اﻟﺘﺎﻟﻴﺔ‬

Page - 1769
Internal Medicine - Neurology - Epilepsy, Seizure, and Status epilepticus

Question 42/82

Question #42

What is the most frequent cause of seizures in elderly patients?

a. Alcohol withdrawal
b. Stroke √
c. Head trauma
d. Hypoglycemia
e. Primary epilepsy

Description

CVA’s are the most common cause of seizures in geriatric patients.

Secondary causes of seizures include:

Metabolic (Hypoglycemia, hypernatremia, hyponatremia, hypocalcemia, hypomagnesemia,


Uremia)
CNS infection (Meningitis, encephalitis, abscess)
Brain tumors, trauma, hypoxia, CVA, hemorrhage
Toxic substances (alcohol, cocaine)

Page - 1770
Internal Medicine - Neurology

Question 43/82

Question #43

A 28-year-old male patent presented with recurrent true seizure attacks for the last 15 minutes.
ere is no returning consciousness between attacks. e patient is suering from which of the
following?

a. Tonic clonic convulsion


b. Absent seizure
c. Jitteriness
d. Status epilepticus
e. Hysterical conversion disorder

‫اﻹﺟﺎﺑﺔ ﻋﲆ اﻟﺼﻔﺤﺔ اﻟﺘﺎﻟﻴﺔ‬

Page - 1771
Internal Medicine - Neurology - Epilepsy, Seizure, and Status epilepticus

Question 43/82

Question #43

A 28-year-old male patent presented with recurrent true seizure attacks for the last 15 minutes.
ere is no returning consciousness between attacks. e patient is suering from which of the
following?

a. Tonic clonic convulsion


b. Absent seizure
c. Jitteriness
d. Status epilepticus √
e. Hysterical conversion disorder

Description

Status epilepticus is a seizure not resolving spontaneously or recurrent convulsions without


recovery of consciousness for > 5 minutes. It can be due to a sub-therapeutic level of anti-epileptic
drugs in a patient known to have epilepsy or may be precipitated by secondary causes like metabolic
infection, tumor, etc.

Page - 1772
Internal Medicine - Neurology

Question 44/82

Question #44

A 24-year-old male patient with a known case of epilepsy presented with an attack of convulsion for
the last 15 minutes without a return of consciousness. Aer maintaining adequate ventilation and
perfusion, what is the first-line pharmacological treatment you will start?

a. Phenytoin
b. Fosphenytoin
c. Naloxone
d. Lorazepam
e. Valproate sodium

‫اﻹﺟﺎﺑﺔ ﻋﲆ اﻟﺼﻔﺤﺔ اﻟﺘﺎﻟﻴﺔ‬

Page - 1773
Internal Medicine - Neurology - Epilepsy, Seizure, and Status epilepticus

Question 44/82

Question #44

A 24-year-old male patient with a known case of epilepsy presented with an attack of convulsion for
the last 15 minutes without a return of consciousness. Aer maintaining adequate ventilation and
perfusion, what is the first-line pharmacological treatment you will start?

a. Phenytoin
b. Fosphenytoin
c. Naloxone
d. Lorazepam √
e. Valproate sodium

Description

Status epilepticus is a seizure not resolving spontaneously or recurrent convulsions without


recovery of consciousness for > 5 minutes

Lorazepam is the preferred agent in the treatment of Status epilepticus.

Approach to treat status epilepticus:

1st line: Lorazepam/ Diazepam


2nd line: Phenytoin/ Fosphenytoin
3rd line: Phenobarbital
4th line: ICU transfer, intubation, ventilation with general anesthesia

Page - 1774
Internal Medicine - Neurology

Question 45/82

Question #45

A 66-year-old female has developed a slowly progressive right-hand tremor for 6 months. It
worsens when she holds them up against gravity but is relieved by resting them. Her mother has the
same problem, and her physical examination and lab investigations appear unremarkable. What
would you prescribe to relieve her tremor?

a. Bromocriptine
b. Carbamazepine
c. Carbidopa/levodopa
d. Escitalopram
e. Propranolol

‫اﻹﺟﺎﺑﺔ ﻋﲆ اﻟﺼﻔﺤﺔ اﻟﺘﺎﻟﻴﺔ‬

Page - 1775
Internal Medicine - Neurology - Essential Tremor

Question 45/82

Question #45

A 66-year-old female has developed a slowly progressive right-hand tremor for 6 months. It
worsens when she holds them up against gravity but is relieved by resting them. Her mother has the
same problem, and her physical examination and lab investigations appear unremarkable. What
would you prescribe to relieve her tremor?

a. Bromocriptine
b. Carbamazepine
c. Carbidopa/levodopa
d. Escitalopram
e. Propranolol √

Description

Essential tremor occurs in both rest and intention


More common in the hands, but the head may be aected
It may aect some skills like handwriting
Caeine makes it worse, and alcohol makes it better
Primidone and propranolol are the best maintenance medical therapy

Page - 1776
Internal Medicine - Neurology

Question 46/82

Question #46

Which of the following is the best to be used in treating essential tremor?

a. Isoniazid
b. Diazepam
c. Propranolol
d. Gabapentin
e. Pregabalin

‫اﻹﺟﺎﺑﺔ ﻋﲆ اﻟﺼﻔﺤﺔ اﻟﺘﺎﻟﻴﺔ‬

Page - 1777
Internal Medicine - Neurology - Essential Tremor

Question 46/82

Question #46

Which of the following is the best to be used in treating essential tremor?

a. Isoniazid
b. Diazepam
c. Propranolol √
d. Gabapentin
e. Pregabalin

Description

Essential tremor occurs in both rest and intention


More common in the hands, but the head may be aected
It may aect some skills like handwriting
Caeine makes it worse, and alcohol makes it better
Primidone and propranolol are the best maintenance medical therapy

Page - 1778
Internal Medicine - Neurology

Question 47/82

Question #47

A 46-year-old male patent presented to you with bilateral lower limb weakness and decreased ankle
jerks. 2 weeks ago, he was diagnosed with C. jejuni diarrhea. All the following are characteristic of
Guillain-Barré syndrome except:

a. Hyporeflexia or Areflexia
b. Sensory deficit
c. Infection with C. jejuni
d. Muscle weakness
e. Steroids used in the treatment

‫اﻹﺟﺎﺑﺔ ﻋﲆ اﻟﺼﻔﺤﺔ اﻟﺘﺎﻟﻴﺔ‬

Page - 1779
Internal Medicine - Neurology - Guillain-Barré syndrome (GBS)

Question 47/82

Question #47

A 46-year-old male patent presented to you with bilateral lower limb weakness and decreased ankle
jerks. 2 weeks ago, he was diagnosed with C. jejuni diarrhea. All the following are characteristic of
Guillain-Barré syndrome except:

a. Hyporeflexia or Areflexia
b. Sensory deficit
c. Infection with C. jejuni
d. Muscle weakness
e. Steroids used in the treatment √

Description

ere is no benefit of steroid use in Guillain-Barre Syndrome (GBS). It is treated with IVIG or plasma
exchange

Guillain-Barre Syndrome (GBS):

Immune-mediated demyelination of the peripheral nervous system


Oen triggered by an infection (classically Campylobacter jejuni)
e clinical features include paresthesia with pain precedes weakness in ascending pattern
e most common cause of death is a respiratory failure (intubation may be needed)
Treatment is achieved by IVIG or plasmapheresis

Page - 1780
Internal Medicine - Neurology

Question 48/82

Question #48

A 46-year-old male patent presented to you with bilateral lower limb weakness and decreased ankle
jerks. 2 weeks ago, he was diagnosed with C. jejuni diarrhea. What is the most likely diagnosis?

a. Guillain-Barré syndrome
b. Multiple sclerosis
c. Myasthenia gravis disease
d. Systemic Lupus Erythematosus (SLE)
e. Cerebrovascular accident (CVA)

‫اﻹﺟﺎﺑﺔ ﻋﲆ اﻟﺼﻔﺤﺔ اﻟﺘﺎﻟﻴﺔ‬

Page - 1781
Internal Medicine - Neurology - Guillain-Barré syndrome (GBS)

Question 48/82

Question #48

A 46-year-old male patent presented to you with bilateral lower limb weakness and decreased ankle
jerks. 2 weeks ago, he was diagnosed with C. jejuni diarrhea. What is the most likely diagnosis?

a. Guillain-Barré syndrome √
b. Multiple sclerosis
c. Myasthenia gravis disease
d. Systemic Lupus Erythematosus (SLE)
e. Cerebrovascular accident (CVA)

Description

Ascending paralysis aer infectious diarrhea is the classic presentation of Guillain-Barre Syndrome
(GBS)

Guillain-Barre Syndrome (GBS):

Immune-mediated demyelination of the peripheral nervous system


Oen triggered by an infection (classically Campylobacter jejuni)
e clinical features include paresthesia with pain precedes weakness in ascending pattern
e most common cause of death is a respiratory failure (intubation may be needed)
Treatment is achieved by IVIG or plasmapheresis

Page - 1782
Internal Medicine - Neurology

Question 49/82

Question #49

A 39-year-old female patient presented with 6 months of recurrent morning headaches and
vomiting. On examination, she has nystagmus. What is the next step in your investigation?

a. Referral to ophthalmologist
b. Brain CT scan
c. EEG
d. Temporal artery biopsy
e. rombolytic therapy

‫اﻹﺟﺎﺑﺔ ﻋﲆ اﻟﺼﻔﺤﺔ اﻟﺘﺎﻟﻴﺔ‬

Page - 1783
Internal Medicine - Neurology - Headaches

Question 49/82

Question #49

A 39-year-old female patient presented with 6 months of recurrent morning headaches and
vomiting. On examination, she has nystagmus. What is the next step in your investigation?

a. Referral to ophthalmologist
b. Brain CT scan √
c. EEG
d. Temporal artery biopsy
e. rombolytic therapy

Description

If an intracranial mass is suspected, a CT or MRI of the head/brain should be done

Page - 1784
Internal Medicine - Neurology

Question 50/82

Question #50

A 49-year-old patient with a known case of diabetes came with constricted pupils, a drooping upper
lid, and dry skin on the le side of his face. Which nerve is the most likely aected?

a. Oculomotor
b. Sympathetic
c. Trochlear
d. Facial
e. Abducent

‫اﻹﺟﺎﺑﺔ ﻋﲆ اﻟﺼﻔﺤﺔ اﻟﺘﺎﻟﻴﺔ‬

Page - 1785
Internal Medicine - Neurology - Horner syndrome

Question 50/82

Question #50

A 49-year-old patient with a known case of diabetes came with constricted pupils, a drooping upper
lid, and dry skin on the le side of his face. Which nerve is the most likely aected?

a. Oculomotor
b. Sympathetic √
c. Trochlear
d. Facial
e. Abducent

Description

Ptosis, myosis, and anhidrosis are classic features of Horner syndrome. It is caused by sympathetic
nerve fibers interruption

Horner's syndrome:

Classically, presented with Ptosis, Miosis, anhidrosis, and Enophthalmos


Cased by interruption of sympathetic nerve supply to the eye
Maybe due to an apical (Pancoast) tumor of the lung

Page - 1786
Internal Medicine - Neurology

Question 51/82

Question #51

A 65-year-old male patient presented with ptosis, myosis, and anhidrosis on the right side of his
face. What is the most likely cause?

a. Tumor-induced exophthalmos
b. Oculomotor nerve palsy
c. Apical pulmonary carcinoma (Pancoast)
d. yroid goiter
e. Vagus nerve palsy

‫اﻹﺟﺎﺑﺔ ﻋﲆ اﻟﺼﻔﺤﺔ اﻟﺘﺎﻟﻴﺔ‬

Page - 1787
Internal Medicine - Neurology - Horner syndrome

Question 51/82

Question #51

A 65-year-old male patient presented with ptosis, myosis, and anhidrosis on the right side of his
face. What is the most likely cause?

a. Tumor-induced exophthalmos
b. Oculomotor nerve palsy
c. Apical pulmonary carcinoma (Pancoast) √
d. yroid goiter
e. Vagus nerve palsy

Description

Ptosis, myosis, and anhidrosis are classic features of Horner syndrome. Pancoast tumor is the most
likely cause.

Horner's syndrome:

Classically, presented with Ptosis, Miosis, anhidrosis, and Enophthalmos


Cased by interruption of sympathetic nerve supply to the eye
Maybe due to an apical (Pancoast) tumor of the lung

Page - 1788
Internal Medicine - Neurology

Question 52/82

Question #52

A 65-year-old male patient presented with ptosis, myosis, and anhidrosis on the right side of his
face. In addition to a complete history and physical examination. Which of the following would be
most appropriate at this point?

a. A chest X-Ray.
b. Brain and orbital MRI
c. yroid function test
d. Fasting glucose level and A1C
e. An acetylcholine receptor antibody level

‫اﻹﺟﺎﺑﺔ ﻋﲆ اﻟﺼﻔﺤﺔ اﻟﺘﺎﻟﻴﺔ‬

Page - 1789
Internal Medicine - Neurology - Horner syndrome

Question 52/82

Question #52

A 65-year-old male patient presented with ptosis, myosis, and anhidrosis on the right side of his
face. In addition to a complete history and physical examination. Which of the following would be
most appropriate at this point?

a. A chest X-Ray. √
b. Brain and orbital MRI
c. yroid function test
d. Fasting glucose level and A1C
e. An acetylcholine receptor antibody level

Description

Ptosis, myosis, and anhidrosis are classic features of Horner syndrome. Pancoast tumor is the most
likely cause.

Horner's syndrome:

Classically, presented with Ptosis, Miosis, anhidrosis, and Enophthalmos


Cased by interruption of sympathetic nerve supply to the eye
Maybe due to an apical (Pancoast) tumor of the lung

Page - 1790
Internal Medicine - Neurology

Question 53/82

Question #53

About the nervous system in the human, all the following are true except:

a. e spinal cord is a part of the central nervous system


b. e Corpus callosum is a highly vascular area connecting the right and le hemispheres
c. e white matter is the inner part of the spinal cord.
d. e cranial nerves V, VI, and VII have their nuclei in the pones
e. e cerebellum function is to maintain coordination, voluntary motor activity, and equilibrium

‫اﻹﺟﺎﺑﺔ ﻋﲆ اﻟﺼﻔﺤﺔ اﻟﺘﺎﻟﻴﺔ‬

Page - 1791
Internal Medicine - Neurology - Introduction to Neurology

Question 53/82

Question #53

About the nervous system in the human, all the following are true except:

a. e spinal cord is a part of the central nervous system


b. e Corpus callosum is a highly vascular area connecting the right and le hemispheres
c. e white matter is the inner part of the spinal cord. √
d. e cranial nerves V, VI, and VII have their nuclei in the pones
e. e cerebellum function is to maintain coordination, voluntary motor activity, and equilibrium

Description

e white matter occupies the inner part of the brain and the outer part of the spinal cord.

Nervous system:

e central nervous system (CNS) is formed of two main parts:


Intracranial part (Cerebrum, brain stem, cerebellum)
Spinal part (spinal cord, cauda equina)
e Cerebrum is formed of two hemispheres connected via the corpus callosum and the upper
part of the brain stem
e Brain stem is formed of the midbrain, pons, and medulla

Page - 1792
Internal Medicine - Neurology

Question 54/82

Question #54

A 40-year-old female patient presented with episodes of vertigo every 4 months. She is complaining
of tinnitus and hearing loss in both ears. Vitals and physical examination are normal. What is the
most likely diagnosis?

a. Meniere’s disease
b. Migraine’s headache
c. Neurofibromatosis
d. Alport syndrome
e. 3rd nerve palsy

‫اﻹﺟﺎﺑﺔ ﻋﲆ اﻟﺼﻔﺤﺔ اﻟﺘﺎﻟﻴﺔ‬

Page - 1793
Internal Medicine - Neurology - Meniere’s Disease

Question 54/82

Question #54

A 40-year-old female patient presented with episodes of vertigo every 4 months. She is complaining
of tinnitus and hearing loss in both ears. Vitals and physical examination are normal. What is the
most likely diagnosis?

a. Meniere’s disease √
b. Migraine’s headache
c. Neurofibromatosis
d. Alport syndrome
e. 3rd nerve palsy

Description

Meniere’s disease is characterized by a triad of vertigo, fluctuating sensorineural hearing loss, and
tinnitus.

Meniere’s disease

It is an episodic attack of tinnitus, hearing loss, and vertigo lasting for minutes to hours
Due to endolymphatic overaccumulation (Endolymphatic hydrops)

Page - 1794
Internal Medicine - Neurology

Question 55/82

Question #55

A 40-year-old female patient presented with episodes of vertigo every 4 months. She is complaining
of tinnitus and hearing loss in both ears. All the following about Meniere’s disease are accurate
except?

a. Endolymphatic overaccumulation is the primary pathology


b. Tinnitus, hearing loss, vertigo, and aural fullness are the main symptoms
c. ere is no diagnostic test for this condition
d. Diuretics and a low-salt diet may decrease the frequency and severity of episodes
e. Surgery never has a rule in the treatment

‫اﻹﺟﺎﺑﺔ ﻋﲆ اﻟﺼﻔﺤﺔ اﻟﺘﺎﻟﻴﺔ‬

Page - 1795
Internal Medicine - Neurology - Meniere’s Disease

Question 55/82

Question #55

A 40-year-old female patient presented with episodes of vertigo every 4 months. She is complaining
of tinnitus and hearing loss in both ears. All the following about Meniere’s disease are accurate
except?

a. Endolymphatic overaccumulation is the primary pathology


b. Tinnitus, hearing loss, vertigo, and aural fullness are the main symptoms
c. ere is no diagnostic test for this condition
d. Diuretics and a low-salt diet may decrease the frequency and severity of episodes
e. Surgery never has a rule in the treatment √

Description

Meniere’s disease is characterized by a triad of vertigo, fluctuating sensorineural hearing loss, and
tinnitus.

Meniere’s disease

It is an episodic attack of tinnitus, hearing loss, and vertigo lasting for minutes to hours
Due to endolymphatic overaccumulation (Endolymphatic hydrops)
Medical treatment includes bed rest, antiemetics, antivertiginous drugs (betahistine,
meclizine, diphenhydramine), and anticholinergics (e.g., scopolamine)
Surgical treatment may be beneficial; the vestibular system can be ablated with topical
gentamicin or surgery

Page - 1796
Internal Medicine - Neurology

Question 56/82

Question #56

Which of the following agents is helpful for prophylactic use for migraine headaches?

a. Dihydroergotamine
b. Amitriptyline
c. Sumatriptan
d. Aspirin
e. Acetaminophen

‫اﻹﺟﺎﺑﺔ ﻋﲆ اﻟﺼﻔﺤﺔ اﻟﺘﺎﻟﻴﺔ‬

Page - 1797
Internal Medicine - Neurology - Migraine Headache

Question 56/82

Question #56

Which of the following agents is helpful for prophylactic use for migraine headaches?

a. Dihydroergotamine
b. Amitriptyline √
c. Sumatriptan
d. Aspirin
e. Acetaminophen

Description

Beta-blockers, antidepressants, anticonvulsants, calcium channel blockers, and serotonin


antagonists are the major classes of drugs used for preventive migraine therapy

Migraine headaches:

Migraine headache is a type of headache that is more common in females.


It can be precipitated by emotional stress, noises, alcohol, caeine, and oral contraceptive pills
It is a recurrent, severe headache that fulfills the POUND criteria: (Disabling, Pulsatile, One-day
duration, Unilateral, and associated with nausea and vomiting).
Patients with more than 3 of the POUND criteria can be diagnosed with migraine without
additional testing
Migraine with aura strongly contributes to stroke when combined with DM, HTN, and obesity.
e episodes of headaches are treated by analgesia (aspirin, paracetamol) and Triptans
Migraine headaches are controlled by prophylactic administration of beta-blockers, valproic
acid, or topiramate.

Page - 1798
Internal Medicine - Neurology

Question 57/82

Question #57

Which of the following is not used as a prophylactic treatment for a migraine headache in adults?

a. Gabapentin
b. Propranolol
c. Amitriptyline
d. Verapamil
e. Naproxen

‫اﻹﺟﺎﺑﺔ ﻋﲆ اﻟﺼﻔﺤﺔ اﻟﺘﺎﻟﻴﺔ‬

Page - 1799
Internal Medicine - Neurology - Migraine Headache

Question 57/82

Question #57

Which of the following is not used as a prophylactic treatment for a migraine headache in adults?

a. Gabapentin
b. Propranolol
c. Amitriptyline
d. Verapamil
e. Naproxen √

Description

Naproxen is used to relieve, not to prevent, migraine headaches

Migraine headaches:

Migraine headache is a type of headache that is more common in females.


It can be precipitated by emotional stress, noises, alcohol, caeine, and oral contraceptive pills
It is a recurrent, severe headache that fulfills the POUND criteria: (Disabling, Pulsatile, One-day
duration, Unilateral, and associated with nausea and vomiting).
Patients with more than 3 of the POUND criteria can be diagnosed with migraine without
additional testing
Migraine with aura strongly contributes to stroke when combined with DM, HTN, and obesity.
e episodes of headaches are treated by analgesia (aspirin, paracetamol) and Triptans
Migraine headaches are controlled by prophylactic administration of beta-blockers, valproic
acid, or topiramate.

Page - 1800
Internal Medicine - Neurology

Question 58/82

Question #58

A 32-year-old female complains of unilateral frontotemporal throbbing headaches for 4 – 6 hours


every 2 – 3 weeks. However, the headache is associated with nausea and occasional vomiting and is
relieved with simple analgesia, sleeping, and massage. Based on history alone, what is the most
likely diagnosis?

a. Acute sinusitis
b. Cluster headache
c. Intracranial mass
d. Migraine headache
e. Tension-type headache

‫اﻹﺟﺎﺑﺔ ﻋﲆ اﻟﺼﻔﺤﺔ اﻟﺘﺎﻟﻴﺔ‬

Page - 1801
Internal Medicine - Neurology - Migraine Headache

Question 58/82

Question #58

A 32-year-old female complains of unilateral frontotemporal throbbing headaches for 4 – 6 hours


every 2 – 3 weeks. However, the headache is associated with nausea and occasional vomiting and is
relieved with simple analgesia, sleeping, and massage. Based on history alone, what is the most
likely diagnosis?

a. Acute sinusitis
b. Cluster headache
c. Intracranial mass
d. Migraine headache √
e. Tension-type headache

Description

Migraine headache is the most likely if young and female. e headaches are unilateral, infrequent,
and throbbing; the headaches are associated with nausea and vomiting, and sleep oers relief.

Migraine headaches:

Migraine headache is a type of headache that is more common in females.


It can be precipitated by emotional stress, noises, alcohol, caeine, and oral contraceptive pills
It is a recurrent, severe headache that fulfills the POUND criteria: (Disabling, Pulsatile, One-day
duration, Unilateral, and associated with nausea and vomiting).
Patients with more than 3 of the POUND criteria can be diagnosed with migraine without
additional testing
Migraine with aura strongly contributes to stroke when combined with DM, HTN, and obesity.
e episodes of headaches are treated by analgesia (aspirin, paracetamol) and Triptans
Migraine headaches are controlled by prophylactic administration of beta-blockers, valproic
acid, or topiramate.

Page - 1802
Internal Medicine - Neurology

Question 59/82

Question #59

A 24-year-old female presents with le-side weakness, blurry vision, and ataxia. 2 years ago, she had
a history of ataxic gait, right-side numbness, and urinary incontinence. What is the most likely
diagnosis?

a. Multiple sclerosis
b. Systemic lupus erythematosus (SLE)
c. Stroke
d. Fibromyalgia
e. Guillain-Barré syndrome

‫اﻹﺟﺎﺑﺔ ﻋﲆ اﻟﺼﻔﺤﺔ اﻟﺘﺎﻟﻴﺔ‬

Page - 1803
Internal Medicine - Neurology - Multiple Sclerosis (MS)

Question 59/82

Question #59

A 24-year-old female presents with le-side weakness, blurry vision, and ataxia. 2 years ago, she had
a history of ataxic gait, right-side numbness, and urinary incontinence. What is the most likely
diagnosis?

a. Multiple sclerosis √
b. Systemic lupus erythematosus (SLE)
c. Stroke
d. Fibromyalgia
e. Guillain-Barré syndrome

Description

MS is diagnosed based on McDonald’s criteria: at least two attacks disseminated in time,


disseminated in place, and typical radiological features.
Multiple sclerosis is defined as a demyelinating autoimmune disease of the CNS
Cell-mediated autoimmune attack of myelin-producing Oligodendrocyte of the CNS
Relapsing-remitting course (80%) – Most common type

Page - 1804
Internal Medicine - Neurology

Question 60/82

Question #60

A 26-year-old female with a history of multiple sclerosis presented with a sudden vision loss in the
right eye. External ocular movements are normal, but funduscopic examination shows pallor of the
optic disk. is patient’s condition is most likely due to demyelination of which of the following?

a. Corpus callosum
b. Oculomotor nerve
c. Trigeminal nerve
d. Optic nerve
e. Visual cortex

‫اﻹﺟﺎﺑﺔ ﻋﲆ اﻟﺼﻔﺤﺔ اﻟﺘﺎﻟﻴﺔ‬

Page - 1805
Internal Medicine - Neurology - Multiple Sclerosis (MS)

Question 60/82

Question #60

A 26-year-old female with a history of multiple sclerosis presented with a sudden vision loss in the
right eye. External ocular movements are normal, but funduscopic examination shows pallor of the
optic disk. is patient’s condition is most likely due to demyelination of which of the following?

a. Corpus callosum
b. Oculomotor nerve
c. Trigeminal nerve
d. Optic nerve √
e. Visual cortex

Description

Localization of the lesion is determined by history and physical examination.


MS is diagnosed based on McDonald’s criteria: at least two attacks disseminated in time,
disseminated in place, and typical radiological features.
Multiple sclerosis is defined as a demyelinating autoimmune disease of the CNS
Cell-mediated autoimmune attack of myelin-producing Oligodendrocyte of the CNS
Relapsing-remitting course (80%) – Most common type

Page - 1806
Internal Medicine - Neurology

Question 61/82

Question #61

A 26-year-old female patient presented with facial numbness, a painful le eye, weak right upper
limb; she intermittently has urine incontinence, and the symptoms are associated with hot weather.
Physical examination shows hyperreflexia. Which of the following will most likely confirm the
diagnosis?

a. EMG
b. EEG
c. Cervical and Brain MRI with contrast
d. Brain CT scan with contrast
e. CSF examination

‫اﻹﺟﺎﺑﺔ ﻋﲆ اﻟﺼﻔﺤﺔ اﻟﺘﺎﻟﻴﺔ‬

Page - 1807
Internal Medicine - Neurology - Multiple Sclerosis (MS)

Question 61/82

Question #61

A 26-year-old female patient presented with facial numbness, a painful le eye, weak right upper
limb; she intermittently has urine incontinence, and the symptoms are associated with hot weather.
Physical examination shows hyperreflexia. Which of the following will most likely confirm the
diagnosis?

a. EMG
b. EEG
c. Cervical and Brain MRI with contrast √
d. Brain CT scan with contrast
e. CSF examination

Description

MRI is the diagnostic modality of choice for multiple sclerosis to ascertain the presence of the
characteristic demyelinating lesions.
MS is diagnosed based on McDonald’s criteria: at least two attacks disseminated in time,
disseminated in place, and typical radiological features.
Multiple sclerosis is defined as a demyelinating autoimmune disease of the CNS
Cell-mediated autoimmune attack of myelin-producing Oligodendrocyte of the CNS
Relapsing-remitting course (80%) – Most common type

Page - 1808
Internal Medicine - Neurology

Question 62/82

Question #62

A 29-year-old female patient presented with diplopia, dysphagia, with choking. You examined her
and noted ptosis, weak facial muscles, and normal tendon reflexes. You suspected a diagnosis of
myasthenia gravis. Which is true regarding this condition?

a. More common in men


b. Anti-acetylcholinesterase antibodies are characteristic
c. ymic hypoplasia is characteristic
d. It may be associated with autoimmune thyroiditis
e. Diagnosed by chest CT or MRI

‫اﻹﺟﺎﺑﺔ ﻋﲆ اﻟﺼﻔﺤﺔ اﻟﺘﺎﻟﻴﺔ‬

Page - 1809
Internal Medicine - Neurology - Myasthenia Gravis (MG)

Question 62/82

Question #62

A 29-year-old female patient presented with diplopia, dysphagia, with choking. You examined her
and noted ptosis, weak facial muscles, and normal tendon reflexes. You suspected a diagnosis of
myasthenia gravis. Which is true regarding this condition?

a. More common in men


b. Anti-acetylcholinesterase antibodies are characteristic
c. ymic hypoplasia is characteristic
d. It may be associated with autoimmune thyroiditis √
e. Diagnosed by chest CT or MRI

Description

Autoimmune diseases are usually associated with other autoimmune diseases.

Myasthenia gravis (MG):

It is an immune-mediated disease in which autoantibodies to acetylcholine receptors in the


neuromuscular junction develop
15% of patients have a thymoma, and many others have thymus hyperplasia
Clinical features include fatigue and weakness, especially the ocular and facial muscles, but all
the body muscles can be aected
e most common cause of death in this disease is respiratory muscle involvement, respiratory
failure, and respiratory infections secondary to aspiration
It is diagnosed by the combination of tensilon test, EMG, and anti-acetylcholine receptor
antibodies > 80% of cases
Chest CT scan should be done for all cases to rule out thymoma, thymus hyperplasia
Pyridostigmine is the treatment of choice for this condition

Page - 1810
Internal Medicine - Neurology

Question 63/82

Question #63

A 29-year-old female patient presented with diplopia, dysphagia, with choking. You examined her
and noted ptosis, weak facial muscles, and normal tendon reflexes. What is the most likely
diagnosis?

a. Guillain-Barré syndrome
b. Myasthenia gravis disease
c. Multiple sclerosis
d. Dermatomyositis
e. Fibromyalgia

‫اﻹﺟﺎﺑﺔ ﻋﲆ اﻟﺼﻔﺤﺔ اﻟﺘﺎﻟﻴﺔ‬

Page - 1811
Internal Medicine - Neurology - Myasthenia Gravis (MG)

Question 63/82

Question #63

A 29-year-old female patient presented with diplopia, dysphagia, with choking. You examined her
and noted ptosis, weak facial muscles, and normal tendon reflexes. What is the most likely
diagnosis?

a. Guillain-Barré syndrome
b. Myasthenia gravis disease √
c. Multiple sclerosis
d. Dermatomyositis
e. Fibromyalgia

Description

e eye muscles are aected first in 40% of people with myasthenia gravis.

Myasthenia gravis (MG):

It is an immune-mediated disease in which autoantibodies to acetylcholine receptors in the


neuromuscular junction develop
15% of patients have a thymoma, and many others have thymus hyperplasia
Clinical features include fatigue and weakness, especially the ocular and facial muscles, but all
the body muscles can be aected
e most common cause of death in this disease is respiratory muscle involvement, respiratory
failure, and respiratory infections secondary to aspiration
It is diagnosed by the combination of tensilon test, EMG, and anti-acetylcholine receptor
antibodies > 80% of cases
Chest CT scan should be done for all cases to rule out thymoma, thymus hyperplasia
Pyridostigmine is the treatment of choice for this condition

Page - 1812
Internal Medicine - Neurology

Question 64/82

Question #64

A 30-year-old female patient with progressive ptosis throughout the day and dysphagia, which of
the following is the most appropriate diagnostic test to confirm the diagnosis of myasthenia gravis?

a. C reactive protein
b. Tensilon test
c. Spinal MRI
d. Chest CT scan
e. Acetylcholine receptor antibody test

‫اﻹﺟﺎﺑﺔ ﻋﲆ اﻟﺼﻔﺤﺔ اﻟﺘﺎﻟﻴﺔ‬

Page - 1813
Internal Medicine - Neurology - Myasthenia Gravis (MG)

Question 64/82

Question #64

A 30-year-old female patient with progressive ptosis throughout the day and dysphagia, which of
the following is the most appropriate diagnostic test to confirm the diagnosis of myasthenia gravis?

a. C reactive protein
b. Tensilon test
c. Spinal MRI
d. Chest CT scan
e. Acetylcholine receptor antibody test √

Description

e anti-acetylcholine receptor antibody test is 100% specific.

It has a sensitivity of 90% in generalized MG but only 50-70% in ocular MG.

Myasthenia gravis (MG):

It is an immune-mediated disease in which autoantibodies to acetylcholine receptors in the


neuromuscular junction develop
15% of patients have a thymoma, and many others have thymus hyperplasia
Clinical features include fatigue and weakness, especially the ocular and facial muscles, but all
the body muscles can be aected
e most common cause of death in this disease is respiratory muscle involvement, respiratory
failure, and respiratory infections secondary to aspiration
It is diagnosed by the combination of tensilon test, EMG, and anti-acetylcholine receptor
antibodies > 80% of cases
Chest CT scan should be done for all cases to rule out thymoma, thymus hyperplasia
Pyridostigmine is the treatment of choice for this condition

Page - 1814
Internal Medicine - Neurology

Question 65/82

Question #65

A 22-year-old depressed lady committed suicide by ingestion of organophosphate insecticide. What


is the primary mechanism of action in organophosphate poisoning?

a. Acetylcholine inhibition
b. Catecholamine activation
c. Cholinesterase activation
d. Cholinesterase inhibition
e. Respiratory center depression

‫اﻹﺟﺎﺑﺔ ﻋﲆ اﻟﺼﻔﺤﺔ اﻟﺘﺎﻟﻴﺔ‬

Page - 1815
Internal Medicine - Neurology - Organophosphate poisoning

Question 65/82

Question #65

A 22-year-old depressed lady committed suicide by ingestion of organophosphate insecticide. What


is the primary mechanism of action in organophosphate poisoning?

a. Acetylcholine inhibition
b. Catecholamine activation
c. Cholinesterase activation
d. Cholinesterase inhibition √
e. Respiratory center depression

Description

Inhibition of the cholinesterase enzyme will lead to enhance in the amount of acetylcholine in the
body and a wide variety of reactions

Organophosphate poisoning:

Organophosphate is a substance that is used widely in insecticides; most of the war gases are
organophosphates as well
e time from exposure to the onset of symptoms is 30 – 120 minutes
Patients should be treated by gastric lavage, washing skin, atropine, Pralidoxime, and other
supportive measures

Page - 1816
Internal Medicine - Neurology

Question 66/82

Question #66

A 22-year-old depressed lady committed suicide by ingestion of organophosphate insecticide. What


is the drug of choice to reverse the condition?

a. Adrenalin
b. Deferoxamine
c. Atropine
d. Acetylcholine
e. Diltiazem

‫اﻹﺟﺎﺑﺔ ﻋﲆ اﻟﺼﻔﺤﺔ اﻟﺘﺎﻟﻴﺔ‬

Page - 1817
Internal Medicine - Neurology - Organophosphate poisoning

Question 66/82

Question #66

A 22-year-old depressed lady committed suicide by ingestion of organophosphate insecticide. What


is the drug of choice to reverse the condition?

a. Adrenalin
b. Deferoxamine
c. Atropine √
d. Acetylcholine
e. Diltiazem

Description

Atropine is the drug of choice to reverse the action of acetylcholine in organophosphate poisoning

Organophosphate poisoning:

Organophosphate is a substance that is used widely in insecticides; most of the war gases are
organophosphates as well
e time from exposure to the onset of symptoms is 30 – 120 minutes
Patients should be treated by gastric lavage, washing skin, atropine, Pralidoxime, and other
supportive measures

Page - 1818
Internal Medicine - Neurology

Question 67/82

Question #67

e correct initial treatment for a patient with Parkinson’s disease is:

a. Sinemet (carbidopa/levodopa) 25mg/100mg tablet three times a day


b. Ginkgo biloba tablets 2 times a day
c. Donepezil
d. Propranolol oral tablets
e. Deep electrical brain stimulation

‫اﻹﺟﺎﺑﺔ ﻋﲆ اﻟﺼﻔﺤﺔ اﻟﺘﺎﻟﻴﺔ‬

Page - 1819
Internal Medicine - Neurology - Parkinson’s Disease (PD)

Question 67/82

Question #67

e correct initial treatment for a patient with Parkinson’s disease is:

a. Sinemet (carbidopa/levodopa) 25mg/100mg tablet three times a day √


b. Ginkgo biloba tablets 2 times a day
c. Donepezil
d. Propranolol oral tablets
e. Deep electrical brain stimulation

Description

Parkinson’s disease (PD) results from the loss of cells in the substantia nigra resulting in decreased
dopamine secretion. It is a clinical syndrome of Bradykinesia, rigidity, tremor, and loss of postural
reflexes.

Sinemet (carbidopa/levodopa) 25mg/100mg tablet three times a day is the best initial treatment for
Parkinson’s disease.

Page - 1820
Internal Medicine - Neurology

Question 68/82

Question #68

Which of the following is the most helpful for confirming the diagnosis of Parkinson’s disease?

a. Brain CT scan
b. Brain MRI
c. Positive response to levodopa
d. Presence of intention tremor
e. Symmetrical bilateral tremor

‫اﻹﺟﺎﺑﺔ ﻋﲆ اﻟﺼﻔﺤﺔ اﻟﺘﺎﻟﻴﺔ‬

Page - 1821
Internal Medicine - Neurology - Parkinson’s Disease (PD)

Question 68/82

Question #68

Which of the following is the most helpful for confirming the diagnosis of Parkinson’s disease?

a. Brain CT scan
b. Brain MRI
c. Positive response to levodopa √
d. Presence of intention tremor
e. Symmetrical bilateral tremor

Description

Patients with Parkinson’s disease should respond to dopamine agonists and levodopa.

Parkinson’s disease (PD) results from the loss of cells in the substantia nigra resulting in decreased
dopamine secretion. It is a clinical syndrome of Bradykinesia, rigidity, tremor, and loss of postural
reflexes.

Sinemet (carbidopa/levodopa) 25mg/100mg tablet three times a day is the best initial treatment for
Parkinson’s disease.

Page - 1822
Internal Medicine - Neurology

Question 69/82

Question #69

A 67-year-old male patent has been a known case of DM and severe Parkinson’s disease for 7 years,
and he is now on Levodopa- Carbidopa. He came to you with constipation, early satiety, GI reflux,
and bloating. Which of the following treatments is contraindicated?

a. Erythromycin
b. Metoclopramide
c. Domperidone
d. Nifedipine
e. Lactulose

‫اﻹﺟﺎﺑﺔ ﻋﲆ اﻟﺼﻔﺤﺔ اﻟﺘﺎﻟﻴﺔ‬

Page - 1823
Internal Medicine - Neurology - Parkinson’s Disease (PD)

Question 69/82

Question #69

A 67-year-old male patent has been a known case of DM and severe Parkinson’s disease for 7 years,
and he is now on Levodopa- Carbidopa. He came to you with constipation, early satiety, GI reflux,
and bloating. Which of the following treatments is contraindicated?

a. Erythromycin
b. Metoclopramide √
c. Domperidone
d. Nifedipine
e. Lactulose

Description

Metoclopramide is a dopamine antagonist and can be used to treat gastroparesis, but it is


contraindicated in patients with severe Parkinson’s disease.

Parkinson’s disease (PD) results from the loss of cells in the substantia nigra resulting in decreased
dopamine secretion. It is a clinical syndrome of Bradykinesia, rigidity, tremor, and loss of postural
reflexes.

Sinemet (carbidopa/levodopa) 25mg/100mg tablet three times a day is the best initial treatment for
Parkinson’s disease.

Page - 1824
Internal Medicine - Neurology

Question 70/82

Question #70

Which of the following diseases is associated with rigidity, tremor, depigmentation of substantia
nigra, and loss of dopaminergic input to basal ganglia?

a. Epilepsy
b. Parkinson disease
c. Bipolar disorder
d. Alzheimer’s disease
e. Huntington disease

‫اﻹﺟﺎﺑﺔ ﻋﲆ اﻟﺼﻔﺤﺔ اﻟﺘﺎﻟﻴﺔ‬

Page - 1825
Internal Medicine - Neurology - Parkinson’s Disease (PD)

Question 70/82

Question #70

Which of the following diseases is associated with rigidity, tremor, depigmentation of substantia
nigra, and loss of dopaminergic input to basal ganglia?

a. Epilepsy
b. Parkinson disease √
c. Bipolar disorder
d. Alzheimer’s disease
e. Huntington disease

Description

Parkinson’s disease (PD) results from the loss of cells in the substantia nigra resulting in decreased
dopamine secretion. It is a clinical syndrome of Bradykinesia, rigidity, tremor, and loss of postural
reflexes.

Sinemet (carbidopa/levodopa) 25mg/100mg tablet three times a day is the best initial treatment for
Parkinson’s disease.

Page - 1826
Internal Medicine - Neurology

Question 71/82

Question #71

A 73-year-old male patient presented with smaller, shuing footsteps and a tendency to fall; all the
following may be found in this patient except:

a. Intention tremor of the hands


b. Loss of facial expressions
c. Rolling resting tremor of the hands
d. Micrographia
e. Bradykinesia

‫اﻹﺟﺎﺑﺔ ﻋﲆ اﻟﺼﻔﺤﺔ اﻟﺘﺎﻟﻴﺔ‬

Page - 1827
Internal Medicine - Neurology - Parkinson’s Disease (PD)

Question 71/82

Question #71

A 73-year-old male patient presented with smaller, shuing footsteps and a tendency to fall; all the
following may be found in this patient except:

a. Intention tremor of the hands √


b. Loss of facial expressions
c. Rolling resting tremor of the hands
d. Micrographia
e. Bradykinesia

Description

Intention tremor is a tremor that gets worse when a person is moving or reaching for an object. It
results from dysfunction of the cerebellum, not Parkinson’s disease.

Parkinson’s disease (PD) results from the loss of cells in the substantia nigra resulting in decreased
dopamine secretion. It is a clinical syndrome of Bradykinesia, rigidity, tremor, and loss of postural
reflexes.

Sinemet (carbidopa/levodopa) 25mg/100mg tablet three times a day is the best initial treatment for
Parkinson’s disease.

Page - 1828
Internal Medicine - Neurology

Question 72/82

Question #72

Which of the following is more likely to manifest in early Parkinson’s disease rather than drug-
induced parkinsonism?

a. Resting tremor
b. Micrographia
c. Shuing gait
d. Bradykinesia
e. Loss of facial expression

‫اﻹﺟﺎﺑﺔ ﻋﲆ اﻟﺼﻔﺤﺔ اﻟﺘﺎﻟﻴﺔ‬

Page - 1829
Internal Medicine - Neurology - Parkinson’s Disease (PD)

Question 72/82

Question #72

Which of the following is more likely to manifest in early Parkinson’s disease rather than drug-
induced parkinsonism?

a. Resting tremor √
b. Micrographia
c. Shuing gait
d. Bradykinesia
e. Loss of facial expression

Description

Resting tremor is more likely to present in Parkinson’s disease than variants such as progressive
supranuclear palsy, multiple system atrophy, and drug-induced parkinsonism

Parkinson’s disease (PD) results from the loss of cells in the substantia nigra resulting in decreased
dopamine secretion. It is a clinical syndrome of Bradykinesia, rigidity, tremor, and loss of postural
reflexes.

Sinemet (carbidopa/levodopa) 25mg/100mg tablet three times a day is the best initial treatment for
Parkinson’s disease.

Page - 1830
Internal Medicine - Neurology

Question 73/82

Question #73

Which one of the following would be the most appropriate treatment to minimize the chance of
post-herpetic neuralgia?

a. Famciclovir
b. Prednisone
c. Capsaicin
d. Carbamazepine
e. Paracetamol

‫اﻹﺟﺎﺑﺔ ﻋﲆ اﻟﺼﻔﺤﺔ اﻟﺘﺎﻟﻴﺔ‬

Page - 1831
Internal Medicine - Neurology - Postherpetic Neuralgia

Question 73/82

Question #73

Which one of the following would be the most appropriate treatment to minimize the chance of
post-herpetic neuralgia?

a. Famciclovir √
b. Prednisone
c. Capsaicin
d. Carbamazepine
e. Paracetamol

Description

Studies show that patients who present for treatment of herpes zoster within 72 hours will benefit
from antiviral therapy such as famciclovir to reduce the pain and decrease the risk of post-herpetic
neuralgia

Tricyclic antidepressants, Pregabalin, Gabapentin, Carbamazepine, and Phenytoin, can be used in


the treatment of post-herpetic neuralgia

Page - 1832
Internal Medicine - Neurology

Question 74/82

Question #74

A 19-year-old obese female comes with bilateral frontal non-throbbing, worsening headaches for 5
months, aggravated by lying flat and relieved by getting up and walking around. Fundus
examination shows mild optic disk blurring, and her brain MRI is normal. e most appropriate next
step is to:

a. Obtain an EEG
b. Neurosurgical consult
c. Perform a lumbar puncture
d. Repeat Brain MRI
e. Start propranolol

‫اﻹﺟﺎﺑﺔ ﻋﲆ اﻟﺼﻔﺤﺔ اﻟﺘﺎﻟﻴﺔ‬

Page - 1833
Internal Medicine - Neurology - Pseudotumor Cerebri

Question 74/82

Question #74

A 19-year-old obese female comes with bilateral frontal non-throbbing, worsening headaches for 5
months, aggravated by lying flat and relieved by getting up and walking around. Fundus
examination shows mild optic disk blurring, and her brain MRI is normal. e most appropriate next
step is to:

a. Obtain an EEG
b. Neurosurgical consult
c. Perform a lumbar puncture √
d. Repeat Brain MRI
e. Start propranolol

Description

For Pseudotumor cerebri, LP will show an elevated opening pressure and normal CSF analysis.

Benign intracranial HTN (BIH):

Also known as Pseudotumor cerebri, or idiopathic intracranial HTN (IIH)


ere is an increase in intracranial pressure without identifiable structural pathology
90% of aected patients are female
Associated with obesity, venous sinus thrombosis, OCP use, and vitamin A toxicity

Page - 1834
Internal Medicine - Neurology

Question 75/82

Question #75

A 22-year-old college student presented with tightening, band-like headaches more frequently in
the morning. What is the most likely diagnosis?

a. Cluster headache
b. Migraine headache
c. Premenstrual headache
d. Sinus headache
e. Tension headache

‫اﻹﺟﺎﺑﺔ ﻋﲆ اﻟﺼﻔﺤﺔ اﻟﺘﺎﻟﻴﺔ‬

Page - 1835
Internal Medicine - Neurology - Tension-type headache

Question 75/82

Question #75

A 22-year-old college student presented with tightening, band-like headaches more frequently in
the morning. What is the most likely diagnosis?

a. Cluster headache
b. Migraine headache
c. Premenstrual headache
d. Sinus headache
e. Tension headache √

Description

A classic presentation of tension-type headache.

Tension-type headache:

It is the most common type of headache


It presents as mild to moderate bilateral squeezing, dull, like pressure in nature, and non-
throbbing. However, it usually radiates forward from the occipital region and is precipitated by
a stressful event.
Treatment is achieved by rest, painkillers (like paracetamol and NSAIDs), and antidepressants.

Page - 1836
Internal Medicine - Neurology

Question 76/82

Question #76

All the following cranial nerves are matched correctly with their type except:

a. Abducent nerve → Motor


b. Facial nerve → Mixed
c. Vagus nerve → Motor.
d. Olfactory nerve → Sensory
e. Optic nerve → Sensory

‫اﻹﺟﺎﺑﺔ ﻋﲆ اﻟﺼﻔﺤﺔ اﻟﺘﺎﻟﻴﺔ‬

Page - 1837
Internal Medicine - Neurology - e Cranial Nerves

Question 76/82

Question #76

All the following cranial nerves are matched correctly with their type except:

a. Abducent nerve → Motor


b. Facial nerve → Mixed
c. Vagus nerve → Motor. √
d. Olfactory nerve → Sensory
e. Optic nerve → Sensory

Description

e Vagus nerve contains parasympathetic nerve fibers (Autonomic).

e Vagus nerve (CN X):

Name: Vagus nerve (autonomic)


Function: Parasympathetic control of the heart, lungs, and digestive tract.
e longest nerve of the autonomic nervous system in the human body

Page - 1838
Internal Medicine - Neurology

Question 77/82

Question #77

A 35-year-old man is a known case of polycystic kidney disease. He came to the emergency
department with a sudden severe headache and neck stiness. However, his brain CT scan is shown
in the picture below. What is the most likely diagnosis?

a. Tension-type headache
b. Subarachnoid hemorrhage
c. Subdural hemorrhage
d. Epidural hemorrhage
e. Interventricular hemorrhage

‫اﻹﺟﺎﺑﺔ ﻋﲆ اﻟﺼﻔﺤﺔ اﻟﺘﺎﻟﻴﺔ‬

Page - 1839
Internal Medicine - Neurology - underclap headache

Question 77/82

Question #77

A 35-year-old man is a known case of polycystic kidney disease. He came to the emergency
department with a sudden severe headache and neck stiness. However, his brain CT scan is shown
in the picture below. What is the most likely diagnosis?

a. Tension-type headache
b. Subarachnoid hemorrhage √
c. Subdural hemorrhage
d. Epidural hemorrhage
e. Interventricular hemorrhage

Description

Berry aneurysm is associated with polycystic kidney disease and carries a risk of rupture and
subarachnoid hemorrhage.

Page - 1840
Subarachnoid hemorrhage (SAH):

It is the most common cause of thunderclap headache

85% are caused by a ruptured saccular berry aneurysm


Clinical features include thunderclap headache, neck rigidity, photophobia, and vomiting.
Because CT scans may be negative in SAH, cerebral angiography is the best investigation.
Lumbar puncture (LP) showing xanthochromia, high protein, high pressure, and high RBC
count is typical, especially 12 hours aer the bleeding onset.

Page - 1841
Internal Medicine - Neurology

Question 78/82

Question #78

A 53-year-old female presents with a sudden thunderclap headache, nausea, and photophobia
associated with a progressively reduced level of consciousness. Which one of the following would be
the best next step?

a. Brain CT angiography
b. Brain CT scan with contrast
c. Brain CT scan without contrast
d. Brain MRI
e. LP and CSF analysis

‫اﻹﺟﺎﺑﺔ ﻋﲆ اﻟﺼﻔﺤﺔ اﻟﺘﺎﻟﻴﺔ‬

Page - 1842
Internal Medicine - Neurology - underclap headache

Question 78/82

Question #78

A 53-year-old female presents with a sudden thunderclap headache, nausea, and photophobia
associated with a progressively reduced level of consciousness. Which one of the following would be
the best next step?

a. Brain CT angiography
b. Brain CT scan with contrast
c. Brain CT scan without contrast √
d. Brain MRI
e. LP and CSF analysis

Description

In a patient with suspected subarachnoid hemorrhage, the first study to do is a CT scan without
contrast

Subarachnoid hemorrhage (SAH):

It is the most common cause of thunderclap headache

85% are caused by a ruptured saccular berry aneurysm


Clinical features include thunderclap headache, neck rigidity, photophobia, and vomiting.
Because CT scans may be negative in SAH, cerebral angiography is the best investigation.
Lumbar puncture (LP) showing xanthochromia, high protein, high pressure, and high RBC
count is typical, especially 12 hours aer the bleeding onset.

Page - 1843
Internal Medicine - Neurology

Question 79/82

Question #79

A 35-year-old female presents with a 14-hour history of headaches described as the “Worst headache
in her life” her BP is 140/90, and the rest of her vital signs are normal. ere are no focal neurologic
findings, but neck stiness is present. Unenhanced CT of the head is negative. Which one of the
following findings in the bloody spinal fluid would indicate a diagnosis of subarachnoid
hemorrhage?

a. Yellowish discoloration of the fluid


b. A WBC:RBC ratio of 1/1000
c. A protein to RBC of 1/1000
d. A glucose level below 2.2 mmol/L
e. A CSF pressure of 17 cmH2O

‫اﻹﺟﺎﺑﺔ ﻋﲆ اﻟﺼﻔﺤﺔ اﻟﺘﺎﻟﻴﺔ‬

Page - 1844
Internal Medicine - Neurology - underclap headache

Question 79/82

Question #79

A 35-year-old female presents with a 14-hour history of headaches described as the “Worst headache
in her life” her BP is 140/90, and the rest of her vital signs are normal. ere are no focal neurologic
findings, but neck stiness is present. Unenhanced CT of the head is negative. Which one of the
following findings in the bloody spinal fluid would indicate a diagnosis of subarachnoid
hemorrhage?

a. Yellowish discoloration of the fluid √


b. A WBC:RBC ratio of 1/1000
c. A protein to RBC of 1/1000
d. A glucose level below 2.2 mmol/L
e. A CSF pressure of 17 cmH2O

Description

Xanthochromia is caused by the lysis of RBCs leading to a yellow-orange color of the CSF

e WBC:RBC ratio is more helpful in diagnosing meningitis.

Subarachnoid hemorrhage (SAH):

It is the most common cause of thunderclap headache

85% are caused by a ruptured saccular berry aneurysm


Clinical features include thunderclap headache, neck rigidity, photophobia, and vomiting.
Because CT scans may be negative in SAH, cerebral angiography is the best investigation.
Lumbar puncture (LP) showing xanthochromia, high protein, high pressure, and high RBC
count is typical, especially 12 hours aer the bleeding onset.

Page - 1845
Internal Medicine - Neurology

Question 80/82

Question #80

A 43-year-old man complains of severe, sharp, lancinating pain in his face that is aggravated by
touching a specific area. Neuro-examination is normal. Brain MRI with contrast shows no
abnormalities of the trigeminal nerve. Which is the most appropriate initial treatment?

a. Anticonvulsants
b. Aspirin
c. Nimodipine
d. NSAIDs
e. Vasodilators

‫اﻹﺟﺎﺑﺔ ﻋﲆ اﻟﺼﻔﺤﺔ اﻟﺘﺎﻟﻴﺔ‬

Page - 1846
Internal Medicine - Neurology - Trigeminal Neuralgia

Question 80/82

Question #80

A 43-year-old man complains of severe, sharp, lancinating pain in his face that is aggravated by
touching a specific area. Neuro-examination is normal. Brain MRI with contrast shows no
abnormalities of the trigeminal nerve. Which is the most appropriate initial treatment?

a. Anticonvulsants √
b. Aspirin
c. Nimodipine
d. NSAIDs
e. Vasodilators

Description

In trigeminal neuralgia, carbamazepine is usually the first choice, but phenytoin has also been used.

Trigeminal neuralgia:

A disease of unknown cause but may be due to compression at the fih cranial nerve root.
Presents with a unilateral lancinating facial pain in the 2nd and 3rd division of trigeminal
nerve territories, which may remit and relapse over a year
e pain may be precipitated by touching the face, mastication, or even pronouncing certain
words.
It is treated by Carbamazepine (First line)
Pregabalin, Gabapentin, and Steroids are used if the patient can’t tolerate carbamazepine)
Surgical treatment is the last resort: Decompression of the trigeminal nerve root

Page - 1847
Internal Medicine - Neurology

Question 81/82

Question #81

A man has shooting pain when he touches his face. You suspect trigeminal neuralgia. What is the
treatment of choice for this condition?

a. SSRI’s
b. Oral Prednisolone
c. Acyclovir
d. Carbamazepine
e. NSAIDs

‫اﻹﺟﺎﺑﺔ ﻋﲆ اﻟﺼﻔﺤﺔ اﻟﺘﺎﻟﻴﺔ‬

Page - 1848
Internal Medicine - Neurology - Trigeminal Neuralgia

Question 81/82

Question #81

A man has shooting pain when he touches his face. You suspect trigeminal neuralgia. What is the
treatment of choice for this condition?

a. SSRI’s
b. Oral Prednisolone
c. Acyclovir
d. Carbamazepine √
e. NSAIDs

Description

carbamazepine or gabapentin can be used in the treatment of trigeminal neuralgia

Trigeminal neuralgia:

A disease of unknown cause but may be due to compression at the fih cranial nerve root.
Presents with a unilateral lancinating facial pain in the 2nd and 3rd division of trigeminal
nerve territories, which may remit and relapse over a year
e pain may be precipitated by touching the face, mastication, or even pronouncing certain
words.
It is treated by Carbamazepine (First line)
Pregabalin, Gabapentin, and Steroids are used if the patient can’t tolerate carbamazepine)
Surgical treatment is the last resort: Decompression of the trigeminal nerve root

Page - 1849
Internal Medicine - Neurology

Question 82/82

Question #82

A 60-year-old-male patient came with right lower limb twitching and weakness. On examination,
he has fasciculations, decreased reflexes, and a down-going plantar reflex in the same leg. is
lesion should be suspected in:

a. A Lower Motor Neuron (LMN)


b. Both upper and lower motor neurons
c. An Upper Motor Neuron (UMN)
d. An Extrapyramidal Neuron
e. e tibiofemoral joint space

‫اﻹﺟﺎﺑﺔ ﻋﲆ اﻟﺼﻔﺤﺔ اﻟﺘﺎﻟﻴﺔ‬

Page - 1850
Internal Medicine - Neurology - Upper vs. Lower Neuron lesion

Question 82/82

Question #82

A 60-year-old-male patient came with right lower limb twitching and weakness. On examination,
he has fasciculations, decreased reflexes, and a down-going plantar reflex in the same leg. is
lesion should be suspected in:

a. A Lower Motor Neuron (LMN) √


b. Both upper and lower motor neurons
c. An Upper Motor Neuron (UMN)
d. An Extrapyramidal Neuron
e. e tibiofemoral joint space

Description

Fasciculations, hyporeflexia, and down-growing plantar reflexes are features of lower motor neuron
lesions.

e following table demonstrates the dierence between upper and lower motor neuron lesions
according to symptoms and physical examination.

Page - 1851
Pulmonology

Page - 1852
Pulmonology

‫ﻣﻮاﺿﻴﻊ اﻷﺳﺌﻠﺔ وأﻋﺪادﻫﺎ‬

1) A-a Gradient => 1 Questions


2) Adult Respiratory Distress Syndrome (ARDS) => 1 Questions
3) Asbestosis => 3 Questions
4) Chronic Obstructive Pulmonary disease (COPD) => 20 Questions
5) Drug-induced lung fibrosis => 1 Questions
6) Eosinophilic lung disease => 1 Questions
7) Extrinsic Allergic Alveolitis (EAA) => 2 Questions
8) Idiopathic Pulmonary Fibrosis (IPF) => 2 Questions
9) Lung Abscess => 3 Questions
10) Pleural Eusion 1 => 7 Questions
11) Pulmonary Function Test (PFT) => 9 Questions
12) Pulmonary Hypertension => 2 Questions
13) Respiratory Alkalosis => 5 Questions
14) Respiratory neoplasms => 3 Questions
15) Sarcoidosis => 12 Questions
16) Silicosis => 1 Questions
17) Tuberculosis (TB) => 8 Questions
18) Asthma => 36 Questions
19) Bronchiectasis => 8 Questions
20) Pneumonia => 13 Questions
21) Sleep apnea syndrome => 6 Questions

Page - 1853
Internal Medicine - Pulmonology

Question 1/144

Question #1

e dierence between Alveolar-arterial PO2 may occur in which one of the following?

a. COPD
b. Hypoventilation
c. Stroke
d. High altitude
e. Anemia

‫اﻹﺟﺎﺑﺔ ﻋﲆ اﻟﺼﻔﺤﺔ اﻟﺘﺎﻟﻴﺔ‬

Page - 1854
Internal Medicine - Pulmonology - A-a Gradient

Question 1/144

Question #1

e dierence between Alveolar-arterial PO2 may occur in which one of the following?

a. COPD √
b. Hypoventilation
c. Stroke
d. High altitude
e. Anemia

Description

A-a gradient measures the ecacy of gas exchange between alveoli (A) and the artery (a)

How to calculate:

A – a gradient = PAO2 – PaO2


PaO2 is measured by ABGs
PAO2 = FiO2 * (760 – 47) – (1.25 * PaCO2)
e normal A-a gradient = 0.3 * age in years

Interpretation:

An abnormal A-a gradient suggests the lungs as a possible cause of hypoxemia


e normal A-a gradient suggests causes external to the lung as a cause for hypoxemia

Page - 1855
Internal Medicine - Pulmonology

Question 2/144

Question #2

A 32-year-old male patient was admitted to the surgical ICU aer sustaining a road trac accident.
e patient developed shortness of breath and tachypnea. His respiratory rate was 39 cycles per
minute, and he was distressed. ABG’s show PH of 7.47, PaO2 of 44 mmHg, PaO2/FiO2 ratio is 140. you
intubated the patient and started mechanical ventilation with high PEEP, but hypoxia is not
improving. What is the most likely diagnosis?

a. Respiratory depression due to brain stem injury


b. Pulmonary edema due to heart failure
c. Adult respiratory distress syndrome
d. Adrenal crisis
e. Tension pneumothorax

‫اﻹﺟﺎﺑﺔ ﻋﲆ اﻟﺼﻔﺤﺔ اﻟﺘﺎﻟﻴﺔ‬

Page - 1856
Internal Medicine - Pulmonology - Adult Respiratory Distress Syndrome (ARDS)

Question 2/144

Question #2

A 32-year-old male patient was admitted to the surgical ICU aer sustaining a road trac accident.
e patient developed shortness of breath and tachypnea. His respiratory rate was 39 cycles per
minute, and he was distressed. ABG’s show PH of 7.47, PaO2 of 44 mmHg, PaO2/FiO2 ratio is 140. you
intubated the patient and started mechanical ventilation with high PEEP, but hypoxia is not
improving. What is the most likely diagnosis?

a. Respiratory depression due to brain stem injury


b. Pulmonary edema due to heart failure
c. Adult respiratory distress syndrome √
d. Adrenal crisis
e. Tension pneumothorax

Description

Respiratory distress and hypoxia aer a road trac accident or any extensive injury along with low
PaO2/FiO2 will raise the suspicion of ARDS, and the patient will need intubation and mechanical
ventilation with high PEEP and low Tidal volume

Adult respiratory distress syndrome:

ARDS: is a respiratory failure from lung injury or due to systemic disease


Presented with Secondary hypoxia + distress + non-cardiogenic pulmonary edema
Prognosis is Poor, Mortality 30-40%
It is induced by damage to the lung (e.g., sepsis, aspiration, PE, etc.)
It is treated by intubation, high PEEP and low Tidal volume, and supportive therapy.

Page - 1857
Internal Medicine - Pulmonology

Question 3/144

Question #3

A 60-year-old male patient with a long history of asbestos exposure presents for follow-up. Which
of the following conditions are not caused by asbestos exposure?

a. High risk of lung cancer


b. Benign pleural plaques
c. Malignant pleural involvement
d. Pleural thickening
e. An obstructive pattern on PFT

‫اﻹﺟﺎﺑﺔ ﻋﲆ اﻟﺼﻔﺤﺔ اﻟﺘﺎﻟﻴﺔ‬

Page - 1858
Internal Medicine - Pulmonology - Asbestosis

Question 3/144

Question #3

A 60-year-old male patient with a long history of asbestos exposure presents for follow-up. Which
of the following conditions are not caused by asbestos exposure?

a. High risk of lung cancer


b. Benign pleural plaques
c. Malignant pleural involvement
d. Pleural thickening
e. An obstructive pattern on PFT √

Description

Asbestos exposure is known to cause lower-zone lung fibrosis. erefore, the pulmonary function
test will show a restrictive pattern, not an obstructive one.

e following are the diseases that are possible to be found in asbestos exposure:

Pleural plaques are the most common asbestos-related lung disease (Benign with no malignant
transformation)

Asbestos may cause pleural thickening


Asbestosis (lower lobe fibrosis)
Lung cancer: Asbestosis and smoking together increase the risk 50 times
Mesothelioma is a malignant asbestos-related disease of the pleura

Page - 1859
Internal Medicine - Pulmonology

Question 4/144

Question #4

A 50-year-old male patient presents with shortness of breath, dry cough, and Velcro-like crepitation
on both lower zones. He previously worked in a factory that manufactures helmets “using asbestos”
for 30 years before retiring 5 years ago. What is the best next step in the management of this
patient?

a. Lung biopsy for asbestos bodies


b. Look for pleural plaques by chest x-ray
c. Look for an obstructive pattern on PFT
d. Order ANA titer
e. Chest CT scan

‫اﻹﺟﺎﺑﺔ ﻋﲆ اﻟﺼﻔﺤﺔ اﻟﺘﺎﻟﻴﺔ‬

Page - 1860
Internal Medicine - Pulmonology - Asbestosis

Question 4/144

Question #4

A 50-year-old male patient presents with shortness of breath, dry cough, and Velcro-like crepitation
on both lower zones. He previously worked in a factory that manufactures helmets “using asbestos”
for 30 years before retiring 5 years ago. What is the best next step in the management of this
patient?

a. Lung biopsy for asbestos bodies


b. Look for pleural plaques by chest x-ray √
c. Look for an obstructive pattern on PFT
d. Order ANA titer
e. Chest CT scan

Description

is patient is suspected of having asbestoses. e chest x-ray is the best initial test for this
condition.

In patients with asbestosis, pulmonary function studies reveal restrictive lung disease, and pleural
plaques in the diaphragmatic pleura on X-ray are considered a reliable indicator of asbestos
exposure

Page - 1861
Internal Medicine - Pulmonology

Question 5/144

Question #5

A 61-year-old male patient is found to have bilateral pleural plaques incidentally during a routine
examination. You suspect occupational exposure. What is the most likely cause of this condition?

a. Silica dust
b. Smoking
c. Asbestos
d. Fungal spores
e. Coal dust

‫اﻹﺟﺎﺑﺔ ﻋﲆ اﻟﺼﻔﺤﺔ اﻟﺘﺎﻟﻴﺔ‬

Page - 1862
Internal Medicine - Pulmonology - Asbestosis

Question 5/144

Question #5

A 61-year-old male patient is found to have bilateral pleural plaques incidentally during a routine
examination. You suspect occupational exposure. What is the most likely cause of this condition?

a. Silica dust
b. Smoking
c. Asbestos √
d. Fungal spores
e. Coal dust

Description

Pleural plaques are the most common asbestos-related lung disease.

is condition is a benign condition with no malignant transformation

Other diseases caused by asbestos exposure are pleural thickening, asbestosis (fibrosis), lung cancer,
and mesothelioma.

Page - 1863
Internal Medicine - Pulmonology

Question 6/144

Question #6

A 65-year-old male patient has had his spirometer that confirms the presence of fixed obstruction.
However, he is a heavy smoker. e following treatments are associated with a decreased mortality
rate in this patient except:

a. Salmeterol
b. Smoking cessation
c. Lung volume reduction surgery
d. Long-term oxygen therapy
e. Pneumococcal vaccination

‫اﻹﺟﺎﺑﺔ ﻋﲆ اﻟﺼﻔﺤﺔ اﻟﺘﺎﻟﻴﺔ‬

Page - 1864
Internal Medicine - Pulmonology - Chronic Obstructive Pulmonary disease (COPD)

Question 6/144

Question #6

A 65-year-old male patient has had his spirometer that confirms the presence of fixed obstruction.
However, he is a heavy smoker. e following treatments are associated with a decreased mortality
rate in this patient except:

a. Salmeterol √
b. Smoking cessation
c. Lung volume reduction surgery
d. Long-term oxygen therapy
e. Pneumococcal vaccination

Description

Salmeterol is a long-acting beta 2 agonist used for the symptomatic treatment of COPD patients,
but it doesn’t reduce the mortality rate.

Treatment of COPD:

Measures to reduce mortality:


Smoking cessation (the most important)
H. Influenza and pneumococcal vaccine
Long-Term Oxygen erapy (LTOT).
Measure to reduce symptoms:
SABA (More rapid onset)
SAMA (Most eective in COPD but slower than SABA)
LABA (More sustained eect)
ICS + LABA (ICS is not used as monotherapy in COPD)
Surgical measures (both morality and symptomatic benefits):
Volume reduction
Lung transplant

Page - 1865
Internal Medicine - Pulmonology

Question 7/144

Question #7

A 66-year-old presents to the ER with COPD exacerbation. He recently developed an increasing


amount of purulent sputum. All the following are indicated in the treatment except:

a. Supplemental Oxygen
b. Ipratropium nebulizer
c. Intravenous methylprednisolone
d. TMP/SMX
e. Inhaled corticosteroids

‫اﻹﺟﺎﺑﺔ ﻋﲆ اﻟﺼﻔﺤﺔ اﻟﺘﺎﻟﻴﺔ‬

Page - 1866
Internal Medicine - Pulmonology - Chronic Obstructive Pulmonary disease (COPD)

Question 7/144

Question #7

A 66-year-old presents to the ER with COPD exacerbation. He recently developed an increasing


amount of purulent sputum. All the following are indicated in the treatment except:

a. Supplemental Oxygen
b. Ipratropium nebulizer
c. Intravenous methylprednisolone
d. TMP/SMX
e. Inhaled corticosteroids √

Description

Inhaled corticosteroids are not indicated in acute COPD exacerbation.

Treatment of COPD exacerbation:

O2 therapy:
O2 saturation target is 88 – 92% until ABGs available
If there is no CO2 retention, make the target 94 – 98%
Bronchodilator (SABA+SAMA) with a back-to-back nebulizer
Give prednisolone 40 mg daily for 5 days
Oral antibiotics (Amoxicillin, Tetracycline, or clarithromycin) only if there are purulent sputum

Page - 1867
Internal Medicine - Pulmonology

Question 8/144

Question #8

A 32-year-old male patient who was previously healthy presented to you with recurrent shortness of
breath for 6 months duration. Aer a pulmonary function test was performed, a diagnosis of COPD
was made. Of the following, what is the most accurate test to diagnose this disease?

a. Chest x-ray
b. Alpha 1 antitrypsin level
c. Bronchoscopy biopsy
d. Positron emission tomography scan
e. Chest CT scan

‫اﻹﺟﺎﺑﺔ ﻋﲆ اﻟﺼﻔﺤﺔ اﻟﺘﺎﻟﻴﺔ‬

Page - 1868
Internal Medicine - Pulmonology - Chronic Obstructive Pulmonary disease (COPD)

Question 8/144

Question #8

A 32-year-old male patient who was previously healthy presented to you with recurrent shortness of
breath for 6 months duration. Aer a pulmonary function test was performed, a diagnosis of COPD
was made. Of the following, what is the most accurate test to diagnose this disease?

a. Chest x-ray
b. Alpha 1 antitrypsin level √
c. Bronchoscopy biopsy
d. Positron emission tomography scan
e. Chest CT scan

Description

Any young patient with COPD should be tested for alpha 1 antitrypsin level. A1AT deficiency is the
most common cause of COPD in young patients.

Alpha-1-antitrypsin deficiency:

Deficiency of alpha-1-antitrypsin due to a genetic problem


Autosomal recessive condition (located on chromosome 14)
Normally, PiMM
Homozygous PiSS (50% normal A1AT levels)
Homozygous PiZZ (10% normal A1AT levels)
Alpha-1-antitrypsin is a protease inhibitor protein synthesized by the liver
A1AT deficiency presents with a combination of COPD and liver cirrhosis
COPD (usually Emphysema) in young (< 40 years) and non-smoker patients is considered A1AT
deficiency until proven otherwise

Page - 1869
Internal Medicine - Pulmonology

Question 9/144

Question #9

A 70-year-old heavy smoker presents with 6 months history of shortness of breath and productive
cough most days. His FEV1 is 43% of predicted, FEV1/FVC ratio is < 70%, and the total lung capacity
(TLC) is elevated. Which of the following is most likely to positively aect his survival?

a. Inhaled corticosteroids
b. Inhaled salmeterol
c. Prophylactic antibiotics
d. Oxygen supplementation
e. Short-acting muscarinic antagonists

‫اﻹﺟﺎﺑﺔ ﻋﲆ اﻟﺼﻔﺤﺔ اﻟﺘﺎﻟﻴﺔ‬

Page - 1870
Internal Medicine - Pulmonology - Chronic Obstructive Pulmonary disease (COPD)

Question 9/144

Question #9

A 70-year-old heavy smoker presents with 6 months history of shortness of breath and productive
cough most days. His FEV1 is 43% of predicted, FEV1/FVC ratio is < 70%, and the total lung capacity
(TLC) is elevated. Which of the following is most likely to positively aect his survival?

a. Inhaled corticosteroids
b. Inhaled salmeterol
c. Prophylactic antibiotics
d. Oxygen supplementation √
e. Short-acting muscarinic antagonists

Description

LTOT is used for those whose PaO2 is less than 55 mmHg

24 hours use is more eective than 12 hours of nocturnal use

It is eective to:

Reduce the mortality rate


Reduces the hemoglobin level
Facilitate sleeping
Increases the exercise tolerance

Indications for LTOT:

PaO2 < 55 mmHg (resting or exercise)


O2 saturation < 88% (resting or exercise)
PaO2 < 60 mmHg with heart disease or polycythemia
O2 saturation < 90% with heart disease or polycythemia

Instructions for LTOT usage:

Maintain O2 saturation > 90% and PaO2 > 60 mmHg


Devices used: Nasal cannula or venturi mask
Duration: at least 15 hours per day
Page - 1871
Page - 1872
Internal Medicine - Pulmonology

Question 10/144

Question #10

A known case of COPD 50-year-old male presents with purulent sputum and shortness of breath for
the last 3 days. On examination, widespread wheezes and coarse crackles are heard on the right lung
base. e patient is not distressed and has stable vital signs. Which of the following is used to treat
his acute COPD exacerbation?

a. Long-acting beta agonists (LABA)


b. Oral prednisolone
c. Metronidazole
d. 100% oxygen
e. Tiotropium

‫اﻹﺟﺎﺑﺔ ﻋﲆ اﻟﺼﻔﺤﺔ اﻟﺘﺎﻟﻴﺔ‬

Page - 1873
Internal Medicine - Pulmonology - Chronic Obstructive Pulmonary disease (COPD)

Question 10/144

Question #10

A known case of COPD 50-year-old male presents with purulent sputum and shortness of breath for
the last 3 days. On examination, widespread wheezes and coarse crackles are heard on the right lung
base. e patient is not distressed and has stable vital signs. Which of the following is used to treat
his acute COPD exacerbation?

a. Long-acting beta agonists (LABA)


b. Oral prednisolone √
c. Metronidazole
d. 100% oxygen
e. Tiotropium

Description

Systemic steroids (oral or IV) shorten hospital stays and decreases treatment failures
LABA and Tiotropium (LAMA) are not used in acute COPD exacerbation but are used to control
the disease and reduce exacerbation recurrence
100% O2 is contraindicated in acute COPD exacerbation as it can cause respiratory failure
Metronidazole is not used in acute COPD exacerbation.
e antibiotic that is used in COPD exacerbation should cover the most likely organisms to
cause this exacerbation (S pneumonia, H influenza, and Moraxella catarrhalis

Treatment of COPD exacerbation:

O2 therapy:
O2 saturation target is 88 – 92% until ABGs available
If there is no CO2 retention, make the target 94 – 98%
Bronchodilator (SABA+SAMA) with a back-to-back nebulizer
Give prednisolone 40 mg daily for 5 days
Oral antibiotics (Amoxicillin, Tetracycline, or clarithromycin) only if there are purulent sputum

Page - 1874
Internal Medicine - Pulmonology

Question 11/144

Question #11

Which of the following does not indicate admission in a patient with COPD exacerbation?

a. Type 2 respiratory failure


b. ABGs showing PH of 7.31
c. Decreased level of consciousness
d. Coughs and Wheezy chest
e. Accessory muscle use

‫اﻹﺟﺎﺑﺔ ﻋﲆ اﻟﺼﻔﺤﺔ اﻟﺘﺎﻟﻴﺔ‬

Page - 1875
Internal Medicine - Pulmonology - Chronic Obstructive Pulmonary disease (COPD)

Question 11/144

Question #11

Which of the following does not indicate admission in a patient with COPD exacerbation?

a. Type 2 respiratory failure


b. ABGs showing PH of 7.31
c. Decreased level of consciousness
d. Coughs and Wheezy chest √
e. Accessory muscle use

Description

In COPD exacerbations, coughs and wheezy chest are expected regardless of the severity of the
exacerbation.

Type 2 respiratory failure and respiratory acidosis require ventilator or NIV support

e use of accessory muscles indicates impending respiratory failure and life-threatening attacks.

Decreased level of consciousness indicates life-threatening COPD exacerbation.

Page - 1876
Internal Medicine - Pulmonology

Question 12/144

Question #12

A 75-year-old male patient with longstanding COPD presents with slowly progressive shortness of
breath over the past year. His medications include a salbutamol inhaler (as required), an ipratropium
bromide inhaler (as required), and a salmeterol inhaler (twice daily). His oxygen saturation is 89%.
Which of the following is considered an indication for long-term oxygen therapy in this patient?

a. PaO2 of 58 mmHg
b. Recurrent infective exacerbations
c. Low exercise tolerance
d. Decreased level of consciousness
e. Presence of polycythemia

‫اﻹﺟﺎﺑﺔ ﻋﲆ اﻟﺼﻔﺤﺔ اﻟﺘﺎﻟﻴﺔ‬

Page - 1877
Internal Medicine - Pulmonology - Chronic Obstructive Pulmonary disease (COPD)

Question 12/144

Question #12

A 75-year-old male patient with longstanding COPD presents with slowly progressive shortness of
breath over the past year. His medications include a salbutamol inhaler (as required), an ipratropium
bromide inhaler (as required), and a salmeterol inhaler (twice daily). His oxygen saturation is 89%.
Which of the following is considered an indication for long-term oxygen therapy in this patient?

a. PaO2 of 58 mmHg
b. Recurrent infective exacerbations
c. Low exercise tolerance
d. Decreased level of consciousness
e. Presence of polycythemia √

Description

LTOT has a mortality benefit in COPD patients

Indications for LTOT:

PaO2 < 55 mmHg (resting or exercise)


O2 saturation < 88% (resting or exercise)
PaO2 < 60 mmHg with heart disease or polycythemia
O2 saturation < 90% with heart disease or polycythemia

Instructions for LTOT usage:

Maintain O2 saturation > 90% and PaO2 > 60 mmHg


Devices used: Nasal cannula or venturi mask
Duration: at least 15 hours per day

Page - 1878
Internal Medicine - Pulmonology

Question 13/144

Question #13

A 70-year-old male who is known case of COPD presents to the ER with COPD exacerbation; all the
following treatments are indicated except:

a. Salbutamol
b. Ipratropium bromide
c. A 100% oxygen flow
d. Prednisolone
e. Intravenous hydrocortisone

‫اﻹﺟﺎﺑﺔ ﻋﲆ اﻟﺼﻔﺤﺔ اﻟﺘﺎﻟﻴﺔ‬

Page - 1879
Internal Medicine - Pulmonology - Chronic Obstructive Pulmonary disease (COPD)

Question 13/144

Question #13

A 70-year-old male who is known case of COPD presents to the ER with COPD exacerbation; all the
following treatments are indicated except:

a. Salbutamol
b. Ipratropium bromide
c. A 100% oxygen flow √
d. Prednisolone
e. Intravenous hydrocortisone

Description

100% oxygen is contraindicated in CO2 retainers patients (COPD and type 2 respiratory failure)
because it can suppress the respiratory center and lead to respiratory failure.

Treatment of COPD exacerbation:

O2 therapy:
O2 saturation target is 88 – 92% until ABGs available
If there is no CO2 retention, make the target 94 – 98%
Bronchodilator (SABA+SAMA) with a back-to-back nebulizer
Give prednisolone 40 mg daily for 5 days
Oral antibiotics (Amoxicillin, Tetracycline, or clarithromycin) only if there are purulent sputum

Page - 1880
Internal Medicine - Pulmonology

Question 14/144

Question #14

A 65-year-old male patient was recently diagnosed with COPD. His pulmonary function test shows
an FEV1 of 33% of predicted and an FEV1/FVC ratio of 60%. e pulmonary function test of this
patient will categorize him as:

a. Stage 1
b. Stage 2
c. Stage 3
d. Stage 4
e. No enough information

‫اﻹﺟﺎﺑﺔ ﻋﲆ اﻟﺼﻔﺤﺔ اﻟﺘﺎﻟﻴﺔ‬

Page - 1881
Internal Medicine - Pulmonology - Chronic Obstructive Pulmonary disease (COPD)

Question 14/144

Question #14

A 65-year-old male patient was recently diagnosed with COPD. His pulmonary function test shows
an FEV1 of 33% of predicted and an FEV1/FVC ratio of 60%. e pulmonary function test of this
patient will categorize him as:

a. Stage 1
b. Stage 2
c. Stage 3 √
d. Stage 4
e. No enough information

Description

All COPD stages have an FEV1/FVC ratio of < 0.7, but the staging depends on FEV1.

FEV1 of 33% predicted is typical for stage 3 COPD.

Stages of COPD:

Stage 1: FEV1 is ≥ 80%


Stage 2: FEV1 is 50 – 79%
Stage 3: FEV1 is 30 – 49%
Stage 4: FEV1 is < 30%

Page - 1882
Internal Medicine - Pulmonology

Question 15/144

Question #15

A 72-year-old male patient has 2 years history of progressive wheezes and shortness of breath and
was diagnosed with COPD secondary to heavy smoking. All the following are correct about this
condition except:

a. Smoking-related Emphysema usually aects the upper zones


b. Inhaled corticosteroid is not used as monotherapy
c. e most eective to stop COPD progression is smoking cessation
d. LTOT is used for symptomatic relief without any mortality benefit
e. DLco is reduced in Emphysema but not aected in chronic bronchitis

‫اﻹﺟﺎﺑﺔ ﻋﲆ اﻟﺼﻔﺤﺔ اﻟﺘﺎﻟﻴﺔ‬

Page - 1883
Internal Medicine - Pulmonology - Chronic Obstructive Pulmonary disease (COPD)

Question 15/144

Question #15

A 72-year-old male patient has 2 years history of progressive wheezes and shortness of breath and
was diagnosed with COPD secondary to heavy smoking. All the following are correct about this
condition except:

a. Smoking-related Emphysema usually aects the upper zones


b. Inhaled corticosteroid is not used as monotherapy
c. e most eective to stop COPD progression is smoking cessation
d. LTOT is used for symptomatic relief without any mortality benefit √
e. DLco is reduced in Emphysema but not aected in chronic bronchitis

Description

Page - 1884
Internal Medicine - Pulmonology

Question 16/144

Question #16

A 66-year-old heavy-smoker male is diagnosed with COPD, and you decided to start Inhaled
corticosteroids as a part of the treatment. In a COPD patient, ICS is shown to do one of the following:

a. Increases the rate of exacerbations


b. Increases the risk of pneumonia
c. Reduce the mortality rate
d. Prevents the progression of COPD
e. Increase the rate of bone fractures

‫اﻹﺟﺎﺑﺔ ﻋﲆ اﻟﺼﻔﺤﺔ اﻟﺘﺎﻟﻴﺔ‬

Page - 1885
Internal Medicine - Pulmonology - Chronic Obstructive Pulmonary disease (COPD)

Question 16/144

Question #16

A 66-year-old heavy-smoker male is diagnosed with COPD, and you decided to start Inhaled
corticosteroids as a part of the treatment. In a COPD patient, ICS is shown to do one of the following:

a. Increases the rate of exacerbations


b. Increases the risk of pneumonia √
c. Reduce the mortality rate
d. Prevents the progression of COPD
e. Increase the rate of bone fractures

Description

ICS increases the risk of candidiasis of the oropharynx and pneumonia, but it does not increase the
risk of bone fractures like systemic steroids.

ICS reduces the risk of exacerbations but does not prevent COPD progression nor reduce mortality.

Page - 1886
Internal Medicine - Pulmonology

Question 17/144

Question #17

A 66-year-old male patient presents with infective COPD exacerbation. He is treated at the
emergency room with 28% oxygen by venturi mask, nebulized SABA and SAMA, and intravenous
hydrocortisone. However, the patient is still symptomatic, and his Arterial blood gases are PH 7.31,
PaCO2 56 mmHg, and PaO2 78 mmHg. What is the most appropriate next step in the management
of this patient?

a. Invasive ventilation
b. Long-acting beta2 agonist
c. Start antibiotics
d. Non-invasive ventilation
e. Continue same treatment

‫اﻹﺟﺎﺑﺔ ﻋﲆ اﻟﺼﻔﺤﺔ اﻟﺘﺎﻟﻴﺔ‬

Page - 1887
Internal Medicine - Pulmonology - Chronic Obstructive Pulmonary disease (COPD)

Question 17/144

Question #17

A 66-year-old male patient presents with infective COPD exacerbation. He is treated at the
emergency room with 28% oxygen by venturi mask, nebulized SABA and SAMA, and intravenous
hydrocortisone. However, the patient is still symptomatic, and his Arterial blood gases are PH 7.31,
PaCO2 56 mmHg, and PaO2 78 mmHg. What is the most appropriate next step in the management
of this patient?

a. Invasive ventilation
b. Long-acting beta2 agonist
c. Start antibiotics
d. Non-invasive ventilation √
e. Continue same treatment

Description

A PH of 7.25 – 7.35 in COPD exacerbation indicates the user of non-invasive ventilation.

Treatment of COPD exacerbation:

O2 therapy:
e O2 saturation target is 88 – 92% until ABGs available
If there is no CO2 retention, make the target 94 – 98%
Bronchodilator (SABA+SAMA) with a back-to-back nebulizer
Give prednisolone 40 mg daily for 5 days
Oral antibiotics (Amoxicillin, Tetracycline, or clarithromycin) only if there are purulent sputum

Indications for non-invasive ventilation:

COPD with respiratory acidosis pH 7.25-7.35


Type II respiratory failure secondary to:
Chest wall deformity
Neuromuscular disease
Obstructive sleep apnea
Cardiogenic pulmonary edema

Page - 1888
Weaning from tracheal intubation

Page - 1889
Internal Medicine - Pulmonology

Question 18/144

Question #18

A 29-year-old male patient presented with recurrent shortness of breath for 3 months. However, the
pulmonary function test shows a fixed obstructive pattern. Which of the following is the most
appropriate test?

a. Alpha 1 antitrypsin level


b. Chest x-ray
c. Bronchoscopy and biopsy
d. High-resolution chest CT scan
e. PET scan

‫اﻹﺟﺎﺑﺔ ﻋﲆ اﻟﺼﻔﺤﺔ اﻟﺘﺎﻟﻴﺔ‬

Page - 1890
Internal Medicine - Pulmonology - Chronic Obstructive Pulmonary disease (COPD)

Question 18/144

Question #18

A 29-year-old male patient presented with recurrent shortness of breath for 3 months. However, the
pulmonary function test shows a fixed obstructive pattern. Which of the following is the most
appropriate test?

a. Alpha 1 antitrypsin level √


b. Chest x-ray
c. Bronchoscopy and biopsy
d. High-resolution chest CT scan
e. PET scan

Description

Any young patient with COPD should be tested for alpha 1 antitrypsin level. A1AT deficiency is the
most common cause of COPD in young patients.

Alpha-1-antitrypsin deficiency:

Deficiency of alpha-1-antitrypsin due to a genetic problem


Autosomal recessive condition (located on chromosome 14)
Normally, PiMM
Homozygous PiSS (50% normal A1AT levels)
Homozygous PiZZ (10% normal A1AT levels)
Alpha-1-antitrypsin is a protease inhibitor protein synthesized by the liver
A1AT deficiency presents with a combination of COPD and liver cirrhosis
COPD (usually Emphysema) in young (< 40 years) and non-smoker patients is considered A1AT
deficiency until proven otherwise

Page - 1891
Internal Medicine - Pulmonology

Question 19/144

Question #19

A 60-year-old male was diagnosed with COPD 8 years ago. Which of the following measures would
most improve the survival of the patient?

a. LAMA and SAMA


b. SAMA and SABA
c. Corticosteroid treatment
d. Oxygen and smoking cessation
e. Oxygen and Steroids

‫اﻹﺟﺎﺑﺔ ﻋﲆ اﻟﺼﻔﺤﺔ اﻟﺘﺎﻟﻴﺔ‬

Page - 1892
Internal Medicine - Pulmonology - Chronic Obstructive Pulmonary disease (COPD)

Question 19/144

Question #19

A 60-year-old male was diagnosed with COPD 8 years ago. Which of the following measures would
most improve the survival of the patient?

a. LAMA and SAMA


b. SAMA and SABA
c. Corticosteroid treatment
d. Oxygen and smoking cessation √
e. Oxygen and Steroids

Description

Long-term oxygen therapy and smoking cessation are indicated to reduce the mortality rate in
patients with COPD

LAMA, LABA, SAMA, SABA, and Steroids are used to treat or prevent exacerbations but not improve
the mortality rate.

Treatment of COPD:

Measures to reduce mortality:


Smoking cessation (the most important)
H. Influenza and pneumococcal vaccine
Long-Term Oxygen erapy (LTOT).
Measure to reduce symptoms:
SABA (More rapid onset)
SAMA (Most eective in COPD but slower than SABA)
LABA (More sustained eect)
ICS + LABA (ICS is not used as monotherapy in COPD)
Surgical measures (both morality and symptomatic benefits):
Volume reduction
Lung transplant

Page - 1893
Internal Medicine - Pulmonology

Question 20/144

Question #20

Which of the following is most essential to prevent the worsening of COPD in a 55-year-old male
patient?

b. Inhaled salmeterol
c. Inhaled beclomethasone
d. Prophylactic antibiotics
e. Stop smoking
a. eophylline

‫اﻹﺟﺎﺑﺔ ﻋﲆ اﻟﺼﻔﺤﺔ اﻟﺘﺎﻟﻴﺔ‬

Page - 1894
Internal Medicine - Pulmonology - Chronic Obstructive Pulmonary disease (COPD)

Question 20/144

Question #20

Which of the following is most essential to prevent the worsening of COPD in a 55-year-old male
patient?

b. Inhaled salmeterol
c. Inhaled beclomethasone
d. Prophylactic antibiotics
e. Stop smoking √
a. eophylline

Description

Smoking cessation is the most crucial step to attenuate the progression of COPD. It may improve
lung function as well.

Treatment of COPD:

Measures to reduce mortality:


Smoking cessation (the most important)
H. Influenza and pneumococcal vaccine
Long-Term Oxygen erapy (LTOT).
Measure to reduce symptoms:
SABA (More rapid onset)
SAMA (Most eective in COPD but slower than SABA)
LABA (More sustained eect)
ICS + LABA (ICS is not used as monotherapy in COPD)
Surgical measures (both morality and symptomatic benefits):
Volume reduction
Lung transplant

Page - 1895
Internal Medicine - Pulmonology

Question 21/144

Question #21

A heavy smoker 55-year-old male patient has chronic productive cough and wheezes. In addition, he
has hyperinflation on his chest x-ray and polycythemia. What is the most likely diagnosis?

a. Interstitial lung disease


b. Granulomatosis with polyangiitis
c. Lung cancer
d. COPD
e. Tuberculosis

‫اﻹﺟﺎﺑﺔ ﻋﲆ اﻟﺼﻔﺤﺔ اﻟﺘﺎﻟﻴﺔ‬

Page - 1896
Internal Medicine - Pulmonology - Chronic Obstructive Pulmonary disease (COPD)

Question 21/144

Question #21

A heavy smoker 55-year-old male patient has chronic productive cough and wheezes. In addition, he
has hyperinflation on his chest x-ray and polycythemia. What is the most likely diagnosis?

a. Interstitial lung disease


b. Granulomatosis with polyangiitis
c. Lung cancer
d. COPD √
e. Tuberculosis

Description

History of heavy smoking, hyperinflated chest x-ray, and polycythemia suggest COPD.

In COPD patients, chronic hypoxia will increase the level of erythropoietin hormone leading to
polycythemia.

A fixed obstructive pattern in PFT is expected in this patient, and smoking cessation is the most
eective step in reducing mortality.

Page - 1897
Internal Medicine - Pulmonology

Question 22/144

Question #22

A 66-year-old male with a history of heavy smoking and COPD presents with a 1-week history of
increasing purulent sputum and shortness of breath. Which of the following is false regarding the
management of this patient?

a. Antibiotic reduces the risk of treatment failure or death


b. A short course of systemic corticosteroids shortens hospital stays and decreases treatment
failures
c. Oral and IV steroids are equivalent in the treatment of this patient
d. ICS should be used in the treatment of acute COPD exacerbation
e. BiPAP is reserved for severe cases only

‫اﻹﺟﺎﺑﺔ ﻋﲆ اﻟﺼﻔﺤﺔ اﻟﺘﺎﻟﻴﺔ‬

Page - 1898
Internal Medicine - Pulmonology - Chronic Obstructive Pulmonary disease (COPD)

Question 22/144

Question #22

A 66-year-old male with a history of heavy smoking and COPD presents with a 1-week history of
increasing purulent sputum and shortness of breath. Which of the following is false regarding the
management of this patient?

a. Antibiotic reduces the risk of treatment failure or death


b. A short course of systemic corticosteroids shortens hospital stays and decreases treatment
failures
c. Oral and IV steroids are equivalent in the treatment of this patient
d. ICS should be used in the treatment of acute COPD exacerbation √
e. BiPAP is reserved for severe cases only

Description

Inhaled corticosteroids are not indicated in acute COPD exacerbation.

Treatment of COPD exacerbation:

O2 therapy:
O2 saturation target is 88 – 92% until ABGs available
If there is no CO2 retention, make the target 94 – 98%
Bronchodilator (SABA+SAMA) with a back-to-back nebulizer
Give prednisolone 40 mg daily for 5 days
Oral antibiotics (Amoxicillin, Tetracycline, or clarithromycin) only if there are purulent sputum

Page - 1899
Internal Medicine - Pulmonology

Question 23/144

Question #23

A 49-year-old male patient with dyspnea and productive cough for the past 2 years presents for
evaluation. His DLco is normal, and his FEV1/FVC is 2.4/3.7. He was given inhaled salbutamol then
spirometry was repeated. e new results show FEV1/FVC of 2.5/3.8. What is the most likely
diagnosis?

a. Chronic bronchitis
b. Emphysema
c. Asthma
d. Interstitial lung disease
e. Kyphosis

‫اﻹﺟﺎﺑﺔ ﻋﲆ اﻟﺼﻔﺤﺔ اﻟﺘﺎﻟﻴﺔ‬

Page - 1900
Internal Medicine - Pulmonology - Chronic Obstructive Pulmonary disease (COPD)

Question 23/144

Question #23

A 49-year-old male patient with dyspnea and productive cough for the past 2 years presents for
evaluation. His DLco is normal, and his FEV1/FVC is 2.4/3.7. He was given inhaled salbutamol then
spirometry was repeated. e new results show FEV1/FVC of 2.5/3.8. What is the most likely
diagnosis?

a. Chronic bronchitis √
b. Emphysema
c. Asthma
d. Interstitial lung disease
e. Kyphosis

Description

FEV1/FVC ratio of less than 0.7 indicates obstructive lung disease.

e bronchodilator challenge test shows less than 12% improvement in FEV1, making COPD the most
likely diagnosis.

Chronic bronchitis has a normal DLco, but Emphysema will manifest with a low DLco level.

In summary, the spirometry that shows a fixed (irreversible) obstructive pattern, and the normal
DLco suggest chronic bronchitis.

Asthma will manifest as a reversible obstructive pattern, while Emphysema will show low DLco.

Interstitial lung disease and Kyphosis will show a restrictive pattern on spirometry.

Page - 1901
Internal Medicine - Pulmonology

Question 24/144

Question #24

A patient with COPD will exhibit hyperinflation of the lungs. All the following are features of a
hyperinflated lung on chest x-ray except:

a. Horizontal wide-spaced ribs


b. Cardiomegaly
c. Hyperlucent lung field
d. Depressed diaphragm
e. Air trapping

‫اﻹﺟﺎﺑﺔ ﻋﲆ اﻟﺼﻔﺤﺔ اﻟﺘﺎﻟﻴﺔ‬

Page - 1902
Internal Medicine - Pulmonology - Chronic Obstructive Pulmonary disease (COPD)

Question 24/144

Question #24

A patient with COPD will exhibit hyperinflation of the lungs. All the following are features of a
hyperinflated lung on chest x-ray except:

a. Horizontal wide-spaced ribs


b. Cardiomegaly √
c. Hyperlucent lung field
d. Depressed diaphragm
e. Air trapping

Description

A slim vertical heart is a feature of hyperinflation on chest x-ray, not cardiomegaly.

Features of hyperinflated chest on chest x-ray:

Horizontal ribs
Wide spaces between ribs
Low set, flat diaphragm
Vertical heart
Hyperlucent lung field

Page - 1903
Internal Medicine - Pulmonology

Question 25/144

Question #25

A 60-year-old male patient with COPD presents for a Pulmonary function test. His spirometry shows
an FEV1 of 45% of the predicted. Which other findings can also be observed in a patient with COPD
on spirometry?

a. Increased tidal volume


b. Decreased total lung capacity
c. Increased residual volume
d. Reduced functional residual capacity
e. Elevated FEV1/FVC ratio

‫اﻹﺟﺎﺑﺔ ﻋﲆ اﻟﺼﻔﺤﺔ اﻟﺘﺎﻟﻴﺔ‬

Page - 1904
Internal Medicine - Pulmonology - Chronic Obstructive Pulmonary disease (COPD)

Question 25/144

Question #25

A 60-year-old male patient with COPD presents for a Pulmonary function test. His spirometry shows
an FEV1 of 45% of the predicted. Which other findings can also be observed in a patient with COPD
on spirometry?

a. Increased tidal volume


b. Decreased total lung capacity
c. Increased residual volume √
d. Reduced functional residual capacity
e. Elevated FEV1/FVC ratio

Description

e following findings are expected to be found in COPD patients.

FEV1 less than 80% of predicted


FEV1/FVC ratio of less than 70%
Increased Total lung capacity (TLC), Functional residual capacity (FRC), Residual volume (RV),
and Vital capacity (VC).
DLco is expected to be low in Emphysema but normal in chronic bronchitis patients

Note: in COPD patients, spirometry should only be tested when the patient’s condition is stable,
ideally six weeks aer the last exacerbation.

Page - 1905
Internal Medicine - Pulmonology

Question 26/144

Question #26

A known case of HTN, migraine headache, and Hodgkin’s lymphoma, a 54-year-old female presents
with fatigue, dry cough, and shortness of breath for a 6-month duration. Her chest examination is
consistent with fine basal crepitations bilaterally. Her PFT is done and shows FEV1 55%, FEV1/FVC
ratio 90% of predicted. You suspect a drug-induced condition. Which of the following is the most
likely cause?

a. Captopril 50mg daily


b. Propranolol 40 mg bid
c. Bleomycin SC 1 unit daily
d. Indapamide 1.5 mg daily
e. Amlodipine 5mg daily

‫اﻹﺟﺎﺑﺔ ﻋﲆ اﻟﺼﻔﺤﺔ اﻟﺘﺎﻟﻴﺔ‬

Page - 1906
Internal Medicine - Pulmonology - Drug-induced lung fibrosis

Question 26/144

Question #26

A known case of HTN, migraine headache, and Hodgkin’s lymphoma, a 54-year-old female presents
with fatigue, dry cough, and shortness of breath for a 6-month duration. Her chest examination is
consistent with fine basal crepitations bilaterally. Her PFT is done and shows FEV1 55%, FEV1/FVC
ratio 90% of predicted. You suspect a drug-induced condition. Which of the following is the most
likely cause?

a. Captopril 50mg daily


b. Propranolol 40 mg bid
c. Bleomycin SC 1 unit daily √
d. Indapamide 1.5 mg daily
e. Amlodipine 5mg daily

Description

Bleomycin is a chemotherapy agent that is a recognized cause of pulmonary fibrosis

Drugs that possibly cause lung fibrosis:

- Bleomycin

- Cyclophosphamide

- Amiodarone

- Procainamide

- Illicit drugs (heroin, methadone)

- Antibiotics: nitrofurantoin, penicillin, sulfonamide

- Anti-Inflammatory Agents: methotrexate, penicillamine

- Gold salts

- Rituximab, anti-TNF agents (Infliximab, Etanercept, Adalimumab)

Page - 1907
Internal Medicine - Pulmonology

Question 27/144

Question #27

All the following are considered eosinophilic lung diseases except:

a. Loer’s syndrome
b. Aspergillosis
c. Churg-Strauss syndrome
d. Cocaine-induced pneumonitis
e. Sarcoidosis

‫اﻹﺟﺎﺑﺔ ﻋﲆ اﻟﺼﻔﺤﺔ اﻟﺘﺎﻟﻴﺔ‬

Page - 1908
Internal Medicine - Pulmonology - Eosinophilic lung disease

Question 27/144

Question #27

All the following are considered eosinophilic lung diseases except:

a. Loer’s syndrome
b. Aspergillosis
c. Churg-Strauss syndrome
d. Cocaine-induced pneumonitis
e. Sarcoidosis √

Description

Sarcoidosis and extrinsic allergic alveolitis are not eosinophilic lung diseases.

Eosinophilic lung diseases:

Diseases that are associated with increased eosinophils in bronchoalveolar lavage and lung
biopsy

Causes:

Loer’s syndrome: pulmonary eosinophilia in response to parasitic infection (e.g., Ascaris,


Strongyloides, Dirofilaria)
Allergic Bronchopulmonary Aspergillosis (ABPA)
Eosinophilic pneumonia (can be viral or bacterial)
Churg-Strauss syndrome (vasculitis)
Drug-induced (cocaine, Phenytoin, some antibiotics, etc.)

Page - 1909
Internal Medicine - Pulmonology

Question 28/144

Question #28

A 45-year-old farmer presents with a dry cough, chest tightness, and shortness of breath for a few
hours; his temperature is 39 °C, respiratory rate is 28 per minute, and blood pressure is normal.
What is the most appropriate diagnosis?

a. Extrinsic allergic alveolitis


b. Pulmonary embolism
c. Granulomatosis with polyangiitis
d. cardiogenic pulmonary edema
e. progressive massive fibrosis

‫اﻹﺟﺎﺑﺔ ﻋﲆ اﻟﺼﻔﺤﺔ اﻟﺘﺎﻟﻴﺔ‬

Page - 1910
Internal Medicine - Pulmonology - Extrinsic Allergic Alveolitis (EAA)

Question 28/144

Question #28

A 45-year-old farmer presents with a dry cough, chest tightness, and shortness of breath for a few
hours; his temperature is 39 °C, respiratory rate is 28 per minute, and blood pressure is normal.
What is the most appropriate diagnosis?

a. Extrinsic allergic alveolitis √


b. Pulmonary embolism
c. Granulomatosis with polyangiitis
d. cardiogenic pulmonary edema
e. progressive massive fibrosis

Description

e occupation of the patient, “Farmer,” and the other signs and symptoms of the patient indicate
acute (type 3) EAA as the most likely diagnosis.

Extrensic allergic alveolitis (EAA)

- Also known as (hypersensitivity pneumonitis)

- Inhalation of organic substance → destruction of alveoli → fibrosis

- Involves type 3 or type 4 hypersensitivity reaction (Type 3 more prominent)

Page - 1911
Internal Medicine - Pulmonology

Question 29/144

Question #29

A 46-year-old patient is suspected of having extrinsic allergic alveolitis. is condition is most likely
acquired from inhalation of which one of the following?

a. Silica dust
b. Organophosphate compounds
c. Chlorine gas
d. Fungal spores
e. Smoking

‫اﻹﺟﺎﺑﺔ ﻋﲆ اﻟﺼﻔﺤﺔ اﻟﺘﺎﻟﻴﺔ‬

Page - 1912
Internal Medicine - Pulmonology - Extrinsic Allergic Alveolitis (EAA)

Question 29/144

Question #29

A 46-year-old patient is suspected of having extrinsic allergic alveolitis. is condition is most likely
acquired from inhalation of which one of the following?

a. Silica dust
b. Organophosphate compounds
c. Chlorine gas
d. Fungal spores √
e. Smoking

Description

EAA is caused by an allergic reaction to inhaled organic substances

Extrensic allergic alveolitis (EAA)

Also known as (hypersensitivity pneumonitis)


Inhalation of organic substance → destruction of alveoli → fibrosis

Involves type 3 or type 4 hypersensitivity reaction (Type 3 more prominent)

Page - 1913
Internal Medicine - Pulmonology

Question 30/144

Question #30

A 56-year-old male presents with Exertional dyspnea, Non-productive cough, Late expiratory
crackles, and clubbing. What is the best investigation to make the diagnosis of this condition?

a. Eosinophilia in the complete blood picture


b. Ground-glass appearance on high-resolution CT scan
c. Increased vascular markings on chest x-ray
d. An obstructive pattern on spirometry
e. Respiratory alkalosis without hypoxia on ABGs

‫اﻹﺟﺎﺑﺔ ﻋﲆ اﻟﺼﻔﺤﺔ اﻟﺘﺎﻟﻴﺔ‬

Page - 1914
Internal Medicine - Pulmonology - Idiopathic Pulmonary Fibrosis (IPF)

Question 30/144

Question #30

A 56-year-old male presents with Exertional dyspnea, Non-productive cough, Late expiratory
crackles, and clubbing. What is the best investigation to make the diagnosis of this condition?

a. Eosinophilia in the complete blood picture


b. Ground-glass appearance on high-resolution CT scan √
c. Increased vascular markings on chest x-ray
d. An obstructive pattern on spirometry
e. Respiratory alkalosis without hypoxia on ABGs

Description

A High-resolution CT scan is the best for the diagnosis of IPF. It will show a ground-glass appearance

A chest x-ray will show a reticulonodular pattern (early stage), or honeycombing (late stage)

Spirometry will show a restrictive pattern

Idiopathic pulmonary fibrosis:

It is an idiopathic progressive disease of an Irreversible process


Clinical features include exertional dyspnea, non-productive cough, late expiratory crackles
(Velcro-like crepitations), and clubbing.
It is diagnosed by a workup of chest x-ray, PFT, and high-resolution CT scan
Biopsy is the most accurate test, but it is usually not indicated

Page - 1915
Internal Medicine - Pulmonology

Question 31/144

Question #31

Digital clubbing and bibasilar inspiratory (Velcro-like) crackles in a 54-year-old previously healthy
male patient indicate one of the following?

a. Asthma
b. GERD
c. Chronic bronchitis
d. Emphysema
e. Idiopathic pulmonary fibrosis (IPF)

‫اﻹﺟﺎﺑﺔ ﻋﲆ اﻟﺼﻔﺤﺔ اﻟﺘﺎﻟﻴﺔ‬

Page - 1916
Internal Medicine - Pulmonology - Idiopathic Pulmonary Fibrosis (IPF)

Question 31/144

Question #31

Digital clubbing and bibasilar inspiratory (Velcro-like) crackles in a 54-year-old previously healthy
male patient indicate one of the following?

a. Asthma
b. GERD
c. Chronic bronchitis
d. Emphysema
e. Idiopathic pulmonary fibrosis (IPF) √

Description

Description:

Exertional dyspnea, Non-productive cough, late expiratory crackles (Velcro-like crepitations), and
clubbing are indicative of IPF

Idiopathic pulmonary fibrosis:

It is an idiopathic progressive disease of an Irreversible process


Clinical features include exertional dyspnea, non-productive cough, late expiratory crackles
(Velcro-like crepitations), and clubbing.
It is diagnosed by a workup of chest x-ray, PFT, and high-resolution CT scan
Biopsy is the most accurate test, but it is usually not indicated

Page - 1917
Internal Medicine - Pulmonology

Question 32/144

Question #32

A 65-year-old male patient who is bedridden due to a Stroke presents with coughs productive of
purulent sputum, hemoptysis, and fever. Chest x-ray shows a cavity lesion with an air-fluid level. A
sputum culture is positive for anaerobic bacteria. What is the treatment of choice for this patient’s
condition?

a. Penicillin
b. Clarithromycin
c. Azithromycin
d. Clindamycin
e. Amoxicillin

‫اﻹﺟﺎﺑﺔ ﻋﲆ اﻟﺼﻔﺤﺔ اﻟﺘﺎﻟﻴﺔ‬

Page - 1918
Internal Medicine - Pulmonology - Lung Abscess

Question 32/144

Question #32

A 65-year-old male patient who is bedridden due to a Stroke presents with coughs productive of
purulent sputum, hemoptysis, and fever. Chest x-ray shows a cavity lesion with an air-fluid level. A
sputum culture is positive for anaerobic bacteria. What is the treatment of choice for this patient’s
condition?

a. Penicillin
b. Clarithromycin
c. Azithromycin
d. Clindamycin √
e. Amoxicillin

Description

is is a case of lung abscess; the leading risk factor here is aspiration which is common in stroke
patients

anaerobic bacteria cause most lung abscesses; clindamycin will be the most eective in most cases.

Page - 1919
Internal Medicine - Pulmonology

Question 33/144

Question #33

For the last 2 weeks, a 34-year-old male patient with epilepsy has presented to you with fever,
anorexia, and persistent cough with smelly sputum. His temperature is 38.9 °C, and his chest
examination reveals rales in the right lower zone. In addition, he has poor dental hygiene. Of the
following, what is the most likely to be expected on a chest x-ray?

a. Bilateral hilar lymphadenopathy


b. Miliary Tuberculosis
c. Pleural eusion
d. Hyperinflated chest
e. Lung abscess

‫اﻹﺟﺎﺑﺔ ﻋﲆ اﻟﺼﻔﺤﺔ اﻟﺘﺎﻟﻴﺔ‬

Page - 1920
Internal Medicine - Pulmonology - Lung Abscess

Question 33/144

Question #33

For the last 2 weeks, a 34-year-old male patient with epilepsy has presented to you with fever,
anorexia, and persistent cough with smelly sputum. His temperature is 38.9 °C, and his chest
examination reveals rales in the right lower zone. In addition, he has poor dental hygiene. Of the
following, what is the most likely to be expected on a chest x-ray?

a. Bilateral hilar lymphadenopathy


b. Miliary Tuberculosis
c. Pleural eusion
d. Hyperinflated chest
e. Lung abscess √

Description

Lung abscess is found in patients at risk of aspiration. Physical examination usually reveals poor
dental hygiene, a foul odor of the breath and sputum, rales, and consolidation.

Page - 1921
Internal Medicine - Pulmonology

Question 34/144

Question #34

A 43-year-old male underwent surgery 3 weeks ago. He presents with cough, sputum, fever, and
right lower zone cavity lesion on chest x-ray. Examination reveals rales and consolidation. Which is
the most likely eective treatment for this patient’s condition?

a. Ceriaxone
b. Clindamycin
c. Metronidazole
d. TMP/SMX
e. Vancomycin

‫اﻹﺟﺎﺑﺔ ﻋﲆ اﻟﺼﻔﺤﺔ اﻟﺘﺎﻟﻴﺔ‬

Page - 1922
Internal Medicine - Pulmonology - Lung Abscess

Question 34/144

Question #34

A 43-year-old male underwent surgery 3 weeks ago. He presents with cough, sputum, fever, and
right lower zone cavity lesion on chest x-ray. Examination reveals rales and consolidation. Which is
the most likely eective treatment for this patient’s condition?

a. Ceriaxone
b. Clindamycin √
c. Metronidazole
d. TMP/SMX
e. Vancomycin

Description

is is a lung abscess; the leading risk factor here is an aspiration common in postoperative patients.

Anaerobic bacteria cause most lung abscesses; clindamycin will be the most eective in most cases.

Page - 1923
Internal Medicine - Pulmonology

Question 35/144

Question #35

A 48-year-old male patient presents with fever, cough, and hemoptysis for the last week. On
examination, there is a decreased breathing sound, dullness to percussion, and reduced TVF on the
right side. e trachea is shied toward the le side. What is the most likely diagnosis of this
patient?

a. Right-side Pneumothorax
b. Right-side Pleural eusion
c. Le-side Pneumothorax
d. Le-side Pleural eusion
e. Right-side lung collapse

‫اﻹﺟﺎﺑﺔ ﻋﲆ اﻟﺼﻔﺤﺔ اﻟﺘﺎﻟﻴﺔ‬

Page - 1924
Internal Medicine - Pulmonology - Pleural Eusion 1

Question 35/144

Question #35

A 48-year-old male patient presents with fever, cough, and hemoptysis for the last week. On
examination, there is a decreased breathing sound, dullness to percussion, and reduced TVF on the
right side. e trachea is shied toward the le side. What is the most likely diagnosis of this
patient?

a. Right-side Pneumothorax
b. Right-side Pleural eusion √
c. Le-side Pneumothorax
d. Le-side Pleural eusion
e. Right-side lung collapse

Description

ere will be an ipsilateral stony dullness in pleural eusion, decreased breathing sound, and
decreased Tactile vocal fremitus (TVF). e trachea will be shied to the opposite side as well.

In the case of pneumothorax, there will be an ipsilateral Hyperresonant chest, decreased breath
sound, and increased TVF. e trachea will be shied to the opposite side as well.

In case of lung collapse, the trachea will be shied to the same side.

Page - 1925
Internal Medicine - Pulmonology

Question 36/144

Question #36

A 37-year-old male patient was diagnosed with pneumonia which is now complicated by empyema.
What is the essential step in the management of this patient?

a. Oxygen therapy
b. Intubation and ventilation
c. Chest tube insertion
d. Pleural biopsy
e. Antibiotics use

‫اﻹﺟﺎﺑﺔ ﻋﲆ اﻟﺼﻔﺤﺔ اﻟﺘﺎﻟﻴﺔ‬

Page - 1926
Internal Medicine - Pulmonology - Pleural Eusion 1

Question 36/144

Question #36

A 37-year-old male patient was diagnosed with pneumonia which is now complicated by empyema.
What is the essential step in the management of this patient?

a. Oxygen therapy
b. Intubation and ventilation
c. Chest tube insertion √
d. Pleural biopsy
e. Antibiotics use

Description

Chest tube insertion is the most important step in the management of empyema.

Antibiotics, intubation, oxygen therapy, and biopsies may be needed to manage, but the patient will
not improve without chest tube insertion.

Page - 1927
Internal Medicine - Pulmonology

Question 37/144

Question #37

A 48-year-old male patient presents with fever, cough, and hemoptysis for the last week. On
examination, there is a decreased breathing sound, dullness to percussion, and reduced TVF on the
right side. e trachea is shied toward the le side. What is the best initial step to take in
managing this patient?

a. Chest x-ray
b. Pleural biopsy
c. oracentesis
d. Chest CT scan
e. Start antibiotics

‫اﻹﺟﺎﺑﺔ ﻋﲆ اﻟﺼﻔﺤﺔ اﻟﺘﺎﻟﻴﺔ‬

Page - 1928
Internal Medicine - Pulmonology - Pleural Eusion 1

Question 37/144

Question #37

A 48-year-old male patient presents with fever, cough, and hemoptysis for the last week. On
examination, there is a decreased breathing sound, dullness to percussion, and reduced TVF on the
right side. e trachea is shied toward the le side. What is the best initial step to take in
managing this patient?

a. Chest x-ray √
b. Pleural biopsy
c. oracentesis
d. Chest CT scan
e. Start antibiotics

Description

A chest x-ray is the best initial test for any patient suspected of pleural eusion.

Chest Ct scan, thoracentesis, pleural biopsy, and antibiotics may be needed later on according to the
underlying condition.

Pleural eusion:

Pleural eusion is an abnormal collection of fluids within the pleural space


Pleural fluids can be Transudates or exudates

Chest X-ray (best initial test) but CT scan (more informative than chest x-ray)
oracentesis (most accurate test): dierentiate exudates from Transudates

Treatment of pleural eusion:

Small eusion needs no treatment


Cardiogenic pleural eusion is highly responsive to diuretics
oracentesis can be therapeutic and diagnostic
For refractory and recurrent pleural eusion, pleurodesis (intrapleural bleomycin or talcum
powder)
For empyema, a chest tube, antibiotics, and surgical drainage may be needed

Page - 1929
Internal Medicine - Pulmonology

Question 38/144

Question #38

A 62-year-old male with a known case of DM and HTN presents with right-side chest pain and
shortness of breath. His chest x-ray is shown below. oracentesis shows a protein concentration of
18 g/L. What is the most likely diagnosis of his condition?

a. Rheumatoid arthritis
b. Hypothyroidism
c. Nephrotic syndrome
d. Congestive heart failure
e. Liver cirrhosis

Page - 1930
‫اﻹﺟﺎﺑﺔ ﻋﲆ اﻟﺼﻔﺤﺔ اﻟﺘﺎﻟﻴﺔ‬

‫‪Page - 1931‬‬
Internal Medicine - Pulmonology - Pleural Eusion 1

Question 38/144

Question #38

A 62-year-old male with a known case of DM and HTN presents with right-side chest pain and
shortness of breath. His chest x-ray is shown below. oracentesis shows a protein concentration of
18 g/L. What is the most likely diagnosis of his condition?

a. Rheumatoid arthritis
b. Hypothyroidism
c. Nephrotic syndrome
d. Congestive heart failure √
e. Liver cirrhosis

Page - 1932
Description

is patient is complaining of right-side pleural eusion. e history and the results of paracentesis
suggest heart failure as the most likely cause.

Rheumatoid arthritis is a possible cause of exudative eusion (not transudates)

e history of HTN, DM, and cardiomegaly on chest x-ray makes CHF more likely than the other
conditions mentioned.

Pleural eusion:

Pleural eusion is an abnormal collection of fluids within the pleural space


Pleural fluids can be Transudates or exudates

Chest X-ray (best initial test) but CT scan (more informative than chest x-ray)
oracentesis (most accurate test): dierentiate exudates from Transudates

Treatment of pleural eusion:

Small eusion needs no treatment


Cardiogenic pleural eusion is highly responsive to diuretics
oracentesis can be therapeutic and diagnostic
For refractory and recurrent pleural eusion, pleurodesis (intrapleural bleomycin or talcum
powder)
For empyema, a chest tube, antibiotics, and surgical drainage may be needed

Page - 1933
Internal Medicine - Pulmonology

Question 39/144

Question #39

A 48-year-old male patient presents with fever, cough, and hemoptysis for the last week. On
examination, there is a decreased breathing sound, dullness to percussion, and reduced TVF on the
right side. e trachea is shied toward the le side. What is the most accurate test to diagnose the
patient’s underlying condition?

a. Chest x-ray
b. Chest CT scan
c. oracentesis
d. Bronchoscopy
e. CT pulmonary angiography

‫اﻹﺟﺎﺑﺔ ﻋﲆ اﻟﺼﻔﺤﺔ اﻟﺘﺎﻟﻴﺔ‬

Page - 1934
Internal Medicine - Pulmonology - Pleural Eusion 1

Question 39/144

Question #39

A 48-year-old male patient presents with fever, cough, and hemoptysis for the last week. On
examination, there is a decreased breathing sound, dullness to percussion, and reduced TVF on the
right side. e trachea is shied toward the le side. What is the most accurate test to diagnose the
patient’s underlying condition?

a. Chest x-ray
b. Chest CT scan
c. oracentesis √
d. Bronchoscopy
e. CT pulmonary angiography

Description

oracentesis is the most accurate test to confirm the presence of pleural eusion and dierentiate
exudates from transudates.

Pleural eusion:

Pleural eusion is an abnormal collection of fluids within the pleural space


Pleural fluids can be Transudates or exudates

Chest X-ray (best initial test) but CT scan (more informative than chest x-ray)
oracentesis (most accurate test): dierentiate exudates from Transudates

oracentesis analysis:

Check for pleural fluid protein:


If > 30 g/l, consider exudates
If < 20 g/l, consider transudates
If 20 – 30 g/l and one of the light’s criteria are found, consider exudates
Modified light’s criteria:
Pleural/serum protein ratio > 0.5
Pleural/serum LDH ratio > 0.6
Pleural fluid LDH > two-thirds the upper limit of normal serum

Page - 1935
Treatment of pleural eusion:

Small eusion needs no treatment


Cardiogenic pleural eusion is highly responsive to diuretics
oracentesis can be therapeutic and diagnostic
For refractory and recurrent pleural eusion, pleurodesis (intrapleural bleomycin or talcum
powder)
For empyema, a chest tube, antibiotics, and surgical drainage may be needed

Page - 1936
Internal Medicine - Pulmonology

Question 40/144

Question #40

A 63-year-old male presents with right-side pleural eusion; the pleural fluid analysis reveals
protein of 24 g/L, LDH level of 35 U/L, Serum total protein is 60 g/L (normally 60 – 83), Serum LDH of
140 U/L (normally 100-105). which of the following is the most likely diagnosis?

a. Pulmonary embolism
b. Liver cirrhosis
c. Tuberculous pleurisy
d. Lung cancer
e. Para-pneumonic eusion

‫اﻹﺟﺎﺑﺔ ﻋﲆ اﻟﺼﻔﺤﺔ اﻟﺘﺎﻟﻴﺔ‬

Page - 1937
Internal Medicine - Pulmonology - Pleural Eusion 1

Question 40/144

Question #40

A 63-year-old male presents with right-side pleural eusion; the pleural fluid analysis reveals
protein of 24 g/L, LDH level of 35 U/L, Serum total protein is 60 g/L (normally 60 – 83), Serum LDH of
140 U/L (normally 100-105). which of the following is the most likely diagnosis?

a. Pulmonary embolism
b. Liver cirrhosis √
c. Tuberculous pleurisy
d. Lung cancer
e. Para-pneumonic eusion

Description

When the pleural protein level is between 20 – 30 g/L (2 – 3 g/dL), you should apply light’s criteria to
know the nature of the eusion (exudate vs. transudate)

e fluid analysis:

e fluid protein is less than 30 g/L

Here the pleural/serum protein ratio = 24/60 = 0.4

e pleural/serum LDH ratio = 70/140 = 0.5

e pleural LDH is less than 0.45 of the normal upper limit of the serum LDH.

So, the pleural fluid analysis is going with transudates

e only cause of transudates among the choices is liver cirrhosis.

Pleural eusion:

Pleural eusion is an abnormal collection of fluids within the pleural space


Pleural fluids can be Transudates or exudates

Chest X-ray (best initial test) but CT scan (more informative than chest x-ray)
oracentesis (most accurate test): dierentiate exudates from Transudates

Page - 1938
oracentesis analysis:

Check for pleural fluid protein:


If > 30 g/l, consider exudates
If < 20 g/l, consider transudates
If 20 – 30 g/l and one of the light’s criteria are found, consider exudates
Modified light’s criteria:
Pleural/serum protein ratio > 0.5
Pleural/serum LDH ratio > 0.6
Pleural fluid LDH > two-thirds the upper limit of normal serum

Treatment of pleural eusion:

Small eusion needs no treatment


Cardiogenic pleural eusion is highly responsive to diuretics
oracentesis can be therapeutic and diagnostic
For refractory and recurrent pleural eusion, pleurodesis (intrapleural bleomycin or talcum
powder)
For empyema, a chest tube, antibiotics, and surgical drainage may be needed

Page - 1939
Internal Medicine - Pulmonology

Question 41/144

Question #41

e following choices are correctly matching with each other except:

a. Pleural eusion → fluid collection in the pleura


b. Empyema → pus collection in the pleura
c. Chylothorax → blood collection in the pleura
d. Ascites → fluid collection in the abdominal cavity
e. Pericardial eusion → fluid collection in the pericardium

‫اﻹﺟﺎﺑﺔ ﻋﲆ اﻟﺼﻔﺤﺔ اﻟﺘﺎﻟﻴﺔ‬

Page - 1940
Internal Medicine - Pulmonology - Pleural Eusion 1

Question 41/144

Question #41

e following choices are correctly matching with each other except:

a. Pleural eusion → fluid collection in the pleura


b. Empyema → pus collection in the pleura
c. Chylothorax → blood collection in the pleura √
d. Ascites → fluid collection in the abdominal cavity
e. Pericardial eusion → fluid collection in the pericardium

Description

Chylothorax is a lymphatic fluid collection in the pleural space.

Note that the blood collection in the pleural space is called Hemothorax.

Pleural space collections:

Pneumothorax: air in the pleural space


Hemothorax: blood in pleural space
Chylothorax: lymphatic fluid in pleural space
Empyema: Pus in the pleural space

Page - 1941
Internal Medicine - Pulmonology

Question 42/144

Question #42

A 45-year-old male patient has pulmonary hemorrhage and hemoptysis. Aer an appropriate
workup, he was diagnosed with Goodpasture syndrome. Which of the following pulmonary function
tests findings is expected to be found?

a. High residual volume


b. Increased TLC
c. Increased tidal volume
d. High DLco
e. Reduced FEV1

‫اﻹﺟﺎﺑﺔ ﻋﲆ اﻟﺼﻔﺤﺔ اﻟﺘﺎﻟﻴﺔ‬

Page - 1942
Internal Medicine - Pulmonology - Pulmonary Function Test (PFT)

Question 42/144

Question #42

A 45-year-old male patient has pulmonary hemorrhage and hemoptysis. Aer an appropriate
workup, he was diagnosed with Goodpasture syndrome. Which of the following pulmonary function
tests findings is expected to be found?

a. High residual volume


b. Increased TLC
c. Increased tidal volume
d. High DLco √
e. Reduced FEV1

Description

is patient is suering from pulmonary hemorrhage.

Regardless of the cause, he is expected to have a high DLco.

e other choices mentioned above are not explicitly related to pulmonary hemorrhage

Page - 1943
Internal Medicine - Pulmonology

Question 43/144

Question #43

A 66-year-old smoker presents with recurrent shortness of breath and a fixed obstructive pattern on
spirometry. Which of the following would most likely dierentiate chronic bronchitis from
Emphysema?

a. Total lung capacity


b. Residual volume
c. Expiratory reserve volume
d. Diusing ling capacity of carbon monoxide
e. e arterial blood gases

‫اﻹﺟﺎﺑﺔ ﻋﲆ اﻟﺼﻔﺤﺔ اﻟﺘﺎﻟﻴﺔ‬

Page - 1944
Internal Medicine - Pulmonology - Pulmonary Function Test (PFT)

Question 43/144

Question #43

A 66-year-old smoker presents with recurrent shortness of breath and a fixed obstructive pattern on
spirometry. Which of the following would most likely dierentiate chronic bronchitis from
Emphysema?

a. Total lung capacity


b. Residual volume
c. Expiratory reserve volume
d. Diusing ling capacity of carbon monoxide √
e. e arterial blood gases

Description

DLco is the central part of lung function that is used to dierentiate COPD vs. Emphysema; it will be
decreased in Emphysema and will not be aected in chronic bronchitis

Page - 1945
Internal Medicine - Pulmonology

Question 44/144

Question #44

A 16-year-old female patient has two episodes of acute cough and wheezes. ese symptoms
respond quickly to bronchodilators, but she is now asymptomatic. In addition, she has a history of
eczema and a strong family history of asthma. Which of the following is the most appropriate
investigation?

a. Chest x-ray
b. Peak flow diary
c. Spirometry
d. Arterial blood gas
e. Chest CT scan

‫اﻹﺟﺎﺑﺔ ﻋﲆ اﻟﺼﻔﺤﺔ اﻟﺘﺎﻟﻴﺔ‬

Page - 1946
Internal Medicine - Pulmonology - Pulmonary Function Test (PFT)

Question 44/144

Question #44

A 16-year-old female patient has two episodes of acute cough and wheezes. ese symptoms
respond quickly to bronchodilators, but she is now asymptomatic. In addition, she has a history of
eczema and a strong family history of asthma. Which of the following is the most appropriate
investigation?

a. Chest x-ray
b. Peak flow diary
c. Spirometry √
d. Arterial blood gas
e. Chest CT scan

Description

Spirometry is done here to assess for an obstructive pattern in this patient who is suspected of
having asthma.

Note that peak flow diary and arterial blood gases are used in acute asthma exacerbations to assess
the severity of asthma, but here there is no acute exacerbation as the patient is asymptomatic at
this moment

Chest x-ray and chest CT scan are not diagnostic in asthma, but chest x-ray can be used to rule out
pneumonia or pneumothorax in case of acute asthma exacerbation, and again, there is no acute
exacerbation at this time in this patient

Page - 1947
Internal Medicine - Pulmonology

Question 45/144

Question #45

About DLco, the following statements are factual except:

a. It is used to dierentiate chronic bronchitis from Emphysema


b. It can distinguish intrinsic vs. Extrinsic restrictive lung disease
c. It is usually increased in case of polycythemia, and male individuals
d. In patients with Emphysema, is it usually elevated
e. In anemia, it is expected to be reduced

‫اﻹﺟﺎﺑﺔ ﻋﲆ اﻟﺼﻔﺤﺔ اﻟﺘﺎﻟﻴﺔ‬

Page - 1948
Internal Medicine - Pulmonology - Pulmonary Function Test (PFT)

Question 45/144

Question #45

About DLco, the following statements are factual except:

a. It is used to dierentiate chronic bronchitis from Emphysema


b. It can distinguish intrinsic vs. Extrinsic restrictive lung disease
c. It is usually increased in case of polycythemia, and male individuals
d. In patients with Emphysema, is it usually elevated √
e. In anemia, it is expected to be reduced

Description

DLco is the diusing capacity or transfer factor of the lung for CO gas
It describes the rate of gas diusion from alveoli to blood.
In COPD, DLco dierentiates Emphysema from chronic bronchitis
In restrictive lung diseases, DLco determines intrinsic vs. extrinsic restrictive lung diseases.

DLco is expected to be low in Emphysema

Page - 1949
Internal Medicine - Pulmonology

Question 46/144

Question #46

A 65-year-old male patient presented with shortness of breath. He is a heavy smoker and drinks
alcohol on occasion. His spirometry shows reduced FEV1 and normal FVC. Of the following, what is
the least likely diagnosis?

a. COPD
b. Asthma
c. Bronchiectasis
d. Kyphosis
e. Asthma COPD overlap syndrome

‫اﻹﺟﺎﺑﺔ ﻋﲆ اﻟﺼﻔﺤﺔ اﻟﺘﺎﻟﻴﺔ‬

Page - 1950
Internal Medicine - Pulmonology - Pulmonary Function Test (PFT)

Question 46/144

Question #46

A 65-year-old male patient presented with shortness of breath. He is a heavy smoker and drinks
alcohol on occasion. His spirometry shows reduced FEV1 and normal FVC. Of the following, what is
the least likely diagnosis?

a. COPD
b. Asthma
c. Bronchiectasis
d. Kyphosis √
e. Asthma COPD overlap syndrome

Description

is is an obstructive pattern in spirometry. e only restrictive disease of the mentioned choices is
Kyphosis.

Obstructive lung disease presents with a reduced FEV1/FVC ratio along with increased TLC

Page - 1951
Internal Medicine - Pulmonology

Question 47/144

Question #47

An 80-year-old male patient presents to you for evaluation due to shortness of breath. Which
changes would typically occur with age?

a. Decreased FVC, FEV1, and PEFR


b. Increased Residual volume
c. TLC remains constant
d. A collapse of the small airways
e. All of the above

‫اﻹﺟﺎﺑﺔ ﻋﲆ اﻟﺼﻔﺤﺔ اﻟﺘﺎﻟﻴﺔ‬

Page - 1952
Internal Medicine - Pulmonology - Pulmonary Function Test (PFT)

Question 47/144

Question #47

An 80-year-old male patient presents to you for evaluation due to shortness of breath. Which
changes would typically occur with age?

a. Decreased FVC, FEV1, and PEFR


b. Increased Residual volume
c. TLC remains constant
d. A collapse of the small airways
e. All of the above √

Description

e following results are typical changes to occur with age:

Vital capacity declines with age


Total lung capacity remains constant.
e reduction in vital capacity (VC) results from an increase in residual volume
Residual volume increases nearly 50% between early adulthood and age 70
FEV1, PEFR, and maximal expiratory flow volume have been shown to decline
Arterial oxygen tension also slowly decreases with age
A Collapse of small airways

Page - 1953
Internal Medicine - Pulmonology

Question 48/144

Question #48

A 65-year-old male patient known to be a heavy smoker for 40 years presented to you to evaluate
his progressively increasing shortness of breath for 6 months. He was a known case of Tuberculosis
20 years ago. His spirometry shows an FVC of 59%, FEV1 of 80%, and FEV1/FVC ratio was 0.81. what is
the most appropriate next step in the management?

a. Order diusion capacity of the lung


b. Methacholine challenge test
c. Order Mantoux test
d. Start short-acting beta-agonists
e. Order a high-resolution CT scan

‫اﻹﺟﺎﺑﺔ ﻋﲆ اﻟﺼﻔﺤﺔ اﻟﺘﺎﻟﻴﺔ‬

Page - 1954
Internal Medicine - Pulmonology - Pulmonary Function Test (PFT)

Question 48/144

Question #48

A 65-year-old male patient known to be a heavy smoker for 40 years presented to you to evaluate
his progressively increasing shortness of breath for 6 months. He was a known case of Tuberculosis
20 years ago. His spirometry shows an FVC of 59%, FEV1 of 80%, and FEV1/FVC ratio was 0.81. what is
the most appropriate next step in the management?

a. Order diusion capacity of the lung √


b. Methacholine challenge test
c. Order Mantoux test
d. Start short-acting beta-agonists
e. Order a high-resolution CT scan

Description

is is a case of restrictive pattern on spirometry, and DLco is the best next step to dierentiate
intrinsic vs. extrinsic restrictive disorders

Page - 1955
Internal Medicine - Pulmonology

Question 49/144

Question #49

A 55-year-old female presents with shortness of breath on exertion for six months; she has been a
heavy smoker for the past 32 years. What is the best diagnostic test to diagnose this patient?

a. Arterial blood gases


b. Chest x-ray
c. Chest CT scan
d. Brain natriuretic peptide
e. Spirometry

‫اﻹﺟﺎﺑﺔ ﻋﲆ اﻟﺼﻔﺤﺔ اﻟﺘﺎﻟﻴﺔ‬

Page - 1956
Internal Medicine - Pulmonology - Pulmonary Function Test (PFT)

Question 49/144

Question #49

A 55-year-old female presents with shortness of breath on exertion for six months; she has been a
heavy smoker for the past 32 years. What is the best diagnostic test to diagnose this patient?

a. Arterial blood gases


b. Chest x-ray
c. Chest CT scan
d. Brain natriuretic peptide
e. Spirometry √

Description

e most likely diagnosis is COPD because she is a heavy smoker and old with chronic dyspnea.

Spirometry is the best tool for the diagnosis of COPD, which will show a fixed obstructive pattern

Page - 1957
Internal Medicine - Pulmonology

Question 50/144

Question #50

A 46-year-old male patient presented with chronic shortness of breath. Aer doing a pulmonary
function test, you find FEV1 and FEV1/FVC ratio to be reduced, and the total lung capacity was
increased. What is the least likely diagnosis?

a. Asthma
b. COPD
c. Bronchiectasis
d. Sarcoidosis
e. cystic fibrosis

‫اﻹﺟﺎﺑﺔ ﻋﲆ اﻟﺼﻔﺤﺔ اﻟﺘﺎﻟﻴﺔ‬

Page - 1958
Internal Medicine - Pulmonology - Pulmonary Function Test (PFT)

Question 50/144

Question #50

A 46-year-old male patient presented with chronic shortness of breath. Aer doing a pulmonary
function test, you find FEV1 and FEV1/FVC ratio to be reduced, and the total lung capacity was
increased. What is the least likely diagnosis?

a. Asthma
b. COPD
c. Bronchiectasis
d. Sarcoidosis √
e. cystic fibrosis

Description

Obstructive lung disease presents with a reduced FEV1/FVC ratio along with increased TLC. e only
restrictive disease from the mentioned choices is sarcoidosis

Page - 1959
Internal Medicine - Pulmonology

Question 51/144

Question #51

A 59-year-old heavy smoker was found to have fixed obstruction on PFT, and the diagnosis of COPD
was made 10 years ago. Despite treatment, he presents with persistent shortness of breath, his O2
saturation is 87%, and his PaO2 is 53 mmHg. e echocardiogram confirms the presence of moderate
pulmonary HTN and the Ejection fraction to be normal. What is the most appropriate treatment at
this time?

a. Continue same treatment


b. Start long-term oxygen therapy
c. Start sildenafil (Viagra)
d. Start Nifedipine
e. Low dose steroid

‫اﻹﺟﺎﺑﺔ ﻋﲆ اﻟﺼﻔﺤﺔ اﻟﺘﺎﻟﻴﺔ‬

Page - 1960
Internal Medicine - Pulmonology - Pulmonary Hypertension

Question 51/144

Question #51

A 59-year-old heavy smoker was found to have fixed obstruction on PFT, and the diagnosis of COPD
was made 10 years ago. Despite treatment, he presents with persistent shortness of breath, his O2
saturation is 87%, and his PaO2 is 53 mmHg. e echocardiogram confirms the presence of moderate
pulmonary HTN and the Ejection fraction to be normal. What is the most appropriate treatment at
this time?

a. Continue same treatment


b. Start long-term oxygen therapy √
c. Start sildenafil (Viagra)
d. Start Nifedipine
e. Low dose steroid

Description

e criteria to use LTOT here are met, and it is the best treatment for this patient’s pulmonary HTN.

Sildenafil and CCBs are used in treating pulmonary HTN not related to COPD or hypoxia.

Page - 1961
Internal Medicine - Pulmonology

Question 52/144

Question #52

A 49-year-old male with a known case of interstitial lung disease presents with shortness of breath
and exertional dyspnea. What is the best way to confirm the presence of pulmonary HTN in this
patient?

a. Chest X-ray
b. Echocardiogram
c. Swan-Ganz catheterization
d. ECG
e. CTPA

‫اﻹﺟﺎﺑﺔ ﻋﲆ اﻟﺼﻔﺤﺔ اﻟﺘﺎﻟﻴﺔ‬

Page - 1962
Internal Medicine - Pulmonology - Pulmonary Hypertension

Question 52/144

Question #52

A 49-year-old male with a known case of interstitial lung disease presents with shortness of breath
and exertional dyspnea. What is the best way to confirm the presence of pulmonary HTN in this
patient?

a. Chest X-ray
b. Echocardiogram
c. Swan-Ganz catheterization √
d. ECG
e. CTPA

Description

Swan-Ganz catheterization is the most accurate test to diagnose pulmonary HTN. It accurately
measures the pulmonary artery pressure

Chest X-ray can point to the possible cause of pulmonary HTN

e echocardiogram will estimate the pulmonary pressure, but the Swan-Ganz catheter is more
accurate

ECG is not specific, but it will show features of RVH, RAH, and cor pulmonale

CTPA is used when recurrent PE is suspected as a cause for pulmonary HTN, but it doesn’t diagnose
the presence of high pulmonary arterial pressure

Page - 1963
Internal Medicine - Pulmonology

Question 53/144

Question #53

Aer the breakup with her boyfriend, a 19-year-old female presents to the ER with lightheadedness,
shortness of breath, headaches, and tingling around her mouth and on her tongue. e patient
appears anxious, and the life-threatening conditions have been ruled out. What would you do next?

a. Order urine and blood toxicology screen


b. Explanation and reassurance
c. Give diazepam intravenously
d. Give Oxygen supplementation
e. Start intravenous fluids

‫اﻹﺟﺎﺑﺔ ﻋﲆ اﻟﺼﻔﺤﺔ اﻟﺘﺎﻟﻴﺔ‬

Page - 1964
Internal Medicine - Pulmonology - Respiratory Alkalosis

Question 53/144

Question #53

Aer the breakup with her boyfriend, a 19-year-old female presents to the ER with lightheadedness,
shortness of breath, headaches, and tingling around her mouth and on her tongue. e patient
appears anxious, and the life-threatening conditions have been ruled out. What would you do next?

a. Order urine and blood toxicology screen


b. Explanation and reassurance √
c. Give diazepam intravenously
d. Give Oxygen supplementation
e. Start intravenous fluids

Description

In young patients with normal physical examination and absence of hypoxemia, hyperventilation is
most likely due to hyperventilation syndrome.

Hyperventilation syndrome needs no treatment other than explanation and reassurance. If it is


recurrent, refer the patient to a psychiatrist.

Hyperventilation syndrome: when a patient is exposed to stress, he starts to hyperventilate, feeling


unwell, then more hyperventilation develops.

Respiratory alkalosis is a feature


A patient may develop symptoms like chest pain, numbness, and weakness
Treat the patients by making them breathe into a bag to reduce the CO2 wash

Page - 1965
Internal Medicine - Pulmonology

Question 54/144

Question #54

A 32-year-old male patient presented with tachypnea and chest tightness. His ABGs show PH 7.56,
PaO2 100 mmHg, and PaCO2 24. What is the most likely diagnosis?

a. Pulmonary embolism
b. Anxiety induced Hyperventilation syndrome
c. Alpha 1 antitrypsin deficiency
d. Tension pneumothorax
e. Acute myocardial infarction

‫اﻹﺟﺎﺑﺔ ﻋﲆ اﻟﺼﻔﺤﺔ اﻟﺘﺎﻟﻴﺔ‬

Page - 1966
Internal Medicine - Pulmonology - Respiratory Alkalosis

Question 54/144

Question #54

A 32-year-old male patient presented with tachypnea and chest tightness. His ABGs show PH 7.56,
PaO2 100 mmHg, and PaCO2 24. What is the most likely diagnosis?

a. Pulmonary embolism
b. Anxiety induced Hyperventilation syndrome √
c. Alpha 1 antitrypsin deficiency
d. Tension pneumothorax
e. Acute myocardial infarction

Description

is patient has respiratory alkalosis without hypoxia.

Anxiety hyperventilation presents with respiratory alkalosis without hypoxia, but PE presents with
hypoxia.

Hyperventilation syndrome: when a patient is exposed to stress, he starts to hyperventilate, feeling


unwell, then more hyperventilation develops.

Respiratory alkalosis is a feature


A patient may develop symptoms like chest pain, numbness, and weakness
Treat the patients by making them breathe into a bag to reduce the CO2 wash

Page - 1967
Internal Medicine - Pulmonology

Question 55/144

Question #55

A 21-year-old female presents to you with recurrent shortness of breath, palpitations, and a tingling
sensation in her hands. is condition usually starts when she is in a crowded area, lasts for about 30
minutes, and then goes away. e patient has no medical history of note, and her vital signs are
normal during the episodes. Which ABGs finding is expected to be encountered in this patient
during the attack?

a. PH: 7.55, PaO2: 60 mmHg, PaCO2: 19 mmHg, HCO3: 26 mmol/l


b. PH: 7.56, PaO2: 99 mmHg, PaCO2: 18 mmHg, HCO3: 13 mmol/l
c. PH: 7.57, PaO2: 99 mmHg, PaCO2: 20 mmHg, HCO3: 25 mmol/l
d. PH: 7.28, PaO2: 105 mmHg, PaCO2: 48 mmHg, HCO3: 24 mmol/l
e. PH: 7.27, PaO2: 62 mmHg, PaCO2: 51 mmHg, HCO3: 24 mmol/l

‫اﻹﺟﺎﺑﺔ ﻋﲆ اﻟﺼﻔﺤﺔ اﻟﺘﺎﻟﻴﺔ‬

Page - 1968
Internal Medicine - Pulmonology - Respiratory Alkalosis

Question 55/144

Question #55

A 21-year-old female presents to you with recurrent shortness of breath, palpitations, and a tingling
sensation in her hands. is condition usually starts when she is in a crowded area, lasts for about 30
minutes, and then goes away. e patient has no medical history of note, and her vital signs are
normal during the episodes. Which ABGs finding is expected to be encountered in this patient
during the attack?

a. PH: 7.55, PaO2: 60 mmHg, PaCO2: 19 mmHg, HCO3: 26 mmol/l


b. PH: 7.56, PaO2: 99 mmHg, PaCO2: 18 mmHg, HCO3: 13 mmol/l
c. PH: 7.57, PaO2: 99 mmHg, PaCO2: 20 mmHg, HCO3: 25 mmol/l √
d. PH: 7.28, PaO2: 105 mmHg, PaCO2: 48 mmHg, HCO3: 24 mmol/l
e. PH: 7.27, PaO2: 62 mmHg, PaCO2: 51 mmHg, HCO3: 24 mmol/l

Description

Description:

is patient is most likely to have acute respiratory alkalosis secondary to hyperventilation
syndrome and agoraphobia.

Hyperventilation will lead to reduced CO2 and elevated PH but no hypoxia.

HCO3 needs time to start changing, so in the case of acute respiratory alkalosis, it is expected to be
normal.

In hyperventilation syndrome (high PH, normal O2, normal HCO3, and low CO2)

Page - 1969
Internal Medicine - Pulmonology

Question 56/144

Question #56

A 56-year-old male patient presents with shortness of breath and chest pain. His ABGs show PH 7.58,
PaCO2 23 mmHg, PaO2 80 mmHg. What is the acid-base balance in this patient?

a. Metabolic alkalosis
b. Respiratory alkalosis
c. Metabolic acidosis
d. Respiratory acidosis
e. Information is not enough

‫اﻹﺟﺎﺑﺔ ﻋﲆ اﻟﺼﻔﺤﺔ اﻟﺘﺎﻟﻴﺔ‬

Page - 1970
Internal Medicine - Pulmonology - Respiratory Alkalosis

Question 56/144

Question #56

A 56-year-old male patient presents with shortness of breath and chest pain. His ABGs show PH 7.58,
PaCO2 23 mmHg, PaO2 80 mmHg. What is the acid-base balance in this patient?

a. Metabolic alkalosis
b. Respiratory alkalosis √
c. Metabolic acidosis
d. Respiratory acidosis
e. Information is not enough

Description

e high PH indicates alkalosis, and the CO2 wash indicates a respiratory cause.

Reduced CO2 due to hyperventilation resulted in elevated PH (Alkalosis)

Page - 1971
Internal Medicine - Pulmonology

Question 57/144

Question #57

A 17-year-old female presents with tachypnea, tachycardia, carpal spasms, and numbness.
Regarding hyperventilation syndrome, all the following statements are true except:

a. CO2 retention is a feature


b. It is common aer stressful condition
c. Respiratory alkalosis is a feature
d. e patient may develop chest pain, numbness, and weakness
e. Explanation and reassurance are the main treatment

‫اﻹﺟﺎﺑﺔ ﻋﲆ اﻟﺼﻔﺤﺔ اﻟﺘﺎﻟﻴﺔ‬

Page - 1972
Internal Medicine - Pulmonology - Respiratory Alkalosis

Question 57/144

Question #57

A 17-year-old female presents with tachypnea, tachycardia, carpal spasms, and numbness.
Regarding hyperventilation syndrome, all the following statements are true except:

a. CO2 retention is a feature √


b. It is common aer stressful condition
c. Respiratory alkalosis is a feature
d. e patient may develop chest pain, numbness, and weakness
e. Explanation and reassurance are the main treatment

Description

In hyperventilation syndrome, CO2 wash, not CO2 retention, is the main ABG finding.

Hyperventilation syndrome: when a patient is exposed to stress, he starts to hyperventilate, feeling


unwell, then more hyperventilation develops.

Respiratory alkalosis is a feature


A patient may develop symptoms like chest pain, numbness, and weakness
Treat the patients by making them breathe into a bag to reduce the CO2 wash

Page - 1973
Internal Medicine - Pulmonology

Question 58/144

Question #58

A 70-year-old male patient who has been a heavy smoker for the last 40 years presents with night
sweats, weight loss, continuous cough, and hoarseness of voice for the previous 6 months. Recently
he got more fatigue and has been confused at times. His lab investigations show serum sodium of
125 mEq/L, serum calcium of 9.2 mg/dL, serum glucose of 90 mg/dL, and serum potassium of 4
mEq/L. His kidney function test and liver function test are normal. What is the most likely diagnosis?

a. Sarcoidosis
b. Squamous cell lung cancer
c. Small cell lung cancer
d. Adenocarcinoma of the lung
e. Large cell lung cancer

‫اﻹﺟﺎﺑﺔ ﻋﲆ اﻟﺼﻔﺤﺔ اﻟﺘﺎﻟﻴﺔ‬

Page - 1974
Internal Medicine - Pulmonology - Respiratory neoplasms

Question 58/144

Question #58

A 70-year-old male patient who has been a heavy smoker for the last 40 years presents with night
sweats, weight loss, continuous cough, and hoarseness of voice for the previous 6 months. Recently
he got more fatigue and has been confused at times. His lab investigations show serum sodium of
125 mEq/L, serum calcium of 9.2 mg/dL, serum glucose of 90 mg/dL, and serum potassium of 4
mEq/L. His kidney function test and liver function test are normal. What is the most likely diagnosis?

a. Sarcoidosis
b. Squamous cell lung cancer
c. Small cell lung cancer √
d. Adenocarcinoma of the lung
e. Large cell lung cancer

Description

e presence of chronic cough, weight loss, night sweating, and a history of heavy smoking will raise
the suspicion of lung cancer.

e type of lung cancer that is associated with hyponatremia as a paraneoplastic feature due to ADH
release is small-cell lung cancer

Page - 1975
Internal Medicine - Pulmonology

Question 59/144

Question #59

A man presents for routine evaluation and is found to have cavitary lesions on a chest x-ray. All the
following are possible causes except:

a. Squamous cell lung cancer


b. Klebsiella pneumonia
c. Lung abscess
d. COVID-19
e. Tuberculosis

‫اﻹﺟﺎﺑﺔ ﻋﲆ اﻟﺼﻔﺤﺔ اﻟﺘﺎﻟﻴﺔ‬

Page - 1976
Internal Medicine - Pulmonology - Respiratory neoplasms

Question 59/144

Question #59

A man presents for routine evaluation and is found to have cavitary lesions on a chest x-ray. All the
following are possible causes except:

a. Squamous cell lung cancer


b. Klebsiella pneumonia
c. Lung abscess
d. COVID-19 √
e. Tuberculosis

Description

Causes of cavitating lesions on chest X-Ray are:

Lung abscess (Staph aureus, Klebsiella, and Pseudomonas)


Squamous cell lung cancer
Pulmonary Tuberculosis
Wegener’s granulomatosis
Pulmonary embolism
Rheumatoid arthritis
Sarcoidosis
Aspergillosis
Histoplasmosis
Coccidioidomycosis

Page - 1977
Internal Medicine - Pulmonology

Question 60/144

Question #60

A patient was diagnosed with squamous cell lung cancer and now develops hypercalcemia. What is
the primary mechanism of the development of hypercalcemia in this patient?

a. Vitamin D activation by the macrophages


b. Parathyroid hormone-related peptide release
c. Antidiuretic hormone release
d. Increased osteoblastic activation
e. Iatrogenic due to treatment

‫اﻹﺟﺎﺑﺔ ﻋﲆ اﻟﺼﻔﺤﺔ اﻟﺘﺎﻟﻴﺔ‬

Page - 1978
Internal Medicine - Pulmonology - Respiratory neoplasms

Question 60/144

Question #60

A patient was diagnosed with squamous cell lung cancer and now develops hypercalcemia. What is
the primary mechanism of the development of hypercalcemia in this patient?

a. Vitamin D activation by the macrophages


b. Parathyroid hormone-related peptide release √
c. Antidiuretic hormone release
d. Increased osteoblastic activation
e. Iatrogenic due to treatment

Description

PTHrP is released by squamous cell lung cancer leading to hypercalcemia as a paraneoplastic


syndrome.

Note that ADH release is a feature of small-cell lung cancer

Vitamin D activation by the macrophages occurs in sarcoidosis

Chemotherapy may cause hypercalcemia, but it is not the primary mechanism in squamous cell lung
cancer.

Page - 1979
Internal Medicine - Pulmonology

Question 61/144

Question #61

A 27-year-old female presents with shortness of breath for 2 months, painful reddish lesions on both
legs, and intermittent joint stiness. Her physical examination is unremarkable, and her chest x-ray
shows bilateral hilar lymphadenopathy and diuse interstitial infiltrations. All the following are true
about her condition except:

a. Hypercalcemia may be a feature


b. A chest x-ray is used for staging in this patient
c. A biopsy is the most accurate test to do
d. Facial palsy may manifest
e. e ACE level is essential in the diagnosis

‫اﻹﺟﺎﺑﺔ ﻋﲆ اﻟﺼﻔﺤﺔ اﻟﺘﺎﻟﻴﺔ‬

Page - 1980
Internal Medicine - Pulmonology - Sarcoidosis

Question 61/144

Question #61

A 27-year-old female presents with shortness of breath for 2 months, painful reddish lesions on both
legs, and intermittent joint stiness. Her physical examination is unremarkable, and her chest x-ray
shows bilateral hilar lymphadenopathy and diuse interstitial infiltrations. All the following are true
about her condition except:

a. Hypercalcemia may be a feature


b. A chest x-ray is used for staging in this patient
c. A biopsy is the most accurate test to do
d. Facial palsy may manifest
e. e ACE level is essential in the diagnosis √

Description

e ACE level may be high (it is not specific, not sensitive, but can be used for the monitoring of the
disease activity →a negligible role in the diagnosis)

Sarcoidosis:

It is a multi-system disease characterized by non-caseating granuloma


It has an unknown etiology but usually occurs in young and black patients, and it is more
common in non-smokers.
e clinical presentation consists of shortness of breath, erythema nodosum, bilateral hilar
lymphadenopathy on chest x-ray, hypercalcemia, and neurological manifestations
Lofgren’s syndrome is an acute sarcoidosis characterized by bilateral hilar lymphadenopathy
(BHL), erythema nodosum, fever, and polyarthralgia. It usually carries an excellent prognosis
Heerfordt’s syndrome (uveoparotid fever), there is parotid enlargement, fever, uveitis, and
facial palsy secondary to sarcoidosis
e most accurate test is a biopsy that shows non-caseating granuloma
e staging system depends on the chest x-ray findings

Page - 1981
Internal Medicine - Pulmonology

Question 62/144

Question #62

A 32-year-old female with shortness of breath and cervical lymphadenopathy presents for
evaluation, she has painful erythematous nodules on her shins, and her chest x-ray shows bilateral
hilar lymphadenopathies. All of the following are considered poor prognostic factors for this patient
except:

a. Elderly patient
b. Presence of erythema nodosum
c. Presence of facial palsy
d. Presence of lupus pernio
e. Stage IV on chest X-ray

‫اﻹﺟﺎﺑﺔ ﻋﲆ اﻟﺼﻔﺤﺔ اﻟﺘﺎﻟﻴﺔ‬

Page - 1982
Internal Medicine - Pulmonology - Sarcoidosis

Question 62/144

Question #62

A 32-year-old female with shortness of breath and cervical lymphadenopathy presents for
evaluation, she has painful erythematous nodules on her shins, and her chest x-ray shows bilateral
hilar lymphadenopathies. All of the following are considered poor prognostic factors for this patient
except:

a. Elderly patient
b. Presence of erythema nodosum √
c. Presence of facial palsy
d. Presence of lupus pernio
e. Stage IV on chest X-ray

Description

is is a typical presentation of sarcoidosis.

Poor prognostic factors in sarcoidosis:

An absence of erythema nodosum


Black people and Older ages
Chest X-Ray: stage III-IV
Extra-pulmonary manifestations: e.g., lupus pernio, splenomegaly
Insidious onset, symptoms > 6 months

Sarcoidosis:

It is a multi-system disease characterized by non-caseating granuloma


It has an unknown etiology but usually occurs in young and black patients, and it is more
common in non-smokers.
e clinical presentation consists of shortness of breath, erythema nodosum, bilateral hilar
lymphadenopathy on chest x-ray, hypercalcemia, and neurological manifestations
Lofgren’s syndrome is an acute sarcoidosis characterized by bilateral hilar lymphadenopathy
(BHL), erythema nodosum, fever, and polyarthralgia. It usually carries an excellent prognosis
Heerfordt’s syndrome (uveoparotid fever), there is parotid enlargement, fever, uveitis, and
facial palsy secondary to sarcoidosis
Page - 1983
e most accurate test is a biopsy that shows non-caseating granuloma
e staging system depends on the chest x-ray findings

Page - 1984
Internal Medicine - Pulmonology

Question 63/144

Question #63

A 32-year-old female with shortness of breath and cervical lymphadenopathy presents for
evaluation, she has painful erythematous nodules on her shins, and her chest x-ray shows bilateral
hilar lymphadenopathies. What is the most likely diagnosis?

a. Idiopathic pulmonary fibrosis


b. Sarcoidosis
c. Alpha 1 antitrypsin deficiency
d. Lymphoma
e. Metastatic thyroid cancer

‫اﻹﺟﺎﺑﺔ ﻋﲆ اﻟﺼﻔﺤﺔ اﻟﺘﺎﻟﻴﺔ‬

Page - 1985
Internal Medicine - Pulmonology - Sarcoidosis

Question 63/144

Question #63

A 32-year-old female with shortness of breath and cervical lymphadenopathy presents for
evaluation, she has painful erythematous nodules on her shins, and her chest x-ray shows bilateral
hilar lymphadenopathies. What is the most likely diagnosis?

a. Idiopathic pulmonary fibrosis


b. Sarcoidosis √
c. Alpha 1 antitrypsin deficiency
d. Lymphoma
e. Metastatic thyroid cancer

Description

Erythema nodosum, bilateral hilar lymphadenopathy, and cervical lymphadenopathy are suggestive
of sarcoidosis

Sarcoidosis:

It is a multi-system disease characterized by non-caseating granuloma


It has an unknown etiology but usually occurs in young and black patients, and it is more
common in non-smokers.
e clinical presentation consists of shortness of breath, erythema nodosum, bilateral hilar
lymphadenopathy on chest x-ray, hypercalcemia, and neurological manifestations
Lofgren’s syndrome is an acute sarcoidosis characterized by bilateral hilar lymphadenopathy
(BHL), erythema nodosum, fever, and polyarthralgia. It usually carries an excellent prognosis
Heerfordt’s syndrome (uveoparotid fever), there is parotid enlargement, fever, uveitis, and
facial palsy secondary to sarcoidosis
e most accurate test is a biopsy that shows non-caseating granuloma
e staging system depends on the chest x-ray findings

Page - 1986
Internal Medicine - Pulmonology

Question 64/144

Question #64

A 32-year-old female with shortness of breath and cervical lymphadenopathy presents for
evaluation, she has painful erythematous nodules on her shins, and her chest x-ray shows bilateral
hilar lymphadenopathies. Which of the following is expected to be found in this patient?

a. Hypocalcemia
b. Hypercalcemia
c. Hypokalemia
d. Hyperkalemia
e. Hypomagnesemia

‫اﻹﺟﺎﺑﺔ ﻋﲆ اﻟﺼﻔﺤﺔ اﻟﺘﺎﻟﻴﺔ‬

Page - 1987
Internal Medicine - Pulmonology - Sarcoidosis

Question 64/144

Question #64

A 32-year-old female with shortness of breath and cervical lymphadenopathy presents for
evaluation, she has painful erythematous nodules on her shins, and her chest x-ray shows bilateral
hilar lymphadenopathies. Which of the following is expected to be found in this patient?

a. Hypocalcemia
b. Hypercalcemia √
c. Hypokalemia
d. Hyperkalemia
e. Hypomagnesemia

Description

Macrophages inside the granulomas cause an increased conversion of vitamin D to its active form
(1,25-dihydroxycholecalciferol), leading to hypercalcemia.

Sarcoidosis:

It is a multi-system disease characterized by non-caseating granuloma


It has an unknown etiology but usually occurs in young and black patients, and it is more
common in non-smokers.
e clinical presentation consists of shortness of breath, erythema nodosum, bilateral hilar
lymphadenopathy on chest x-ray, hypercalcemia, and neurological manifestations
Lofgren’s syndrome is an acute sarcoidosis characterized by bilateral hilar lymphadenopathy
(BHL), erythema nodosum, fever, and polyarthralgia. It usually carries an excellent prognosis
Heerfordt’s syndrome (uveoparotid fever), there is parotid enlargement, fever, uveitis, and
facial palsy secondary to sarcoidosis
e most accurate test is a biopsy that shows non-caseating granuloma
e staging system depends on the chest x-ray findings

Page - 1988
Internal Medicine - Pulmonology

Question 65/144

Question #65

A 29-year-old male patient complained of progressive shortness of breath and dry cough over the
past six weeks. His chest x-ray shows bilateral hilar lymphadenopathy, and his shins show nodular
cherry-sized lumps. What is the most likely diagnosis?

d. Cystic fibrosis
e. Pneumonia
a. Bronchial asthma
b. Sarcoidosis
c. Alpha 1 antitrypsin deficiency

‫اﻹﺟﺎﺑﺔ ﻋﲆ اﻟﺼﻔﺤﺔ اﻟﺘﺎﻟﻴﺔ‬

Page - 1989
Internal Medicine - Pulmonology - Sarcoidosis

Question 65/144

Question #65

A 29-year-old male patient complained of progressive shortness of breath and dry cough over the
past six weeks. His chest x-ray shows bilateral hilar lymphadenopathy, and his shins show nodular
cherry-sized lumps. What is the most likely diagnosis?

d. Cystic fibrosis
e. Pneumonia
a. Bronchial asthma
b. Sarcoidosis √
c. Alpha 1 antitrypsin deficiency

Description

Any questions in the exam mention shin lesions and bilateral hilar lymphadenopathy, you should
think about sarcoidosis. is lesion is most likely to be erythema nodosum.

Sarcoidosis:

It is a multi-system disease characterized by non-caseating granuloma


It has an unknown etiology but usually occurs in young and black patients, and it is more
common in non-smokers.
e clinical presentation consists of shortness of breath, erythema nodosum, bilateral hilar
lymphadenopathy on chest x-ray, hypercalcemia, and neurological manifestations
Lofgren’s syndrome is an acute sarcoidosis characterized by bilateral hilar lymphadenopathy
(BHL), erythema nodosum, fever, and polyarthralgia. It usually carries an excellent prognosis
Heerfordt’s syndrome (uveoparotid fever), there is parotid enlargement, fever, uveitis, and
facial palsy secondary to sarcoidosis
e most accurate test is a biopsy that shows non-caseating granuloma
e staging system depends on the chest x-ray findings

Page - 1990
Internal Medicine - Pulmonology

Question 66/144

Question #66

You confirmed a diagnosis of sarcoidosis in a 32-year-old male patient. Which of the following tests
should be done next?

a. Electroencephalogram (EEG)
b. Genetic testing
c. Liver biopsy
d. Renal biopsy
e. Slit-lamp examination

‫اﻹﺟﺎﺑﺔ ﻋﲆ اﻟﺼﻔﺤﺔ اﻟﺘﺎﻟﻴﺔ‬

Page - 1991
Internal Medicine - Pulmonology - Sarcoidosis

Question 66/144

Question #66

You confirmed a diagnosis of sarcoidosis in a 32-year-old male patient. Which of the following tests
should be done next?

a. Electroencephalogram (EEG)
b. Genetic testing
c. Liver biopsy
d. Renal biopsy
e. Slit-lamp examination √

Description

e necessary tests to be done in a patient with sarcoidosis aer the diagnosis are:

Serum calcium level (to rule out hypercalcemia)


Pulmonary function testing
Slit-lamp examination (to rule out uveitis)
ECG
Serum ACE level

Sarcoidosis:

It is a multi-system disease characterized by non-caseating granuloma


It has an unknown etiology but usually occurs in young and black patients, and it is more
common in non-smokers.
e clinical presentation consists of shortness of breath, erythema nodosum, bilateral hilar
lymphadenopathy on chest x-ray, hypercalcemia, and neurological manifestations
Lofgren’s syndrome is an acute sarcoidosis characterized by bilateral hilar lymphadenopathy
(BHL), erythema nodosum, fever, and polyarthralgia. It usually carries an excellent prognosis
Heerfordt’s syndrome (uveoparotid fever), there is parotid enlargement, fever, uveitis, and
facial palsy secondary to sarcoidosis
e most accurate test is a biopsy that shows non-caseating granuloma
e staging system depends on the chest x-ray findings

Treatment of sarcoidosis:
Page - 1992
Stages 1&2: spontaneous resolution
Steroids may be needed If:
Stages > 2 with progressive symptoms
Hypercalcemia
Eye-heart or CNS involvement

Page - 1993
Internal Medicine - Pulmonology

Question 67/144

Question #67

A 32-year-old female with shortness of breath and cervical lymphadenopathy presents for
evaluation, she has painful erythematous nodules on her shins, and her chest x-ray shows bilateral
hilar lymphadenopathies. What is the most accurate test to confirm the diagnosis?

a. Chest CT scan
b. Lymph node biopsy
c. ESR level
d. ACE level
e. Spirometry

‫اﻹﺟﺎﺑﺔ ﻋﲆ اﻟﺼﻔﺤﺔ اﻟﺘﺎﻟﻴﺔ‬

Page - 1994
Internal Medicine - Pulmonology - Sarcoidosis

Question 67/144

Question #67

A 32-year-old female with shortness of breath and cervical lymphadenopathy presents for
evaluation, she has painful erythematous nodules on her shins, and her chest x-ray shows bilateral
hilar lymphadenopathies. What is the most accurate test to confirm the diagnosis?

a. Chest CT scan
b. Lymph node biopsy √
c. ESR level
d. ACE level
e. Spirometry

Description

is is a typical scenario of sarcoidosis. erefore, the most accurate test is a biopsy (from the lung or
the lymph nodes) that shows non-caseating granuloma

Sarcoidosis:

It is a multi-system disease characterized by non-caseating granuloma


It has an unknown etiology but usually occurs in young and black patients, and it is more
common in non-smokers.
e clinical presentation consists of shortness of breath, erythema nodosum, bilateral hilar
lymphadenopathy on chest x-ray, hypercalcemia, and neurological manifestations
Lofgren’s syndrome is an acute sarcoidosis characterized by bilateral hilar lymphadenopathy
(BHL), erythema nodosum, fever, and polyarthralgia. It usually carries an excellent prognosis
Heerfordt’s syndrome (uveoparotid fever), there is parotid enlargement, fever, uveitis, and
facial palsy secondary to sarcoidosis
e most accurate test is a biopsy that shows non-caseating granuloma
e staging system depends on the chest x-ray findings

Page - 1995
Internal Medicine - Pulmonology

Question 68/144

Question #68

A 32-year-old female with shortness of breath and cervical lymphadenopathy presents for
evaluation, she has painful erythematous nodules on her shins, and her chest x-ray shows bilateral
hilar lymphadenopathies. Her lab investigation and the rest of the physical examination are normal,
Slit-lamp test and ECG are normal. What is the treatment indicated for this patient?

a. Inhaled corticosteroids
b. No treatment is indicated at this stage
c. Provide short-acting beta-agonist
d. Refer to a hematologist for evaluation
e. Start systemic steroid therapy

‫اﻹﺟﺎﺑﺔ ﻋﲆ اﻟﺼﻔﺤﺔ اﻟﺘﺎﻟﻴﺔ‬

Page - 1996
Internal Medicine - Pulmonology - Sarcoidosis

Question 68/144

Question #68

A 32-year-old female with shortness of breath and cervical lymphadenopathy presents for
evaluation, she has painful erythematous nodules on her shins, and her chest x-ray shows bilateral
hilar lymphadenopathies. Her lab investigation and the rest of the physical examination are normal,
Slit-lamp test and ECG are normal. What is the treatment indicated for this patient?

a. Inhaled corticosteroids
b. No treatment is indicated at this stage √
c. Provide short-acting beta-agonist
d. Refer to a hematologist for evaluation
e. Start systemic steroid therapy

Description

is is stage 1 of sarcoidosis, with no hypocalcemia, eye, heart, or CNS involvement. So, a
spontaneous resolution will occur mostly, and no steroid therapy is indicated in this case.

Sarcoidosis:

It is a multi-system disease characterized by non-caseating granuloma


It has an unknown etiology but usually occurs in young and black patients, and it is more
common in non-smokers.
e clinical presentation consists of shortness of breath, erythema nodosum, bilateral hilar
lymphadenopathy on chest x-ray, hypercalcemia, and neurological manifestations
Lofgren’s syndrome is an acute sarcoidosis characterized by bilateral hilar lymphadenopathy
(BHL), erythema nodosum, fever, and polyarthralgia. It usually carries an excellent prognosis
Heerfordt’s syndrome (uveoparotid fever), there is parotid enlargement, fever, uveitis, and
facial palsy secondary to sarcoidosis
e most accurate test is a biopsy that shows non-caseating granuloma
e staging system depends on the chest x-ray findings

Treatment of sarcoidosis:

Stages 1&2: spontaneous resolution


Steroids may be needed If:
Page - 1997
Stages > 2 with progressive symptoms
Hypercalcemia
Eye-heart or CNS involvement

Page - 1998
Internal Medicine - Pulmonology

Question 69/144

Question #69

A 34-year-old black female presents for routine checkups. Her physical examination shows enlarged,
non-tender supraclavicular lymph nodes. Her chest x-ray is shown below. She has no cough,
shortness of breath, night sweats, weight loss, or fever. e rest of the physical examination is
normal, her PPD test is negative, and her septic workup is negative. Her PFT and ECG are normal.
What is the next step in the management of this patient?

a. Periodic chest x-ray, PFT without any treatment


b. Hospital admission and start IVIG
c. Start INH and do sputum culture again
d. Start systemic steroids and monitor the lymph node size
e. Start methotrexate and order lymph node biopsy

‫اﻹﺟﺎﺑﺔ ﻋﲆ اﻟﺼﻔﺤﺔ اﻟﺘﺎﻟﻴﺔ‬

Page - 1999
Internal Medicine - Pulmonology - Sarcoidosis

Question 69/144

Question #69

A 34-year-old black female presents for routine checkups. Her physical examination shows enlarged,
non-tender supraclavicular lymph nodes. Her chest x-ray is shown below. She has no cough,
shortness of breath, night sweats, weight loss, or fever. e rest of the physical examination is
normal, her PPD test is negative, and her septic workup is negative. Her PFT and ECG are normal.
What is the next step in the management of this patient?

a. Periodic chest x-ray, PFT without any treatment √


b. Hospital admission and start IVIG
c. Start INH and do sputum culture again
d. Start systemic steroids and monitor the lymph node size
e. Start methotrexate and order lymph node biopsy

Description

is is an asymptomatic patient with sarcoidosis. It is stage 1, which is usually spontaneously


resolved and has no indication of steroid use or hospital admission

e chest x-ray shows (Garland’s triad) Bilateral hilar lymphadenopathy and paratracheal
Page - 2000
lymphadenopathy (see the following image!).

Page - 2001
Internal Medicine - Pulmonology

Question 70/144

Question #70

A 32-year-old female with shortness of breath and cervical lymphadenopathy presents for
evaluation. She has painful erythematous nodules on her shins. Her chest x-ray shows bilateral hilar
lymphadenopathies without infiltration. Which stage of sarcoidosis does this patient have if she is
confirmed by biopsy to have the disease?

a. Stage 1
b. Stage 2
c. Stage 3
d. Stage 4
e. e information is not enough

‫اﻹﺟﺎﺑﺔ ﻋﲆ اﻟﺼﻔﺤﺔ اﻟﺘﺎﻟﻴﺔ‬

Page - 2002
Internal Medicine - Pulmonology - Sarcoidosis

Question 70/144

Question #70

A 32-year-old female with shortness of breath and cervical lymphadenopathy presents for
evaluation. She has painful erythematous nodules on her shins. Her chest x-ray shows bilateral hilar
lymphadenopathies without infiltration. Which stage of sarcoidosis does this patient have if she is
confirmed by biopsy to have the disease?

a. Stage 1 √
b. Stage 2
c. Stage 3
d. Stage 4
e. e information is not enough

Description

is is a typical scenario of sarcoidosis, and Her chest x-ray shows bilateral hilar lymphadenopathies
without infiltration. erefore, her Sarcoidosis stage is 1.

e staging system depends on the chest x-ray findings as the following:

Stage 1 = Bilateral Hilar Lymphadenopathy (BHL)


Stage 2 = BHL + interstitial infiltrates
Stage 3 = diuse interstitial infiltrates only
Stage 4 = diuse fibrosis

Sarcoidosis:

It is a multi-system disease characterized by non-caseating granuloma


It has an unknown etiology but usually occurs in young and black patients, and it is more
common in non-smokers.
e clinical presentation consists of shortness of breath, erythema nodosum, bilateral hilar
lymphadenopathy on chest x-ray, hypercalcemia, and neurological manifestations
Lofgren’s syndrome is an acute sarcoidosis characterized by bilateral hilar lymphadenopathy
(BHL), erythema nodosum, fever, and polyarthralgia. It usually carries an excellent prognosis
Heerfordt’s syndrome (uveoparotid fever), there is parotid enlargement, fever, uveitis, and
facial palsy secondary to sarcoidosis
Page - 2003
e most accurate test is a biopsy that shows non-caseating granuloma

Page - 2004
Internal Medicine - Pulmonology

Question 71/144

Question #71

A 35-year-old female is complaining of breathlessness. Her chest x-ray shows bilateral hilar
lymphadenopathy. In addition, there are red tender lesions on her shins. What is the most likely
diagnosis?

a. Asthma
b. Cystic fibrosis
c. Sarcoidosis
d. Silicosis
e. Bronchiectasis

‫اﻹﺟﺎﺑﺔ ﻋﲆ اﻟﺼﻔﺤﺔ اﻟﺘﺎﻟﻴﺔ‬

Page - 2005
Internal Medicine - Pulmonology - Sarcoidosis

Question 71/144

Question #71

A 35-year-old female is complaining of breathlessness. Her chest x-ray shows bilateral hilar
lymphadenopathy. In addition, there are red tender lesions on her shins. What is the most likely
diagnosis?

a. Asthma
b. Cystic fibrosis
c. Sarcoidosis √
d. Silicosis
e. Bronchiectasis

Description

In any questions in the exam, if shin lesions and bilateral hilar lymphadenopathy are present, you
should think about sarcoidosis. is lesion is most likely to be erythema nodosum

Sarcoidosis:

It is a multi-system disease characterized by non-caseating granuloma


It has an unknown etiology but usually occurs in young and black patients, and it is more
common in non-smokers.
e clinical presentation consists of shortness of breath, erythema nodosum, bilateral hilar
lymphadenopathy on chest x-ray, hypercalcemia, and neurological manifestations
Lofgren’s syndrome is an acute sarcoidosis characterized by bilateral hilar lymphadenopathy
(BHL), erythema nodosum, fever, and polyarthralgia. It usually carries an excellent prognosis
Heerfordt’s syndrome (uveoparotid fever), there is parotid enlargement, fever, uveitis, and
facial palsy secondary to sarcoidosis
e most accurate test is a biopsy that shows non-caseating granuloma
e staging system depends on the chest x-ray findings

Page - 2006
Internal Medicine - Pulmonology

Question 72/144

Question #72

A 25-year-old male has had dyspnea, cough, and fatigue for 3 months. On examination, a bilateral
facial nerve palsy is present, and his chest x-ray shows bilateral hilar lymphadenopathy. What is the
most likely diagnosis?

a. Lymphoma
b. Sarcoidosis
c. Polyarteritis nodosa
d. Benign pulmonary nodules
e. Extrinsic allergic alveolitis

‫اﻹﺟﺎﺑﺔ ﻋﲆ اﻟﺼﻔﺤﺔ اﻟﺘﺎﻟﻴﺔ‬

Page - 2007
Internal Medicine - Pulmonology - Sarcoidosis

Question 72/144

Question #72

A 25-year-old male has had dyspnea, cough, and fatigue for 3 months. On examination, a bilateral
facial nerve palsy is present, and his chest x-ray shows bilateral hilar lymphadenopathy. What is the
most likely diagnosis?

a. Lymphoma
b. Sarcoidosis √
c. Polyarteritis nodosa
d. Benign pulmonary nodules
e. Extrinsic allergic alveolitis

Description

is is a case of sarcoidosis with neurological involvement; steroids are indicated for the treatment

Sarcoidosis:

It is a multi-system disease characterized by non-caseating granuloma


It has an unknown etiology but usually occurs in young and black patients, and it is more
common in non-smokers.
e clinical presentation consists of shortness of breath, erythema nodosum, bilateral hilar
lymphadenopathy on chest x-ray, hypercalcemia, and neurological manifestations
Lofgren’s syndrome is an acute sarcoidosis characterized by bilateral hilar lymphadenopathy
(BHL), erythema nodosum, fever, and polyarthralgia. It usually carries an excellent prognosis
Heerfordt’s syndrome (uveoparotid fever), there is parotid enlargement, fever, uveitis, and
facial palsy secondary to sarcoidosis
e most accurate test is a biopsy that shows non-caseating granuloma
e staging system depends on the chest x-ray findings

Page - 2008
Internal Medicine - Pulmonology

Question 73/144

Question #73

A 52-year-old man presents with shortness of breath, fatigue, and progressive dry cough. His chest
x-ray shows multiple coin-like lesions on the upper lobe of the right lung. He recently retired from a
cement manufacturing company in Al-Zarqa’a, Jordan. His Tuberculosis test is negative. What is the
most likely diagnosis?

a. Sarcoidosis
b. Silicosis
c. Tuberculosis
d. Aspergillosis
e. Atypical mycobacteria

‫اﻹﺟﺎﺑﺔ ﻋﲆ اﻟﺼﻔﺤﺔ اﻟﺘﺎﻟﻴﺔ‬

Page - 2009
Internal Medicine - Pulmonology - Silicosis

Question 73/144

Question #73

A 52-year-old man presents with shortness of breath, fatigue, and progressive dry cough. His chest
x-ray shows multiple coin-like lesions on the upper lobe of the right lung. He recently retired from a
cement manufacturing company in Al-Zarqa’a, Jordan. His Tuberculosis test is negative. What is the
most likely diagnosis?

a. Sarcoidosis
b. Silicosis √
c. Tuberculosis
d. Aspergillosis
e. Atypical mycobacteria

Description

is patient has occupational exposure to cement, which is rich in silicon.

Silicosis due to inhalation of silica dust is usually a well-known case of upper zone fibrosis. Diagnosis
is by clinical history and chest x-ray, and no specific treatment is available

Page - 2010
Internal Medicine - Pulmonology

Question 74/144

Question #74

A 33-year-old male patient presents with fever, dyspnea, and hemoptysis; he was started on INH,
Rifampicin, and Ethambutol aer being diagnosed with TB. Aer 2 months, he developed a pins and
needles sensation in his hands and feet, and neurological examination is otherwise normal. What is
the best next step in management?

a. Start vitamin B12 supplementation


b. Start gabapentin orally
c. Start pyridoxine supplementation
d. Start vitamin D supplementation
e. Start thiamin (B1) supplementation

‫اﻹﺟﺎﺑﺔ ﻋﲆ اﻟﺼﻔﺤﺔ اﻟﺘﺎﻟﻴﺔ‬

Page - 2011
Internal Medicine - Pulmonology - Tuberculosis (TB)

Question 74/144

Question #74

A 33-year-old male patient presents with fever, dyspnea, and hemoptysis; he was started on INH,
Rifampicin, and Ethambutol aer being diagnosed with TB. Aer 2 months, he developed a pins and
needles sensation in his hands and feet, and neurological examination is otherwise normal. What is
the best next step in management?

a. Start vitamin B12 supplementation


b. Start gabapentin orally
c. Start pyridoxine supplementation √
d. Start vitamin D supplementation
e. Start thiamin (B1) supplementation

Description

Isoniazid interferes with the metabolism of vitamin B6, leading to neuropathy. is condition may
result in coma and seizures and is reversed by vitamin B6 (pyridoxine)

Page - 2012
Internal Medicine - Pulmonology

Question 75/144

Question #75

A 55-year-old male previously worked at a silica mine, and he is known to have silicosis. He presents
today with 6 months history of cough and weight loss; his chest x-ray shows upper zone infiltration
and fibrosis, Sputum culture is sent, and results are pending. What is the most likely diagnosis?

a. COPD
b. Mesothelioma
c. Pneumonia
d. Rapidly progressive silicosis
e. Tuberculosis

‫اﻹﺟﺎﺑﺔ ﻋﲆ اﻟﺼﻔﺤﺔ اﻟﺘﺎﻟﻴﺔ‬

Page - 2013
Internal Medicine - Pulmonology - Tuberculosis (TB)

Question 75/144

Question #75

A 55-year-old male previously worked at a silica mine, and he is known to have silicosis. He presents
today with 6 months history of cough and weight loss; his chest x-ray shows upper zone infiltration
and fibrosis, Sputum culture is sent, and results are pending. What is the most likely diagnosis?

a. COPD
b. Mesothelioma
c. Pneumonia
d. Rapidly progressive silicosis
e. Tuberculosis √

Description

Since silica is toxic to macrophages, Tuberculosis is associated with silica exposure.

Tuberculosis (TB):

It is an infection caused by Mycobacterium Tuberculosis (an Acid-fast bacilli bacterium that


can cause caseating granuloma)
It can be latent in 95% and primary active disease in 5%, or secondary active disease (activation
of latent infection)
It can be pulmonary, extrapulmonary, or both.
Tuberculosis has an airborne mode of transmission. en, the bacteria invade the immunity,
survive and replicate in the macrophages
Clinical features are: pneumonia resistant to antibiotics, constitutional symptoms, upper zone
infiltration on the chest x-ray

Page - 2014
Internal Medicine - Pulmonology

Question 76/144

Question #76

e threshold for diagnosing a patient with Tuberculosis aer PPD screening in an HIV patient is an
induration of more than which of the following?

a. 3 mm
b. 5 mm
c. 7 mm
d. 10 mm
e. 15 mm

‫اﻹﺟﺎﺑﺔ ﻋﲆ اﻟﺼﻔﺤﺔ اﻟﺘﺎﻟﻴﺔ‬

Page - 2015
Internal Medicine - Pulmonology - Tuberculosis (TB)

Question 76/144

Question #76

e threshold for diagnosing a patient with Tuberculosis aer PPD screening in an HIV patient is an
induration of more than which of the following?

a. 3 mm
b. 5 mm √
c. 7 mm
d. 10 mm
e. 15 mm

Description

Induration ≥ 5 mm induration in an HIV patient is considered a positive PPD test

e following table demonstrates the Mantoux test interpretations:

Mantoux test is considered positive in the following conditions:

Induration ≥ 15 mm induration (for any patient)


Induration ≥ 10 mm induration and one of the following:
A recent arrival from a high prevalent country
IV drug user
Child < 4 years
Induration ≥ 5 mm induration and one of the following:
HIV positive
Recent contact with TB patient
Typical TB changes on chest x-ray
Organ transplant

Page - 2016
Internal Medicine - Pulmonology

Question 77/144

Question #77

A 22-year-old male has had recent contact with a TB patient; his tuberculin test was negative 6
months ago but now shows a 12 mm induration. Which of the following is essential to check before
starting latent TB treatment with INH?

a. Vitamin B6 level
b. Presence of neuropathy
c. Active Tuberculosis
d. History of Hepatitis A
e. Diabetes mellitus

‫اﻹﺟﺎﺑﺔ ﻋﲆ اﻟﺼﻔﺤﺔ اﻟﺘﺎﻟﻴﺔ‬

Page - 2017
Internal Medicine - Pulmonology - Tuberculosis (TB)

Question 77/144

Question #77

A 22-year-old male has had recent contact with a TB patient; his tuberculin test was negative 6
months ago but now shows a 12 mm induration. Which of the following is essential to check before
starting latent TB treatment with INH?

a. Vitamin B6 level
b. Presence of neuropathy
c. Active Tuberculosis √
d. History of Hepatitis A
e. Diabetes mellitus

Description

Starting INH alone in latent TB for 9 months is eective, but it carries a high risk of drug resistance if
started alone in active TB patients. A chest x-ray should be done to exclude active Tuberculosis
should be done first before starting INH for latent TB

Page - 2018
Internal Medicine - Pulmonology

Question 78/144

Question #78

A 34-year-old male patient was diagnosed with TB and is now on treatment. Which of the following
drugs is the most active against mycobacterium tuberculosis?

a. Isoniazid
b. Rifampicin
c. Ethambutol
d. Pyrazinamide
e. Streptomycin

‫اﻹﺟﺎﺑﺔ ﻋﲆ اﻟﺼﻔﺤﺔ اﻟﺘﺎﻟﻴﺔ‬

Page - 2019
Internal Medicine - Pulmonology - Tuberculosis (TB)

Question 78/144

Question #78

A 34-year-old male patient was diagnosed with TB and is now on treatment. Which of the following
drugs is the most active against mycobacterium tuberculosis?

a. Isoniazid √
b. Rifampicin
c. Ethambutol
d. Pyrazinamide
e. Streptomycin

Description

Isoniazid (INH) is the most active drug against Mycobacterium tuberculosis. It penetrates most body
fluids and accumulates in caseated lesions.

INH is used alone for 9 months in latent TB. In addition, it should be administrated with other anti-
TB drugs in active TB to avoid the development of drug resistance.

Page - 2020
Internal Medicine - Pulmonology

Question 79/144

Question #79

A 33-year-old male patient presents with fever, dyspnea, and hemoptysis; he was started on INH,
Rifampicin, and Ethambutol aer being diagnosed with TB. Aer 2 months, he developed a pins and
needles sensation in his hands and feet, and neurological examination is otherwise normal. What is
the most likely cause of his symptoms?

a. Isoniazid
b. Rifampicin
c. Ethambutol
d. B12 deficiency
e. Cerebrovascular accident

‫اﻹﺟﺎﺑﺔ ﻋﲆ اﻟﺼﻔﺤﺔ اﻟﺘﺎﻟﻴﺔ‬

Page - 2021
Internal Medicine - Pulmonology - Tuberculosis (TB)

Question 79/144

Question #79

A 33-year-old male patient presents with fever, dyspnea, and hemoptysis; he was started on INH,
Rifampicin, and Ethambutol aer being diagnosed with TB. Aer 2 months, he developed a pins and
needles sensation in his hands and feet, and neurological examination is otherwise normal. What is
the most likely cause of his symptoms?

a. Isoniazid √
b. Rifampicin
c. Ethambutol
d. B12 deficiency
e. Cerebrovascular accident

Description

Isoniazid interferes with the metabolism of vitamin B6, leading to neuropathy. is condition may
result in coma and seizures and is reversed by vitamin B6 (pyridoxine).

e following table demonstrates the adverse eects of anti-TB medications and their treatments:

Page - 2022
Internal Medicine - Pulmonology

Question 80/144

Question #80

A 32-year-old male who returns from south Africa presents with several months of hemoptysis and
weight loss. Physical examination, including vital signs, chest, heart, and abdomen, are all
unremarkable. His chest x-ray shows right upper zone cavitation, the tuberculin test is positive, the
sputum smear is negative for AFB, and cultures are pending. Which is the best initial treatment for
his condition?

a. Wait for culture results, then start treatment


b. INH for 9 months duration
c. Rifampicin and pyrazinamide for 9 months
d. Rifampicin for 6 weeks
e. INH, Rifampicin, Ethambutol, and pyrazinamide for 6 months

‫اﻹﺟﺎﺑﺔ ﻋﲆ اﻟﺼﻔﺤﺔ اﻟﺘﺎﻟﻴﺔ‬

Page - 2023
Internal Medicine - Pulmonology - Tuberculosis (TB)

Question 80/144

Question #80

A 32-year-old male who returns from south Africa presents with several months of hemoptysis and
weight loss. Physical examination, including vital signs, chest, heart, and abdomen, are all
unremarkable. His chest x-ray shows right upper zone cavitation, the tuberculin test is positive, the
sputum smear is negative for AFB, and cultures are pending. Which is the best initial treatment for
his condition?

a. Wait for culture results, then start treatment


b. INH for 9 months duration
c. Rifampicin and pyrazinamide for 9 months
d. Rifampicin for 6 weeks
e. INH, Rifampicin, Ethambutol, and pyrazinamide for 6 months √

Description

South Africa is known to have a high prevalence of Tuberculosis. Treatment shouldn’t be delayed for
those with high clinical suspicion, as in this case.

INH alone has a high risk of resistance, so 4 drugs should be started for 2 months, and then 2 drugs
for 4 months are the best regimen.

Page - 2024
Internal Medicine - Pulmonology

Question 81/144

Question #81

An alcoholic 49-year-old male patient presents with general weakness, weight loss, and cough. His
temperature is 39 °C, hepatomegaly is noted, but his alpha-fetoprotein is normal. e chest x-ray is
shown below. What is the most likely diagnosis?

a. Reactivation pulmonary Tuberculosis


b. Lobar pneumonia
c. Bronchiectasis
d. Miliary TB
e. Hepatic cell carcinoma

‫اﻹﺟﺎﺑﺔ ﻋﲆ اﻟﺼﻔﺤﺔ اﻟﺘﺎﻟﻴﺔ‬

Page - 2025
Internal Medicine - Pulmonology - Tuberculosis (TB)

Question 81/144

Question #81

An alcoholic 49-year-old male patient presents with general weakness, weight loss, and cough. His
temperature is 39 °C, hepatomegaly is noted, but his alpha-fetoprotein is normal. e chest x-ray is
shown below. What is the most likely diagnosis?

a. Reactivation pulmonary Tuberculosis


b. Lobar pneumonia
c. Bronchiectasis
d. Miliary TB √
e. Hepatic cell carcinoma

Description

e chest x-ray shows diuse, well-defined nodules less than 5 mm in diameter, characteristic of
Miliary Tuberculosis. e negative alpha-fetoprotein excludes hepatic cancer. Page - 2026
Features of Miliary TB:

Widespread hematogenous dissemination of TB to the lung, abdominal organs, CNS, etc.


e lesions are usually yellowish granulomas of 1 – 2 mm in diameter
Fever, night sweats, anorexia, weakness, and weight loss are presenting symptoms in the
majority of cases

Page - 2027
Internal Medicine - Pulmonology

Question 82/144

Question #82

A 15-year-old male has presented with recurrent shortness of breath for the last 6 months. His
dyspnea occurs more at night. His little brother has a history of eczema. Which of the following is
most likely to present in the Pulmonary function test?

a. Normal FEV1, high FEV1/FVC ratio, High TLC


b. Low FEV1, Low FEV1/FVC ratio, Low TLC
c. Low FEV1, Normal FEV1/FVC ratio, High TLC
d. Low FEV1, Low FEV1/FVC ratio, High TLC
e. Normal FEV1, Normal FEV1/FVC ratio, Normal TLC

‫اﻹﺟﺎﺑﺔ ﻋﲆ اﻟﺼﻔﺤﺔ اﻟﺘﺎﻟﻴﺔ‬

Page - 2028
Internal Medicine - Pulmonology - Asthma

Question 82/144

Question #82

A 15-year-old male has presented with recurrent shortness of breath for the last 6 months. His
dyspnea occurs more at night. His little brother has a history of eczema. Which of the following is
most likely to present in the Pulmonary function test?

a. Normal FEV1, high FEV1/FVC ratio, High TLC


b. Low FEV1, Low FEV1/FVC ratio, Low TLC
c. Low FEV1, Normal FEV1/FVC ratio, High TLC
d. Low FEV1, Low FEV1/FVC ratio, High TLC √
e. Normal FEV1, Normal FEV1/FVC ratio, Normal TLC

Description

is patient complains of recurrent shortness of breath, night symptoms, and a family history of an
IgE-related disease (eczema in her brother). erefore, asthma is the most likely diagnosis.

Asthma is known to manifest as an obstructive lung disease on the pulmonary function test

In pure obstructive lung disease, FEV1/FVC ratio will be low, and the TLC will be high.

Asthma:

Asthma is a chronic disease that causes inflammation with reversible & variable narrowing of
the airways (e main dierence between asthma and COPD is reversibility and variability)
Asthma is usually associated with a history or family history of other IgE-related diseases like
atopic dermatitis (eczema) and atopic rhinitis (hay fever)
Triggers are: dust, smoking, air pollution, Drugs – NSAID, Beta-blockers, exercise, emotions,
sudden changes in air temperature

Page - 2029
Internal Medicine - Pulmonology

Question 83/144

Question #83

A 30-year-old male patient presents with 6 months history of shortness of breath. His symptoms get
better at weekends but get worse on workdays. What is the most likely diagnosis?

a. Atopic asthma
b. Occupational asthma
c. Chronic bronchitis
d. Alpha 1 antitrypsin deficiency
e. Cough variant asthma

‫اﻹﺟﺎﺑﺔ ﻋﲆ اﻟﺼﻔﺤﺔ اﻟﺘﺎﻟﻴﺔ‬

Page - 2030
Internal Medicine - Pulmonology - Asthma

Question 83/144

Question #83

A 30-year-old male patient presents with 6 months history of shortness of breath. His symptoms get
better at weekends but get worse on workdays. What is the most likely diagnosis?

a. Atopic asthma
b. Occupational asthma √
c. Chronic bronchitis
d. Alpha 1 antitrypsin deficiency
e. Cough variant asthma

Description

is patient complains of variable asthmatic symptoms that are worse on workdays and better at
weekends. erefore, Occupational asthma is the most likely diagnosis.

To diagnose occupational asthma, perform pulmonary function tests on work days and weekends to
compare the results.

Note: in occupational asthma, chemicals at work worsen symptoms, and the patient is better at
weekends.

Page - 2031
Internal Medicine - Pulmonology

Question 84/144

Question #84

A 22-year-old male is a known case of asthma. He presents with acute asthma exacerbation for 2
hours. He is not improving despite the use of SABA and SAMA. On examination, the patient is using
accessory muscles in respiration. His blood pressure is 100/60. Chest auscultation shows loud
expiratory wheezing, ABGs show PH 7.45, PaCO2 39 mmHg, and HCO3 was 26. What is the next step
in the management of this patient?

a. Admission to the hospital and provide Oxygen by mask


b. Inhaled high dose beclomethasone
c. Repeat O2, SABA, SAMA, and prednisolone
d. Intubation, 100% oxygen, and IV steroids
e. Reassurance

‫اﻹﺟﺎﺑﺔ ﻋﲆ اﻟﺼﻔﺤﺔ اﻟﺘﺎﻟﻴﺔ‬

Page - 2032
Internal Medicine - Pulmonology - Asthma

Question 84/144

Question #84

A 22-year-old male is a known case of asthma. He presents with acute asthma exacerbation for 2
hours. He is not improving despite the use of SABA and SAMA. On examination, the patient is using
accessory muscles in respiration. His blood pressure is 100/60. Chest auscultation shows loud
expiratory wheezing, ABGs show PH 7.45, PaCO2 39 mmHg, and HCO3 was 26. What is the next step
in the management of this patient?

a. Admission to the hospital and provide Oxygen by mask


b. Inhaled high dose beclomethasone
c. Repeat O2, SABA, SAMA, and prednisolone
d. Intubation, 100% oxygen, and IV steroids √
e. Reassurance

Description

Normal PaCO2 in acute asthma exacerbation is a sign of life-threatening asthma. erefore,


intubation is warranted at this point.

Based on PEFR, ABGs, and clinical features, asthma exacerbations are classified as moderate, severe,
and life-threatening.

e following table demonstrates the features of acute asthma classes:

Page - 2033
Internal Medicine - Pulmonology

Question 85/144

Question #85

A 15-year-old male has presented with recurrent shortness of breath for the last 6 months. His
dyspnea occurs more at night. In addition, his little brother has a history of eczema. Which of the
following is the best to establish a diagnosis of his condition?

a. Chest X-ray
b. Chest CT scan
c. Symptomatic control on bronchodilators
d. Bronchoscopy
e. DLco

‫اﻹﺟﺎﺑﺔ ﻋﲆ اﻟﺼﻔﺤﺔ اﻟﺘﺎﻟﻴﺔ‬

Page - 2034
Internal Medicine - Pulmonology - Asthma

Question 85/144

Question #85

A 15-year-old male has presented with recurrent shortness of breath for the last 6 months. His
dyspnea occurs more at night. In addition, his little brother has a history of eczema. Which of the
following is the best to establish a diagnosis of his condition?

a. Chest X-ray
b. Chest CT scan
c. Symptomatic control on bronchodilators √
d. Bronchoscopy
e. DLco

Description

is patient complains of recurrent shortness of breath, night symptoms, and a family history of an
IgE-related disease (eczema in her brother). erefore, asthma is the most likely diagnosis.

An improvement of the dyspnea aer administration of SABA is indicative of reversibility and is


diagnostic of asthma in this patient.

Asthma:

Asthma is a chronic disease that causes inflammation with reversible & variable narrowing of
the airways (e main dierence between asthma and COPD is reversibility and variability)
Asthma is usually associated with a history or family history of other IgE-related diseases like
atopic dermatitis (eczema) and atopic rhinitis (hay fever)
Triggers are: dust, smoking, air pollution, Drugs – NSAID, Beta-blockers, exercise, emotions,
sudden changes in air temperature

Page - 2035
Internal Medicine - Pulmonology

Question 86/144

Question #86

A 24-year-old male patient presents with acute asthma exacerbation. He has received nebulized
SABA, 200 mg hydrocortisone IV, and 100% oxygen for the last 20 minutes. Unfortunately, he is still
symptomatic. His chest is wheezy, and his oxygen saturation is 90%. What is the best next step in
the management of this patient?

a. Prednisolone 40 mg PO
b. Nebulized Ipratropium bromide 0.5 mg
c. Intramuscular adrenalin
d. Stop oxygen administration
e. Magnesium sulfate intravenously

‫اﻹﺟﺎﺑﺔ ﻋﲆ اﻟﺼﻔﺤﺔ اﻟﺘﺎﻟﻴﺔ‬

Page - 2036
Internal Medicine - Pulmonology - Asthma

Question 86/144

Question #86

A 24-year-old male patient presents with acute asthma exacerbation. He has received nebulized
SABA, 200 mg hydrocortisone IV, and 100% oxygen for the last 20 minutes. Unfortunately, he is still
symptomatic. His chest is wheezy, and his oxygen saturation is 90%. What is the best next step in
the management of this patient?

a. Prednisolone 40 mg PO
b. Nebulized Ipratropium bromide 0.5 mg √
c. Intramuscular adrenalin
d. Stop oxygen administration
e. Magnesium sulfate intravenously

Description

Ipratropium bromide is a short-acting muscarinic antagonist used as an asthma reliever for patients
who do not respond to SAMA and early steroid administration.

Treatment of asthma exacerbation:

e first step:
Oxygen: For all hypoxic patients, given by venturi mask
SABA: 5 – 10 pus by MDI and spacer (SABA nebulizer is indicated in severe cases)- can be
repeated up to 3 doses in 1 hour.
Early administration of systemic steroid (40 – 50 mg oral prednisolone)
e second step: Nebulized SAMA (Short-acting muscarinic antagonists, e.g., Ipratropium)
e third step: Magnesium sulfate IV (1.2 – 2 g over 30 minutes)
Not routinely used
Only if the previous measures fail to improve the patient
Intubation and mechanical ventilation should be done in any patient with life-threatening or
near-fatal asthma.

Page - 2037
Internal Medicine - Pulmonology

Question 87/144

Question #87

An 18-year-old male patient with asthma developed sudden chest pain and shortness of breath
while running for a bus. His breath sounds are reduced on the right side. His oxygen saturation is
95% on room air. What is the single most appropriate investigation at this moment?

a. ECG
b. Chest x-ray
c. Chest CT scan
d. D Dimer
e. Arterial blood gas

‫اﻹﺟﺎﺑﺔ ﻋﲆ اﻟﺼﻔﺤﺔ اﻟﺘﺎﻟﻴﺔ‬

Page - 2038
Internal Medicine - Pulmonology - Asthma

Question 87/144

Question #87

An 18-year-old male patient with asthma developed sudden chest pain and shortness of breath
while running for a bus. His breath sounds are reduced on the right side. His oxygen saturation is
95% on room air. What is the single most appropriate investigation at this moment?

a. ECG
b. Chest x-ray √
c. Chest CT scan
d. D Dimer
e. Arterial blood gas

Description

is patient is most likely to have acute asthma exacerbation secondary to pneumothorax.

Primary spontaneous pneumothorax here in this patient is likely because of the presence of
unilaterally reduced breathing sound and the history of running

A standard erect chest x-ray is recommended as an initial diagnostic investigation for this patient.

Page - 2039
Internal Medicine - Pulmonology

Question 88/144

Question #88

A 22-year-old male patient presents with acute asthma exacerbation. He is treated with 100%
oxygen, nebulized salbutamol, oral prednisolone, and nebulized ipratropium bromide.
Unfortunately, the patient is still symptomatic and tachypneic. What is the best next step in the
management of this patient?

a. Intravenous aminophylline
b. Intravenous MgSO4
c. Intravenous salbutamol
d. Intravenous adrenalin
e. Intramuscular adrenalin

‫اﻹﺟﺎﺑﺔ ﻋﲆ اﻟﺼﻔﺤﺔ اﻟﺘﺎﻟﻴﺔ‬

Page - 2040
Internal Medicine - Pulmonology - Asthma

Question 88/144

Question #88

A 22-year-old male patient presents with acute asthma exacerbation. He is treated with 100%
oxygen, nebulized salbutamol, oral prednisolone, and nebulized ipratropium bromide.
Unfortunately, the patient is still symptomatic and tachypneic. What is the best next step in the
management of this patient?

a. Intravenous aminophylline
b. Intravenous MgSO4 √
c. Intravenous salbutamol
d. Intravenous adrenalin
e. Intramuscular adrenalin

Description

is patient with acute asthma exacerbation didn’t respond to SABA, steroids, and SAMA. erefore,
you should provide Magnesium sulfate intravenously at this step.

Treatment of asthma exacerbation:

e first step:
Oxygen: For all hypoxic patients, given by venturi mask
SABA: 5 – 10 pus by MDI and spacer (SABA nebulizer is indicated in severe cases)- can be
repeated up to 3 doses in 1 hour.
Early administration of systemic steroid (40 – 50 mg oral prednisolone)
e second step: Nebulized SAMA (Short-acting muscarinic antagonists, e.g., Ipratropium)
e third step: Magnesium sulfate IV (1.2 – 2 g over 30 minutes)
Not routinely used
Only if the previous measures fail to improve the patient
Intubation and mechanical ventilation should be done in any patient with life-threatening or
near-fatal asthma.

Page - 2041
Internal Medicine - Pulmonology

Question 89/144

Question #89

A 33-year-old female with a long history of asthma uses a salbutamol inhaler as needed. She needs it
3 times per week in the daytime and 3 times monthly at night as the symptoms awaken her from
sleep. What would you do next?

a. Add LABA
b. Add LTRA
c. Add low-dose ICS
d. Continue same treatment
e. Increase the dose of salbutamol

‫اﻹﺟﺎﺑﺔ ﻋﲆ اﻟﺼﻔﺤﺔ اﻟﺘﺎﻟﻴﺔ‬

Page - 2042
Internal Medicine - Pulmonology - Asthma

Question 89/144

Question #89

A 33-year-old female with a long history of asthma uses a salbutamol inhaler as needed. She needs it
3 times per week in the daytime and 3 times monthly at night as the symptoms awaken her from
sleep. What would you do next?

a. Add LABA
b. Add LTRA
c. Add low-dose ICS √
d. Continue same treatment
e. Increase the dose of salbutamol

Description

e patient is still uncontrolled because she uses her inhaler more than 2 times weekly.

You should add a low-dose ICS to her regimen to control her asthma.

e following diagram demonstrates the recent guidelines in asthma management:

Page - 2043
Internal Medicine - Pulmonology

Question 90/144

Question #90

A 25-year-old male patient known case of asthma complains of daily symptoms of shortness of
breath and wheezing. He occasionally wakes up from sleeping due to chest tightness and dyspnea.
He is on a salbutamol inhaler as required. e next step in the management of this patient is:

a. Increase the dose of salbutamol


b. Give oral prednisolone
c. Add ipratropium bromide in two pus 4 times a day
d. Add low dose beclomethasone two pus BID
e. Stop salbutamol and start long-acting beta-agonist

‫اﻹﺟﺎﺑﺔ ﻋﲆ اﻟﺼﻔﺤﺔ اﻟﺘﺎﻟﻴﺔ‬

Page - 2044
Internal Medicine - Pulmonology - Asthma

Question 90/144

Question #90

A 25-year-old male patient known case of asthma complains of daily symptoms of shortness of
breath and wheezing. He occasionally wakes up from sleeping due to chest tightness and dyspnea.
He is on a salbutamol inhaler as required. e next step in the management of this patient is:

a. Increase the dose of salbutamol


b. Give oral prednisolone
c. Add ipratropium bromide in two pus 4 times a day
d. Add low dose beclomethasone two pus BID √
e. Stop salbutamol and start long-acting beta-agonist

Description

is patent is in the first step of his treatment, and he is not responsive. erefore, a step up of his
treatment by adding a low dose of ICS is recommended.

You should add a low-dose inhaled steroid (fluticasone, beclomethasone) to the patient’s treatment.

e following diagram demonstrates the recent guidelines in asthma management:

Page - 2045
Internal Medicine - Pulmonology

Question 91/144

Question #91

A 29-year-old dentist presents to you with coughs, wheezing, and shortness of breath. He exercises
regularly and experiences his symptoms about 5 minutes aer starting his exercises. His physical
examination and lab investigations are all normal. What is the most likely diagnosis?

a. Occupational asthma
b. Atopic asthma
c. Drug-induced asthma
d. Exercise-induced asthma
e. Cough variant asthma

‫اﻹﺟﺎﺑﺔ ﻋﲆ اﻟﺼﻔﺤﺔ اﻟﺘﺎﻟﻴﺔ‬

Page - 2046
Internal Medicine - Pulmonology - Asthma

Question 91/144

Question #91

A 29-year-old dentist presents to you with coughs, wheezing, and shortness of breath. He exercises
regularly and experiences his symptoms about 5 minutes aer starting his exercises. His physical
examination and lab investigations are all normal. What is the most likely diagnosis?

a. Occupational asthma
b. Atopic asthma
c. Drug-induced asthma
d. Exercise-induced asthma √
e. Cough variant asthma

Description

is patient has bronchospasms that are induced by exercise. Bronchodilators should be provided as
needed for this patient

Page - 2047
Internal Medicine - Pulmonology

Question 92/144

Question #92

A 30-year-old male patient presents with a 6-month history of shortness of breath. His symptoms
get better at weekends but get worse on workdays. What is the best tool to establish the most likely
diagnosis?

a. Chest X-ray
b. Lung volume study
c. DLco
d. Spirometry at workdays and weekends
e. Methacholine challenge test

‫اﻹﺟﺎﺑﺔ ﻋﲆ اﻟﺼﻔﺤﺔ اﻟﺘﺎﻟﻴﺔ‬

Page - 2048
Internal Medicine - Pulmonology - Asthma

Question 92/144

Question #92

A 30-year-old male patient presents with a 6-month history of shortness of breath. His symptoms
get better at weekends but get worse on workdays. What is the best tool to establish the most likely
diagnosis?

a. Chest X-ray
b. Lung volume study
c. DLco
d. Spirometry at workdays and weekends √
e. Methacholine challenge test

Description

is patient complains of variable asthmatic symptoms that are worse on workdays and better at
weekends. erefore, Occupational asthma is the most likely diagnosis.

To diagnose occupational asthma, perform pulmonary function tests on work days and weekends to
compare the results.

Note: in occupational asthma, chemicals at work worsen symptoms, and the patient is better at
weekends.

Note: Spirometry showing obstructive pattern at workdays which improves at weekends, is


suggestive of occupational asthma

Page - 2049
Internal Medicine - Pulmonology

Question 93/144

Question #93

A 22-year-old female patient suers from wheezing and chest tightness, which is more frequent at
night and when exposed to cold weather. What is the most likely diagnosis?

a. COPD
b. Pneumonia
c. Asthma
d. Bronchiectasis
e. Interstitial lung disease

‫اﻹﺟﺎﺑﺔ ﻋﲆ اﻟﺼﻔﺤﺔ اﻟﺘﺎﻟﻴﺔ‬

Page - 2050
Internal Medicine - Pulmonology - Asthma

Question 93/144

Question #93

A 22-year-old female patient suers from wheezing and chest tightness, which is more frequent at
night and when exposed to cold weather. What is the most likely diagnosis?

a. COPD
b. Pneumonia
c. Asthma √
d. Bronchiectasis
e. Interstitial lung disease

Description

is patient is suering from symptoms of asthma aer she exposed to a precipitant

Asthma is a chronic disease that causes inflammation with reversible & variable narrowing of the
airways (e main dierence between asthma and COPD is reversibility and variability)

Asthma:

Asthma is a chronic disease that causes inflammation with reversible & variable narrowing of
the airways (e main dierence between asthma and COPD is reversibility and variability)
Asthma is usually associated with a history or family history of other IgE-related diseases like
atopic dermatitis (eczema) and atopic rhinitis (hay fever)
Triggers are: dust, smoking, air pollution, Drugs – NSAID, Beta-blockers, exercise, emotions,
sudden changes in air temperature

Page - 2051
Internal Medicine - Pulmonology

Question 94/144

Question #94

What is the best test to dierentiate between asthma and COPD in a patient with recurrent
shortness of breath?

a. Methacholine challenge test


b. Bronchodilation challenge test
c. 6-Minute walk test
d. DLco
e. Chest x-ray

‫اﻹﺟﺎﺑﺔ ﻋﲆ اﻟﺼﻔﺤﺔ اﻟﺘﺎﻟﻴﺔ‬

Page - 2052
Internal Medicine - Pulmonology - Asthma

Question 94/144

Question #94

What is the best test to dierentiate between asthma and COPD in a patient with recurrent
shortness of breath?

a. Methacholine challenge test


b. Bronchodilation challenge test √
c. 6-Minute walk test
d. DLco
e. Chest x-ray

Description

Asthma:

Asthma is a chronic disease that causes inflammation with reversible & variable narrowing of the
airways (e main dierence between asthma and COPD is reversibility and variability)

Reversibility of more than 12% and 200 mL in FEV1 aer the administration of SABA is the main test
to identify reversibility.

Page - 2053
Internal Medicine - Pulmonology

Question 95/144

Question #95

A 25-year-old male patient with acute asthma exacerbation is treated at the ER initially with 100%
oxygen and salbutamol nebulizers. e patient’s respiratory rate is 21 per minute, O2 saturation is
93%, and his heart rate is 90 bpm. What would you do next in the management of this patient?

a. No additional treatment
b. Administrate oral prednisolone 40 mg
c. Administrate MgSO4 1.5 grams intravenously
d. Intubation and mechanical ventilation
e. Intramuscular adrenalin

‫اﻹﺟﺎﺑﺔ ﻋﲆ اﻟﺼﻔﺤﺔ اﻟﺘﺎﻟﻴﺔ‬

Page - 2054
Internal Medicine - Pulmonology - Asthma

Question 95/144

Question #95

A 25-year-old male patient with acute asthma exacerbation is treated at the ER initially with 100%
oxygen and salbutamol nebulizers. e patient’s respiratory rate is 21 per minute, O2 saturation is
93%, and his heart rate is 90 bpm. What would you do next in the management of this patient?

a. No additional treatment
b. Administrate oral prednisolone 40 mg √
c. Administrate MgSO4 1.5 grams intravenously
d. Intubation and mechanical ventilation
e. Intramuscular adrenalin

Description

Here the patient has mild acute asthma exacerbation. However, early administration of steroids is
vital in acute asthma exacerbation

Treatment of asthma exacerbation:

e first step:
Oxygen: For all hypoxic patients, given by venturi mask
SABA: 5 – 10 pus by MDI and spacer (SABA nebulizer is indicated in severe cases)- can be
repeated up to 3 doses in 1 hour.
Early administration of systemic steroid (40 – 50 mg oral prednisolone)
e second step: Nebulized SAMA (Short-acting muscarinic antagonists, e.g., Ipratropium)
e third step: Magnesium sulfate IV (1.2 – 2 g over 30 minutes)
Not routinely used
Only if the previous measures fail to improve the patient
Intubation and mechanical ventilation should be done in any patient with life-threatening or
near-fatal asthma.

Page - 2055
Internal Medicine - Pulmonology

Question 96/144

Question #96

Compared to the regular nebulizer, the use of a metered-dose inhaler (MDI) in asthma attack
treatment in the emergency department will result in all of the following except:

a. Reduces the required salbutamol dose


b. Shorter duration in the ER
c. Better improvement in PEFR
d. Better improvement on ABGs
e. Increases the recurrence of exacerbations

‫اﻹﺟﺎﺑﺔ ﻋﲆ اﻟﺼﻔﺤﺔ اﻟﺘﺎﻟﻴﺔ‬

Page - 2056
Internal Medicine - Pulmonology - Asthma

Question 96/144

Question #96

Compared to the regular nebulizer, the use of a metered-dose inhaler (MDI) in asthma attack
treatment in the emergency department will result in all of the following except:

a. Reduces the required salbutamol dose


b. Shorter duration in the ER
c. Better improvement in PEFR
d. Better improvement on ABGs
e. Increases the recurrence of exacerbations √

Description

Compared to nebulizers, MDIs with spacers have been shown to reduce the recurrence of
exacerbations.

Treatment of asthma exacerbation:

e first step:
Oxygen: For all hypoxic patients, given by venturi mask
SABA: 5 – 10 pus by MDI and spacer (SABA nebulizer is indicated in severe cases)- can be
repeated up to 3 doses in 1 hour.
Early administration of systemic steroid (40 – 50 mg oral prednisolone)
e second step: Nebulized SAMA (Short-acting muscarinic antagonists, e.g., Ipratropium)
e third step: Magnesium sulfate IV (1.2 – 2 g over 30 minutes)
Not routinely used
Only if the previous measures fail to improve the patient
Intubation and mechanical ventilation should be done in any patient with life-threatening or
near-fatal asthma.

Page - 2057
Internal Medicine - Pulmonology

Question 97/144

Question #97

An asthmatic 21-year-old male patient presents to the ER with shortness of breath that is not
improving on his inhaled salbutamol. However, physical examination shows a dyspneic conscious,
and alert person. His vital signs are normal. What is the next step best to be done at this point?

a. Start inhaled corticosteroids


b. IV methylprednisolone
c. Oxygen with bag and mask ventilation
d. Start oral corticosteroids
e. Methacholine challenge test

‫اﻹﺟﺎﺑﺔ ﻋﲆ اﻟﺼﻔﺤﺔ اﻟﺘﺎﻟﻴﺔ‬

Page - 2058
Internal Medicine - Pulmonology - Asthma

Question 97/144

Question #97

An asthmatic 21-year-old male patient presents to the ER with shortness of breath that is not
improving on his inhaled salbutamol. However, physical examination shows a dyspneic conscious,
and alert person. His vital signs are normal. What is the next step best to be done at this point?

a. Start inhaled corticosteroids


b. IV methylprednisolone
c. Oxygen with bag and mask ventilation
d. Start oral corticosteroids √
e. Methacholine challenge test

Description

Early administration of steroids is vital in acute asthma exacerbation

Bag-mask ventilation is not indicated in stable patients.

IV steroids are equivalent to oral steroids in the treatment of asthma

Treatment of asthma exacerbation:

e first step:
Oxygen: For all hypoxic patients, given by venturi mask
SABA: 5 – 10 pus by MDI and spacer (SABA nebulizer is indicated in severe cases)- can be
repeated up to 3 doses in 1 hour.
Early administration of systemic steroid (40 – 50 mg oral prednisolone)
e second step: Nebulized SAMA (Short-acting muscarinic antagonists, e.g., Ipratropium)
e third step: Magnesium sulfate IV (1.2 – 2 g over 30 minutes)
Not routinely used
Only if the previous measures fail to improve the patient
Intubation and mechanical ventilation should be done in any patient with life-threatening or
near-fatal asthma.

Page - 2059
Internal Medicine - Pulmonology

Question 98/144

Question #98

A 19-year-old male patient is a known case of asthma. He presented with frequent night coughs and
exercise-induced shortness of breath. He is on inhaled salbutamol as needed and twice daily inhaled
corticosteroids. His inhaler technique is good. What would you do next in the management of this
patient?

a. Add Salmeterol inhaler


b. Add oral theophylline
c. Add nebulized albuterol
d. Add regular oral corticosteroids
e. Increase the dose of ICS

‫اﻹﺟﺎﺑﺔ ﻋﲆ اﻟﺼﻔﺤﺔ اﻟﺘﺎﻟﻴﺔ‬

Page - 2060
Internal Medicine - Pulmonology - Asthma

Question 98/144

Question #98

A 19-year-old male patient is a known case of asthma. He presented with frequent night coughs and
exercise-induced shortness of breath. He is on inhaled salbutamol as needed and twice daily inhaled
corticosteroids. His inhaler technique is good. What would you do next in the management of this
patient?

a. Add Salmeterol inhaler √


b. Add oral theophylline
c. Add nebulized albuterol
d. Add regular oral corticosteroids
e. Increase the dose of ICS

Description

is patient is taking SABA and Low dose ICS. erefore, the best next step is to step up his
treatment and start LABA (salmeterol) inhaler in his regimen.

e following diagram demonstrates the recent guidelines in asthma management:

Page - 2061
Internal Medicine - Pulmonology

Question 99/144

Question #99

A 21-year-old male patient with a history of eczema presents with fever, shortness of breath, and
wheezes. Pulmonary function test shows FEV1/FCV of 55%. is ratio became 72% aer the
administration of bronchodilators. What is the most likely diagnosis?

a. Asthma exacerbation
b. COPD
c. Tuberculosis
d. Bronchiectasis
e. Chronic bronchitis

‫اﻹﺟﺎﺑﺔ ﻋﲆ اﻟﺼﻔﺤﺔ اﻟﺘﺎﻟﻴﺔ‬

Page - 2062
Internal Medicine - Pulmonology - Asthma

Question 99/144

Question #99

A 21-year-old male patient with a history of eczema presents with fever, shortness of breath, and
wheezes. Pulmonary function test shows FEV1/FCV of 55%. is ratio became 72% aer the
administration of bronchodilators. What is the most likely diagnosis?

a. Asthma exacerbation √
b. COPD
c. Tuberculosis
d. Bronchiectasis
e. Chronic bronchitis

Description

e history of eczema, SOB, and Wheezes are typical asthma presentations.

e most likely diagnosis is an infective exacerbation of asthma (fever, SOB, and Wheezes).

e most accurate test to confirm asthma is to perform a Pulmonary function test that is expected
to show a reversible obstruction pattern in the asthma patients, i.e., reduced FEV1 and FEV1/FVC
ratio; this will improve by more than 200 ml and 12% aer the administration of SABA.

Asthma:

Asthma is a chronic disease that causes inflammation with reversible & variable narrowing of
the airways (e main dierence between asthma and COPD is reversibility and variability)
Asthma is usually associated with a history or family history of other IgE-related diseases like
atopic dermatitis (eczema) and atopic rhinitis (hay fever)
Triggers are: dust, smoking, air pollution, Drugs – NSAID, Beta-blockers, exercise, emotions,
sudden changes in air temperature

Page - 2063
Internal Medicine - Pulmonology

Question 100/144

Question #100

A 22-year-old male with mild persistent asthma is on inhaled corticosteroids. Which of the following
is true regarding this drug?

a. ey are used as needed (to relieve attack)


b. ey are ineective in a patient who takes beta-blockers
c. ey are less eective than SABA in controlling asthma
d. ey are acting by causing bronchodilatation
e. ey are eective within 5 minutes for 6 hours duration

‫اﻹﺟﺎﺑﺔ ﻋﲆ اﻟﺼﻔﺤﺔ اﻟﺘﺎﻟﻴﺔ‬

Page - 2064
Internal Medicine - Pulmonology - Asthma

Question 100/144

Question #100

A 22-year-old male with mild persistent asthma is on inhaled corticosteroids. Which of the following
is true regarding this drug?

a. ey are used as needed (to relieve attack)


b. ey are ineective in a patient who takes beta-blockers
c. ey are less eective than SABA in controlling asthma
d. ey are acting by causing bronchodilatation
e. ey are eective within 5 minutes for 6 hours duration √

Description

e eect of ICS starts within 5 minutes and lasts up to 4 – 6 hours.

It is used regularly, twice daily (not as needed)

SABA is not used to control asthma. It is used to relieve it

ICS acts by anti-inflammatory eect (Not a bronchodilator)

Page - 2065
Internal Medicine - Pulmonology

Question 101/144

Question #101

A 22-year-old male patient with a known case of asthma presents with palpitations and tachycardia.
His heart rate is 115 bpm and regular. His peak expiratory flow rate is normal. What is the most
appropriate management of this patient?

a. Lifestyle changes
b. Holter 24-hour monitoring
c. Admission to the CCU for more investigations
d. Do Chest x-ray
e. Review his medications

‫اﻹﺟﺎﺑﺔ ﻋﲆ اﻟﺼﻔﺤﺔ اﻟﺘﺎﻟﻴﺔ‬

Page - 2066
Internal Medicine - Pulmonology - Asthma

Question 101/144

Question #101

A 22-year-old male patient with a known case of asthma presents with palpitations and tachycardia.
His heart rate is 115 bpm and regular. His peak expiratory flow rate is normal. What is the most
appropriate management of this patient?

a. Lifestyle changes
b. Holter 24-hour monitoring
c. Admission to the CCU for more investigations
d. Do Chest x-ray
e. Review his medications √

Description

e patient most likely suers from a common side eect due to beta-agonist use.

SABA: Short-acting beta 2 agonists:

Used as reliever, not controller (cause bronchodilation)


Example: salbutamol, albuterol, levalbuterol
Side eects: hypokalemia, tachycardia, and tremor
SABA alone has more risk of exacerbation compared to SABA + ICS use. erefore, according to
the recent guidelines, ensure the patient will adhere to ICS therapy before prescribing SABA.

Page - 2067
Internal Medicine - Pulmonology

Question 102/144

Question #102

A patient with a known case of asthma who has acute shortness of breath daily and night-time
symptoms once weekly, which of the following asthma classes does he have?

a. Mild intermittent asthma


b. Mild persistent asthma
c. Moderate persistent asthma
d. Severe persistent asthma
e. Moderate intermittent asthma

‫اﻹﺟﺎﺑﺔ ﻋﲆ اﻟﺼﻔﺤﺔ اﻟﺘﺎﻟﻴﺔ‬

Page - 2068
Internal Medicine - Pulmonology - Asthma

Question 102/144

Question #102

A patient with a known case of asthma who has acute shortness of breath daily and night-time
symptoms once weekly, which of the following asthma classes does he have?

a. Mild intermittent asthma


b. Mild persistent asthma
c. Moderate persistent asthma √
d. Severe persistent asthma
e. Moderate intermittent asthma

Description

Daily daytime and weekly night-time symptoms of asthma are typical for moderate persistent
asthma

e following table shows the classification of asthma according to the frequency of symptoms and
the FEV1.

Page - 2069
Internal Medicine - Pulmonology

Question 103/144

Question #103

Which is the most appropriate first-line treatment for a 22-year-old male patient with acute asthma
exacerbation?

a. Oxygen
b. Salbutamol intravenously
c. Salbutamol nebulizer
d. Salmeterol nebulizer
e. Intravenous MgSO4

‫اﻹﺟﺎﺑﺔ ﻋﲆ اﻟﺼﻔﺤﺔ اﻟﺘﺎﻟﻴﺔ‬

Page - 2070
Internal Medicine - Pulmonology - Asthma

Question 103/144

Question #103

Which is the most appropriate first-line treatment for a 22-year-old male patient with acute asthma
exacerbation?

a. Oxygen
b. Salbutamol intravenously
c. Salbutamol nebulizer √
d. Salmeterol nebulizer
e. Intravenous MgSO4

Description

It is essential to know that Oxygen is beneficial for acute asthma exacerbation patients, but the
first-line treatment is SABA (Salbutamol, Albuterol, Levalbuterol).

Treatment of asthma exacerbation:

e first step:
Oxygen: For all hypoxic patients, given by venturi mask
SABA: 5 – 10 pus by MDI and spacer (SABA nebulizer is indicated in severe cases)- can be
repeated up to 3 doses in 1 hour.
Early administration of systemic steroid (40 – 50 mg oral prednisolone)
e second step: Nebulized SAMA (Short-acting muscarinic antagonists, e.g., Ipratropium)
e third step: Magnesium sulfate IV (1.2 – 2 g over 30 minutes)
Not routinely used
Only if the previous measures fail to improve the patient
Intubation and mechanical ventilation should be done in any patient with life-threatening or
near-fatal asthma.

Page - 2071
Internal Medicine - Pulmonology

Question 104/144

Question #104

A 55-year-old male patient with a history of HTN and Asthma developed wheezes and chest
tightness 2 days aer changing his medications, which his GP prescribed. What is the most likely
cause of his symptoms?

a. Bisoprolol
b. Enalapril
c. Verapamil
d. Carvedilol
e. Aldactone

‫اﻹﺟﺎﺑﺔ ﻋﲆ اﻟﺼﻔﺤﺔ اﻟﺘﺎﻟﻴﺔ‬

Page - 2072
Internal Medicine - Pulmonology - Asthma

Question 104/144

Question #104

A 55-year-old male patient with a history of HTN and Asthma developed wheezes and chest
tightness 2 days aer changing his medications, which his GP prescribed. What is the most likely
cause of his symptoms?

a. Bisoprolol
b. Enalapril
c. Verapamil
d. Carvedilol √
e. Aldactone

Description

Patients with asthma should avoid using aspirin, NSAIDs, and non-selective Beta-blockers because
they can exacerbate their symptoms.

Bisoprolol is a selective beta 1 blocker and will not cause bronchospasm if used wisely.

Note: Even the selective beta-blockers in high doses will lose their selectivity, so do not use selective
beta-blockers in high doses for asthma patients

Page - 2073
Internal Medicine - Pulmonology

Question 105/144

Question #105

Which of the following asthmatic patient would be preferred to be treated with low-dose inhaled
corticosteroids alone?

a. A 19-year-old male with once-weekly symptoms


b. A 20-year-old female with symptoms of 3 times weekly but less than once daily
c. A 20-year-old male with exercise-induced asthma
d. A 21-year-old female with asthma symptoms several times a day
e. A18-year-old male with daily asthmatic symptoms

‫اﻹﺟﺎﺑﺔ ﻋﲆ اﻟﺼﻔﺤﺔ اﻟﺘﺎﻟﻴﺔ‬

Page - 2074
Internal Medicine - Pulmonology - Asthma

Question 105/144

Question #105

Which of the following asthmatic patient would be preferred to be treated with low-dose inhaled
corticosteroids alone?

a. A 19-year-old male with once-weekly symptoms


b. A 20-year-old female with symptoms of 3 times weekly but less than once daily √
c. A 20-year-old male with exercise-induced asthma
d. A 21-year-old female with asthma symptoms several times a day
e. A18-year-old male with daily asthmatic symptoms

Description

According to recent guidelines, the initial asthma management depends on the class of asthma as
the following:

Mild intermittent: No maintenance therapy required.


Mild persistent: Low dose ICS alone
Moderate persistent: Low dose ICS + LABA
Severe persistent: high dose ICS + LABA
Very severe persistent: high dose ICS + LABA + oral steroids

e second patient (Choice B) complains of 3 times weekly symptoms but less than once daily.
erefore, he has mild persistent asthma. He needs maintenance treatment with ICS alone. Which
makes this the correct answer

e following diagram demonstrates the recent guidelines in asthma management:

Page - 2075
Page - 2076
Internal Medicine - Pulmonology

Question 106/144

Question #106

A 15-year-old male has presented with recurrent shortness of breath for the last 6 months. His
dyspnea occurs more at night. His little brother has a history of eczema. e following drugs are not
safe to use in this patient except:

a. Propranolol
b. Naproxen
c. Aspirin
d. Celecoxib
e. Prednisolone

‫اﻹﺟﺎﺑﺔ ﻋﲆ اﻟﺼﻔﺤﺔ اﻟﺘﺎﻟﻴﺔ‬

Page - 2077
Internal Medicine - Pulmonology - Asthma

Question 106/144

Question #106

A 15-year-old male has presented with recurrent shortness of breath for the last 6 months. His
dyspnea occurs more at night. His little brother has a history of eczema. e following drugs are not
safe to use in this patient except:

a. Propranolol
b. Naproxen
c. Aspirin
d. Celecoxib
e. Prednisolone √

Description

is patient complains of recurrent shortness of breath, night symptoms, and a family history of an
IgE-related disease (eczema in her brother). erefore, asthma is the most likely diagnosis.

Prednisolone is safe to use in the treatment of asthma.

e rest of the drugs mentioned can cause bronchospasm and should be avoided in asthmatic
patients.

Page - 2078
Internal Medicine - Pulmonology

Question 107/144

Question #107

All the following patients with acute status asthmaticus should be admitted to the hospital except:

a. A patient with a history of life-threatening asthma


b. A pregnant lady with a moderate asthma attack
c. A patient on oral steroids presents with a moderate asthma attack
d. A patient with normal PaCO2 on ABGs during the event
e. None of the above

‫اﻹﺟﺎﺑﺔ ﻋﲆ اﻟﺼﻔﺤﺔ اﻟﺘﺎﻟﻴﺔ‬

Page - 2079
Internal Medicine - Pulmonology - Asthma

Question 107/144

Question #107

All the following patients with acute status asthmaticus should be admitted to the hospital except:

a. A patient with a history of life-threatening asthma


b. A pregnant lady with a moderate asthma attack
c. A patient on oral steroids presents with a moderate asthma attack
d. A patient with normal PaCO2 on ABGs during the event
e. None of the above √

Description

All of the mentioned patients should be admitted to the hospital.

Indications for admission in acute asthma attacks are:

All patients with life-threatening asthma


Severe asthma attack not responding to the initial treatment
Previous near-fatal asthma attack
Pregnant women
An attack despite the use of oral steroids

Page - 2080
Internal Medicine - Pulmonology

Question 108/144

Question #108

A 29-year-old male patient presents to your clinic because he has had uncontrolled asthma over the
past month. He experiences asthmatic symptoms more than 5 times weekly and recurrent night-
time exacerbations. However, his current medications include salbutamol with ICS as required,
regular low-dose ICS, and a regular Salmeterol inhaler. What is the best next step in the
management of this patient?

a. Keep the same treatment


b. Add oral steroid
c. Add ipratropium bromide inhaler PRN
d. Add theophylline
e. Increase the dose of ICS

‫اﻹﺟﺎﺑﺔ ﻋﲆ اﻟﺼﻔﺤﺔ اﻟﺘﺎﻟﻴﺔ‬

Page - 2081
Internal Medicine - Pulmonology - Asthma

Question 108/144

Question #108

A 29-year-old male patient presents to your clinic because he has had uncontrolled asthma over the
past month. He experiences asthmatic symptoms more than 5 times weekly and recurrent night-
time exacerbations. However, his current medications include salbutamol with ICS as required,
regular low-dose ICS, and a regular Salmeterol inhaler. What is the best next step in the
management of this patient?

a. Keep the same treatment


b. Add oral steroid
c. Add ipratropium bromide inhaler PRN
d. Add theophylline
e. Increase the dose of ICS √

Description

is patient takes low-dose ICS + LABA for asthma maintenance and still has uncontrolled
symptoms. erefore, high-dose ICS should be used instead of low doses.

e following diagram demonstrates the recent guidelines in asthma management:

Page - 2082
Internal Medicine - Pulmonology

Question 109/144

Question #109

A 22-year-old gentleman with a known case of asthma presents with acute exacerbation. Which of
the following is the first line to use in acute exacerbation of his asthma?

a. Oxygen
b. Salbutamol nebulizer
c. Prednisolone
d. Ipratropium bromide
e. Sodium cromolyn

‫اﻹﺟﺎﺑﺔ ﻋﲆ اﻟﺼﻔﺤﺔ اﻟﺘﺎﻟﻴﺔ‬

Page - 2083
Internal Medicine - Pulmonology - Asthma

Question 109/144

Question #109

A 22-year-old gentleman with a known case of asthma presents with acute exacerbation. Which of
the following is the first line to use in acute exacerbation of his asthma?

a. Oxygen
b. Salbutamol nebulizer √
c. Prednisolone
d. Ipratropium bromide
e. Sodium cromolyn

Description

e first-line treatment for acute attacks of asthma is short-acting beta-agonists

Treatment of asthma exacerbation:

e first step:
Oxygen: For all hypoxic patients, given by venturi mask
SABA: 5 – 10 pus by MDI and spacer (SABA nebulizer is indicated in severe cases)- can be
repeated up to 3 doses in 1 hour.
Early administration of systemic steroid (40 – 50 mg oral prednisolone)
e second step: Nebulized SAMA (Short-acting muscarinic antagonists, e.g., Ipratropium)
e third step: Magnesium sulfate IV (1.2 – 2 g over 30 minutes)
Not routinely used
Only if the previous measures fail to improve the patient
Intubation and mechanical ventilation should be done in any patient with life-threatening or
near-fatal asthma.

Page - 2084
Internal Medicine - Pulmonology

Question 110/144

Question #110

A 29-year-old female is a known case of asthma and is on a salbutamol inhaler as needed. She uses
her inhaler 4 – 5 times weekly with a good response. What is the next step in management?

a. Keep the patient on the same treatment


b. Add Long-acting beta 2 agonist
c. Add a low dose of beclomethasone
d. Add a high-dose beclomethasone
e. Add ipratropium bromide as needed

‫اﻹﺟﺎﺑﺔ ﻋﲆ اﻟﺼﻔﺤﺔ اﻟﺘﺎﻟﻴﺔ‬

Page - 2085
Internal Medicine - Pulmonology - Asthma

Question 110/144

Question #110

A 29-year-old female is a known case of asthma and is on a salbutamol inhaler as needed. She uses
her inhaler 4 – 5 times weekly with a good response. What is the next step in management?

a. Keep the patient on the same treatment


b. Add Long-acting beta 2 agonist
c. Add a low dose of beclomethasone √
d. Add a high-dose beclomethasone
e. Add ipratropium bromide as needed

Description

e patient is still uncontrolled because she uses her inhaler more than 2 times weekly.

You should add a low-dose ICS to her regimen to control her asthma.

e following diagram demonstrates the recent guidelines in asthma management:

Page - 2086
Internal Medicine - Pulmonology

Question 111/144

Question #111

A 22-year-old gentleman with a known case of asthma presents with acute exacerbation. Which of
the following is not used in treating acute asthmatic attacks?

a. Short-acting beta-agonist
b. Inhaled corticosteroids
c. Oral Prednisolone
d. Ipratropium bromide
e. Adrenalin

‫اﻹﺟﺎﺑﺔ ﻋﲆ اﻟﺼﻔﺤﺔ اﻟﺘﺎﻟﻴﺔ‬

Page - 2087
Internal Medicine - Pulmonology - Asthma

Question 111/144

Question #111

A 22-year-old gentleman with a known case of asthma presents with acute exacerbation. Which of
the following is not used in treating acute asthmatic attacks?

a. Short-acting beta-agonist
b. Inhaled corticosteroids √
c. Oral Prednisolone
d. Ipratropium bromide
e. Adrenalin

Description

Inhaled corticosteroids are used in preventing asthmatic attacks, not as a reliever

Treatment of asthma exacerbation:

e first step:
Oxygen: For all hypoxic patients, given by venturi mask
SABA: 5 – 10 pus by MDI and spacer (SABA nebulizer is indicated in severe cases)- can be
repeated up to 3 doses in 1 hour.
Early administration of systemic steroid (40 – 50 mg oral prednisolone)
e second step: Nebulized SAMA (Short-acting muscarinic antagonists, e.g., Ipratropium)
e third step: Magnesium sulfate IV (1.2 – 2 g over 30 minutes)
Not routinely used
Only if the previous measures fail to improve the patient
Intubation and mechanical ventilation should be done in any patient with life-threatening or
near-fatal asthma.

Page - 2088
Internal Medicine - Pulmonology

Question 112/144

Question #112

A 15-year-old male has presented with recurrent shortness of breath for the last 6 months. His
dyspnea occurs more at night, and his little brother has a history of eczema. What is the most likely
diagnosis?

a. COPD
b. Sarcoidosis
c. Asthma
d. Allergic rhinitis
e. Cystic fibrosis

‫اﻹﺟﺎﺑﺔ ﻋﲆ اﻟﺼﻔﺤﺔ اﻟﺘﺎﻟﻴﺔ‬

Page - 2089
Internal Medicine - Pulmonology - Asthma

Question 112/144

Question #112

A 15-year-old male has presented with recurrent shortness of breath for the last 6 months. His
dyspnea occurs more at night, and his little brother has a history of eczema. What is the most likely
diagnosis?

a. COPD
b. Sarcoidosis
c. Asthma √
d. Allergic rhinitis
e. Cystic fibrosis

Description

Recurrent dyspnea and a family history of an IgE-related disease (Eczema) strongly suggest asthma
in this patient.

Asthma:

Asthma is a chronic disease that causes inflammation with reversible & variable narrowing of
the airways (e main dierence between asthma and COPD is reversibility and variability)
Asthma is usually associated with a history or family history of other IgE-related diseases like
atopic dermatitis (eczema) and atopic rhinitis (hay fever)
Triggers are: dust, smoking, air pollution, Drugs – NSAID, Beta-blockers, exercise, emotions,
sudden changes in air temperature

Page - 2090
Internal Medicine - Pulmonology

Question 113/144

Question #113

A 19-year-old male patient was brought to the emergency department aer falling on his knee. e
patient is in pain, and you decided to give him analgesia. However, his medications include only
salbutamol for asthma as needed. What is the most appropriate analgesia to administer?

a. Aspirin
b. Diclofenac
c. Ibuprofen
d. Paracetamol
e. Celecoxib

‫اﻹﺟﺎﺑﺔ ﻋﲆ اﻟﺼﻔﺤﺔ اﻟﺘﺎﻟﻴﺔ‬

Page - 2091
Internal Medicine - Pulmonology - Asthma

Question 113/144

Question #113

A 19-year-old male patient was brought to the emergency department aer falling on his knee. e
patient is in pain, and you decided to give him analgesia. However, his medications include only
salbutamol for asthma as needed. What is the most appropriate analgesia to administer?

a. Aspirin
b. Diclofenac
c. Ibuprofen
d. Paracetamol √
e. Celecoxib

Description

Aspirin, NSAIDs, and beta-blockers may exacerbate asthma. ese drugs should be avoided here

e safest drug to give here is paracetamol

Triggers of asthma: dust, smoking, air pollution, Drugs – NSAID, Beta-blockers, exercise, emotions,
sudden changes in air temperature

Page - 2092
Internal Medicine - Pulmonology

Question 114/144

Question #114

A 22-year-old male patient who is known case of asthma presents with acute asthma exacerbation;
the following features indicate a life-threatening asthma attack except:

a. PEFR < 33%


b. Normal or high CO2
c. Respiratory acidosis
d. Wheezy chest
e. Bradycardia and Hypotension

‫اﻹﺟﺎﺑﺔ ﻋﲆ اﻟﺼﻔﺤﺔ اﻟﺘﺎﻟﻴﺔ‬

Page - 2093
Internal Medicine - Pulmonology - Asthma

Question 114/144

Question #114

A 22-year-old male patient who is known case of asthma presents with acute asthma exacerbation;
the following features indicate a life-threatening asthma attack except:

a. PEFR < 33%


b. Normal or high CO2
c. Respiratory acidosis
d. Wheezy chest √
e. Bradycardia and Hypotension

Description

A silent chest, not a wheezy chest, is a feature of life-threatening asthma.

Based on PEFR, ABGs, and clinical features, asthma exacerbations are classified as moderate, severe,
and life-threatening.

e following table demonstrates the features of acute asthma classes:

Page - 2094
Internal Medicine - Pulmonology

Question 115/144

Question #115

A 25-year-old male patient presented with acute asthma exacerbation. His respiratory rate is 25
breaths per minute. His pulse rate is 110 bpm, the patient has a wheezy chest, and he looks drowsy.
Which of the following is a criterion for life-threatening asthma?

a. Respiratory rate > 24 breaths per minute


b. PEFR 33 – 50% of predicted
c. Heart rate of more than 90 bpm
d. Bilateral severe wheezes
e. Altered mental status and drowsiness

‫اﻹﺟﺎﺑﺔ ﻋﲆ اﻟﺼﻔﺤﺔ اﻟﺘﺎﻟﻴﺔ‬

Page - 2095
Internal Medicine - Pulmonology - Asthma

Question 115/144

Question #115

A 25-year-old male patient presented with acute asthma exacerbation. His respiratory rate is 25
breaths per minute. His pulse rate is 110 bpm, the patient has a wheezy chest, and he looks drowsy.
Which of the following is a criterion for life-threatening asthma?

a. Respiratory rate > 24 breaths per minute


b. PEFR 33 – 50% of predicted
c. Heart rate of more than 90 bpm
d. Bilateral severe wheezes
e. Altered mental status and drowsiness √

Description

Based on PEFR, ABGs, and clinical features, asthma exacerbations are classified as moderate, severe,
and life-threatening.

e following table demonstrates the features of acute asthma classes:

Page - 2096
Internal Medicine - Pulmonology

Question 116/144

Question #116

A 36-year-old patient with a known case of asthma presents to the ER with a status asthmaticus. All
of the following may be included in the therapy except:

a. Inhaled beta 2 agonist


b. IV hydration
c. Ipratropium bromide
d. Intubation and ventilation
e. Inhaled corticosteroids

‫اﻹﺟﺎﺑﺔ ﻋﲆ اﻟﺼﻔﺤﺔ اﻟﺘﺎﻟﻴﺔ‬

Page - 2097
Internal Medicine - Pulmonology - Asthma

Question 116/144

Question #116

A 36-year-old patient with a known case of asthma presents to the ER with a status asthmaticus. All
of the following may be included in the therapy except:

a. Inhaled beta 2 agonist


b. IV hydration
c. Ipratropium bromide
d. Intubation and ventilation
e. Inhaled corticosteroids √

Description

Inhaled steroids are not used in acute asthma exacerbation.

Oral or IV steroids are equivalent to each other in acute asthma management.

Treatment of asthma exacerbation:

e first step:
Oxygen: For all hypoxic patients, given by venturi mask
SABA: 5 – 10 pus by MDI and spacer (SABA nebulizer is indicated in severe cases)- can be
repeated up to 3 doses in 1 hour.
Early administration of systemic steroid (40 – 50 mg oral prednisolone)
e second step: Nebulized SAMA (Short-acting muscarinic antagonists, e.g., Ipratropium)
e third step: Magnesium sulfate IV (1.2 – 2 g over 30 minutes)
Not routinely used
Only if the previous measures fail to improve the patient
Intubation and mechanical ventilation should be done in any patient with life-threatening or
near-fatal asthma.

Page - 2098
Internal Medicine - Pulmonology

Question 117/144

Question #117

A 24-year-old male presents with a cough, shortness of breath, and wheezes. e symptoms start 10
minutes aer jogging. What test would you do next for your patient?

a. Chest x-ray
b. Allergy skin test
c. Spirometry
d. Peak flow rate
e. Arterial blood gas

‫اﻹﺟﺎﺑﺔ ﻋﲆ اﻟﺼﻔﺤﺔ اﻟﺘﺎﻟﻴﺔ‬

Page - 2099
Internal Medicine - Pulmonology - Asthma

Question 117/144

Question #117

A 24-year-old male presents with a cough, shortness of breath, and wheezes. e symptoms start 10
minutes aer jogging. What test would you do next for your patient?

a. Chest x-ray
b. Allergy skin test
c. Spirometry √
d. Peak flow rate
e. Arterial blood gas

Description

Description:

is is a suspected case of exercise-induced asthma; spirometry showing a reversible obstructive


pattern is diagnostic in this condition.

Note that if the patient improves on the salbutamol inhaler, this will be diagnostic for asthma

Page - 2100
Internal Medicine - Pulmonology

Question 118/144

Question #118

A patient with recurrent chest infections presents with persistent coughs productive of a copious
amount of purulent sputum. Physical examination shows inspiratory crepitation and finger
clubbing. His chest x-ray shows a tram track appearance. What is the most likely diagnosis?

a. Sarcoidosis
b. Bronchiectasis
c. Asthma
d. COPD
e. Pulmonary fibrosis

‫اﻹﺟﺎﺑﺔ ﻋﲆ اﻟﺼﻔﺤﺔ اﻟﺘﺎﻟﻴﺔ‬

Page - 2101
Internal Medicine - Pulmonology - Bronchiectasis

Question 118/144

Question #118

A patient with recurrent chest infections presents with persistent coughs productive of a copious
amount of purulent sputum. Physical examination shows inspiratory crepitation and finger
clubbing. His chest x-ray shows a tram track appearance. What is the most likely diagnosis?

a. Sarcoidosis
b. Bronchiectasis √
c. Asthma
d. COPD
e. Pulmonary fibrosis

Description

Several points in the question point to bronchiectasis as the most likely diagnosis.

e presence of recurrent chest infection, copious amount of purulent sputum, clubbing, and tram
track appearance on chest x-ray

Bronchiectasis is an irreversible abnormal dilatation of small and medium-sized bronchi with


chronic airway inflammation. It is associated with regular sputum production, chronic cough,
recurrent acute chest infections, and airflow obstruction.

Page - 2102
Internal Medicine - Pulmonology

Question 119/144

Question #119

A young adult who has had recurrent chest infections since childhood presents with excessive
purulent sputum and drumstick-shaped fingers. What is the most likely diagnosis?

a. Sarcoidosis
b. Bronchiectasis
c. Asthma
d. COPD
e. Pulmonary fibrosis

‫اﻹﺟﺎﺑﺔ ﻋﲆ اﻟﺼﻔﺤﺔ اﻟﺘﺎﻟﻴﺔ‬

Page - 2103
Internal Medicine - Pulmonology - Bronchiectasis

Question 119/144

Question #119

A young adult who has had recurrent chest infections since childhood presents with excessive
purulent sputum and drumstick-shaped fingers. What is the most likely diagnosis?

a. Sarcoidosis
b. Bronchiectasis √
c. Asthma
d. COPD
e. Pulmonary fibrosis

Description

Bronchiectasis is an irreversible abnormal dilatation of small and medium-sized bronchi with


chronic airway inflammation. It is associated with regular sputum production, chronic cough,
recurrent acute chest infections, and airflow obstruction.

Asthma and COPD have not been known to cause figure clubbing

Pulmonary fibrosis manifests with dry cough

Page - 2104
Internal Medicine - Pulmonology

Question 120/144

Question #120

A 29-year-old male has had recurrent chest infections and chronic coughs since childhood. His cough
is exacerbated by lying flat and is associated with bloody, foul-smelling sputum. Examination shows
clubbing fingers, and coarse crepitations are heard over the lung bases. Which is the most likely
diagnosis?

a. Emphysema
b. Chronic bronchitis
c. Lung cancer
d. Bronchiectasis
e. Atopic asthma

‫اﻹﺟﺎﺑﺔ ﻋﲆ اﻟﺼﻔﺤﺔ اﻟﺘﺎﻟﻴﺔ‬

Page - 2105
Internal Medicine - Pulmonology - Bronchiectasis

Question 120/144

Question #120

A 29-year-old male has had recurrent chest infections and chronic coughs since childhood. His cough
is exacerbated by lying flat and is associated with bloody, foul-smelling sputum. Examination shows
clubbing fingers, and coarse crepitations are heard over the lung bases. Which is the most likely
diagnosis?

a. Emphysema
b. Chronic bronchitis
c. Lung cancer
d. Bronchiectasis √
e. Atopic asthma

Description

History of chronic or recurrent chest infection, Cough with foul-smelling sputum that changes with
body position, and Presence of coarse crepitations are all indicating bronchiectasis

Bronchiectasis:

It is a dilatation of the airways due to chronic lung infection in childhood (anatomic defect)
It usually aects medium-sized airways
Cystic fibrosis is the most common cause of bronchiectasis

Page - 2106
Internal Medicine - Pulmonology

Question 121/144

Question #121

A 34-year-old male patient has been known to have a history of cystic fibrosis and recurrent chest
infections since childhood. In addition, he presents with chronic coughs; his chest x-ray is shown in
the picture below. What is the most likely diagnosis?

a. Emphysema
b. Chronic bronchitis
c. Bronchiectasis
d. Asthma
e. Acute lobar pneumonia

‫اﻹﺟﺎﺑﺔ ﻋﲆ اﻟﺼﻔﺤﺔ اﻟﺘﺎﻟﻴﺔ‬


Page - 2107
Internal Medicine - Pulmonology - Bronchiectasis

Question 121/144

Question #121

A 34-year-old male patient has been known to have a history of cystic fibrosis and recurrent chest
infections since childhood. In addition, he presents with chronic coughs; his chest x-ray is shown in
the picture below. What is the most likely diagnosis?

a. Emphysema
b. Chronic bronchitis
c. Bronchiectasis √
d. Asthma
e. Acute lobar pneumonia

Description
Page - 2108
Tram tack appearance on chest X-ray, History of chronic or recurrent chest infection, Cough with
foul-smelling sputum that changes with body position, and Presence of coarse crepitations are all
indicating bronchiectasis

Bronchiectasis:

It is a dilatation of the airways due to chronic lung infection in childhood (anatomic defect)
It usually aects medium-sized airways
Cystic fibrosis is the most common cause of bronchiectasis

Page - 2109
Internal Medicine - Pulmonology

Question 122/144

Question #122

A 42-year-old male patient with 3 years history of daily cough and malodourous sputum was treated
with antibiotics for a recurrent chest infection and hemoptysis. His FEV1/FVC ratio is 0.60. His CT
scan shows bronchial wall thickening and dilatation of the bronchi. What is the most likely
diagnosis?

a. Idiopathic pulmonary fibrosis


b. Sarcoidosis
c. Bronchiectasis
d. Bronchitis
e. Emphysema

‫اﻹﺟﺎﺑﺔ ﻋﲆ اﻟﺼﻔﺤﺔ اﻟﺘﺎﻟﻴﺔ‬

Page - 2110
Internal Medicine - Pulmonology - Bronchiectasis

Question 122/144

Question #122

A 42-year-old male patient with 3 years history of daily cough and malodourous sputum was treated
with antibiotics for a recurrent chest infection and hemoptysis. His FEV1/FVC ratio is 0.60. His CT
scan shows bronchial wall thickening and dilatation of the bronchi. What is the most likely
diagnosis?

a. Idiopathic pulmonary fibrosis


b. Sarcoidosis
c. Bronchiectasis √
d. Bronchitis
e. Emphysema

Description

History of recurrent lung infection, foul-smelling (malodourous) sputum, the CT scan findings, and
Spirometry findings are suggestive of bronchiectasis

Bronchiectasis:

It is a dilatation of the airways due to chronic lung infection in childhood (anatomic defect)
It usually aects medium-sized airways
Cystic fibrosis is the most common cause of bronchiectasis

Page - 2111
Internal Medicine - Pulmonology

Question 123/144

Question #123

A 60-year-old male patient has a productive cough, foul-smelling sputum, clubbing, coarse
crepitations, and peri-bronchial thickening on chest X-ray. What is the next step in managing this
patient?

a. Bronchoscopy
b. High-resolution CT scan
c. Sputum culture
d. Lung biopsy
e. Electrocardiogram

‫اﻹﺟﺎﺑﺔ ﻋﲆ اﻟﺼﻔﺤﺔ اﻟﺘﺎﻟﻴﺔ‬

Page - 2112
Internal Medicine - Pulmonology - Bronchiectasis

Question 123/144

Question #123

A 60-year-old male patient has a productive cough, foul-smelling sputum, clubbing, coarse
crepitations, and peri-bronchial thickening on chest X-ray. What is the next step in managing this
patient?

a. Bronchoscopy
b. High-resolution CT scan √
c. Sputum culture
d. Lung biopsy
e. Electrocardiogram

Description

Description:

is is a typical scenario for bronchiectasis. However, a High-resolution CT scan of the lung is the
diagnostic modality of choice for bronchiectasis

Bronchiectasis:

It is a dilatation of the airways due to chronic lung infection in childhood (anatomic defect)
It usually aects medium-sized airways
Cystic fibrosis is the most common cause of bronchiectasis

Page - 2113
Internal Medicine - Pulmonology

Question 124/144

Question #124

An HIV-positive, 32-year-old male patient presents with persistent productive coughs with a large
amount of purulent sputum. His chest examination shows inspiratory crepitations. However, His
chest x-ray shows a tram track appearance. What is the most likely diagnosis?

a. Sarcoidosis
b. Bronchiectasis
c. Asthma
d. COPD
e. Pulmonary fibrosis

‫اﻹﺟﺎﺑﺔ ﻋﲆ اﻟﺼﻔﺤﺔ اﻟﺘﺎﻟﻴﺔ‬

Page - 2114
Internal Medicine - Pulmonology - Bronchiectasis

Question 124/144

Question #124

An HIV-positive, 32-year-old male patient presents with persistent productive coughs with a large
amount of purulent sputum. His chest examination shows inspiratory crepitations. However, His
chest x-ray shows a tram track appearance. What is the most likely diagnosis?

a. Sarcoidosis
b. Bronchiectasis √
c. Asthma
d. COPD
e. Pulmonary fibrosis

Description

Several points in the question point to bronchiectasis as the most likely diagnosis.

e possible presence of recurrent chest infection secondary to HIV, copious amount of purulent
sputum, and tram track appearance on chest x-ray

Bronchiectasis is an irreversible abnormal dilatation of small and medium-sized bronchi with


chronic airway inflammation. It is associated with regular sputum production, chronic cough,
recurrent acute chest infections, and airflow obstruction.

Page - 2115
Internal Medicine - Pulmonology

Question 125/144

Question #125

A 34-year-old male patient has been known to have a history of cystic fibrosis and recurrent chest
infections since childhood. He presents with chronic coughs; his chest CT scan is shown in the
picture below. What is the name of this sign?

a. Tram track appearance


b. Meniscus sign
c. Signet-ring sign
d. Peri-bronchial edema
e. Ground glass appearance

‫اﻹﺟﺎﺑﺔ ﻋﲆ اﻟﺼﻔﺤﺔ اﻟﺘﺎﻟﻴﺔ‬

Page - 2116
Internal Medicine - Pulmonology - Bronchiectasis

Question 125/144

Question #125

A 34-year-old male patient has been known to have a history of cystic fibrosis and recurrent chest
infections since childhood. He presents with chronic coughs; his chest CT scan is shown in the
picture below. What is the name of this sign?

a. Tram track appearance


b. Meniscus sign
c. Signet-ring sign √
d. Peri-bronchial edema
e. Ground glass appearance

Description

Signet-ring sign on chest CT scan, history of chronic or recurrent chest infection, Cough with foul-
Page - 2117
smelling sputum that changes with body position, and Presence of coarse crepitations are all
indicating bronchiectasis

Bronchiectasis:

It is a dilatation of the airways due to chronic lung infection in childhood (anatomic defect)
It usually aects medium-sized airways
Cystic fibrosis is the most common cause of bronchiectasis

Page - 2118
Internal Medicine - Pulmonology

Question 126/144

Question #126

A 20-year-old male patient presents with a dry cough, headache, fatigue, and low-grade fever for
one week. He is in good general condition. His chest x-ray shows bilateral pneumonia without
pleural eusion. Which is the most likely cause of this patient’s condition?

a. H. influenzas type B
b. Streptococcus pneumonia
c. Staphylococci
d. Klebsiella
e. Mycoplasma

‫اﻹﺟﺎﺑﺔ ﻋﲆ اﻟﺼﻔﺤﺔ اﻟﺘﺎﻟﻴﺔ‬

Page - 2119
Internal Medicine - Pulmonology - Pneumonia

Question 126/144

Question #126

A 20-year-old male patient presents with a dry cough, headache, fatigue, and low-grade fever for
one week. He is in good general condition. His chest x-ray shows bilateral pneumonia without
pleural eusion. Which is the most likely cause of this patient’s condition?

a. H. influenzas type B
b. Streptococcus pneumonia
c. Staphylococci
d. Klebsiella
e. Mycoplasma √

Description

is is atypical pneumonia (dry cough and looking well patient). e most common cause of atypical
pneumonia is mycoplasma pneumonia.

Pneumonia:

Pneumonia is defined as an infection of the lung, which can be bacterial, viral, or fungal
Low immunity, reduced protective mechanisms, and low ciliary movement are risk factors for
pneumonia
It is classified into community-acquired (CAP) or Hospital-acquired (HAP)
CAP is further classified into typical and atypical pneumonia.

Causes of pneumonia:

Pneumococcal pneumonia is the most common Typical CAP


Mycoplasma pneumonia is the most common atypical CAP
Staph aureus is the most common cause of HAP
PCP is the most common in HIV and immunosuppressed patients
Legionella is common in air conditioning system workers
Klebsiella pneumonia is common in alcoholics and DM patients
Aerobics are common in those with aspiration or poor dentation

Page - 2120
Internal Medicine - Pulmonology

Question 127/144

Question #127

An alcoholic 49-year-old male presents with shortness of breath, cough, fever, and bloody sputum.
His chest x-ray shows an early cavitary lesion. What is the most common cause of his condition?

a. Pneumococcal pneumonia
b. Legionella pneumonia
c. Mycoplasma pneumonia
d. PCP
e. Klebsiella pneumonia

‫اﻹﺟﺎﺑﺔ ﻋﲆ اﻟﺼﻔﺤﺔ اﻟﺘﺎﻟﻴﺔ‬

Page - 2121
Internal Medicine - Pulmonology - Pneumonia

Question 127/144

Question #127

An alcoholic 49-year-old male presents with shortness of breath, cough, fever, and bloody sputum.
His chest x-ray shows an early cavitary lesion. What is the most common cause of his condition?

a. Pneumococcal pneumonia
b. Legionella pneumonia
c. Mycoplasma pneumonia
d. PCP
e. Klebsiella pneumonia √

Description

Klebsiella pneumonia is common in alcoholic patients and is a known cause of cavitary lesions on
chest x-ray. e presence of Red-Current jelly will support the diagnosis.

Klebsiella pneumonia:

It is classically in alcoholics and DM patients


Presented with Red-Current jelly sputum (hemoptysis from necrotizing disease)
Present with aspiration pneumonia
A common cause of Lung Abscess and cavitating lesions on chest x-ray

Page - 2122
Internal Medicine - Pulmonology

Question 128/144

Question #128

Which of the following CD4 count are the HIV patients at increased risk of pneumocystis carinii
pneumonia (PCP)?

a. Above 500 cells/mm


b. 200 – 500 cells/mm
c. 100 – 200 cells/mm
d. 50 – 100 cells/mm
e. Less than 50 cells/mm

‫اﻹﺟﺎﺑﺔ ﻋﲆ اﻟﺼﻔﺤﺔ اﻟﺘﺎﻟﻴﺔ‬

Page - 2123
Internal Medicine - Pulmonology - Pneumonia

Question 128/144

Question #128

Which of the following CD4 count are the HIV patients at increased risk of pneumocystis carinii
pneumonia (PCP)?

a. Above 500 cells/mm


b. 200 – 500 cells/mm
c. 100 – 200 cells/mm √
d. 50 – 100 cells/mm
e. Less than 50 cells/mm

Description

PCP is the most common type of pneumonia that occurs in HIV patients.

Pneumocystis Carinii Pneumonia (PCP):

It is caused by yeast-like fungus pneumocystis jiroveci


Usually seen in patients with cancer, chemotherapy, HIV, and drug abusers
It is the most common cause of pneumonia in HIV patients (with CD4 of 100 – 200/mm3)
Pneumothorax is a common complication

e following table demonstrates the most common diseases aecting the HIV

Patients and the CD4 count at which the diseases most commonly occur:

Page - 2124
Page - 2125
Internal Medicine - Pulmonology

Question 129/144

Question #129

A 48-year-old male patient presents with a fever of 39.2 °C, rigors, chills, and cough productive of
yellow sputum. He is previously healthy and never experienced a similar condition before. Chest x-
ray shows le lower lobe consolidation. e most likely cause of his pneumonia is:

a. Staphylococcus aureus
b. Klebsiella
c. Streptococcus pneumonia
d. Legionella pneumonia
e. Mycoplasma pneumonia

‫اﻹﺟﺎﺑﺔ ﻋﲆ اﻟﺼﻔﺤﺔ اﻟﺘﺎﻟﻴﺔ‬

Page - 2126
Internal Medicine - Pulmonology - Pneumonia

Question 129/144

Question #129

A 48-year-old male patient presents with a fever of 39.2 °C, rigors, chills, and cough productive of
yellow sputum. He is previously healthy and never experienced a similar condition before. Chest x-
ray shows le lower lobe consolidation. e most likely cause of his pneumonia is:

a. Staphylococcus aureus
b. Klebsiella
c. Streptococcus pneumonia √
d. Legionella pneumonia
e. Mycoplasma pneumonia

Description

is is typical pneumonia; the most common cause of typical pneumonia is streptococcus
pneumonia.

Pneumonia:

Pneumonia is defined as an infection of the lung, which can be bacterial, viral, or fungal
Low immunity, reduced protective mechanisms, and low ciliary movement are risk factors for
pneumonia
It is classified into community-acquired (CAP) or Hospital-acquired (HAP)
CAP is further classified into typical and atypical pneumonia.

Causes of pneumonia:

Pneumococcal pneumonia is the most common Typical CAP


Mycoplasma pneumonia is the most common atypical CAP
Staph aureus is the most common cause of HAP
PCP is the most common in HIV and immunosuppressed patients
Legionella is common in air conditioning system workers
Klebsiella pneumonia is common in alcoholics and DM patients
Aerobics are common in those with aspiration or poor dentation

Page - 2127
Internal Medicine - Pulmonology

Question 130/144

Question #130

A 24-year-old female presents with 2 days history of fever, shortness of breath, cough, and sputum.
Chest x-ray reveals pneumonia. 2 months ago, she was treated for sinusitis with amoxicillin. Which
of the following is the best choice for antibiotic treatment?

a. Amoxicillin
b. Azithromycin
c. Ceriaxone
d. Levofloxacin
e. Ciprofloxacin

‫اﻹﺟﺎﺑﺔ ﻋﲆ اﻟﺼﻔﺤﺔ اﻟﺘﺎﻟﻴﺔ‬

Page - 2128
Internal Medicine - Pulmonology - Pneumonia

Question 130/144

Question #130

A 24-year-old female presents with 2 days history of fever, shortness of breath, cough, and sputum.
Chest x-ray reveals pneumonia. 2 months ago, she was treated for sinusitis with amoxicillin. Which
of the following is the best choice for antibiotic treatment?

a. Amoxicillin
b. Azithromycin
c. Ceriaxone
d. Levofloxacin √
e. Ciprofloxacin

Description

Because the patient was using antibiotics in the last 3 months, respiratory fluoroquinolones are
indicated.

If the patient wasn’t using antibiotics in the last 3 months, macrolides are the first-line treatment.

e choice of antibiotic in typical pneumonia treatment:

Outpatient cases:
If previously healthy with no antibiotic use in the past 3 months, use macrolides
(azithromycin or clarithromycin) or Doxycycline
If there are comorbidities or use of antibiotics in the past 3 months, use respiratory
fluoroquinolones (Levofloxacin or Moxifloxacin)
Inpatient cases:
Respiratory fluoroquinolone (Levofloxacin or Moxifloxacin) or
Ceriaxone with Azithromycin

Page - 2129
Internal Medicine - Pulmonology

Question 131/144

Question #131

A 15-year-old patient was injured during a road trac accident; he had a resection of his lacerated
spleen at that time. Today aer 1 year, he presents with pneumonia. Which of the following is the
most likely cause of pneumonia?

a. Staphylococcus aureus
b. Streptococcus pneumonia
c. E. coli
d. Klebsiella pneumonia
e. Neisseria meningitidis

‫اﻹﺟﺎﺑﺔ ﻋﲆ اﻟﺼﻔﺤﺔ اﻟﺘﺎﻟﻴﺔ‬

Page - 2130
Internal Medicine - Pulmonology - Pneumonia

Question 131/144

Question #131

A 15-year-old patient was injured during a road trac accident; he had a resection of his lacerated
spleen at that time. Today aer 1 year, he presents with pneumonia. Which of the following is the
most likely cause of pneumonia?

a. Staphylococcus aureus
b. Streptococcus pneumonia √
c. E. coli
d. Klebsiella pneumonia
e. Neisseria meningitidis

Description

Streptococcus pneumonia is the most important pathogen in asplenic patients

Pneumonia:

Pneumonia is defined as an infection of the lung, which can be bacterial, viral, or fungal
Low immunity, reduced protective mechanisms, and low ciliary movement are risk factors for
pneumonia
It is classified into community-acquired (CAP) or Hospital-acquired (HAP)
CAP is further classified into typical and atypical pneumonia.

Causes of pneumonia:

Pneumococcal pneumonia is the most common Typical CAP


Mycoplasma pneumonia is the most common atypical CAP
Staph aureus is the most common cause of HAP
PCP is the most common in HIV and immunosuppressed patients
Legionella is common in air conditioning system workers
Klebsiella pneumonia is common in alcoholics and DM patients
Aerobics are common in those with aspiration or poor dentation

Page - 2131
Internal Medicine - Pulmonology

Question 132/144

Question #132

A 29-year-old male patient presents with coughs and shortness of breath and is diagnosed with
pneumonia. On physical examination, you find a rash of Kaposi’s sarcoma. What is the most likely
cause of his pneumonia?

a. Streptococcal pneumonia
b. Staphylococcus aureus
c. Pneumocystis carinii
d. Mycobacterium avium intercellulare
e. Mycoplasma pneumonia

‫اﻹﺟﺎﺑﺔ ﻋﲆ اﻟﺼﻔﺤﺔ اﻟﺘﺎﻟﻴﺔ‬

Page - 2132
Internal Medicine - Pulmonology - Pneumonia

Question 132/144

Question #132

A 29-year-old male patient presents with coughs and shortness of breath and is diagnosed with
pneumonia. On physical examination, you find a rash of Kaposi’s sarcoma. What is the most likely
cause of his pneumonia?

a. Streptococcal pneumonia
b. Staphylococcus aureus
c. Pneumocystis carinii √
d. Mycobacterium avium intercellulare
e. Mycoplasma pneumonia

Description

Kaposi’s sarcoma is usually found in Immunocompromised patients (e.g., AIDS); the most common
pneumonia in HIV patients is PCP.

Pneumonia:

Pneumonia is defined as an infection of the lung, which can be bacterial, viral, or fungal
Low immunity, reduced protective mechanisms, and low ciliary movement are risk factors for
pneumonia
It is classified into community-acquired (CAP) or Hospital-acquired (HAP)
CAP is further classified into typical and atypical pneumonia.

Causes of pneumonia:

Pneumococcal pneumonia is the most common Typical CAP


Mycoplasma pneumonia is the most common atypical CAP
Staph aureus is the most common cause of HAP
PCP is the most common in HIV and immunosuppressed patients
Legionella is common in air conditioning system workers
Klebsiella pneumonia is common in alcoholics and DM patients
Aerobics are common in those with aspiration or poor dentation

Page - 2133
Internal Medicine - Pulmonology

Question 133/144

Question #133

A 58-year-old male patient with a known COPD case presents with shortness of breath and cough
and an increasing amount of purulent sputum for 4 days. Chest examination is consistent with
crackles, dullness to percussion, and bronchial breathing. His temperature is 38.1 °C, BP is 130/80,
and pulse is 82 bpm. You diagnosed him with community-acquired pneumonia. What is the true
statement about this condition?

a. Amoxicillin is the preferred agent


b. A chest x-ray is indicated
c. A sputum sample is the most accurate test
d. Supportive treatment is only indicated
e. Wait until culture results are available before starting antibiotics

‫اﻹﺟﺎﺑﺔ ﻋﲆ اﻟﺼﻔﺤﺔ اﻟﺘﺎﻟﻴﺔ‬

Page - 2134
Internal Medicine - Pulmonology - Pneumonia

Question 133/144

Question #133

A 58-year-old male patient with a known COPD case presents with shortness of breath and cough
and an increasing amount of purulent sputum for 4 days. Chest examination is consistent with
crackles, dullness to percussion, and bronchial breathing. His temperature is 38.1 °C, BP is 130/80,
and pulse is 82 bpm. You diagnosed him with community-acquired pneumonia. What is the true
statement about this condition?

a. Amoxicillin is the preferred agent


b. A chest x-ray is indicated √
c. A sputum sample is the most accurate test
d. Supportive treatment is only indicated
e. Wait until culture results are available before starting antibiotics

Description

A chest x-ray is the best initial test; the patient is suspected of having pneumonia complicated by
pleural eusion. Treatment should include bronchodilators, steroids, antibiotics, and supportive
care.

Pneumonia:

Pneumonia is defined as an infection of the lung, which can be bacterial, viral, or fungal
Low immunity, reduced protective mechanisms, and low ciliary movement are risk factors for
pneumonia
It is classified into community-acquired (CAP) or Hospital-acquired (HAP)
CAP is further classified into typical and atypical pneumonia.

Causes of pneumonia:

Pneumococcal pneumonia is the most common Typical CAP


Mycoplasma pneumonia is the most common atypical CAP
Staph aureus is the most common cause of HAP
PCP is the most common in HIV and immunosuppressed patients
Legionella is common in air conditioning system workers
Klebsiella pneumonia is common in alcoholics and DM patients
Page - 2135
Aerobics are common in those with aspiration or poor dentation

Page - 2136
Internal Medicine - Pulmonology

Question 134/144

Question #134

A 22-year-old previously healthy male presents with low-grade fever, dry cough, and bilateral basal
infiltration on a chest x-ray. He is clinically well. You suspect mycoplasma pneumonia. What is the
treatment of choice for this condition?

a. Amoxicillin
b. Azithromycin
c. Ceriaxone
d. Vancomycin
e. Paracetamol

‫اﻹﺟﺎﺑﺔ ﻋﲆ اﻟﺼﻔﺤﺔ اﻟﺘﺎﻟﻴﺔ‬

Page - 2137
Internal Medicine - Pulmonology - Pneumonia

Question 134/144

Question #134

A 22-year-old previously healthy male presents with low-grade fever, dry cough, and bilateral basal
infiltration on a chest x-ray. He is clinically well. You suspect mycoplasma pneumonia. What is the
treatment of choice for this condition?

a. Amoxicillin
b. Azithromycin √
c. Ceriaxone
d. Vancomycin
e. Paracetamol

Description

Macrolides are the drugs of choice to treat mycoplasma pneumonia. Mycoplasma doesn’t have a cell
wall, so it will not be aected by antibiotics that inhibit cell wall synthesis.

Pneumonia:

Pneumonia is defined as an infection of the lung, which can be bacterial, viral, or fungal
Low immunity, reduced protective mechanisms, and low ciliary movement are risk factors for
pneumonia
It is classified into community-acquired (CAP) or Hospital-acquired (HAP)
CAP is further classified into typical and atypical pneumonia.

Causes of pneumonia:

Pneumococcal pneumonia is the most common Typical CAP


Mycoplasma pneumonia is the most common atypical CAP
Staph aureus is the most common cause of HAP
PCP is the most common in HIV and immunosuppressed patients
Legionella is common in air conditioning system workers
Klebsiella pneumonia is common in alcoholics and DM patients
Aerobics are common in those with aspiration or poor dentation

Page - 2138
Internal Medicine - Pulmonology

Question 135/144

Question #135

A 22-year-old male presents with community-acquired pneumonia. He is conscious-oriented and


alert and doesn’t use any medication. His temperature is 39.9 °C, his blood pressure is 91/62, and his
respiratory rate is 23 per minute. His lab investigations show hemoglobin of 14 g/dL, WBC of 25,000,
and normal kidney and liver function tests. Which of the following is the best action to take?

a. Discharge home on azithromycin and follow up aer 24 hours


b. Discharge home on Levofloxacin and follow up aer 24 hours
c. Admission to the hospital and start intravenous ceriaxone
d. Admission to the ICU and start intravenous antibiotics
e. Discharge home on paracetamol without antibiotics

‫اﻹﺟﺎﺑﺔ ﻋﲆ اﻟﺼﻔﺤﺔ اﻟﺘﺎﻟﻴﺔ‬

Page - 2139
Internal Medicine - Pulmonology - Pneumonia

Question 135/144

Question #135

A 22-year-old male presents with community-acquired pneumonia. He is conscious-oriented and


alert and doesn’t use any medication. His temperature is 39.9 °C, his blood pressure is 91/62, and his
respiratory rate is 23 per minute. His lab investigations show hemoglobin of 14 g/dL, WBC of 25,000,
and normal kidney and liver function tests. Which of the following is the best action to take?

a. Discharge home on azithromycin and follow up aer 24 hours √


b. Discharge home on Levofloxacin and follow up aer 24 hours
c. Admission to the hospital and start intravenous ceriaxone
d. Admission to the ICU and start intravenous antibiotics
e. Discharge home on paracetamol without antibiotics

Description

According to the CURB 65 score, this patient is not indicated to be admitted to the hospital.

Because the patient wasn’t using antibiotics in the last 3 months, macrolides are the first-line
treatment

Respiratory fluoroquinolones are indicated if the patient was using antibiotics in the last 3 months.

e choice of antibiotic in typical pneumonia treatment:

Outpatient cases:
If previously healthy with no antibiotic use in the past 3 months, use macrolides
(azithromycin or clarithromycin) or Doxycycline
If there are comorbidities or use of antibiotics in the past 3 months, use respiratory
fluoroquinolones (Levofloxacin or Moxifloxacin)
Inpatient cases:
Respiratory fluoroquinolone (Levofloxacin or Moxifloxacin) or
Ceriaxone with Azithromycin

e following table shows the CURB65 criteria for pneumonia:

Page - 2140
Page - 2141
Internal Medicine - Pulmonology

Question 136/144

Question #136

Aer returning from a holiday, a 48-year-old patient developed shortness of breath, diarrhea, cough,
and chest pain. His chest x-ray shows bilateral infiltration, and his labs show serum sodium of 125
mEq/L, Potassium of 3.2 mEq/L, but otherwise normal. What is the most likely diagnosis?

a. Pneumococcal pneumonia
b. Legionella pneumonia
c. Mycoplasma pneumonia
d. PCP
e. E. coli infection

‫اﻹﺟﺎﺑﺔ ﻋﲆ اﻟﺼﻔﺤﺔ اﻟﺘﺎﻟﻴﺔ‬

Page - 2142
Internal Medicine - Pulmonology - Pneumonia

Question 136/144

Question #136

Aer returning from a holiday, a 48-year-old patient developed shortness of breath, diarrhea, cough,
and chest pain. His chest x-ray shows bilateral infiltration, and his labs show serum sodium of 125
mEq/L, Potassium of 3.2 mEq/L, but otherwise normal. What is the most likely diagnosis?

a. Pneumococcal pneumonia
b. Legionella pneumonia √
c. Mycoplasma pneumonia
d. PCP
e. E. coli infection

Description

Diarrhea, hyponatremia, and a history of travel suggest legionella infection.

Legionella pneumonia:

Commonly in air-conditioning systems, workers


Associated with:
GI symptoms (abdominal pain and diarrhea)
CNS symptoms (e.g., headache and confusion)
Hyponatremia
Lymphopenia is a feature
Diagnosis by Urine antigen test
Treatment: Macrolides (e.g., Erythromycin)

Page - 2143
Internal Medicine - Pulmonology

Question 137/144

Question #137

A 30-year-old man presents with shortness of breath, cough, and pleuritic chest pain for a 1-week
duration. Physical examination reveals a temperature of 38.4 °C, generalized lymphadenopathy,
bibasilar rales, and multiple perianal contusions suggestive of homosexuality. Which of the
following is the best action at this time?

a. Admission and administration of TMP/SMX intravenously


b. Admission and starting Intravenous ceriaxone
c. Start Anti-TB treatment
d. Erythromycin orally and follow up in 24 hours
e. Start acetaminophen and bed rest at home

‫اﻹﺟﺎﺑﺔ ﻋﲆ اﻟﺼﻔﺤﺔ اﻟﺘﺎﻟﻴﺔ‬

Page - 2144
Internal Medicine - Pulmonology - Pneumonia

Question 137/144

Question #137

A 30-year-old man presents with shortness of breath, cough, and pleuritic chest pain for a 1-week
duration. Physical examination reveals a temperature of 38.4 °C, generalized lymphadenopathy,
bibasilar rales, and multiple perianal contusions suggestive of homosexuality. Which of the
following is the best action at this time?

a. Admission and administration of TMP/SMX intravenously √


b. Admission and starting Intravenous ceriaxone
c. Start Anti-TB treatment
d. Erythromycin orally and follow up in 24 hours
e. Start acetaminophen and bed rest at home

Description

e perianal contusions are suggestive of homosexuality and will raise the suspicion of HIV
infection. e most common cause of Pneumonia in HIV patients is PCP which is treated with
TMP/SMX

Pneumonia:

Pneumonia is defined as an infection of the lung, which can be bacterial, viral, or fungal
Low immunity, reduced protective mechanisms, and low ciliary movement are risk factors for
pneumonia
It is classified into community-acquired (CAP) or Hospital-acquired (HAP)
CAP is further classified into typical and atypical pneumonia.

Causes of pneumonia:

Pneumococcal pneumonia is the most common Typical CAP


Mycoplasma pneumonia is the most common atypical CAP
Staph aureus is the most common cause of HAP
PCP is the most common in HIV and immunosuppressed patients
Legionella is common in air conditioning system workers
Klebsiella pneumonia is common in alcoholics and DM patients
Aerobics are common in those with aspiration or poor dentation
Page - 2145
Page - 2146
Internal Medicine - Pulmonology

Question 138/144

Question #138

A 28-year-old healthy male presents with a fever of 38.9˚C, dry cough and shortness of breath,
bronchial breathing, and crepitations that are auscultated in the middle lung zone. What is the best
next step for this patient?

a. Start empiric antibiotic treatment


b. Give antibiotics only if the chest x-ray is abnormal
c. Give antibiotics if cultures are positive
d. Supportive treatment only
e. Give bronchodilators and antibiotics if PFT is abnormal

‫اﻹﺟﺎﺑﺔ ﻋﲆ اﻟﺼﻔﺤﺔ اﻟﺘﺎﻟﻴﺔ‬

Page - 2147
Internal Medicine - Pulmonology - Pneumonia

Question 138/144

Question #138

A 28-year-old healthy male presents with a fever of 38.9˚C, dry cough and shortness of breath,
bronchial breathing, and crepitations that are auscultated in the middle lung zone. What is the best
next step for this patient?

a. Start empiric antibiotic treatment √


b. Give antibiotics only if the chest x-ray is abnormal
c. Give antibiotics if cultures are positive
d. Supportive treatment only
e. Give bronchodilators and antibiotics if PFT is abnormal

Description

is is a case of atypical bacterial pneumonia. Antibiotics (macrolides are the first line) are indicated
regardless of the chest x-ray. e chest x-ray may be normal in the early stages.

Pneumonia:

Pneumonia is defined as an infection of the lung, which can be bacterial, viral, or fungal
Low immunity, reduced protective mechanisms, and low ciliary movement are risk factors for
pneumonia
It is classified into community-acquired (CAP) or Hospital-acquired (HAP)
CAP is further classified into typical and atypical pneumonia.

Causes of pneumonia:

Pneumococcal pneumonia is the most common Typical CAP


Mycoplasma pneumonia is the most common atypical CAP
Staph aureus is the most common cause of HAP
PCP is the most common in HIV and immunosuppressed patients
Legionella is common in air conditioning system workers
Klebsiella pneumonia is common in alcoholics and DM patients
Aerobics are common in those with aspiration or poor dentation

Page - 2148
Internal Medicine - Pulmonology

Question 139/144

Question #139

An obese 52-yar-old male presents with excessive daytime sleepiness, choking, and snoring at night.
His vital signs are normal except for stage 1 HTN. Which of the following is the first-line treatment
for his condition?

a. Weight loss
b. CPAP
c. BiPAP
d. Uvulopalatopharyngoplasty
e. Long-term Oxygen therapy (LTOT)

‫اﻹﺟﺎﺑﺔ ﻋﲆ اﻟﺼﻔﺤﺔ اﻟﺘﺎﻟﻴﺔ‬

Page - 2149
Internal Medicine - Pulmonology - Sleep apnea syndrome

Question 139/144

Question #139

An obese 52-yar-old male presents with excessive daytime sleepiness, choking, and snoring at night.
His vital signs are normal except for stage 1 HTN. Which of the following is the first-line treatment
for his condition?

a. Weight loss
b. CPAP √
c. BiPAP
d. Uvulopalatopharyngoplasty
e. Long-term Oxygen therapy (LTOT)

Description

is patient’s most likely diagnosis is Obstructive sleep apnea.

e first-line treatment for his condition is CPAP.

Sleep apnea syndrome:

It is defined as a repetitive period of apnea during sleep leading to distorted sleeping, snoring,
daytime sleepiness, and hypertension
A period of more than 10 seconds without breathing is considered an apneic episode
Obesity is the most common cause of obstructive sleep apnea
e clinical features include Daytime somnolence, headache, impaired memory, and Loud
snoring (usually noted by a sleep partner).
Polysomnography (Sleep study) is the most accurate test: a pulse oximeter, EEG, airflow, and
thoracoabdominal movements may be monitored during sleep.
C-PAP (continuous positive airway pressure) is the first line treatment of obstructive sleep
apnea

Page - 2150
Internal Medicine - Pulmonology

Question 140/144

Question #140

A 59-year-old male patient was diagnosed with obstructive sleep apnea (OSA). Which of the
following is most commonly expected to manifest in this disease?

a. Snoring
b. Headache
c. Esophageal reflux
d. Daytime fatigue and sleepiness
e. Choking

‫اﻹﺟﺎﺑﺔ ﻋﲆ اﻟﺼﻔﺤﺔ اﻟﺘﺎﻟﻴﺔ‬

Page - 2151
Internal Medicine - Pulmonology - Sleep apnea syndrome

Question 140/144

Question #140

A 59-year-old male patient was diagnosed with obstructive sleep apnea (OSA). Which of the
following is most commonly expected to manifest in this disease?

a. Snoring
b. Headache
c. Esophageal reflux
d. Daytime fatigue and sleepiness √
e. Choking

Description

e most common presenting symptom of obstructive sleep apnea is excessive daytime sleepiness.

Sleep apnea syndrome:

It is defined as a repetitive period of apnea during sleep leading to distorted sleeping, snoring,
daytime sleepiness, and hypertension
A period of more than 10 seconds without breathing is considered an apneic episode
Obesity is the most common cause of obstructive sleep apnea
e clinical features include Daytime somnolence, headache, impaired memory, and Loud
snoring (usually noted by a sleep partner).
Polysomnography (Sleep study) is the most accurate test: a pulse oximeter, EEG, airflow, and
thoracoabdominal movements may be monitored during sleep.
C-PAP (continuous positive airway pressure) is the first line treatment of obstructive sleep
apnea

Page - 2152
Internal Medicine - Pulmonology

Question 141/144

Question #141

A 45-year-old male presents with general fatigue and poor concentration for the last year, his wife
state that he snores loudly, and at times he looks like he is choking and stops breathing during sleep.
Which of the following is associated with this patient’s condition?

a. Carpel tunnel syndrome


b. COPD
c. Hypercalcemia
d. Hypertension
e. Vitiligo

‫اﻹﺟﺎﺑﺔ ﻋﲆ اﻟﺼﻔﺤﺔ اﻟﺘﺎﻟﻴﺔ‬

Page - 2153
Internal Medicine - Pulmonology - Sleep apnea syndrome

Question 141/144

Question #141

A 45-year-old male presents with general fatigue and poor concentration for the last year, his wife
state that he snores loudly, and at times he looks like he is choking and stops breathing during sleep.
Which of the following is associated with this patient’s condition?

a. Carpel tunnel syndrome


b. COPD
c. Hypercalcemia
d. Hypertension √
e. Vitiligo

Description

Obstructive sleep apnea is associated with HTN and may be caused by HTN. Many patients with OSA
improve when their blood pressure is controlled.

Sleep apnea syndrome:

It is defined as a repetitive period of apnea during sleep leading to distorted sleeping, snoring,
daytime sleepiness, and hypertension
A period of more than 10 seconds without breathing is considered an apneic episode
Obesity is the most common cause of obstructive sleep apnea
e clinical features include Daytime somnolence, headache, impaired memory, and Loud
snoring (usually noted by a sleep partner).
Polysomnography (Sleep study) is the most accurate test: a pulse oximeter, EEG, airflow, and
thoracoabdominal movements may be monitored during sleep.
C-PAP (continuous positive airway pressure) is the first line treatment of obstructive sleep
apnea

Page - 2154
Internal Medicine - Pulmonology

Question 142/144

Question #142

A 54-year-old male patient is treated with CPAP for obstructive sleep apnea. What is the major
problem encountered in the use of CPAP?

a. Recurrent coughing
b. Excessive daytime sleeping
c. Recurrent sneezing
d. Non-compliance
e. Dry mucous membranes

‫اﻹﺟﺎﺑﺔ ﻋﲆ اﻟﺼﻔﺤﺔ اﻟﺘﺎﻟﻴﺔ‬

Page - 2155
Internal Medicine - Pulmonology - Sleep apnea syndrome

Question 142/144

Question #142

A 54-year-old male patient is treated with CPAP for obstructive sleep apnea. What is the major
problem encountered in the use of CPAP?

a. Recurrent coughing
b. Excessive daytime sleeping
c. Recurrent sneezing
d. Non-compliance √
e. Dry mucous membranes

Description

Sneezing, rhinorrhea, and dry mucous membranes are common results of CPAP, but non-compliance
is the major problem in its use.

e patients usually complain of discomfort, claustrophobia, panic attacks, and noise from the
machine.

Sleep apnea syndrome:

It is defined as a repetitive period of apnea during sleep leading to distorted sleeping, snoring,
daytime sleepiness, and hypertension
A period of more than 10 seconds without breathing is considered an apneic episode
Obesity is the most common cause of obstructive sleep apnea
e clinical features include Daytime somnolence, headache, impaired memory, and Loud
snoring (usually noted by a sleep partner).
Polysomnography (Sleep study) is the most accurate test: a pulse oximeter, EEG, airflow, and
thoracoabdominal movements may be monitored during sleep.
C-PAP (continuous positive airway pressure) is the first line treatment of obstructive sleep
apnea

Page - 2156
Internal Medicine - Pulmonology

Question 143/144

Question #143

A 45-year-old male presents with general fatigue and poor concentration for the last year, his wife
state that he snores loudly, and at times he looks like he is choking and stops breathing during sleep.
What is the diagnostic procedure of choice for his condition?

a. Chest x-ray
b. Chest CT scan
c. Pulmonary function test
d. Polysomnography
e. Bronchoscopy

‫اﻹﺟﺎﺑﺔ ﻋﲆ اﻟﺼﻔﺤﺔ اﻟﺘﺎﻟﻴﺔ‬

Page - 2157
Internal Medicine - Pulmonology - Sleep apnea syndrome

Question 143/144

Question #143

A 45-year-old male presents with general fatigue and poor concentration for the last year, his wife
state that he snores loudly, and at times he looks like he is choking and stops breathing during sleep.
What is the diagnostic procedure of choice for his condition?

a. Chest x-ray
b. Chest CT scan
c. Pulmonary function test
d. Polysomnography √
e. Bronchoscopy

Description

is patient’s most likely diagnosis is Obstructive sleep apnea.

Polysomnography is the diagnostic test of choice for OSA.

Sleep apnea syndrome:

It is defined as a repetitive period of apnea during sleep leading to distorted sleeping, snoring,
daytime sleepiness, and hypertension
A period of more than 10 seconds without breathing is considered an apneic episode
Obesity is the most common cause of obstructive sleep apnea
e clinical features include Daytime somnolence, headache, impaired memory, and Loud
snoring (usually noted by a sleep partner).
Polysomnography (Sleep study) is the most accurate test: a pulse oximeter, EEG, airflow, and
thoracoabdominal movements may be monitored during sleep.
C-PAP (continuous positive airway pressure) is the first line treatment of obstructive sleep
apnea

Page - 2158
Internal Medicine - Pulmonology

Question 144/144

Question #144

What is the most eective treatment for OSA in a 50-year-old patient?

a. Sildenafil (Viagra)
b. CPAP
c. Weight reduction
d. Bronchodilators
e. Inhaled steroid

‫اﻹﺟﺎﺑﺔ ﻋﲆ اﻟﺼﻔﺤﺔ اﻟﺘﺎﻟﻴﺔ‬

Page - 2159
Internal Medicine - Pulmonology - Sleep apnea syndrome

Question 144/144

Question #144

What is the most eective treatment for OSA in a 50-year-old patient?

a. Sildenafil (Viagra)
b. CPAP √
c. Weight reduction
d. Bronchodilators
e. Inhaled steroid

Description

e first-line and the most eective treatment for his condition is CPAP

Sleep apnea syndrome:

It is defined as a repetitive period of apnea during sleep leading to distorted sleeping, snoring,
daytime sleepiness, and hypertension
A period of more than 10 seconds without breathing is considered an apneic episode
Obesity is the most common cause of obstructive sleep apnea
e clinical features include Daytime somnolence, headache, impaired memory, and Loud
snoring (usually noted by a sleep partner).
Polysomnography (Sleep study) is the most accurate test: a pulse oximeter, EEG, airflow, and
thoracoabdominal movements may be monitored during sleep.
C-PAP (continuous positive airway pressure) is the first line treatment of obstructive sleep
apnea

Page - 2160
Rheumatology

Page - 2161
Rheumatology

‫ﻣﻮاﺿﻴﻊ اﻷﺳﺌﻠﺔ وأﻋﺪادﻫﺎ‬

1) Behçet’s Syndrome => 2 Questions


2) Dermatomyositis => 3 Questions
3) Fibromyalgia => 3 Questions
4) Giant cell arteritis (temporal arteritis) => 6 Questions
5) Gout (podagra) => 9 Questions
6) Infective arthritis => 6 Questions
7) Polymyalgia Rheumatica (PMR) => 4 Questions
8) Polymyositis => 2 Questions
9) Pseudogout => 4 Questions
10) Rheumatoid Arthritis (RA) => 10 Questions
11) Sjögren's syndrome => 5 Questions
12) Spondyloarthropathies => 6 Questions
13) Systemic Lupus Erythematosus (SLE) => 9 Questions
14) Systemic sclerosis (SS) => 2 Questions

Page - 2162
Internal Medicine - Rheumatology

Question 1/71

Question #1

A 30-year-old female patient presents with painful ulcers in her mouth and vagina. She denies
sexual activity, and her Tzanck test is negative for herpes simplex virus. You performed a blood
laboratory workup and asked her to visit you again within 48 hours. However, she came to you on
the second visit with a new pustule at the venipuncture site. What is the most likely diagnosis?

a. Reactive arthritis
b. Syphilis infection
c. Behçet’s syndrome
d. HIV infection
e. Kawasaki’s disease

‫اﻹﺟﺎﺑﺔ ﻋﲆ اﻟﺼﻔﺤﺔ اﻟﺘﺎﻟﻴﺔ‬

Page - 2163
Internal Medicine - Rheumatology - Behçet’s Syndrome

Question 1/71

Question #1

A 30-year-old female patient presents with painful ulcers in her mouth and vagina. She denies
sexual activity, and her Tzanck test is negative for herpes simplex virus. You performed a blood
laboratory workup and asked her to visit you again within 48 hours. However, she came to you on
the second visit with a new pustule at the venipuncture site. What is the most likely diagnosis?

a. Reactive arthritis
b. Syphilis infection
c. Behçet’s syndrome √
d. HIV infection
e. Kawasaki’s disease

Description

Oral ulcers, genital ulcers, and the positive pathergy test strongly suggest the disease.

Behçet’s syndrome is a vasculitis of an unknown cause that involves venules. It is associated with
HLA B5

e clinical features of Behçet’s syndrome may include:

Oral ulcers, Genital ulcers, inflammatory arthritis


Skin: Erythema nodosum, acne
Eye: Anterior uveitis, retinal Vasculitis, iritis
Neurological: aseptic meningitis
Superior vena cava obstruction may occur (venous involvement)

Positive pathergy test is suggestive (puncture site following needle prick becomes inflamed with
small pustule forming)

Page - 2164
Internal Medicine - Rheumatology

Question 2/71

Question #2

e following signs and symptoms are associated with Behçet’s syndrome except:

a. Mouth ulcers
b. Genital ulcers
c. Positive pathergy test
d. Erythema nodosum
e. Viral meningitis

‫اﻹﺟﺎﺑﺔ ﻋﲆ اﻟﺼﻔﺤﺔ اﻟﺘﺎﻟﻴﺔ‬

Page - 2165
Internal Medicine - Rheumatology - Behçet’s Syndrome

Question 2/71

Question #2

e following signs and symptoms are associated with Behçet’s syndrome except:

a. Mouth ulcers
b. Genital ulcers
c. Positive pathergy test
d. Erythema nodosum
e. Viral meningitis √

Description

Aseptic meningitis is the manifestation here, not viral meningitis.

Behçet’s syndrome is a vasculitis of an unknown cause that involves venules. It is associated with
HLA B5

e clinical features of Behçet’s syndrome may include:

Oral ulcers, Genital ulcers, inflammatory arthritis


Skin: Erythema nodosum, acne
Eye: Anterior uveitis, retinal Vasculitis, iritis
Neurological: aseptic meningitis
Superior vena cava obstruction may occur (venous involvement)

Positive pathergy test is suggestive (puncture site following needle prick becomes inflamed with
small pustule forming)

Page - 2166
Internal Medicine - Rheumatology

Question 3/71

Question #3

A 22-year-old female presents with muscle weakness. Physical examination demonstrates the
finding shown in the picture below. What is the most likely diagnosis?

a. Scleroderma
b. Dermatomyositis
c. Polymyalgia rheumatica
d. Fibromyalgia
e. Fungal infection

Page - 2167
‫اﻹﺟﺎﺑﺔ ﻋﲆ اﻟﺼﻔﺤﺔ اﻟﺘﺎﻟﻴﺔ‬

‫‪Page - 2168‬‬
Internal Medicine - Rheumatology - Dermatomyositis

Question 3/71

Question #3

A 22-year-old female presents with muscle weakness. Physical examination demonstrates the
finding shown in the picture below. What is the most likely diagnosis?

a. Scleroderma
b. Dermatomyositis √
c. Polymyalgia rheumatica
d. Fibromyalgia
e. Fungal infection

Page - 2169
Description

is picture shows Gottron’s papule. e presence of this skin finding and muscle weakness is typical
for dermatomyositis.

Dermatomyositis is an inflammatory disorder causing symmetrical, proximal muscle weakness and


characteristic skin lesions.

Clinical findings include:

Symmetrical proximal muscle weakness


Photosensitivity (Malar involvement)
Photo distributed poikiloderma:
Shawl sign: erythema of the back and shoulders
V sign: neck and back or the neck and upper chest
Holser sign: along the lateral thigh
Heliotrope rash: edematous and purplish eyelids
Mechanic’s hand: palmer fishering and hyperkeratosis
Gottron’s papules: scaly patches at PIP and MCP joints

Lab investigations will show high CPK, positive ANA (60%), and positive Anti-Mi-2 (25%). Muscle
biopsy is the most accurate test.

Page - 2170
Internal Medicine - Rheumatology

Question 4/71

Question #4

Which of the following is mostly associated with dermatomyositis?

a. Anti-Ro
b. Anti-La
c. Anti-Mi-2
d. Anti-Jo-1
e. Anti-sm

‫اﻹﺟﺎﺑﺔ ﻋﲆ اﻟﺼﻔﺤﺔ اﻟﺘﺎﻟﻴﺔ‬

Page - 2171
Internal Medicine - Rheumatology - Dermatomyositis

Question 4/71

Question #4

Which of the following is mostly associated with dermatomyositis?

a. Anti-Ro
b. Anti-La
c. Anti-Mi-2 √
d. Anti-Jo-1
e. Anti-sm

Description

e anti-Mi-2 antibody is the most specific for dermatomyositis

Dermatomyositis is an inflammatory disorder causing symmetrical, proximal muscle weakness and


characteristic skin lesions.

Clinical findings include:

Symmetrical proximal muscle weakness


Photosensitivity (Malar involvement)
Photo distributed poikiloderma:
Shawl sign: erythema of the back and shoulders
V sign: neck and back or the neck and upper chest
Holser sign: along the lateral thigh
Heliotrope rash: edematous and purplish eyelids
Mechanic’s hand: palmer fishering and hyperkeratosis
Gottron’s papules: scaly patches at PIP and MCP joints

Lab investigations will show high CPK, positive ANA (60%), and positive Anti-Mi-2 (25%)

Muscle biopsy is the most accurate test.

Page - 2172
Internal Medicine - Rheumatology

Question 5/71

Question #5

A 55-year-old male patient presents with muscle weakness. On physical examination, you found an
erythematous scaly rash on his knuckles and an edematous right upper eyelid. In addition, his
creatine phosphokinase is elevated. What is the most likely diagnosis?

a. Psoriasis
b. Dermatomyositis
c. Polymyositis
d. Fibromyalgia
e. Atopic dermatitis

‫اﻹﺟﺎﺑﺔ ﻋﲆ اﻟﺼﻔﺤﺔ اﻟﺘﺎﻟﻴﺔ‬

Page - 2173
Internal Medicine - Rheumatology - Dermatomyositis

Question 5/71

Question #5

A 55-year-old male patient presents with muscle weakness. On physical examination, you found an
erythematous scaly rash on his knuckles and an edematous right upper eyelid. In addition, his
creatine phosphokinase is elevated. What is the most likely diagnosis?

a. Psoriasis
b. Dermatomyositis √
c. Polymyositis
d. Fibromyalgia
e. Atopic dermatitis

Description

Gottron’s papules, Heliotrope rash, high CPK, and muscle weakness strongly suggest the diagnosis
of dermatomyositis.

Dermatomyositis is an inflammatory disorder causing symmetrical, proximal muscle weakness and


characteristic skin lesions.

Clinical findings include:

Symmetrical proximal muscle weakness


Photosensitivity (Malar involvement)
Photo distributed poikiloderma:
Shawl sign: erythema of the back and shoulders
V sign: neck and back or the neck and upper chest
Holser sign: along the lateral thigh
Heliotrope rash: edematous and purplish eyelids
Mechanic’s hand: palmer fishering and hyperkeratosis
Gottron’s papules: scaly patches at PIP and MCP joints

Lab investigations will show high CPK, positive ANA (60%), and positive Anti-Mi-2 (25%)

Muscle biopsy is the most accurate test.

Page - 2174
Internal Medicine - Rheumatology

Question 6/71

Question #6

A 29-year-old female presents with generalized pain and is suspected of having fibromyalgia. Which
of the following is associated with this condition?

a. Elevated ESR and CRP


b. Positive muscle biopsy
c. Sleep disturbances
d. Convulsions
e. Anemia

‫اﻹﺟﺎﺑﺔ ﻋﲆ اﻟﺼﻔﺤﺔ اﻟﺘﺎﻟﻴﺔ‬

Page - 2175
Internal Medicine - Rheumatology - Fibromyalgia

Question 6/71

Question #6

A 29-year-old female presents with generalized pain and is suspected of having fibromyalgia. Which
of the following is associated with this condition?

a. Elevated ESR and CRP


b. Positive muscle biopsy
c. Sleep disturbances √
d. Convulsions
e. Anemia

Description

Fibromyalgia is a disease of unknown cause characterized by chronic generalized pain and


tenderness in multiple sites of the body, stiness, and sleeping disorders.

Fibromyalgia usually aects females (the females-to-males ratio is 9:1)

Treatment of this condition is achieved by the following:

Aerobic exercise (the best for functional improvement)


Cognitive therapy
Medications:
Anticonvulsant (Pregabalin)
Dual serotonin-norepinephrine reuptake inhibitor (Duloxetine, Milnacipran)
SSRI and TCA may be used

Page - 2176
Internal Medicine - Rheumatology

Question 7/71

Question #7

A 25-year-old female patient has complained of fatigue, shoulder and back pain, and insomnia for
several months. Physical examination is unremarkable except for tenderness over the costochondral
junctions, the medial aspect of the knee joint, and the trapezius muscle. Lab investigations,
including CBC, ESR, and CRP, are unremarkable. What is the most likely diagnosis?

a. Dermatomyositis
b. Polymyositis
c. Fibromyalgia
d. Polymyalgia rheumatica
e. Duchenne muscular dystrophy

‫اﻹﺟﺎﺑﺔ ﻋﲆ اﻟﺼﻔﺤﺔ اﻟﺘﺎﻟﻴﺔ‬

Page - 2177
Internal Medicine - Rheumatology - Fibromyalgia

Question 7/71

Question #7

A 25-year-old female patient has complained of fatigue, shoulder and back pain, and insomnia for
several months. Physical examination is unremarkable except for tenderness over the costochondral
junctions, the medial aspect of the knee joint, and the trapezius muscle. Lab investigations,
including CBC, ESR, and CRP, are unremarkable. What is the most likely diagnosis?

a. Dermatomyositis
b. Polymyositis
c. Fibromyalgia √
d. Polymyalgia rheumatica
e. Duchenne muscular dystrophy

Description

Fibromyalgia is a disease of unknown cause characterized by chronic generalized pain and


tenderness in multiple sites of the body, stiness, and sleeping disorders.

Fibromyalgia usually aects females (the females-to-males ratio is 9:1)

Treatment of this condition is achieved by the following:

Aerobic exercise (the best for functional improvement)


Cognitive therapy
Medications:
Anticonvulsant (Pregabalin)
Dual serotonin-norepinephrine reuptake inhibitor (Duloxetine, Milnacipran)
SSRI and TCA may be used

Page - 2178
Internal Medicine - Rheumatology

Question 8/71

Question #8

A 28-year-old female presents with 6 months history of neck, back, and shoulder pains associated
with fatigue and insomnia. Her physical examination shows ten dierent tender points in her body
but otherwise normal. Her lab investigations are unremarkable. What is the most appropriate
treatment for this patient’s condition?

a. NSAIDs
b. Opioids
c. Steroids
d. Cognitive therapy
e. Massage therapy

‫اﻹﺟﺎﺑﺔ ﻋﲆ اﻟﺼﻔﺤﺔ اﻟﺘﺎﻟﻴﺔ‬

Page - 2179
Internal Medicine - Rheumatology - Fibromyalgia

Question 8/71

Question #8

A 28-year-old female presents with 6 months history of neck, back, and shoulder pains associated
with fatigue and insomnia. Her physical examination shows ten dierent tender points in her body
but otherwise normal. Her lab investigations are unremarkable. What is the most appropriate
treatment for this patient’s condition?

a. NSAIDs
b. Opioids
c. Steroids
d. Cognitive therapy √
e. Massage therapy

Description

Cognitive therapy is the most eective in the treatment of fibromyalgia.

Fibromyalgia is a disease of unknown cause characterized by chronic generalized pain and


tenderness in multiple sites of the body, stiness, and sleeping disorders.

Fibromyalgia usually aects females (the females-to-males ratio is 9:1)

Treatment of this condition is achieved by the following:

Aerobic exercise (the best for functional improvement)


Cognitive therapy
Medications:
Anticonvulsant (Pregabalin)
Dual serotonin-norepinephrine reuptake inhibitor (Duloxetine, Milnacipran)
SSRI and TCA may be used

Page - 2180
Internal Medicine - Rheumatology

Question 9/71

Question #9

A 69-year-old female patient complains of shoulder stiness and pain and le-sided facial pain
aggravated by mastication. In addition, the show recently complained of weight loss, fever, and
fatigue. Her lab investigations show CPK of 80 mcg/L and ESR of 120 mm/h. Which one of the
following is the greatest immediate risk for this patient?

a. Hemiparesis
b. Convulsions
c. Unilateral loss of vision
d. Pulmonary embolism
e. Sudden death

‫اﻹﺟﺎﺑﺔ ﻋﲆ اﻟﺼﻔﺤﺔ اﻟﺘﺎﻟﻴﺔ‬

Page - 2181
Internal Medicine - Rheumatology - Giant cell arteritis (temporal arteritis)

Question 9/71

Question #9

A 69-year-old female patient complains of shoulder stiness and pain and le-sided facial pain
aggravated by mastication. In addition, the show recently complained of weight loss, fever, and
fatigue. Her lab investigations show CPK of 80 mcg/L and ESR of 120 mm/h. Which one of the
following is the greatest immediate risk for this patient?

a. Hemiparesis
b. Convulsions
c. Unilateral loss of vision √
d. Pulmonary embolism
e. Sudden death

Description

e patient complains of jaw claudication, shoulder stiness and pain, and markedly elevated ESR.

is is a case of temporal arteritis as a spectrum of polymyalgia rheumatica (PMR).

Patients with temporal arteritis are at considerable risk of vision loss and should receive steroids as
soon as possible.

Polymyalgia Rheumatica (PMR) usually occurs in those over 50 years of age

It may present with systemic features of fever, malaise, and weight loss.

Severe pain and stiness in the shoulder, neck, and pelvic girdle muscles are characteristic.

In PMR, ESR is typically extremely high (as high as 100 mm/h), but CPK is not elevated (no muscle
destruction).

Because temporal arteritis is strongly associated with PMR, temporal artery biopsy is important in
the workup.

Treatment: Steroid (there will be a rapid response even at low dose)

Page - 2182
Internal Medicine - Rheumatology

Question 10/71

Question #10

A 52-year-old female with a severe headache and jaw pain when she chews food. Bilateral shoulder
pain and wrist weakness. Her ESR is 82 mm/h. What is the most likely diagnosis?

a. Rheumatoid Arthritis
b. Temporal Arteritis
c. Polymyositis
d. Sarcoidosis
e. Otitis media

‫اﻹﺟﺎﺑﺔ ﻋﲆ اﻟﺼﻔﺤﺔ اﻟﺘﺎﻟﻴﺔ‬

Page - 2183
Internal Medicine - Rheumatology - Giant cell arteritis (temporal arteritis)

Question 10/71

Question #10

A 52-year-old female with a severe headache and jaw pain when she chews food. Bilateral shoulder
pain and wrist weakness. Her ESR is 82 mm/h. What is the most likely diagnosis?

a. Rheumatoid Arthritis
b. Temporal Arteritis √
c. Polymyositis
d. Sarcoidosis
e. Otitis media

Description

Headache, jaw claudication, and high ESR point toward Temporal arteritis, shoulder pain, and
muscle weakness are due to Simultaneous polymyalgia rheumatica

Page - 2184
Internal Medicine - Rheumatology

Question 11/71

Question #11

A 70-year-old woman could not brush her teeth because of arm pain and weakness. e pain is
improved with NSAIDs, and physical examination shows le-side tenderness at the scalp. Which of
the following is the most appropriate next step?

a. Increase the dose of the NSAIDs


b. Order ESR level
c. Check for RA antibody
d. Cervical spine x-ray
e. Refer her for psychiatric counseling

‫اﻹﺟﺎﺑﺔ ﻋﲆ اﻟﺼﻔﺤﺔ اﻟﺘﺎﻟﻴﺔ‬

Page - 2185
Internal Medicine - Rheumatology - Giant cell arteritis (temporal arteritis)

Question 11/71

Question #11

A 70-year-old woman could not brush her teeth because of arm pain and weakness. e pain is
improved with NSAIDs, and physical examination shows le-side tenderness at the scalp. Which of
the following is the most appropriate next step?

a. Increase the dose of the NSAIDs


b. Order ESR level √
c. Check for RA antibody
d. Cervical spine x-ray
e. Refer her for psychiatric counseling

Description

Temporal arteritis is suspected, high ESR level is sensitive to temporal arteritis

Temporal artery biopsy is the most accurate test

Page - 2186
Internal Medicine - Rheumatology

Question 12/71

Question #12

A 50-year-old female recently became unable to comb her hair due to severe shoulder pain and
stiness. She presents to you with severe unilateral headache and facial pain. On examination,
severe pain was elicited by touching her temporal area. Her labs show an ESR level of 115 mm/h.
What is the best next step in the management of this patient?

a. Temporal artery biopsy


b. Administrate high-dose steroids
c. Administrate intravenous opiate
d. Give paracetamol
e. Order brain MRI

‫اﻹﺟﺎﺑﺔ ﻋﲆ اﻟﺼﻔﺤﺔ اﻟﺘﺎﻟﻴﺔ‬

Page - 2187
Internal Medicine - Rheumatology - Giant cell arteritis (temporal arteritis)

Question 12/71

Question #12

A 50-year-old female recently became unable to comb her hair due to severe shoulder pain and
stiness. She presents to you with severe unilateral headache and facial pain. On examination,
severe pain was elicited by touching her temporal area. Her labs show an ESR level of 115 mm/h.
What is the best next step in the management of this patient?

a. Temporal artery biopsy


b. Administrate high-dose steroids √
c. Administrate intravenous opiate
d. Give paracetamol
e. Order brain MRI

Description

Although temporal artery biopsy is the most accurate test, steroids should not be delayed, and they
are the most appropriate next step.

Prednisolone should be started when you suspect temporal arteritis if ESR is elevated. However, do
not wait till you get a temporal artery biopsy.

Giant cell arteritis (temporal arteritis) is a disease that is on a spectrum of PMR.

It is characterized by severe headache, scalp tenderness, and jaw claudication

is condition may present with sudden painless vision loss (permanent or temporary)

Investigations will show an extremely high ESR and high CRP. However, the most accurate test is
temporal artery biopsy.

Temporal artery biopsy will be false negative aer 36 hours of steroid therapy. However, high-dose
steroids should not be delayed waiting for a biopsy.

Page - 2188
Internal Medicine - Rheumatology

Question 13/71

Question #13

A 60-year-old male patient complains of headache, blurry vision, and jaw pain associated with
chewing. What is the most accurate investigation?

a. ESR
b. Brain CT scan
c. Fundoscopic examination
d. Visual evoked potential
e. Temporal artery biopsy

‫اﻹﺟﺎﺑﺔ ﻋﲆ اﻟﺼﻔﺤﺔ اﻟﺘﺎﻟﻴﺔ‬

Page - 2189
Internal Medicine - Rheumatology - Giant cell arteritis (temporal arteritis)

Question 13/71

Question #13

A 60-year-old male patient complains of headache, blurry vision, and jaw pain associated with
chewing. What is the most accurate investigation?

a. ESR
b. Brain CT scan
c. Fundoscopic examination
d. Visual evoked potential
e. Temporal artery biopsy √

Description

Giant cell arteritis (temporal arteritis) is a disease that is on a spectrum of PMR.

It is characterized by severe headache, scalp tenderness, and jaw claudication

is condition may present with sudden painless vision loss (permanent or temporary)

Investigations will show an extremely high ESR and high CRP. However, the most accurate test is
temporal artery biopsy.

Temporal artery biopsy will be false negative aer 36 hours of steroid therapy. However, high-dose
steroids should not be delayed waiting for a biopsy.

Page - 2190
Internal Medicine - Rheumatology

Question 14/71

Question #14

A 59-year-old female patient presented with headaches, and a tender cordlike structure is palpable
just anterior to her ear and extending up to her lateral scalp. Her labs show an ESR of 102mm/h.
Which one of the following would be most appropriate at this point?

a. Clopidogrel
b. High-dose corticosteroids
c. NSAIDs
d. Aspirin
e. Paracetamol

‫اﻹﺟﺎﺑﺔ ﻋﲆ اﻟﺼﻔﺤﺔ اﻟﺘﺎﻟﻴﺔ‬

Page - 2191
Internal Medicine - Rheumatology - Giant cell arteritis (temporal arteritis)

Question 14/71

Question #14

A 59-year-old female patient presented with headaches, and a tender cordlike structure is palpable
just anterior to her ear and extending up to her lateral scalp. Her labs show an ESR of 102mm/h.
Which one of the following would be most appropriate at this point?

a. Clopidogrel
b. High-dose corticosteroids √
c. NSAIDs
d. Aspirin
e. Paracetamol

Description

is is a typical scenario of temporal arteritis.

High-dose steroids are indicated in temporal arteritis to prevent vision loss.

Page - 2192
Internal Medicine - Rheumatology

Question 15/71

Question #15

Aer a night of binge alcohol drinking, a 40-year-old male patient presented with sudden, painful
swelling of the big toe associated with hotness and tenderness. Joint aspiration reveals
monosodium Urate crystals (needle shape). What is the most likely diagnosis?

a. Septic arthritis
b. Rheumatoid arthritis
c. Gout
d. Pseudogout
e. Osteoarthritis

‫اﻹﺟﺎﺑﺔ ﻋﲆ اﻟﺼﻔﺤﺔ اﻟﺘﺎﻟﻴﺔ‬

Page - 2193
Internal Medicine - Rheumatology - Gout (podagra)

Question 15/71

Question #15

Aer a night of binge alcohol drinking, a 40-year-old male patient presented with sudden, painful
swelling of the big toe associated with hotness and tenderness. Joint aspiration reveals
monosodium Urate crystals (needle shape). What is the most likely diagnosis?

a. Septic arthritis
b. Rheumatoid arthritis
c. Gout √
d. Pseudogout
e. Osteoarthritis

Description

Gout is a disease of uric acid metabolism which leads to the accumulation of sodium Urate crystals
in (Joints, So tissues, and urinary tract stones)

e presence of negative birefringent needle-shaped sodium urate crystals in the joint aspiration is
the most accurate test to confirm the disease.

Page - 2194
Internal Medicine - Rheumatology

Question 16/71

Question #16

A 66-year-old male patient develops sudden onset pain, redness, and swelling of his right knee joint.
His medical history is significant for heart failure, for which he has received treatment for the past 2
years. What is the most appropriate method for confirming the diagnosis of his painful knee?

a. Joint aspiration for microscopy


b. Joint aspiration for culture and sensitivity
c. Serum urate level
d. ESR and CRP
e. Knee x-ray

‫اﻹﺟﺎﺑﺔ ﻋﲆ اﻟﺼﻔﺤﺔ اﻟﺘﺎﻟﻴﺔ‬

Page - 2195
Internal Medicine - Rheumatology - Gout (podagra)

Question 16/71

Question #16

A 66-year-old male patient develops sudden onset pain, redness, and swelling of his right knee joint.
His medical history is significant for heart failure, for which he has received treatment for the past 2
years. What is the most appropriate method for confirming the diagnosis of his painful knee?

a. Joint aspiration for microscopy √


b. Joint aspiration for culture and sensitivity
c. Serum urate level
d. ESR and CRP
e. Knee x-ray

Description

e presence of heart failure for which the patient should receive furosemide) will raise the
suspicion of acute gouty arthritis due to the side eect of loop diuretics.

Joint aspiration is the best diagnostic test; it will show urate crystals under the microscope.

Synovial fluid culture is better when suspected infective arthritis

Serum uric acid level is not always elevated in acute gouty arthritis

Despite high sensitivity, ESR and CRP are not specific for gouty arthritis

Page - 2196
Internal Medicine - Rheumatology

Question 17/71

Question #17

A 39-year-old alcoholic male patient develops right first toe pain and swelling for 5 days. On
examination, hotness, redness, and tenderness are present. What is the most appropriate
investigation at this time?

a. Joint x-ray
b. Joint aspiration
c. ESR and CRP
d. Kidney function test
e. Joint CT scan

‫اﻹﺟﺎﺑﺔ ﻋﲆ اﻟﺼﻔﺤﺔ اﻟﺘﺎﻟﻴﺔ‬

Page - 2197
Internal Medicine - Rheumatology - Gout (podagra)

Question 17/71

Question #17

A 39-year-old alcoholic male patient develops right first toe pain and swelling for 5 days. On
examination, hotness, redness, and tenderness are present. What is the most appropriate
investigation at this time?

a. Joint x-ray
b. Joint aspiration √
c. ESR and CRP
d. Kidney function test
e. Joint CT scan

Description

is patient has gout.

Gout is a disease of uric acid metabolism which leads to the accumulation of sodium Urate crystals
in (Joints, So tissues, and urinary tract stones)

e presence of negative birefringent needle-shaped sodium urate crystals in the joint aspiration is
the most accurate test to confirm the disease.

Page - 2198
Internal Medicine - Rheumatology

Question 18/71

Question #18

A 66-year-old male patient develops sudden onset pain, redness, and swelling of his right knee joint.
His medical history is significant for heart failure, HTN, DM, and dyslipidemia, for which he has
received treatment for the past 2 years. You performed joint aspiration and found urate crystals.
Which of the following is the most likely cause of his acute gouty arthritis?

a. Simvastatin
b. Metformin
c. Aspirin
d. Furosemide
e. Losartan

‫اﻹﺟﺎﺑﺔ ﻋﲆ اﻟﺼﻔﺤﺔ اﻟﺘﺎﻟﻴﺔ‬

Page - 2199
Internal Medicine - Rheumatology - Gout (podagra)

Question 18/71

Question #18

A 66-year-old male patient develops sudden onset pain, redness, and swelling of his right knee joint.
His medical history is significant for heart failure, HTN, DM, and dyslipidemia, for which he has
received treatment for the past 2 years. You performed joint aspiration and found urate crystals.
Which of the following is the most likely cause of his acute gouty arthritis?

a. Simvastatin
b. Metformin
c. Aspirin
d. Furosemide √
e. Losartan

Description

Furosemide may raise uric acid levels and precipitate gout

Among the mentioned medications, it is the most likely cause of this patient’s condition.

Page - 2200
Internal Medicine - Rheumatology

Question 19/71

Question #19

A 72-year-old male patient with a history of HTN, DM, and gout presents to you for follow-up and
evaluation. Which medication is the most appropriate to use in his scenario?

a. Enalapril
b. Losartan
c. Valsartan
d. Amlodipine
e. Hydrochlorothiazide

‫اﻹﺟﺎﺑﺔ ﻋﲆ اﻟﺼﻔﺤﺔ اﻟﺘﺎﻟﻴﺔ‬

Page - 2201
Internal Medicine - Rheumatology - Gout (podagra)

Question 19/71

Question #19

A 72-year-old male patient with a history of HTN, DM, and gout presents to you for follow-up and
evaluation. Which medication is the most appropriate to use in his scenario?

a. Enalapril
b. Losartan √
c. Valsartan
d. Amlodipine
e. Hydrochlorothiazide

Description

ACE inhibitors or ARBs are the most appropriate antihypertensive medication in the presence of DM.

Among ARBs, Losartan is the first-line treatment of HTN in patients with gout.

Losartan will be beneficial as a nephroprotective agent and control HTN without exacerbating gout.

iazide diuretics will increase uric acid and worsen Gout symptoms.

Page - 2202
Internal Medicine - Rheumatology

Question 20/71

Question #20

A 41-year-old male patient presents with le metatarsophalangeal joint pain and tenderness. e x-
ray is shown below. What is the most appropriate treatment?

a. Allopurinol
b. Febuxostat
c. Colchicine
d. Prednisolone
e. Naproxen

‫اﻹﺟﺎﺑﺔ ﻋﲆ اﻟﺼﻔﺤﺔ اﻟﺘﺎﻟﻴﺔ‬

Page - 2203
Internal Medicine - Rheumatology - Gout (podagra)

Question 20/71

Question #20

A 41-year-old male patient presents with le metatarsophalangeal joint pain and tenderness. e x-
ray is shown below. What is the most appropriate treatment?

a. Allopurinol
b. Febuxostat
c. Colchicine
d. Prednisolone
e. Naproxen √

Description

e clinical scenario and the imaging are typical for acute gouty arthritis.

e image above shows punched-out erosion.

NSAIDs are the first-line treatment of acute gouty arthritis

In the critical presentation of gout, the treatment is as the following:

NSAIDs (better than colchicine) – the first line


Colchicine (if NSAIDs or steroids cannot be used)
Corticosteroids (if no NSAIDs are ineective or contraindicated)
If the patient is taking allopurinol, do not stop it.

Page - 2204
Internal Medicine - Rheumatology

Question 21/71

Question #21

A 39-year-old alcoholic male patient develops right first toe pain and swelling for 5 days. On
examination, hotness, redness, and tenderness are present. What is the first-line treatment for this
patient’s condition?

a. Allopurinol
b. Febuxostat
c. Aspirin
d. Naproxen
e. Corticosteroids

‫اﻹﺟﺎﺑﺔ ﻋﲆ اﻟﺼﻔﺤﺔ اﻟﺘﺎﻟﻴﺔ‬

Page - 2205
Internal Medicine - Rheumatology - Gout (podagra)

Question 21/71

Question #21

A 39-year-old alcoholic male patient develops right first toe pain and swelling for 5 days. On
examination, hotness, redness, and tenderness are present. What is the first-line treatment for this
patient’s condition?

a. Allopurinol
b. Febuxostat
c. Aspirin
d. Naproxen √
e. Corticosteroids

Description

is patient has gout.

Gout is a disease of uric acid metabolism which leads to the accumulation of sodium Urate crystals
in (Joints, So tissues, and urinary tract stones)

e presence of negative birefringent needle-shaped sodium urate crystals in the joint aspiration is
the most accurate test to confirm the disease.

Page - 2206
Internal Medicine - Rheumatology

Question 22/71

Question #22

A 54-year-old male patient has a history of gout and recurrent acute gouty arthritis and is on
allopurinol. He told you that he is not compliant with his medications because of severe diarrhea
caused by this drug. What is the most appropriate treatment to lower his uric acid level?

a. Colchicine
b. Prednisolone
c. Febuxostat
d. Naproxen
e. iazide diuretics

‫اﻹﺟﺎﺑﺔ ﻋﲆ اﻟﺼﻔﺤﺔ اﻟﺘﺎﻟﻴﺔ‬

Page - 2207
Internal Medicine - Rheumatology - Gout (podagra)

Question 22/71

Question #22

A 54-year-old male patient has a history of gout and recurrent acute gouty arthritis and is on
allopurinol. He told you that he is not compliant with his medications because of severe diarrhea
caused by this drug. What is the most appropriate treatment to lower his uric acid level?

a. Colchicine
b. Prednisolone
c. Febuxostat √
d. Naproxen
e. iazide diuretics

Description

Drug treatments for gout:

Allopurinol (inhibits the formation of Uric acid)


Febuxostat (used when the patient cannot tolerate allopurinol)
Colchicine
Always stop thiazide because it exacerbates the condition.

Page - 2208
Internal Medicine - Rheumatology

Question 23/71

Question #23

A 66-year-old male patient complains of sudden, painful ankle joint swelling. Joint aspiration
demonstrates monosodium urate crystals. What is the most likely diagnosis?

a. Septic arthritis
b. Rheumatoid arthritis
c. Gout
d. Pseudogout
e. Osteoarthritis

‫اﻹﺟﺎﺑﺔ ﻋﲆ اﻟﺼﻔﺤﺔ اﻟﺘﺎﻟﻴﺔ‬

Page - 2209
Internal Medicine - Rheumatology - Gout (podagra)

Question 23/71

Question #23

A 66-year-old male patient complains of sudden, painful ankle joint swelling. Joint aspiration
demonstrates monosodium urate crystals. What is the most likely diagnosis?

a. Septic arthritis
b. Rheumatoid arthritis
c. Gout √
d. Pseudogout
e. Osteoarthritis

Description

Gout is a disease of uric acid metabolism which leads to the accumulation of sodium Urate crystals
in (Joints, So tissues, and urinary tract stones)

e presence of negative birefringent needle-shaped sodium urate crystals in the joint aspiration is
the most accurate test to confirm the disease.

Page - 2210
Internal Medicine - Rheumatology

Question 24/71

Question #24

A 28-year-old male patient complains of knee pain and fever. Which of the following, if negative, has
the highest negative predictive value of septic arthritis?

a. Blood culture
b. ESR and CRP
c. Synovial fluid culture
d. Synovial fluid gram staining
e. Synovial fluid WBC

‫اﻹﺟﺎﺑﺔ ﻋﲆ اﻟﺼﻔﺤﺔ اﻟﺘﺎﻟﻴﺔ‬

Page - 2211
Internal Medicine - Rheumatology - Infective arthritis

Question 24/71

Question #24

A 28-year-old male patient complains of knee pain and fever. Which of the following, if negative, has
the highest negative predictive value of septic arthritis?

a. Blood culture
b. ESR and CRP √
c. Synovial fluid culture
d. Synovial fluid gram staining
e. Synovial fluid WBC

Description

e negative predictive value of both CRP and ESR together is almost 90%.

e negative predictive value means the percentage of the cases giving non-false negative test
results.

Negative predictive value = true negative / (total negative results)

See the following table:

Page - 2212
Internal Medicine - Rheumatology

Question 25/71

Question #25

A 20-year-old sexually active female develops right-side knee, ankle, and le wrist swellings.
Physical examination demonstrates petechial rashes on both her legs. On x-ray, knee joint eusion
is noted. However, her RA antibody test is negative. What is the most likely diagnosis?

a. Psoriatic arthritis
b. Rheumatoid arthritis
c. Reactive arthritis
d. Gonococcal arthritis
e. Systemic lupus erythematosus

‫اﻹﺟﺎﺑﺔ ﻋﲆ اﻟﺼﻔﺤﺔ اﻟﺘﺎﻟﻴﺔ‬

Page - 2213
Internal Medicine - Rheumatology - Infective arthritis

Question 25/71

Question #25

A 20-year-old sexually active female develops right-side knee, ankle, and le wrist swellings.
Physical examination demonstrates petechial rashes on both her legs. On x-ray, knee joint eusion
is noted. However, her RA antibody test is negative. What is the most likely diagnosis?

a. Psoriatic arthritis
b. Rheumatoid arthritis
c. Reactive arthritis
d. Gonococcal arthritis √
e. Systemic lupus erythematosus

Description

Gonococcal arthritis is one of the common causes of septic arthritis in a previously sexually active
patient

Blood cultures are 40% positive; culture may be positive from genitalia, throat, and rectum

e dierence in presentation from septic arthritis; Gonococcal arthritis will have polyarticular
involvement, tenosynovitis, and petechial rash.

Treatment is achieved by ceriaxone 1g IV for 2 days, then ciprofloxacin 500mg PO BID for 7 days

Page - 2214
Internal Medicine - Rheumatology

Question 26/71

Question #26

A 43-year-old female patient has complained of acute hot, red, swollen right knee associated with
fever and general weakness. In addition, there is a reduced active and passive range of motion in
that joint. What is the most appropriate investigation at this time?

a. Knee two-view x-ray


b. Joint aspirate with culture and sensitivity
c. Knee ultrasound
d. Knee MRI
e. ESR and CRP

‫اﻹﺟﺎﺑﺔ ﻋﲆ اﻟﺼﻔﺤﺔ اﻟﺘﺎﻟﻴﺔ‬

Page - 2215
Internal Medicine - Rheumatology - Infective arthritis

Question 26/71

Question #26

A 43-year-old female patient has complained of acute hot, red, swollen right knee associated with
fever and general weakness. In addition, there is a reduced active and passive range of motion in
that joint. What is the most appropriate investigation at this time?

a. Knee two-view x-ray


b. Joint aspirate with culture and sensitivity √
c. Knee ultrasound
d. Knee MRI
e. ESR and CRP

Description

is patient most likely suers from septic arthritis.

Joint aspiration with cytology and culture is the most accurate test to confirm the diagnosis,
determine the infective organism, and determine the appropriate antibiotics that can be used
eectively.

Page - 2216
Internal Medicine - Rheumatology

Question 27/71

Question #27

A 42-year-old male patient presents with a painful Knee joint. In addition, the patient mentioned a
history of trauma at the same joint 2 weeks ago. On examination, the joint is swollen, hot, red, and
tender, and his temperature is 38.9 °C. You perform arthrocentesis, and the results are pending.
Which of the following is most likely to appear in the synovial fluid culture?

a. E. coli
b. Staphylococcus aureus
c. Streptococcus pneumonia
d. Pseudomonas
e. Klebsiella

‫اﻹﺟﺎﺑﺔ ﻋﲆ اﻟﺼﻔﺤﺔ اﻟﺘﺎﻟﻴﺔ‬

Page - 2217
Internal Medicine - Rheumatology - Infective arthritis

Question 27/71

Question #27

A 42-year-old male patient presents with a painful Knee joint. In addition, the patient mentioned a
history of trauma at the same joint 2 weeks ago. On examination, the joint is swollen, hot, red, and
tender, and his temperature is 38.9 °C. You perform arthrocentesis, and the results are pending.
Which of the following is most likely to appear in the synovial fluid culture?

a. E. coli
b. Staphylococcus aureus √
c. Streptococcus pneumonia
d. Pseudomonas
e. Klebsiella

Description

e causes of septic arthritis are as the following:

e most common organism is Staphylococcus aureus (40%)


Others: (streptococci, E. coli, Pseudomonas)
e most common cause of septic arthritis in a recently placed artificial joint is Staphylococcus
epidermidis.

Page - 2218
Internal Medicine - Rheumatology

Question 28/71

Question #28

A 42-year-old male patient presents with a painful elbow joint. In addition, the patient mentioned a
history of trauma at the same joint 2 weeks ago. On examination, the joint is swollen, hot, red, and
tender, and his temperature is 38.9 °C. Which of the following is the most accurate test in this
scenario?

a. Blood culture
b. Arthrocentesis
c. CT scan of the aected joint
d. X-ray of the aected joint
e. ESR and CRP

‫اﻹﺟﺎﺑﺔ ﻋﲆ اﻟﺼﻔﺤﺔ اﻟﺘﺎﻟﻴﺔ‬

Page - 2219
Internal Medicine - Rheumatology - Infective arthritis

Question 28/71

Question #28

A 42-year-old male patient presents with a painful elbow joint. In addition, the patient mentioned a
history of trauma at the same joint 2 weeks ago. On examination, the joint is swollen, hot, red, and
tender, and his temperature is 38.9 °C. Which of the following is the most accurate test in this
scenario?

a. Blood culture
b. Arthrocentesis √
c. CT scan of the aected joint
d. X-ray of the aected joint
e. ESR and CRP

Description

is patient most likely suers from septic arthritis.

Joint aspiration with cytology and culture is the most accurate test to confirm the diagnosis,
determine the infective organism, and determine the appropriate antibiotics that can be used
eectively.

Page - 2220
Internal Medicine - Rheumatology

Question 29/71

Question #29

A 42-year-old male patient presents with a painful Knee joint. In addition, the patient mentioned a
history of trauma at the same joint 2 weeks ago. On examination, the joint is swollen, hot, red, and
tender, and his temperature is 38.9 °C. What are the most appropriate empirical antibiotics to use
for this patient?

a. Flucloxacillin
b. Amoxicillin
c. Azithromycin
d. Ampicillin
e. Rifampicin

‫اﻹﺟﺎﺑﺔ ﻋﲆ اﻟﺼﻔﺤﺔ اﻟﺘﺎﻟﻴﺔ‬

Page - 2221
Internal Medicine - Rheumatology - Infective arthritis

Question 29/71

Question #29

A 42-year-old male patient presents with a painful Knee joint. In addition, the patient mentioned a
history of trauma at the same joint 2 weeks ago. On examination, the joint is swollen, hot, red, and
tender, and his temperature is 38.9 °C. What are the most appropriate empirical antibiotics to use
for this patient?

a. Flucloxacillin √
b. Amoxicillin
c. Azithromycin
d. Ampicillin
e. Rifampicin

Description

Antibiotics treatment in septic arthritis:

Flucloxacillin for 4 to 6 weeks


If penicillin-allergic patients: use clindamycin
If gonococcal arthritis: use cefotaxime or ceriaxone
If the infection is not responding to antibiotics: perform repeated percutaneous aspiration

In general, intravenous antibiotics are used for 7 days until the swelling subsides and the blood
cultures become negative. is is followed by a 4-week course of oral antibiotics.

Page - 2222
Internal Medicine - Rheumatology

Question 30/71

Question #30

A 53-year-old male patient complains of fever, malaise, and weight loss. In addition, he was recently
complaining of sti, painful shoulders and neck. His lab investigations show hemoglobin of 11g/dL,
MCV of 85 fl, and ESR of 102 mm/h. what is the most likely diagnosis?

a. Polyarteritis nodosa
b. Polymyalgia rheumatica
c. Granulomatosis with polyangiitis
d. Churg Strauss syndrome
e. Behçet’s syndrome

‫اﻹﺟﺎﺑﺔ ﻋﲆ اﻟﺼﻔﺤﺔ اﻟﺘﺎﻟﻴﺔ‬

Page - 2223
Internal Medicine - Rheumatology - Polymyalgia Rheumatica (PMR)

Question 30/71

Question #30

A 53-year-old male patient complains of fever, malaise, and weight loss. In addition, he was recently
complaining of sti, painful shoulders and neck. His lab investigations show hemoglobin of 11g/dL,
MCV of 85 fl, and ESR of 102 mm/h. what is the most likely diagnosis?

a. Polyarteritis nodosa
b. Polymyalgia rheumatica √
c. Granulomatosis with polyangiitis
d. Churg Strauss syndrome
e. Behçet’s syndrome

Description

Polymyalgia Rheumatica (PMR) usually occurs in those over 50 years of age

It may present with systemic features of fever, malaise, and weight loss.

Severe pain and stiness in the shoulder, neck and pelvic girdle muscles are characteristic.

In PMR, ESR is typically extremely high (as high as 100 mm/h), but CPK is not elevated (no muscle
destruction).

Because temporal arteritis is strongly associated with PMR, temporal artery biopsy is important in
the workup.

Treatment: Steroid (there will be a rapid response even at low dose)

Page - 2224
Internal Medicine - Rheumatology

Question 31/71

Question #31

A 66-year-old male patient complains of diculty getting out of a chair because of pain and
morning stiness in his shoulders and proximal arms. His ESR is extremely high. What is the most
likely diagnosis?

a. Polyarteritis nodosa
b. Polymyalgia rheumatica
c. Granulomatosis with polyangiitis
d. Churg Strauss syndrome
e. Behçet’s syndrome

‫اﻹﺟﺎﺑﺔ ﻋﲆ اﻟﺼﻔﺤﺔ اﻟﺘﺎﻟﻴﺔ‬

Page - 2225
Internal Medicine - Rheumatology - Polymyalgia Rheumatica (PMR)

Question 31/71

Question #31

A 66-year-old male patient complains of diculty getting out of a chair because of pain and
morning stiness in his shoulders and proximal arms. His ESR is extremely high. What is the most
likely diagnosis?

a. Polyarteritis nodosa
b. Polymyalgia rheumatica √
c. Granulomatosis with polyangiitis
d. Churg Strauss syndrome
e. Behçet’s syndrome

Description

Polymyalgia Rheumatica (PMR) usually occurs in those over 50 years of age

It may present with systemic features of fever, malaise, and weight loss.

Severe pain and stiness in the shoulders, neck, and pelvic girdle muscles are characteristic.

In PMR, ESR is typically extremely high (as high as 100 mm/h), but CPK is not elevated (no muscle
destruction).

Because temporal arteritis is strongly associated with PMR, temporal artery biopsy is important in
the workup.

Treatment: Steroid (there will be a rapid response even at low dose)

Page - 2226
Internal Medicine - Rheumatology

Question 32/71

Question #32

Which of the following condition is associated with polymyalgia rheumatica?

a. Temporal arteritis
b. Takayasu’s arteritis
c. Behçet’s disease
d. Polyarteritis nodosa
e. Fibromyalgia

‫اﻹﺟﺎﺑﺔ ﻋﲆ اﻟﺼﻔﺤﺔ اﻟﺘﺎﻟﻴﺔ‬

Page - 2227
Internal Medicine - Rheumatology - Polymyalgia Rheumatica (PMR)

Question 32/71

Question #32

Which of the following condition is associated with polymyalgia rheumatica?

a. Temporal arteritis √
b. Takayasu’s arteritis
c. Behçet’s disease
d. Polyarteritis nodosa
e. Fibromyalgia

Description

Giant cell arteritis (temporal arteritis) is on a spectrum of polymyalgia rheumatica (PMR).

It is characterized by severe headache and tenderness and may be complicated by blindness

Page - 2228
Internal Medicine - Rheumatology

Question 33/71

Question #33

A 66-year-old male patient is suspected of having polymyalgia rheumatica (PMR). Which of the
following is necessary to make this diagnosis?

a. Joint swelling and redness


b. Early morning stiness
c. Positive response to the NSAIDs
d. Extremely high ESR
e. Bilateral shoulder pain and stiness

‫اﻹﺟﺎﺑﺔ ﻋﲆ اﻟﺼﻔﺤﺔ اﻟﺘﺎﻟﻴﺔ‬

Page - 2229
Internal Medicine - Rheumatology - Polymyalgia Rheumatica (PMR)

Question 33/71

Question #33

A 66-year-old male patient is suspected of having polymyalgia rheumatica (PMR). Which of the
following is necessary to make this diagnosis?

a. Joint swelling and redness


b. Early morning stiness
c. Positive response to the NSAIDs
d. Extremely high ESR
e. Bilateral shoulder pain and stiness √

Description

To diagnose PMR, at least one 1-month of shoulder or hip stiness and pain should present.

Polymyalgia Rheumatica (PMR) usually occurs in those over 50 years of age

It may present with systemic features of fever, malaise, and weight loss.

Severe pain and stiness in the shoulders, neck, and pelvic girdle muscles are characteristic.

In PMR, ESR is typically extremely high (as high as 100 mm/h), but CPK is not elevated (no muscle
destruction).

Because temporal arteritis is strongly associated with PMR, temporal artery biopsy is important in
the workup.

Treatment: Steroid (there will be a rapid response even at low dose)

Page - 2230
Internal Medicine - Rheumatology

Question 34/71

Question #34

A 40-year-old male patient is unable to get up from a chair due to proximal muscle weakness. He
denies visual symptoms and informs you his symptoms improved aer using steroids. However, lab
investigations demonstrate elevated creatine kinase and high CRP. What is the most likely
diagnosis?

a. Myasthenia gravis disease


b. Amyotrophic lateral sclerosis
c. Multiple sclerosis
d. Cerebrovascular accident
e. Polymyositis

‫اﻹﺟﺎﺑﺔ ﻋﲆ اﻟﺼﻔﺤﺔ اﻟﺘﺎﻟﻴﺔ‬

Page - 2231
Internal Medicine - Rheumatology - Polymyositis

Question 34/71

Question #34

A 40-year-old male patient is unable to get up from a chair due to proximal muscle weakness. He
denies visual symptoms and informs you his symptoms improved aer using steroids. However, lab
investigations demonstrate elevated creatine kinase and high CRP. What is the most likely
diagnosis?

a. Myasthenia gravis disease


b. Amyotrophic lateral sclerosis
c. Multiple sclerosis
d. Cerebrovascular accident
e. Polymyositis √

Description

Polymyositis is an inflammatory disorder with symmetrical muscle weakness. However, there is no


skin involvement like dermatomyositis.

Usually aects middle-aged females; the females-to-males ratio is 3:1

Clinical and laboratory manifestations may include the following:

Weak and tender proximal muscles


Dysphagia
Respiratory muscle involvement
Elevated muscle enzymes (CPK, AST, LDH)
Anti-synthetase antibodies (anti-Jo-1 antibodies) are positive
No skin involvement

Page - 2232
Internal Medicine - Rheumatology

Question 35/71

Question #35

A 60-year-old male patient complains of dysphagia, muscle weakness, and inability to climb stairs.
His lab investigations demonstrate high CPK, AST, and LDH but normal alkaline phosphatase; the
Anti-Jo-1 antibody is positive. What is the most likely diagnosis?

a. Polymyositis
b. Polymyalgia rheumatica
c. Duchenne muscle dystrophy
d. Osteoarthritis
e. Liver disease

‫اﻹﺟﺎﺑﺔ ﻋﲆ اﻟﺼﻔﺤﺔ اﻟﺘﺎﻟﻴﺔ‬

Page - 2233
Internal Medicine - Rheumatology - Polymyositis

Question 35/71

Question #35

A 60-year-old male patient complains of dysphagia, muscle weakness, and inability to climb stairs.
His lab investigations demonstrate high CPK, AST, and LDH but normal alkaline phosphatase; the
Anti-Jo-1 antibody is positive. What is the most likely diagnosis?

a. Polymyositis √
b. Polymyalgia rheumatica
c. Duchenne muscle dystrophy
d. Osteoarthritis
e. Liver disease

Description

Polymyositis is an inflammatory disorder with symmetrical muscle weakness but no skin


involvement, like dermatomyositis.

Usually aects middle-aged females; the females-to-males ratio is 3:1

Clinical and laboratory manifestations may include the following:

Weak and tender proximal muscles


Dysphagia
Respiratory muscle involvement
Elevated muscle enzymes (CPK, AST, LDH)
Anti-synthetase antibodies (anti-Jo-1 antibodies) are positive
No skin involvement

Page - 2234
Internal Medicine - Rheumatology

Question 36/71

Question #36

A 54-year-old male patient complains of a red and inflamed right knee. His arthrocentesis result
shows rhomboid positively birefringent crystals. What is the most appropriate treatment for his
condition?

a. Indomethacin
b. Colchicine
c. Prednisolone
d. Allopurinol
e. Febuxostat

‫اﻹﺟﺎﺑﺔ ﻋﲆ اﻟﺼﻔﺤﺔ اﻟﺘﺎﻟﻴﺔ‬

Page - 2235
Internal Medicine - Rheumatology - Pseudogout

Question 36/71

Question #36

A 54-year-old male patient complains of a red and inflamed right knee. His arthrocentesis result
shows rhomboid positively birefringent crystals. What is the most appropriate treatment for his
condition?

a. Indomethacin √
b. Colchicine
c. Prednisolone
d. Allopurinol
e. Febuxostat

Description

NSAIDs are the first-line treatment of Pseudogout.

Pseudogout, also known as calcium pyruvate deposition disease, is a disease of unknown cause
(despite the association with hemochromatosis, hyperparathyroidism, hypothyroidism, and true
gout in some patients)

Synovial fluid aspiration will show positive birefringent cuboidal calcium pyrophosphate crystals
and bloodstained or turbid aspirated fluid.

Joint X-Ray usually shows chondrocalcinosis due to calcium deposition in the cartilage.

Treatment is achieved by the following measures:

NSAIDs (best initial treatment)


Intraarticular steroid (if NSAID’s resistant)
Joint aspiration to relieve swelling and pain
Colchicine for prophylaxis

Page - 2236
Internal Medicine - Rheumatology

Question 37/71

Question #37

Chondrocalcinosis is a typical feature of which of the following?

a. Gout
b. Pseudogout
c. Osteomalacia
d. Osteoporosis
e. Ankylosing spondylitis

‫اﻹﺟﺎﺑﺔ ﻋﲆ اﻟﺼﻔﺤﺔ اﻟﺘﺎﻟﻴﺔ‬

Page - 2237
Internal Medicine - Rheumatology - Pseudogout

Question 37/71

Question #37

Chondrocalcinosis is a typical feature of which of the following?

a. Gout
b. Pseudogout √
c. Osteomalacia
d. Osteoporosis
e. Ankylosing spondylitis

Description

Chondrocalcinosis is a linear calcification of the articular cartilage in the joints, especially the knee
joint. It is associated with pseudogout.

e following picture shows Chondrocalcinosis:

Page - 2238
Internal Medicine - Rheumatology

Question 38/71

Question #38

A 54-year-old male patient complains of a red and inflamed right knee. His arthrocentesis result
shows rhomboid positively birefringent crystals. What is the most likely diagnosis?

a. Gout
b. Pseudogout
c. Rheumatoid arthritis
d. Septic arthritis
e. Reactive arthritis

‫اﻹﺟﺎﺑﺔ ﻋﲆ اﻟﺼﻔﺤﺔ اﻟﺘﺎﻟﻴﺔ‬

Page - 2239
Internal Medicine - Rheumatology - Pseudogout

Question 38/71

Question #38

A 54-year-old male patient complains of a red and inflamed right knee. His arthrocentesis result
shows rhomboid positively birefringent crystals. What is the most likely diagnosis?

a. Gout
b. Pseudogout √
c. Rheumatoid arthritis
d. Septic arthritis
e. Reactive arthritis

Description

Pseudogout, also known as calcium pyruvate deposition disease, is a disease of unknown cause
(despite the association with hemochromatosis, hyperparathyroidism, hypothyroidism, and true
gout in some patients)

Synovial fluid aspiration will show positive birefringent cuboidal calcium pyrophosphate crystals
and bloodstained or turbid aspirated fluid.

Joint X-Ray usually shows chondrocalcinosis due to calcium deposition in the cartilage.

Treatment is achieved by the following measures:

NSAIDs (best initial treatment)


Intraarticular steroid (if NSAID’s resistant)
Joint aspiration to relieve swelling and pain
Colchicine for prophylaxis

Page - 2240
Internal Medicine - Rheumatology

Question 39/71

Question #39

A patient complains of knee pain, redness, and hotness. His knee x-ray is shown below. What is the
most likely diagnosis?

a. Gout
b. Infective arthritis
c. Osteoarthritis
d. Pseudogout
e. Rheumatoid arthritis

‫اﻹﺟﺎﺑﺔ ﻋﲆ اﻟﺼﻔﺤﺔ اﻟﺘﺎﻟﻴﺔ‬

Page - 2241
Internal Medicine - Rheumatology - Pseudogout

Question 39/71

Question #39

A patient complains of knee pain, redness, and hotness. His knee x-ray is shown below. What is the
most likely diagnosis?

a. Gout
b. Infective arthritis
c. Osteoarthritis
d. Pseudogout √
e. Rheumatoid arthritis

Description

is picture shows a typical finding of Chondrocalcinosis.

Chondrocalcinosis is a typical fining of Pseudogout in which calcium pyrophosphate crystals deposit


in the joints.

Pseudogout, also known as calcium pyruvate deposition disease, is a disease of unknown cause
Page - 2242
(despite the association with hemochromatosis, hyperparathyroidism, hypothyroidism, and true
gout in some patients)

Synovial fluid aspiration will show positive birefringent cuboidal calcium pyrophosphate crystals
and bloodstained or turbid aspirated fluid.

Joint X-Ray usually shows chondrocalcinosis due to calcium deposition in the cartilage.

Treatment is achieved by the following measures:

NSAIDs (best initial treatment)


Intraarticular steroid (if NSAID’s resistant)
Joint aspiration to relieve swelling and pain
Colchicine for prophylaxis

Page - 2243
Internal Medicine - Rheumatology

Question 40/71

Question #40

You suspect a diagnosis of rheumatoid arthritis in a 55-year-old male patient. Which of the following
is the most distinctive with the diagnosis?

a. Large joint involvement


b. Unilateral joint involvement
c. Hepatosplenomegaly
d. Morning joint stiness
e. Maculopapular rash

‫اﻹﺟﺎﺑﺔ ﻋﲆ اﻟﺼﻔﺤﺔ اﻟﺘﺎﻟﻴﺔ‬

Page - 2244
Internal Medicine - Rheumatology - Rheumatoid Arthritis (RA)

Question 40/71

Question #40

You suspect a diagnosis of rheumatoid arthritis in a 55-year-old male patient. Which of the following
is the most distinctive with the diagnosis?

a. Large joint involvement


b. Unilateral joint involvement
c. Hepatosplenomegaly
d. Morning joint stiness √
e. Maculopapular rash

Description

Rheumatoid arthritis is an inflammatory deforming arthritis that usually aects the small joints,
typically symmetrically bilateral, more prominent in the hands, and associated with morning joint
stiness for more than 1 hour.

Page - 2245
Internal Medicine - Rheumatology

Question 41/71

Question #41

A 66-year-old male patient is treated for Rheumatoid arthritis. Which drug is hepatotoxic and needs
follow-up with a complete blood count and pulmonary function test?

a. Methotrexate
b. Prednisone
c. Rituximab
d. Sulfasalazine
e. Hydroxychloroquine

‫اﻹﺟﺎﺑﺔ ﻋﲆ اﻟﺼﻔﺤﺔ اﻟﺘﺎﻟﻴﺔ‬

Page - 2246
Internal Medicine - Rheumatology - Rheumatoid Arthritis (RA)

Question 41/71

Question #41

A 66-year-old male patient is treated for Rheumatoid arthritis. Which drug is hepatotoxic and needs
follow-up with a complete blood count and pulmonary function test?

a. Methotrexate √
b. Prednisone
c. Rituximab
d. Sulfasalazine
e. Hydroxychloroquine

Description

While preparing for your medical exam, you should identify the common adverse eects of the
drugs used to treat rheumatoid arthritis.

Methotrexate is a hepatotoxic drug that causes bone marrow suppression and pulmonary fibrosis.
So, a patient on methotrexate should be followed with regular CBC, LFT, and PFT.

e following table shows the common drugs used in rheumatoid arthritis and their possible side
eects:

Page - 2247
Internal Medicine - Rheumatology

Question 42/71

Question #42

A 60-year-old male patient with a background of rheumatoid arthritis was started on NSAIDs and
methotrexate. However, his routine follow-up laboratory results are as follows:

Hemoglobin: 10.5 g/dL

Platelet count: 95 * 10^9

WBC: 2.9 * 10^9

What is the most appropriate management?

a. Start ferrous sulfate


b. Start folic acid
c. Blood transfusion
d. Bone marrow biopsy
e. Start Folinic acid

‫اﻹﺟﺎﺑﺔ ﻋﲆ اﻟﺼﻔﺤﺔ اﻟﺘﺎﻟﻴﺔ‬

Page - 2248
Internal Medicine - Rheumatology - Rheumatoid Arthritis (RA)

Question 42/71

Question #42

A 60-year-old male patient with a background of rheumatoid arthritis was started on NSAIDs and
methotrexate. However, his routine follow-up laboratory results are as follows:

Hemoglobin: 10.5 g/dL

Platelet count: 95 * 10^9

WBC: 2.9 * 10^9

What is the most appropriate management?

a. Start ferrous sulfate


b. Start folic acid
c. Blood transfusion
d. Bone marrow biopsy
e. Start Folinic acid √

Description

e patient suers from myelosuppression secondary to her methotrexate therapy.

Folic acid is used to prevent methotrexate-associated side eects, but Folinic acid is the
recommended treatment of myelosuppression due to methotrexate therapy.

Page - 2249
Internal Medicine - Rheumatology

Question 43/71

Question #43

A heavy smoker 55-year-old female complains of a bilaterally limited range of motion in her wrists
and hands, and you suspect rheumatoid arthritis (RA). What is the most specific test for the
diagnosis of RA?

a. Antinuclear antibody titer (ANA)


b. Erythrocyte sedimentation rate (ESR)
c. Rheumatoid factor (RA)
d. Anti-smith antibody (Anti-sm)
e. Anti-cyclic citrullinated peptide (anti-CCP) antibody

‫اﻹﺟﺎﺑﺔ ﻋﲆ اﻟﺼﻔﺤﺔ اﻟﺘﺎﻟﻴﺔ‬

Page - 2250
Internal Medicine - Rheumatology - Rheumatoid Arthritis (RA)

Question 43/71

Question #43

A heavy smoker 55-year-old female complains of a bilaterally limited range of motion in her wrists
and hands, and you suspect rheumatoid arthritis (RA). What is the most specific test for the
diagnosis of RA?

a. Antinuclear antibody titer (ANA)


b. Erythrocyte sedimentation rate (ESR)
c. Rheumatoid factor (RA)
d. Anti-smith antibody (Anti-sm)
e. Anti-cyclic citrullinated peptide (anti-CCP) antibody √

Description

Anti-CCP is the most specific antibody in Rheumatoid arthritis

It has 95% specificity and 70% sensitivity.

RA and Anti-CCP antibodies are neither sucient nor mandatory to diagnose rheumatoid arthritis.

Page - 2251
Internal Medicine - Rheumatology

Question 44/71

Question #44

A 65-year-old male patient is a known case of rheumatoid arthritis. However, his medications
include Ibuprofen and methotrexate. Which of the following vitamins and minerals should the
patient receive in addition to his medications?

a. Vitamin b12
b. Iron supplement
c. Folic acid supplement
d. Zinc supplement
e. Selenium

‫اﻹﺟﺎﺑﺔ ﻋﲆ اﻟﺼﻔﺤﺔ اﻟﺘﺎﻟﻴﺔ‬

Page - 2252
Internal Medicine - Rheumatology - Rheumatoid Arthritis (RA)

Question 44/71

Question #44

A 65-year-old male patient is a known case of rheumatoid arthritis. However, his medications
include Ibuprofen and methotrexate. Which of the following vitamins and minerals should the
patient receive in addition to his medications?

a. Vitamin b12
b. Iron supplement
c. Folic acid supplement √
d. Zinc supplement
e. Selenium

Description

Methotrexate is well known to cause folic acid deficiency. erefore, all patients who receive
methotrexate for more than 3 weeks should receive a folic acid supplement to prevent hematologic
and neurologic adverse outcomes.

Vitamin B12 should be received in patients who have pernicious anemia, gastrectomy, terminal ilium
resection, or those who are vegans.

Iron supplements should be received for dietary insuciency, malabsorption, high demands, or
chronic blood loss.

Selenium is beneficial in the case of Hashimoto’s thyroiditis; it helps to enhance thyroid function.

Page - 2253
Internal Medicine - Rheumatology

Question 45/71

Question #45

Which test should be done for a patient with Rheumatoid arthritis before starting on TNF inhibitors?

a. Blood film
b. Pulmonary function test
c. Mantoux test
d. Serum albumin level
e. Kidney function test

‫اﻹﺟﺎﺑﺔ ﻋﲆ اﻟﺼﻔﺤﺔ اﻟﺘﺎﻟﻴﺔ‬

Page - 2254
Internal Medicine - Rheumatology - Rheumatoid Arthritis (RA)

Question 45/71

Question #45

Which test should be done for a patient with Rheumatoid arthritis before starting on TNF inhibitors?

a. Blood film
b. Pulmonary function test
c. Mantoux test √
d. Serum albumin level
e. Kidney function test

Description

TNF inhibitors (e.g., etanercept) are known to cause reactivation of latent TB, so the Mantoux test
should be performed before starting these drugs.

e following table shows the common drugs used in rheumatoid arthritis and their possible side
eects:

Page - 2255
Internal Medicine - Rheumatology

Question 46/71

Question #46

A 39-year-old male patient complains of 8 months history of painful tender wrists and hands.
However, he has more than 1 hour of morning stiness but no rash, fever, or skin changes. On
examination, you noted symmetrical swelling and redness in the proximal interphalangeal and
metacarpophalangeal joints. His blood tests are negative for ANA and RA antibodies, and his ESR is
elevated. What is the most likely diagnosis?

a. Rheumatoid arthritis
b. Systemic lupus erythematosus
c. Infective arthritis
d. Osteoarthritis
e. Acute gouty arthritis

‫اﻹﺟﺎﺑﺔ ﻋﲆ اﻟﺼﻔﺤﺔ اﻟﺘﺎﻟﻴﺔ‬

Page - 2256
Internal Medicine - Rheumatology - Rheumatoid Arthritis (RA)

Question 46/71

Question #46

A 39-year-old male patient complains of 8 months history of painful tender wrists and hands.
However, he has more than 1 hour of morning stiness but no rash, fever, or skin changes. On
examination, you noted symmetrical swelling and redness in the proximal interphalangeal and
metacarpophalangeal joints. His blood tests are negative for ANA and RA antibodies, and his ESR is
elevated. What is the most likely diagnosis?

a. Rheumatoid arthritis √
b. Systemic lupus erythematosus
c. Infective arthritis
d. Osteoarthritis
e. Acute gouty arthritis

Description

is patient most likely has rheumatoid arthritis.

e history of more than 6 weeks, the presence of symmetrical bilateral small joint involvement that
does not aect the distal interphalangeal joint, the presence of morning stiness of more than 1
hour, and the presence of high ESR are strongly aected, suggestive of the disease.

e following table demonstrates the criteria for the diagnosis of rheumatoid arthritis:

Page - 2257
Page - 2258
Internal Medicine - Rheumatology

Question 47/71

Question #47

A 55-year-old female is diagnosed with Rheumatoid arthritis. Which of the following is known to
delay the progression of the disease?

a. Ibuprofen
b. Intraarticular corticosteroids
c. Indomethacin
d. Hydroxychloroquine
e. Aspirin

‫اﻹﺟﺎﺑﺔ ﻋﲆ اﻟﺼﻔﺤﺔ اﻟﺘﺎﻟﻴﺔ‬

Page - 2259
Internal Medicine - Rheumatology - Rheumatoid Arthritis (RA)

Question 47/71

Question #47

A 55-year-old female is diagnosed with Rheumatoid arthritis. Which of the following is known to
delay the progression of the disease?

a. Ibuprofen
b. Intraarticular corticosteroids
c. Indomethacin
d. Hydroxychloroquine √
e. Aspirin

Description

Aspirin and Ibuprofen are known to reduce pain and the mortality rate but do not aect the
progression of the disease.

Hydroxychloroquine is a disease-modifying anti-Rheumatic agent (DMARD) that delays the


progression of rheumatoid arthritis.

Page - 2260
Internal Medicine - Rheumatology

Question 48/71

Question #48

A 60-year-old male patient with a background of rheumatoid arthritis was started on NSAIDs and
methotrexate. However, he came to you with a clinical feature of UTI. Which of the following is
contraindicated in this patient?

a. Amoxicillin
b. Ciprofloxacin
c. Nitrofurantoin
d. Trimethoprim/sulfamethoxazole
e. Ceriaxone

‫اﻹﺟﺎﺑﺔ ﻋﲆ اﻟﺼﻔﺤﺔ اﻟﺘﺎﻟﻴﺔ‬

Page - 2261
Internal Medicine - Rheumatology - Rheumatoid Arthritis (RA)

Question 48/71

Question #48

A 60-year-old male patient with a background of rheumatoid arthritis was started on NSAIDs and
methotrexate. However, he came to you with a clinical feature of UTI. Which of the following is
contraindicated in this patient?

a. Amoxicillin
b. Ciprofloxacin
c. Nitrofurantoin
d. Trimethoprim/sulfamethoxazole √
e. Ceriaxone

Description

e concomitant use of Trimethoprim and methotrexate is dangerous because of the risk of severe
myelosuppression and cytopenia that could be fatal.

Page - 2262
Internal Medicine - Rheumatology

Question 49/71

Question #49

A 66-year-old male patient with rheumatoid arthritis complains of severe joint pains in his hands
and feet. His medical history is otherwise unremarkable. What is the most appropriate drug to
control his pain?

a. Methotrexate
b. Hydroxychloroquine
c. Paracetamol
d. Ibuprofen
e. Morphine

‫اﻹﺟﺎﺑﺔ ﻋﲆ اﻟﺼﻔﺤﺔ اﻟﺘﺎﻟﻴﺔ‬

Page - 2263
Internal Medicine - Rheumatology - Rheumatoid Arthritis (RA)

Question 49/71

Question #49

A 66-year-old male patient with rheumatoid arthritis complains of severe joint pains in his hands
and feet. His medical history is otherwise unremarkable. What is the most appropriate drug to
control his pain?

a. Methotrexate
b. Hydroxychloroquine
c. Paracetamol
d. Ibuprofen √
e. Morphine

Description

e most appropriate drugs used in treating pain in Rheumatoid arthritis are NSAIDs (e.g.,
Ibuprofen), but they should never be used as a monotherapy.

Page - 2264
Internal Medicine - Rheumatology

Question 50/71

Question #50

Schirmer’s test is used in which of the following disorders?

a. Ankylosing spondylitis
b. Pernicious anemia
c. Sjögren’s syndrome
d. Myasthenia graves
e. Behçet’s syndrome

‫اﻹﺟﺎﺑﺔ ﻋﲆ اﻟﺼﻔﺤﺔ اﻟﺘﺎﻟﻴﺔ‬

Page - 2265
Internal Medicine - Rheumatology - Sjögren's syndrome

Question 50/71

Question #50

Schirmer’s test is used in which of the following disorders?

a. Ankylosing spondylitis
b. Pernicious anemia
c. Sjögren’s syndrome √
d. Myasthenia graves
e. Behçet’s syndrome

Description

In Schirmer’s test, a filter paper is placed to measure tear formation. It is used in Sjögren’s syndrome
and is considered positive if reduced teat formation is present.

e test used in ankylosing spondylitis is called Schöber’s test

e test used in pernicious anemia is called schilling test

e test used in Behçet’s syndrome is called pathergy test

e test used in myasthenia graves is called tensilon test

Page - 2266
Internal Medicine - Rheumatology

Question 51/71

Question #51

A 50-year-old female patient presents with a burning sensation in her eyes. She has a long history of
dry eyes and mouth, altered taste sensation, and dysphagia. In addition, she develops hoarseness of
voice when she talks for a long time. What is the most likely diagnosis?

a. Sjögren’s syndrome
b. Systemic lupus erythematosus
c. Hypothyroidism
d. Graves disease
e. Scleroderma

‫اﻹﺟﺎﺑﺔ ﻋﲆ اﻟﺼﻔﺤﺔ اﻟﺘﺎﻟﻴﺔ‬

Page - 2267
Internal Medicine - Rheumatology - Sjögren's syndrome

Question 51/71

Question #51

A 50-year-old female patient presents with a burning sensation in her eyes. She has a long history of
dry eyes and mouth, altered taste sensation, and dysphagia. In addition, she develops hoarseness of
voice when she talks for a long time. What is the most likely diagnosis?

a. Sjögren’s syndrome √
b. Systemic lupus erythematosus
c. Hypothyroidism
d. Graves disease
e. Scleroderma

Description

Sjögren’s syndrome is an autoimmune disorder of unknown cause resulting from autoantibodies


against lacrimal and salivary glands.

is will result in dry eyes, dry mouth, pancreatic dysfunction, and other exocrine dysfunctions

Page - 2268
Internal Medicine - Rheumatology

Question 52/71

Question #52

A 45-year-old female complains of blurry vision and dry eyes that ate now getting worse. In addition,
she mentioned diculty in swallowing and the need for water drinking to swallow food. Lab
investigations demonstrate positive SSA and SSB antibodies. e following are appropriate for the
treatment except:

a. Artificial tears
b. Pancreatic enzyme replacement
c. Pilocarpine administration
d. Artificial saliva
e. Antihistamines

‫اﻹﺟﺎﺑﺔ ﻋﲆ اﻟﺼﻔﺤﺔ اﻟﺘﺎﻟﻴﺔ‬

Page - 2269
Internal Medicine - Rheumatology - Sjögren's syndrome

Question 52/71

Question #52

A 45-year-old female complains of blurry vision and dry eyes that ate now getting worse. In addition,
she mentioned diculty in swallowing and the need for water drinking to swallow food. Lab
investigations demonstrate positive SSA and SSB antibodies. e following are appropriate for the
treatment except:

a. Artificial tears
b. Pancreatic enzyme replacement
c. Pilocarpine administration
d. Artificial saliva
e. Antihistamines √

Description

Antihistamines will cause more dryness and worsen the condition.

is is a typical scenario of Sjögren’s syndrome

Sjögren’s syndrome is an autoimmune disorder of unknown cause resulting from autoantibodies


against lacrimal and salivary glands.

is will result in dry eyes, dry mouth, pancreatic dysfunction, and other exocrine dysfunctions

Treatment of Sjögren’s syndrome is as the following:

Artificial Tears
Artificial saliva
Pilocarpine may stimulate saliva production
Pancreatic enzyme replacement

Page - 2270
Internal Medicine - Rheumatology

Question 53/71

Question #53

A 50-year-old male patient is known to have rheumatoid arthritis. He complains of dry eye and
dysphagia. In addition, his Schirmer test shows reduced tear formation. What is the most likely
diagnosis?

a. Sjögren’s syndrome
b. Systemic lupus erythematosus
c. Hypothyroidism
d. Medication side eects
e. Systemic sclerosis

‫اﻹﺟﺎﺑﺔ ﻋﲆ اﻟﺼﻔﺤﺔ اﻟﺘﺎﻟﻴﺔ‬

Page - 2271
Internal Medicine - Rheumatology - Sjögren's syndrome

Question 53/71

Question #53

A 50-year-old male patient is known to have rheumatoid arthritis. He complains of dry eye and
dysphagia. In addition, his Schirmer test shows reduced tear formation. What is the most likely
diagnosis?

a. Sjögren’s syndrome √
b. Systemic lupus erythematosus
c. Hypothyroidism
d. Medication side eects
e. Systemic sclerosis

Description

Sjögren’s syndrome is an autoimmune disorder of unknown cause resulting from autoantibodies


against lacrimal and salivary glands.

is will result in dry eyes and mouth, pancreatic dysfunction, and other exocrine dysfunctions.

Page - 2272
Internal Medicine - Rheumatology

Question 54/71

Question #54

A 45-year-old male patient complains of reduced tear and saliva production and extensive dental
caries. However, he denies taking any medication. What is the most likely diagnosis?

a. Sjögren’s syndrome
b. Systemic lupus erythematosus
c. Hypothyroidism
d. Medication side eects
e. Systemic sclerosis

‫اﻹﺟﺎﺑﺔ ﻋﲆ اﻟﺼﻔﺤﺔ اﻟﺘﺎﻟﻴﺔ‬

Page - 2273
Internal Medicine - Rheumatology - Sjögren's syndrome

Question 54/71

Question #54

A 45-year-old male patient complains of reduced tear and saliva production and extensive dental
caries. However, he denies taking any medication. What is the most likely diagnosis?

a. Sjögren’s syndrome √
b. Systemic lupus erythematosus
c. Hypothyroidism
d. Medication side eects
e. Systemic sclerosis

Description

Sjögren’s syndrome is an autoimmune disorder of unknown cause resulting from autoantibodies


against lacrimal and salivary glands.

is will result in dry eye and mouth, pancreatic dysfunction, and other exocrine dysfunctions.

Page - 2274
Internal Medicine - Rheumatology

Question 55/71

Question #55

e following disorders are seronegative Spondyloarthropathies except:

a. Ankylosing spondylitis
b. Reactive arthritis
c. Enteropathic arthritis
d. Psoriatic arthritis
e. Rheumatoid arthritis

‫اﻹﺟﺎﺑﺔ ﻋﲆ اﻟﺼﻔﺤﺔ اﻟﺘﺎﻟﻴﺔ‬

Page - 2275
Internal Medicine - Rheumatology - Spondyloarthropathies

Question 55/71

Question #55

e following disorders are seronegative Spondyloarthropathies except:

a. Ankylosing spondylitis
b. Reactive arthritis
c. Enteropathic arthritis
d. Psoriatic arthritis
e. Rheumatoid arthritis √

Description

Spondyloarthropathies (seronegative arthritis):

e gene most strongly associated with Spondyloarthropathies is HLA B27


Usually involves men under the age of 40 years

is group includes:

Ankylosing spondylitis
Reactive arthritis (Reiter’s syndrome)
Psoriatic arthritis
Enteropathic arthritis (associated with IBD)

Page - 2276
Internal Medicine - Rheumatology

Question 56/71

Question #56

A 33-year-old female has had sudden onset arthritis aer a febrile illness. On examination, a
maculopapular rash is noted on her soles of feet, she has conjunctivitis, and red, painful, and
swollen joints of her feet and knees are noted. What is the most likely diagnosis?

a. Psoriatic arthritis
b. Reactive arthritis
c. Rheumatoid arthritis
d. Septic arthritis
e. Systemic lupus erythematosus

‫اﻹﺟﺎﺑﺔ ﻋﲆ اﻟﺼﻔﺤﺔ اﻟﺘﺎﻟﻴﺔ‬

Page - 2277
Internal Medicine - Rheumatology - Spondyloarthropathies

Question 56/71

Question #56

A 33-year-old female has had sudden onset arthritis aer a febrile illness. On examination, a
maculopapular rash is noted on her soles of feet, she has conjunctivitis, and red, painful, and
swollen joints of her feet and knees are noted. What is the most likely diagnosis?

a. Psoriatic arthritis
b. Reactive arthritis √
c. Rheumatoid arthritis
d. Septic arthritis
e. Systemic lupus erythematosus

Description

Reactive arthritis is caused when a joint reacts to an infection elsewhere in the body.
May develop days or weeks aer GI or GU infection with Salmonella, Shigella, Campylobacter,
Chlamydia, or Ureaplasma urealyticum
Male to female ratio is 20:1
Associated with HLD B27 (70%)

Symptoms include:

Conjunctivitis 50%
Urethritis
Arthritis (Knee and ankle is the most commonest sites)
Keratoderma blennorrhagica 10% (psoriasiform rash on sole and feet)
Circinate balanitis (psoriasiform rash on the penis)
Systemic features: Fever, weight loss, Carditis, Aortic regurgitation

Page - 2278
Internal Medicine - Rheumatology

Question 57/71

Question #57

When compared with rheumatoid arthritis, all are correct about Spondyloarthropathies except:

a. ey aect the axial skeleton more than the small joints
b. ey cause asymmetrical arthritis
c. Rheumatoid factor is negative
d. More common in males than females
e. Steroids are the best treatment option for Spondyloarthropathies

‫اﻹﺟﺎﺑﺔ ﻋﲆ اﻟﺼﻔﺤﺔ اﻟﺘﺎﻟﻴﺔ‬

Page - 2279
Internal Medicine - Rheumatology - Spondyloarthropathies

Question 57/71

Question #57

When compared with rheumatoid arthritis, all are correct about Spondyloarthropathies except:

a. ey aect the axial skeleton more than the small joints
b. ey cause asymmetrical arthritis
c. Rheumatoid factor is negative
d. More common in males than females
e. Steroids are the best treatment option for Spondyloarthropathies √

Description

Spondyloarthropathies (seronegative arthritis):

e gene most strongly associated with Spondyloarthropathies is HLA B27


Usually involves men under the age of 40 years

is group includes:

Ankylosing spondylitis
Reactive arthritis (Reiter’s syndrome)
Psoriatic arthritis
Enteropathic arthritis (associated with IBD)

e following table demonstrates the dierences between seropositive and seronegative arthritis:

Page - 2280
Internal Medicine - Rheumatology

Question 58/71

Question #58

You suspect ankylosing spondylitis in a 25-year-old male patient. Which of the following is correct
about this condition?

a. More in women than men


b. Most commonly aects the small joints of the hands
c. Morning stiness that improves with activity
d. Schirmer test is used in the diagnosis
e. Peak incidence at the age of 70s

‫اﻹﺟﺎﺑﺔ ﻋﲆ اﻟﺼﻔﺤﺔ اﻟﺘﺎﻟﻴﺔ‬

Page - 2281
Internal Medicine - Rheumatology - Spondyloarthropathies

Question 58/71

Question #58

You suspect ankylosing spondylitis in a 25-year-old male patient. Which of the following is correct
about this condition?

a. More in women than men


b. Most commonly aects the small joints of the hands
c. Morning stiness that improves with activity √
d. Schirmer test is used in the diagnosis
e. Peak incidence at the age of 70s

Description

Ankylosing spondylitis:

Chronic inflammation aects the sacroiliac joint and spine


Most cases begin in late adolescence or early adulthood
It involves the SI joint and progress cranially without skipping any region
Associated with progressive stiening and fusion of axial skeleton
Male to female ratio is 3: 1
Associated with HLA B27 (70%)
Low Back pain worse at rest (early morning and inactivity) is the main feature

Page - 2282
Internal Medicine - Rheumatology

Question 59/71

Question #59

A 25-year-old male presents with chronic lower back pain and stiness. On further questioning, he
mentioned that his symptoms worsen in the morning or at rest. His Schober test is positive. What is
the most appropriate investigation to perform?

a. HLA B27
b. Sacroiliac x-ray
c. oracic spine x-ray
d. ESR
e. RA antibody titer

‫اﻹﺟﺎﺑﺔ ﻋﲆ اﻟﺼﻔﺤﺔ اﻟﺘﺎﻟﻴﺔ‬

Page - 2283
Internal Medicine - Rheumatology - Spondyloarthropathies

Question 59/71

Question #59

A 25-year-old male presents with chronic lower back pain and stiness. On further questioning, he
mentioned that his symptoms worsen in the morning or at rest. His Schober test is positive. What is
the most appropriate investigation to perform?

a. HLA B27
b. Sacroiliac x-ray √
c. oracic spine x-ray
d. ESR
e. RA antibody titer

Description

is is a typical presentation of ankylosing spondylitis; a sacroiliac x-ray is the best next step in the
management of this patient

ESR is not specific

HLA B27 test is not recommended nor sucient to diagnose spondyloarthropathies

Diagnosis of Ankylosing spondylitis:

ESR – high, RF - absent


Lumber X-Ray:
Maybe normal early in the disease
Sacroiliitis: subchondral erosions and sclerosis.
Square vertebrae with tramline appearance
Bamboo spine: a late finding with a fusion of vertebrae
Lumbar MRI (most accurate)

Page - 2284
Internal Medicine - Rheumatology

Question 60/71

Question #60

A 22-year-old male patient presents with urethral discharge, knee pain, le second toe pain, oral
ulcers, and intermittent diarrhea. On further questioning, the patient admits that he is bisexual. You
performed a physical examination and found conjunctival edema and erythema. What is the most
likely diagnosis?

a. Rheumatoid arthritis
b. Reactive arthritis
c. Behçet’s disease
d. Gonococcal arthritis
e. Psoriatic arthritis

‫اﻹﺟﺎﺑﺔ ﻋﲆ اﻟﺼﻔﺤﺔ اﻟﺘﺎﻟﻴﺔ‬

Page - 2285
Internal Medicine - Rheumatology - Spondyloarthropathies

Question 60/71

Question #60

A 22-year-old male patient presents with urethral discharge, knee pain, le second toe pain, oral
ulcers, and intermittent diarrhea. On further questioning, the patient admits that he is bisexual. You
performed a physical examination and found conjunctival edema and erythema. What is the most
likely diagnosis?

a. Rheumatoid arthritis
b. Reactive arthritis √
c. Behçet’s disease
d. Gonococcal arthritis
e. Psoriatic arthritis

Description

Reactive arthritis is caused when a joint reacts to an infection elsewhere in the body.
May develop days or weeks aer GI or GU infection with Salmonella, Shigella, Campylobacter,
Chlamydia, or Ureaplasma urealyticum
Male to female ratio is 20:1
Associated with HLD B27 (70%)

Symptoms include:

Conjunctivitis 50%
Urethritis
Arthritis (Knee and ankle are the most common sites)
Keratoderma blennorrhagica 10% (psoriasiform rash on sole and feet)
Circinate balanitis (psoriasiform rash on the penis)
Systemic features: Fever, weight loss, Carditis, Aortic regurgitation

Page - 2286
Internal Medicine - Rheumatology

Question 61/71

Question #61

A young lady was recently diagnosed with systemic lupus erythematosus. e following findings
indicate active disease except:

a. High ESR
b. High CRP
c. high Anti-DsDNA titer
d. low C3
e. low C4

‫اﻹﺟﺎﺑﺔ ﻋﲆ اﻟﺼﻔﺤﺔ اﻟﺘﺎﻟﻴﺔ‬

Page - 2287
Internal Medicine - Rheumatology - Systemic Lupus Erythematosus (SLE)

Question 61/71

Question #61

A young lady was recently diagnosed with systemic lupus erythematosus. e following findings
indicate active disease except:

a. High ESR
b. High CRP √
c. high Anti-DsDNA titer
d. low C3
e. low C4

Description

CRP is reduced in the case of active SLE (not elevated).

Signs of active SLE are as the following:

Elevated ESR and Anti-DsDNA

Reduced CRP, C3, and C4

Page - 2288
Internal Medicine - Rheumatology

Question 62/71

Question #62

A 50-year-old female complains of arthritis, fever, serositis, and proteinuria. In addition, her medical
history is significant for atrial fibrillation, for which she takes procainamide. Which of the following
antibodies is the most specific for the suspected disease?

a. Anti-sm antibody
b. Anti-DsDNA antibody
c. Anti-Histone antibody
d. Antinuclear antibody
e. Ant-La (SSB)

‫اﻹﺟﺎﺑﺔ ﻋﲆ اﻟﺼﻔﺤﺔ اﻟﺘﺎﻟﻴﺔ‬

Page - 2289
Internal Medicine - Rheumatology - Systemic Lupus Erythematosus (SLE)

Question 62/71

Question #62

A 50-year-old female complains of arthritis, fever, serositis, and proteinuria. In addition, her medical
history is significant for atrial fibrillation, for which she takes procainamide. Which of the following
antibodies is the most specific for the suspected disease?

a. Anti-sm antibody √
b. Anti-DsDNA antibody
c. Anti-Histone antibody
d. Antinuclear antibody
e. Ant-La (SSB)

Description

Although procainamide can cause drug-induced lupus, drugs can not cause renal involvement in
SLE.

In drug-induced lupus, the symptoms are limited to arthritis, fever, and serositis (do not order Anti-
histone antibodies in a patient with renal involvement of SLE).

Page - 2290
Internal Medicine - Rheumatology

Question 63/71

Question #63

A young female is found to have a positive antinuclear antibody. Her history is significant for
arthralgia, malar rash, ankle edema, and proteinuria. What is the most likely diagnosis?

a. Systemic lupus erythematosus


b. Rheumatoid arthritis
c. Isolated Reynaud’s disease
d. Systemic sclerosis
e. Multiple sclerosis

‫اﻹﺟﺎﺑﺔ ﻋﲆ اﻟﺼﻔﺤﺔ اﻟﺘﺎﻟﻴﺔ‬

Page - 2291
Internal Medicine - Rheumatology - Systemic Lupus Erythematosus (SLE)

Question 63/71

Question #63

A young female is found to have a positive antinuclear antibody. Her history is significant for
arthralgia, malar rash, ankle edema, and proteinuria. What is the most likely diagnosis?

a. Systemic lupus erythematosus √


b. Rheumatoid arthritis
c. Isolated Reynaud’s disease
d. Systemic sclerosis
e. Multiple sclerosis

Description

Antinuclear antibody (ANA) is the most sensitive test in SLE, but it is not specific.

e presence of arthralgia, malar rash, and renal involvement strongly suggest SLE.

Page - 2292
Internal Medicine - Rheumatology

Question 64/71

Question #64

A 30-year-old female presents with a malar rash and arthralgia. You suspect SLE. Which of the
following is the best initial test to perform now?

a. ANA
b. Anti-DsDNA
c. Anti-sm
d. Anti-Histone
e. Complement level

‫اﻹﺟﺎﺑﺔ ﻋﲆ اﻟﺼﻔﺤﺔ اﻟﺘﺎﻟﻴﺔ‬

Page - 2293
Internal Medicine - Rheumatology - Systemic Lupus Erythematosus (SLE)

Question 64/71

Question #64

A 30-year-old female presents with a malar rash and arthralgia. You suspect SLE. Which of the
following is the best initial test to perform now?

a. ANA √
b. Anti-DsDNA
c. Anti-sm
d. Anti-Histone
e. Complement level

Description

ANA is the best initial and the most sensitive test in SLE

It is 99% sensitive but only 70% specific

e following are the antibodies associated with SLE and their key details:

Antinuclear antibody (ANA): Highly Sensitive (99%) but not specific

RF: 20% sensitive, not specific

Anti-Smith: Most specific (> 99%), sensitivity (30%)

Anti-Double stranded DNA (Anti-dsDNA): Specific (99%) but only 70% sensitive (Indicates active
disease)

Anti-Ro (SSA)

Anti-La (SSB)

Anti-Histone antibody (for drug-induced SLE)

SLE can be associated with antiphospholipid antibodies

Page - 2294
Internal Medicine - Rheumatology

Question 65/71

Question #65

A 30-year-old female presents with a malar rash and arthralgia. You suspect SLE. Which of the
following is the most specific test to perform?

a. ANA
b. Anti-DsDNA
c. Anti-sm
d. Anti-Histone
e. Complement level

‫اﻹﺟﺎﺑﺔ ﻋﲆ اﻟﺼﻔﺤﺔ اﻟﺘﺎﻟﻴﺔ‬

Page - 2295
Internal Medicine - Rheumatology - Systemic Lupus Erythematosus (SLE)

Question 65/71

Question #65

A 30-year-old female presents with a malar rash and arthralgia. You suspect SLE. Which of the
following is the most specific test to perform?

a. ANA
b. Anti-DsDNA
c. Anti-sm √
d. Anti-Histone
e. Complement level

Description

Anti-sm is the most specific (>99%) for SLE while Anti-DsDNA is 99% specific; so, Anti-sm is slightly
more specific than Anti-DsDNA.

ANA is the best initial and the most sensitive test in SLE

It is 99% sensitive but only 70% specific

e following are the antibodies associated with SLE and their key details:

Antinuclear antibody (ANA): Highly Sensitive (99%) but not specific

RF: 20% sensitive, not specific

Anti-Smith: Most specific (> 99%), sensitivity (30%)

Anti-Double stranded DNA (Anti-dsDNA): Specific (99%) but only 70% sensitive (Indicates active
disease)

Anti-Ro (SSA)

Anti-La (SSB)

Anti-Histone antibody (for drug-induced SLE)

SLE can be associated with antiphospholipid antibodies

Page - 2296
Internal Medicine - Rheumatology

Question 66/71

Question #66

A 52-year-old female patient is known to have HTN, for which she receives hydrochlorothiazide,
hydralazine, and enalapril. She presents with a 1-month history of symmetrical arthralgia of hands
and arms and knee pain. On further evaluation, you found a pleural friction rub, elevated ANA titer,
and negative rheumatoid factor. What is the most appropriate initial management?

a. Stop enalapril
b. Stop hydrochlorothiazide
c. Stop hydralazine
d. Start on steroid therapy
e. Order Anti-sm antibody titer

‫اﻹﺟﺎﺑﺔ ﻋﲆ اﻟﺼﻔﺤﺔ اﻟﺘﺎﻟﻴﺔ‬

Page - 2297
Internal Medicine - Rheumatology - Systemic Lupus Erythematosus (SLE)

Question 66/71

Question #66

A 52-year-old female patient is known to have HTN, for which she receives hydrochlorothiazide,
hydralazine, and enalapril. She presents with a 1-month history of symmetrical arthralgia of hands
and arms and knee pain. On further evaluation, you found a pleural friction rub, elevated ANA titer,
and negative rheumatoid factor. What is the most appropriate initial management?

a. Stop enalapril
b. Stop hydrochlorothiazide
c. Stop hydralazine √
d. Start on steroid therapy
e. Order Anti-sm antibody titer

Description

is is a case of drug-induced lupus.

e most common drugs that can cause drug-induced lupus are:

Procainamide (the most common oender)


Hydralazine (the second most common oender)
Isoniazid
Tumor necrosis factor (TNF) alpha inhibitors (such as etanercept, infliximab, and adalimumab)
Minocycline
Quinidine

Page - 2298
Internal Medicine - Rheumatology

Question 67/71

Question #67

A 30-year-old lady complains of symmetrical arthritis, facial rash at the sun exposure distribution,
and mouth ulcer. Which of the following is the most likely diagnosis?

a. Gonococcal arthritis
b. Psoriatic arthritis
c. Reactive arthritis
d. Systemic lupus erythematosus
e. Rheumatoid arthritis

‫اﻹﺟﺎﺑﺔ ﻋﲆ اﻟﺼﻔﺤﺔ اﻟﺘﺎﻟﻴﺔ‬

Page - 2299
Internal Medicine - Rheumatology - Systemic Lupus Erythematosus (SLE)

Question 67/71

Question #67

A 30-year-old lady complains of symmetrical arthritis, facial rash at the sun exposure distribution,
and mouth ulcer. Which of the following is the most likely diagnosis?

a. Gonococcal arthritis
b. Psoriatic arthritis
c. Reactive arthritis
d. Systemic lupus erythematosus √
e. Rheumatoid arthritis

Description

SLE is an autoimmune disorder with several autoantibodies that are Associated with HLA B8, DR2,
DR3

e females are more aected than males (F: M ratio is 9:1)

ere are several Autoantibodies associated with this condition (the most specific one is an anti-
smith antibody (Anti-sm)

Page - 2300
Internal Medicine - Rheumatology

Question 68/71

Question #68

A 24-year-old female presents with fever, malaise, malar rash, and arthralgia. Anti-sm antibody is
positive. Which of the following is the most likely diagnosis?

a. Antiphospholipid antibody syndrome


b. Systemic lupus erythematosus
c. Rheumatoid arthritis
d. Psoriatic arthritis
e. Polymyalgia

‫اﻹﺟﺎﺑﺔ ﻋﲆ اﻟﺼﻔﺤﺔ اﻟﺘﺎﻟﻴﺔ‬

Page - 2301
Internal Medicine - Rheumatology - Systemic Lupus Erythematosus (SLE)

Question 68/71

Question #68

A 24-year-old female presents with fever, malaise, malar rash, and arthralgia. Anti-sm antibody is
positive. Which of the following is the most likely diagnosis?

a. Antiphospholipid antibody syndrome


b. Systemic lupus erythematosus √
c. Rheumatoid arthritis
d. Psoriatic arthritis
e. Polymyalgia

Description

SLE is an autoimmune disorder with several autoantibodies that are Associated with HLA B8, DR2,
DR3

e females are more aected than males (F: M ratio is 9:1)

ere are several Autoantibodies associated with this condition (the most specific one is an anti-
smith antibody (Anti-sm)

Page - 2302
Internal Medicine - Rheumatology

Question 69/71

Question #69

A 50-year-old female complains of arthritis, fever, and serositis. Her medical history is significant for
atrial fibrillation, for which she takes procainamide. Which of the following antibodies would mostly
help reach the suspected diagnosis?

a. Anti-sm antibody
b. Anti-DsDNA antibody
c. Anti-Histone antibody
d. Antinuclear antibody
e. Ant-La (SSB)

‫اﻹﺟﺎﺑﺔ ﻋﲆ اﻟﺼﻔﺤﺔ اﻟﺘﺎﻟﻴﺔ‬

Page - 2303
Internal Medicine - Rheumatology - Systemic Lupus Erythematosus (SLE)

Question 69/71

Question #69

A 50-year-old female complains of arthritis, fever, and serositis. Her medical history is significant for
atrial fibrillation, for which she takes procainamide. Which of the following antibodies would mostly
help reach the suspected diagnosis?

a. Anti-sm antibody
b. Anti-DsDNA antibody
c. Anti-Histone antibody √
d. Antinuclear antibody
e. Ant-La (SSB)

Description

is is a case of drug-induced lupus. Procainamide is the most common drug to cause lupus.

e most common drugs that can cause drug-induced lupus are:

Procainamide (the most common oender)


Hydralazine (the second most common oender)
Isoniazid
Tumor necrosis factor (TNF) alpha inhibitors (such as etanercept, infliximab, and adalimumab)
Minocycline
Quinidine

Page - 2304
Internal Medicine - Rheumatology

Question 70/71

Question #70

A 50-year-old female complains of both hands pain related to cold exposure. In addition, she has
dysphagia and exertional shortness of breath. What is the most likely diagnosis?

a. Esophageal carcinoma
b. Systemic sclerosis
c. Nasopharyngeal carcinoma
d. Benign esophageal stricture
e. Lupus erythematosus

‫اﻹﺟﺎﺑﺔ ﻋﲆ اﻟﺼﻔﺤﺔ اﻟﺘﺎﻟﻴﺔ‬

Page - 2305
Internal Medicine - Rheumatology - Systemic sclerosis (SS)

Question 70/71

Question #70

A 50-year-old female complains of both hands pain related to cold exposure. In addition, she has
dysphagia and exertional shortness of breath. What is the most likely diagnosis?

a. Esophageal carcinoma
b. Systemic sclerosis √
c. Nasopharyngeal carcinoma
d. Benign esophageal stricture
e. Lupus erythematosus

Description

e hand’s pain on cold exposure is typical for Raynaud’s phenomena.

e presence of exertional shortness of breath may indicate pulmonary fibrosis or cardiac


involvement, while dysphagia results from esophageal involvement.

is clinical scenario is typical for systemic sclerosis (SS).

SS is a generalized disorder of connective tissue of unknown cause characterized by skin fibrosis and
blood vessels and visceral involvement

e lungs, heart, or GI tract may be involved in the disease.

Page - 2306
Internal Medicine - Rheumatology

Question 71/71

Question #71

A 45-year-old female complains of symmetrical arthritis, pallor, and cyanosis of her fingers aer
exposure to cold weather. On examination, you noted telangiectasia on her lips and figures. Which
of the following is the most likely diagnosis?

a. Rheumatoid arthritis
b. Systemic sclerosis
c. Systemic lupus erythematosus
d. Reactive arthritis
e. Antiphospholipid antibody syndrome

‫اﻹﺟﺎﺑﺔ ﻋﲆ اﻟﺼﻔﺤﺔ اﻟﺘﺎﻟﻴﺔ‬

Page - 2307
Internal Medicine - Rheumatology - Systemic sclerosis (SS)

Question 71/71

Question #71

A 45-year-old female complains of symmetrical arthritis, pallor, and cyanosis of her fingers aer
exposure to cold weather. On examination, you noted telangiectasia on her lips and figures. Which
of the following is the most likely diagnosis?

a. Rheumatoid arthritis
b. Systemic sclerosis √
c. Systemic lupus erythematosus
d. Reactive arthritis
e. Antiphospholipid antibody syndrome

Description

Systemic sclerosis (SS) is a generalized disorder of connective tissue of unknown cause characterized
by skin fibrosis and blood vessels and visceral involvement

e lungs, heart, or GI tract may be involved in the disease.

In this scenario, the patient only has skin manifestations of SS, making limited type SS the most
likely diagnosis.

Page - 2308
Page - 2309

You might also like